Nothing Special   »   [go: up one dir, main page]

Academia.eduAcademia.edu
a LANGE medical book Katzung & Trevor’s Pharmacology Examination & Board Review Eleventh Edition Anthony J. Trevor, PhD Professor Emeritus of Pharmacology and Toxicology Department of Cellular & Molecular Pharmacology University of California, San Francisco Bertram G. Katzung, MD, PhD Professor Emeritus of Pharmacology Department of Cellular & Molecular Pharmacology University of California, San Francisco Marieke Kruidering-Hall, PhD Associate Professor & Academy Chair of Pharmacology Education Department of Cellular & Molecular Pharmacology University of California, San Francisco New York Chicago San Francisco Athens London Madrid Mexico City Milan New Delhi Singapore Sydney Toronto Copyright © 2015, 2013, 2010, 2008, 2005, 2002 by McGraw-Hill Education. All rights reserved. Except as permitted under the United States Copyright Act of 1976, no part of this publication may be reproduced or distributed in any form or by any means, or stored in a database or retrieval system, without the prior written permission of the publisher, with the exception that the program listings may be entered, stored, and executed in a computer system, but they may not be reproduced for publication. ISBN: 978-0-07-182639-6 MHID: 0-07-182639-4 The material in this eBook also appears in the print version of this title: ISBN: 978-0-07-182635-8, MHID: 0-07-182635-1. eBook conversion by codeMantra Version 1.0 All trademarks are trademarks of their respective owners. Rather than put a trademark symbol after every occurrence of a trademarked name, we use names in an editorial fashion only, and to the beneit of the trademark owner, with no intention of infringement of the trademark. Where such designations appear in this book, they have been printed with initial caps. McGraw-Hill Education eBooks are available at special quantity discounts to use as premiums and sales promotions or for use in corporate training programs. To contact a representative, please visit the Contact Us page at www.mhprofessional.com. Previous editions copyright © 1998, 1995, 1993, 1990 by Appleton & Lange. Notice Medicine is an ever-changing science. As new research and clinical experience broaden our knowledge, changes in treatment and drug therapy are required. The authors and the publisher of this work have checked with sources believed to be reliable in their efforts to provide information that is complete and generally in accord with the standards accepted at the time of publication. However, in view of the possibility of human error or changes in medical sciences, neither the authors nor the publisher nor any other party who has been involved in the preparation or publication of this work warrants that the information contained herein is in every respect accurate or complete, and they disclaim all responsibility for any errors or omissions or for the results obtained from use of the information contained in this work. Readers are encouraged to conirm the information contained herein with other sources. For example and in particular, readers are advised to check the product information sheet included in the package of each drug they plan to administer to be certain that the information contained in this work is accurate and that changes have not been made in the recommended dose or in the contraindications for administration. This recommendation is of particular importance in connection with new or infrequently used drugs. TERMS OF USE This is a copyrighted work and McGraw-Hill Education and its licensors reserve all rights in and to the work. Use of this work is subject to these terms. Except as permitted under the Copyright Act of 1976 and the right to store and retrieve one copy of the work, you may not decompile, disassemble, reverse engineer, reproduce, modify, create derivative works based upon, transmit, distribute, disseminate, sell, publish or sublicense the work or any part of it without McGraw-Hill Education’s prior consent. You may use the work for your own noncommercial and personal use; any other use of the work is strictly prohibited. Your right to use the work may be terminated if you fail to comply with these terms. THE WORK IS PROVIDED “AS IS.” McGRAW-HILL EDUCATION AND ITS LICENSORS MAKE NO GUARANTEES OR WARRANTIES AS TO THE ACCURACY, ADEQUACY OR COMPLETENESS OF OR RESULTS TO BE OBTAINED FROM USING THE WORK, INCLUDING ANY INFORMATION THAT CAN BE ACCESSED THROUGH THE WORK VIA HYPERLINK OR OTHERWISE, AND EXPRESSLY DISCLAIM ANY WARRANTY, EXPRESS OR IMPLIED, INCLUDING BUT NOT LIMITED TO IMPLIED WARRANTIES OF MERCHANTABILITY OR FITNESS FOR A PARTICULAR PURPOSE. McGraw-Hill Education and its licensors do not warrant or guarantee that the functions contained in the work will meet your requirements or that its operation will be uninterrupted or error free. Neither McGraw-Hill Education nor its licensors shall be liable to you or anyone else for any inaccuracy, error or omission, regardless of cause, in the work or for any damages resulting therefrom. McGraw-Hill Education has no responsibility for the content of any information accessed through the work. Under no circumstances shall McGraw-Hill Education and/or its licensors be liable for any indirect, incidental, special, punitive, consequential or similar damages that result from the use of or inability to use the work, even if any of them has been advised of the possibility of such damages. This limitation of liability shall apply to any claim or cause whatsoever whether such claim or cause arises in contract, tort or otherwise. Contents Preface part v part I BASIC PRINCIPLES 1. Introduction 26 4. Drug Metabolism 35 5. Pharmacogenomics 16. Histamine, Serotonin, & the Ergot Alkaloids 143 16 3. Pharmacokinetics part DRUGS WITH IMPORTANT ACTIONS ON SMOOTH MUSCLE 143 1 1 2. Pharmacodynamics IV 17. Vasoactive Peptides 152 18. Prostaglandins & Other Eicosanoids 158 19. Nitric Oxide, Donors, & Inhibitors 165 41 20. Drugs Used in Asthma & Chronic Obstructive Pulmonary Disease 169 II AUTONOMIC DRUGS 47 part 6. Introduction to Autonomic Pharmacology 47 7. Cholinoceptor-Activating & Cholinesterase-Inhibiting Drugs 60 V DRUGS THAT ACT IN THE CENTRAL NERVOUS SYSTEM 179 21. Introduction to CNS Pharmacology 179 8. Cholinoceptor Blockers & Cholinesterase Regenerators 69 22. Sedative-Hypnotic Drugs 186 9. Sympathomimetics 23. Alcohols 76 10. Adrenoceptor Blockers part 194 24. Antiseizure Drugs 201 85 25. General Anesthetics 208 III CARDIOVASCULAR DRUGS 93 26. Local Anesthetics 216 27. Skeletal Muscle Relaxants 221 11. Drugs Used in Hypertension 93 12. Drugs Used in the Treatment of Angina Pectoris 103 13. Drugs Used in Heart Failure 112 14. Antiarrhythmic Drugs 121 15. Diuretics & Other Drugs That Act on the Kidney 132 28. Drugs Used in Parkinsonism & Other Movement Disorders 229 29. Antipsychotic Agents & Lithium 236 30. Antidepressants 244 31. Opioid Analgesics & Antagonists 252 32. Drugs of Abuse 260 iii iv CONTENTS part 47. Antimycobacterial Drugs 389 VI DRUGS WITH IMPORTANT ACTIONS ON BLOOD, INFLAMMATION, & GOUT 267 33. Agents Used in Cytopenias; Hematopoietic Growth Factors 267 48. Antifungal Agents 395 49. Antiviral Chemotherapy & Prophylaxis 402 50. Miscellaneous Antimicrobial Agents & Urinary Antiseptics 414 51. Clinical Use of Antimicrobials 420 34. Drugs Used in Coagulation Disorders 276 52. Antiprotozoal Drugs 426 35. Agents Used in Dyslipidemia 288 53. Antihelminthic Drugs 434 36. NSAIDs, Acetaminophen, & Drugs Used in Rheumatoid Arthritis & Gout 296 54. Cancer Chemotherapy 440 55. Immunopharmacology part VII part ENDOCRINE DRUGS 307 IX TOXICOLOGY 37. Hypothalamic & Pituitary Hormones 307 452 463 38. Thyroid & Antithyroid Drugs 316 56. Environmental & Occupational Toxicology 463 39. Corticosteroids & Antagonists 322 57. Heavy Metals 469 40. Gonadal Hormones & Inhibitors 329 41. Pancreatic Hormones, Antidiabetic Agents, & Glucagon 340 42. Drugs That Affect Bone Mineral Homeostasis 349 part part X SPECIAL TOPICS 483 59. Drugs Used in Gastrointestinal Disorders 483 VIII CHEMOTHERAPEUTIC DRUGS 58. Management of the Poisoned Patient 475 359 60. Dietary Supplements & Herbal Medications 492 61. Drug Interactions 497 43. Beta-Lactam Antibiotics & Other Cell Wall Synthesis Inhibitors 360 44. Chloramphenicol, Tetracyclines, Macrolides, Clindamycin, Streptogramins, & Linezolid 369 45. Aminoglycosides 377 46. Sulfonamides, Trimethoprim, & Fluoroquinolones 382 Appendix I. Strategies for Improving Test Performance 503 Appendix II. Key Words for Key Drugs 506 Appendix III. Examination 1 518 Appendix IV. Examination 2 534 Index 549 Preface This book is designed to help students review pharmacology and to prepare for both regular course examinations and board examinations. The eleventh edition has been revised to make such preparation as active and efficient as possible. As with earlier editions, rigorous standards of accuracy and currency have been maintained in keeping with the book’s status as the companion to the Basic & Clinical Pharmacology textbook. This review book divides pharmacology into the topics used in most courses and textbooks. Major introductory chapters (eg, autonomic pharmacology and CNS pharmacology) are included for integration with relevant physiology and biochemistry. The chapter-based approach facilitates use of this book in conjunction with course notes or a larger text. We recommend several strategies to make reviewing more effective (Appendix I contains a summary of learning and test-taking strategies that most students find useful). First, each chapter has a short discussion of the major concepts that underlie its basic principles or the specific drug group, accompanied by explanatory figures and tables. The figures are in full color and some are new to this edition. Students are advised to read the text thoroughly before they attempt to answer the study questions at the end of each chapter. If a concept is found to be difficult or confusing, the student is advised to consult a regular textbook such as Basic & Clinical Pharmacology, 13th edition. Second, each drug-oriented chapter opens with an “Overview” that organizes the group of drugs visually in diagrammatic form. We recommend that students practice reproducing the overview diagram from memory. Third, a list of High Yield Terms to Learn and their definitions is near the front of most chapters. Make sure that you are able to define those terms. Fourth, many chapters include a “Skill Keeper” question that prompts the student to review previous material and to see links between related topics. We suggest that students try to answer Skill Keeper questions on their own before checking the answers that are provided at the end of the chapter. Fifth, each of the sixty-one chapters contains up to ten sample questions followed by a set of answers with explanations. For most effective learning, you should take each set of sample questions as if it were a real examination. After you have answered every question, work through the answers. When you are analyzing the answers, make sure that you understand why each choice is either correct or incorrect. Sixth, each chapter includes a Checklist of focused tasks that you should be able to do once you have finished the chapter. Seventh, most chapters end with a Summary Table that lists the most important drugs and includes key information concerning their mechanisms of action, effects, clinical uses, pharmacokinetics, drug interactions, and toxicities. Eighth, when preparing for a comprehensive examination, you should review the strategies described in Appendix I if you have not already done so. Then review the list of drugs in Appendix II: Key Words for Key Drugs. Students are also advised to check this appendix as they work through the chapters so they can begin to identify drugs out of the context of a chapter that reviews a restricted set of drugs. Ninth, after you have worked your way through most or all of the chapters and have a good grasp of the Key Drugs, you should take the comprehensive examinations, each of 100 questions, presented in Appendices III and IV. These examinations are followed by a list of answers, each with a short explanation or rationale underlying the correct choice and the numbers of the chapters in which more information can be found if needed. We recommend that you take an entire examination or a block of questions as if it were a real examination: commit to answers for the whole set before you check the answers. As you work through the answers, make sure that you understand why each answer is either correct or incorrect. If you need to, return to the relevant chapters(s) to review the text that covers key concepts and facts that form the basis for the question. We recommend that this book be used with a regular text. Basic & Clinical Pharmacology, 13th edition (McGraw-Hill, 2015), follows the chapter sequence used here. However, this review book is designed to complement any standard medical pharmacology text. The student who completes and understands Pharmacology: Examination & Board Review will greatly improve his or her performance and will have an excellent command of pharmacology. Because it was developed in parallel with the textbook Basic & Clinical Pharmacology, this review book represents the authors’ interpretations of chapters written by contributors to that text. We are grateful to those contributors, to our other v vi PREFACE faculty colleagues, and to our students, who have taught us most of what we know about teaching. We very much appreciate the invaluable contributions to this text afforded by the editorial team of Karen Edmonson, Rachel D’Annucci Henriquez, Shruti Awasthi, Harriet Lebowitz, and Michael Weitz. The authors also thank Katharine Katzung for her excellent proofreading contributions to this edition. Anthony J. Trevor, PhD Bertram G. Katzung, MD, PhD Marieke Kruidering-Hall, PhD PART I BASIC PRINCIPLES C H A P T E R 1 Introduction Pharmacology is the body of knowledge concerned with the action of chemicals on biologic systems. Medical pharmacology is the area of pharmacology concerned with the use of chemicals in the prevention, diagnosis, and treatment of disease, especially in humans. Toxicology is the area of pharmacology concerned with the undesirable effects of chemicals on biologic systems. Pharmacokinetics describes the effects of the body on drugs, eg, absorption, excretion, etc. Pharmacodynamics denotes the actions of the drug on the body, such as mechanism of action and therapeutic and toxic effects. The first part of this chapter reviews the basic principles of pharmacokinetics and pharmacodynamics that will be applied in subsequent chapters. The second part of the chapter reviews the development and regulation of drugs. Nature of drugs Pharmacokinetics Pharmacodynamics Receptor, receptor sites Inert binding sites Movement of drugs in body Absorption Distribution Metabolism Elimination Drug development & regulation Safety & efficacy Animal testing Clinical trials Patents & generic drugs 1 2 PART I Basic Principles ■ I. THE NATURE OF DRUGS PHARMACODYNAMIC PRINCIPLES Drugs in common use include inorganic ions, nonpeptide organic molecules, small peptides and proteins, nucleic acids, lipids, and carbohydrates. Some are found in plants or animals, and others are partially or completely synthetic. Many drugs found in nature are alkaloids, which are molecules that have a basic pH in solution, usually as a result of amine groups in their structure. Many biologically important endogenous molecules and exogenous drugs are optically active; that is, they contain one or more asymmetric centers and can exist as enantiomers. The enantiomers of optically active drugs usually differ, sometimes more than 1000-fold, in their affinity for biologic receptor sites. Furthermore, such enantiomers may be metabolized at different rates in the body, with important clinical consequences. A. Receptors Drug actions are mediated through the effects of drug ligand molecules on drug receptors in the body. Most receptors are large regulatory molecules that influence important biochemical processes (eg, enzymes involved in glucose metabolism) or physiologic processes (eg, ion channel receptors, neurotransmitter reuptake transporters, and ion transporters). If drug-receptor binding results in activation of the receptor, the drug is termed an agonist; if inhibition results, the drug is considered an antagonist. Some drugs mimic agonist molecules by inhibiting metabolic enzymes, eg, acetylcholinesterase inhibitors. As suggested in Figure 1–1, a receptor molecule may have several binding sites. Quantitation of the effects of drug-receptor binding as a function of dose yields dose-response curves that provide information about the nature of the drug-receptor interaction. Dose-response phenomena are discussed in more detail in Chapter 2. A few drugs are enzymes themselves (eg, thrombolytic enzymes, pancreatic enzymes). These drugs do not act on endogenous receptors but on substrate molecules. A. Size and Molecular Weight Drugs vary in size from molecular weight (MW) 7 (lithium) to over MW 50,000 (thrombolytic enzymes, antibodies, other proteins). Most drugs, however, have MWs between 100 and 1000. Drugs smaller than MW 100 are rarely sufficiently selective in their actions, whereas drugs much larger than MW 1000 are often poorly absorbed and poorly distributed in the body. Most protein drugs (“biologicals”) are commercially produced in cell, bacteria, or yeast cultures using recombinant DNA technology. B. Drug-Receptor Bonds Drugs bind to receptors with a variety of chemical bonds. These include very strong covalent bonds (which usually result in irreversible action), somewhat weaker electrostatic bonds (eg, between a cation and an anion), and much weaker interactions (eg, hydrogen, van der Waals, and hydrophobic bonds). B. Receptor and Inert Binding Sites Because most ligand molecules are much smaller than their receptor molecules (discussed in the text that follows), specific regions of receptor molecules provide the local areas responsible for drug binding. Such areas are termed receptor sites or recognition sites. In addition, drugs bind to some nonregulatory molecules in the body without producing a discernible effect. Such binding sites are termed inert binding sites. In some compartments of the High-Yield Terms to Learn Drugs Substances that act on biologic systems at the chemical (molecular) level and alter their functions Drug receptors The molecular components of the body with which drugs interact to bring about their effects Distribution phase The phase of drug movement from the site of administration into the tissues Elimination phase The phase of drug inactivation or removal from the body by metabolism or excretion Endocytosis, exocytosis Endocytosis: Absorption of material across a cell membrane by enclosing it in cell membrane material and pulling it into the cell, where it can be processed or released. Exocytosis: Expulsion of material from vesicles in the cell into the extracellular space Permeation Movement of a molecule (eg, drug) through the biologic medium Pharmacodynamics The actions of a drug on the body, including receptor interactions, dose-response phenomena, and mechanisms of therapeutic and toxic actions Pharmacokinetics The actions of the body on the drug, including absorption, distribution, metabolism, and elimination. Elimination of a drug may be achieved by metabolism or by excretion. Biodisposition is a term sometimes used to describe the processes of metabolism and excretion Transporter A specialized molecule, usually a protein, that carries a drug, transmitter, or other molecule across a membrane in which it is not permeable, eg, Na+/K+ ATPase, serotonin reuptake transporter, etc Mutagenic An effect on the inheritable characteristics of a cell or organism—a mutation in the DNA; usually tested in microorganisms with the Ames test Carcinogenic An effect of inducing malignant characteristics Teratogenic An effect on the in utero development of an organism resulting in abnormal structure or function; not generally heritable CHAPTER 1 Introduction 3 High-Yield Terms to Learn (continued) Placebo An inactive “dummy” medication made up to resemble the active investigational formulation as much as possible but lacking therapeutic effect Single-blind study A clinical trial in which the investigators—but not the subjects—know which subjects are receiving active drug and which are receiving placebos Double-blind study A clinical trial in which neither the subjects nor the investigators know which subjects are receiving placebos; the code is held by a third party IND Investigational New Drug Exemption; an application for FDA approval to carry out new drug trials in humans; requires animal data NDA New Drug Application; seeks FDA approval to market a new drug for ordinary clinical use; requires data from clinical trials as well as preclinical (animal) data Phases 1, 2, and 3 of clinical trials Three parts of a clinical trial that are usually carried out before submitting an NDA to the FDA Positive control A known standard therapy, to be used along with placebo, to evaluate the superiority or inferiority of a new drug in relation to the other drugs available Orphan drugs Drugs developed for diseases in which the expected number of patients is small. Some countries bestow certain commercial advantages on companies that develop drugs for uncommon diseases Drug Receptor Effects A + – Response Agonist A+C A alone A+B B A+D Competitive inhibitor Log Dose C Allosteric activator D Allosteric inhibitor FIGURE 1–1 Potential mechanisms of drug interaction with a receptor. Possible effects resulting from these interactions are diagrammed in the dose-response curves at the right. The traditional agonist (drug A)-receptor binding process results in the dose-response curve denoted “A alone.” B is a pharmacologic antagonist drug that competes with the agonist for binding to the receptor site. The dose-response curve produced by increasing doses of A in the presence of a fixed concentration of B is indicated by the curve “A+B.” Drugs C and D act at different sites on the receptor molecule; they are allosteric activators or inhibitors. Note that allosteric inhibitors do not compete with the agonist drug for binding to the receptor, and they may bind reversibly or irreversibly. (Reproduced, with permission, from Katzung BG, editor: Basic & Clinical Pharmacology, 12th ed. McGraw-Hill, 2012: Fig. 1–3.) 4 PART I Basic Principles body (eg, the plasma), inert binding sites play an important role in buffering the concentration of a drug because bound drug does not contribute directly to the concentration gradient that drives diffusion. Albumin and orosomucoid (α1-acid glycoprotein) are two important plasma proteins with significant drug-binding capacity. PHARMACOKINETIC PRINCIPLES To produce useful therapeutic effects, most drugs must be absorbed, distributed, and eliminated. Pharmacokinetic principles make rational dosing possible by quantifying these processes. The Movement of Drugs in the Body To reach its receptors and bring about a biologic effect, a drug molecule (eg, a benzodiazepine sedative) must travel from the site of administration (eg, the gastrointestinal tract) to the site of action (eg, the brain). A. Permeation Permeation is the movement of drug molecules into and within the biologic environment. It involves several processes, the most important of which are discussed next. 1. Aqueous diffusion—Aqueous diffusion is the movement of molecules through the watery extracellular and intracellular spaces. The membranes of most capillaries have small water-filled pores that permit the aqueous diffusion of molecules up to the size of small proteins between the blood and the extravascular space. This is a passive process governed by Fick’s law (see later discussion). The capillaries in the brain, testes, and some other organs lack aqueous pores, and these tissues are less exposed to some drugs. 2. Lipid diffusion—Lipid diffusion is the passive movement of molecules through membranes and other lipid barriers. Like aqueous diffusion, this process is governed by Fick’s law. 3. Transport by special carriers—Drugs that do not readily diffuse through membranes may be transported across barriers by mechanisms that carry similar endogenous substances. A very large number of such transporter molecules have been identified, and many of these are important in the movement of drugs or as targets of drug action. Unlike aqueous and lipid diffusion, carrier transport is not governed by Fick’s law and is capacitylimited. Important examples are transporters for ions (eg, Na+/ K+ ATPase), for neurotransmitters (eg, transporters for serotonin, norepinephrine), for metabolites (eg, glucose, amino acids), and for foreign molecules (xenobiotics) such as anticancer drugs. After release, amine neurotransmitters (dopamine, norepinephrine, and serotonin) and some other transmitters are recycled into nerve endings by transport molecules. Selective inhibitors for these transporters often have clinical value; for example, several antidepressants act by inhibiting the transport of amine neurotransmitters back into the nerve endings from which they have been released. 4. Endocytosis—Endocytosis occurs through binding of the transported molecule to specialized components (receptors) on cell membranes, with subsequent internalization by infolding of that area of the membrane. The contents of the resulting intracellular vesicle are subsequently released into the cytoplasm of the cell. Endocytosis permits very large or very lipid-insoluble chemicals to enter cells. For example, large molecules such as proteins may cross cell membranes by endocytosis. Smaller, polar substances such as vitamin B12 and iron combine with special proteins (B12 with intrinsic factor and iron with transferrin), and the complexes enter cells by this mechanism. Because the substance to be transported must combine with a membrane receptor, endocytotic transport can be quite selective. Exocytosis is the reverse process, that is, the expulsion of material that is membrane-encapsulated inside the cell from the cell. Most neurotransmitters are released by exocytosis. B. Fick’s Law of Diffusion Fick’s law predicts the rate of movement of molecules across a barrier. The concentration gradient (C1 − C2) and permeability coefficient for the drug and the area and thickness of the barrier membrane are used to compute the rate as follows: Rate = C1 − C2 × Permeability coefficient × Area Thickness (1) Thus, drug absorption is faster from organs with large surface areas, such as the small intestine, than from organs with smaller absorbing areas (the stomach). Furthermore, drug absorption is faster from organs with thin membrane barriers (eg, the lung) than from those with thick barriers (eg, the skin). C. Water and Lipid Solubility of Drugs 1. Solubility—The aqueous solubility of a drug is often a function of the electrostatic charge (degree of ionization, polarity) of the molecule, because water molecules behave as dipoles and are attracted to charged drug molecules, forming an aqueous shell around them. Conversely, the lipid solubility of a molecule is inversely proportional to its charge. Many drugs are weak bases or weak acids. For such molecules, the pH of the medium determines the fraction of molecules charged (ionized) versus uncharged (nonionized). If the pKa of the drug and the pH of the medium are known, the fraction of molecules in the ionized state can be predicted by means of the Henderson-Hasselbalch equation:  Protonated form  log  = pK a − pH  Unprotonated form (2) “Protonated” means associated with a proton (a hydrogen ion); this form of the equation applies to both acids and bases. 2. Ionization of weak acids and bases—Weak bases are ionized—and therefore more polar and more water-soluble—when they are protonated. Weak acids are not ionized—and so are less water-soluble—when they are protonated. CHAPTER 1 Introduction Absorption of Drugs The following equations summarize these points: RNH3+  protonated weak base (charged, more water-soluble) RCOOH  protonated weak acid (uncharged, more lipid-soluble) RNH2 + H+ Unprotonated weak proton base (uncharged, more lipid-soluble) (3) + H+ RCOO – Unprotonated weak proton acid (charged, (4) more water-soluble) The Henderson-Hasselbalch relationship is clinically important when it is necessary to estimate or alter the partition of drugs between compartments of differing pH. For example, most drugs are freely filtered at the glomerulus, but lipid-soluble drugs can be rapidly reabsorbed from the tubular urine. If a patient takes an overdose of a weak acid drug, for example, aspirin, the excretion of this drug is faster in alkaline urine. This is because a drug that is a weak acid dissociates to its charged, polar form in alkaline solution, and this form cannot readily diffuse from the renal tubule back into the blood; that is, the drug is trapped in the tubule. Conversely, excretion of a weak base (eg, pyrimethamine, amphetamine) is faster in acidic urine (Figure 1–2). Blood pH 7.4 H N Urine pH 6.0 Lipid diffusion 1.0 µM R Membranes of the nephron H TABLE 1–1 Common routes of drug administration. Oral (swallowed) Offers maximal convenience; absorption is often slower. Subject to the first-pass effect, in which a significant amount of the agent is metabolized in the gut wall, portal circulation, and liver before it reaches the systemic circulation Buccal and sublingual (not swallowed) Direct absorption into the systemic venous circulation, bypassing the hepatic portal circuit and first-pass metabolism Intravenous Instantaneous and complete absorption (by definition, bioavailability is 100%). Potentially more dangerous Intramuscular Often faster and more complete (higher bioavailability) than with oral administration. Large volumes may be given if the drug is not too irritating. First-pass metabolism is avoided Subcutaneous Slower absorption than the intramuscular route. First-pass metabolism is avoided. Rectal (suppository) The rectal route offers partial avoidance of the first-pass effect. Larger amounts of drug and drugs with unpleasant tastes are better administered rectally than by the buccal or sublingual routes Inhalation Route offers delivery closest to respiratory tissues (eg, for asthma). Usually very rapid absorption (eg, for anesthetic gases) Topical The topical route includes application to the skin or to the mucous membrane of the eye, ear, nose, throat, airway, or vagina for local effect Transdermal The transdermal route involves application to the skin for systemic effect. Absorption usually occurs very slowly (because of the thickness of the skin), but the first-pass effect is avoided H N H H+ H+ R A. Routes of Administration Drugs usually enter the body at sites remote from the target tissue or organ and thus require transport by the circulation to the intended site of action. To enter the bloodstream, a drug must be absorbed from its site of administration (unless the drug has been injected directly into the vascular compartment). The rate and efficiency of absorption differ depending on a drug’s route of administration. In fact, for some drugs, the amount absorbed may be only a small fraction of the dose administered when given by certain routes. The amount absorbed into the systemic circulation divided by the amount of drug administered constitutes its bioavailability by that route. Common routes of administration and some of their features are listed in Table 1–1. 1.0 µM R H N+ H R H N+ H H H 0.4 µM 10.0 µM 1.4 µM total 5 11.0 µM total FIGURE 1–2 The Henderson-Hasselbalch principle applied to drug excretion in the urine. Because the nonionized form diffuses readily across the lipid barriers of the nephron, this form may reach equal concentrations in the blood and urine; in contrast, the ionized form does not diffuse as readily. Protonation occurs within the blood and the urine according to the Henderson-Hasselbalch equation. Pyrimethamine, a weak base of pKa 7.0, is used in this example. At blood pH, only 0.4 μmol of the protonated species will be present for each 1.0 μmol of the unprotonated form. The total concentration in the blood will thus be 1.4 μmol/L if the concentration of the unprotonated form is 1.0 μmol/L. In the urine at pH 6.0, 10 μmol of the nondiffusible ionized form will be present for each 1.0 μmol of the unprotonated, diffusible form. Therefore, the total urine concentration (11 μmol/L) may be almost 8 times higher than the blood concentration. 6 PART I Basic Principles B. Blood Flow Blood flow influences absorption from intramuscular and subcutaneous sites and, in shock, from the gastrointestinal tract as well. High blood flow maintains a high drug depot-to-blood concentration gradient and thus facilitates absorption. C. Concentration The concentration of drug at the site of administration is important in determining the concentration gradient relative to the blood as noted previously. As indicated by Fick’s law (Equation 1), the concentration gradient is a major determinant of the rate of absorption. Drug concentration in the vehicle is particularly important in the absorption of drugs applied topically. Distribution of Drugs A. Determinants of Distribution 1. Size of the organ—The size of the organ determines the concentration gradient between blood and the organ. For example, skeletal muscle can take up a large amount of drug because the concentration in the muscle tissue remains low (and the bloodtissue gradient high) even after relatively large amounts of drug have been transferred; this occurs because skeletal muscle is a very large organ. In contrast, because the brain is smaller, distribution of a smaller amount of drug into it will raise the tissue concentration and reduce to zero the blood-tissue concentration gradient, preventing further uptake of drug unless it is actively transported. 2. Blood flow—Blood flow to the tissue is an important determinant of the rate of uptake of drug, although blood flow may not affect the amount of drug in the tissue at equilibrium. As a result, well-perfused tissues (eg, brain, heart, kidneys, and splanchnic organs) usually achieve high tissue concentrations sooner than poorly perfused tissues (eg, fat, bone). 3. Solubility—The solubility of a drug in tissue influences the concentration of the drug in the extracellular fluid surrounding the blood vessels. If the drug is very soluble in the cells, the concentration in the perivascular extracellular space will be lower and diffusion from the vessel into the extravascular tissue space will be facilitated. For example, some organs (such as the brain) have a high lipid content and thus dissolve a high concentration of lipid-soluble agents rapidly. 4. Binding—Binding of a drug to macromolecules in the blood or a tissue compartment tends to increase the drug’s concentration in that compartment. For example, warfarin is strongly bound to plasma albumin, which restricts warfarin’s diffusion out of the vascular compartment. Conversely, chloroquine is strongly bound to extravascular tissue proteins, which results in a marked reduction in the plasma concentration of chloroquine. B. Apparent Volume of Distribution and Physical Volumes The apparent volume of distribution (Vd) is an important pharmacokinetic parameter that reflects the above determinants of the TABLE 1–2 Average values for some physical volumes within the adult human body. Compartment Volume (L/kg body weight) Plasma 0.04 Blood 0.08 Extracellular water 0.2 Total body water 0.6 Fat 0.2–0.35 distribution of a drug in the body. Vd relates the amount of drug in the body to the concentration in the plasma (Chapter 3). In contrast, the physical volumes of various body compartments are less important in pharmacokinetics (Table 1–2). However, obesity alters the ratios of total body water to body weight and fat to total body weight, and this may be important when using highly lipid-soluble drugs. A simple approximate rule for the aqueous compartments of the normal body is as follows: 40% of total body weight is intracellular water and 20% is extracellular water; thus, water constitutes approximately 60% of body weight. Metabolism of Drugs Drug disposition is a term sometimes used to refer to metabolism and elimination of drugs. Some authorities use disposition to denote distribution as well as metabolism and elimination. Metabolism of a drug sometimes terminates its action, but other effects of drug metabolism are also important. Some drugs when given orally are metabolized before they enter the systemic circulation. This first-pass metabolism was referred to in Table 1–1 as one cause of low bioavailability. Drug metabolism occurs primarily in the liver and is discussed in greater detail in Chapter 4. A. Drug Metabolism as a Mechanism of Activation or Termination of Drug Action The action of many drugs (eg, sympathomimetics, phenothiazines) is terminated before they are excreted because they are metabolized to biologically inactive derivatives. Conversion to an inactive metabolite is a form of elimination. In contrast, prodrugs (eg, levodopa, minoxidil) are inactive as administered and must be metabolized in the body to become active. Many drugs are active as administered and have active metabolites as well (eg, morphine, some benzodiazepines). B. Drug Elimination Without Metabolism Some drugs (eg, lithium, many others) are not modified by the body; they continue to act until they are excreted. Elimination of Drugs Along with the dosage, the rate of elimination following the last dose (disappearance of the active molecules from the site of action, the bloodstream, and the body) determines the duration of action CHAPTER 1 Introduction for many drugs. Therefore, knowledge of the time course of concentration in plasma is important in predicting the intensity and duration of effect for most drugs. Note: Drug elimination is not the same as drug excretion: A drug may be eliminated by metabolism long before the modified molecules are excreted from the body. For most drugs and their metabolites, excretion is primarily by way of the kidney. Volatile anesthetic gases, a major exception, are excreted primarily by the lungs. For drugs with active metabolites (eg, diazepam), elimination of the parent molecule by metabolism is not synonymous with termination of action. For drugs that are not metabolized, excretion is the mode of elimination. A small number of drugs combine irreversibly with their receptors, so that disappearance from the bloodstream is not equivalent to cessation of drug action: These drugs may have a very prolonged action. For example, phenoxybenzamine, an irreversible inhibitor of α adrenoceptors, is eliminated from the bloodstream in less than 1 h after administration. The drug’s action, however, lasts for 48 h, the time required for turnover of the receptors. Such drugs do not have a constant half-life. This is typical of ethanol (over most of its plasma concentration range) and of phenytoin and aspirin at high therapeutic or toxic concentrations. Pharmacokinetic Models A. Multicompartment Distribution After absorption into the circulation, many drugs undergo an early distribution phase followed by a slower elimination phase. Mathematically, this behavior can be simulated by means of a “two-compartment model” as shown in Figure 1–4. The two compartments consist of the blood and the extravascular tissues. (Note that each phase is associated with a characteristic half-life: t1/2α for the first phase, t1/2β for the second phase. Note also that when concentration is plotted on a logarithmic axis, the elimination phase for a first-order drug is a straight line.) B. Other Distribution Models A few drugs behave as if they were distributed to only 1 compartment (eg, if they are restricted to the vascular compartment). Others have more complex distributions that require more than 2 compartments for construction of accurate mathematical models. A. First-Order Elimination The term first-order elimination indicates that the rate of elimination is proportional to the concentration (ie, the higher the concentration, the greater the amount of drug eliminated per unit time). The result is that the drug’s concentration in plasma decreases exponentially with time (Figure 1–3, left). Drugs with first-order elimination have a characteristic half-life of elimination that is constant regardless of the amount of drug in the body. The concentration of such a drug in the blood will decrease by 50% for every half-life. Most drugs in clinical use demonstrate first-order kinetics. ■ II. DRUG DEVELOPMENT & REGULATION The sale and use of drugs are regulated in almost all countries by governmental agencies. In the United States, regulation is by the Food and Drug Administration (FDA). New drugs are developed in industrial or academic laboratories. Before a new drug can be approved for regular therapeutic use in humans, a series of animal and experimental human studies (clinical trials) must be carried out. New drugs may emerge from a variety of sources. Some are the result of identification of a new target for a disease. B. Zero-Order Elimination The term zero-order elimination implies that the rate of elimination is constant regardless of concentration (Figure 1–3, right). This occurs with drugs that saturate their elimination mechanisms at concentrations of clinical interest. As a result, the concentrations of these drugs in plasma decrease in a linear fashion over time. 2.5 units/h 1.25 units/h Time (h) Zero-order elimination Plasma concentration Plasma concentration First-order elimination 5 units/h elimination rate 7 2.5 units/h elimination rate 2.5 units/h 2.5 units/h Time (h) FIGURE 1–3 Comparison of first-order and zero-order elimination. For drugs with first-order kinetics (left), rate of elimination (units per hour) is proportional to concentration; this is the more common process. In the case of zero-order elimination (right), the rate is constant and independent of concentration. 8 PART I Basic Principles Serum concentration (C) (µg/mL) (logarithmic scale) 64.0 Dose Distribution Distribution phase 32.0 Blood Tissues t1/2α Elimination t1/2β 16.0 8.0 Elimination phase 4.0 t1/2β 2.0 1.0 0 2 4 6 12 18 24 Time (h) (linear scale) FIGURE 1–4 Serum concentration-time curve after administration of a drug as an intravenous bolus. This drug follows first-order kinetics and appears to occupy two compartments. The initial curvilinear portion of the data represents the distribution phase, with drug equilibrating between the blood compartment and the tissue compartment. The linear portion of the curve represents drug elimination. The elimination half-life (t1/2β) can be extracted graphically as shown by measuring the time between any two plasma concentration points on the elimination phase that differ by twofold. (See Chapter 3 for additional details.) Rational molecular design or screening is then used to find a molecule that selectively alters the function of the target. New drugs may result from the screening of hundreds of compounds against model diseases in animals. In contrast, many (so-called “me-too” drugs) are the result of simple chemical alteration of the pharmacokinetic properties of the original prototype agent. SAFETY & EFFICACY Because society expects prescription drugs to be safe and effective, governments regulate the development and marketing of new drugs. Current regulations in the USA require evidence of relative safety (derived from acute and subacute toxicity testing in animals) and probable therapeutic action (from the pharmacologic profile in animals) before human testing is permitted. Some information about the pharmacokinetics of a compound is also required before clinical evaluation is begun. Chronic toxicity test results are generally not required, but testing must be underway before human studies are started. The development of a new drug and its pathway through various levels of testing and regulation are illustrated in Figure 1–5. The cost of development of a new drug, including false starts and discarded molecules, is often greater than 500 million dollars. ANIMAL TESTING The animal testing of a specific drug that is required before human studies can begin is a function of its proposed use and the urgency of the application. Thus, a drug proposed for occasional topical use requires less extensive testing than one destined for chronic systemic administration. Because of the urgent need, anticancer drugs and anti-HIV drugs require less evidence of safety than do drugs used in treatment of less threatening diseases. Urgently needed drugs are often investigated and approved on an accelerated schedule. A. Acute Toxicity Acute toxicity studies are required for all new drugs. These studies involve administration of incrementing doses of the agent up to the lethal level in at least 2 species (eg, 1 rodent and 1 nonrodent). B. Subacute and Chronic Toxicity Subacute and chronic toxicity testing is required for most agents, especially those intended for chronic use. Tests are usually conducted for 2–4 weeks (subacute) and 6–24 months (chronic), in at least 2 species. CHAPTER 1 Introduction In vitro studies Animal testing Clinical testing Biologic products 20–100 subjects 100–200 patients Phase 3 (Does it work, double blind?) 1000–6000 patients Chemical synthesis Generics become available (Does it work in patients?) Phase 2 Efficacy, Lead compound selectivity, mechanism Marketing (Is it safe, pharmacokinetics?) Phase 1 9 Phase 4 (Postmarketing surveillance) Drug metabolism, safety assessment 0 2 Years (average) 4 IND (Investigational New Drug) 8–9 NDA (New Drug Application) 20 (Patent expires 20 years after filing of application) FIGURE 1–5 The development and testing process required to bring a new drug to market in the United States. Some requirements may be different for drugs used in life-threatening diseases. (Reproduced, with permission, from Katzung BG, editor: Basic & Clinical Pharmacology, 12th ed. McGraw-Hill, 2012: Fig. 5–1.) TYPES OF ANIMAL TESTS A. Pharmacologic Profile The pharmacologic profile is a description of all the pharmacologic effects of a drug (eg, effects on cardiovascular function, gastrointestinal activity, respiration, hepatic and renal function, endocrine function, CNS). Both graded and quantal doseresponse data are gathered. B. Reproductive Toxicity Reproductive toxicity testing involves the study of the fertility effects of the candidate drug and its teratogenic and mutagenic toxicity. The FDA has used a 5-level descriptive scale to summarize information regarding the safety of drugs in pregnancy (Table 1–3). Teratogenesis can be defined as the induction of developmental defects in the somatic tissues of the fetus (eg, by exposure of the fetus to a chemical, infection, or radiation). Teratogenesis is studied by treating pregnant female animals of at least 2 species at selected times during early pregnancy when organogenesis is known to take place and by later examining the fetuses or neonates for abnormalities. Examples of drugs known to have teratogenic effects include thalidomide, isotretinoin, valproic acid, ethanol, glucocorticoids, warfarin, lithium, and androgens. Mutagenesis denotes induction of changes in the genetic material of animals of any age and therefore induction of heritable abnormalities. The Ames test, the standard in vitro test for mutagenicity, uses a special strain of salmonella bacteria that depends on specific nutrients in the culture medium. Loss of this dependence as a result of exposure to the test drug signals a mutation. Many carcinogens (eg, aflatoxin, cancer chemotherapeutic drugs, and other agents that TABLE 1–3 FDA ratings of drug safety in pregnancy. Category Description A Controlled studies in women fail to demonstrate a risk to the fetus in the first trimester (and there is no evidence of a risk in later trimesters), and the possibility of fetal harm appears remote B Either animal reproduction studies have not demonstrated a fetal risk but there are no controlled studies in pregnant women, or animal reproduction studies have shown an adverse effect (other than a decrease in fertility) that was not confirmed in controlled studies in women in the first trimester (and there is no evidence of a risk in later trimesters) C Either studies in animals have revealed adverse effects on the fetus (teratogenic or embryocidal or other) and there are no controlled studies in women, or studies in women and animals are not available. Drugs should be given only when the potential benefit justifies the potential risk to the fetus D There is positive evidence of human fetal risk, but the benefits from use in pregnant women may be acceptable despite the risk (eg, if the drug is needed in a life-threatening situation or for a serious disease for which safer drugs cannot be used or are ineffective) X Studies in animals or human beings have demonstrated fetal abnormalities or there is evidence of fetal risk based on human experience or both, and the risk of the use of the drug in pregnant women clearly outweighs any possible benefit. The drug is contraindicated in women who are or may become pregnant 10 PART I Basic Principles bind to DNA) have mutagenic effects and test positive in the Ames test. The dominant lethal test is an in vivo mutagenicity test carried out in mice. Male animals are exposed to the test substance before mating. Abnormalities in the results of subsequent mating (eg, loss of embryos, deformed fetuses) signal a mutation in the male’s germ cells. conditions, and patients are closely monitored, often in a hospital research ward. The goal is to determine whether the agent has the desired efficacy (ie, produces adequate therapeutic response) at doses that are tolerated by sick patients. Detailed data are collected regarding the pharmacokinetics and pharmacodynamics of the drug in this patient population. C. Carcinogenesis Carcinogenesis is the induction of malignant characteristics in cells. Carcinogenicity is difficult and expensive to study, and the Ames test is often used to screen chemicals because there is a moderately high degree of correlation between mutagenicity in the Ames test and carcinogenicity in some animal tests, as previously noted. Agents with known carcinogenic effects include coal tar, aflatoxin, dimethylnitrosamine and other nitrosamines, urethane, vinyl chloride, and the polycyclic aromatic hydrocarbons in tobacco smoke (eg, benzo[a]pyrene) and other tobacco products. C. Phase 3 A phase 3 trial usually involves many patients (eg, 1000–6000 or more, in many centers) and many clinicians who are using the drug in the manner proposed for its ultimate general use (eg, in outpatients). Such studies usually include placebo and positive controls in a double-blind crossover design. The goals are to explore further, under the conditions of the proposed clinical use, the spectrum of beneficial actions of the new drug, to compare it with placebo (negative control) and older therapy (positive control), and to discover toxicities, if any, that occur so infrequently as to be undetectable in phase 2 studies. Very large amounts of data are collected and these studies are usually very expensive. Unfortunately, relatively few phase 3 trials include the current standard of care as a positive control. If the drug successfully completes phase 3, an NDA is submitted to the FDA. If the NDA is approved, the drug can be marketed and phase 4 begins. CLINICAL TRIALS Human testing of new drugs in the United States requires approval by institutional committees that monitor the ethical (informed consent, patient safety) and scientific aspects (study design, statistical power) of the proposed tests. Such testing also requires the prior approval by the FDA of an Investigational New Drug Exemption application (IND), which is submitted by the manufacturer to the FDA (Figure 1–5). The IND includes all the preclinical data collected up to the time of submission and the detailed proposal for clinical trials. The major clinical testing process is usually divided into 3 phases that are carried out to provide information for a New Drug Application (NDA). The NDA includes all the results of preclinical and clinical testing and constitutes the request for FDA approval of general marketing of the new agent for prescription use. A fourth phase of study (the surveillance phase) follows NDA approval. In particularly lethal conditions, the FDA may permit carefully monitored treatment of patients before phases 2 and 3 are completed. A. Phase 1 A phase 1 trial consists of careful evaluation of the dose-response relationship and the pharmacokinetics of the new drug in a small number of normal human volunteers (eg, 20–100). An exception is the phase 1 trials of cancer chemotherapeutic agents and other highly toxic drugs; these are carried out by administering the agents to volunteer patients with the target disease. In phase 1 studies, the acute effects of the agent are studied over a broad range of dosages, starting with one that produces no detectable effect and progressing to one that produces either a significant physiologic response or a very minor toxic effect. B. Phase 2 A phase 2 trial involves evaluation of a drug in a moderate number of sick patients (eg, 100–200) with the target disease. A placebo or positive control drug is included in a single-blind or double-blind design. The study is carried out under very carefully controlled D. Phase 4 Phase 4 represents the postmarketing surveillance phase of evaluation, in which it is hoped that toxicities that occur very infrequently will be detected and reported early enough to prevent major therapeutic disasters. Manufacturers are required to inform the FDA at regular intervals of all reported untoward drug reactions. Unlike the first 3 phases, phase 4 has not been rigidly regulated by the FDA in the past. Because so many drugs have been found to be unacceptably toxic only after they have been marketed, there is considerable current interest in making phase 4 surveillance more consistent, effective, and informative. DRUG PATENTS & GENERIC DRUGS A patent application is usually submitted around the time that a new drug enters animal testing (Figure 1–5). In the United States, approval of the patent and completion of the NDA approval process give the originator the right to market the drug without competition from other firms for a period of 10–14 years from the NDA approval date. After expiration of the patent, any company may apply to the FDA for permission to market a generic version of the same drug if they demonstrate that their generic drug molecule is bioequivalent (ie, meets certain requirements for content, purity, and bioavailability) to the original product. DRUG LEGISLATION Many laws regulating drugs in the United States were passed during the 20th century. Refer to Table 1–4 for a partial list of this legislation. CHAPTER 1 Introduction TABLE 1–4 Selected legislation pertaining to drugs in the United States. Law Purpose and Effect Pure Food and Drug Act of 1906 Prohibited mislabeling and adulteration of foods and drugs (but no requirement for efficacy or safety) Harrison Narcotics Act of 1914 Established regulations for the use of opium, opioids, and cocaine (marijuana added in 1937) Food, Drug, and Cosmetics Act of 1938 Required that new drugs be tested for safety as well as purity Kefauver-Harris Amendment (1962) Required proof of efficacy as well as safety for new drugs Dietary Supplement and Health Education Act (1994) Amended the Food, Drug, and Cosmetics act of 1938 to establish standards for dietary supplements but prohibited the FDA from applying drug efficacy and safety standards to supplements ORPHAN DRUGS An orphan drug is a drug for a rare disease (one affecting fewer than 200,000 people in the United States). The study of such agents has often been neglected because profits from the sales of an effective agent for an uncommon ailment might not pay the costs of development. In the United States, current legislation provides for tax relief and other incentives designed to encourage the development of orphan drugs. QUESTIONS 1. A 3-year-old is brought to the emergency department having just ingested a large overdose of tolbutamide, an oral antidiabetic drug. Tolbutamide is a weak acid with a pKa of 5.3. It is capable of entering most tissues, including the brain. On physical examination, the heart rate is 100/min, blood pressure 90/50 mm Hg, and respiratory rate 20/min. Which of the following statements about this case of tolbutamide overdose is most correct? (A) Urinary excretion would be accelerated by administration of NH4Cl, an acidifying agent (B) Urinary excretion would be accelerated by giving NaHCO3, an alkalinizing agent (C) Less of the drug would be ionized at blood pH than at stomach pH (D) Absorption of the drug would be slower from the stomach than from the small intestine (E) Hemodialysis is the only effective therapy 11 2. Botulinum toxin is a large protein molecule. Its action on cholinergic transmission depends on an intracellular action within nerve endings. Which one of the following processes is best suited for permeation of very large protein molecules into cells? (A) Aqueous diffusion (B) Endocytosis (C) First-pass effect (D) Lipid diffusion (E) Special carrier transport 3. A 12-year-old child has bacterial pharyngitis and is to receive an oral antibiotic. She complains of a sore throat and pain on swallowing. The tympanic membranes are slightly reddened bilaterally, but she does not complain of earache. Blood pressure is 105/70 mm Hg, heart rate 100/mm, temperature 37.8 °C (100.1 °F). Ampicillin is a weak organic acid with a pKa of 2.5. What percentage of a given dose will be in the lipid-soluble form in the duodenum at a pH of 4.5? (A) About 1% (B) About 10% (C) About 50% (D) About 90% (E) About 99% 4. Ampicillin is eliminated by first-order kinetics. Which of the following statements best describes the process by which the plasma concentration of this drug declines? (A) There is only 1 metabolic path for drug elimination (B) The half-life is the same regardless of the plasma concentration (C) The drug is largely metabolized in the liver after oral administration and has low bioavailability (D) The rate of elimination is proportional to the rate of administration at all times (E) The drug is distributed to only 1 compartment outside the vascular system 5. The pharmacokinetics of a new drug are under study in a phase 1 clinical trial. Which statement about the distribution of drugs to specific tissues is most correct? (A) Distribution to an organ is independent of blood flow (B) Distribution is independent of the solubility of the drug in that tissue (C) Distribution into a tissue depends on the unbound drug concentration gradient between blood and the tissue (D) Distribution is increased for drugs that are strongly bound to plasma proteins (E) Distribution has no effect on the half-life of the drug 6. The pharmacokinetic process or property that distinguishes the elimination of ethanol and high doses of phenytoin and aspirin from the elimination of most other drugs is called (A) Distribution (B) Excretion (C) First-pass effect (D) First-order elimination (E) Zero-order elimination 12 PART I Basic Principles 7. A new drug was administered intravenously, and its plasma levels were measured for several hours. A graph was prepared as shown below, with the plasma levels plotted on a logarithmic ordinate and time on a linear abscissa. It was concluded that the drug has first-order kinetics. From this graph, what is the best estimate of the half-life? 11. Which of the following would probably not be included in an optimal phase 3 clinical trial of a new analgesic drug for mild pain? (A) A negative control (placebo) (B) A positive control (current standard analgesic therapy) (C) Double-blind protocol (in which neither the patient nor immediate observers of the patient know which agent is active) (D) A group of 1000–5000 subjects with a clinical condition requiring analgesia (E) Prior submission of an NDA (new drug application) to the FDA Plasma concentration 32 16 8 4 2 1 0 1 2 3 4 5 6 7 Time (h) (A) (B) (C) (D) (E) 10. The “dominant lethal” test involves the treatment of a male adult animal with a chemical before mating; the pregnant female is later examined for fetal death and abnormalities. The dominant lethal test therefore is a test of (A) Teratogenicity (B) Mutagenicity (C) Carcinogenicity (D) Sperm viability 0.5 h 1h 3h 4h 7h 8. A large pharmaceutical company has conducted extensive animal testing of a new drug for the treatment of advanced prostate cancer. The chief of research and development recommends that the company now submit an IND application in order to start clinical trials. Which of the following statements is most correct regarding clinical trials of new drugs? (A) Phase 1 involves the study of a small number of normal volunteers by highly trained clinical pharmacologists (B) Phase 2 involves the use of the new drug in a large number of patients (1000–5000) who have the disease to be treated under conditions of proposed use (eg, outpatients) (C) Chronic animal toxicity studies must be complete and reported in the IND (D) Phase 4 involves the detailed study of toxic effects that have been discovered in phase 3 (E) Phase 2 requires the use of a positive control (a known effective drug) and a placebo 9. Which of the following statements about animal testing of potential new therapeutic agents is most correct? (A) Extends at least 3 years to discover late toxicities (B) Requires at least 1 primate species (eg, rhesus monkey) (C) Requires the submission of histopathologic slides and specimens to the FDA for evaluation by government scientists (D) Has good predictability for drug allergy-type reactions (E) May be abbreviated in the case of some very toxic agents used in cancer 12. Which of the following statements about the testing of new compounds for potential therapeutic use in the treatment of hypertension is most correct? (A) Animal tests cannot be used to predict the types of clinical toxicities that may occur because there is no correlation with human toxicity (B) Human studies in normal individuals will be done before the drug is used in individuals with hypertension (C) The degree of risk must be assessed in at least 3 species of animals, including 1 primate species (D) The animal therapeutic index must be known before trial of the agents in humans 13. The Ames test is frequently carried out before clinical trials are begun. The Ames test is a method that detects (A) Carcinogenesis in primates (B) Carcinogenesis in rodents (C) Mutagenesis in bacteria (D) Teratogenesis in any mammalian species (E) Teratogenesis in primates 14. Which of the following statements about new drug development is most correct? (A) Drugs that test positive for teratogenicity, mutagenicity, or carcinogenicity can be tested in humans (B) Food supplements and herbal (botanical) remedies are subject to the same FDA regulation as ordinary drugs (C) All new drugs must be studied in at least 1 primate species before NDA submission (D) Orphan drugs are drugs that are no longer produced by the original manufacturer (E) Phase 4 (surveillance) is the most rigidly regulated phase of clinical drug trials CHAPTER 1 Introduction ANSWERS 1. Questions that deal with acid-base (Henderson-Hasselbalch) manipulations are common on examinations. Since absorption involves permeation across lipid membranes, we can in theory treat an overdose by decreasing absorption from the gut and reabsorption from the tubular urine by making the drug less lipid-soluble. Ionization attracts water molecules and decreases lipid solubility. Tolbutamide is a weak acid, which means that it is less ionized when protonated, ie, at acid pH. Choice C suggests that the drug would be less ionized at pH 7.4 than at pH 2.0, which is clearly wrong for weak acids. Choice D says (in effect) that the more ionized form is absorbed faster, which is incorrect. A and B are opposites because NH4Cl is an acidifying salt and sodium bicarbonate an alkalinizing one. (From the point of view of test strategy, opposites in a list of answers always deserve careful attention.) E is a distracter. Because an alkaline environment favors ionization of a weak acid, we should give bicarbonate. The answer is B. Note that clinical management of overdose involves many other considerations in addition to trapping the drug in urine; manipulation of urine pH may be contraindicated for other reasons. 2. Endocytosis is an important mechanism for transport of very large molecules across membranes. Aqueous diffusion is not involved in transport across the lipid barrier of cell membranes. Lipid diffusion and special carrier transport are common for smaller molecules. The first-pass effect has nothing to do with the mechanisms of permeation; rather, it denotes drug metabolism or excretion before absorption into the systemic circulation. The answer is B. 3. U.S. Medical Licensing Examination (USMLE)-type questions often contain a lengthy clinical description in the stem. One can often determine the relevance of the clinical data by scanning the last sentence in the stem and the list of answers, see Appendix IV. In this question, the emphasis is clearly on pharmacokinetic principles. Ampicillin is an acid, so it is more ionized at alkaline pH and less ionized at acidic pH. The Henderson-Hasselbalch equation predicts that the ratio changes from 50/50 at the pH equal to the pKa to 1/10 (protonated/unprotonated) at 1 pH unit more alkaline than the pKa and 1/100 at 2 pH units more alkaline. For acids, the protonated form is the nonionized, more lipid-soluble form. The answer is A. 4. “First-order” means that the elimination rate is proportional to the concentration perfusing the organ of elimination. The half-life is a constant. The rate of elimination is proportional to the rate of administration only at steady state. The order of elimination is independent of the number of compartments into which a drug distributes. The answer is B. 5. This is a straightforward question of pharmacokinetic distribution concepts. From the list of determinants of drug distribution given on page 6, choice C is correct. 6. The excretion of most drugs follows first-order kinetics. However, ethanol and, in higher doses, aspirin and phenytoin follow zero-order kinetics; that is, their elimination rates are constant regardless of blood concentration. The answer is E. 7. Drugs with first-order kinetics have constant half-lives, and when the log of the concentration in a body compartment 8. 9. 10. 11. 12. 13. 14. 13 is plotted versus time, a straight line results. The half-life is defined as the time required for the concentration to decrease by 50%. As shown in the graph, the concentration decreased from 16 units at 1 h to 8 units at 4 h and 4 units at 7 h; therefore, the half-life is 7 h minus 4 h or 3 h. The answer is C. Except for known toxic drugs (eg, cancer chemotherapy drugs), phase 1 is carried out in 25–50 normal volunteers. Phase 2 is carried out in several hundred closely monitored patients with the disease. Results of chronic toxicity studies in animals are required in the NDA and are usually underway at the time of IND submission. However, they do not have to be completed and reported in the IND. Phase 4 is the general surveillance phase that follows marketing of the new drug. It is not targeted at specific effects. Positive controls and placebos are not a rigid requirement of any phase of clinical trials, although placebos are often used in phase 2 and phase 3 studies. The answer is A. Drugs proposed for short-term use may not require longterm chronic testing. For some drugs, no primates are used; for other agents, only 1 species is used. The data from the tests, not the evidence itself, must be submitted to the FDA. Prediction of human drug allergy from animal testing is useful but not definitive (see answer 12). The answer is E. The description of the test indicates that a chromosomal change (passed from father to fetus) is the toxicity detected. This is a mutation. The answer is B. The first 4 items (A–D) are correct; they would be included. An NDA cannot be acted upon until the first 3 phases of clinical trials have been completed. (The IND must be approved before clinical trials can be conducted.) The answer is E. Animal tests in a single species do not always predict human toxicities. However, when these tests are carried out in several species, most acute toxicities that occur in humans also appear in at least 1 animal species. According to current FDA rules, the “degree of risk” must be determined in at least 2 species. Use of primates is not always required. The therapeutic index is not required. Except for cancer chemotherapeutic agents and antivirals used in AIDS, phase 1 clinical trials are carried out in normal subjects. The answer is B. The Ames test is carried out in Salmonella and detects mutations in the bacterial DNA. Because mutagenic potential is associated with carcinogenic risk for many chemicals, a positive Ames test is often used to suggest that a particular agent may be a carcinogen. However, the test itself only detects mutations. The answer is C. Food supplements and botanicals are much more loosely regulated than conventional drugs. Primates are not required in any phase of new drug testing, although they are sometimes used. Orphan drugs are those for which the anticipated patient population is smaller than 200,000 patients in the United States. Phase 4 surveillance is the most loosely regulated phase of clinical trials. Many drugs in current clinical use test positive for teratogenicity, mutagenicity, or carcinogenicity. Such drugs are usually labeled with warnings about these toxicities and, in the case of teratogenicity, are labeled as contraindicated in pregnancy. The answer is A. 14 PART I Basic Principles CHECKLIST When you complete this chapter, you should be able to: ❑ Define and describe the terms receptor and receptor site. ❑ Distinguish between a competitive inhibitor and an allosteric inhibitor. ❑ Predict the relative ease of permeation of a weak acid or base from knowledge of its pKa, the pH of the medium, and the Henderson-Hasselbalch equation. ❑ List and discuss the common routes of drug administration and excretion. ❑ Draw graphs of the blood level versus time for drugs subject to zero-order elimination and for drugs subject to first-order elimination. Label the axes appropriately. ❑ Describe the major animal and clinical studies carried out in drug development. ❑ Describe the purpose of the Investigational New Drug (IND) Exemption and the New Drug Application (NDA). ❑ Define carcinogenesis, mutagenesis, and teratogenesis. ❑ Describe the difference between the FDA regulations for ordinary drugs and those for botanical remedies. CHAPTER 1 Summary Table Major Concept Description Nature of drugs Drugs are chemicals that modify body functions. They may be ions, carbohydrates, lipids, or proteins. They vary in size from lithium (MW 7) to proteins (MW ≥ 50,000) Drug permeation Most drugs are administered at a site distant from their target tissue. To reach the target, they must permeate through both lipid and aqueous pathways. Movement of drugs occurs by means of aqueous diffusion, lipid diffusion, transport by special carriers, or by exocytosis and endocytosis Rate of diffusion Aqueous diffusion and lipid diffusion are predicted by Fick’s law and are directly proportional to gradient, area, and permeability coefficient and inversely proportional to the length or thickness of the diffusion path Drug trapping Because the permeability coefficient of a weak base or weak acid varies with the pH according to the Henderson-Hasselbalch equation, drugs may be trapped in a cellular compartment in which the pH is such as to reduce their solubility in the barriers surrounding the compartment Routes of administration Drugs are usually administered by one of the following routes of administration: oral, buccal, sublingual, topical, transdermal, intravenous, subcutaneous, intramuscular, rectal, or by inhalation Drug distribution After absorption, drugs are distributed to different parts of the body depending on concentration gradient, blood flow, solubility, and binding in the tissue Drug elimination Drugs are eliminated by reducing their concentration or amount in the body. This occurs when the drug is inactivated by metabolism or excreted from the body Elimination kinetics The rate of elimination of drugs may be zero order (ie, constant regardless of concentration) or first order (ie, proportional to the concentration) (Continued ) CHAPTER 1 Introduction 15 CHAPTER 1 Summary Table (Continued ) Major Concept Description Drug safety and efficacy Standards of safety and efficacy for drugs developed slowly during the 20th century and are still incomplete. Because of heavy lobbying by manufacturers, these standards are still not applied to nutritional supplements and many so-called botanical or herbal medications. A few of the relevant US laws are listed in Table 1–4 Preclinical drug testing All new drugs undergo extensive preclinical testing in broken tissue preparations and cell cultures, isolated animal organ preparations, and intact animals. Efforts are made to determine the full range of toxic and therapeutic effects. See Figure 1–5 Clinical drug trials All new drugs proposed for use in humans must undergo a series of tests in humans. These tests are regulated by the FDA and may be accelerated or retarded depending on the perceived clinical need and possible toxicities. The trials are often divided into 3 phases before marketing is allowed. See Figure 1–5 C A P T E R 2 Pharmacodynamics Pharmacodynamics deals with the effects of drugs on biologic systems, whereas pharmacokinetics (Chapter 3) deals with actions of the biologic system on the drug. The principles of H pharmacodynamics apply to all biologic systems, from isolated receptors in the test tube to patients with specific diseases. Pharmacodynamics Receptors, effectors Dose-response curves Agonists, partial agonists, antagonists, inverse agonists RECEPTORS Receptors are the specific molecules in a biologic system with which drugs interact to produce changes in the function of the system. Receptors must be selective in their ligand-binding characteristics (so as to respond to the proper chemical signal and not to meaningless ones). Receptors must also be modifiable when they bind a drug molecule (so as to bring about the functional change). Many receptors have been identified, purified, chemically characterized, and cloned. Most are proteins; a few are other macromolecules such as DNA. Some authorities consider enzymes as a separate category; for the purposes of this book, enzymes that are affected by drugs are considered receptors. The receptor site (also known as the recognition site) for a drug is the specific binding region of the receptor macromolecule and has a relatively high and selective affinity for the drug molecule. The interaction of a drug with its receptor is the fundamental event that initiates the action of the drug, and many drugs are classified on the basis of their primary receptor affinity. EFFECTORS Effectors are molecules that translate the drug-receptor interaction into a change in cellular activity. The best examples of effectors are enzymes such as adenylyl cyclase. Some receptors are also effectors in that a 16 Signalling Signalling mechanisms mechanisms Receptor regulation single molecule may incorporate both the drug-binding site and the effector mechanism. For example, a tyrosine kinase effector enzyme is part of the insulin receptor molecule, and a sodium-potassium channel is the effector part of the nicotinic acetylcholine receptor. GRADED DOSE-RESPONSE RELATIONSHIPS When the response of a particular receptor-effector system is measured against increasing concentrations of a drug, the graph of the response versus the drug concentration or dose is called a graded dose-response curve (Figure 2–1A). Plotting the same data on a logarithmic concentration axis usually results in a sigmoid curve, which simplifies the mathematical manipulation of the doseresponse data (Figure 2–1B). The efficacy (Emax) and potency (EC50 or ED50) parameters are derived from these data. The smaller the EC50 (or ED50), the greater the potency of the drug. GRADED DOSE-BINDING RELATIONSHIP & BINDING AFFINITY It is possible to measure the percentage of receptors bound by a drug, and by plotting this percentage against the log of the CHAPTER 2 Pharmacodynamics 17 High-Yield Terms to Learn Receptor A molecule to which a drug binds to bring about a change in function of the biologic system Inert binding molecule or site A molecule to which a drug may bind without changing any function Receptor site Specific region of the receptor molecule to which the drug binds Spare receptor Receptor that does not bind drug when the drug concentration is sufficient to produce maximal effect; present when Kd > EC50 Effector Component of a system that accomplishes the biologic effect after the receptor is activated by an agonist; often a channel, transporter, or enzyme molecule, may be part of the receptor molecule Agonist A drug that activates its receptor upon binding Pharmacologic antagonist A drug that binds without activating its receptor and thereby prevents activation by an agonist Competitive antagonist A pharmacologic antagonist that can be overcome by increasing the concentration of agonist Irreversible antagonist A pharmacologic antagonist that cannot be overcome by increasing agonist concentration Physiologic antagonist A drug that counters the effects of another by binding to a different receptor and causing opposing effects Chemical antagonist A drug that counters the effects of another by binding the agonist drug (not the receptor) Allosteric agonist, antagonist A drug that binds to a receptor molecule without interfering with normal agonist binding but alters the response to the normal agonist Partial agonist A drug that binds to its receptor but produces a smaller effect (Emax) at full dosage than a full agonist Constitutive activity Activity of a receptor-effector system in the absence of an agonist ligand Inverse agonist A drug that binds to the non-active state of receptor molecules and decreases constitutive activity (see text) Graded dose-response curve A graph of the increasing response to increasing drug concentration or dose Quantal dose-response curve A graph of the increasing fraction of a population that shows a specified response at progressively increasing doses EC50, ED50, TD50, etc In graded dose-response curves, the concentration or dose that causes 50% of the maximal effect or toxicity. In quantal dose-response curves, the concentration or dose that causes a specified response in 50% of the population under study Kd The concentration of drug that binds 50% of the receptors in the system Efficacy, maximal efficacy The largest effect that can be achieved with a particular drug, regardless of dose, Emax Potency The amount or concentration of drug required to produce a specified effect, usually EC50 or ED50 50 EC50 0 10 20 30 Dose (linear scale) 200 C Emax 100 Change in heart rate (beats/min) Change in heart rate (beats/min) B Emax 50 EC50 0.5 5 Bmax 100 Percent of receptors bound A 100 50 500 Dose (log scale) 50 Kd 0.5 5 50 500 Dose (log scale) FIGURE 2–1 Graded dose-response and dose-binding graphs. (In isolated tissue preparations, concentration is usually used as the measure of dose.) A. Relation between drug dose or concentration (abscissa) and drug effect (ordinate). When the dose axis is linear, a hyperbolic curve is commonly obtained. B. Same data, logarithmic dose axis. The dose or concentration at which effect is half-maximal is denoted EC50, whereas the maximal effect is Emax. C. If the percentage of receptors that bind drug is plotted against drug concentration, a similar curve is obtained, and the concentration at which 50% of the receptors are bound is denoted Kd, and the maximal number of receptors bound is termed Bmax. 18 PART I Basic Principles concentration of the drug, a dose-binding graph similar to the dose-response curve is obtained (Figure 2–1C). The concentration of drug required to bind 50% of the receptor sites is denoted by the dissociation constant (Kd) and is a useful measure of the affinity of a drug molecule for its binding site on the receptor molecule. The smaller the Kd, the greater the affinity of the drug for its receptor. If the number of binding sites on each receptor molecule is known, it is possible to determine the total number of receptors in the system from the Bmax. derived from experiments carried out in this manner. Because the magnitude of the specified effect is arbitrarily determined, the ED50 determined by quantal dose-response measurements has no direct relation to the ED50 determined from graded doseresponse curves. Unlike the graded dose-response determination, no attempt is made to determine the maximal effect of the drug. Quantal dose-response data provide information about the variation in sensitivity to the drug in a given population, and if the variation is small, the curve is steep. QUANTAL DOSE-RESPONSE RELATIONSHIPS EFFICACY When the minimum dose required to produce a specified response is determined in each member of a population, the quantal dose-response relationship is defined (Figure 2–2). For example, a blood pressure-lowering drug might be studied by measuring the dose required to lower the mean arterial pressure by 20 mm Hg in 100 hypertensive patients. When plotted as the percentage of the population that shows this response at each dose versus the log of the dose administered, a cumulative quantal dose-response curve, usually sigmoid in shape, is obtained. The median effective dose (ED50), median toxic dose (TD50), and (in animals) median lethal dose (LD50) are Percent individuals responding 100 Cumulative percent exhibiting therapeutic effect Cumulative percent dead at each dose 50 Percent requiring dose to achieve desired effect 1.25 2.5 5 ED50 10 20 40 Dose (mg) Percent requiring dose for a lethal effect 80 160 320 640 LD50 FIGURE 2–2 Quantal dose-response plots from a study of the therapeutic and lethal effects of a new drug in mice. Shaded boxes (and the accompanying bell-shaped curves) indicate the frequency distribution of doses of drug required to produce a specified effect, that is, the percentage of animals that required a particular dose to exhibit the effect. The open boxes (and corresponding sigmoidal curves) indicate the cumulative frequency distribution of responses, which are lognormally distributed. (Modified and reproduced, with permission, from Katzung BG, editor: Basic & Clinical Pharmacology, 12th ed. McGraw-Hill, 2012: Fig. 2–16.) Efficacy—often called maximal efficacy—is the greatest effect (Emax) an agonist can produce if the dose is taken to the highest tolerated level. Efficacy is determined mainly by the nature of the drug and the receptor and its associated effector system. It can be measured with a graded dose-response curve (Figure 2–1) but not with a quantal dose-response curve. By definition, partial agonists have lower maximal efficacy than full agonists (see later discussion). POTENCY Potency denotes the amount of drug needed to produce a given effect. In graded dose-response measurements, the effect usually chosen is 50% of the maximal effect and the concentration or dose causing this effect is called the EC50 or ED50 (Figure 2–1A and B). Potency is determined mainly by the affinity of the receptor for the drug and the number of receptors available. In quantal doseresponse measurements, ED50, TD50, and LD50 are also potency variables (median effective, toxic, and lethal doses, respectively, in 50% of the population studied). Thus, a measure of potency can be determined from either graded or quantal dose-response curves (eg, Figures 2–1 and 2–2, respectively), but the numbers obtained are not identical and they have different meanings. SPARE RECEPTORS Spare receptors are said to exist if the maximal drug response (Emax) is obtained at less than 100% occupation of the receptors (Bmax). In practice, the determination is usually made by comparing the concentration for 50% of maximal effect (EC50) with the concentration for 50% of maximal binding (Kd). If the EC50 is less than the Kd, spare receptors are said to exist (Figure 2–3). This might result from 1 of 2 mechanisms. First, the duration of the effector activation may be much greater than the duration of the drug-receptor interaction. Second, the actual number of receptors may exceed the number of effector molecules available. The presence of spare receptors increases sensitivity to the agonist because the likelihood of a drug-receptor interaction increases in proportion to the number of receptors available. (For contrast, the system depicted in Figure 2–1, panels B and C, does not have spare receptors, since the EC50 and the Kd are equal.) CHAPTER 2 Pharmacodynamics 19 100 Effect Ra Percent of maximum Ri Drug effect D D Drug binding 50 Ri – D Ra – D Effect Kd EC50 100% 0 0.1 1.0 10 100 Ra + Da Full agonist 1000 FIGURE 2–3 In a system with spare receptors, the EC50 is lower than the Kd, indicating that to achieve 50% of maximal effect, less than 50% of the receptors must be activated. Explanations for this phenomenon are discussed in the text. Activity Dose (log scale) Ra + Dpa Constitutive activity AGONISTS, PARTIAL AGONISTS, & INVERSE AGONISTS Modern concepts of drug-receptor interactions consider the receptor to have at least 2 states—active and inactive. In the absence of ligand, a receptor might be fully active or completely inactive; alternatively, an equilibrium state might exist with some receptors in the activated state and with most in the inactive state (Ra + Ri; Figure 2–4). Many receptor systems exhibit some activity in the absence of ligand, suggesting that some fraction of the receptor is always in the activated state. Activity in the absence of ligand is called constitutive activity. A full agonist is a drug capable of fully activating the effector system when it binds to the receptor. In the model system illustrated in Figure 2–4, a full agonist has high affinity for the activated receptor conformation, and sufficiently high concentrations result in all the receptors achieving the activated state (Ra – Da). A partial agonist produces less than the full effect, even when it has saturated the receptors (Ra-Dpa + Ri-Dpa), presumably by combining with both receptor conformations, but favoring the active state. In the presence of a full agonist, a partial agonist acts as an inhibitor. In this model, neutral antagonists bind with equal affinity to the Ri and Ra states, preventing binding by an agonist and preventing any deviation from the level of constitutive activity. In contrast, inverse agonists have a higher affinity for the inactive Ri state than for Ra and decrease or abolish any constitutive activity. ANTAGONISTS A. Competitive and Irreversible Pharmacologic Antagonists Competitive antagonists are drugs that bind to, or very close to, the agonist receptor site in a reversible way without activating the Partial agonist Ra + Ri 0 Ra + Dant + Ri + Dant Antagonist Ri + Di Inverse agonist Log Dose FIGURE 2–4 Upper: One model of drug-receptor interactions. The receptor is able to assume 2 conformations, Ri and Ra. In the Ri state, it is inactive and produces no effect, even when combined with a drug (D) molecule. In the Ra state, it activates its effectors and an effect is recorded, even in the absence of ligand. In the absence of drug, the equilibrium between Ri and Ra determines the degree of constitutive activity. Lower: A full agonist drug (Da) has a much higher affinity for the Ra than for the Ri receptor conformation, and a maximal effect is produced at sufficiently high drug concentration. A partial agonist drug (Dpa) has somewhat greater affinity for the Ra than for the Ri conformation and produces less effect, even at saturating concentrations. A neutral antagonist (Dant) binds with equal affinity to both receptor conformations and prevents binding of agonist. An inverse agonist (Di) binds much more avidly to the Ri receptor conformation, prevents conversion to the Ra state, and reduces constitutive activity. (Modified and reproduced, with permission, from Katzung BG, editor: Basic & Clinical Pharmacology, 12th ed. McGraw-Hill, 2012: Fig. 1–4.) effector system for that receptor. Neutral antagonists bind the receptor without shifting the ratio of Ra to Ri (Figure 2–4). In the presence of a competitive antagonist, the dose-response curve for an agonist is shifted to higher doses (ie, horizontally to the right on the dose axis), but the same maximal effect is reached (Figure 2–5A). The agonist, if given in a high enough concentration, can displace the antagonist and fully activate the receptors. In contrast, an irreversible antagonist causes a downward shift of the maximum, with no shift of the curve on the dose axis unless spare receptors are present (Figure 2–5B). Unlike the effects of a competitive antagonist, the effects of an irreversible antagonist cannot be overcome by adding more agonist. Competitive antagonists increase the ED50; irreversible antagonists do not (unless spare receptors are present). 20 PART I Basic Principles A B 100 100 Agonist alone Agonist plus competitive antagonist 50 Effect of antagonist 0 Percent of maximum Percent of maximum Agonist alone Effect of antagonist 50 Agonist plus irreversible antagonist 0 0.1 1.0 10 100 1000 Agonist dose (log scale) 0.1 1.0 10 100 1000 Agonist dose (log scale) FIGURE 2–5 Agonist dose-response curves in the presence of competitive and irreversible antagonists. Note the use of a logarithmic scale for drug concentration. A. A competitive antagonist has an effect illustrated by the shift of the agonist curve to the right. B. An irreversible (or noncompetitive) antagonist shifts the agonist curve downward. A noncompetitive antagonist that acts at an allosteric site of the receptor (see Figure 1–1) may bind reversibly or irreversibly; a noncompetitive antagonist that acts at the receptor site binds irreversibly. B. Physiologic Antagonists A physiologic antagonist binds to a different receptor molecule, producing an effect opposite to that produced by the drug it antagonizes. Thus, it differs from a pharmacologic antagonist, which interacts with the same receptor as the drug it inhibits. Typical examples of physiologic antagonists are the antagonism of the bronchoconstrictor action of histamine by epinephrine’s bronchodilator action and glucagon’s antagonism of the cardiac depressant effects of propranolol. C. Chemical Antagonists A chemical antagonist interacts directly with the drug being antagonized to remove it or to prevent it from binding to its target. A chemical antagonist does not depend on interaction with the agonist’s receptor (although such interaction may occur). Common examples of chemical antagonists are dimercaprol, a chelator of lead and some other toxic metals, and pralidoxime, which combines avidly with the phosphorus in organophosphate cholinesterase inhibitors. SKILL KEEPER: ALLOSTERIC ANTAGONISTS (SEE CHAPTER 1) Describe the difference between a pharmacologic antagonist and an allosteric inhibitor. How could you differentiate these two experimentally? THERAPEUTIC INDEX & THERAPEUTIC WINDOW The therapeutic index is the ratio of the TD50 (or LD50) to the ED50, determined from quantal dose-response curves. The therapeutic index represents an estimate of the safety of a drug, because a very safe drug might be expected to have a very large toxic dose and a much smaller effective dose. For example, in Figure 2–2, the ED50 is approximately 3 mg, and the LD50 is approximately 150 mg. The therapeutic index is therefore approximately 150/3, or 50, in mice. Obviously, a full range of toxic doses cannot be ethically studied in humans. Furthermore, factors such as the varying slopes of dose-response curves make this estimate a poor safety index even in animals. The therapeutic window, a more clinically useful index of safety, describes the dosage range between the minimum effective therapeutic concentration or dose, and the minimum toxic concentration or dose. For example, if the average minimum therapeutic plasma concentration of theophylline is 8 mg/L and toxic effects are observed at 18 mg/L, the therapeutic window is 8–18 mg/L. Both the therapeutic index and the therapeutic window depend on the specific toxic effect used in the determination. SIGNALING MECHANISMS Once an agonist drug has bound to its receptor, some effector mechanism is activated. The receptor-effector system may be an enzyme in the intracellular space (eg, cyclooxygenase, a target of nonsteroidal anti-inflammatory drugs) or in the membrane or extracellular space (eg, acetylcholinesterase). Neurotransmitter reuptake CHAPTER 2 Pharmacodynamics 1 Steroid Drug 2 Tyrosine kinase 3 JAK-STAT 4 Ion Channel 21 5 GPCR Outside cell Membrane G Inside cell JAK A B Y Y~P X Y STAT FIGURE 2–6 Signaling mechanisms for drug effects. Five major cross-membrane signaling mechanisms are recognized: (1) transmembrane diffusion of the drug to bind to an intracellular receptor; (2) transmembrane enzyme receptors, whose outer domain provides the receptor function and inner domain provides the effector mechanism converting A to B; (3) transmembrane receptors that, after activation by an appropriate ligand, activate separate cytoplasmic tyrosine kinase molecules (JAKs), which phosphorylate STAT molecules that regulate transcription (Y, tyrosine; P, phosphate); (4) transmembrane channels that are gated open or closed by the binding of a drug to the receptor site; and (5) G protein-coupled receptors, which use a coupling protein to activate a separate effector molecule. (Modified and reproduced, with permission, from Katzung BG, editor: Basic & Clinical Pharmacology, 12th ed. McGraw-Hill, 2012: Fig. 2–5.) transporters (eg, the norepinephrine transporter, NET, and the dopamine transporter, DAT) are receptors for many drugs, eg, antidepressants and cocaine. Most antiarrhythmic drugs target voltageactivated ion channels in the membrane for sodium, potassium, or calcium. For the largest group of drug-receptor interactions, the drug is present in the extracellular space, whereas the effector mechanism resides inside the cell and modifies some intracellular process. These classic drug-receptor interactions involve signaling across the membrane. Five major types of transmembrane-signaling mechanisms for receptor-effector systems have been defined (Figure 2–6, Table 2–1). RECEPTOR REGULATION Receptors are dynamically regulated in number, location, and interaction with other molecules. Changes can occur over short times (minutes) and longer periods (days). TABLE 2–1 Types of transmembrane signaling receptors. Receptor Type Description Intracellular, often steroid receptor-like Steroids, vitamin D, nitric oxide, and a few other highly membrane-permeant agents cross the membrane and activate intracellular receptors. The effector molecule may be part of the receptor or separate Membrane-spanning receptoreffector enzymes Insulin, epidermal growth factor, and similar agents bind to the extracellular domain of molecules that incorporate tyrosine kinase enzyme activity in their intracellular domains. Most of these receptors dimerize upon activation Membrane receptors that bind intracellular tyrosine kinase enzymes (JAK-STAT receptors) Many cytokines activate receptor molecules that bind intracellular tyrosine kinase enzymes (Janus kinases, JAKs) that activate transcription regulators (signal transducers and activators of transcription, STATs) that migrate to the nucleus to bring about the final effect Ligand-activated or modulated membrane ion channels Certain Na+/K+ channels are activated by drugs: acetylcholine activates nicotinic Na+/K+ channels, serotonin activates 5-HT3 Na+/K+ channels. Benzodiazepines, barbiturates, and several other sedative hypnotics allosterically modulate GABA-activated Cl– channels G-protein-coupled receptors (GPCRs) GPCRs consist of 7 transmembrane (7-TM) domains and when activated by extracellular ligands, bind trimeric G proteins at the inner membrane surface and cause the release of activated Gα and Gβγ units. These activated units, in turn, modulate cytoplasmic effectors. The effectors commonly synthesize or release second messengers such as cAMP, IP3, and DAG. GPCRs are the most common type of receptors in the body cAMP, cyclic adenosine monophosphate; IP3, inositol trisphosphate; DAG, diacylglycerol. PART I Basic Principles Frequent or continuous exposure to agonists often results in shortterm diminution of the receptor response, sometimes called tachyphylaxis. Several mechanisms are responsible for this phenomenon. First, intracellular molecules may block access of a G protein to the activated receptor molecule. For example, the molecule β-arrestin has been shown to bind to an intracellular loop of the β adrenoceptor when the receptor is continuously activated. Beta-arrestin prevents access of the Gs-coupling protein and thus desensitizes the tissue to further β-agonist activation within minutes. Removal of the β agonist results in removal of β-arrestin and restoration of the full response after a few minutes or hours. Second, agonist-bound receptors may be internalized by endocytosis, removing them from further exposure to extracellular molecules. The internalized receptor molecule may then be either reinserted into the membrane (eg, morphine receptors) or degraded (eg, β adrenoceptors, epidermal growth factor receptors). In some cases, a cyclic internalization-reinsertion process may actually be necessary for normal functioning of the receptor-effector system. Third, continuous activation of the receptor-effector system may lead to depletion of some essential substrate required for downstream effects. For example, depletion of thiol cofactors may be responsible for tolerance to nitroglycerin. In some cases, repletion of the missing substrate (eg, by administration of glutathione) can reverse the tolerance. Long-term reductions in receptor number (downregulation) may occur in response to continuous exposure to agonists. The opposite change (upregulation) occurs when receptor activation is blocked for prolonged periods (usually several days) by pharmacologic antagonists or by denervation. QUESTIONS 1. A 55-year-old woman with hypertension is to be treated with a thiazide diuretic. Thiazide A in a dose of 5 mg produces the same decrease in blood pressure as 500 mg of thiazide B. Which of the following statements best describes these results? (A) Thiazide A is more efficacious than thiazide B (B) Thiazide A is about 100 times more potent than thiazide B (C) Toxicity of thiazide A is less than that of thiazide B (D) Thiazide A has a wider therapeutic window than thiazide B (E) Thiazide A has a longer half-life than thiazide B 2. Graded and quantal dose-response curves are being used for evaluation of a new antiasthmatic drug in the animal laboratory and in clinical trials. Which of the following statements best describes graded dose-response curves? (A) More precisely quantitated than quantal dose-response curves (B) Obtainable from isolated tissue preparations but not from the study of intact subjects (C) Used to determine the maximal efficacy of the drug (D) Used to determine the therapeutic index of the drug (E) Used to determine the variation in sensitivity of subjects to the drug 3. Prior to clinical trials in patients with heart failure, an animal study was carried out to compare two new positive inotropic drugs (A and B) to a current standard agent (C). The results of cardiac output measurements are shown in the graph below. B Increase in cardiac output 22 C A Log dose Which of the following statements is correct? (A) Drug A is most effective (B) Drug B is least potent (C) Drug C is most potent (D) Drug B is more potent than drug C and more effective than drug A (E) Drug A is more potent than drug B and more effective than drug C 4. A study was carried out in isolated intestinal smooth muscle preparations to determine the action of a new drug “novamine,” which in separate studies bound to the same receptors as acetylcholine. In the absence of other drugs, acetylcholine caused contraction of the muscle. Novamine alone caused relaxation of the preparation. In the presence of a low concentration of novamine, the EC50 of acetylcholine was unchanged, but the Emax was reduced. In the presence of a high concentration of novamine, extremely high concentrations of acetylcholine had no effect. Which of the following expressions best describes novamine? (A) A chemical antagonist (B) An irreversible antagonist (C) A partial agonist (D) A physiologic antagonist (E) A spare receptor agonist 5. Beta adrenoceptors in the heart regulate cardiac rate and contractile strength. Several studies have indicated that in humans and experimental animals, about 90% of β adrenoceptors in the heart are spare receptors. Which of the following statements about spare receptors is most correct? (A) Spare receptors, in the absence of drug, are sequestered in the cytoplasm (B) Spare receptors may be detected by finding that the drug-receptor interaction lasts longer than the intracellular effect (C) Spare receptors influence the maximal efficacy of the drug-receptor system (D) Spare receptors activate the effector machinery of the cell without the need for a drug (E) Spare receptors may be detected by the finding that the EC50 is smaller than the Kd for the agonist CHAPTER 2 Pharmacodynamics 6. Two cholesterol-lowering drugs, X and Y, were studied in a large group of patients, and the percentages of the group showing a specific therapeutic effect (35% reduction in lowdensity lipoprotein [LDL] cholesterol) were determined. The results are shown in the following table. Drug Dose (mg) 5 10 20 50 100 200 Percent Responding to Drug X Percent Responding to Drug Y 1 5 10 50 70 90 10 20 50 70 90 100 Which of the following statements about these results is correct? (A) Drug X is safer than drug Y (B) Drug Y is more effective than drug X (C) The 2 drugs act on the same receptors (D) Drug X is less potent than drug Y (E) The therapeutic index of drug Y is 10 7. Sugammadex is a new drug that reverses the action of rocuronium and certain other skeletal muscle-relaxing agents (nondepolarizing neuromuscular blocking agents). It appears to interact directly with the rocuronium molecule and not at all with the rocuronium receptor. Which of the following terms best describes sugammadex? (A) Chemical antagonist (B) Noncompetitive antagonist (C) Partial agonist (D) Pharmacologic antagonist (E) Physiologic antagonist DIRECTIONS: 8–10. Each of the curves in the graph below may be considered a concentration-effect curve or a concentration-binding curve. Curve 1 100 Percent of maximum Curve 3 50 Curve 2 Curve 4 Curve 5 Log dose 8. Which of the curves in the graph describes the percentage of binding of a large dose of full agonist to its receptors as the concentration of a partial agonist is increased from low to very high levels? (A) Curve 1 (B) Curve 2 (C) Curve 3 (D) Curve 4 (E) Curve 5 23 9. Which of the curves in the graph describes the percentage effect observed when a large dose of full agonist is present throughout the experiment and the concentration of a partial agonist is increased from low to very high levels? (A) Curve 1 (B) Curve 2 (C) Curve 3 (D) Curve 4 (E) Curve 5 10. Which of the curves in the graph describes the percentage of binding of the partial agonist whose effect is shown by Curve 4 if the system has many spare receptors? (A) Curve 1 (B) Curve 2 (C) Curve 3 (D) Curve 4 (E) Curve 5 ANSWERS 1. No information is given regarding the maximal antihypertensive response to either drug. Similarly, no information about half-life or toxicity is provided. The fact that a given response is achieved with a smaller dose of thiazide A indicates that A is more potent than B in the ratio of 500:5. The answer is B. 2. Precise quantitation is possible with both types of doseresponse curves. Quantal dose-response curves show the frequency of occurrence of a specified response, which may be therapeutically effective (ED) or toxic (TD). Thus, quantal studies are used to determine the therapeutic index and the variation in sensitivity to the drug. Graded (not quantal) dose-response curves are used to determine maximal efficacy (maximal response). The answer is C. 3. Drug A produces 50% of its maximal effect at a lower dose than either B or C and thus is the most potent; drug C is the least potent. However, drug A, a partial agonist, is less efficacious than drugs B and C. The answer is D. 4. Choices involving chemical or physiologic antagonism are incorrect because novamine is said to act at the same receptors as acetylcholine. When given alone, the novamine effect is opposite to that of acetylcholine, so choice C is incorrect. “Spare receptor agonist” is a nonsense distracter. The answer is B. 5. There is no difference in location between “spare” and other receptors. Spare receptors may be defined as those that are not needed for binding drug to achieve the maximal effect. Spare receptors influence the sensitivity of the system to an agonist because the statistical probability of a drug-receptor interaction increases with the total number of receptors. They do not alter the maximal efficacy. If they do not bind an agonist molecule, spare receptors do not activate an effector molecule. EC50 less than Kd is an indication of the presence of spare receptors. The answer is E. 6. No information is presented regarding the safety of these drugs. Similarly, no information on efficacy (maximal effect) is presented; this requires graded dose-response curves. Although both drugs are said to be producing a therapeutic effect, no information on their receptor mechanisms is given. Since no data on toxicity are available, the therapeutic index cannot be determined. The answer is D because the ED50 of drug Y (20 mg/d) is less than that of drug X (50 mg/d). 24 PART I Basic Principles 7. Sugammadex interacts directly with rocuronium and not with the rocuronium receptor; therefore, it is a chemical antagonist. The answer is A. 8. The binding of a full agonist decreases as the concentration of a partial agonist is increased to very high levels. As the partial agonist displaces more and more of the full agonist, the percentage of receptors that bind the full agonist drops to zero, that is, Curve 5. The answer is E. 9. Curve 1 describes the response of the system when a full agonist is displaced by increasing concentrations of partial agonist. This is because the increasing percentage of receptors binding the partial agonist finally produce the maximal effect typical of the partial agonist. The answer is A. 10. Partial agonists, like full agonists, bind 100% of their receptors when present in a high enough concentration. Therefore, the binding curve (but not the effect curve) will go to 100%. If the effect curve is Curve 4 and many spare receptors are present, the binding curve must be displaced to the right of Curve 4 (Kd > EC50). Therefore, Curve 3 fits the description better than Curve 2. The answer is C. SKILL KEEPER ANSWER: ALLOSTERIC ANTAGONISTS Allosteric antagonists do not bind to the agonist receptor site; they bind to some other region of the receptor molecule that results in inhibition of the response to agonists (see Figure 1–1). They do not prevent binding of the agonist. In contrast, pharmacologic antagonists bind to the agonist site and prevent access of the agonist. The difference can be detected experimentally by evaluating competition between the binding of radioisotopically labeled antagonist and the agonist. High concentrations of agonist displace or prevent the binding of a pharmacologic antagonist but not an allosteric antagonist. CHECKLIST When you complete this chapter, you should be able to: ❑ Compare the efficacy and the potency of 2 drugs on the basis of their graded dose- response curves. ❑ Predict the effect of a partial agonist in a patient in the presence and in the absence of a full agonist. ❑ Name the types of antagonists used in therapeutics. ❑ Describe the difference between an inverse agonist and a pharmacologic antagonist. ❑ Specify whether a pharmacologic antagonist is competitive or irreversible based on its effects on the dose-response curve and the dose-binding curve of an agonist in the presence of the antagonist. ❑ Give examples of competitive and irreversible pharmacologic antagonists and of physiologic and chemical antagonists. ❑ Name 5 transmembrane signaling methods by which drug-receptor interactions exert their effects. ❑ Describe 2 mechanisms of receptor regulation. CHAPTER 2 Pharmacodynamics 25 CHAPTER 2 Summary Table Major Concept Description Graded vs quantal responses Responses are graded when they increment gradually (eg, heart rate change) as the dose of drug increases; they are quantal when they switch from no effect to a specified effect at a certain dose (eg, from arrhythmia to normal sinus rhythm) or if they are measured as positive upon reaching a specified response Graded vs quantal dose response curves Graded dose response curves plot the increment in physiologic or biochemical response as dose or concentration is increased. Quantal dose response curves plot the increment in the percent of the population under study that responds with a specified effect as the dose is increased Efficacy vs potency Efficacy represents the maximal effect (Emax) of a drug at the highest tolerated dose, whereas potency reflects the amount of drug (the dose or concentration) required to cause a specific amount of effect, eg, the EC50 for a half-maximal effect. A drug may have high efficacy but low potency or vice versa Agonism and antagonism The ability to activate (agonism) or inhibit (antagonism) a biologic system or effect. Different drugs may have very different effects on a receptor. The effect may be to activate, partially activate, or inhibit the receptor’s function. In addition, the binding of a drug may be at the site that an endogenous ligand binds that receptor, or at a different site Transmembrane signaling Many drugs act on intracellular functions but reach their targets in the extracellular space. On reaching the target tissue, some drugs diffuse through the cell membrane and act on intracellular receptors. Most act on receptors on the extracellular face of the cell membrane and modify the intracellular function of those receptors by transmembrane signaling Receptor regulation Receptors are in dynamic equilibrium, being synthesized in the interior of the cell, inserted into the cell membranes, sequestered out of the membranes, and degraded at various rates. These changes are noted as upregulation or downregulation of the receptor numbers and usually take days to accomplish. More rapid changes (minutes or hours) in response to agonists may occur as a result of block of access of intracellular coupling molecules to activated receptors, resulting in tachyphylaxis or tolerance C A P T E R 3 Pharmacokinetics Pharmacokinetics denotes the effects of biologic systems on drugs. The major processes involved in pharmacokinetics are absorption, distribution, and elimination. Appropriate H application of pharmacokinetic data and a few simple formulas makes it possible to calculate loading and maintenance doses. Pharmacokinetics Volume of distribution Clearance Bioavailability Dosing Maintenance Half-life First pass effect Loading High-Yield Terms to Learn Volume of distribution (apparent) The ratio of the amount of drug in the body to the drug concentration in the plasma or blood. Units: liters Clearance The ratio of the rate of elimination of a drug to the concentration of the drug in the plasma or blood. Units: volume/time, eg, mL/min or L/h Half-life The time required for the amount of drug in the body or blood to fall by 50%. For drugs eliminated by first-order kinetics, this number is a constant regardless of the concentration. Units: time Bioavailability The fraction (or percentage) of the administered dose of drug that reaches the systemic circulation Area under the curve (AUC) The graphic area under a plot of drug concentration versus time after a single dose or during a single dosing interval. Units: concentration × time; eg, mg min/mL Peak and trough concentrations The maximum and minimum drug concentrations achieved during repeated dosing cycles Minimum effective concentration (MEC) The plasma drug concentration below which a patient’s response is too small for clinical benefit First-pass effect, presystemic elimination The elimination of drug that occurs after administration but before it enters the systemic circulation (eg, during passage through the gut wall, portal circulation, or liver for an orally administered drug) Steady state In pharmacokinetics, the condition in which the average total amount of drug in the body does not change over multiple dosing cycles (ie, the condition in which the rate of drug elimination equals the rate of administration) Biodisposition Often used as a synonym for pharmacokinetics; the processes of drug absorption, distribution, and elimination. Sometimes used more narrowly to describe elimination 26 CHAPTER 3 Pharmacokinetics 27 EFFECTIVE DRUG CONCENTRATION VOLUME OF DISTRIBUTION The effective drug concentration is the concentration of a drug at the receptor site. In patients, drug concentrations are more readily measured in the blood. Except for topically applied agents, the concentration at the receptor site is usually proportional to the drug’s concentration in the plasma or whole blood at equilibrium. The plasma concentration is a function of the rate of input of the drug (by absorption) into the plasma, the rate of distribution, and the rate of elimination. If the rate of input is known, the remaining processes are well described by 2 primary parameters: apparent volume of distribution (Vd) and clearance (CL). These parameters are unique for a particular drug and a particular patient but have average values in large populations that can be used to predict drug concentrations. The volume of distribution (Vd) relates the amount of drug in the body to the plasma concentration according to the following equation: Amount of drug in the body Vd = Plasma drug concentration (1) (Units = Volume) Vd = Amount of drug in the body Concentration in the blood 2 units 18 units A Vascular compartment B B B B B BB B B B BB B B B B B B A A Vd = 20 = 10 2 A B Vd = 20 = 1.1 18 B 2 units C C 2 units A A A Extravascular compartments of the body 18 units C A A A A A A A A A A A A A The calculated parameter for the Vd has no direct physical equivalent; therefore, it is usually denoted as the apparent Vd. Because the size of the compartments to which the drug may be distributed can vary with body size, Vd is sometimes expressed as Vd per kilogram of body weight (Vd/kg). A drug that is completely retained in the plasma compartment (Figure 3–1) will have a Vd equal to the plasma C CC CC CC CC CC CC CC CC CC C CC CC CC CC CC CC CC CC C CC C C C C C C C C C C C C C C C C C C C C C C C C C C C C C C C C CC CC CC CC CC CC CC CC CC C C C C C C C C C C C C C C C C C C C CC C CC C CC C CC C CC C CC C CC C C C CC C CC C CC CC CC CC CC CC CC CC CC CC CC CC C CC CC CC CC CC CC CC C CC CC CC CC CC CC CC CC CC CC Vd = 200 = 100 2 198 units FIGURE 3–1 Effect of drug binding on volume of distribution. Drug A diffuses freely between the 2 compartments and does not bind to macromolecules (heavy wavy lines) in the vascular or the extravascular compartments of the hypothetical organism in the diagram. With 20 units of the drug in the body, the steady-state distribution leaves a blood concentration of 2 units. Drug B, on the other hand, binds avidly to proteins in the blood. At equilibrium, only 2 units of the total are present in the extravascular volume, leaving 18 units still in the blood. In each case, the total amount of drug in the body is the same (20 units), but the apparent volumes of distribution are very different. Drug C is avidly bound to molecules in peripheral tissues, so that a larger total dose (200 units) is required to achieve measurable plasma concentrations. At equilibrium, 198 units are found in the peripheral tissues and only 2 units in the plasma, so that the calculated volume of distribution is greater than the physical volume of the system. 28 PART I Basic Principles volume (about 4% of body weight). The Vd of drugs that are normally bound to plasma proteins such as albumin can be altered by liver disease (through reduced protein synthesis) and kidney disease (through urinary protein loss). On the other hand, if a drug is avidly bound in peripheral tissues, the drug’s concentration in plasma may drop to very low values even though the total amount in the body is large. As a result, the Vd may greatly exceed the total physical volume of the body. For example, 50,000 liters is the average Vd for the drug quinacrine in persons whose average physical body volume is 70 liters. CLEARANCE Clearance (CL) relates the rate of elimination to the plasma concentration: Rate of elimination of drug Plasma drug concentration (Units = Volume per unit time) CL = (2) For a drug eliminated with first-order kinetics, clearance is a constant; that is, the ratio of rate of elimination to plasma concentration is the same over a broad range of plasma concentration (Figure 3–2). As in the case of Vd, clearance is sometimes expressed as CL per kg of body weight. The magnitudes of clearance for different drugs range from a small percentage of the blood flow to a maximum of the total blood flow to the organs of elimination. Clearance depends on the drug, blood flow, and the condition of the organs of elimination in the patient. The clearance of a particular drug by an individual organ is equivalent to the extraction capability of that organ for that drug times the rate of delivery of drug to the organ. Thus, the clearance of a drug that is very effectively extracted by an organ (ie, the blood is completely cleared of the drug as it passes through the organ) is often flow-limited. For such a drug, the total clearance from the body is a function of blood flow through the eliminating organ and is limited by the blood flow to that organ. In this situation, other conditions—cardiac disease, or other drugs that change blood flow—may have more dramatic effects on clearance than disease of the organ of elimination. Note that for drugs eliminated with zero-order kinetics (see Figure 1–3, right), elimination rate is constant and clearance is not constant. SKILL KEEPER 1: ZERO-ORDER ELIMINATION (SEE CHAPTER 1) Most drugs in clinical use obey the first-order kinetics rule described in the text. Can you name 3 important drugs that do not? The Skill Keeper Answer appears at the end of the chapter. HALF-LIFE Half-life (t1/2) is a derived parameter, completely determined by Vd and CL. Like clearance, half-life is a constant for drugs that follow first-order kinetics. Half-life can be determined graphically from a plot of the blood level versus time (eg, Figure 1–4) or from the following relationship: 0.693 × Vd CL (Units = Time) t V2 = Rate of elimination Clearance (CL) = (3) Plasma concentration (Cp) One must know both primary variables (Vd and CL) to predict changes in half-life. Disease, age, and other variables usually alter the clearance of a drug much more than they alter its Vd. The half-life determines the rate at which blood concentration rises during a constant infusion and falls after administration is stopped (Figure 3–3). The effect of a drug at 87–90% of its steady-state concentration is clinically indistinguishable from the steady-state effect; thus, 3–4 half-lives of dosing at a constant rate are considered adequate to produce the effect to be expected at steady state. Plasma concentration (Cp) Rate of elimination = CL x Cp 5 units/h elimination 2.5 units/h 1.25 units/h BIOAVAILABILITY Time (h) FIGURE 3–2 The clearance of the great majority of drugs is relatively constant over a broad range of plasma concentrations (Cp). Since elimination rate is equal to clearance times plasma concentration, the elimination rate will be rapid at first and slow as the concentration decreases. The bioavailability of a drug is the fraction (F) of the administered dose that reaches the systemic circulation. Bioavailability is defined as unity (or 100%) in the case of intravenous administration. After administration by other routes, bioavailability is generally reduced by incomplete absorption (and in the intestine, expulsion of drug by intestinal transporters), first-pass metabolism, and any distribution into other tissues that occurs before Percent of maximum CHAPTER 3 Pharmacokinetics 29 100 Stop infusion 75 50 Start infusion 25 0 0 2 4 6 8 2 4 6 8 10 Time (number of half-lives) FIGURE 3–3 Plasma concentration (plotted as percentage of maximum) of a drug given by constant intravenous infusion for 8 half-lives and then stopped. The concentration rises smoothly with time and always reaches 50% of steady state after 1 half-life, 75% after 2 half-lives, 87.5% after 3 half-lives, and so on. The decline in concentration after stopping drug administration follows the same type of curve: 50% is left after 1 half-life, 25% after 2 half-lives, and so on. The asymptotic approach to steady state on both increasing and decreasing limbs of the curve is characteristic of drugs that have first-order kinetics. the drug enters the systemic circulation. Even for drugs with equal bioavailabilities, entry into the systemic circulation occurs over varying periods of time, depending on the drug formulation and other factors. To account for such factors, the concentration appearing in the plasma is integrated over time to obtain an integrated total area under the plasma concentration curve (AUC, Figure 3–4). EXTRACTION SKILL KEEPER 2: FIRST-PASS EFFECT (SEE CHAPTER 1) The oral route of administration is the most likely to have a large first-pass effect and therefore low bioavailability. What tissues contribute to this effect? The Skill Keeper Answer appears at the end of the chapter. DOSAGE REGIMENS Removal of a drug by an organ can be specified as the extraction ratio, that is, the fraction or percentage of the drug removed from the perfusing blood during its passage through the organ (Figure 3–5). After steady-state concentration in plasma has been achieved, the extraction ratio is one measure of the elimination of the drug by that organ. Drugs that have a high hepatic extraction ratio have a large first-pass effect and the bioavailability of these drugs after oral administration is low. A dosage regimen is a plan for drug administration over a period of time. An optimal dosage regimen results in the achievement of therapeutic levels of the drug in the blood without exceeding the minimum toxic concentration. To maintain the plasma concentration within a specified range over long periods of therapy, a schedule of maintenance doses is used. If it is necessary to achieve the target plasma level rapidly, a loading dose may be used to “load” the Vd with the drug. Ideally, the dosing plan Multiple doses Plasma concentration (Cp) Plasma concentration (Cp) Single dose Intravenous AUC 20 Oral AUC 10 0 0 5 10 Time (h) 15 AUC 20 10 0 0 5 10 15 Time (h) FIGURE 3–4 The area under the curve (AUC) is used to calculate the bioavailability of a drug. The AUC can be derived from either singledose studies (left) or multiple-dose measurements (right). Bioavailability is calculated from AUC(route)/AUC(IV). 30 PART I Basic Principles Q Portal circulation Ci Co CLliver Oral dose Gut B. Loading Dosage If the therapeutic concentration must be achieved rapidly and the Vd is large, a large loading dose may be needed at the onset of therapy. This can be calculated from the following equation: Q Liver Systemic circulation Intravenous dose Loading dose = Remainder CLrenal FIGURE 3–5 The principles of organ extraction and first-pass effect are illustrated. Part of the administered oral dose (blue) is lost in the gut in the feces or to metabolism, and lost to metabolism in the liver before it enters the systemic circulation: This is the first-pass effect. The extraction of drug from the circulation by the liver is equal to blood flow (Q) times the difference between entering and leaving drug concentration, ie, Q × (Ci – Co). CL, clearance. (Modified and reproduced, with permission, from Katzung BG, editor: Basic & Clinical Pharmacology, 8th ed. McGraw-Hill, 2001.) is based on knowledge of both the minimum therapeutic and minimum toxic concentrations for the drug, as well as its clearance and Vd. A. Maintenance Dosage Because the maintenance rate of drug administration is equal to the rate of elimination at steady state (this is the definition of steady state), the maintenance dosage is a function of clearance (from Equation 2). Dosing rate = CL × Desired plasma concentratition Bioavailability (5) Note that clearance does not enter into this computation. If the loading dose is large (Vd much larger than blood volume), the dose should be given slowly to prevent toxicity due to excessively high plasma levels during the distribution phase. of the body CLother Vd × Desired plasma concentration Bioavailability THERAPEUTIC WINDOW The therapeutic window is the safe range between the minimum therapeutic concentration and the minimum toxic concentration of a drug. These data are used to determine the acceptable range of plasma levels when designing a dosing regimen. Thus, the minimum effective concentration usually determines the desired trough levels of a drug given intermittently, whereas the minimum toxic concentration determines the permissible peak plasma concentration. For example, the drug theophylline has a therapeutic concentration range of 8–20 mg/L but may be toxic at concentrations of more than 15–20 mg/L. The therapeutic window for a particular patient might thus be 8–16 mg/L (Figure 3–6). Unfortunately, for some drugs the therapeutic and toxic concentrations vary so greatly among patients that it is impossible to predict the therapeutic window in a given patient. Such drugs must be titrated individually in each patient. (4) Minimum toxic concentration Note that Vd is not involved in the calculation of maintenance dosing rate. The dosing rate computed for maintenance dosage is the average dose per unit time. When performing such calculations, make certain that the units are in agreement throughout. For example, if clearance is given in mL/min, the resulting dosing rate is a per minute rate. Because convenience of administration is desirable for chronic therapy, doses should be given orally if possible and only once or a few times per day. The size of the daily dose (dose per minute × 60 min/h × 24 h/d) is a simple extension of the preceding information. The number of doses to be given per day is usually determined by the half-life of the drug and the difference between the minimum therapeutic and toxic concentrations (see Therapeutic Window, below). If it is important to maintain a concentration above the minimum therapeutic level at all times, either a larger dose is given at long intervals or smaller doses at more frequent intervals. If the difference between the toxic and therapeutic concentrations is small, then smaller and more frequent doses must be administered to prevent toxicity. Cp (mg/L) 20 Therapeutic window 10 Minimum effective concentration 0 0 5 10 15 Time (h) FIGURE 3–6 The therapeutic window for theophylline in a typical patient. The minimum effective concentration in this patient was found to be 8 mg/L; the minimum toxic concentration was found to be 16 mg/L. The therapeutic window is indicated by the blue area. To maintain the plasma concentration (Cp) within the window, this drug must be given at least once every half-life (7.5 h in this patient) because the minimum effective concentration is half the minimum toxic concentration and Cp will decay by 50% in 1 half-life. (Note: This concept applies to drugs given in the ordinary, promptrelease form. Slow-release formulations can often be given at longer intervals.) CHAPTER 3 Pharmacokinetics QUESTIONS ADJUSTMENT OF DOSAGE WHEN ELIMINATION IS ALTERED BY DISEASE Renal disease or reduced cardiac output often reduces the clearance of drugs that depend on renal elimination. Alteration of clearance by liver disease is less common but may also occur. Impairment of hepatic clearance occurs (for high extraction drugs) when liver blood flow is reduced, as in heart failure, and in severe cirrhosis and other forms of liver failure. Because it is important in the elimination of drugs, assessing renal function is important in estimating dosage in patients. The most important renal variable in drug elimination is glomerular filtration rate (GFR), and creatinine clearance (CLcr) is a convenient approximation of GFR. The dosage in a patient with renal impairment may be corrected by multiplying the average dosage for a normal person times the ratio of the patient’s altered creatinine clearance (CLcr) to normal creatinine clearance (approximately 100 mL/min, or 6 L/h in a young adult). Corrected dosage = Average dosage × Patient’s CL cr 100 mL/min (6) This simplified approach ignores nonrenal routes of clearance that may be significant. If a drug is cleared partly by the kidney and partly by other routes, Equation 6 should be applied to the part of the dose that is eliminated by the kidney. For example, if a drug is 50% cleared by the kidney and 50% by the liver and the normal dosage is 200 mg/d, the hepatic and renal elimination rates are each 100 mg/d. Therefore, the corrected dosage in a patient with a creatinine clearance of 20 mL/min will be: Dosage = 100 mg/d (liver) + 100 mg/d 20 mL/min (kidney) × 100 mL/min Dosage = 100 mg/d + 20 mg/d = 120 mg/d (7) Renal function is altered by many diseases and is often decreased in older patients. CLcr can be measured directly, but this requires careful measurement of both serum creatinine concentration and a timed total urine creatinine. A common shortcut that requires only the serum (or plasma) creatinine measurement (Scr) is the use of an equation. One such equation in common use is the CockcroftGault equation: CL cr (mL/min) = (140 − Age) × body weight (kg) 72 × S cr (8) The result is multiplied by 0.85 for females. A similar equation for GFR is the MDRD equation: GFR (mL/min/1.73 m2 body surface area) = 175 × (0.742 if female) × (1.212 if African American) S1.154 × Age 0.203 cr 31 (9) 1. Mr Jones has zero kidney function and is undergoing hemodialysis while awaiting a kidney transplant. He takes metformin for type 2 diabetes mellitus and was previously stabilized (while his kidney function was adequate) at a dosage of 500 mg twice daily, given orally. The plasma concentration at this dosage with normal kidney function was found to be 1.4 mg/L. He has been on dialysis for 10 days and metformin toxicity is suspected. A blood sample now shows a metformin concentration of 4.2 mg/L. What was Mr. Jones’ clearance of metformin while his kidney function was normal? (A) 238 L/d (B) 29.8 L/h (C) 3 L/d (D) 238 L/h (E) 30 L/min 2. Ms Smith, a 65-year-old woman with pneumonia, was given tobramycin, 150 mg, intravenously. After 20 minutes, the plasma concentration was measured and was found to be 3 mg/L. Assuming no elimination of the drug in 20 minutes, what is the apparent volume of distribution of tobramycin in Ms Smith? (A) 3 L/min (B) 3 L (C) 50 L (D) 7 L (E) 0.1 mg/min 3. St John’s Wort, a popular botanical remedy, is a potent inducer of hepatic phase I CYP3A4 enzymes. Verapamil and phenytoin are both eliminated from the body by metabolism in the liver. Verapamil has a clearance of 1.5 L/min, approximately equal to liver blood flow, whereas phenytoin has a clearance of 0.1 L/min. Based on this fact, which of the following is most correct? (A) St John’s Wort will increase the half-life of phenytoin and verapamil (B) St John’s Wort will decrease the volume of distribution of CYP3A4 substrates (C) St John’s Wort will decrease the hepatic extraction of phenytoin (D) St John’s Wort will decrease the first-pass effect for verapamil (E) St John’s Wort will increase the clearance of phenytoin 32 PART I Basic Principles 4. A 55-year-old man with severe rheumatoid arthritis has elected to participate in the trial of a new immunosuppressive agent. It is given by constant intravenous infusion of 8 mg/h. Plasma concentrations (Cp) are measured with the results shown in the following table. Time After Start of Infusion (h) Plasma Concentration (mg/L) 1 0.8 2 1.2 8 3.0 10 3.6 20 3.84 40 4.0 What conclusion can be drawn from these data? (A) Clearance is 2 L/h (B) Doubling the rate of infusion would result in a plasma concentration of 16 mg/L at 40 h (C) Elimination follows zero-order kinetics (D) Half-life is 8 h (E) Volume of distribution is 30 L 5. You are the only physician in a clinic that is cut off from the outside world by violent storms, flooding, and landslides. A 15-year-old girl is brought to the clinic with severe asthmatic wheezing. Because of the lack of other drugs, you decide to use intravenous theophylline for treatment. The pharmacokinetics of theophylline include the following average parameters: Vd 35 L; CL 48 mL/min; half-life 8 h. If an intravenous infusion of theophylline is started at a rate of 0.48 mg/min, how long would it take to reach 93.75% of the final steadystate concentration? (A) Approximately 48 min (B) Approximately 7.4 h (C) Approximately 8 h (D) Approximately 24 h (E) Approximately 32 h 6. A 74-year-old retired mechanic is admitted with a myocardial infarction and a severe acute cardiac arrhythmia. You decide to give lidocaine to correct the arrhythmia. A continuous intravenous infusion of lidocaine, 1.92 mg/min, is started at 8 am. The average pharmacokinetic parameters of lidocaine are: Vd 77 L; clearance 640 mL/min; half-life 1.4 h. What is the expected steady-state plasma concentration? (A) 40 mg/L (B) 3.0 mg/L (C) 0.025 mg/L (D) 7.2 mg/L (E) 3.46 mg/L 7. A new drug is under study in phase 1 trials. It is found that this molecule is avidly taken up by extravascular tissues so that the final total amount in the extravascular compartment at steady state is 100 times the amount remaining in the blood plasma. What is the probable volume of distribution in a hypothetical person with 8 L of blood and 4 L of plasma? (A) Insufficient data to calculate (B) 8 L (C) 14.14 L (D) 100 L (E) 404 L 8. A 63-year-old woman in the intensive care unit requires an infusion of procainamide. Its half-life is 2 h. The infusion is begun at 9 am. At 1 pm on the same day, a blood sample is taken; the drug concentration is found to be 3 mg/L. What is the probable steady-state drug concentration after 16 or more hours of infusion? (A) 3 mg/L (B) 4 mg/L (C) 6 mg/L (D) 9.9 mg/L (E) 15 mg/L 9. A 30-year-old man is brought to the emergency department in a deep coma. Respiration is severely depressed and he has pinpoint pupils. His friends state that he self-administered a large dose of morphine 6 h earlier. An immediate blood analysis shows a morphine blood level of 0.25 mg/L. Assuming that the Vd of morphine in this patient is 200 L and the half-life is 3 h, how much morphine did the patient inject 6 h earlier? (A) 25 mg (B) 50 mg (C) 100 mg (D) 200 mg (E) Not enough data to predict 10. Gentamicin, an aminoglycoside antibiotic, is sometimes given in intermittent intravenous bolus doses of 100 mg 3 times a day to achieve target peak plasma concentrations of about 5 mg/L. Gentamicin’s clearance (normally 5.4 L/h/70 kg) is almost entirely by glomerular filtration. Your patient, however, is found to have a creatinine clearance one third of normal. What should your modified dosage regimen for this patient be? (A) 20 mg 3 times a day (B) 33 mg 3 times a day (C) 72 mg 3 times a day (D) 100 mg 2 times a day (E) 150 mg 2 times a day CHAPTER 3 Pharmacokinetics ANSWERS 1. Examination questions often provide more information than is needed—to test the student’s ability to classify and organize data. In question 1, the data provided for Mr Jones on dialysis is irrelevant, even though choice A, 238 L/d, is the correct clearance while on dialysis. By definition, clearance is calculated by dividing the rate of elimination by the plasma concentration: Rate in = rate out (elimination rate) at steady state (ss) 6. The drug is being administered continuously and the steadystate concentration (Cpss) for a continuously administered drug is given by the equation in question 1. Thus, Dosage = Plasma levelss × Clearance 1.92 mg/min = Cp ss × CL Rearranging: CL = rate in Cp(ss) Cp ss = 1.92 mg/min CL CL = 1000 mg/24 h 1.4 mg/L Cp ss = 1.92 mg/min 640 mL/min CL = 29.8 L/h The answer is B. 2. The volume of distribution (Vd) is the apparent volume into which the loading dose is distributed. It is calculated by dividing the dose by the resulting plasma concentration, Cp: loading dose Cp 150 mg Vd = 3 mg/L Vd = Vd = 50 L The answer is C. 3. Induction of phase I metabolizing enzymes will decrease the half-life of substrates of these enzymes. P450 enzyme induction has no effect on volume of distribution. Hepatic extraction, the first-pass effect, and clearance for CYP3A4 substrates will be increased by inducers. However, the extraction of verapamil is already equal to the hepatic blood flow, so further increase in metabolism will not increase clearance of this drug. The answer is E. 4. By inspection of the data in the table, it is clear that the steady-state plasma concentration is approximately 4 mg/L. None of the measured concentrations is equal to one half of the steady state value, so the half-life is not immediately apparent. However, according to the constant infusion principle (Figure 3–3), 2 half-lives are required to reach 75% of the final concentration; 75% (3.0 mg/L) of the final steady-state concentration was reached at 8 h. If 8 h equals 2 half-lives, the half-life must be 4 h. Rearranging the equation for maintenance dosing (dosing rate = CL × Cp), it can be determined that the clearance (CL) = dosing rate/plasma concentration (Cp), or 2 L/h. The volume of distribution (Vd) can be calculated from the half-life equation (t1/2 = 0.693 × Vd/CL) and is equal to 11.5 L. This drug follows first-order kinetics, as indicated by the progressive approach to the steady-state plasma concentration. The answer is A. 5. The approach of the drug plasma concentration to steady-state concentration during continuous infusion follows a stereotypical curve (Figure 3–3) that rises rapidly at first and gradually reaches a plateau. It reaches 50% of steady state at 1 half-life, 75% at 2 half-lives, 87.5% at 3, 93.75% at 4, and progressively halves the difference between its current level and 100% of steady state with each half-life. The answer is E, 32 h, or 4 half-lives. 33 Cp ss = 0.003 mg/mL or 3 mg/L The answer is B. 7. Let Z be the amount in the blood plasma. If the amount in the rest of the body is 100 times greater, then the total amount in the body is 101Z. The concentration in the blood plasma (Cp) is Z/4 L. According to the definition: Vd = amount in body Cp Vd = 101Z = 101 × 4 L = 404 L Z/4 L The answer is E. 8. According to the curve that relates plasma concentration to infusion time (Figure 3–3), a drug reaches 50% of its final steady-state concentration in 1 half-life, 75% in 2 half-lives, etc. From 9 am to 1 pm is 4 h, or 2 half-lives. Therefore, the measured concentration at 1 pm is 75% of the steady-state value (0.75 × Cpss). The steady-state concentration is 3 mg/L divided by 0.75, or 4 mg/L. The answer is B. 9. According to the curve that relates the decline of plasma concentration to time as the drug is eliminated (Figure 3–3), the plasma concentration of morphine was 4 times higher immediately after administration than at the time of the measurement, which occurred 6 h, or 2 half-lives, later. Therefore, the initial plasma concentration was 1 mg/L. Since the amount in the body at any time is equal to Vd × plasma concentration (text Equation 1), the amount injected was 200 L × 1 mg/L, or 200 mg. The answer is D. 10. If the drug is cleared almost entirely by the kidney and creatinine clearance is reduced to one third of normal, the total daily dose should also be reduced to one third. The answer is B. SKILL KEEPER 1 ANSWER: ZERO-ORDER ELIMINATION (SEE CHAPTER 1) The 3 important drugs that follow zero-order rather than firstorder kinetics are ethanol, aspirin, and phenytoin. 34 PART I Basic Principles SKILL KEEPER 2 ANSWER: FIRST-PASS EFFECT (SEE CHAPTER 1) The oral route of administration entails passage of the drug through the gastric and intestinal contents, the epithelium and other tissues of the intestinal wall, the portal blood, and the liver before it enters the systemic circulation for distribution to the body. Metabolism by enzymes in any of these tissues, expulsion by drug transporters, and excretion into the bile all may contribute to the first-pass effect of oral administration. CHECKLIST When you complete this chapter, you should be able to: ❑ Estimate the half-life of a drug based on its clearance and volume of distribution or from a graph of its plasma concentration over time. ❑ Calculate loading and maintenance dosage regimens for oral or intravenous admin- istration of a drug when given the following information: minimum therapeutic concentration, minimum toxic concentration, oral bioavailability, clearance, and volume of distribution. ❑ Calculate the dosage adjustment required for a patient with impaired renal function. CHAPTER 3 Summary Table Major Concept Description Loading dose The dose required to achieve a specific plasma drug concentration level (Cp) with a single administration. Because this requires filling the volume of distribution (Vd), the calculation uses the volume of distribution (Vd) equation as: Loading dose = Cp(target) × Vd; has units of mg Maintenance dose The dose required for regular administration to maintain a target plasma level. Because this requires restoring the amount of drug lost to elimination (clearance, CL), the calculation uses the clearance equation as: Maintenance dose = Cp(target) × CL; has units of mg per time Half-life The half-life concept is useful in predicting the time course of falling drug levels after administration is stopped, and in predicting the time course of increase in drug level when repeated administration is begun—see Figure 3–3 Therapeutic window The therapeutic window is much more useful as a clinical measure of drug safety and as a guide to dosage than the older therapeutic index. The classic therapeutic index, TI, determined from animal measures of therapeutically effective dosage and lethal dosage, is inapplicable to human therapeutics, whereas the minimum therapeutic dosage and the minimum toxic dosage are readily determined in clinical trials Bioavailability The fraction or percentage of the dose of a drug that reaches the systemic circulation. The bioavailability of a drug given intravenously is therefore 100% C H A P T E R 4 Drug Metabolism All organisms are exposed to foreign chemical compounds (xenobiotics) in the air, water, and food. To ensure elimination of pharmacologically active xenobiotics as well as to terminate the action of many endogenous substances, evolution has provided metabolic pathways that alter such compounds’ activity and their susceptibility to excretion. Drug metabolism Phase I reactions Phase II reactions Genetic factors THE NEED FOR DRUG METABOLISM Many cells in tissues that act as portals for entry of external molecules into the body (eg, pulmonary epithelium, intestinal epithelium) contain transporter molecules (MDR family [P-glycoproteins], MRP family, others) that expel unwanted molecules immediately after absorption. However, many foreign molecules evade these gatekeepers and are absorbed. Therefore, all higher organisms, especially terrestrial animals, require mechanisms for ridding themselves of toxic foreign molecules after they are absorbed, as well as Induction of drug metabolism Inhibition of drug metabolism mechanisms for excreting undesirable substances produced within the body. Biotransformation of drugs is one such process. It is an important mechanism by which the body terminates the action of many drugs. In some cases, it serves to activate prodrugs. Most drugs are relatively lipid-soluble as given, a characteristic needed for absorption across membranes. The same property would result in very slow removal from the body because the unchanged molecule would also be readily reabsorbed from the urine in the renal tubule. The body hastens excretion by transforming many drugs to less lipid-soluble, less readily reabsorbed forms. High-Yield Terms to Learn Phase I reactions Reactions that convert the parent drug to a more polar (water-soluble) or more reactive product by unmasking or inserting a polar functional group such as ´OH, ´SH, or ´NH2 Phase II reactions Reactions that increase water solubility by conjugation of the drug molecule with a polar moiety such as glucuronate, acetate, or sulfate CYP isozymes Cytochrome P450 enzyme species (eg, CYP2D6 and CYP3A4) that are responsible for much of drug metabolism. Many isoforms of CYP have been recognized Enzyme induction Stimulation of drug-metabolizing capacity; usually manifested in the liver by increased synthesis of smooth endoplasmic reticulum (which contains high concentrations of phase I enzymes) P-glycoprotein, MDR-1 An ATP-dependent transport molecule found in many epithelial and cancer cells. The transporter expels drug molecules from the cytoplasm into the extracellular space. In epithelial cells, expulsion is via the external or luminal face 35 36 PART I Basic Principles TABLE 4–1 Examples of phase I drug-metabolizing reactions. Reaction Type Typical Drug Substrates Oxidations, P450 dependent Hydroxylation N-dealkylation O-dealkylation N-oxidation S-oxidation Deamination Amphetamines, barbiturates, phenytoin, warfarin Caffeine, morphine, theophylline Codeine Acetaminophen, nicotine Chlorpromazine, cimetidine, thioridazine Amphetamine, diazepam Oxidations, P450 independent Amine oxidation Dehydrogenation Epinephrine Chloral hydrate, ethanol Reductions Chloramphenicol, clonazepam, dantrolene, naloxone Hydrolyses Esters Amides Aspirin, clofibrate, procaine, succinylcholine Indomethacin, lidocaine, procainamide TYPES OF METABOLIC REACTIONS A. Phase I Reactions Phase I reactions include oxidation (especially by the cytochrome P450 group of enzymes, also called mixed-function oxidases), reduction, deamination, and hydrolysis. Examples of phase I drug substrates are listed in Table 4–1. These enzymes are found in high concentrations in the smooth endoplasmic reticulum of the liver. They are not highly selective in their substrates, so a relatively small number of P450 isoforms are able to metabolize thousands of drugs. Of the drugs metabolized by phase I cytochrome P450s, approximately 75% are metabolized by just two: CYP3A4/5 or CYP2D6. CYP3A4 and CYP3A5 alone are responsible for the metabolism of approximately 50% of drugs. Nevertheless, some selectivity can be detected, and optical enantiomers, in particular, are often metabolized at different rates. B. Phase II Reactions Phase II reactions are synthetic reactions that involve addition (conjugation) of subgroups to —OH, —NH2, and —SH functions on the drug molecule. The subgroups that are added include glucuronate, acetate, glutathione, glycine, sulfate, and methyl groups. Most of these groups are relatively polar and make the product less lipid-soluble than the original drug molecule. Examples of phase II reactions are listed in Table 4–2. Like phase I enzymes, phase II enzymes are not very selective. Drugs that are metabolized by both routes may undergo phase II metabolism before or after phase I. SITES OF DRUG METABOLISM The most important organ for drug metabolism is the liver. The kidneys play an important role in the metabolism of some drugs. A few drugs (eg, esters) are metabolized in many tissues (eg, liver, TABLE 4–2 Examples of phase II drug-metabolizing reactions. Reaction Type Typical Drug Substrates Glucuronidation Acetaminophen, diazepam, digoxin, morphine, sulfamethiazole Acetylation Clonazepam, dapsone, isoniazid, mescaline, sulfonamides Glutathione conjugation Ethacrynic acid, reactive phase I metabolite of acetaminophen Glycine conjugation Deoxycholic acid, nicotinic acid (niacin), salicylic acid Sulfation Acetaminophen, methyldopa Methylation Dopamine, epinephrine, histamine, norepinephrine, thiouracil Adapted, with permission, from Katzung BG, editor: Basic & Clinical Pharmacology, 12th ed. McGraw-Hill, 2012. CHAPTER 4 Drug Metabolism blood, intestinal wall) because of the wide distribution of their enzymes. DETERMINANTS OF BIOTRANSFORMATION RATE The rate of biotransformation of a drug may vary markedly among different individuals. This variation is most often due to genetic or drug-induced differences. For a few drugs, age or disease-related differences in drug metabolism are significant. In humans, gender is important for only a few drugs. (First-pass metabolism of ethanol is greater in men than in women.) On the other hand, a variety of drugs may induce or inhibit drug-metabolizing enzymes to a very significant extent. Smoking is a common cause of enzyme induction in the liver and lung and may increase the metabolism of some drugs. Because the rate of biotransformation is often the primary determinant of clearance, variations in drug metabolism must be considered carefully when designing or modifying a dosage regimen. A. Genetic Factors Several drug-metabolizing systems have long been known to differ among families or populations in genetically determined ways. Because recent advances in genomic techniques are making it possible to screen for a huge variety of polymorphisms, it is expected that pharmacogenomics will become an important part of patient evaluation in the near future, influencing both drug choice and drug dosing (see Chapter 5). B. Effects of Other Drugs Coadministration of certain agents may alter the disposition of many drugs. Mechanisms include the following: 1. Enzyme induction—Induction (increased rate and extent of metabolism) usually results from increased synthesis of 37 cytochrome P450 drug-oxidizing enzymes in the liver as well as the cofactor, heme. Several cytoplasmic drug receptors have been identified that result in activation of the genes for P450 isoforms. Drugs and other xenobiotics that increase enzyme activity are known as inducers. Many isozymes of the P450 family exist, and most inducers selectively increase one or more subgroups of isozymes. Common inducers of a few of these isozymes and the drugs whose metabolism is increased are listed in Table 4–3. Several days are usually required to reach maximum induction; a similar amount of time is required to regress after withdrawal of the inducer. The most common strong inducers of drug metabolism are carbamazepine, phenobarbital, phenytoin, and rifampin. 2. Enzyme inhibition—A few common inhibitors and the drugs whose metabolism is diminished are listed in Table 4–4. The inhibitors of drug metabolism most likely to be involved in serious drug interactions are amiodarone, cimetidine, furanocoumarins present in grapefruit juice, azole antifungals, and the HIV protease inhibitor ritonavir. Suicide inhibitors are drugs that are metabolized to products that irreversibly inhibit the metabolizing enzyme. Such agents include ethinyl estradiol, norethindrone, spironolactone, secobarbital, allopurinol, fluroxene, and propylthiouracil. Metabolism may also be decreased by pharmacodynamic factors such as a reduction in blood flow to the metabolizing organ (eg, propranolol reduces hepatic blood flow). 3. Inhibitors of intestinal P-glycoprotein—MDR-1, also known as P-glycoprotein (P-gp), is an important modulator of intestinal drug transport and usually functions to expel drugs from the intestinal mucosa into the lumen, thus contributing to presystemic (first pass) elimination. P-gp and other members of the MDR family are also found in the blood-brain barrier and in drug-resistant cancer cells. Drugs that inhibit intestinal P-gp TABLE 4–3 A partial list of drugs that significantly induce P450-mediated drug metabolism in humans. CYP Family Induced Important Inducers Drugs Whose Metabolism Is Induced 1A2 Benzo[a]pyrene (from tobacco smoke), carbamazepine, phenobarbital, rifampin, omeprazole Acetaminophen, clozapine, haloperidol, theophylline, tricyclic antidepressants, (R)-warfarin 2C9 Barbiturates, especially phenobarbital, phenytoin, primidone, rifampin Barbiturates, celecoxib, chloramphenicol, doxorubicin, ibuprofen, phenytoin, chlorpromazine, steroids, tolbutamide, (S)-warfarin 2C19 Carbamazepine, phenobarbital, phenytoin, rifampin Diazepam, phenytoin, topiramate, tricyclic antidepressants, (R)-warfarin 2E1 Ethanol, isoniazid Acetaminophen, enflurane, ethanol (minor), halothane 3A4 Barbiturates, carbamazepine, corticosteroids, efavirenz, phenytoin, rifampin, pioglitazone, St. John’s wort Antiarrhythmics, antidepressants, azole antifungals, benzodiazepines, calcium channel blockers, cyclosporine, delavirdine, doxorubicin, efavirenz, erythromycin, estrogens, HIV protease inhibitors, nefazodone, paclitaxel, proton pump inhibitors, HMG-CoA reductase inhibitors, rifabutin, rifampin, sildenafil, SSRIs, tamoxifen, trazodone, vinca alkaloids SSRIs, selective serotonin reuptake inhibitors. 38 PART I Basic Principles TABLE 4–4 A partial list of drugs that significantly inhibit P450-mediated drug metabolism in humans. CYP Family Inhibited Inhibitors Drugs Whose Metabolism Is Inhibited 1A2 Cimetidine, fluoroquinolones, grapefruit juice, macrolides, isoniazid, zileuton Acetaminophen, clozapine, haloperidol, theophylline, tricyclic antidepressants, (R)-warfarin 2C9 Amiodarone, chloramphenicol, cimetidine, isoniazid, metronidazole, SSRIs, zafirlukast Barbiturates, celecoxib, chloramphenicol, doxorubicin, ibuprofen, phenytoin, chlorpromazine, steroids, tolbutamide, (S)-warfarin 2C19 Fluconazole, omeprazole, SSRIs Diazepam, phenytoin, topiramate, (R)-warfarin 2D6 Amiodarone, cimetidine, quinidine, SSRIs Antiarrhythmics, antidepressants, beta blockers, clozapine, flecainide, lidocaine, mexiletine, opioids 3A4 Amiodarone, azole antifungals, cimetidine, clarithromycin, cyclosporine, diltiazem, erythromycin, fluoroquinolones, grapefruit juice, HIV protease inhibitors, metronidazole, quinine, SSRIs, tacrolimus Antiarrhythmics, antidepressants, azole antifungals, benzodiazepines, calcium channel blockers, cyclosporine, delavirdine, doxorubicin, efavirenz, erythromycin, estrogens, HIV protease inhibitors, nefazodone, paclitaxel, proton pump inhibitors, HMG-CoA reductase inhibitors, rifabutin, rifampin, sildenafil, SSRIs, tamoxifen, trazodone, vinca alkaloids SSRIs, selective serotonin reuptake inhibitors. mimic drug metabolism inhibitors by increasing bioavailability; coadministration of P-gp inhibitors may result in toxic plasma concentrations of drugs given at normally nontoxic dosage. P-gp inhibitors include verapamil, mibefradil (a calcium channel blocker no longer on the market), and furanocoumarin components of grapefruit juice. Important drugs that are normally expelled by P-gp (and are therefore potentially more toxic when given with a P-gp inhibitor) include digoxin, cyclosporine, and saquinavir. Enzyme inducers (eg, ethanol) may increase acetaminophen toxicity because they increase phase I metabolism more than phase II metabolism, thus resulting in increased production of the reactive metabolite. (Phase II) Drug metabolism is not synonymous with drug inactivation. Some drugs are converted to active products by metabolism. If these products are toxic, severe injury may result under some circumstances. An important example is acetaminophen when taken in large overdoses (Figure 4–1). Acetaminophen is conjugated to harmless glucuronide and sulfate metabolites when it is taken in recommended doses by patients with normal liver function. If a large overdose is taken, however, the phase II metabolic pathways are overwhelmed, and a phase I P450-dependent system converts some of the drug to a reactive intermediate (N-acetyl-p-benzoquinoneimine). This intermediate is conjugated with glutathione to a third harmless product if glutathione stores are adequate. If glutathione stores are exhausted, however, the reactive intermediate combines with sulfhydryl groups on essential hepatic cell proteins, resulting in cell death. Prompt administration of other sulfhydryl donors (eg, acetylcysteine) may be life-saving after an overdose. In severe liver disease, stores of glucuronide, sulfate, and glutathione may be depleted, making the patient more susceptible to hepatic toxicity with near-normal doses of acetaminophen. P450 induction + Ac-sulfate Cytochrome P450 (Phase I) Reactive electrophilic compound (Ac*) Liver disease GSH − TOXIC METABOLISM (Phase II) Ac Ac-glucuronide Gs-Ac* Ac-mercapturate Cell macromolecules (protein) Ac*- protein Hepatic cell death FIGURE 4–1 Metabolism of acetaminophen (Ac) to harmless conjugates or to toxic metabolites. Acetaminophen glucuronide, acetaminophen sulfate, and the mercapturate conjugate of acetaminophen all are nontoxic phase II conjugates. Ac* is the toxic, reactive phase I metabolite, N-acetyl-p-benzoquinoneimine. Transformation to the reactive metabolite occurs when hepatic stores of sulfate, glucuronide, and glutathione (GSH, Gs) are depleted or overwhelmed or when phase I enzymes have been induced. CHAPTER 4 Drug Metabolism QUESTIONS Questions 1–2. You are planning to treat chronic major depression in a 35-year-old patient with recurrent suicidal thoughts. She has several comorbid conditions that require drug therapy. You are concerned about drug interactions caused by changes in drug metabolism in this patient. 1. Drug metabolism in humans usually results in a product that is (A) Less lipid soluble than the original drug (B) More likely to distribute intracellularly (C) More likely to be reabsorbed by kidney tubules (D) More lipid soluble than the original drug (E) Less water soluble than the original drug 2. If therapy with multiple drugs causes induction of drug metabolism in your depressed patient, it will (A) Be associated with increased smooth endoplasmic reticulum (B) Be associated with increased rough endoplasmic reticulum (C) Be associated with decreased enzymes in the soluble cytoplasmic fraction (D) Require 3–4 months to reach completion (E) Be irreversible 3. Which of the following factors is likely to increase the duration of action of a drug that is metabolized by CYP3A4 in the liver? (A) Chronic administration of rifampin during therapy with the drug in question (B) Chronic therapy with amiodarone (C) Displacement from tissue-binding sites by another drug (D) Increased cardiac output (E) Chronic administration of carbamazepine 4. Reports of cardiac arrhythmias caused by unusually high blood levels of 2 antihistamines, terfenadine and astemizole, led to their removal from the market. Which of the following best explains these effects? (A) Concomitant treatment with rifampin (B) Use of these drugs by chronic alcoholics (C) Use of these drugs by chronic smokers (D) Treatment of these patients with ketoconazole, an azole antifungal agent 5. Which of the following agents, when used in combination with other anti-HIV drugs, permits dose reductions? (A) Cimetidine (B) Efavirenz (C) Ketoconazole (D) Procainamide (E) Quinidine (F) Ritonavir (G) Succinylcholine (H) Verapamil 39 6. Which of the following drugs may inhibit the hepatic microsomal P450 responsible for warfarin metabolism? (A) Amiodarone (B) Ethanol (C) Phenobarbital (D) Procainamide (E) Rifampin 7. Which of the following drugs, if used chronically, is most likely to increase the toxicity of acetaminophen? (A) Cimetidine (B) Ethanol (C) Ketoconazole (D) Procainamide (E) Quinidine (F) Ritonavir (G) Succinylcholine (H) Verapamil 8. Which of the following drugs has higher first-pass metabolism in men than in women? (A) Cimetidine (B) Ethanol (C) Ketoconazole (D) Procainamide (E) Quinidine (F) Ritonavir (G) Succinylcholine (H) Verapamil 9. Which of the following drugs is an established inhibitor of P-glycoprotein (P-gp) drug transporters? (A) Cimetidine (B) Ethanol (C) Ketoconazole (D) Procainamide (E) Quinidine (F) Ritonavir (G) Succinylcholine (H) Verapamil 10. Which of the following cytochrome isoforms is responsible for metabolizing the largest number of drugs? (A) CYP1A2 (B) CYP2C9 (C) CYP2C19 (D) CYP2D6 (E) CYP3A4 ANSWERS 1. Biotransformation usually results in a product that is less lipidsoluble. This facilitates elimination of drugs that would otherwise be reabsorbed from the renal tubule. The answer is A. 2. The smooth endoplasmic reticulum, which contains the mixed-function oxidase drug-metabolizing enzymes, is selectively increased by inducers. The answer is A. 40 PART I Basic Principles 3. Rifampin and carbamazepine can induce drug-metabolizing enzymes and thereby may reduce the duration of drug action. Displacement of drug from tissue may transiently increase the intensity of the effect but decreases the volume of distribution. Amiodarone is recognized as an inhibitor of P450 and may decrease clearance of drugs metabolized by CYP2C9, CYP2D6, and CYP3A4. The answer is B. 4. Treatment with rifampin and chronic alcohol use are associated with increased drug metabolism and lower, not higher, blood levels. Ketoconazole, itraconazole, erythromycin, and some substances in grapefruit juice slow the metabolism of certain older non-sedating antihistamines (Chapter 16). The answer is D. 5. Ritonavir inhibits hepatic drug metabolism, and its use at low doses in combination regimens has permitted dose reductions of other HIV protease inhibitors (eg, indinavir). The answer is F. 6. Amiodarone is an important antiarrhythmic drug and has a well-documented ability to inhibit the hepatic metabolism of many drugs. The answer is A. 7. Acetaminophen is normally eliminated by phase II conjugation reactions. The drug’s toxicity is caused by an oxidized reactive metabolite produced by phase I oxidizing P450 enzymes. Ethanol and certain other drugs induce P450 enzymes and thus reduce the hepatotoxic dose. Alcoholic cirrhosis reduces the hepatotoxic dose even more. The answer is B. 8. Ethanol is subject to metabolism in the stomach as well as in the liver. Independent of body weight and other factors, men have greater gastric ethanol metabolism and thus a lower ethanol bioavailability than women. The answer is B. 9. Verapamil is an inhibitor of P-glycoprotein drug transporters and has been used to enhance the cytotoxic actions of methotrexate in cancer chemotherapy. The answer is H. 10. While CYP2D6 is responsible for metabolizing approximately 25% of drugs, CYP3A4 is involved in almost 50% of such reactions. The answer is E. CHECKLIST When you complete this chapter, you should be able to: ❑ List the major phase I and phase II metabolic reactions. Know which P450 isoform is responsible for the greatest number of important reactions. ❑ Describe the mechanism of hepatic enzyme induction and list 3 drugs that are known to cause it. ❑ List 3 drugs that inhibit the metabolism of other drugs. ❑ Describe some of the effects of smoking, liver disease, and kidney disease on drug elimination. ❑ Describe the pathways by which acetaminophen is metabolized (1) to harmless prod- ucts if normal doses are taken and (2) to hepatotoxic products if an overdose is taken. CHAPTER 4 Summary Table Major Concept Description Drug metabolism vs drug elimination Termination of drug action requires either removal of the drug from the body (excretion) or modification of the drug molecule (metabolism) so that it no longer has an effect. Both methods constitute drug elimination, and both are very important in the clinical use of drugs. Almost all drugs (or their metabolites) are eventually excreted, but for many, excretion occurs only some time after they have been metabolized to inactive products Induction and inhibition of drug metabolism A large number of drugs alter their own metabolism and the metabolism of other drugs either by inducing the synthesis of larger amounts of the metabolizing enzymes (usually P450 enzymes in the liver) or by inhibiting those enzymes. Some drugs both inhibit (acutely) and induce (with chronic administration) drug metabolism Pharmacogenomic variation in drug metabolism Genetic variations in drug metabolism undoubtedly occur for many drugs. Specific differences have been defined for (1) succinylcholine and similar esters, (2) procainamide and similar amines, and (3) a miscellaneous group that includes β blockers, antidepressants, and others (see Chapter 5) Toxic metabolism Some substances are metabolized to toxic molecules by drug-metabolizing enzymes. Important examples include methyl alcohol, ethylene glycol, and, at high doses or in the presence of liver disease, acetaminophen. See Figure 4–1 and Chapter 23 C A P T E R 5 Pharmacogenomics Pharmacogenomics is a rapidly growing area of knowledge regarding the genetic variations that influence drug metabolism and drug effects. Most of the research in this field to date has involved phase I or phase II drug metabolism and H drug transport. Application of genomic analysis of individual patients to selection of specific drugs and drug dosage is under investigation. Pharmacogenomics Definitions Enzymes INTRODUCTION The inheritance of genetic information via the double DNA helix is now well-understood. The decoding of the human genome and of many animal and plant genomes has opened a field of research into the molecular basis of variations between individuals and among populations. The identification of the specific genes (or groups of genes) that affect drug responses is still incomplete, but knowledge about a small number of these genes of pharmacologic significance has suggested the possibility that “personalized medicine” is possible and may become practical in the near future. Personalized medicine denotes clinical treatment that takes into account the genetic factors that contribute to disease and the pharmacogenomic factors that influence the response to drug treatment in specific individuals. Intense academic and commercial research is currently directed at discovering these factors. Research is also directed at developing accurate and inexpensive tests for pharmacogenetic factors in individual patients. As noted in Chapter 4, important genetic variations in drug metabolism exist between individuals. Furthermore, genetic diseases alter many functions that are drug targets. The identification of specific genes that control the expression of the molecules involved and the variants (polymorphisms) of those genes has become the subject of intense research over the last 10–20 years. At present, much data are available regarding the variants of the Transporters Immune system genes for some phase I and phase II enzymes and some drug transporters. Examples of these genetic determinants of drug metabolism and transport are the subject of this chapter. PHASE I ENZYMES CYP2D6, CYP2C19, CYP3A4/5, and dihydropyrimidine dehydrogenase are among the drug-metabolizing enzymes most carefully studied (Table 5–1). A. CYP2D6 This enzyme is responsible for the hepatic metabolism of 20% of commonly used drugs. More than 100 polymorphisms of the CYP2D6 gene have been discovered, but only 9 are common. CYP2D6 polymorphisms are especially important in patients receiving codeine because this enzyme converts codeine to its active metabolite, morphine. Several deaths due to respiratory depression have been reported in children who were believed to be ultrarapid metabolizers. B. CYP2C19 CYP2C19 is responsible for the hepatic metabolism of a small number of very important drugs (clopidogrel, propranolol, omeprazole, diazepam, and tricyclic antidepressants). Because reduced metabolism of clopidogrel results in lower 41 42 PART I Basic Principles High-Yield Terms to Learn Pharmacogenetics Synonym for pharmacogenomics; the study of genetic factors that affect drug responses Single nucleotide polymorphism (SNP) A single base pair substitution in the genome that occurs in >1% of a subject population (cf mutation) Mutation A polymorphism that occurs in the genome of <1% of a population; more generally, any change in the genetic material Allele One of 2 or more alternative forms of a gene. Almost all genes are represented by 2 alleles in the genome (because 22 of the 23 human chromosomes are paired). Allele variants are denoted “∗3,” “∗5,” etc Diplotype Representation of the alleles for a specific gene on both chromosomes of a pair. Thus, the gene for the enzyme CYP2D6 with allele ∗3 on one chromosome and ∗5 on the other would be denoted CYP2D6∗3/∗5 Haplotype A series of alleles found in a linked locus on a chromosome Genotype, phenotype Characteristics of the DNA (genotype) and the physiology and biochemistry (phenotype) expressed by the DNA of an individual or population Indels Insertions or deletions of one or more nucleotide bases in genes Synonymous SNP A single nucleotide variation (SNP) that codes for the same amino acid when read out; no change of function (phenotype) results Nonsynonymous (missense) SNP An SNP that results in substitution of a different amino acid when read out; a change in function may result Copy number variation (CNV) Variation in the number of copies of a gene. An increased number of copies commonly results in a gain of function phenotype and vice versa PM, IM, EM, UM Poor metabolizer, intermediate metabolizer, extensive metabolizer, and ultrarapid metabolizer, respectively. These terms describe individuals with varying rates of metabolism of a specific drug or the genomes responsible in such individuals mtDNA, Y-DNA mtDNA is the DNA found in mitochondria; it is normally inherited only through the maternal line. Y-DNA is the DNA found in the Y chromosome and is therefore inherited through the paternal line Genome-Wide Association Study (GWAS) Analysis of the complete genomes of a population of individuals with regard to the frequency of association of specific allelic variations with a specific phenotype concentrations of its active metabolite, reduced function polymorphisms in this enzyme reduce the efficacy of clopidogrel and increase the risk of clotting in patients with coronary artery disease. Conversely, gain of function results in increased risk of bleeding. Poor metabolizers and IMs should receive alternative drugs prasugrel or ticagrelor, not clopidogrel. C. CYP3A4 and CYP3A5 CYP3A4/5 are responsible for the metabolism of over 50% of drugs in common use. Some polymorphisms with important ethnic variability have been described, but relatively few appear to alter pharmacokinetics to a clinically significant degree. D. Dihydropyrimidine Dehydrogenase (DPD) DPD is responsible for the clearance of 5-fluorouracil (5-FU), a first-line prodrug agent for the treatment of colorectal cancer. Capecitabine and tegafur are oral prodrugs converted in the body to 5-FU. In the body, 5-FU is converted to cytotoxic 5-fluorouridine 5′-monophosphate (5-FUMP) and 5-fluoro-2′-deoxyuridine5′-monophosphate (5-FdUMP) (see Chapter 54). Nonfunctional polymorphisms in the DYPD gene result in increased toxicity and require reduced dosage. E. Multiple Enzyme Polymorphisms: CYP2C9 and VCORC1 CYP2C9 and vitamin K epoxide reductase complex subunit 1 (VCORC1) are responsible for the inactivation of S-warfarin. Some mutations of the VCORC1 gene lead to spontaneous bleeding disorders. Reduced function polymorphisms in both genes result in increased warfarin action and enhanced risk of bleeding. Algorithms have been developed to predict the optimal dosage of warfarin, but clinical trials of these algorithms have not shown improved anticoagulant control thus far. PHASE II ENZYMES A. Uridine 5-diphospho-(UDP) glucuronosyltransferase (UGT1A1) UGT1A1 is involved in the hepatic excretion of small molecules into the bile. UGT1A1 contributes to the clearance of SN-38, the bioactive metabolite of irinotecan, a cytotoxic agent used in the CHAPTER 5 Pharmacogenomics 43 TABLE 5–1 Polymorphisms associated with altered drug responses. Functional Element Alleles or SNPs of Major Importance Examples of Drugs Affected Phase I enzyme CYP2C9 ∗2, ∗3: decreased function Warfarin, phenytoin, antidiabetic sulfonylurea metabolism slowed, toxicity increased CYP2C19 ∗17: increased function, Increased or decreased clopidogrel active metabolite ∗2, ∗3: decreased function CYP2D6 ∗1, ∗2: increased function Codeine converted to morphine. Increased function associated with increased toxicity; decreased function associated with decreased analgesia. Increased toxicity of many other drugs ∗3, ∗4, ∗5: decreased function CYP3A4, ∗1, ∗8, ∗11, ∗13, ∗16, ∗17: decreased function 3A5 (SNPs more common in 3A5) *3, *5, *6, *7: decreased function Dihydropyrimidine dehydrogenase (DPD) DPYD ∗2A, ∗13, rs67376798: reduced function Increased toxicity from pyrimidine cancer chemotherapeutic agents, eg, 5-FU UGT1A1 UGT1A1∗28 Increased irinotecan toxicity TPMT ∗2, ∗3 Increased thiopurine (azathioprine, 6-mercaptopurine, 6-thioguanine) toxicity G6PD Mediterranean, Canton, Kaiping: decreased function Greatly increased susceptibility to hemolysis and other toxicities from oxidative stressors but increased resistance to malaria Transporters OATP (P-gp, etc) rs4149056: decreased function Increased risk of simvastatin myopathy. Many other drugs but effects inconclusive Receptors Beta1 adrenoceptor ADRB1 Arg389Gly Increased efficacy of metoprolol Metabolism of some dihydropyridines, cyclosporine, tacrolimus reduced; increased toxicity Phase II enzyme treatment of colorectal cancer. Reduced function polymorphisms result in increased irinotecan-induced bone marrow depression and diarrhea and require a reduction in dosage. B. Thiopurine S-methyltransferase (TPMT) TPMT is important in the inactivation of chemotherapeutic purine derivatives, eg, 6-mercaptopurine (6-MP), azathioprine, a prodrug of 6-MP, and 6-thioguanine (6-TG). Reduced function polymorphisms result in altered therapeutic efficacy as well as altered toxicity. TRANSPORTERS The organic anion transporter (OATP) 1B1 expressed by the SLCO1B1 gene transports drugs and endogenous compounds from the blood into hepatocytes. Substrates include statins and methotrexate. Numerous SNPs are recognized in the SLCO1B1 gene and some are associated with reduced function. Reduced function alleles result in elevated concentrations of some statins, especially simvastatin, and increased risk of skeletal muscle myopathy. The P-glycoprotein is a very promiscuous transporter found in blood-tissue interfaces. Its former name, multidrug resistance transporter-1 (MDR1), reflects its importance in expelling cytotoxic drugs from resistant cancer cells. It is encoded by the ABCB1 gene and over 100 SNPs have been identified in its coding regions. Association studies with drug pharmacokinetics have yielded mixed results. HUMAN LEUKOCYTE ANTIGEN (HLA) POLYMORPHISMS HLA polymorphisms are associated with variations in immunologic responses to drugs, including liver injury, Stevens-Johnson syndrome, and toxic epidermal necrosis. Examples are given in Table 5–1. Polymorphisms have been associated with reactions to abacavir, flucloxacillin, allopurinol, and carbamazepine. SKILL KEEPER: MECHANISM AND TREATMENT OF ACETAMINOPHEN TOXICITY (SEE CHAPTER 4) A 17-year-old boy is admitted to the emergency department and acetaminophen overdose is suspected. What is the mechanism of acetaminophen toxicity and how is it treated? The Skill Keeper Answer appears at the end of the chapter. 44 PART I Basic Principles QUESTIONS 1. A 59-year-old man with acute coronary syndrome is admitted to the hospital for emergency percutaneous insertion of a coronary stent. Which of the following drugs might cause unexpected results based on the patient’s CYP2C19 genotype? (A) Clopidogrel (B) Codeine (C) Prasugrel (D) Ticagrelor (E) Warfarin 2. A 62-year-old woman with advanced colon cancer is treated with intravenous 5-fluorouracil. Within a few days, she develops severe diarrhea, and within a week, she shows severe neutropenia. Which of the following polymorphisms is most likely to be responsible? (A) CYP2D6 ∗1x3 (B) CYP2C19∗2 (C) CYP2C9∗3 (D) DYPD ∗2A (E) UGT1A1∗28 3. A 38-year-old man is being treated for HIV-induced acquired immunodeficiency syndrome (AIDS). When abacavir therapy is begun, he develops a severe skin rash. Which of the following pharmacogenomic diagnoses might explain this skin rash? (A) CYP2D6 ∗3 (PM) (B) CYP3A5 ∗3 (PM) (C) HLA-B ∗57:01 (EM) (D) SLCO1B1∗5 (PM) 4. A college student volunteers to have his genome decoded as part of a population-wide study of polymorphisms. He receives a call from the principal investigator informing him that his genome unexpectedly contains an important single nucleotide polymorphism. Which of the following polymorphisms is associated with risk of hemolysis and increased resistance to malaria? (A) CYP2D6 ∗3 (B) CYP2D19∗2 (C) TPMT ∗2 (D) UGT1A1∗28 (E) G6PD-(A)–Canton 5. A 7-year-old child is brought to the emergency department in coma with cyanosis. Her mother states that the girl was given codeine with acetaminophen because of severe bruising after a fall. Shortly after the first dose, the child became unresponsive and “turned blue.” Which of the following alleles might be responsible for this presentation? (A) CYP2D6 ∗1x3 (B) CYP2C19∗2 (C) CYP2C9∗3 (D) DYPD∗2A (E) UGT1A1∗28 ANSWERS 1. Clopidogrel is a prodrug that must be metabolized to an active platelet-inhibiting metabolite by CYP2C19. Poor metabolizers achieve inadequate platelet inhibition, and EMs and UMs may have excess effect and bleed. Prasugrel and ticagrelor do not require P450 activation and are not subject to this risk. The answer is A. 2. CYP2D6 ∗1x3 is a gain-of-function allele and is associated with increased effect and toxicity of codeine. CYP2C19∗2 is a nonfunctional allele associated with reduced efficacy of clopidogrel. CYP2C9∗3 with a reduced function allele of VCORC1 is associated with reduced warfarin clearance. UGT1A1∗28 is a reduced function allele for uridine 5′-diphospho-(UDP) glucuronosyltransferase and enhances irinotecan toxicity. 5-Fluorouracil is cleared by dihydropyrimidine dehydrogenase (DPD). The DYPD∗2A allele is nonfunctional. The answer is D. 3. Poor metabolizers of the CYP2D6 ∗3 genotype are prone to reduced efficacy of codeine. Poor metabolizers of the CYP3A5∗3 type show reduced tacrolimus clearance. Simvastatin toxicity (myopathy) is enhanced in SLCO1B1 poor metabolizers. Enhanced metabolizers of the HLA-B∗57:01 type are prone to abacavir rashes and flucloxacillin liver damage. The answer is C. 4. CYP2D6 ∗3 is associated with reduced codeine efficacy. CYP2D19∗2 results in reduced clopidogrel conversion to its active metabolite. TPMT ∗2 is associated with decreased clearance of 6-mercaptopurine and increased toxicity. UGT1A1∗28 results in decreased clearance and increased toxicity of SN-38, the active metabolite of irinotecan. G6PD-(A)–Canton is a reduced function allele of the glucose 6-phosphate dehydrogenase gene that decreases intracellular stores of glutathione, increasing the risk of hemolysis but reducing susceptibility to malaria. The answer is E. 5. As noted in answer 4, SNPs in CYP2D6 may increase or decrease the efficacy and toxicity of codeine because the CYP2D6 enzyme is responsible for conversion of codeine to its active metabolite, morphine. CYP2D6 ∗1xN and ∗2xN are gain-of-function polymorphisms that result in more efficient conversion to morphine and increased risk of opioid-induced respiratory depression. The answer is A. SKILL KEEPER ANSWER: ACETAMINOPHEN TOXICITY AND TREATMENT In normal dosages, and in individuals with normal liver function, acetaminophen is converted to harmless glucuronide and sulfate conjugates and is excreted. Overdoses or high therapeutic doses in individuals with impaired liver function overwhelm the phase II systems and result in intracellular accumulation of a reactive intermediate that can combine with essential cellular proteins and cause hepatic necrosis. Treatment attempts to maximize free radical scavenger activity with N-acetylcysteine. CHAPTER 5 Pharmacogenomics 45 CHECKLIST When you complete this chapter, you should be able to: ❑ Name 3 gene polymorphisms that increase or decrease drug efficacy or toxicity. ❑ Name 3 drugs that may require dosage adjustments in specific genetic populations. ❑ Name 1 drug that is more toxic due to a polymorphism. ❑ Name 1 drug that is less effective due to a loss of function polymorphism. CHAPTER 5 Summary Table Major Concept Description Genetic gain of function Increased function of the enzyme or transporter target due to multiple copies of the gene or gene polymorphism resulting in altered structure of the resulting target molecule Genetic loss of function Decreased function of the enzyme or transporter target due to failure of expression of the gene or altered structure of the resulting target molecule Synonymous and nonsynonymous SNPs If an SNP results in no change in the amino acid specified by a given DNA base triad, it is referred to as a synonymous SNP and no change in phenotype is expected. If the SNP results in coding of a different amino acid, it is nonsynonymous and a change in function may or may not result This page intentionally left blank PART II AUTONOMIC DRUGS C Introduction to Autonomic Pharmacology The autonomic nervous system (ANS) is the major involuntary, automatic portion of the nervous system and contrasts in several ways with the somatic (voluntary) nervous system. The anatomy, neurotransmitter chemistry, receptor characteristics, H A P T E R 6 and functional integration of the ANS are discussed in this chapter. Major autonomic drug groups are discussed in Chapters 7 through 10. Drugs in many other groups have significant autonomic effects, many of which are undesirable. Autonomic introduction ANS anatomy Transmitter types: acetylcholine, norepinephrine, peptides, purines Transmitter synthesis, storage, release, termination ANATOMIC ASPECTS OF THE ANS The motor (efferent) portion of the ANS is the major neural pathway for information transmission from the central nervous system (CNS) to the involuntary effector tissues (smooth muscle, cardiac muscle, and exocrine glands; Figure 6–1). Its 2 major subdivisions are the parasympathetic ANS (PANS) and the sympathetic ANS (SANS). The enteric nervous system (ENS) is a semiautonomous part of the ANS located in the gastrointestinal tract, with specific functions for the control of this organ system. The neuron cell Receptor types M, N α, β, D ANS effects, regulation NANC bodies of the ENS are located in the myenteric plexus (plexus of Auerbach) and the submucous plexus (plexus of Meissner); these neurons send motor and sensory axons to the motor and secretory cells; they also provide sensory input to the parasympathetic and sympathetic nervous systems and receive motor output from them. There are many sensory (afferent) fibers in autonomic nerves. These are of considerable importance for the physiologic control of the involuntary organs but are directly influenced by only a few drugs. In contrast, many drugs have important effects on the motor functions of these organs. 47 48 PART II Autonomic Drugs High-Yield Terms to Learn Adrenergic A nerve ending that releases norepinephrine as the primary transmitter; also, a synapse in which norepinephrine is the primary transmitter Adrenoceptor, adrenergic receptor A receptor that binds, and is activated by, one of the catecholamine transmitters or hormones (norepinephrine, epinephrine, dopamine) and related drugs The neuronal homeostatic mechanism that maintains a constant mean arterial blood pressure; the sensory limb originates in the baroreceptors of the carotid sinus and aortic arch; efferent pathways run in parasympathetic and sympathetic nerves A nerve ending that releases acetylcholine; also, a synapse in which the primary transmitter is acetylcholine A receptor that binds, and is activated by, acetylcholine and related drugs A nerve ending that releases dopamine as the primary transmitter; also a synapse in which dopamine is the primary transmitter A compensatory mechanism for maintaining a body function at a predetermined level, for example, the baroreceptor reflex for blood pressure control Nerve fibers associated with autonomic nerves that release any transmitter other than norepinephrine or acetylcholine The part of the autonomic nervous system that originates in the cranial nerves and sacral part of the spinal cord; the craniosacral autonomic system A receptor located on the distal side of a synapse, for example, on a postganglionic neuron or an autonomic effector cell A receptor located on the nerve ending from which the transmitter is released into the synapse; modulates the release of transmitter Baroreceptor reflex Cholinergic Cholinoceptor, cholinergic receptor Dopaminergic Homeostatic reflex Nonadrenergic, noncholinergic (NANC) system Parasympathetic Postsynaptic receptor Presynaptic receptor Sympathetic The part of the autonomic nervous system that originates in the thoracic and lumbar parts of the spinal cord; the thoracolumbar autonomic system The parasympathetic preganglionic motor fibers originate in cranial nerve nuclei III, VII, IX, and X and in sacral segments (usually S2–S4) of the spinal cord. The sympathetic preganglionic fibers originate in the thoracic (T1–T12) and lumbar (L1–L5) segments of the cord. Most of the sympathetic ganglia are located in 2 paravertebral chains that lie along the sides of the spinal column in the thorax and abdomen. A few (the prevertebral ganglia) are located on the anterior aspect of the abdominal aorta. Most of the parasympathetic ganglia are located in the organs innervated and more distant from the spinal cord. Because of the locations of the ganglia, the preganglionic sympathetic fibers are short and the postganglionic fibers are long. The opposite is true for the parasympathetic system: preganglionic fibers are longer and postganglionic fibers are short. Some receptors that respond to autonomic transmitters and drugs receive no innervation. These include muscarinic receptors on the endothelium of blood vessels, some presynaptic receptors on nerve endings, and, in some species, the adrenoceptors on apocrine sweat glands and α2 and β adrenoceptors in blood vessels. NEUROTRANSMITTER ASPECTS OF THE ANS The synthesis, storage, release, receptor interactions, and termination of action of the neurotransmitters all contribute to the action of autonomic drugs (Figure 6–2). A. Cholinergic Transmission Acetylcholine (ACh) is the primary transmitter in all autonomic ganglia and at the synapses between parasympathetic postganglionic neurons and their effector cells. It is the transmitter at postganglionic sympathetic neurons to the thermoregulatory sweat glands. It is also the primary transmitter at the somatic (voluntary) skeletal muscle neuromuscular junction (Figure 6–1). 1. Synthesis and storage—Acetylcholine is synthesized in the nerve terminal by the enzyme choline acetyltransferase (ChAT) from acetyl-CoA (produced in mitochondria) and choline (transported across the cell membrane) (Figure 6–2). The rate-limiting step is probably the transport of choline into the nerve terminal. This transport can be inhibited by the research drug hemicholinium. Acetylcholine is actively transported into its vesicles for storage by the vesicle-associated transporter, VAT. This process can be inhibited by another research drug, vesamicol. 2. Release of acetylcholine—Release of transmitter stores from vesicles in the nerve ending requires the entry of calcium through calcium channels and triggering of an interaction between SNARE (soluble N-ethylmaleimide-sensitive-factor attachment protein receptor) proteins. SNARE proteins include v-SNARES associated with the vesicles (VAMPs, vesicle-associated membrane proteins: synaptobrevin, synaptotagmin) and t-SNARE proteins associated with the nerve terminal membrane (SNAPs, synaptosome-associated proteins: SNAP25, syntaxin, and others). This interaction results in docking of the vesicle to the terminal membrane and, with influx CHAPTER 6 Introduction to Autonomic Pharmacology N ACh 49 Parasympathetic Cardiac and smooth muscle, gland cells, nerve terminals ACh M Medulla Sympathetic ganglia ACh ACh Spinal cord ACh ACh M N N NE α, β Sympathetic Sweat glands (eccrine) Sympathetic Cardiac and smooth muscle, gland cells, nerve terminals N D Sympathetic Renal vascular smooth muscle D1 N Adrenal medulla Epi, NE ACh N Somatic Skeletal muscle Voluntary motor nerve FIGURE 6–1 Schematic diagram comparing some features of the parasympathetic and sympathetic divisions of the autonomic nervous system with the somatic motor system. Parasympathetic ganglia are not shown as discrete structures because most of them are diffusely distributed in the walls of the organs innervated. Only 3 of the more than 20 sympathetic ganglia are shown. α and β, alpha and beta adrenoceptors; ACh, acetylcholine; D, dopamine; D1, dopamine1 receptors; Epi, epinephrine; M, muscarinic; N, nicotinic; NE, norepinephrine. (Modified and reproduced, with permission, from Katzung BG, editor: Basic & Clinical Pharmacology, 12th ed. McGraw-Hill, 2012: Fig. 6–1.) of calcium, fusion of the membranes of the vesicles with the nerveending membranes, the opening of a pore to the extracellular space, and the release of the stored transmitter. The several types of botulinum toxins are able to enter cholinergic nerve terminals and enzymatically alter synaptobrevin or one of the other docking or fusion proteins to prevent the release process. 3. Termination of action of acetylcholine—The action of acetylcholine in the synapse is normally terminated by metabolism to acetate and choline by the enzyme acetylcholinesterase in the synaptic cleft. The products are not excreted but are recycled in the body. Inhibition of acetylcholinesterase is an important therapeutic (and potentially toxic) effect of several drugs. 4. Drug effects on synthesis, storage, release, and termination of action of acetylcholine—Drugs that block the synthesis of acetylcholine (eg, hemicholinium), its storage (eg, vesamicol), or its release (eg, botulinum toxin) are not very useful for systemic therapy because their effects are not sufficiently selective (ie, PANS and SANS ganglia and somatic neuromuscular junctions all may be blocked). However, because botulinum toxin is a very large molecule and diffuses very slowly, it can be used by injection for relatively selective local effects. SKILL KEEPER: DRUG PERMEATION (SEE CHAPTER 1) Botulinum toxin is a very large protein molecule and does not diffuse readily when injected into tissue. In spite of this property, it is able to enter cholinergic nerve endings from the extracellular space and inhibit the release of acetylcholine. How might it cross the lipid membrane barrier? The Skill Keeper Answer appears at the end of the chapter. 50 PART II Autonomic Drugs CHOLINERGIC NORADRENERGIC Hemicholinium − Tyrosine Tyrosine Choline TH Acetyl-CoA + Choline Metyrosine − DOPA ChAT ACh Dopamine − Vesamicol − NE ACh Ca2+ VAMPs Ca2+ NE + + Botulinum ACh SNAPs ACh Reserpine − AChE Cholinoceptor Guanethidine Uptake 1 (NET) − Cocaine, TCA Choline + Acetate Postsynaptic membrane NE NE − Diffusion, metabolism Adrenoceptor FIGURE 6–2 Characteristics of transmitter synthesis, storage, release, and termination of action at cholinergic and noradrenergic nerve terminals are shown from the top downward. Circles represent transporters; ACh, acetylcholine; AChE, acetylcholinesterase; ChAT, choline acetyltransferase; DOPA, dihydroxyphenylalanine; NE, norepinephrine; NET, norepinephrine transporter; TCA, tricyclic antidepressant; TH, tyrosine hydroxylase. B. Adrenergic Transmission Norepinephrine (NE) is the primary transmitter at the sympathetic postganglionic neuron-effector cell synapses in most tissues. Important exceptions include sympathetic fibers to thermoregulatory (eccrine) sweat glands and probably vasodilator sympathetic fibers in skeletal muscle, which release acetylcholine. Dopamine may be a vasodilator transmitter in renal blood vessels, but norepinephrine is a vasoconstrictor of these vessels. 1. Synthesis and storage—The synthesis of dopamine and norepinephrine requires several steps (Figure 6–2). After transport across the cell membrane, tyrosine is hydroxylated by tyrosine hydroxylase (the rate-limiting step) to DOPA (dihydroxyphenylalanine), decarboxylated to dopamine, and (inside the vesicle) hydroxylated to norepinephrine. Tyrosine hydroxylase can be inhibited by metyrosine. Norepinephrine and dopamine are transported into vesicles by the vesicular monoamine transporter (VMAT) and are stored there. Monoamine oxidase (MAO) is present on mitochondria in the adrenergic nerve ending and inactivates a portion of the dopamine and norepinephrine in the cytoplasm. Therefore, MAO inhibitors may increase the stores of these transmitters and other amines in the nerve endings (Chapter 30). VMAT can be inhibited by reserpine, resulting in depletion of transmitter stores. 2. Release and termination of action—Dopamine and norepinephrine are released from their nerve endings by the same calcium-dependent mechanism responsible for acetylcholine release (see prior discussion). In contrast to cholinergic neurons, noradrenergic and dopaminergic neurons lack receptors for botulinum and do not transport this toxin into the nerve terminal. Termination of action is also quite different from the cholinergic system. Metabolism is not responsible for termination of action of the catecholamine transmitters, norepinephrine and dopamine. Rather, diffusion and reuptake (especially uptake-1, Figure 6–2, by the norepinephrine transporter, NET, or the dopamine transporter, DAT) reduce their concentration in the synaptic cleft and stop their action. Outside the cleft, these transmitters can be metabolized—by MAO and catechol-O-methyltransferase (COMT)—and the products of these enzymatic reactions are excreted. Determination of the 24-h excretion of metanephrine, normetanephrine, 3-methoxy-4-hydroxymandelic acid (VMA), and other metabolites provides a measure of the total body production of catecholamines, a determination useful in diagnosing conditions such as pheochromocytoma. Inhibition of MAO increases stores of catecholamines in nerve endings and has both therapeutic and toxic potential. Inhibition of COMT in the brain is useful in Parkinson’s disease (Chapter 28). CHAPTER 6 Introduction to Autonomic Pharmacology 3. Drug effects on adrenergic transmission—Drugs that block norepinephrine synthesis (eg, metyrosine) or catecholamine storage (eg, reserpine) or release (eg, guanethidine) were used in treatment of several diseases (eg, hypertension) because they block sympathetic but not parasympathetic functions. Other drugs promote catecholamine release (eg, the amphetamines) and predictably cause sympathomimetic effects. C. Cotransmitters Many (probably all) autonomic nerves have transmitter vesicles that contain other transmitter molecules in addition to the primary agents (acetylcholine or norepinephrine) previously described. These cotransmitters may be localized in the same vesicles as the primary transmitter or in a separate population of vesicles. Substances recognized to date as cotransmitters include ATP (adenosine triphosphate), enkephalins, vasoactive intestinal peptide, neuropeptide Y, substance P, neurotensin, somatostatin, and others. Their main role in autonomic function appears to involve modulation of synaptic transmission. The same substances function as primary transmitters in other synapses. 51 heart, vascular endothelium, smooth muscle, presynaptic nerve terminals, and exocrine glands). Evidence (including their genes) has been found for 5 subtypes, of which 3 appear to be important in peripheral autonomic transmission. All 5 are G-proteincoupled receptors (see Chapter 2). 2. Nicotinic receptors—These receptors are located on Na+-K+ ion channels and respond to acetylcholine and nicotine, another acetylcholine mimic (but not to muscarine) by opening the channel. The 2 major nicotinic subtypes are located in ganglia and in skeletal muscle end plates. The nicotinic receptors are the primary receptors for transmission at these sites. B. Adrenoceptors Also referred to as adrenergic receptors, adrenoceptors are divided into several subtypes (Table 6–2). 1. Alpha receptors—These are located on vascular smooth muscle, presynaptic nerve terminals, blood platelets, fat cells (lipocytes), and neurons in the brain. Alpha receptors are further divided into 2 major types, α1 and α2. These 2 subtypes constitute different families and use different G-coupling proteins. ANS RECEPTORS The major receptor systems in the ANS include cholinoceptors, adrenoceptors, and dopamine receptors, which have been studied in detail. The numerous receptors for cotransmitter substances have not been as fully characterized. A. Cholinoceptors Also referred to as cholinergic receptors, these molecules respond to acetylcholine and its analogs. Cholinoceptors are subdivided as follows (Table 6–1): 1. Muscarinic receptors—As their name suggests, these receptors respond to muscarine (an alkaloid) as well as to acetylcholine. The effects of activation of these receptors resemble those of postganglionic parasympathetic nerve stimulation. Muscarinic receptors are located primarily on autonomic effector cells (including 2. Beta receptors—These receptors are located on most types of smooth muscle, cardiac muscle, some presynaptic nerve terminals, and lipocytes. Beta receptors are divided into 3 major subtypes, β1, β2, and β3. These subtypes are rather similar and use the same G-coupling protein. C. Dopamine Receptors Dopamine (D, DA) receptors are a subclass of adrenoceptors but with rather different distribution and function. Dopamine receptors are especially important in the renal and splanchnic vessels and in the brain. Although at least 5 subtypes exist, the D1 subtype appears to be the most important dopamine receptor on peripheral effector cells. D2 receptors are found on presynaptic nerve terminals. D1, D2, and other types of dopamine receptors also occur in the CNS. TABLE 6–1 Characteristics of the most important cholinoceptors in the peripheral nervous system. Receptor Location Mechanism Major Functions M1 Nerve endings Gq-coupled ↑ IP3, DAG cascade M2 Heart, some nerve endings Gi-coupled ↓ cAMP, activates K+ channels M3 Effector cells: smooth muscle, glands, endothelium Gq-coupled ↑ IP3, DAG cascade NN ANS ganglia Na+-K+ ion channel Depolarizes, evokes action potential NM Neuromuscular end plate Na+-K+ ion channel Depolarizes, evokes action potential IP3, inositol trisphosphate; DAG, diacylglycerol; cAMP, cyclic adenosine monophosphate. 52 PART II Autonomic Drugs TABLE 6–2 Characteristics of the most important adrenoceptors in the ANS. Receptor Location G Protein Second Messenger Major Functions Alpha1 (α1) Effector tissues: smooth muscle, glands Gq ↑ IP3, DAG ↑ Ca2+, causes contraction, secretion Alpha2 (α2) Nerve endings, some smooth muscle Gi ↓ cAMP ↓ Transmitter release (nerves), causes contraction (muscle) Beta1 (β1) Cardiac muscle, juxtaglomerular apparatus Gs ↑ cAMP ↑ Heart rate, ↑ force; ↑ renin release Beta2 (β2) Smooth muscle, liver, heart Gs ↑ cAMP Relax smooth muscle; ↑ glycogenolysis; ↑ heart rate, force Beta3 (β3) Adipose cells Gs ↑ cAMP ↑ Lipolysis Dopamine1 (D1) Smooth muscle Gs ↑ cAMP Relax renal vascular smooth muscle ANS, autonomic nervous system, IP3, inositol trisphosphate; DAG, diacylglycerol; cAMP, cyclic adenosine monophosphate. EFFECTS OF ACTIVATING AUTONOMIC NERVES Each division of the ANS has specific effects on organ systems. These effects, summarized in Table 6–3, should be memorized. Dually innervated organs such as the iris of the eye and the sinoatrial node of the heart receive both sympathetic and parasympathetic innervation. The pupil has a natural, intrinsic diameter to which it returns when both divisions of the ANS are blocked. Pharmacologic ganglion blockade, therefore, causes it to move to its intrinsic size. Similarly, the cardiac sinus node pacemaker has an intrinsic rate (about 100–110/min) in the absence of both ANS inputs. How will these variables change (increase or decrease) if the ganglia are blocked? The answer is predictable if one knows which system is dominant. For example, both the pupil and, at rest, the sinoatrial node are dominated by the parasympathetic system. Thus, blockade of both systems, with removal of the dominant PANS and nondominant SANS effects, result in mydriasis and tachycardia. effector” fibers because, when activated by a sensory input, they are capable of releasing transmitter peptides from the sensory ending itself, from local axon branches, and from collaterals that terminate in the autonomic ganglia. In addition to their neurotransmitter roles, these peptides are potent agonists in many autonomic effector tissues, especially smooth muscle (see Chapter 17). SITES OF AUTONOMIC DRUG ACTION Because of the number of steps in the transmission of autonomic commands from the CNS to the effector cells, there are many sites at which autonomic drugs may act. These sites include the CNS centers; the ganglia; the postganglionic nerve terminals; the effector cell receptors; and the mechanisms responsible for transmitter synthesis, storage, release, and termination of action. The most selective effect is achieved by drugs acting at receptors that mediate very selective actions (Table 6–4). Many natural and synthetic toxins have significant effects on autonomic and somatic nerve function. NONADRENERGIC, NONCHOLINERGIC (NANC) TRANSMISSION INTEGRATION OF AUTONOMIC FUNCTION Some nerve fibers in autonomic effector tissues do not show the histochemical characteristics of either cholinergic or adrenergic fibers. Some of these are motor fibers that cause the release of ATP and other purines related to it. Purine-evoked responses have been identified in the bronchi, gastrointestinal tract, and urinary tract. Other motor fibers are peptidergic, that is, they release peptides as the primary transmitters (see list in earlier Cotransmitters section). Some fibers may release nitric oxide, a highly permeant gas that is not stored but is synthesized on demand (see Chapter 19). Other nonadrenergic, noncholinergic fibers have the anatomic characteristics of sensory fibers and contain peptides, such as substance P, that are stored in and released from the fiber terminals. These fibers have been termed “sensory-efferent” or “sensory-local Functional integration in the ANS is provided mainly through the mechanism of negative feedback and is extremely important in determining the overall response to endogenous and exogenous ANS transmitters and their analogs. This process uses modulatory pre- and postsynaptic receptors at the local level and homeostatic reflexes at the system level. A. Local Integration Local feedback control has been found at the level of the nerve endings in all systems investigated. The best documented of these is the negative feedback of norepinephrine upon its own release from adrenergic nerve terminals. This effect is mediated by α2 receptors located on the presynaptic nerve membrane (Figure 6–3). CHAPTER 6 Introduction to Autonomic Pharmacology 53 TABLE 6–3 Direct effects of autonomic nerve activity on some organ systems. Effect of Sympathetic Parasympathetic Organ Actiona Receptorb Actiona Receptorb Eye Iris Radial muscle Circular muscle Ciliary muscle Contracts ... [Relaxes] α1 ... β ... Contracts Contracts ... M3 M3 Heart Sinoatrial node Ectopic pacemakers Contractility Accelerates Accelerates Increases β1, β2 β1, β2 β1, β2 Decelerates ... Decreases (atria) M2 ... [M2] Contracts Relaxes Contracts [Relaxes] α β2 α [Mc] ... ... ... ... ... ... ... ... Bronchiolar smooth muscle Relaxes β2 Contracts M3 Gastrointestinal tract Smooth muscle Walls Sphincters Secretion Myenteric plexus Relaxes Contracts Inhibits ... α2,d β2 α1 α2 ... Contracts Relaxes Increases Activates M3 M3 M3 M1 Relaxes Contracts Relaxes Contracts Ejaculation β2 α1 β2 α α Contracts Relaxes ... Contracts Erection M3 M3 ... M3 M Contracts α Increases Increases M α ... ... ... ... ... ... ... ... Metabolic functions Liver Liver Fat cells Kidney Gluconeogenesis Glycogenolysis Lipolysis Renin release β2, α β2, α β3 β1 ... ... ... ... ... ... ... ... Autonomic nerve endings Sympathetic Parasympathetic ... Decreases ACh release ... α Decreases NE release ... Me ... Blood vessels Skin, splanchnic vessels Skeletal muscle vessels Genitourinary smooth muscle Bladder wall Sphincter Uterus, pregnant Penis, seminal vesicles Skin Pilomotor smooth muscle Sweat glands Thermoregulatory Apocrine (stress) a Less important actions are shown in brackets. b Specific receptor type: α, alpha; β, beta; M, muscarinic. c Vascular smooth muscle in skeletal muscle has sympathetic cholinergic dilator fibers. d Probably through presynaptic inhibition of parasympathetic activity. e Probably M1, but M2 may participate in some locations. ACh, acetylcholine; NE, norepinephrine. Modified and reproduced, with permission, from Katzung BG, editor: Basic & Clinical Pharmacology, 12th ed. McGraw-Hill, 2012. Presynaptic receptors that bind the primary transmitter substance and thereby regulate its release are called autoreceptors. Transmitter release is also modulated by other presynaptic receptors (heteroreceptors); in the case of adrenergic nerve terminals, receptors for acetylcholine, histamine, serotonin, prostaglandins, peptides, and other substances have been found. Presynaptic regulation by a variety of endogenous chemicals probably occurs in all nerve fibers. 54 PART II Autonomic Drugs TABLE 6–4 Steps in autonomic transmission: effects of drugs. Process Drug Example Site Action Action potential propagation Local anesthetics, tetrodotoxin,a saxitoxinb Nerve axons Block sodium channels; block conduction Transmitter synthesis Hemicholinium Alpha-methyltyrosine (metyrosine) Cholinergic nerve terminals: membrane Adrenergic nerve terminals and adrenal medulla: cytoplasm Blocks uptake of choline and slows synthesis of acetylcholine Slows synthesis of norepinephrine Transmitter storage Vesamicol Reserpine Cholinergic terminals: vesicles Adrenergic terminals: vesicles Prevents storage, depletes Prevents storage, depletes Transmitter release Manyc Nerve terminal membrane receptors Modulates release ω-Conotoxin GVIAd Nerve terminal calcium channels Reduces release Botulinum toxin Alpha-latrotoxine Tyramine, amphetamine Cholinergic vesicles Cholinergic and adrenergic vesicles Adrenergic nerve terminals Prevents release Causes explosive release Promotes release Cocaine, tricyclic antidepressants 6-Hydroxydopamine Adrenergic nerve terminals Adrenergic nerve terminals Inhibit uptake; increase transmitter effect on postsynaptic receptors Destroys the terminals Norepinephrine Receptors at adrenergic junctions Binds α receptors; causes activation Phentolamine Receptors at adrenergic junctions Binds α receptors; prevents activation Isoproterenol Receptors at adrenergic junctions Binds β receptors; activates adenylyl cyclase Propranolol Receptors at adrenergic junctions Nicotine Receptors at nicotinic cholinergic junctions (autonomic ganglia, neuromuscular end plates) Ganglionic nicotinic receptors Neuromuscular end plates Parasympathetic effector cells (smooth muscle, glands) Parasympathetic effector cells Binds β receptors; prevents activation Binds nicotinic receptors; opens ion channel in post-synaptic membrane Transmitter uptake after release Receptor activation or blockade Hexamethonium Tubocurarine Bethanechol Atropine Enzymatic inactivation of transmitter Prevents activation of NN receptors Prevents activation of NM receptors Binds and activates muscarinic receptors Binds muscarinic receptors; prevents activation Neostigmine Cholinergic synapses (acetylcholinesterase) Inhibits enzyme; prolongs and intensifies transmitter action Tranylcypromine Adrenergic nerve terminals (monoamine oxidase) Inhibits enzyme; increases stored transmitter pool a Toxin of puffer fish, California newt. b Toxin of Gonyaulax (red tide organism). c Norepinephrine, dopamine, acetylcholine, angiotensin II, various prostaglandins, etc. d Toxin of marine snails of the genus Conus. e Black widow spider venom. Modified and reproduced, with permission, from Katzung BG, editor: Basic & Clinical Pharmacology, 12th ed. McGraw-Hill, 2012. Postsynaptic modulatory receptors, including M1 and M2 muscarinic receptors and at least 1 type of peptidergic receptor, have been found in ganglionic synapses, where nicotinic transmission is primary. These receptors may facilitate or inhibit transmission by evoking slow excitatory or inhibitory postsynaptic potentials (EPSPs or IPSPs). B. Systemic Reflexes System reflexes regulate blood pressure, gastrointestinal motility, bladder tone, airway smooth muscle, and other processes. The control of blood pressure—by the baroreceptor neural reflex and the renin-angiotensin-aldosterone hormonal response—is especially important (Figure 6–4). These homeostatic mechanisms have evolved to maintain mean arterial blood pressure at a level determined by the vasomotor center and renal sensors. Any deviation from this blood pressure “set point” causes a change in ANS activity and renin-angiotensin-aldosterone levels. For example, a decrease in blood pressure caused by hemorrhage results in increased SANS discharge and renin release. CHAPTER 6 Introduction to Autonomic Pharmacology 55 Noradrenergic nerve terminal Release-modulating receptors M NE − AT1 + α2 − NE Uptake 1 (NET) FIGURE 6–3 Local control of autonomic nervous system function via modulation of transmitter release. In the example shown, release of norepinephrine (NE) from a sympathetic nerve ending is modulated by norepinephrine itself, acting on presynaptic α2 autoreceptors, and by acetylcholine and angiotensin II, acting on heteroreceptors. Many other modulators (see text) influence the release process. AT1, angiotensin II receptor; M, muscarinic receptor; NET, norepinephrine transporter. Negative feedback NE β Adrenoceptor Cardiac muscle cell (sinoatrial node) Autonomic feedback loop VASOMOTOR CENTER Parasympathetic autonomic nervous system Baroreceptors + Peripheral vascular resistance Mean arterial pressure Hormonal feedback loop Sympathetic autonomic nervous system Cardiac output – + + Heart rate + Contractile force Stroke volume Venous tone Venous return Blood volume Aldosterone Renal blood flow/pressure Renin Angiotensin FIGURE 6–4 Autonomic and hormonal control of cardiovascular function. Note that 2 feedback loops are present: the autonomic nervous system loop and the hormonal loop. Each major loop has several components. In the neuronal loop, sensory input to the vasomotor center is via afferent fibers in the ninth and tenth cranial (PANS) nerves. On the efferent side, the sympathetic nervous system directly influences 4 major variables: peripheral vascular resistance, heart rate, contractile force, and venous tone. The parasympathetic nervous system directly influences heart rate. In addition, angiotensin II directly increases peripheral vascular resistance (not shown), and sympathetic nervous system discharge directly increases renin secretion (not shown). Because these control mechanisms have evolved to maintain normal blood pressure, the net feedback effect of each loop is negative; feedback tends to compensate for the change in arterial blood pressure that evoked the response. Thus, decreased blood pressure due to blood loss would be compensated by increased sympathetic outflow and renin release. Conversely, elevated pressure due to the administration of a vasoconstrictor drug would cause reduced sympathetic outflow, decreased renin release, and increased parasympathetic (vagal) outflow. (Modified and reproduced, with permission, from Katzung BG, editor: Basic & Clinical Pharmacology, 12th ed. McGraw-Hill, 2012: Fig. 6–7.) 56 PART II Autonomic Drugs Cornea Canal of Schlemm Anterior chamber Trabecular meshwork Dilator (α) Sphincter (M) Sclera Iris Lens Ciliary epithelium (β) Ciliary muscle (M) FIGURE 6–5 Some pharmacologic targets in the eye. The diagram illustrates clinically important structures and their receptors. The heavy arrow (blue) illustrates the flow of aqueous humor from its secretion by the ciliary epithelium to its drainage through the canal of Schlemm. M, muscarinic receptor; α, alpha receptor; β, beta receptor. (Modified and reproduced, with permission, from Katzung BG, editor: Basic & Clinical Pharmacology, 12th ed. McGraw-Hill, 2012: Fig. 6–9.) Consequently, peripheral vascular resistance, venous tone, heart rate, and cardiac force are increased by norepinephrine released from sympathetic nerves. This ANS response can be blocked with ganglion-blocking drugs such as hexamethonium. Blood volume is replenished by retention of salt and water in the kidney under the influence of increased levels of aldosterone. These compensatory responses may be large enough to overcome some of the actions of drugs. For example, the chronic treatment of hypertension with a vasodilator such as hydralazine will be unsuccessful if compensatory tachycardia (via the baroreceptor reflex) and salt and water retention (via the renin system response) are not prevented through the use of additional drugs. C. Complex Organ Control: The Eye The eye contains multiple tissues, several of them under autonomic control (Figure 6–5). The pupil, discussed previously, is under reciprocal control by the SANS (via α receptors on the pupillary dilator muscle) and the PANS (via muscarinic receptors on the pupillary constrictor). The ciliary muscle, which controls accommodation, is under primary control of muscarinic receptors innervated by the PANS, with insignificant contributions from the SANS. The ciliary epithelium, on the other hand, has important β receptors that have a permissive effect on aqueous humor secretion. Each of these receptors is an important target of drugs that are discussed in the following chapters. QUESTIONS 1. A 3-year-old child has been admitted to the emergency department having swallowed the contents of 2 bottles of a nasal decongestant. The active ingredient of the medication is a potent, selective α-adrenoceptor agonist drug. Which of the following is a sign of α-receptor activation that may occur in this patient? (A) Bronchodilation (B) Cardiac acceleration (tachycardia) (C) Pupillary dilation (mydriasis) (D) Renin release from the kidneys (E) Vasodilation of the blood vessels of the skin 2. Mr Green is a 60-year-old man with poorly controlled hypertension of 170/110 mm Hg. He is to receive minoxidil. Minoxidil is a powerful arteriolar vasodilator that does not act on autonomic receptors. Which of the following effects will be observed if no other drugs are used? (A) Tachycardia and increased cardiac contractility (B) Tachycardia and decreased cardiac output (C) Decreased mean arterial pressure and decreased cardiac contractility (D) Decreased mean arterial pressure and increased salt and water excretion by the kidney (E) No change in mean arterial pressure and decreased cardiac contractility CHAPTER 6 Introduction to Autonomic Pharmacology 3. Full activation of the parasympathetic nervous system is likely to produce which of the following effects? (A) Bronchodilation (B) Decreased intestinal motility (C) Increased thermoregulatory sweating (D) Increased pupillary constrictor tone (miosis) (E) Increased heart rate (tachycardia) Questions 4–5. For these questions, use the accompanying diagram. Assume that the diagram can represent either the sympathetic or the parasympathetic system. Questions 9–10. Assume that the diagram below represents a sympathetic postganglionic nerve ending. Terminal Axon 2 4 1 Enzyme 7 4 1 3 2 Spinal cord 57 x 3 4 4 y z 1 5 6 Effector cell 4. Assuming the structure is part of the thoracolumbar system, norepinephrine acts at which of the following sites in the diagram? (A) Sites 1 and 2 (B) Sites 3 and 4 (C) Sites 5 and 6 5. If the effector cell in the diagram is a pupillary constrictor smooth muscle cell, which of the following receptor types is denoted by structure 6? (A) Alpha1 adrenoceptor (B) Beta2 adrenoceptor (C) M3 cholinoceptor (D) Ng cholinoceptor 6. Nicotinic receptor sites do not include which one of the following sites? (A) Bronchial smooth muscle (B) Adrenal medullary cells (C) Parasympathetic ganglia (D) Skeletal muscle end plates (E) Sympathetic ganglia 7. Several children at a summer camp were hospitalized with symptoms thought to be due to ingestion of food containing botulinum toxin. Which one of the following signs or symptoms is consistent with the diagnosis of botulinum poisoning? (A) Bronchospasm (B) Cycloplegia (C) Diarrhea (D) Skeletal muscle spasms (E) Hyperventilation 8. Which one of the following is the primary neurotransmitter agent normally released in the sinoatrial node of the heart in response to a blood pressure increase? (A) Acetylcholine (B) Dopamine (C) Epinephrine (D) Glutamate (E) Norepinephrine 9. Which of the following blocks the carrier represented by “z” in the diagram? (A) Amphetamine (B) Botulinum toxin (C) Cocaine (D) Hemicholinium (E) Reserpine 10. Which of the following inhibits the carrier denoted “y” in the diagram? (A) Cocaine (B) Dopamine (C) Hemicholinium (D) Reserpine (E) Vesamicol ANSWERS 1. Mydriasis can be caused by contraction of the radial fibers of the iris; these smooth muscle cells have α receptors. All the other listed responses are mediated by β adrenoceptors (Table 6–4). The answer is C. 2. Because of the compensatory responses, a drug that directly decreases blood pressure through a decrease in peripheral vascular resistance will cause a reflex increase in sympathetic outflow, an increase in renin release, and a decrease in parasympathetic outflow. As a result, heart rate and cardiac force will increase. In addition, salt and water retention will occur. The answer is A. 3. Parasympathetic discharge causes bronchial and intestinal smooth muscle contraction and bradycardia. Thermoregulatory (eccrine) sweat glands are innervated by sympathetic cholinergic fibers, not parasympathetic. The answer is D. 4. Norepinephrine acts at presynaptic α2 regulatory receptors (site 5) and postsynaptic α1 adrenoceptors (site 6). It may be metabolized by enzymes outside the synapse or transported back into the nerve terminal. The answer is C. 5. The nerves innervating the pupillary constrictor muscle are postganglionic parasympathetic cholinergic nerves. The pupillary dilator muscle contains α1 adrenoceptors. The answer is C. 58 PART II Autonomic Drugs 6. Both types of ganglia and the skeletal muscle neuromuscular junction have nicotinic cholinoceptors, as does the adrenal medulla (a modified form of sympathetic ganglionic neuron tissue). Bronchial smooth muscle contains muscarinic cholinoceptors and noncholinergic receptors. The answer is A. 7. Botulinum toxin impairs all types of cholinergic transmission, including transmission at ganglionic synapses and somatic motor nerve endings. Botulinum toxin prevents discharge of vesicular transmitter content from cholinergic nerve endings. All of the signs listed except cycloplegia indicate increased muscle contraction; cycloplegia (paralysis of accommodation) results in blurred near vision. The answer is B. 8. When blood pressure increases, the parasympathetic system is activated and heart rate decreases. Acetylcholine is the transmitter at parasympathetic nerve endings innervating the sinus node (nerve endings of the vagus nerve). The answer is A. 9. The reuptake carrier “z” (also known as NET) transports norepinephrine back into the nerve ending after release and is blocked by cocaine. The answer is C. 10. The vesicular carrier “y” in the diagram transports dopamine and norepinephrine into the vesicles for storage. It can be blocked by reserpine. Hemicholiniums and vesamicol block transporters in cholinergic nerves. The answer is D. SKILL KEEPER ANSWER: DRUG PERMEATION (SEE CHAPTER 1) Botulinum toxin is too large to cross membranes by means of lipid or aqueous diffusion. It must bind to membrane receptors and enter by endocytosis. Botulinum-binding receptors for endocytosis are present on cholinergic neurons but not adrenergic neurons. CHECKLIST When you complete this chapter, you should be able to: ❑ Describe the steps in the synthesis, storage, release, and termination of action of the major autonomic transmitters. ❑ Name 2 cotransmitter substances. ❑ Name the major types and subtypes of autonomic receptors and the tissues in which they are found. ❑ Describe the organ system effects of stimulation of the parasympathetic and sympathetic systems. ❑ Name examples of inhibitors of acetylcholine and norepinephrine synthesis, storage, and release. Predict the effects of these inhibitors on the function of the major organ systems. ❑ List the determinants of blood pressure and describe the baroreceptor reflex response for the following perturbations: (1) blood loss, (2) administration of a vasodilator, (3) a vasoconstrictor, (4) a cardiac stimulant, (5) a cardiac depressant. ❑ Describe the results of transplantation of the heart (with interruption of its autonomic nerves) on cardiac function. ❑ Describe the actions of several toxins that affect nerve function: tetrodotoxin, saxitoxin, botulinum toxins, and latrotoxin. CHAPTER 6 Introduction to Autonomic Pharmacology SUMMARY TABLE: Introduction–Autonomic Drugs Drug Comment Acetylcholine Primary transmitter at cholinergic nerve endings (preganglionic ANS, postganglionic parasympathetic, postganglionic sympathetic to thermoregulatory sweat glands, and somatic neuromuscular end plates) Amphetamine Sympathomimetic drug that facilitates the release of catecholamines from adrenergic nerve endings Botulinum toxin Bacterial toxin that enzymatically disables release of acetylcholine from cholinergic nerve endings Cocaine Sympathomimetic drug that impairs reuptake of catecholamine transmitters (norepinephrine, dopamine) by adrenergic nerve endings; it is also a local anesthetic Dopamine Important central nervous system (CNS) transmitter with some peripheral effects (renal vasodilation, cardiac stimulation) Epinephrine Hormone released from adrenal medulla, neurotransmitter in CNS Hemicholiniums Research drugs that inhibit transport of choline into cholinergic nerve endings Hexamethonium Research drug that blocks all ANS ganglia and prevents autonomic compensatory reflexes Metanephrine Product of epinephrine and norepinephrine metabolism Metyrosine Inhibitor of tyrosine hydroxylase, the rate-limiting enzyme in norepinephrine synthesis Norepinephrine Primary transmitter at most sympathetic postganglionic nerve endings; important CNS transmitter Reserpine Drug that inhibits VMAT, transporter of dopamine and norepinephrine into transmitter vesicles of adrenergic nerves Tetrodotoxin, saxitoxin Toxins that block sodium channels and thereby limit transmission in all nerve fibers Vesamicol Drug that inhibits VAT, transporter of acetylcholine into its transmitter vesicles 59 C Cholinoceptor-Activating & Cholinesterase-Inhibiting Drugs Drugs with acetylcholine-like effects (cholinomimetics) consist of 2 major subgroups on the basis of their mode of action (ie, whether they act directly at the acetylcholine receptor or indirectly through inhibition of cholinesterase). Drugs in the direct-acting subgroup are further subdivided on the basis of H A P T E R 7 their spectrum of action (ie, whether they act on muscarinic or nicotinic cholinoceptors). Acetylcholine may be considered the prototype that acts directly at both muscarinic and nicotinic receptors. Neostigmine is a prototype for the indirect-acting cholinesterase inhibitors. Cholinomimetic (cholinergic) drugs Direct-acting Muscarinic Indirect-acting Nicotinic Organophosphates (very long acting) (parathion) Carbamates (intermediate to long acting) (neostigmine) Choline esters (acetylcholine) Alkaloids (pilocarpine) Edrophonium (short acting) DIRECT-ACTING CHOLINOMIMETIC AGONISTS This class comprises a group of choline esters (acetylcholine, methacholine, carbachol, bethanechol) and a second group of naturally occurring alkaloids (muscarine, pilocarpine, nicotine, lobeline). Newer drugs are occasionally introduced for special applications. The members differ in their spectrum of action (amount of muscarinic versus nicotinic stimulation) and in their pharmacokinetics (Table 7–1). Both factors influence their clinical use. 60 A. Classification Muscarinic agonists are parasympathomimetic; that is, they mimic the actions of parasympathetic nerve stimulation in addition to other effects. Five subgroups of muscarinic receptors have been identified (Table 7–2), but the muscarinic agonists available for clinical use activate them nonselectively. Nicotinic agonists act on both ganglionic or neuromuscular cholinoceptors; agonist selectivity is limited. On the other hand, a few slightly selective muscarinic antagonists and a separate group of relatively selective nicotinic receptor antagonists are available (Chapter 8). CHAPTER 7 Cholinoceptor-Activating & Cholinesterase-Inhibiting Drugs High-Yield Terms to Learn Choline esters A cholinomimetic drug consisting of choline (an alcohol) or a choline derivative, esterified with an acidic substance (eg, acetic or carbamic acid); usually poorly lipid-soluble Cholinergic crisis The clinical condition of excessive activation of cholinoceptors; it may include skeletal muscle weakness as well as parasympathetic effects, usually caused by cholinesterase inhibitors; cf myasthenic crisis Cholinomimetic alkaloids A drug with weakly alkaline properties (usually an amine of plant origin) whose effects resemble those of acetylcholine; usually lipid-soluble Cyclospasm Marked contraction of the ciliary muscle; maximum accommodation for close vision Direct-acting cholinomimetic A drug that binds and activates cholinoceptors; the effects mimic those of acetylcholine Endothelium-derived relaxing factor (EDRF) A potent vasodilator substance, largely nitric oxide (NO), that is released from vascular endothelial cells Indirect-acting cholinomimetic A drug that amplifies the effects of endogenous acetylcholine by inhibiting acetylcholinesterase Muscarinic agonist A cholinomimetic drug that binds muscarinic receptors and has primarily muscarine-like actions Myasthenic crisis In patients with myasthenia, an acute worsening of symptoms; usually relieved by increasing cholinesterase inhibitor treatment; cf cholinergic crisis Nicotinic agonist A cholinomimetic drug that binds nicotinic receptors and has primarily nicotine-like actions Organophosphate An ester of phosphoric acid and an alcohol that inhibits cholinesterase Organophosphate aging A process whereby the organophosphate, after binding to cholinesterase, is chemically modified and becomes more firmly bound to the enzyme Parasympathomimetic A drug whose effects resemble those of stimulating the parasympathetic nerves TABLE 7–1 Some cholinomimetics: spectrum of action and pharmacokinetics. a Drug Spectrum of Actiona Pharmacokinetic Features Direct-acting Acetylcholine B Bethanechol Carbachol Pilocarpine Nicotine Varenicline M B M N N Rapidly hydrolyzed by cholinesterase (ChE); duration of action 5–30 s; poor lipid solubility Resistant to ChE; orally active, poor lipid solubility; duration of action 30 min to 2 h Like bethanechol Not an ester, good lipid solubility; duration of action 30 min to 2 h Not an ester; duration of action 1–6 h; high lipid solubility Partial agonist at N receptors, high lipid solubility; duration 12–24 h Indirect-acting Edrophonium B Neostigmine B Physostigmine B Pyridostigmine Echothiophate Parathion Sarin B B B B B, both M and N; M, muscarinic; N, nicotinic. Alcohol, quaternary amine, poor lipid solubility, not orally active; duration of action 5–15 min Carbamate, quaternary amine, poor lipid solubility, orally active; duration of action 30 min to 2 h or more Carbamate, tertiary amine, good lipid solubility, orally active; duration of action 30 min to 2 h Carbamate, like neostigmine, but longer duration of action (4–8 h) Organophosphate, moderate lipid solubility; duration of action 2–7 days Organophosphate, high lipid solubility; duration of action 7–30 days; insecticide Organophosphate, very high lipid solubility, nerve gas 61 62 PART II Autonomic Drugs TABLE 7–2 Cholinoceptor types and their postreceptor mechanisms. Receptor Type G Protein Postreceptor Mechanisms M1 Gq ↑ IP3, DAG cascade M2 Gi ↓ cAMP synthesis M3 Gq ↑ IP3, DAG cascade M4 Gi ↓ cAMP synthesis M5 Gq ↑ IP3, DAG cascade NM None Na+/K+ depolarizing current NN None Na+/K+ depolarizing current cAMP, cyclic adenosine monophosphate; DAG, diacylglycerol; IP3, inositol1,4,5-trisphosphate. SKILL KEEPER: DRUG METABOLISM (SEE CHAPTER 4) Acetylcholine is metabolized in the body by hydrolysis of the ester bond. Is this a phase I or phase II metabolic reaction? The Skill Keeper Answer appears at the end of the chapter. B. Molecular Mechanisms of Action 1. Muscarinic mechanisms—Muscarinic receptors are G proteincoupled receptors (GPCRs) (Table 7–2). Gq protein coupling of M1 and M3 muscarinic receptors to phospholipase C, a membrane-bound enzyme, leads to the release of the second messengers, diacylglycerol (DAG) and inositol-1,4,5-trisphosphate (IP3). DAG modulates the action of protein kinase C, an enzyme important in secretion, whereas IP3 evokes the release of calcium from intracellular storage sites, which in smooth muscle results in contraction. M2 muscarinic receptors couple to adenylyl cyclase through the inhibitory Gi-coupling protein. A third mechanism couples the same M2 receptors via the βγ subunit of the G protein to potassium channels in the heart and elsewhere; muscarinic agonists facilitate opening of these channels. M4 and M5 receptors may be important in the central nervous system (CNS) but have not been shown to play major roles in peripheral organs. 2. Nicotinic mechanism—The mechanism of nicotinic action has been clearly defined. The nicotinic acetylcholine receptor is located on a channel protein that is selective for sodium and potassium. When the receptor is activated, the channel opens and depolarization of the cell occurs as a direct result of the influx of sodium, causing an excitatory postsynaptic potential (EPSP). If large enough, the EPSP evokes a propagated action potential in the surrounding membrane. The nicotinic receptors on sympathetic and parasympathetic ganglion neurons (NN, also denoted NG) differ slightly from those on neuromuscular end plates (NM). C. Tissue and Organ Effects The tissue and organ system effects of cholinomimetics are summarized in Table 7–3. Note that vasodilation is not a parasympathomimetic response (ie, it is not evoked by parasympathetic nerve discharge, even though directly acting cholinomimetics cause vasodilation). This vasodilation results from the release of endothelium-derived relaxing factor (EDRF; nitric oxide and possibly other substances) in the vessels, mediated by uninnervated muscarinic receptors on the endothelial cells. Note also that decreased blood pressure evokes the baroreceptor reflex, resulting in strong compensatory sympathetic discharge to the heart. As a result, injections of small to moderate amounts of direct-acting muscarinic cholinomimetics often cause tachycardia, whereas parasympathetic (vagal) nerve discharge to the heart causes bradycardia. Another effect seen with cholinomimetic drugs but not with parasympathetic nerve stimulation is thermoregulatory (eccrine) sweating; this is a sympathetic cholinergic effect (see Chapter 6). The tissue and organ level effects of nicotinic ganglionic stimulation depend on the autonomic innervation of the organ involved. The blood vessels are dominated by sympathetic innervation; therefore, nicotinic receptor activation results in vasoconstriction mediated by sympathetic postganglionic nerve discharge. The gut is dominated by parasympathetic control; nicotinic drugs increase motility and secretion because of increased parasympathetic postganglionic neuron discharge. Nicotinic neuromuscular end plate activation by direct-acting drugs results in fasciculations and spasm of the muscles involved. Prolonged activation results in paralysis (see Chapter 27), which is an important hazard of exposure to nicotine-containing and organophosphate insecticides. D. Clinical Use Several clinical conditions benefit from an increase in cholinergic activity, including glaucoma, Sjogren’s syndrome, and loss of normal PANS activity in the bowel and bladder. Direct-acting nicotinic agonists are used in smoking cessation and to produce skeletal muscle paralysis (succinylcholine, Chapter 27). Indirect-acting agents are used when increased nicotinic activation is needed at the neuromuscular junction (see discussion of myasthenia gravis). Nicotine and related neonicotinoids are used as insecticides despite reported toxic effects on bee colonies. Varenicline is a newer nicotinic agonist with partial agonist properties. It appears to reduce craving in persons addicted to nicotine through a nonautonomic action. E. Toxicity The signs and symptoms of overdosage are readily predicted from the general pharmacology of acetylcholine. 1. Muscarinic toxicity—These effects include CNS stimulation (uncommon with choline esters and pilocarpine), miosis, spasm of accommodation, bronchoconstriction, excessive gastrointestinal and genitourinary smooth muscle activity, increased secretory activity (sweat glands, airway, gastrointestinal tract, lacrimal glands), and vasodilation. Transient bradycardia occurs, followed by reflex tachycardia if the drug is administered as an intravenous bolus; reflex tachycardia occurs otherwise. Muscarine and similar alkaloids CHAPTER 7 Cholinoceptor-Activating & Cholinesterase-Inhibiting Drugs 63 TABLE 7–3 Effects of cholinomimetics on major organ systems. Organ Responsea CNS Complex stimulatory effects. Nicotine: elevation of mood, alerting, addiction (nicotine-naïve individuals often suffer nausea and vomiting on initial exposure); physostigmine: convulsions; excessive concentrations may cause coma Eye Sphincter muscle of iris Ciliary muscle Contraction (miosis) Contraction (accommodation for near vision), cyclospasm Heart Sinoatrial node Atria Atrioventricular node Ventricles Decrease in rate (negative chronotropy), but note important reflex response in intact subject (see text) Decrease in contractile force (negative inotropy); decrease in refractory period Decrease in conduction velocity (negative dromotropy), increase in refractory period Small decrease in contractile force Blood vessels Dilation via release of EDRF from endothelium Bronchi Contraction (bronchoconstriction) Gastrointestinal tract Motility Sphincters Increase in smooth muscle contraction, peristalsis Decrease in tone, relaxation (Exception: gastroesophageal sphincter contracts) Urinary bladder Detrusor Trigone and sphincter Increase in contraction Relaxation; voiding Skeletal muscle Activation of neuromuscular end plates, contraction Glands (exocrine) Increased secretion (thermoregulatory sweating, lacrimation, salivation, bronchial secretion, gastrointestinal glands) a Only the direct effects are indicated; homeostatic responses to these direct actions may be important (see text). EDRF, endothelium-derived relaxing factor (primarily nitric oxide). are found in certain mushrooms (Inocybe species and Amanita muscaria) and are responsible for the short-duration type of mushroom poisoning, which is characterized by nausea, vomiting, and diarrhea. (The much more dangerous and potentially lethal form of mushroom poisoning from Amanita phalloides and related species involves initial vomiting and diarrhea but is followed by hepatic and renal necrosis. It is not caused by muscarinic agonists but by amanitin and phalloidin, RNA polymerase inhibitors.) 2. Nicotinic toxicity—Toxic effects include ganglionic stimulation and block and neuromuscular end plate depolarization leading to fasciculations and then paralysis. The neuromuscular effects are described in greater detail in Chapter 27. CNS toxicity includes stimulation (including convulsions) followed by depression. Nicotine in small doses, ie, via smoking, is strongly addicting. INDIRECT-ACTING AGONISTS A. Classification and Prototypes Hundreds of indirect-acting cholinomimetic drugs have been synthesized in 2 major chemical classes: carbamic acid esters (carbamates) and phosphoric acid esters (organophosphates). These drugs are acetylcholinesterase (AChE) inhibitors. Neostigmine is a prototypic carbamate, whereas parathion, an important insecticide, is a prototypic organophosphate. A third class has only one clinically useful member: edrophonium is an alcohol (not an ester) with a very short duration of action. B. Mechanism of Action Both carbamate and organophosphate inhibitors bind to cholinesterase and undergo prompt hydrolysis. The alcohol portion of the molecule is then released. The acidic portion (carbamate ion or phosphate ion) is released much more slowly from the enzyme active site, preventing the binding and hydrolysis of endogenous acetylcholine. As a result, these drugs amplify acetylcholine effects wherever the transmitter is released. Edrophonium, though not an ester, has sufficient affinity for the enzyme active site to similarly prevent access of acetylcholine for 5–15 min. After hydrolysis, carbamates are released by cholinesterase over a period of 2–8 h. Organophosphates are long-acting drugs; they form an extremely stable phosphate complex with the enzyme. After initial hydrolysis, the phosphoric acid residue is released over periods of days to weeks. Recovery is due in part to synthesis of new enzyme. 64 PART II Autonomic Drugs C. Effects By inhibiting cholinesterase, these agents cause an increase in the concentration, half-life, and actions of acetylcholine in synapses where acetylcholine is released physiologically. Therefore, the indirect agents have muscarinic or nicotinic effects depending on which organ system is under consideration. Cholinesterase inhibitors do not have significant actions at uninnervated sites where acetylcholine is not normally released (eg, vascular endothelial cells). D. Clinical Uses The clinical applications of the AChE inhibitors are predictable from a consideration of the organs and the diseases that benefit from an amplification of cholinergic activity. These applications are summarized in the Drug Summary Table. Carbamates, which include neostigmine, physostigmine, pyridostigmine, and ambenonium, are used far more often in therapeutics than are organophosphates. The treatment of myasthenia is especially important. (Because myasthenia is an autoimmune disorder, treatment may also include thymectomy and immunosuppressant drugs.) Rivastigmine, a carbamate, and several other cholinesterase inhibitors are used exclusively in Alzheimer’s disease. A portion of their action may be due to other, unknown mechanisms. Although their effects are modest and temporary, these drugs are frequently used in this devastating condition. Some carbamates (eg, carbaryl) are used in agriculture as insecticides. Two organophosphates used in medicine are malathion (a scabicide) and metrifonate (an antihelminthic agent). Edrophonium is used for the rapid reversal of nondepolarizing neuromuscular blockade (Chapter 27), in the diagnosis of myasthenia, and in differentiating myasthenic crisis from cholinergic crisis in patients with this disease. Because cholinergic crisis can result in muscle weakness like that of myasthenic crisis, distinguishing the 2 conditions may be difficult. Administration of a short-acting cholinomimetic, such as edrophonium, will improve muscle strength in myasthenic crisis but weaken it in cholinergic crisis. E. Toxicity In addition to their therapeutic uses, some AChE inhibitors (especially organophosphates) have clinical importance because of accidental exposures to toxic amounts of pesticides. The most toxic of these drugs (eg, parathion) can be rapidly fatal if exposure is not immediately recognized and treated. After standard protection of vital signs (see Chapter 58), the antidote of first choice is the antimuscarinic agent atropine, but this drug has no effect on the nicotinic signs of toxicity. Nicotinic toxicity is treated by regenerating active cholinesterase. Immediately after binding to cholinesterase, most organophosphate inhibitors can be removed from the enzyme by the use of regenerator compounds such as pralidoxim (see Chapter 8), and this may reverse both nicotinic and muscarinic signs. If the enzyme-phosphate binding is allowed to persist, however, aging (a further chemical change) occurs and regenerator drugs can no longer remove the inhibitor. Treatment is described in more detail in Chapter 8. Because of their toxicity and short persistence in the environment, organophosphates are used extensively in agriculture as insecticides and antihelminthic agents; examples are malathion and parathion. Some of these agents (eg, malathion, dichlorvos) are relatively safe in humans because they are metabolized rapidly to inactive products in mammals (and birds) but not in insects. Some are prodrugs (eg, malathion, parathion) and must be metabolized to the active product (malaoxon from malathion, paraoxon from parathion). The signs and symptoms of poisoning are the same as those described for the direct-acting agents, with the following exceptions: vasodilation is a late and uncommon effect; bradycardia is more common than tachycardia; CNS stimulation is common with organophosphate and physostigmine overdosage and includes convulsions, followed by respiratory and cardiovascular depression. The spectrum of toxicity can be remembered with the aid of the mnemonic DUMBBELSS (diarrhea, urination, miosis, bronchoconstriction, bradycardia, excitation [of skeletal muscle and CNS], lacrimation, and salivation and sweating). QUESTIONS 1. A 30-year-old woman undergoes abdominal surgery. In spite of minimal tissue damage, complete ileus (absence of bowel motility) follows, and she complains of severe bloating. She also finds it difficult to urinate. Mild cholinomimetic stimulation with bethanechol or neostigmine is often effective in relieving these complications of surgery. Neostigmine and bethanechol in moderate doses have significantly different effects on which one of the following? (A) Gastric secretory cells (B) Vascular endothelium (C) Salivary glands (D) Sweat glands (E) Ureteral tone 2. Parathion has which one of the following characteristics? (A) It is inactivated by conversion to paraoxon (B) It is less toxic to humans than malathion (C) It is more persistent in the environment than DDT (D) It is poorly absorbed through skin and lungs (E) If treated early, its toxicity may be partly reversed by pralidoxime 3. Ms Brown has been treated for myasthenia gravis for several years. She reports to the emergency department complaining of recent onset of weakness of her hands, diplopia, and difficulty swallowing. She may be suffering from a change in response to her myasthenia therapy, that is, a cholinergic or a myasthenic crisis. Which of the following is the best drug for distinguishing between myasthenic crisis (insufficient therapy) and cholinergic crisis (excessive therapy)? (A) Atropine (B) Edrophonium (C) Physostigmine (D) Pralidoxime (E) Pyridostigmine CHAPTER 7 Cholinoceptor-Activating & Cholinesterase-Inhibiting Drugs 4. A crop duster pilot has been accidentally exposed to a high concentration of a highly toxic agricultural organophosphate insecticide. If untreated, the cause of death from such exposure would probably be (A) Cardiac arrhythmia (B) Gastrointestinal bleeding (C) Heart failure (D) Hypotension (E) Respiratory failure 5. Mr Green has just been diagnosed with dysautonomia (chronic idiopathic autonomic insufficiency). You are considering different therapies for his disease. Pyridostigmine and neostigmine may cause which one of the following in this patient? (A) Bronchodilation (B) Cycloplegia (C) Diarrhea (D) Irreversible inhibition of acetylcholinesterase (E) Reduced gastric acid secretion 6. Parasympathetic nerve stimulation and a slow infusion of bethanechol will each (A) Cause ganglion cell depolarization (B) Cause skeletal muscle end plate depolarization (C) Cause vasodilation (D) Increase bladder tone (E) Increase heart rate 7. Actions and clinical uses of muscarinic cholinoceptor agonists include which one of the following? (A) Bronchodilation (treatment of asthma) (B) Miosis (treatment of glaucoma) (C) Decreased gastrointestinal motility (treatment of diarrhea) (D) Decreased neuromuscular transmission and relaxation of skeletal muscle (during surgical anesthesia) (E) Increased sweating (treatment of fever) 8. Which of the following is a direct-acting cholinomimetic that is lipid-soluble and is used to facilitate smoking cessation? (A) Acetylcholine (B) Bethanechol (C) Neostigmine (D) Physostigmine (E) Varenicline 9. A 3-year-old child is admitted to the emergency department after taking a drug from her parents’ medicine cabinet. The signs suggest that the drug is an indirect-acting cholinomimetic with little or no CNS effect and a duration of action of about 2–4 h. Which of the following is the most likely cause of these effects? (A) Acetylcholine (B) Bethanechol (C) Neostigmine (D) Physostigmine (E) Pilocarpine 65 10. Which of the following is the primary second-messenger process in the contraction of the ciliary muscle when focusing on near objects? (A) cAMP (cyclic adenosine monophosphate) (B) DAG (diacylglycerol) (C) Depolarizing influx of sodium ions via a channel (D) IP3 (inositol 1,4,5-trisphosphate) (E) NO (nitric oxide) ANSWERS 1. Because neostigmine acts on the enzyme cholinesterase, which is present at all cholinergic synapses, this drug increases acetylcholine effects at nicotinic junctions as well as muscarinic ones. Bethanechol, on the other hand, is a direct-acting agent that is selective for muscarinic receptors regardless of whether the receptors are innervated or not. The muscarinic receptors on vascular endothelial cells are not innervated and respond only to direct-acting drugs. The answer is B. 2. The “-thion” organophosphates (those containing the P:S bond) are activated, not inactivated, by conversion to “-oxon” (P:O) derivatives. They are less stable than halogenated hydrocarbon insecticides of the DDT type; therefore, they are less persistent in the environment. Parathion is more toxic than malathion. It is very lipid-soluble and rapidly absorbed through the lungs and skin. Pralidoxime has very high affinity for the phosphorus atom and is a chemical antagonist of organophosphates. The answer is E. 3. Any of the cholinesterase inhibitors (choices B, C, or E) would effectively correct myasthenic crisis. However, because cholinergic crisis (if that is what is causing the symptoms) would be worsened by a cholinomimetic, we choose the shortest-acting cholinesterase inhibitor, edrophonium. The answer is B. 4. Respiratory failure, from neuromuscular paralysis or CNS depression, is the most important cause of acute deaths in cholinesterase inhibitor toxicity. The answer is E. 5. Cholinesterase inhibition is typically associated with increased (never decreased) bowel activity. (Fortunately, many patients become tolerant to this effect.) The answer is C. 6. Choice (E) is not correct because the vagus slows the heart. Parasympathetic nerve stimulation does not cause vasodilation (most vessels do not receive parasympathetic innervation), so choice (C) is incorrect. Ganglion cells and the end plate contain nicotinic receptors, which are not affected by bethanechol, a direct-acting muscarinic agonist. The answer is D. 7. Muscarinic agonists cause accommodation and cyclospasm, the opposite of paralysis of accommodation (cycloplegia). In acute angle-closure glaucoma and chronic open-angle glaucoma, this may result in a desirable increased outflow of aqueous and decreased intraocular pressure. These agents may cause bronchospasm but have no effect on neuromuscular transmission. They may cause diarrhea and are not used in its treatment. Muscarinic agonists may also cause sweating, but drug-induced sweating is of no value in the treatment of fever. The answer is B. 66 PART II Autonomic Drugs 8. Varenicline is a lipid-soluble partial agonist at nicotinic receptors and is used to reduce craving for tobacco in smokers. The answer is E. 9. Neostigmine is the prototypical indirect-acting cholinomimetic; it is a quaternary (charged) substance with poor lipid solubility; its duration of action is about 2–4 h. Physostigmine is similar but has good lipid solubility and significant CNS effects. The answer is C. 10. Cholinomimetics cause smooth muscle contraction mainly through the release of intracellular calcium. This release is triggered by an increase in IP3 acting on receptors in the endoplasmic reticulum. The answer is D. SKILL KEEPER ANSWER: DRUG METABOLISM (SEE CHAPTER 4) The esters acetylcholine and methacholine are hydrolyzed by acetylcholinesterase. Hydrolytic drug metabolism reactions are classified as phase I. CHECKLIST When you complete this chapter, you should be able to: ❑ List the locations and types of acetylcholine receptors in the major organ systems (CNS, autonomic ganglia, eye, heart, vessels, bronchi, gut, genitourinary tract, skeletal muscle, exocrine glands). ❑ Describe the second messengers involved and the effects of acetylcholine on the major organs. ❑ List the major clinical uses of cholinomimetic agonists. ❑ Describe the pharmacodynamic differences between direct-acting and indirect- acting cholinomimetic agents. ❑ List the major pharmacokinetic differences of the direct- and indirect-acting cholinomimetics. ❑ List the major signs and symptoms of (1) organophosphate insecticide poisoning and (2) acute nicotine toxicity. DRUG SUMMARY TABLE: Cholinoceptor-Activating & Cholinesterase-Inhibiting Drugs. Subclass Mechanism of Action Clinical and Other Applications Pharmacokinetics Toxicities, Interactions Direct-acting, muscarinic agonists Bethanechol Activates muscarinic (M) receptors • increases IP3 and DAG Bladder and bowel atony, for example, after surgery or spinal cord injury Oral, IM activity Poor lipid solubility: does not enter CNS Duration: 0.3–2 h All parasympathomimetic effects: cyclospasm, diarrhea, urinary urgency, plus vasodilation, reflex tachycardia, and sweating Pilocarpine Same as bethanechol • may also activate EPSP via M receptors in ganglia Sjögren’s syndrome (increases salivation) • was used in glaucoma (causes miosis, cyclospasm) Oral, IM activity Good lipid solubility, topical activity in eye Similar to bethanechol but may cause vasoconstriction via ganglionic effect Muscarine Same as bethanechol Alkaloid found in mushrooms Low lipid solubility but readily absorbed from gut Mushroom poisoning of fast-onset type Smoking cessation (also used as insecticide) High lipid solubility, absorbed by all routes Generalized ganglionic stimulation: hypertension, tachycardia, nausea, vomiting, diarrhea • For smoking cessation, usually used as gum or transdermal patch Duration: 4–6 h Major overdose: convulsions, paralysis, coma Direct-acting, nicotinic agonists Nicotine Activates all nicotinic (N) receptors • opens Na+-K+ channels in ganglia and neuromuscular end plates Varenicline A partial agonist at N receptors Smoking cessation High lipid solubility, oral activity • Duration: ~12 h Hypertension, sweating, sensory disturbance, diarrhea, polyuria, menstrual disturbance Succinylcholine N-receptor agonist, moderately selective for neuromuscular end plate (NM receptors) Muscle relaxation (see Chapter 27) Highly polar, used IV • Duration: 5–10 min Initial muscle spasms and postoperative pain • Prolonged action in persons with abnormal butyrylcholinesterase Inhibitor of cholinesterase • amplifier of endogenously released Ach Reversal of NM block by nondepolarizing drugs • diagnosis of myasthenia gravis Highly polar • used IV • Duration: 5–10 min Increased parasympathetic effects, especially nausea, vomiting, diarrhea, urinary urgency Indirect-acting, alcohol Edrophonium Indirect-acting, carbamates Neostigmine Like edrophonium plus small direct nicotinic agonist action Reversal of NM block, treatment of myasthenia Moderately polar but orally active • Duration: 2–4 h Like edrophonium but longer duration Pyridostigmine Like edrophonium Treatment of myasthenia Moderately polar but orally active • Duration: 4–8 h Like edrophonium but longer duration Physostigmine Like edrophonium Reversal of severe atropine poisoning (IV) • occasionally used in acute glaucoma (topical) Lipid soluble • can be used topically in the eye • Duration: 2–4 h Like edrophonium but longer duration plus CNS effects: seizures 67 (Continued ) 68 DRUG SUMMARY TABLE: Cholinoceptor-Activating & Cholinesterase-Inhibiting Drugs. (Continued ) Subclass Mechanism of Action Clinical and Other Applications Pharmacokinetics Toxicities, Interactions Indirect-acting, organophosphates Parathion Like edrophonium Insecticide only Duration: days to weeks Highly lipid-soluble Highly dangerous insecticide • causes all parasympathetic effects plus muscle paralysis and coma Malathion Like edrophonium Insecticide and scabicide (topical) Duration: days Highly lipid-soluble but metabolized to inactive products in mammals and birds Much safer insecticide than parathion Sarin, tabun, others Like parathion Nerve gases • terrorist threat Like parathion but more rapid action Rapidly lethal Alzheimer’s disease Lipid soluble, enter CNS • Half-lives: 1.5–70 h Nausea, vomiting Indirect-acting, for Alzheimer’s disease Rivastigmine, galantamine, donepezil; tacrine is obsolete Cholinesterase inhibition plus variable other poorly understood effects ACh, acetylcholine; DAG, diacylglycerol; EPSP, excitatory postsynaptic potential; IP 3 , inositol-1,4,5-trisphosphate. C Cholinoceptor Blockers & Cholinesterase Regenerators The cholinoceptor antagonists consist of 2 subclasses based on their spectrum of action (ie, block of muscarinic versus nicotinic receptors). These drugs are pharmacologic antagonists or inverse agonists (eg, atropine). A third, special, subgroup, the M1-selective (pirenzepine) Nonselective (atropine) A P T E R 8 cholinesterase regenerators, are not receptor blockers but rather are chemical antagonists of organophosphate acetylcholinesterase (AChE) inhibitors. Anticholinergic drugs Antimuscarinic H Cholinesterase regenerators Antinicotinic Ganglion blockers (hexamethonium) MUSCARINIC ANTAGONISTS A. Classification and Pharmacokinetics Muscarinic antagonists can be subdivided according to their selectivity for specific M receptors or their lack of such selectivity. Although the division of muscarinic receptors into subgroups is well documented (Chapters 6 and 7), only 2 distinctly receptor-selective M1 antagonists have reached clinical trials (eg, pirenzepine, telenzepine, neither of which is used in the United States). However, as noted later, a few agents in use in the United States are somewhat selective for the M3 subtype. Most of the antimuscarinic drugs in use are relatively nonselective. The muscarinic blockers can also be subdivided on the basis of their primary clinical target organs (central nervous system [CNS], eye, bronchi, or gastrointestinal and genitourinary tracts). Drugs used for their effects on the CNS or the eye must be sufficiently lipid-soluble to cross lipid barriers. A major determinant of this property is the presence or absence of a permanently charged (quaternary) amine group in the drug molecule because charged molecules are less lipid-soluble (see Chapter 1). Neuromuscular blockers (tubocurarine) Oximes (pralidoxime) Atropine is the prototypical nonselective muscarinic blocker. This alkaloid is found in Atropa belladonna and many other plants. Because it is a tertiary amine, atropine is relatively lipid-soluble and readily crosses membrane barriers. The drug is well distributed into the CNS, the eye, and other organs. It is eliminated partially by metabolism in the liver and partially unchanged in the urine; half-life is approximately 2 h; and duration of action of normal doses is 4–8 h except in the eye (see Drug Summary Table). In ophthalmology, topical activity (the ability to enter the eye after conjunctival administration) and duration of action are important in determining the usefulness of several antimuscarinic drugs (see Clinical Uses). Similar ability to cross lipid barriers is essential for the agents used in parkinsonism. In contrast, the drugs used for their antisecretory or antispastic actions in the gut, bladder, and bronchi are often selected for minimum CNS activity; these drugs may incorporate quaternary amine groups to limit penetration through the blood– brain barrier. 69 70 PART II Autonomic Drugs High-Yield Terms to Learn Anticholinergic A drug that blocks muscarinic or nicotinic receptors, but commonly used to mean antimuscarinic Antimuscarinic A drug that blocks muscarinic but not nicotinic receptors Atropine fever Hyperthermia induced by antimuscarinic drugs; caused mainly by inhibition of sweating Atropine flush Marked cutaneous vasodilation of the arms and upper torso and head by toxic doses of antimuscarinic drugs, especially atropine; mechanism unknown Cholinesterase regenerator A chemical antagonist that binds the phosphorus of organophosphates and displaces AChE Cycloplegia Paralysis of accommodation; inability to focus on close objects Depolarizing blockade Flaccid skeletal muscle paralysis caused by persistent depolarization of the neuromuscular end plate Miotic A drug that constricts the pupil Mydriatic A drug that dilates the pupil Nondepolarizing blockade Flaccid skeletal muscle paralysis caused by blockade of the nicotinic receptor and prevention of end plate depolarization Parasympatholytic, parasympathoplegic A drug that reduces the effects of parasympathetic nerve stimulation, usually by blockade of the muscarinic receptors of autonomic effector tissues B. Mechanism of Action Although several are inverse agonists, muscarinic blocking agents act like competitive (surmountable) pharmacologic antagonists; their blocking effects can be overcome by increased concentrations of muscarinic agonists. D. Clinical Uses The muscarinic blockers have several useful therapeutic applications in the CNS, eye, bronchi, gut, and urinary bladder. These uses are listed in the Drug Summary Table at the end of this chapter. C. Effects The peripheral actions of muscarinic blockers are mostly predictable effects derived from cholinoceptor blockade (Table 8–1). These include the ocular, gastrointestinal, genitourinary, and secretory effects. The CNS effects are less predictable. CNS effects seen at therapeutic concentrations include sedation, reduction of motion sickness, and, as previously noted, reduction of some of the signs of parkinsonism. Cardiovascular effects at therapeutic doses include an initial slowing of heart rate caused by central effects or blockade of inhibitory presynaptic muscarinic receptors on vagus nerve endings. These are followed by the tachycardia and decreased atrioventricular conduction time that would be predicted from blockade of postsynaptic muscarinic receptors in the sinus node. M1-selective agents (not currently available in the United States) may be somewhat selective for the gastrointestinal tract. TABLE 8–1 Effects of muscarinic blocking drugs. Organ Effect Mechanism CNS Sedation, anti-motion sickness action, antiparkinson action, amnesia, delirium Block of muscarinic receptors, several subtypes Eye Bronchi Cycloplegia, mydriasis Bronchodilation, especially if constricted Relaxation, slowed peristalsis, reduced salivation Relaxation of bladder wall, urinary retention Initial bradycardia, especially at low doses, then tachycardia Block of muscarinic vasodilation; not manifest unless a muscarinic agonist is present Marked reduction of salivation; moderate reduction of lacrimation, sweating; less reduction of gastric secretion Block of M3 receptors Block of M3 receptors Gastrointestinal tract Genitourinary tract Heart Blood vessels SKILL KEEPER: DRUG IONIZATION (SEE CHAPTER 1) The pKa of atropine, a weak base, is 9.7. What fraction of atropine (an amine) is in the lipid-soluble form in urine of pH 7.7? The Skill Keeper Answer appears at the end of the chapter. Glands Skeletal muscle None Block of M1, M3 receptors Block of M3 and possibly M1 receptors Tachycardia from block of M2 receptors in the sinoatrial node Block of M3 receptors on endothelium of vessels Block of M1, M3 receptors CHAPTER 8 Cholinoceptor Blockers & Cholinesterase Regenerators 1. CNS—Scopolamine is standard therapy for motion sickness; it is one of the most effective agents available for this condition. A transdermal patch formulation is available. Benztropine, biperiden, and trihexyphenidyl are representative of several antimuscarinic agents used in parkinsonism. Although not as effective as levodopa (see Chapter 28), these agents may be useful as adjuncts or when patients become unresponsive to levodopa. Benztropine is sometimes used parenterally to treat acute dystonias caused by firstgeneration antipsychotic medications. 2. Eye—Antimuscarinic drugs are used to cause mydriasis, as indicated by the origin of the name belladonna (“beautiful lady”) from the ancient cosmetic use of extracts of the Atropa belladonna plant to dilate the pupils. They also cause cycloplegia and prevent accommodation. In descending order of duration of action, these drugs are atropine (>72 h), homatropine (24 h), cyclopentolate (2–12 h), and tropicamide (0.5–4 h). These agents are all well absorbed from the conjunctival sac into the eye. 3. Bronchi—Parenteral atropine has long been used to reduce airway secretions during general anesthesia. Ipratropium is a quaternary antimuscarinic agent used by inhalation to promote bronchodilation in asthma and chronic obstructive pulmonary disease (COPD). Although not as efficacious as β agonists, ipratropium is less likely to cause tachycardia and cardiac arrhythmias in sensitive patients. It has very few antimuscarinic effects outside the lungs because it is poorly absorbed and rapidly metabolized. Tiotropium is an analog with a longer duration of action. Aclidinium is a newer long-acting antimuscarinic drug available in combination with a long-acting β2-adrenoceptor agonist for the treatment of COPD. 4. Gut—Atropine, methscopolamine, and propantheline were used in the past to reduce acid secretion in acid-peptic disease, but are now obsolete for this indication because they are not as effective as H2 blockers (Chapter 16) and proton pump inhibitors (Chapter 59), and they cause far more frequent and severe adverse effects. The M1-selective inhibitor pirenzepine is available in Europe for the treatment of peptic ulcer. Muscarinic blockers can also be used to reduce cramping and hypermotility in transient diarrheas, but drugs such as diphenoxylate and loperamide (Chapters 31, 59) are more effective. 5. Bladder—Oxybutynin, tolterodine, or similar agents may be used to reduce urgency in mild cystitis and to reduce bladder spasms after urologic surgery. Tolterodine, darifenacin, solifenacin, fesoterodine, and propiverine are slightly selective for M3 receptors and are promoted for the treatment of stress incontinence. 6. Cholinesterase inhibitor intoxication—Atropine, given parenterally in large doses, reduces the muscarinic signs of poisoning with AChE inhibitors. Pralidoxime (see below) is used to regenerate active AChE. E. Toxicity A traditional mnemonic for atropine toxicity is “Dry as a bone, hot as a pistol, red as a beet, mad as a hatter.” This description reflects both predictable antimuscarinic effects and some unpredictable actions. 71 1. Predictable toxicities—Antimuscarinic actions lead to several important and potentially dangerous effects. Blockade of thermoregulatory sweating may result in hyperthermia or atropine fever (“hot as a pistol”). This is the most dangerous effect of the antimuscarinic drugs in children and is potentially lethal in infants. Sweating, salivation, and lacrimation are all significantly reduced or stopped (“dry as a bone”). Moderate tachycardia is common, and severe tachycardia or arrhythmias are common with large overdoses. In the elderly, important toxicities include acute angle-closure glaucoma and urinary retention, especially in men with prostatic hyperplasia. Constipation and blurred vision are common adverse effects in all age groups. 2. Other toxicities—Toxicities not predictable from peripheral autonomic actions include CNS and cardiovascular effects. CNS toxicity includes sedation, amnesia, and delirium or hallucinations (“mad as a hatter”); convulsions may also occur. Central muscarinic receptors are probably involved. Other drug groups with antimuscarinic effects, for example, tricyclic antidepressants, may cause hallucinations or delirium in the elderly, who are especially susceptible to antimuscarinic toxicity. At very high doses, intraventricular conduction may be blocked; this action is probably not mediated by muscarinic blockade and is difficult to treat. Dilation of the cutaneous vessels of the arms, head, neck, and trunk also occurs at these doses; the resulting “atropine flush” (“red as a beet”) may be diagnostic of overdose with these drugs. The mechanism is unknown. 3. Treatment of toxicity—Treatment of toxicity is usually symptomatic. Severe tachycardia may require cautious administration of small doses of physostigmine. Hyperthermia can usually be managed with cooling blankets or evaporative cooling. F. Contraindications The antimuscarinic agents should be used cautiously in infants because of the danger of hyperthermia. The drugs are relatively contraindicated in persons with glaucoma, especially the closedangle form, and in men with prostatic hyperplasia. NICOTINIC ANTAGONISTS A. Ganglion-Blocking Drugs Blockers of ganglionic nicotinic receptors act like competitive pharmacologic antagonists, although there is evidence that some also block the pore of the nicotinic channel itself. These drugs were the first successful agents for the treatment of hypertension. Hexamethonium (C6, a prototype), mecamylamine, and several other ganglion blockers were extensively used for this disease. Unfortunately, the adverse effects of ganglion blockade in hypertension are so severe (both sympathetic and parasympathetic divisions are blocked) that patients were unable to tolerate them for long periods (Table 8–2). Trimethaphan was the ganglion blocker most recently used in clinical practice, but it too has been almost abandoned. It is poorly lipid-soluble, inactive orally, and 72 PART II Autonomic Drugs TABLE 8–2 Effects of ganglion-blocking drugs. Organ Effects CNS Antinicotinic action may include reduction of nicotine craving and amelioration of Tourette’s syndrome (mecamylamine only) Eye Moderate mydriasis and cycloplegia Bronchi Little effect; asthmatic patients may note some bronchodilation Gastrointestinal tract Marked reduction of motility, constipation may be severe Genitourinary tract Reduced contractility of the bladder; impairment of erection (parasympathetic block) and ejaculation (sympathetic block) Heart Moderate tachycardia and reduction in force and cardiac output at rest; block of exerciseinduced changes Vessels Reduction in arteriolar and venous tone, dosedependent reduction in blood pressure; orthostatic hypotension usually marked Glands Reductions in salivation, lacrimation, sweating, and gastric secretion Skeletal muscle No significant effect has a short half-life. It was used intravenously to treat severe accelerated hypertension (malignant hypertension) and to produce controlled hypotension. These drugs are still used in research. Recent interest has focused on nicotinic receptors in the CNS and their relation to nicotine addiction and to Tourette’s syndrome. Paradoxically, nicotine (in the form of nicotine gum or patches), varenicline (a partial agonist given by mouth), and mecamylamine, a nicotinic ganglion blocker that enters the CNS, have all been shown to have some benefit in smoking cessation. Because ganglion blockers interrupt sympathetic control of venous tone, they cause marked venous pooling; postural hypotension is a major manifestation of this effect. Other toxicities of ganglion-blocking drugs include dry mouth, blurred vision, constipation, and severe sexual dysfunction (Table 8–2). As a result, ganglion blockers are rarely used. B. Neuromuscular-Blocking Drugs Neuromuscular-blocking drugs are important for producing marked skeletal muscle relaxation that is important in surgery and in mechanical ventilation of patients. They are discussed in detail in Chapter 27. CHOLINESTERASE REGENERATORS Pralidoxime is the prototype cholinesterase regenerator. These chemical antagonists contain an oxime group, which has an extremely high affinity for the phosphorus atom in organophosphate insecticides. Because the affinity of the oxime group for phosphorus exceeds the affinity of the enzyme-active site for phosphorus, these agents are able to bind the inhibitor and displace the enzyme if aging has not occurred. The active enzyme is thus regenerated. Pralidoxime, the oxime currently available in the United States, is used to treat patients exposed to high doses of organophosphate AChE inhibitor insecticides, such as parathion, or to nerve gases. It is not recommended for use in carbamate AChE inhibitor overdosage. QUESTIONS 1. A 27-year old compulsive drug user injected a drug he thought was methamphetamine, but he has not developed any signs of methamphetamine action. He has been admitted to the emergency department and antimuscarinic drug overdose is suspected. Probable signs of atropine overdose include which one of the following? (A) Gastrointestinal smooth muscle cramping (B) Increased heart rate (C) Increased gastric secretion (D) Pupillary constriction (E) Urinary frequency 2. Which of the following is the most dangerous effect of belladonna alkaloids in infants and toddlers? (A) Dehydration (B) Hallucinations (C) Hypertension (D) Hyperthermia (E) Intraventricular heart block 3. Which one of the following can be blocked by atropine? (A) Decreased blood pressure caused by hexamethonium (B) Increased blood pressure caused by nicotine (C) Increased skeletal muscle strength caused by neostigmine (D) Tachycardia caused by exercise (E) Sweating caused by exercise Questions 4–5. Two new synthetic drugs (X and Y) are to be studied for their cardiovascular effects. The drugs are given to three anesthetized animals while the blood pressure is recorded. The first animal has received no pretreatment (control), the second has received an effective dose of a long-acting ganglion blocker, and the third has received an effective dose of a long-acting muscarinic antagonist. 4. Drug X caused a 50 mm Hg rise in mean blood pressure in the control animal, no blood pressure change in the ganglionblocked animal, and a 75 mm mean blood pressure rise in the atropine-pretreated animal. Drug X is probably a drug similar to (A) Acetylcholine (B) Atropine (C) Epinephrine (D) Hexamethonium (E) Nicotine CHAPTER 8 Cholinoceptor Blockers & Cholinesterase Regenerators 5. The net changes in heart rate induced by drug Y in these experiments are shown in the following graph. Percent change in heart rate + 50% 0 No blocker Ganglion blocker Y Muscarinic blocker Y 73 10. Which one of the following drugs has a very high affinity for the phosphorus atom in parathion and is often used to treat life-threatening insecticide toxicity? (A) Atropine (B) Benztropine (C) Bethanechol (D) Botulinum (E) Cyclopentolate (F) Neostigmine (G) Pralidoxime Y – 50% Drug Y is probably a drug similar to (A) Acetylcholine (B) Edrophonium (C) Hexamethonium (D) Nicotine (E) Pralidoxime 6. A 30-year-old man has been treated with several autonomic drugs for 4 weeks. He is now admitted to the emergency department showing signs of drug toxicity. Which of the following signs would distinguish between an overdose of a ganglion blocker versus a muscarinic blocker? (A) Cycloplegia (B) Dry skin in a warm environment (C) Miosis (D) Postural hypotension (E) Tachycardia 7. Which of the following is an accepted therapeutic indication for the use of antimuscarinic drugs? (A) Atrial fibrillation (B) Botulinum poisoning (C) Chronic obstructive pulmonary disease (COPD) (D) Glaucoma (E) Postoperative urinary retention 8. Which of the following is an expected effect of a therapeutic dose of an antimuscarinic drug? (A) Decreased cAMP (cyclic adenosine monophosphate) in cardiac muscle (B) Decreased DAG (diacylglycerol) in salivary gland tissue (C) Increased IP3 (inositol trisphosphate) in intestinal smooth muscle (D) Increased potassium efflux from smooth muscle (E) Increased sodium influx into the skeletal muscle end plate 9. Which one of the following drugs causes vasodilation that can be blocked by atropine? (A) Benztropine (B) Bethanechol (C) Botulinum toxin (D) Cyclopentolate (E) Edrophonium (F) Neostigmine (G) Pralidoxime ANSWERS 1. Tachycardia is a characteristic atropine overdose effect. Bradycardia is sometimes observed after small doses. None of the other choices are typical of atropine or methamphetamine overdose. The answer is B. 2. Choices B, D, and E are all possible effects of the atropine group. In infants, however, the most dangerous effect is hyperthermia. Deaths with body temperatures in excess of 42°C have occurred after the use of atropine-containing eye drops in children. The answer is D. 3. Atropine blocks muscarinic receptors and inhibits parasympathomimetic effects. Nicotine can induce both parasympathomimetic and sympathomimetic effects by virtue of its ganglion-stimulating action. Hypertension and exercise-induced tachycardia reflect sympathetic discharge with norepinephrine release and therefore would not be blocked by atropine. Exerciseinduced sweating is another sympathomimetic response, but it is mediated by acetylcholine released from sympathetic nerve fibers at eccrine sweat glands. The answer is E. 4. Drug X causes an increase in blood pressure that is blocked by a ganglion blocker but not by a muscarinic blocker. The pressor response is actually increased by pretreatment with atropine, a muscarinic blocker, suggesting that compensatory vagal discharge might have blunted the full response. This description fits a ganglion stimulant like nicotine but not epinephrine, since epinephrine’s pressor effects are produced at α receptors, not in the ganglia. The answer is E. 5. Drug Y causes an increase in heart rate that is blocked by a muscarinic blocker but reversed by a ganglion blocker. The fact that a ganglion blocker reverses the unknown drug’s effect suggests that the control response (tachycardia) involves the baroreceptor reflex. The description fits a directacting muscarinic stimulant such as acetylcholine (given in a dosage that causes a significant drop in blood pressure). An indirect-acting cholinomimetic (cholinesterase inhibitor, B) would not produce this pattern because the vascular muscarinic receptors involved in the depressor response are not innervated and are unresponsive to indirectly acting agents. The answer is A. 6. Neither ganglion blockers nor muscarinic blockers cause miosis; they cause mydriasis. Both classes of cholinoceptor blockers increase resting heart rate and cause cycloplegia, because these are determined largely by parasympathetic tone. Similarly, both can cause dry skin, since this requires cholinergic transmission. Postural hypotension, on the other hand, is a sign of sympathetic blockade, which would occur with ganglion blockers but not muscarinic blockers (Chapter 6). The answer is D. 74 PART II Autonomic Drugs 7. Atrial fibrillation and other arrhythmias are not responsive to antimuscarinic agents. Botulinum poisoning is associated with parasympathetic blockade. Parkinson’s disease, not Huntington’s, is partially responsive to antimuscarinic drugs. Antimuscarinic drugs tend to cause urinary retention and may precipitate or exacerbate glaucoma. Bronchospasm is mediated in part by vagal outflow in many patients with COPD and in some with asthma. The answer is C. 8. Muscarinic M1 and M3 receptors mediate increases in IP3 and DAG in target tissues (intestine, salivary glands). M2 receptors (heart) mediate a decrease in cAMP and an increase in potassium permeability. Antimuscarinic agents block these effects. The answer is B. 9. Bethanechol (Chapter 7) causes vasodilation by directly activating muscarinic receptors on the endothelium of blood vessels. This effect can be blocked by atropine. Indirectly acting agents (AChE inhibitors) do not typically cause vasodilation because the endothelial receptors are not innervated and acetylcholine is not released at this site. Pralidoxime is a distracter in this answer list. The answer is B. 10. Pralidoxime has a very high affinity for the phosphorus atom in organophosphate insecticides. The answer is G. SKILL KEEPER ANSWER: DRUG IONIZATION (SEE CHAPTER 1) The pKa of atropine is 9.7. According to the HendersonHasselbalch equation, Log (protonated / unprotonated) = pK a - pH Log (P / U) = 9.7 - 7.7 Log (P / U) = 2 P / U = antilog (2) = 100 /1 Therefore, about 99% of the drug is in the protonated form, 1% in the unprotonated form. Since atropine is a weak base, it is the unprotonated form that is lipid soluble. Therefore, about 1% of the atropine in the urine is lipid soluble. CHECKLIST When you complete this chapter, you should be able to: ❑ Describe the effects of atropine on the major organ systems (CNS, eye, heart, ves- sels, bronchi, gut, genitourinary tract, exocrine glands, skeletal muscle). ❑ List the signs, symptoms, and treatment of atropine overdose. ❑ List the major clinical indications and contraindications for the use of muscarinic antagonists. ❑ Describe the effects of the ganglion-blocking nicotinic antagonists. ❑ List one antimuscarinic agent promoted for each of the following uses: to produce mydriasis and cycloplegia; to treat parkinsonism, asthma, bladder spasm, and the muscarinic toxicity of insecticides ❑ Describe the mechanism of action and clinical use of pralidoxime. CHAPTER 8 Cholinoceptor Blockers & Cholinesterase Regenerators 75 DRUG SUMMARY TABLE: Cholinoceptor Blockers & Cholinesterase Regenerators Subclass Mechanism of Action Clinical Applications Pharmacokinetics Toxicities, Interactions Competitive pharmacologic antagonist (inverse agonist) at all M receptors Mydriatic, cycloplegic • antidote for cholinesterase inhibitor toxicity Lipid-soluble Duration: 2–4 h except in eye: ≥72 h All parasympatholytic effects plus sedation, delirium, hyperthermia, flushing Antimuscarinic, nonselective Atropine Benztropine, others: antiparkinsonism; oral and parenteral Dicyclomine, glycopyrrolate: oral, parenteral for gastrointestinal applications Homatropine, cyclopentolate, tropicamide: topical ophthalmic use to produce mydriasis, cycloplegia Ipratropium, tiotropium, aclidinium: inhaled for asthma, chronic obstructive pulmonary disease Oxybutynin: oral, transdermal, promoted for urinary urgency, incontinence Scopolamine: anti-motion sickness via transdermal patch Trospium: oral, for urinary urgency Antimuscarinic, selective Darifenacin, fesoterodine, solifenacin, tolterodine Pirenzepine, telenzepine Like atropine, but modest selectivity for M3 receptors Significant M1 selectivity Urinary urgency, incontinence Oral Duration: 12–24 h Excessive parasympatholytic effects Peptic disease (not available in USA) Oral Excessive parasympatholytic effects Obsolete; was used for hypertension Oral, parenteral Block of all autonomic effects Antinicotinic ganglion blockers Hexamethonium Selective block of NN receptors Trimethaphan: IV only, short-acting; was used for hypertensive emergencies and controlled hypotension Mecamylamine: oral, enters CNS; investigational use for smoking cessation Antinicotinic neuromuscular blockers See Chapter 27 AChE regenerator Pralidoxime Chemical antagonist of organophosphates Organophosphate poisoning Parenteral Muscle weakness C H A P T E R 9 Sympathomimetics their spectrum of action (α-, β-, or dopamine-receptor affinity) or mode of action (direct or indirect). The sympathomimetics constitute a very important group of drugs used for cardiovascular, respiratory, and other conditions. They are readily divided into subgroups on the basis of Sympathomimetic agonists Direct-acting Alpha agonists Indirect-acting Beta agonists Releasers Reuptake inhibitors (amphetamine) (cocaine) Beta2-selective (albuterol) Beta1-selective (dobutamine) Nonselective (isoproterenol) Alpha2-selective (clonidine) Alpha1-selective (phenylephrine) Nonselective (norepinephrine) CLASSIFICATION A. Spectrum of Action Adrenoceptors are classified as α, β, or dopamine receptors; these groups are further subdivided into subgroups. The distribution of these receptors is set forth in Table 9–1. Epinephrine may be considered a single prototype agonist with effects at all α- and β-receptor types. Alternatively, separate prototypes, phenylephrine (an α agonist) and isoproterenol (β), may be defined. The just-mentioned drugs have relatively little effect on dopamine receptors, but dopamine itself is a potent dopamine-receptor 76 agonist and, when given as a drug, can also activate β receptors (intermediate doses) and α receptors (larger doses). B. Mode of Action Sympathomimetic agonists may directly activate their adrenoceptors, or they may act indirectly to increase the concentration of endogenous catecholamine transmitter in the synapse. Amphetamine derivatives and tyramine cause the release of stored catecholamines; they are therefore mainly indirect in their mode of action. Cocaine and the tricyclic antidepressants exhibit another form of indirect action; these drugs inhibit reuptake of CHAPTER 9 Sympathomimetics 77 High-Yield Terms to Learn Anorexiant A drug that decreases appetite (causes anorexia) Catecholamine A dihydroxyphenylethylamine derivative (eg, norepinephrine, epinephrine), a relatively polar molecule that is readily metabolized by catechol-O-methyltransferase Decongestant An α-agonist drug that reduces conjunctival, nasal, or oropharyngeal mucosal vasodilation by constricting blood vessels in the submucosal tissue Mydriatic A drug that causes dilation of the pupil; opposite of miotic Phenylisopropylamine A synthetic sympathomimetic with isopropylamine in its structure (eg, amphetamine, ephedrine). Unlike catecholamines, phenylisopropylamines usually have oral activity, a long half-life, CNS activity, and cause release of stored catecholamines Selective ` or a adrenoceptor agonist Drugs that have relatively greater effects on α or β adrenoceptors; none are absolutely selective or specific Sympathomimetic A drug that mimics stimulation of the sympathetic autonomic nervous system Reuptake inhibitor An indirect-acting drug that increases the activity of transmitters in the synapse by inhibiting their reuptake into the presynaptic nerve ending. May act selectively on noradrenergic, serotonergic, or both types of nerve endings TABLE 9–1 Types of adrenoceptors, some of the peripheral tissues in which they are found, and their major effects. Type Tissue Actions Alpha1 Most vascular smooth muscle Pupillary dilator muscle Pilomotor smooth muscle Bladder trigone, prostatic smooth muscle Liver (in some species, eg, rat) Contracts (↑ vascular resistance) Contracts (mydriasis) Contracts (erects hair) Contraction Stimulates glycogenolysis Adrenergic and cholinergic nerve terminals Platelets Some vascular smooth muscle Fat cells Inhibits transmitter release Stimulates aggregation Contracts Inhibits lipolysis Pancreatic β (B) cells Inhibits insulin release Beta1 Heart Juxtaglomerular cells of kidney Stimulates rate and force Stimulates renin release Beta2 Airways, uterine, and vascular smooth muscle Liver (human) Relaxes Stimulates glycogenolysis Alpha2 Pancreatic β (B) cells Stimulates insulin release Somatic motor neuron terminals (voluntary muscle) Heart Causes tremor Stimulates rate and force Beta3 Fat cells Stimulates lipolysis Dopamine1 (D1) Renal and other splanchnic blood vessels Dilates (↓ resistance) Dopamine2 (D2) Nerve terminals Inhibits adenylyl cyclase 78 PART II Autonomic Drugs catecholamines by the norepinephrine transporter (NET) and the dopamine transporter (DAT) in nerve terminals (see Figure 6–2) and thus increase the synaptic activity of released transmitter. Blockade of metabolism (ie, block of catechol-O-methyltransferase [COMT] and monoamine oxidase [MAO]) has little direct effect on autonomic activity, but MAO inhibition increases the stores of catecholamines and related molecules in adrenergic synaptic vesicles and thus may potentiate the action of indirect-acting sympathomimetics (eg, amphetamines) that cause the release of stored transmitter. CHEMISTRY & PHARMACOKINETICS The endogenous adrenoceptor agonists (epinephrine, norepinephrine, and dopamine) are catecholamines and are rapidly metabolized by COMT and MAO as described in Chapter 6. If used as drugs, these adrenoceptor agonists are relatively inactive by the oral route and must be given parenterally. When released from nerve endings, they are subsequently taken up (by NET or DAT) into nerve endings and into perisynaptic cells; this uptake may also occur with exogenous norepinephrine, epinephrine, and dopamine given as drugs. These agonists have a short duration of action. When given parenterally, they do not enter the central nervous system (CNS) in significant amounts. Isoproterenol, a synthetic catecholamine, is similar to the endogenous transmitters but is not readily taken up into nerve endings. Phenylisopropylamines, for example, amphetamines, are resistant to MAO; most of them are not catecholamines and are therefore also resistant to COMT. Phenylisopropylamines are orally active; they enter the CNS, and their effects last much longer than do those of catecholamines. Tyramine, which is not a phenylisopropylamine, is rapidly metabolized by MAO except in patients who are taking an MAO inhibitor drug. MAO inhibitors are sometimes used in the treatment of depression (see Chapter 30). MECHANISMS OF ACTION A. Alpha-Receptor Effects Alpha-receptor effects are mediated primarily by the trimeric coupling protein Gq. When Gq is activated, the alpha moiety of this protein activates the enzyme phospholipase C, resulting in the release of inositol-1,4,5-trisphosphate (IP3) and diacylglycerol (DAG) from membrane lipids. Calcium is subsequently released from stores in smooth muscle cells by IP3, and enzymes are activated by DAG. Direct gating of calcium channels may also play a role in increasing intracellular calcium concentration. Alpha2-receptor activation results in inhibition of adenylyl cyclase via the coupling protein Gi. B. Beta-Receptor Effects All β receptors (β1, β2, and β3) stimulate adenylyl cyclase via the coupling protein Gs, which leads to an increase in cyclic adenosine monophosphate (cAMP) concentration in the cell. Some evidence suggests that β receptors may exert G-protein-independent effects after binding β-arrestin. C. Dopamine-Receptor Effects Dopamine D1 receptors activate adenylyl cyclase via Gs and increase cAMP in neurons and vascular smooth muscle. Dopamine D2 receptors are more important in the brain but probably also play a significant role as presynaptic receptors on peripheral nerves. These receptors reduce the synthesis of cAMP via Gi. ORGAN SYSTEM EFFECTS A. Central Nervous System Catecholamines do not enter the CNS readily. Sympathomimetics that do enter the CNS (eg, amphetamines, cocaine) have a spectrum of stimulant effects, beginning with mild alerting or reduction of fatigue and progressing to anorexia, euphoria, and insomnia. These CNS effects reflect the release and amplification of dopamine's action in the ventral tegmental area and other CNS nuclei (see Chapter 32). Repeated dosing of amphetamines results in the rapid development of tolerance and dependence. Very high doses of amphetamines lead to marked anxiety or aggressiveness, paranoia, and, less commonly, seizures. Overdoses of cocaine very commonly result in seizures. Some α2-selective agonists (eg, clonidine) cause vasoconstriction when administered intravenously or locally into the conjunctival sac. However, when given chronically, they are readily taken up into the CNS and reduce sympathetic outflow, probably by activating α2 adrenoceptors on presynaptic nerve endings. As a result, they can lower blood pressure (see also Chapter 11). B. Eye The smooth muscle of the pupillary dilator responds to topical phenylephrine and similar α agonists with contraction and mydriasis. Accommodation is not significantly affected. Outflow of aqueous humor may be facilitated by nonselective α agonists, with a subsequent reduction of intraocular pressure. This probably occurs via the uveoscleral drainage system. Alpha2-selective agonists also reduce intraocular pressure, apparently by reducing synthesis of aqueous humor. C. Bronchi The smooth muscle of the bronchi relaxes markedly in response to β2 agonists, eg, isoproterenol and albuterol. These agents are the most efficacious and reliable drugs for reversing bronchospasm. D. Gastrointestinal Tract The gastrointestinal tract is well endowed with both α and β receptors, located both on smooth muscle and on neurons of the enteric nervous system. Activation of either α or β receptors leads to relaxation of the smooth muscle. Alpha2 agonists may also decrease salt and water secretion into the intestine. E. Genitourinary Tract The genitourinary tract contains α receptors in the bladder trigone and sphincter area; these receptors mediate contraction of the sphincter. In men, α1 receptors mediate prostatic smooth muscle contraction. Sympathomimetics are sometimes used to increase sphincter tone. Beta2 agonists may cause significant CHAPTER 9 Sympathomimetics 79 TABLE 9–2 Effects of prototypical sympathomimetics on vascular resistance, blood pressure, and heart rate. Effect on Drug Skin, Splanchnic Vascular Resistance Skeletal Muscle Vascular Resistance Renal Vascular Resistance Mean Blood Pressure Heart Rate Phenylephrine ↑↑↑ ↑ ↑ ↑↑ ↓a Isoproterenol — ↓↓ — ↓↓ ↑↑ Norepinephrine ↑↑↑↑ ↑↑ ↑ ↑↑↑ ↓a, ↑b a Compensatory reflex response. b Direct response (if reflexes blocked). uterine relaxation in pregnant women near term, but the doses required also cause significant tachycardia. F. Vascular System Different vascular beds respond differently, depending on their dominant receptor type (Tables 9–1 and 9–2). 1. Alpha1 agonists—Alpha1 agonists (eg, phenylephrine) contract vascular smooth muscle, especially in skin and splanchnic blood vessels, and increase peripheral vascular resistance and venous pressure. Because these drugs increase blood pressure, they often evoke a compensatory reflex bradycardia. 2. Alpha2 agonists—Alpha2 agonists (eg, clonidine) cause vasoconstriction when administered intravenously or topically (eg, as a nasal spray), but when given orally they accumulate in the CNS and reduce sympathetic outflow and blood pressure as described in Chapter 11. 3. Beta agonists—Beta2 agonists (eg, albuterol, metaproterenol, terbutaline) and nonselective β agonists (eg, isoproterenol) cause significant reduction in arteriolar tone in the skeletal muscle vascular bed and can reduce peripheral vascular resistance and arterial blood pressure. Beta1 agonists have relatively little effect on vessels. 4. Dopamine—Dopamine causes vasodilation in the splanchnic and renal vascular beds by activating D1 receptors. This effect can be useful in the treatment of renal failure associated with shock. At higher doses, dopamine activates β receptors in the heart and elsewhere; at still higher doses, α receptors are activated. G. Heart The heart is well supplied with β1 and β2 receptors. The β1 receptors predominate in some parts of the heart; both β1 and β2 receptors mediate increased rate of cardiac pacemakers (normal and abnormal), increased atrioventricular node conduction velocity, and increased cardiac force. H. Net Cardiovascular Actions Sympathomimetics with both α and β1 effects (eg, norepinephrine) may cause a reflex increase in vagal outflow because they increase blood pressure and evoke the baroreceptor reflex. This reflex vagal effect may dominate any direct beta effects on the heart rate, so that a slow infusion of norepinephrine typically causes increased blood pressure and bradycardia (Figure 9–1; Table 9–2). If the reflex is blocked (eg, by a ganglion blocker or antimuscarinic drug), norepinephrine will cause a direct β1mediated tachycardia. A pure α agonist (eg, phenylephrine) routinely slows heart rate via the baroreceptor reflex, whereas a pure β agonist (eg, isoproterenol) almost always increases heart rate. Diastolic blood pressure is affected mainly by peripheral vascular resistance and the heart rate. (The heart rate is important because the diastolic interval determines the outflow of blood from the arterial compartment.) The adrenoceptors with the greatest effects on vascular resistance are α and β2 receptors. The pulse pressure (the systolic minus the diastolic pressure) is determined mainly by the stroke volume (a function of force of cardiac contraction), which is influenced by β1 receptors. The systolic pressure is the sum of the diastolic and the pulse pressures and is therefore a function of both α and β effects. I. Metabolic and Hormonal Effects Beta1 agonists increase renin secretion. Beta2 agonists increase insulin secretion. They also increase glycogenolysis in the liver and the resulting hyperglycemia is countered by the increased insulin levels. Transport of glucose out of the liver is associated initially with hyperkalemia; transport into peripheral organs (especially skeletal muscle) is accompanied by movement of potassium into these cells, resulting in a later hypokalemia. All β agonists appear to stimulate lipolysis via the β3 receptor. SKILL KEEPER: BLOOD PRESSURE CONTROL MECHANISMS IN PHEOCHROMOCYTOMA (SEE CHAPTER 6) Patients with pheochromocytoma may have this tumor for several months or even years before symptoms or signs lead to a diagnosis. Predict the probable compensatory responses to a chronic increase in blood pressure caused by a tumor releasing large amounts of norepinephrine. The Skill Keeper Answer appears at the end of the chapter. 80 PART II Autonomic Drugs Heart rate (beats/min) Blood pressure (mm Hg) Norepinephrine Isoproterenol 150 Pulse pressure 100 Systolic 50 Mean Diastolic 100 50 Time FIGURE 9–1 Typical effects of norepinephrine and isoproterenol on blood pressure and heart rate. Note that the pulse pressure is only slightly increased by norepinephrine but is markedly increased by isoproterenol (see text). The reduction in heart rate caused by norepinephrine is the result of baroreceptor reflex activation of vagal outflow to the heart. CLINICAL USES Pharmacokinetic characteristics and clinical applications of selected sympathomimetics are shown in the Drug Summary Table. A. Anaphylaxis Epinephrine is the drug of choice for the immediate treatment of anaphylactic shock (hypotension, bronchospasm, angioedema) because it is an effective physiologic antagonist of many of the mediators of anaphylaxis. Antihistamines and corticosteroids may also be used, but these agents are neither as efficacious as epinephrine nor as rapid acting. for glaucoma and include apraclonidine and brimonidine. As noted, the α2-selective agonists appear to reduce synthesis of aqueous humor. See Table 10–3 for a summary of drugs used in glaucoma. D. Bronchi The β agonists, especially the β2-selective agonists, are drugs of choice in the treatment of acute asthmatic bronchoconstriction. The short-acting β2-selective agonists (eg, albuterol, metaproterenol, terbutaline) are not recommended for prophylaxis, but they are safe and effective and may be lifesaving in the treatment of acute bronchospasm. Much longer-acting β2-selective agonists, salmeterol, formoterol, indacaterol, olodaterol, and vilanterol are used in combination with corticosteroids or antimuscarinic agents for prophylaxis in asthma or chronic obstructive pulmonary disease (COPD); they are not indicated for the treatment of acute symptoms (see Chapter 20). B. Central Nervous System The phenylisopropylamines such as amphetamine are widely used and abused for their CNS effects. Legitimate indications include narcolepsy and, with appropriate adjuncts, weight reduction. The anorexiant effect may be helpful in initiating weight loss but is insufficient to maintain the loss unless patients also receive intensive dietary and psychological counseling and support. Methylphenidate and other amphetamine analogs are heavily used in attention deficit hyperkinetic disorder (ADHD). The drugs are abused or misused for the purpose of deferring sleep and for their mood-elevating, euphoria-producing action. Cocaine is abused for its mood-elevating effect. These drugs have a high addiction liability (see Chapter 32). E. Cardiovascular Applications 1. Conditions in which an increase in blood flow is desired—In acute heart failure and some types of shock, an increase in cardiac output and blood flow to the tissues is needed. Beta1 agonists may be useful in this situation because they increase cardiac contractility and reduce (to some degree) afterload by decreasing the impedance to ventricular ejection through a small β2 effect. Norepinephrine, in contrast to earlier recommendations, is an effective agent in septic and cardiogenic shock when used properly. Dobutamine and dopamine are also used. Unfortunately, the arrhythmogenic effects of these drugs may be dose-limiting. C. Eye The α agonists, especially phenylephrine and tetrahydrozoline, are often used to reduce the conjunctival itching and congestion caused by irritation or allergy. Phenylephrine is also an effective mydriatic. These drugs do not cause cycloplegia. Newer α2 agonists are in current use 2. Conditions in which a decrease in blood flow or increase in blood pressure is desired—Alpha1 agonists are useful in situations in which vasoconstriction is appropriate. These include local hemostatic (epinephrine) and decongestant effects (phenylephrine) as well as shock (norepinephrine, phenylephrine), CHAPTER 9 Sympathomimetics in which temporary maintenance of blood pressure may help maintain perfusion of the brain, heart, and kidneys. High doses of vasoconstrictors may worsen shock due to septicemia or myocardial infarction because cardiac reserve is marginal. Alpha agonists are often mixed with local anesthetics to reduce the loss of anesthetic from the area of injection into the circulation. Chronic orthostatic hypotension due to inadequate sympathetic tone can be treated with oral ephedrine or a newer orally active α1 agonist, midodrine. 3. Conditions in which acute cardiac stimulation is desired—Epinephrine has been used in cardiac arrest by intravenous and direct intracardiac injection. Isoproterenol has been used for atrioventricular (AV) block. F. Genitourinary Tract Beta2 agonists (ritodrine, terbutaline) are sometimes used to suppress premature labor, but the cardiac stimulant effect may be hazardous to both mother and fetus. Nonsteroidal anti-inflammatory drugs, calcium channel blockers, and magnesium are also used for this indication. Long-acting oral sympathomimetics such as ephedrine are sometimes used to improve urinary continence in the elderly and in children with enuresis. This action is mediated by α receptors in the trigone of the bladder and, in men, the smooth muscle of the prostate. TOXICITY Because of their limited penetration into the brain, catecholamines have little CNS toxicity when given systemically. In the periphery, their adverse effects are extensions of their pharmacologic alpha or beta actions: excessive vasoconstriction, cardiac arrhythmias, myocardial infarction, hemorrhagic stroke, and pulmonary edema or hemorrhage. The phenylisopropylamines may produce mild to severe CNS toxicity, depending on dosage. In moderate doses, they may induce nervousness, anorexia, and insomnia; in higher doses, they may cause anxiety, aggressiveness, or paranoid behavior. Convulsions may occur. Peripherally acting agents have toxicities that are predictable on the basis of the receptors they activate. Thus, α1 agonists cause hypertension, and β1 agonists cause sinus tachycardia and serious arrhythmias. Beta2 agonists cause skeletal muscle tremor. It is important to note that none of these drugs is perfectly selective; at high doses, β1-selective agents have β2 actions and vice versa. Cocaine is of special importance as a drug of abuse: its major toxicities include cardiac arrhythmias or infarction and seizures. A fatal outcome is more common with acute cocaine overdose than with any other sympathomimetic. 81 QUESTIONS Questions 1 and 2. A 7-year-old boy with a previous history of bee sting allergy is brought to the emergency department after being stung by 3 bees. 1. Which of the following are probable signs of the anaphylactic reaction to bee stings? (A) Bronchodilation, tachycardia, hypertension, vomiting, diarrhea (B) Bronchospasm, tachycardia, hypotension, laryngeal edema (C) Diarrhea, bradycardia, vomiting (D) Laryngeal edema, bradycardia, hypotension, diarrhea (E) Miosis, tachycardia, vomiting, diarrhea 2. If this child has signs of anaphylaxis, what is the treatment of choice? (A) Diphenhydramine (an antihistamine) (B) Ephedrine (C) Epinephrine (D) Isoproterenol (E) Methylprednisolone (a corticosteroid) 3. A 65-year-old woman with impaired renal function and a necrotic ulcer in the sole of her right foot is admitted to the ward from the emergency department. She has long-standing type 2 diabetes mellitus and you wish to examine her retinas for possible vascular changes. Which of the following drugs is a good choice when pupillary dilation—but not cycloplegia—is desired? (A) Isoproterenol (B) Norepinephrine (C) Phenylephrine (D) Pilocarpine (E) Tropicamide 4. A 60-year-old immigrant from Latin America was told she had hypertension and should be taking antihypertensive medication. She decides to take an herbal medication from an online “holistic pharmacy.” One week after starting the medication, she is found unconscious in her apartment. In the emergency department, her blood pressure is 50/0 mm Hg and heart rate is 40 bpm. Respirations are 20/min; pupils are slightly constricted. Bowel sounds are present. Which of the following would be the most effective cardiovascular stimulant? (A) Amphetamine (B) Clonidine (C) Isoproterenol (D) Norepinephrine (E) Tyramine 5. A group of volunteers are involved in a phase 1 clinical trial of a new autonomic drug. When administered by intravenous bolus, the blood pressure increases. When given orally for 1 week, the blood pressure decreases. Which of the following standard agents does the new drug most resemble? (A) Atropine (B) Clonidine (C) Phentolamine (an α blocker) (D) Phenylephrine (E) Propranolol (a β blocker) 82 PART II Autonomic Drugs 6. Your 30-year-old patient has moderately severe new onset asthma, and you prescribe a highly selective β2 agonist inhaler to be used when needed. In considering the possible drug effects in this patient, you would note that β2 stimulants frequently cause (A) Direct stimulation of renin release (B) Hypoglycemia (C) Itching due to increased cGMP (cyclic guanine monophosphate) in mast cells (D) Skeletal muscle tremor (E) Vasodilation in the skin 7. Mr Green, a 54-year-old banker, had a cardiac transplant 6 months ago. His current blood pressure is 120/70 mm Hg and heart rate is 100 bpm. Which of the following drugs would have the least effect on Mr Green's heart rate? (A) Albuterol (B) Epinephrine (C) Isoproterenol (D) Norepinephrine (E) Phenylephrine Blood pressure (mm Hg) Questions 8 and 9. Several new drugs with autonomic actions were studied in preclinical trials in animals. Autonomic drugs X and Y were given in moderate doses as intravenous boluses. The systolic and diastolic blood pressures changed as shown in the diagram below. 200 180 160 140 120 100 80 60 40 20 y x Systolic Systolic Diastolic Diastolic 8. Which of the following drugs most resembles drug X? (A) Atropine (B) Bethanechol (C) Epinephrine (D) Isoproterenol (E) Phenylephrine 9. Which of the following most resembles drug Y? (A) Atropine (B) Bethanechol (C) Epinephrine (D) Isoproterenol (E) Phenylephrine 10. A new drug was given by subcutaneous injection to 25 normal subjects in a phase 1 clinical trial. The cardiovascular effects are summarized in the table below. Variable Control Peak Drug Effect Systolic BP (mm Hg) 116 156 Diastolic BP (mm Hg) 76 96 Cardiac output (L/min) 5.0 7.7 Heart rate (beats/min) 71.2 94.3 Which of the following drugs does the new experimental agent most resemble? (A) Atropine (B) Epinephrine (C) Isoproterenol (D) Phenylephrine (E) Physostigmine ANSWERS 1. Anaphylaxis is caused by the release of several mediators. Leukotrienes and certain proteins are the most important of these. They cause bronchospasm and laryngeal edema and marked vasodilation with severe hypotension. Tachycardia is a common reflex response to the hypotension. Gastrointestinal disturbance is not as common nor as dangerous. The answer is B. 2. The treatment of anaphylaxis requires a powerful physiologic antagonist with the ability to cause rapid bronchodilation (β2 effect), and vasoconstriction (α effect). Epinephrine is the most effective agent with these properties. Antihistamines and corticosteroids are sometimes used as supplementary agents, but the prompt parenteral use of epinephrine is mandatory. The answer is C. 3. Antimuscarinics (tropicamide) are mydriatic and cycloplegic; α-sympathomimetic agonists are only mydriatic in the eye. Isoproterenol has negligible effects on the eye. Norepinephrine penetrates the conjunctiva poorly and would produce intense vasoconstriction. Pilocarpine causes miosis. Phenylephrine is well-absorbed from the conjunctival sac and produces useful mydriasis for 10–30 minutes. The answer is C. 4. “Herbal” medications often contain potent synthetic drugs in addition to (or instead of) the advertised constituents. This patient shows signs of sympathetic autonomic failure: hypotension, inappropriate bradycardia, constricted pupils. These signs are compatible with a large overdose of a drug that causes marked depletion of stored catecholamine transmitter such as reserpine, an obsolete but inexpensive antihypertensive agent. The indirect-acting agents (amphetamines and tyramine) act through catecholamines in (or released from) the nerve terminal and would therefore be ineffective in this patient. Clonidine acts primarily on presynaptic nerve endings although it can activate α2 receptors located elsewhere. Isoproterenol would stimulate the heart but has no α-agonist action and might exacerbate the hypotension. Norepinephrine has the necessary combination of direct action and a spectrum that includes α1, α2, and β1 effects. The answer is D. CHAPTER 9 Sympathomimetics 5. The dual blood pressure effects of the drug suggest that initially it is causing a direct α-agonist vasoconstrictor effect, but when given for a week, it is accumulating in a blood pressure-controlling center, eg, the CNS, and reducing sympathetic outflow. The answer is B. 6. Tremor is a common β2 effect. Blood vessels in the skin have almost exclusively α (vasoconstrictor) receptors. Stimulation of renin release is a β1 effect. Beta2 agonists cause hyperglycemia and have little effect on cGMP. The answer is D. 7. Heart transplantation involves cutting of the autonomic nerves to the heart. As a result, autonomic nerve endings degenerate, and cardiac transmitter stores are absent for 2 years or longer after surgery. Therefore, indirect-acting sympathomimetics are ineffective in changing heart rate. All the drugs listed are direct-acting, and all but phenylephrine have significant effects on β receptors. Phenylephrine usually causes reflex bradycardia, which requires intact vagal innervation. The answer is E. (Note that denervation may result in upregulation of both β1 and β2 receptors so that direct-acting β agonists may have a greater than normal effect.) 8. The drug X dose caused a decrease in diastolic blood pressure and little change in systolic pressure. Thus, there was a large increase in pulse pressure. The decrease in diastolic pressure suggests that the drug decreased vascular resistance, that is, it must have significant muscarinic or β-agonist effects. The fact that it also markedly increased pulse pressure suggests that it strongly increased stroke volume, a β-agonist effect. The drug with these beta effects is isoproterenol (Figure 9–1). The answer is D. 9. Drug Y caused a marked increase in diastolic pressure, suggesting strong α vasoconstrictor effects. It caused little or no increase in pulse pressure, suggesting negligible β-agonist action. [An increase in stroke volume may result from 83 increased venous return (an α-agonist effect) and stroke volume.] The drug that best matches this description is phenylephrine. The answer is E. 10. The investigational agent caused a marked increase in systolic and diastolic pressures and a moderate increase in pulse pressure (from 40 to 60 mm Hg). These changes suggest a strong alpha effect on vessels and an increase in stroke volume, a β-agonist action in the heart. The heart rate increased significantly, reflecting a β response. Note that the stroke volume also increased (cardiac output divided by heart rate—from 70.2 to 81.7 mL). The drug behaves most like a mixed α and β agonist. The answer is B. SKILL KEEPER ANSWER: BLOOD PRESSURE CONTROL MECHANISMS IN PHEOCHROMOCYTOMA (SEE CHAPTER 6) Because the control mechanisms that attempt to maintain blood pressure constant are intact in patients with pheochromocytoma (they are not intact, they are reset in patients with ordinary “essential” hypertension), a number of compensatory changes are observed in pheochromocytoma patients (see Figure 6–4). These include reduced renin, angiotensin, and aldosterone levels in the blood. Reduced aldosterone causes more salt and water to be excreted by the kidney, reducing blood volume. Since the red cell mass is not affected, hematocrit is often increased. If the tumor releases only norepinephrine, a compensatory bradycardia may also be present, but most patients release enough epinephrine to maintain heart rate at a normal or even increased level. CHECKLIST When you complete this chapter, you should be able to: ❑ Name a typical nonselective α agonist, a selective α2 agonist, a nonselective β agonist, a selective β1 agonist, selective β2 agonists, an α1, α2, β1 agonist, and an α1, α2, β1, β2 agonist. ❑ List tissues that contain significant numbers of α1 or α2 receptors. ❑ List tissues that contain significant numbers of β1 or β2 receptors. ❑ Describe the major organ system effects of a pure α agonist, a pure β agonist, and a mixed α and β agonist ❑ Describe a clinical situation in which the effects of an indirect sympathomimetic would differ from those of a direct agonist. ❑ List the major clinical applications of the adrenoceptor agonists. 84 PART II Autonomic Drugs DRUG SUMMARY TABLE: Sympathomimetics Subclass Mechanism of Action Clinical Applications Pharmacokinetics Toxicities, Interactions Direct-acting catecholamines Epinephrine α1, α2, β1, β2, β3 agonist Anaphylaxis • hemostatic • cardiac arrest Parenteral and topical only • does not enter CNS • Duration: short Hypertension, arrhythmia, stroke, myocardial infarction, pulmonary edema Norepinephrine α1, α2, β1, β3 agonist Shock Like epinephrine • IV only Vasospasm, tissue necrosis, excessive blood pressure increase, arrhythmias, infarction Dopamine D1, α1, α2, β1, β3, agonist Shock, especially with renal shutdown • sometimes used in heart failure Like epinephrine • IV only Cardiovascular disturbance, arrhythmias Isoproterenol: β1, β2, β3 agonist; primary use is by nebulizer (in acute asthma) and IV (in AV block) Dobutamine: β1 agonist; primary use is in acute heart failure to increase cardiac output Noncatecholamines Phenylephrine α1, α2 agonist Decongestant, mydriatic, neurogenic hypotension Oral, topical, and parenteral • Duration: 15–60 min Hypertension, stroke, myocardial infarction Prompt onset for acute bronchospasm Inhalant via aerosol canister • Duration: 2–6 h Tachycardia, tremor Noncatecholamine β2-selective Albuterol, metaproterenol, terbutaline β2 agonist Salmeterol, formoterol, indacaterol, vilanterol, olodaterol: β2 agonists; slow onset, long action. Not useful in acute bronchospasm, used only with corticosteroids for prophylaxis of asthma or with antimuscarinics for COPD Indirect-acting phenylisopropylamines Amphetamine, methamphetamine Displaces stored catecholamines from nerve endings Anorexiant, ADHD, narcolepsy Oral and parenteral • Duration: ≥4–6 h High addiction liability. Paranoia, aggression; insomnia; hypertension Ephedrine: displacer like amphetamine plus some direct activity; oral activity; duration 4–6 h. Sometimes used for narcolepsy, idiopathic postural hypotension, enuresis. Lower addiction liability than amphetamines Cocaine Cocaine Blocks norepinephrine reuptake (NET) and dopamine reuptake (DAT) Local anesthetic with intrinsic hemostatic action Parenteral only (topical nasal, IV, local injection) Duration: 2 h Very high addiction liability. Hypertension, arrhythmias, seizures Displaces stored catecholamines No clinical use but found in fermented foods Normally high first-pass effect, but in patients taking MAO inhibitors it is absorbed Hypertension, arrhythmias, stroke, myocardial infarction Tyramine Tyramine ADHD, attention deficit hyperactivity disorder; COPD, chronic obstructive pulmonary disease; CNS, central nervous system; DAT, dopamine transporter; MAO, monoamine oxidase; NET, norepinephrine transporter. C A P T E R 10 Adrenoceptor Blockers Alpha- and beta-adrenoceptor-blocking agents are divided into primary subgroups on the basis of their receptor selectivity. All of these agents are pharmacologic antagonists or partial agonists and most are reversible and competitive in action. Because H α and β blockers differ markedly in their effects and clinical applications, these drugs are considered separately in the following discussion. Adrenoceptor antagonists Alpha blockers Beta blockers Alpha2-selective (yohimbine) Alpha1-selective (prazosin) Nonselective Irreversible (phenoxybenzamine) Beta2-selective (butoxamine) Beta1-selective (atenolol) Nonselective (propranolol) Reversible (phentolamine) ALPHA-BLOCKING DRUGS A. Classification Subdivisions of the α blockers are based on selective affinity for α1 versus α2 receptors or a lack thereof. Other features used to classify the α-blocking drugs are their reversibility and duration of action. Irreversible, long-acting—Phenoxybenzamine is the prototypical long-acting α blocker; it differs from other adrenoceptor blockers in being irreversible in action. It is slightly α1-selective. Reversible, shorter-acting—Phentolamine is a competitive, reversible blocking agent that does not distinguish between α1 and α2 receptors. Alpha1-selective—Prazosin is a highly selective, reversible pharmacologic α1 blocker. Doxazosin, terazosin, and tamsulosin are similar drugs. The advantage of α1 selectivity is discussed in the following text. Alpha2-selective—Yohimbine and rauwolscine are α2-selective competitive pharmacologic antagonists. They are used primarily in research applications. B. Pharmacokinetics Alpha-blocking drugs are all active by the oral as well as the parenteral route, although phentolamine is rarely given orally. Phenoxybenzamine has a short elimination half-life but a long duration of action—about 48 h—because it binds covalently to its receptor. Phentolamine has a duration of action of 2–4 h when used orally and 20–40 min when given parenterally. Prazosin and the other α1-selective blockers act for 8–24 h. C. Mechanism of Action Phenoxybenzamine binds covalently to the α receptor, thereby producing an irreversible (insurmountable) blockade. The other α-blocking agents are competitive antagonists, and their effects 85 86 PART II Autonomic Drugs High-Yield Terms to Learn Competitive blocker A surmountable antagonist (eg, phentolamine); one that can be overcome by increasing the dose of agonist Epinephrine reversal Conversion of the pressor response to epinephrine (typical of large doses) to a blood pressure– lowering effect; caused by α blockers, which unmask the β2 vasodilating effects of epinephrine Intrinsic sympathomimetic activity (ISA) Partial agonist action by adrenoceptor blockers; typical of several β blockers (eg, pindolol, acebutolol) Irreversible blocker A nonsurmountable inhibitor, usually because of covalent bond formation (eg, phenoxybenzamine) Membrane-stabilizing activity (MSA) Local anesthetic action; typical of several β blockers (eg, propranolol) Orthostatic hypotension Hypotension that is most marked in the upright position; caused by venous pooling (typical of α blockade) or inadequate blood volume (caused by blood loss or excessive diuresis) Partial agonist A drug (eg, pindolol) that produces a smaller maximal effect than a full agonist and therefore can inhibit the effect of a full agonist Pheochromocytoma A tumor consisting of cells that release varying amounts of norepinephrine and epinephrine into the circulation can be surmounted by increased concentrations of agonist. This difference may be important in the treatment of pheochromocytoma because a massive release of catecholamines from the tumor may overcome a reversible blockade. D. Effects 1. Nonselective blockers—These agents cause a predictable blockade of α-mediated responses to sympathetic nervous system discharge and exogenous sympathomimetics (ie, the α responses listed in Table 9–1). The most important effects of nonselective α blockers are those on the cardiovascular system: a reduction in vascular tone with a reduction of both arterial and venous pressures. There are no significant direct cardiac effects. However, the nonselective α blockers do cause baroreceptor reflex-mediated tachycardia as a result of the drop in mean arterial pressure (see Figure 6–4). This tachycardia may be exaggerated because the α2 receptors on adrenergic nerve terminals in the heart, which normally reduce the net release of norepinephrine, are also blocked (see Figure 6–3). Epinephrine reversal (Figure 10–1) is a predictable effect in a patient who has received an α blocker. The term refers to a reversal of the blood pressure effect of large doses of epinephrine, from a pressor response (mediated by α receptors) to a depressor response (mediated by β2 receptors). The effect is not observed with phenylephrine or norepinephrine because these drugs lack sufficient β2 effects. Epinephrine reversal, manifested as orthostatic hypotension, is occasionally seen as an unexpected (but predictable) effect of drugs for which α blockade is an adverse effect (eg, some phenothiazine antipsychotic agents, antihistamines). 2. Selective α blockers—Because prazosin and its analogs block vascular α1 receptors much more effectively than the α2modulatory receptors associated with cardiac sympathetic nerve endings, these drugs reduce blood pressure with much less reflex tachycardia than the nonselective α blockers. These drugs also have useful relaxing effects on smooth muscle in the prostate. E. Clinical Uses 1. Nonselective α blockers—Nonselective α blockers have limited clinical applications. The best-documented application is in the presurgical management of pheochromocytoma. Such patients may have severe hypertension and reduced blood volume, which should be corrected before subjecting the patient to the stress of surgery. Phenoxybenzamine is usually used during this preparatory phase; phentolamine is sometimes used during surgery. Phenoxybenzamine also has serotonin receptor-blocking effects, which justify its occasional use in carcinoid tumor, as well as H1 antihistaminic effects, which lead to its use in mastocytosis. Accidental local infiltration of potent α agonists such as norepinephrine may lead to severe tissue ischemia and necrosis if not promptly reversed; infiltration of the ischemic area with phentolamine is sometimes used to prevent tissue damage. Overdose with drugs of abuse such as amphetamine, cocaine, or phenylpropanolamine may lead to severe hypertension because of their indirect sympathomimetic actions. This hypertension usually responds well to α blockers. Sudden cessation of clonidine therapy leads to rebound hypertension (Chapter 11); this phenomenon is often treated with phentolamine. Raynaud’s phenomenon sometimes responds to α blockers, but their efficacy in this condition is not well documented. Phentolamine or yohimbine has been used by direct injection to cause penile erection in men with erectile dysfunction, but phosphodiesterase inhibitors are more popular (see Chapter 12). 2. Selective α blockers—Prazosin, doxazosin, and terazosin are used in hypertension (Chapter 11). These α1 blockers, as well as tamsulosin and silodosin are also used to reduce urinary hesitancy and prevent urinary retention in men with benign prostatic hyperplasia. CHAPTER 10 Adrenoceptor Blockers Before alpha blockade After alpha blockade Epi (large dose) Blood pressure Epi (large dose) 87 Time Net pressor effect Phenylephrine Blood pressure Phenylephrine Net depressor effect Net pressor effect Suppression of pressor effect FIGURE 10–1 The effects of an α blocker, for example, phentolamine, on the blood pressure responses to epinephrine (epi) and phenylephrine. The epinephrine response exhibits reversal of the mean blood pressure change from a net increase (the α response) to a net decrease (the β2 response). The response to phenylephrine is suppressed but not reversed, because phenylephrine lacks β action. F. Toxicity The most important toxicities of the α blockers are simple extensions of their α-blocking effects. The main manifestations are orthostatic hypotension and, in the case of the nonselective agents, marked reflex tachycardia. Tachycardia is less common and less severe with α1-selective blockers. Phentolamine also has some non-alpha-mediated vasodilating effects. In patients with coronary disease, angina may be precipitated by the tachycardia. Oral administration of some of these drugs can cause nausea and vomiting. The α1-selective agents are associated with an exaggerated orthostatic hypotensive response to the first dose in some patients. Therefore, the first dose is usually small and taken just before going to bed. BETA-BLOCKING DRUGS A. Classification, Subgroups, and Mechanisms All of the β blockers used clinically are competitive pharmacologic antagonists. Propranolol is the prototype. Drugs in this group are usually classified into subgroups on the basis of β1 selectivity, partial agonist activity, local anesthetic action, and lipid-solubility (Table 10–1). 1. Receptor selectivity—Beta1-receptor selectivity (β1 block > β2 block) is a property of acebutolol, atenolol, esmolol, metoprolol, and several other β blockers. This property may be an advantage when treating patients with asthma because functioning β2 receptors are important in preventing bronchospasm in such patients. Nadolol, propranolol, and timolol are typical nonselective β blockers. Note that, except for β blockers that start with the letter “c,” blockers with names starting with letters “a” through “m” are β1 selective. Labetalol and carvedilol have combined α- and β-blocking actions. These drugs are optically active, and different isomers have α- or β-blocking action. Nebivolol has vasodilating action in addition to dose-dependent β1-selective antagonism. 2. Partial agonist activity—Partial agonist activity (“intrinsic sympathomimetic activity”) may be an advantage in treating patients with asthma because these drugs (eg, pindolol, acebutolol)—at least in theory—are less likely to cause bronchospasm. In contrast, full antagonists such as propranolol are more likely to cause severe bronchospasm in patients with airway disease. 3. Local anesthetic activity—Local anesthetic activity (“membrane-stabilizing activity”) is a disadvantage when β blockers are used topically in the eye because it decreases protective reflexes and increases the risk of corneal ulceration. Local anesthetic effects are absent from timolol and several other β blockers that are useful in glaucoma. 4. Pharmacokinetics—Most of the systemic agents have been developed for chronic oral use, but bioavailability and duration of action vary widely (Table 10–1). Esmolol is a short-acting ester β blocker that is used only parenterally. Nadolol is the longest-acting β blocker. Acebutolol, atenolol, and nadolol are less lipid-soluble than other β blockers and probably enter the central nervous system (CNS) to a lesser extent. 88 PART II Autonomic Drugs TABLE 10–1 Properties of several β-adrenoceptor-blocking drugs. Drug Selectivity Partial Agonist Activity Local Anesthetic Activity Lipid Solubility Elimination Half-Life Acebutolol β1 Yes Yes Low 3–4 h Atenolol β1 No No Low 6–9 h Carvedilol None No No Moderate 7–10 h Esmolol β1 No No Low 10 min; IV only Labetalola None Yes, β2 only Yes Low 5h Metoprolol β1 No Yes Moderate 3–4 h Nadolol None No No Low 14–24 h Nebivolol β1 at low doses No No Low 11–20 h Pindolol None Yes Yes Moderate 3–4 h Propranolol None No Yes High 3.5–6 h Timolol None No No Moderate 4–5 h a b a Also causes α-receptor blockade. b Also causes vasodilation by causing release of nitric oxide from vascular endothelium. Modified, with permission, from Katzung BG, editor: Basic & Clinical Pharmacology, 12th ed., McGraw-Hill, 2012: p. 159. SKILL KEEPER: PARTIAL AGONIST ACTION (SEE CHAPTER 2) Draw a concentration-response graph showing the effect of increasing concentrations of albuterol on airway diameter (as a percentage of maximum) in the presence of a large concentration of pindolol. On the same graph, draw the curves for the percentage of receptors bound to albuterol and to pindolol at each concentration. The Skill Keeper Answer appears at the end of the chapter. B. Effects and Clinical Uses Most of the organ-level effects of β blockers are predictable from blockade of the β-receptor–mediated effects of sympathetic discharge. The clinical applications of β blockade are remarkably broad (see the Drug Summary Table). The treatment of openangle glaucoma involves the use of several groups of autonomic drugs as well as other agents (Table 10–2). The cardiovascular applications of β blockers—especially in hypertension, angina, and arrhythmias—are extremely important. Treatment of chronic (not acute) heart failure has become an important application of β blockers. Several large clinical trials have shown that some, but not all, β blockers can reduce morbidity and mortality when used properly in heart failure (see Chapter 13). Labetalol, carvedilol, and metoprolol have documented benefits in this application. Pheochromocytoma is sometimes treated with combined α- and β-blocking agents (eg, labetalol), especially if the tumor is producing large amounts of epinephrine as well as norepinephrine. A novel and unexplained beneficial reduction in the size of infantile hemangiomas has been reported for propranolol. C. Toxicity Cardiovascular adverse effects, which are extensions of the β blockade, include bradycardia, atrioventricular blockade, and heart failure. Patients with airway disease may suffer severe asthma attacks. Beta blockers have been shown experimentally to reduce insulin secretion, but this does not appear to be a clinically important effect. However, premonitory symptoms of hypoglycemia from insulin overdosage (tachycardia, tremor, and anxiety) may be masked by β blockers, and mobilization of glucose from the liver and sequestration of K+ in skeletal muscle may be impaired. CNS adverse effects include sedation, fatigue, and sleep alterations. Atenolol, nadolol, and several other less lipid-soluble β blockers are claimed to have less marked CNS action because they do not enter the CNS as readily as other members of this group. Sexual dysfunction has been reported for most of the β blockers in some patients. CHAPTER 10 Adrenoceptor Blockers 89 TABLE 10–2 Drugs used in glaucoma. Group, Drugs Mechanism Method of Administration Decreased secretion of aqueous humor from the ciliary epithelium Topical drops Increased aqueous outflow Topical drops Ciliary muscle contraction, opening of trabecular meshwork, increased outflow Topical drops or gel, plastic film slowrelease insert Alpha agonists Nonselective: epinephrine Increased outflow via uveoscleral veins Topical drops (obsolete) Alpha2-selective agonists Apraclonidine, brimonidine Decreased aqueous secretion Topical drops Carbonic anhydrase inhibitors Acetazolamide, dorzolamide Decreased aqueous secretion due to lack of HCO3− Oral (acetazolamide) or topical (others) Osmotic agents Mannitol Removal of water from eye IV (for acute closed-angle glaucoma) Beta blockers Timolol, others Prostaglandins Latanoprost, others Cholinomimetics Pilocarpine, physostigmine Modified and reproduced, with permission, from Katzung BG, editor: Basic & Clinical Pharmacology, 12th ed. McGraw-Hill, 2012, p. 161. QUESTIONS 1. A patient is to receive epinephrine. She has previously received an adrenoceptor-blocking agent. Which of the following effects of epinephrine would be blocked by phentolamine but not by metoprolol? (A) Cardiac stimulation (B) Increase of cAMP (cyclic adenosine monophosphate) in fat (C) Mydriasis (D) Relaxation of bronchial smooth muscle (E) Relaxation of the uterus 2. Clinical studies have shown that adrenoceptor blockers have many useful effects in patients. However, a number of drug toxicities have been documented. Adverse effects that limit the use of adrenoceptor blockers include which one of the following? (A) Bronchoconstriction from α-blocking agents (B) Acute heart failure exacerbation from β blockers (C) Impaired blood sugar response with α blockers (D) Increased intraocular pressure with β blockers (E) Sleep disturbances from α-blocking drugs Questions 3–6. Four new synthetic drugs (designated W, X, Y, and Z) are to be studied for their cardiovascular effects. They are given to 4 anesthetized animals while the heart rate is recorded. The first animal has received no pretreatment (control); the second has received an effective dose of hexamethonium; the third has received an effective dose of atropine; and the fourth has received an effective dose of phenoxybenzamine. The net changes induced by W, X, Y, and Z in the animals are described in the following questions. 3. Drug W increased heart rate in the control animal, the atropine-pretreated animal, and the phenoxybenzaminepretreated animal. However, drug W had no effect on heart rate in the hexamethonium-pretreated animal. Drug W is probably a drug similar to (A) Acetylcholine (B) Edrophonium (C) Isoproterenol (D) Nicotine (E) Norepinephrine PART II Autonomic Drugs 4. Drug X had the effects shown in the table below. Heart Rate Response to Drug X Was No pretreatment ↓ Hexamethonium ↑ Atropine ↑ Phenoxybenzamine ↑ Drug X is probably a drug similar to (A) Acetylcholine (B) Albuterol (C) Edrophonium (D) Isoproterenol (E) Norepinephrine 5. Drug Y had the effects shown in the table below. In the Animal Receiving Heart Rate Response to Drug Y Was No pretreatment ↑ Hexamethonium ↑ Atropine ↑ Phenoxybenzamine ↑ 200 Percent change in heart rate 6. The results of the test of drug Z are shown in the graph. 0 Hexamethonium Atropine Z Z – 100 Drug Z is probably a drug similar to (A) Acetylcholine (B) Edrophonium (C) Isoproterenol (D) Norepinephrine (E) Pralidoxime 9. A 56-year-old man has hypertension and an enlarged prostate, which biopsy shows to be benign prostatic hyperplasia. He complains of urinary retention. Which of the following drugs would be the most appropriate initial therapy? (A) Albuterol (B) Atenolol (C) Metoprolol (D) Prazosin (E) Timolol 10. A new drug was administered to an anesthetized animal with the results shown here. A large dose of epinephrine (epi) was administered before and after the new agent for comparison. Drug Y is probably a drug similar to (A) Acetylcholine (B) Edrophonium (C) Isoproterenol (D) Norepinephrine (E) Prazosin No 100 pretreatment Z 8. Your 75-year-old patient with angina and glaucoma is to receive a β-blocking drug. Which of the following statements is most correct regarding β-blocking drugs? (A) Esmolol’s pharmacokinetics are compatible with chronic topical use (B) Metoprolol blocks β2 receptors selectively (C) Nadolol lacks β2-blocking action (D) Pindolol is a β antagonist with high membrane-stabilizing (local anesthetic) activity (E) Timolol lacks the local anesthetic effects of propranolol Phenoxybenzamine Z Blood pressure (mm Hg) In the Animal Receiving 7. When given to a patient, phentolamine blocks which one of the following? (A) Bradycardia induced by phenylephrine (B) Bronchodilation induced by epinephrine (C) Increased cardiac contractile force induced by norepinephrine (D) Miosis induced by acetylcholine (E) Vasodilation induced by isoproterenol New drug Epi Epi Epi Epi 100 0 Cardiac force 90 Which of the following agents does the new drug most closely resemble? (A) Atenolol (B) Atropine (C) Labetalol (D) Phenoxybenzamine (E) Propranolol CHAPTER 10 Adrenoceptor Blockers 1. Mydriasis caused by contraction of the pupillary dilator radial smooth muscle is mediated by α receptors. All the other effects listed are mediated by β receptors. The answer is C. 2. Although chronic heart failure is often treated with certain β blockers, acute heart failure can be precipitated by these drugs. Choices A, C, and E reverse the correct pairing of receptor subtype (α versus β) with effect. Choice D reverses the direction of change of intraocular pressure. The answer is B. 3. In developing a strategy for this type of question, consider first the actions of the known blocking drugs. Hexamethonium blocks reflexes as well as the direct action of nicotine. Atropine would block direct muscarinic effects of an unknown drug (if it had any) or reflex slowing of the heart mediated by the vagus. Phenoxybenzamine blocks only α-receptor-mediated processes. If the response produced in the nonpretreated animal is blocked or reversed by hexamethonium, it is probably a direct nicotinic effect or a reflex response to hypotension. In that case, consider all the receptors involved in mediating the reflex. Drug W causes tachycardia that is prevented by ganglion blockade. The only drug in the list of choices that causes hypotension and tachycardia that is not blocked by atropine is isoproterenol, and the tachycardia caused by isoproterenol is not blocked by ganglionic blockade. Thus, drug W must be nicotine or a drug similar to it. The answer is D. 4. Drug X causes slowing of the heart rate, but this is converted into tachycardia by hexamethonium and atropine, demonstrating that when it occurs, the bradycardia is caused by reflex vagal discharge. Phenoxybenzamine also reverses the bradycardia to tachycardia, suggesting that α receptors are needed to induce the reflex bradycardia and that X also has direct β-agonist actions. The choices that evoke a vagal reflex bradycardia (vasoconstrictors) but can also cause direct tachycardia (β agonists) are limited; the answer is E. 5. Drug Y causes tachycardia that is not significantly influenced by any of the blockers; therefore, drug Y must have a direct β-agonist effect on the heart. The answer is C. 6. Drug Z causes tachycardia that is converted to bradycardia by hexamethonium and blocked completely by atropine. This indicates that the tachycardia is a reflex evoked by muscarinic vasodilation. Drug Z causes bradycardia when the ganglia are blocked, indicating that it also has a direct muscarinic action on the heart. This is confirmed by the ability of atropine to block both the tachycardia and the bradycardia. The answer is A. 7. Phenylephrine, an α agonist, increases blood pressure and causes bradycardia through the baroreceptor reflex. Blockade of this drug’s α-mediated vasoconstrictor effect prevents the bradycardia. The answer is A. 8. Esmolol is a short-acting β blocker for parenteral use only. Nadolol is a nonselective β blocker, and metoprolol is a β1-selective blocker. Timolol is useful in glaucoma because it does not anesthetize the cornea. The answer is E. 9. An α blocker is appropriate therapy in a man with both hypertension and benign prostatic hyperplasia because both conditions involve contraction of smooth muscle containing α receptors. The answer is D. 10. The new drug blocks both the α-mediated effects (increased diastolic and mean arterial blood pressure) and β-mediated action (increased cardiac force). In addition, it does not cause epinephrine reversal. Therefore, the drug must have both α- and β-blocking effects. The answer is C. SKILL KEEPER ANSWER: PARTIAL AGONIST ACTION (SEE CHAPTER 2) Because pindolol is a partial agonist at β receptors, the concentration–response curve will show a bronchodilating effect at zero albuterol concentration. As albuterol concentration increases, the airway diameter also increases. The binding curves will show pindolol binding starting at 100% of receptors and going to zero as albuterol concentration increases, with albuterol binding starting at zero and going to 100%. Pindolol binding 100 Percent of maximum ANSWERS Total effect Albuterol binding 50 0 0 Very high Concentration of albuterol CHECKLIST When you complete this chapter, you should be able to: ❑ Describe and compare the effects of an α blocker on the blood pressure and heart rate responses to epinephrine, norepinephrine, and phenylephrine. ❑ Compare the pharmacodynamics of propranolol, labetalol, metoprolol, and pindolol. ❑ Compare the pharmacokinetics of propranolol, atenolol, esmolol, and nadolol. ❑ Describe the clinical indications and toxicities of typical α and β blockers. ❑ List and describe several drugs useful in glaucoma. 91 92 PART II Autonomic Drugs DRUG SUMMARY TABLE: Adrenoceptor Blockers Subclass Mechanism of Action Clinical Applications Pharmacokinetics Toxicities, Interactions Nonselective ` blockers Phentolamine Competitive pharmacologic antagonism at α receptors Pheochromocytoma, antidote to overdose of α agonists Oral, IV • short half-life Duration: 2–4 h Orthostatic hypotension • reflex tachycardia Phenoxybenzamine Irreversible (covalent) binding to α receptors Pheochromocytoma, carcinoid, mastocytosis, Raynaud’s phenomenon Oral, short half-life but long duration of action (24–48 h) Orthostatic hypotension, reflex tachycardia • gastrointestinal irritation Hypertension, benign prostatic hyperplasia Oral Duration: 8 h Orthostatic hypotension (especially first dose), but little reflex tachycardia Alpha1-selective blockers Prazosin Competitive antagonism at α1 receptors Doxazosin, terazosin: like prazosin; longer duration of action (12–24 h) Tamsulosin, silodosin: like prazosin, approved only for benign prostatic hyperplasia Alpha2-selective blockers Yohimbine Competitive antagonism at α2 receptors Obsolete use for erectile dysfunction • research use Oral, parenteral Tachycardia • gastrointestinal upset Competitive block of β receptors, local anesthetic effect Angina, arrhythmias (treatment and prophylaxis), hypertension, thyrotoxicosis, tremor, stage fright, migraine Oral and IV Duration: 4–6 h. Ready entry into CNS Excessive β blockade: bronchospasm (can be fatal in asthmatics), atrioventricular block, heart failure • CNS sedation, lethargy, sleep disturbances Oral Duration: 6–9 h Like propranolol with somewhat less danger of bronchospasm Nonselective a blockers Propranolol Timolol, betaxolol, others: lack local anesthetic action; useful in glaucoma Pindolol: partial agonist action; possibly safer in asthma Nadolol: like propranolol but longer action (up to 24 h) and less CNS effect Beta1-selective blockers Atenolol Competitive block of β1 receptors Hypertension, angina, arrhythmias Esmolol: IV agent for perioperative and thyroid storm arrhythmias, hypertensive emergency Metoprolol: like atenolol, oral, shown to reduce mortality in heart failure Nebivolol: oral β1-selective blocker with additional nitric oxide-dependent vasodilating action Beta2-selective blockers Butoxamine Competitive block of β2 receptors None • research use only — Bronchospasm Four isomers; 2 bind and block both α and β receptors Hypertension, hypertensive emergencies (IV) Oral and IV Duration: 5 h Like atenolol Alpha + beta blockers Labetalol Carvedilol: like labetalol, 2 isomers; shown to reduce mortality in heart failure PART III CARDIOVASCULAR DRUGS C H A P T E R 11 Drugs Used in Hypertension Hypertension is recognized as a major risk factor for several potentially lethal cardiac conditions, including myocardial infarction and heart failure. Antihypertensive drugs are organized around a clinical indication—the need to treat a disease—rather than a single receptor type. The drugs covered in this unit have a variety of mechanisms of action including diuresis, sympathoplegia, vasodilation, and antagonism of the renin-angiotensin-aldosterone system, and many agents are available in most categories. Drugs used in hypertension Diuretics Sympathoplegics— blockers of Alpha or beta receptors (prazosin, propranolol) Nerve terminals (guanethidine, reserpine) Ganglia (hexamethonium) Vasodilat ors Angiotensin antagonists ACE inhibitors (captopril) Renin inhibitor (aliskiren) Receptor blockers (losartan) Parenteral vasodilators Calcium (nitroprusside) blockers Older oral vasodilators (nifedipine) (hydralazine) CNS sympathetic outflow (clonidine) 93 94 PART III Cardiovascular Drugs High-Yield Terms to Learn Baroreceptor reflex Primary autonomic mechanism for blood pressure homeostasis; involves sensory input from carotid sinus and aorta to the vasomotor center and output via the parasympathetic and sympathetic motor nerves Catecholamine reuptake pump (norepinephrine transporter [NET]) Nerve terminal transporter responsible for recycling norepinephrine after release into the synapse Catecholamine vesicle pump Storage vesicle transporter that pumps catecholamine from neural cytoplasm into the storage vesicle; also called vesicle monoamine transporter (VMAT) End-organ damage Vascular damage in heart, kidney, retina, or brain Essential hypertension Hypertension of unknown etiology; also called primary hypertension False transmitter Substance, for example, octopamine, stored in vesicles and released into synaptic cleft but lacking the effect of the true transmitter, norepinephrine Hypertensive emergency (“malignant hypertension”) An accelerated form of severe hypertension associated with rising blood pressure and rapidly progressing damage to vessels and end organs. Often signaled by renal damage, encephalopathy, and retinal hemorrhages or by angina, stroke, or myocardial infarction Orthostatic hypotension Hypotension on assuming upright posture; postural hypotension Postganglionic neuron blocker Drug that blocks transmission by an action in the terminals of the postganglionic nerves Rebound hypertension Elevated blood pressure (usually above pretreatment levels) resulting from loss of antihypertensive drug effect Reflex tachycardia Tachycardia resulting from lowering of blood pressure; mediated by the baroreceptor reflex Secondary hypertension Hypertension caused by a diagnosable abnormality, eg, aortic coarctation, renal artery stenosis, adrenal tumor, etc. Compare essential hypertension. Stepped care Progressive addition of drugs to an antihypertensive regimen, starting with one (usually a diuretic) and adding in stepwise fashion an angiotensin inhibitor, a sympatholytic, and a vasodilator Sympatholytic, sympathoplegic Drug that reduces effects of the sympathetic nervous system Less than 20% of cases of hypertension are due to (“secondary” to) factors that can be clearly defined and corrected. This type of hypertension is associated with pheochromocytoma, coarctation of the aorta, renal vascular disease, adrenal cortical tumors, and a few other rare conditions. Most cases of hypertension are idiopathic, also called “primary” or “essential” hypertension. The strategies for treating idiopathic hypertension are based on the determinants of arterial pressure (see Figure 6–4). These strategies include reductions of blood volume, sympathetic effects, vascular smooth muscle tension, and angiotensin effects. Unfortunately, the baroreceptor reflex and the renin response in primary hypertension are reset to maintain the higher blood pressure. As a result, they respond to a therapeutically lowered blood pressure with compensatory homeostatic responses, which may be significant (Table 11–1). As indicated in Figure 11–1, these compensatory responses can be counteracted with β blockers and diuretics or angiotensin antagonists. DIURETICS Diuretics are covered in greater detail in Chapter 15 but are mentioned here because of their importance in hypertension. These drugs lower blood pressure by reduction of blood volume and probably also by a direct vascular effect that is not fully understood. The diuretics most important for treating hypertension are the thiazides (eg, chlorthalidone, hydrochlorothiazide) and the loop diuretics (eg, furosemide). Thiazides may be adequate in mild and moderate hypertension, but the loop agents are used in severe hypertension and in hypertensive emergencies. Compensatory responses to blood pressure lowering by diuretics are minimal (Table 11–1). When thiazides are given, the maximal antihypertensive effect is often achieved with doses lower than those required for the maximal diuretic effect. SKILL KEEPER 1: DEVELOPMENT OF NEW ANTIHYPERTENSIVE DRUGS (SEE CHAPTER 1) A new drug is under development for the treatment of hypertension. What types of data will the producer of this drug have to provide before beginning clinical trials? What data will be needed to market the drug? The Skill Keeper Answer appears at the end of the chapter. CHAPTER 11 Drugs Used in Hypertension Compensatory responses are significant for some of these agents (Table 11–1). Sympathoplegics are subdivided by anatomic site of action (Figure 11–2). TABLE 11–1 Compensatory responses to antihypertensive drugs. Class and Drug Compensatory Responses Diuretics (thiazides, loop agents) Minimal Sympathoplegics Centrally acting (clonidine, methyldopa) Ganglion blockers (obsolete) Alpha1-selective blockers Beta blockers Vasodilators Hydralazine Salt and water retention Salt and water retention Salt and water retention, slight tachycardia Minimal Salt and water retention, moderate tachycardia Marked salt and water retention, marked tachycardia Minor salt and water retention Minoxidil Nifedipine, other calcium channel blockers Nitroprusside Salt and water retention Angiotensin-renin antagonists (ACE inhibitors, ARBs, aliskiren) 95 Minimal SYMPATHOPLEGICS Sympathoplegic drugs interfere with sympathetic (SANS) control of cardiovascular function. The result is a reduction of one or more of the following: venous tone, heart rate, contractile force of the heart, cardiac output, and total peripheral resistance (see Figure 6–4). A. Baroreceptor-Sensitizing Agents A few natural products, such as veratrum alkaloids, appear to increase sensitivity of baroreceptor sensory nerves and reduce SANS outflow while increasing vagal tone to the heart. These agents are toxic and no clinically available drugs act at this site. B. Sympathoplegics That Act in the Central Nervous System Alpha2-selective agonists (eg, clonidine, methyldopa) cause a decrease in sympathetic outflow by activation of α2 receptors in the CNS. These drugs readily enter the CNS when given orally. Methyldopa is a prodrug; it is transported into the brain and then converted to methylnorepinephrine. Clonidine and methyldopa reduce blood pressure by reducing cardiac output, vascular resistance, or both. The major compensatory response is salt retention. Sudden discontinuation of clonidine causes rebound hypertension, which may be severe. This rebound increase in blood pressure can be controlled by reinstitution of clonidine therapy or administration of α blockers such as phentolamine. Methyldopa occasionally causes hematologic immunotoxicity, detected initially by test tube agglutination of red blood cells (positive Coombs’ test) and in some patients progressing to hemolytic anemia. Both drugs may cause sedation—methyldopa more so at therapeutic dosage. Early studies suggested that methyldopa protected kidney function and was safe in pregnancy; it is sometimes preferred for hypertension in pregnancy. Hypertension Initial treatment Decreased blood pressure Compensatory increased sympathetic outflow − − Beta blockers Compensatory increased renin secretion Tachycardia Diuretics, ACE inhibitors Salt and water retention Increased blood pressure FIGURE 11–1 Compensatory responses (orange boxes) to decreased blood pressure when treating hypertension. The initial treatment that causes the compensatory responses might be a vasodilator. Arrows with minus signs indicate drugs used (white boxes) to minimize the compensatory responses. ACE, angiotensin-converting enzyme. 96 PART III Cardiovascular Drugs B. Nucleus of the tractus solitarius and vasomotor center Brain stem Sensory fiber A. Baroreceptor in carotid sinus X XI Inhibitory interneurons Arterial blood pressure XII Motor fibers D Spinal cord C. Autonomic ganglion D. Sympathetic nerve ending E E. Alpha or beta receptor FIGURE 11–2 Baroreceptor reflex arc and sites of action of sympathoplegic drugs. The letters (A–E) indicate potential sites of action of subgroups of sympathoplegics described in the text. No clinically useful drugs act at the baroreceptor (site A), but drugs are available for each of the other sites. C. Ganglion-Blocking Drugs Nicotinic blockers that act in the ganglia are very efficacious, but because their adverse effects are severe, they are now considered obsolete. Hexamethonium and trimethaphan are extremely powerful blood pressure-lowering drugs. D. Postganglionic Sympathetic Nerve Terminal Blockers Drugs that deplete the adrenergic nerve terminal of its norepinephrine stores (eg, reserpine) or that deplete and block release of the stores (eg, guanethidine, guanadrel) can lower blood pressure. The major compensatory response is salt and water retention. In high dosages, these drugs are very efficacious but produce severe adverse effects and are now considered obsolete for hypertension. Monoamine oxidase (MAO) inhibitors were once used in hypertension because they cause the formation of a false transmitter (octopamine) in sympathetic postganglionic neuron terminals and lower blood pressure. Octopamine is stored, along with increased amounts of norepinephrine, in the transmitter vesicles. SANS nerve impulses then release a mixture of octopamine (which has very low efficacy) and norepinephrine, resulting in a smaller than normal increase in vascular resistance. Large doses of indirect-acting sympathomimetics, on the other hand (eg, the tyramine in a meal of fermented foods), may cause release of very large amounts of stored norepinephrine (along with the octopamine) and result in a hypertensive crisis. (Recall that tyramine normally has very low bioavailability because of metabolism by MAO. In the presence of MAO inhibitors, it has much higher bioavailability.) Because of this risk and the availability of better drugs, MAO inhibitors are no longer used in hypertension. However, they are still occasionally used for treatment of severe depressive disorder (Chapter 30). E. Adrenoceptor Blockers Alpha1-selective agents (eg, prazosin, doxazosin, terazosin) are moderately effective antihypertensive drugs. Alpha blockers reduce vascular resistance and venous return. The nonselective α blockers (phentolamine, phenoxybenzamine) are of no value in chronic hypertension because of excessive tachycardia. Alpha1selective adrenoceptor blockers are relatively free of the severe adverse effects of the nonselective α blockers and postganglionic nerve terminal sympathoplegic agents. They do, however, cause orthostatic hypotension, especially with the first few doses. On the other hand, they relax smooth muscle in the prostate, which is useful in benign prostatic hyperplasia. Beta blockers are used very heavily in the treatment of hypertension. Propranolol is the prototype, and atenolol, metoprolol, and carvedilol are among the most popular. They initially reduce cardiac output, but in chronic use their action may include a decrease in vascular resistance as a contributing effect. The latter effect may result from reduced angiotensin levels (β blockers reduce renin release from the kidney). Nebivolol is a newer β blocker with some direct vasodilator action caused by nitric oxide release. Potential adverse effects are listed in the Drug Summary Table. As noted in Chapter 10, β1-selective blockers with fewer CNS effects may have some advantages over the nonselective and more lipid-soluble agents. VASODILATORS Drugs that dilate blood vessels by acting directly on smooth muscle cells through nonautonomic mechanisms are useful in treating some hypertensive patients. Vasodilators act by four major mechanisms: blockade of calcium channels, release of nitric CHAPTER 11 Drugs Used in Hypertension TABLE 11–2 Mechanisms of action of vasodilators. Mechanism of Smooth Muscle Relaxation Examples Reduction of calcium influx via L-type channels Dihydropyridines: vessels > heart Release of nitric oxide from drug or vascular endothelium Hyperpolarization of vascular smooth muscle through opening of potassium channels Nitroprusside, hydralazine Activation of dopamine D1 receptors Fenoldopam Verapamil, diltiazem: heart ≥ vessels Minoxidil sulfate, diazoxide oxide, opening of potassium channels (which leads to hyperpolarization), and activation of D1 dopamine receptors (Table 11–2). Compensatory responses are listed in Table 11–1. A. Calcium Channel-Blocking Agents Calcium channel blockers (eg, nifedipine, verapamil, diltiazem) are effective vasodilators. Because they are moderately efficacious and orally active, these drugs are suitable for chronic use in hypertension of any severity. Verapamil and diltiazem also reduce cardiac output in most patients. Nifedipine is the prototype dihydropyridine calcium channel blocker, and many other dihydropyridines are available (amlodipine, felodipine, isradipine, etc). Because they are well-tolerated and produce fewer compensatory responses, the calcium channel blockers are much more commonly used than hydralazine or minoxidil. They are discussed in greater detail in Chapter 12. B. Hydralazine and Minoxidil These older vasodilators have more effect on arterioles than on veins. They are orally active and suitable for chronic therapy. Hydralazine apparently acts through the release of nitric oxide from endothelial cells. It causes significant baroreceptor homeostatic responses and must be combined with other drugs, usually diuretics and β blockers. However, it is rarely used at high dosage because of its toxicity (Drug Summary Table). Hydralazineinduced lupus erythematosus is reversible upon stopping the drug, and lupus is less common at dosages below 200 mg/d. Minoxidil is extremely efficacious, and systemic administration is reserved for severe hypertension. Minoxidil is a prodrug; its metabolite, minoxidil sulfate, is a potassium channel opener that hyperpolarizes and relaxes vascular smooth muscle. The compensatory responses to minoxidil (Figure 11–1) require the concomitant use of diuretics and β blockers. Because it can cause hirsutism, minoxidil is also available as a topical agent for the treatment of baldness. C. Nitroprusside, Diazoxide, and Fenoldopam These parenteral vasodilators are used in hypertensive emergencies. Nitroprusside is a light-sensitive, short-acting agent (duration 97 of action is a few minutes) that must be infused continuously. The release of nitric oxide (from the drug molecule itself) stimulates guanylyl cyclase and increases cyclic guanine monophosphate (cGMP) concentration and relaxation in vascular smooth muscle. Diazoxide is a thiazide derivative but lacks diuretic properties. It is given as intravenous boluses or as an infusion and has several hours’ duration of action. Diazoxide opens potassium channels, thus hyperpolarizing and relaxing smooth muscle cells. This drug also reduces insulin release and can be used to treat hypoglycemia caused by insulin-producing tumors. Dopamine D1 receptor activation by fenoldopam causes prompt, marked arteriolar vasodilation. This drug is given by intravenous infusion. It has a short duration of action (10 min) and, like nitroprusside and diazoxide, is used for hypertensive emergencies. ANGIOTENSIN ANTAGONISTS & A RENIN INHIBITOR The two primary groups of angiotensin antagonists are the angiotensin-converting enzyme (ACE) inhibitors and the angiotensin II receptor blockers (ARBs). ACE inhibitors (eg, captopril), which inhibit the enzyme variously known as angiotensin-converting enzyme, kininase II, and peptidyl dipeptidase, cause a reduction in blood levels of angiotensin II and aldosterone and an increase in endogenous vasodilators of the kinin family (bradykinin; Figure 11–3). ACE inhibitors have a low incidence of serious adverse effects (except in pregnancy) when given in normal dosage and produce minimal compensatory responses (Table 11–1). The ACE inhibitors are useful in heart failure and diabetes as well as in hypertension. The toxicities of ACE inhibitors include cough (up to 30% of patients), hyperkalemia, and renal damage in occasional patients with preexisting renal vascular disease (although they protect the diabetic kidney). They cause major renal damage in the fetus and are absolutely contraindicated in pregnancy. The second group of angiotensin antagonists, the receptor blockers, is represented by the orally active agents losartan, valsartan, irbesartan, candesartan, and other ARBs, which competitively inhibit angiotensin II at its AT1 receptor site. ARBs appear to be as effective in lowering blood pressure as the ACE inhibitors and have the advantage of a lower incidence of cough, although they do cause hyperkalemia. Like the ACE inhibitors, they cause fetal renal toxicity and are thus contraindicated in pregnancy. Aliskiren is a newer drug in the antihypertensive group and inhibits renin’s action on its substrate, angiotensinogen (Figure 11-3). It thus reduces the formation of angiotensin I and, in consequence, angiotensin II. Toxicities include headache and diarrhea. It does not appear to cause cough, but it is not yet known whether it has the other toxicities of the angiotensin antagonists. It does not show reproductive toxicity in animals but is considered to be contraindicated in pregnancy because of the toxicity of ACE inhibitors and ARBs. Angiotensin antagonists and renin inhibitors reduce aldosterone levels (angiotensin II is a major stimulant of aldosterone release) and cause potassium retention. If the patient has renal 98 PART III Cardiovascular Drugs Angiotensinogen − Renin Bradykinin (active vasodilator) Angiotensinconverting enzyme Angiotensin II (active vasoconstrictor) − Angiotensin I (inactive decapeptide) Aliskiren ACE inhibitors Inactive metabolites AT1 receptor blockers − AT1 receptor FIGURE 11–3 Actions of aliskiren, angiotensin-converting enzyme inhibitors, and AT1 receptor blockers. Renin converts angiotensinogen to angiotensin I. Block by aliskiren blocks the sequence at its start. ACE is responsible for activating angiotensin I to angiotensin II and for inactivating bradykinin, a vasodilator normally present in very low concentrations. Block of this enzyme thus decreases the concentration of a vasoconstrictor and increases the concentration of a vasodilator. The AT1 receptor antagonists lack the effect on bradykinin levels, which may explain the lower incidence of cough observed with these agents. impairment, is consuming a high-potassium diet, or is taking other drugs that tend to conserve potassium, potassium concentrations may reach toxic levels. CLINICAL USES OF ANTIHYPERTENSIVE DRUGS A. Stepped Care (Polypharmacy) Therapy of hypertension is complex because the disease is symptomless until far advanced and because the drugs may cause major compensatory responses and significant toxicities. However, overall toxicity can be reduced and compensatory responses minimized by the use of multiple drugs at lower dosages in patients with moderate or severe hypertension. Typically, drugs are added to a patient’s regimen in stepwise fashion; each additional agent is chosen from a different subgroup until adequate blood pressure control has been achieved. The usual steps include (1) lifestyle measures such as salt restriction and weight reduction, (2) diuretics (a thiazide), (3) sympathoplegics (a β blocker), (4) ACE inhibitors, and (5) vasodilators. The vasodilator chosen first is usually a calcium channel blocker. The ability of drugs in steps 2 and 3 to control the compensatory responses induced by the others should be noted (eg, propranolol reduces the tachycardia induced by hydralazine). Thus, rational polypharmacy minimizes toxicities while producing additive or supra-additive therapeutic effects. SKILL KEEPER 2: COMPENSATORY RESPONSES TO ANTIHYPERTENSIVE DRUGS (SEE CHAPTER 6) If hydralazine is administered in moderate dosage for several weeks, compensatory cardiac and renal responses will be observed. Specify the exact mechanisms and structures involved in these responses. The Skill Keeper Answer appears at the end of the chapter. B. Monotherapy It has been found in large clinical studies that many patients do well on a single drug (eg, an ACE inhibitor, calcium channel blocker, or combined α and β blocker). This approach to the treatment of mild and moderate hypertension has become more popular than stepped care because of its simplicity, better patient compliance, and—with modern drugs—a relatively low incidence of toxicity. C. Age and Ethnicity Older patients of most races respond better to diuretics and β blockers than to ACE inhibitors. African Americans of all ages respond better to diuretics and calcium channel blockers, and they respond less well to ACE inhibitors. There is considerable interindividual variability in metabolism of β blockers. D. Hypertensive Emergency Hypertensive emergency (formerly called malignant hypertension) is an accelerated form of severe hypertension associated with rising blood pressure and rapidly progressing damage to vessels and end organs. Management of hypertensive emergency must be carried out on an urgent basis in the hospital. Powerful vasodilators (nitroprusside, fenoldopam, or diazoxide) are combined with diuretics (furosemide) and β blockers to lower blood pressure to the 140–160/90–110 mm Hg range promptly (within a few hours). Further reduction is then pursued more slowly. QUESTIONS 1. A 32-year-old woman with hypertension wishes to become pregnant. Her physician informs her that she will have to switch to another antihypertensive drug. Which of the following drugs is absolutely contraindicated in pregnancy? (A) Atenolol (B) Losartan (C) Methyldopa (D) Nifedipine (E) Propranolol CHAPTER 11 Drugs Used in Hypertension 2. A patient is admitted to the emergency department with severe tachycardia after a drug overdose. His family reports that he has been depressed about his hypertension. Which one of the following drugs increases the heart rate in a dosedependent manner? (A) Captopril (B) Hydrochlorothiazide (C) Losartan (D) Minoxidil (E) Verapamil 3. Which one of the following is characteristic of nifedipine treatment in patients with essential hypertension? (A) Competitively blocks angiotensin II at its receptor (B) Decreases calcium efflux from skeletal muscle (C) Decreases renin concentration in the blood (D) Decreases calcium influx into smooth muscle (E) Decreases calcium flux into the urine 4. A 73-year-old man with a history of a recent change in his treatment for moderately severe hypertension is brought to the emergency department because of a fall at home. Which of the following drug groups is most likely to cause postural hypotension and thus an increased risk of falls? (A) ACE inhibitors (B) Alpha1-selective receptor blockers (C) Arteriolar dilators (D) Beta1-selective receptor blockers (E) Nonselective β blockers 5. A significant number of patients started on ACE inhibitor therapy for hypertension are intolerant and must be switched to a different class of drug. What is the most common manifestation of this intolerance? (A) Angioedema (B) Glaucoma (C) Headache (D) Incessant cough (E) Ventricular arrhythmias 6. Which one of the following is a significant unwanted effect of the drug named? (A) Constipation with verapamil (B) Heart failure with hydralazine (C) Hemolytic anemia with atenolol (D) Hypokalemia with aliskiren (E) Lupus-like syndrome with hydrochlorothiazide 7. Comparison of prazosin with atenolol shows that (A) Both decrease heart rate (B) Both increase cardiac output (C) Both increase renin secretion (D) Both increase sympathetic outflow from the CNS (E) Both produce orthostatic hypotension 8. A patient with hypertension and angina is referred for treatment. Metoprolol and verapamil are among the drugs considered. Both metoprolol and verapamil are associated with which one of the following? (A) Diarrhea (B) Hypoglycemia (C) Increased PR interval (D) Tachycardia (E) Thyrotoxicosis 99 9. A 45-year-old man is brought to the emergency department with mental obtundation. He is found to have a blood pressure of 220/160 and retinal hemorrhages. Which one of the following is used in severe hypertensive emergencies, is shortacting, acts on a G protein-coupled receptor, and must be given by intravenous infusion? (A) Aliskiren (B) Captopril (C) Fenoldopam (D) Hydralazine (E) Losartan (F) Metoprolol (G) Nitroprusside (H) Prazosin (I) Propranolol 10. Which of the following is very short-acting and acts by releasing nitric oxide? (A) Atenolol (B) Captopril (C) Diltiazem (D) Fenoldopam (E) Hydrochlorothiazide (F) Losartan (G) Minoxidil (H) Nitroprusside (I) Prazosin ANSWERS 1. Methyldopa is often recommended in pregnant patients because it has a good safety record. Calcium channel blockers (choice D) and β blockers (choices A and E) are not contraindicated. In contrast, ACE inhibitors and ARBs (choice B) have been shown to be teratogenic. The answer is B. 2. ACE inhibitors (choice A), ARBs (choice C), and diuretics (choice B) do not significantly increase heart rate. Although dihydropyridine calcium channel blockers do not usually reduce rate markedly (and may increase it), verapamil (choice E) and diltiazem do inhibit the sinoatrial node and predictably decrease rate. Other direct vasodilators (choice D) regularly increase heart rate, and minoxidil, a very efficacious vasodilator, causes severe tachycardia that must be controlled with β blockers. The answer is D. 3. Nifedipine is a prototype L-type calcium channel blocker and lowers blood pressure by reducing calcium influx into vascular smooth muscle. It has no effect on angiotensin-converting enzyme. Calcium efflux from skeletal muscle cells does not involve the L-type Ca channel. The plasma renin level may increase as a result of the compensatory response to reduced blood pressure. Calcium channel blockers have negligible effects on urine calcium. The answer is D. 4. Drug-induced postural (orthostatic) hypotension is usually due to venous pooling or excessive diuresis and inadequate blood volume. Venous pooling is normally prevented by α-receptor activation in vascular smooth muscle; thus, orthostatic hypotension is caused or exacerbated by α1 blockers, eg, prazosin. The answer is B. 100 PART III Cardiovascular Drugs 5. Chronic, intolerable cough is an important adverse effect of captopril and other ACE inhibitors. It may be reduced or prevented by prior administration of aspirin. These drugs are very commonly used in hypertensive diabetic patients because of their proven benefits in reducing diabetic renal damage. The ACE inhibitors are not associated with glaucoma; angioedema is not as common as cough; and headache and arrhythmias are rare. The answer is D. 6. Hydralazine (choice B) is sometimes used in heart failure. Beta blockers (choice C) are not associated with hematologic abnormalities, but methyldopa is. The thiazide diuretics (choice E) often cause mild hyperglycemia, hyperuricemia, and hyperlipidemia but not lupus; hydralazine is associated with a lupus-like syndrome. Aliskiren (choice D) and other inhibitors of the renin-angiotensin-aldosterone system may cause hyperkalemia, not hypokalemia. Verapamil (choice A) often causes constipation, probably by blocking L-type calcium channels in the colon. The answer is A. 7. Atenolol, but not prazosin, may decrease heart rate (choice A). Prazosin—but not atenolol—may increase cardiac output, a compensatory effect (choice B). Prazosin may increase renin output (a compensatory response), but β blockers inhibit its release by the kidney (choice C). By reducing blood pressure, both may increase central sympathetic outflow (a compensatory response). Beta blockers do not cause orthostatic hypotension. The answer is D. 8. Neither β blockers nor calcium channel blockers cause diarrhea. Hypoglycemia is not a common effect of any of the antihypertensive drugs. Thyroid disorders are not associated with either drug group. However, calcium blockers, especially verapamil and diltiazem, and β blockers are associated with depression of calcium-dependent processes in the heart, for example, contractility, heart rate, and atrioventricular conduction. Therefore, bradycardia and increased PR interval may be expected. The dihydropyridines do not often cause cardiac depression, probably because they evoke increased sympathetic outflow as a result of their dominant vascular effects. The answer is C. 9. Fenoldopam, nitroprusside, and propranolol are the drugs in the list that have been used in hypertensive emergencies. Fenoldopam and nitroprusside are used by infusion only, but nitroprusside releases nitric oxide, which acts on intracellular guanylyl cyclase. The answer is C. 10. The two agents in this list that act via a nitric oxide mechanism are hydralazine and nitroprusside (see Table 11–2). However, hydralazine has a duration of action of hours, whereas nitroprusside acts for seconds to minutes and must be given by intravenous infusion. The answer is H. SKILL KEEPER 1 ANSWER: DEVELOPMENT OF NEW ANTIHYPERTENSIVE DRUGS (SEE CHAPTER 1) The FDA requires a broad range of animal data, provided by the developer in an investigational new drug (IND) application, before clinical trials can be started. These data must show that the drug has the expected effects on blood pressure in animals and has low and well-defined toxicity in at least two species. A new drug application (NDA) must be submitted and approved before marketing can begin. This application usually requires data on pharmacokinetics in volunteers (phase 1), efficacy and safety in a small group of closely observed patients (phase 2), and efficacy and safety in a much larger group of patients under conditions of actual use (phase 3). SKILL KEEPER 2 ANSWER: COMPENSATORY RESPONSES TO ANTIHYPERTENSIVE DRUGS (SEE CHAPTER 6) The compensatory responses to hydralazine are tachycardia and salt and water retention. These responses are generated by the baroreceptor and renin-angiotensin-aldosterone mechanisms summarized in Figures 6–4 and 11–1. The motor limb of the sympathetic response consists of outflow from the vasomotor center to the heart and vessels, as shown in Figure 11–2. You should be able to reproduce these diagrams from memory. CHECKLIST When you complete this chapter, you should be able to: ❑ List 4 major groups of antihypertensive drugs, and give examples of drugs in each group. (Renin inhibitors are not considered an independent major group; can you name the one available drug that acts by this mechanism?) ❑ Describe the compensatory responses, if any, to each of the 4 major types of antihypertensive drugs. ❑ List the major sites of action of sympathoplegic drugs in clinical use, and give examples of drugs that act at each site. ❑ List the 4 mechanisms of action of vasodilator drugs. ❑ List the major antihypertensive vasodilator drugs and describe their effects. ❑ Describe the differences between the 2 types of angiotensin antagonists. ❑ List the major toxicities of the prototype antihypertensive agents. CHAPTER 11 Drugs Used in Hypertension 101 DRUG SUMMARY TABLE: Drugs Used in Hypertension Subclass Mechanism of Action Clinical Applications Pharmacokinetics Toxicities, Interactions Diuretics (see also Chapter 15) Hydrochlorothiazide, chlorthalidone Block Na/Cl transporter in distal convoluted tubule Hypertension, mild edema Oral Duration: 8–12 h Furosemide Block Na/K/2Cl transporter in thick ascending limb Hypertension, heart failure, edema, hypercalcemia Oral, parenteral Duration: 2–3 h Hypokalemia, hyperglycemia, hyperuricemia, hyperlipidemia Hypokalemia, hypovolemia, ototoxicity Agonist at α2 receptors • in CNS this results in decreased SANS outflow Prodrug converted to methylnorepinephrine in CNS, with effects like clonidine Hypertension Oral and transdermal Oral duration: 2–3 days • transdermal 1 wk Oral Duration: 12–24 h Sedation, danger of severe rebound hypertension if suddenly stopped Sedation, induces hemolytic antibodies Obsolete prototype nicotinic acetylcholine (ACh) receptor blocker in ganglia • blocks all ANS transmission None Oral, parenteral; no CNS effect Severe orthostatic hypotension, constipation, blurred vision, sexual dysfunction Sympathoplegics Centrally acting Clonidine Methyldopa Ganglion blockers Hexamethonium Hypertension Trimethaphan: IV, obsolete short-acting ganglion blocker for hypertensive emergencies, controlled hypotension Mecamylamine: oral ganglion blocker, several hours’ duration, enters CNS Postganglionic neuron blockers Reserpine Blocks vesicular pump (VMAT) in adrenergic neurons Obsolete in hypertension, Huntington’s disease Oral Duration: 5 days Sedation • severe psychiatric depression (high doses) Guanadrel: blocks release of norepinephrine, depletes stores; oral, long duration; severe orthostatic hypotension (guanethidine, a similar drug, was withdrawn in the United States) Alpha blockers Prazosin Selective α1 blocker • reduces peripheral vascular resistance • prostatic smooth muscle tone Mild hypertension, benign prostatic hyperplasia Oral Duration: 6–8 h First dose orthostatic hypotension Oral, parenteral Duration: 6–8 h (extended release forms available) Bronchospasm in asthmatics • excessive cardiac depression, sexual dysfunction, sedation, sleep disturbances Doxazosin, terazosin: similar to prazosin but longer duration of action Beta blockers Propranolol Prototype nonselective β blocker • reduces cardiac output • possible secondary reduction in renin release Hypertension • many other applications (see Chapter 10) Atenolol, metoprolol: like propranolol but β1-selective; fewer adverse effects Labetalol, carvedilol: combined α and β blockade; oral and parenteral (Continued ) 102 PART III Cardiovascular Drugs DRUG SUMMARY TABLE: Drugs Used in Hypertension (Continued) Subclass Mechanism of Action Clinical Applications Pharmacokinetics Toxicities, Interactions Prototype L-type calcium channel blockers • combine moderate vascular effect with weak cardiac effect Hypertension, angina Oral Constipation Vasodilators, oral Calcium channel blockers Nifedipine, other dihydropyridines Duration: 6–24 h Verapamil, diltiazem oral and parenteral; also used in arrhythmias; greater cardiodepressant effects than dihydropyridines; verapamil blocks P-glycoprotein transporter (see Chapter 5) Older oral vasodilators Hydralazine Probably causes release of nitric acid (NO) by endothelial cells • causes arteriolar dilation Prodrug, sulfate metabolite opens K+ channels, causes arteriolar smooth muscle hyperpolarization and vasodilation Hypertension (also used in heart failure in combination with isosorbide dinitrate) Severe hypertension • male-pattern baldness Oral Duration: 6–8 h Tachycardia, salt and water retention, lupus-like syndrome Oral, topical Duration: 6–8 h Marked tachycardia, salt and water retention • hirsutism Releases NO from drug molecule Hypertensive emergencies • cardiac decompensation Parenteral only Duration: minutes • requires constant infusion Excessive hypotension • prolonged infusion may cause thiocyanate and cyanide toxicity Diazoxide K+ channel opener in smooth muscle, secretory cells Hypertensive emergencies • hypoglycemia due to insulin-secreting tumors Parenteral for hypertension, oral for insulinoma Hyperglycemia • edema, excessive hypotension Fenoldopam D1 agonist • causes arteriolar dilation Hypertensive emergencies Parenteral only, very short duration Excessive hypotension Renin inhibitor • reduces angiotensin I synthesis Hypertension Oral Duration: 12 h Angioedema, renal impairment ACE inhibitor • reduces angiotensin II synthesis Hypertension, diabetic renal disease, heart failure Oral Cough • hyperkalemia • teratogen Minoxidil Vasodilators, parenteral Nitroprusside Renin antagonist Aliskiren Angiotensin antagonists ACE inhibitors Captopril Half-life: 2.2 h but large doses provide duration of 12 h Benazepril, enalapril, lisinopril, others: like captopril but longer half-lives Angiotensin II receptor blockers (ARBs) Losartan Blocks AT1 receptors Hypertension Oral Hyperkalemia • teratogen Duration: 6–8 h Candesartan, irbesartan, others: like losartan ACE, angiotensin-converting enzyme; ANS, autonomic nervous system; CNS, central nervous system; SANS, sympathetic autonomic nervous system. C A P T E R 12 Drugs Used in the Treatment of Angina Pectoris Angina pectoris refers to a strangling or pressure-like pain caused by cardiac ischemia. The pain is usually located substernally but is sometimes perceived in the neck, shoulder and arm, or epigastrium. Women develop angina at a later age than H men and are less likely to have classic substernal pain. Drugs used in angina exploit two main strategies: reduction of oxygen demand and increase of oxygen delivery to the myocardium. Drugs used in angina pectoris Vasodilators Cardiac depressants Calcium blockers (verapamil) Nitrates Beta blockers (propranolol) Other drugs Metabolism modifiers; rate inhibitors Long duration (transdermal nitroglycerin) Intermediate (oral nitroglycerin) Short duration (sublingual nitroglycerin) PATHOPHYSIOLOGY OF ANGINA A. Types of Angina 1. Atherosclerotic angina—Atherosclerotic angina is also known as angina of effort or classic angina. It is associated with atheromatous plaques that partially occlude one or more coronary arteries. When cardiac work increases (eg, in exercise), the obstruction of flow and inadequate oxygen delivery results in the accumulation of metabolites, eg, lactic acid, and ischemic changes that stimulate myocardial pain endings. Rest, by reducing cardiac work, usually leads to complete relief of the pain within 15 min. Atherosclerotic angina constitutes about 90% of angina cases. 103 104 PART III Cardiovascular Drugs High-Yield Terms to Learn Angina of effort, classic angina, atherosclerotic angina Angina pectoris (crushing, strangling chest pain) that is precipitated by exertion Vasospastic angina, variant angina, Prinzmetal’s angina Angina precipitated by reversible spasm of coronary vessels, often at rest Coronary vasodilator Older, incorrect name for drugs useful in angina; some potent coronary vasodilators are ineffective in angina “Monday disease” Industrial disease caused by chronic exposure to vasodilating concentrations of organic nitrates in the workplace; characterized by headache, dizziness, and tachycardia on return to work after 2 days absence Nitrate tolerance, tachyphylaxis Loss of effect of a nitrate vasodilator when exposure is prolonged beyond 10–12 h Unstable angina Rapidly progressing increase in frequency and severity of anginal attacks; an acute coronary syndrome that often heralds imminent myocardial infarction Preload Filling pressure of the heart, dependent on venous tone and blood volume; determines enddiastolic fiber length and tension Afterload Impedance to ejection of stroke volume; determined by vascular resistance (arterial blood pressure) and arterial stiffness; determines systolic fiber tension Intramyocardial fiber tension Force exerted by myocardial fibers, especially ventricular fibers at any given time; a primary determinant of myocardial O2 requirement Double product The product of heart rate and systolic blood pressure; an estimate of cardiac work Myocardial revascularization Mechanical intervention to improve O2 delivery to the myocardium by angioplasty or bypass grafting 2. Vasospastic angina—Vasospastic angina, also known as rest angina, variant angina, or Prinzmetal’s angina, is responsible for less than 10% of angina cases. It involves reversible spasm of coronaries, usually at the site of an atherosclerotic plaque. Spasm may occur at any time, even during sleep. Vasospastic angina may deteriorate into unstable angina. is thought to be the immediate precursor of a myocardial infarction and is treated as a medical emergency. 3. Unstable angina—A third type of angina—unstable or crescendo angina, also known as acute coronary syndrome—is characterized by increased frequency and severity of attacks that result from a combination of atherosclerotic plaques, platelet aggregation at fractured plaques, and vasospasm. Unstable angina The pharmacologic treatment of coronary insufficiency is based on the physiologic factors that control myocardial oxygen requirement. A major determinant is myocardial fiber tension (the higher the tension, the greater the oxygen requirement). Several variables contribute to fiber tension (Figure 12–1), as discussed next. DETERMINANTS OF CARDIAC OXYGEN REQUIREMENT Systolic factors Diastolic factors Blood volume Venous tone∗ + + Peripheral resistance∗ + Heart rate∗ + Heart force∗ + Ejection time∗ + Intramyocardial fiber tension Myocardial O2 requirement FIGURE 12–1 Determinants of the volume of oxygen required by the heart. Both diastolic and systolic factors contribute to the oxygen requirement; most of these factors are directly influenced by sympathetic discharge (venous tone, peripheral resistance, heart rate, and heart force) as noted by the asterisks. CHAPTER 12 Drugs Used in the Treatment of Angina Pectoris THERAPEUTIC STRATEGIES The defect that causes anginal pain is inadequate coronary oxygen delivery relative to the myocardial oxygen requirement. This defect can be corrected—at present—in 2 ways: by increasing oxygen delivery and by reducing oxygen requirement (Figure 12–2). Traditional pharmacologic therapies include the nitrates, the calcium channel blockers, and the β blockers. A newer strategy attempts to increase the efficiency of oxygen utilization by shifting the energy substrate preference of the heart from fatty acids to glucose. Drugs that may act by this mechanism are termed partial fatty acid oxidation inhibitors (pFOX inhibitors) and include ranolazine and trimetazidine. However, more recent evidence suggests that the major mechanism of action of ranolazine is inhibition of late sodium current (see below). Another new group of antianginal drugs selectively reduces heart rate (and O2 requirement) with no other detectable hemodynamic effects. These investigational drugs (ivabradine is the prototype) act by inhibition of the sinoatrial pacemaker current, If. The nitrates, calcium blockers, and β blockers all reduce the oxygen requirement in atherosclerotic angina. Nitrates and calcium channel blockers (but not β blockers) can also increase oxygen delivery by reducing spasm in vasospastic angina. Myocardial revascularization corrects coronary obstruction either by bypass grafting or by angioplasty (enlargement of the coronary lumen by means of a special catheter). Therapy of unstable angina differs from that of stable angina in that urgent angioplasty is the treatment of choice in most patients and platelet clotting is the Coronary obstruction Normal Oxygen delivery Note that several of these variables are increased by sympathetic discharge. Preload (diastolic filling pressure) is a function of blood volume and venous tone. Venous tone is mainly controlled by sympathetic activity. Afterload is determined by arterial blood pressure and large artery stiffness. It is one of the systolic determinants of oxygen requirement. Heart rate contributes to total fiber tension because at fast heart rates, fibers spend more time at systolic tension levels. Furthermore, at faster rates, diastole is abbreviated, and diastole constitutes the time available for coronary flow (coronary blood flow is low or nil during systole). Heart rate and systolic blood pressure may be multiplied to yield the double product, a measure of cardiac work and therefore of oxygen requirement. As intensity of exercise (eg, running on a treadmill) increases, demand for cardiac output increases, so the double product also increases. However, the double product is sensitive to sympathetic tone, as is cardiac oxygen demand (Figure 12–1). In patients with atherosclerotic angina, effective drugs reduce the double product by reducing cardiac work without reducing exercise capacity. Force of cardiac contraction is another systolic factor controlled mainly by sympathetic outflow to the heart. Ejection time for ventricular contraction is inversely related to force of contraction but is also influenced by impedance to outflow. Increased ejection time (prolonged systole) increases oxygen requirement. Coronary vasodilation 105 ∗ ∗ ∗∗ ∗ = Anginal pain Oxygen requirement FIGURE 12–2 Strategies for the treatment of effort angina. When coronary flow is adequate, O2 delivery increases as O2 requirement increases with exercise (black line). Angina is characterized by reduced coronary oxygen delivery versus oxygen requirement (curve in red line), and anginal pain occurs as the oxygen debt increases. In some cases, this can be corrected by increasing oxygen delivery (revascularization or, in the case of reversible vasospasm, nitrates and calcium channel blockers, brown line). More often, drugs are used to reduce oxygen requirement (nitrates, β blockers, and calcium channel blockers) and slow progress along the red line. major target of drug therapy. A variety of platelet inhibitors are used in this condition (see Chapter 34). Intravenous nitroglycerin is sometimes of value. NITRATES A. Classification and Pharmacokinetics Nitroglycerin (the active ingredient in dynamite) is the most important of the therapeutic nitrates and is available in forms that provide a range of durations of action from 10–20 min (sublingual for relief of acute attacks) to 8–10 h (transdermal for prophylaxis) (see the Drug Summary Table at the end of the chapter). Nitroglycerin (glyceryl trinitrate) is rapidly denitrated in the liver and in smooth muscle—first to the 2 dinitrates (glyceryl dinitrate), which retain a significant vasodilating effect; and more slowly to the mononitrates, which are much less active. Because of the high enzyme activity in the liver, the first-pass effect for nitroglycerin is about 90%. The efficacy of oral (swallowed) nitroglycerin probably results from the high levels of glyceryl dinitrate in the blood. The effects of sublingual and transdermal nitroglycerin are mainly the result of the unchanged drug because these routes avoid the first-pass effect (see Chapters 1 and 3). Other nitrates are similar to nitroglycerin in their pharmacokinetics and pharmacodynamics. Isosorbide dinitrate is another commonly used nitrate; it is available in sublingual and oral forms. Isosorbide dinitrate is rapidly denitrated in the liver and smooth muscle to isosorbide mononitrate, which is also active. Isosorbide mononitrate is available as a separate drug for oral use. Several other nitrates are available for oral use and, like the oral nitroglycerin 106 PART III Cardiovascular Drugs preparation, have an intermediate duration of action (4–6 h). Amyl nitrite is a volatile and rapid-acting vasodilator that was used for angina by the inhalational route but is now rarely prescribed. Note that this mechanism is identical to that of nitroprusside (see Chapter 11). C. Organ System Effects B. Mechanism of Action Nitrates release nitric oxide (NO) within smooth muscle cells, probably through the action of the mitochondrial enzyme aldehyde dehydrogenase-2 (ALDH2). NO stimulates soluble (cytoplasmic) guanylyl cyclase and causes an increase of the second messenger cGMP (cyclic guanosine monophosphate); the latter results in smooth muscle relaxation by stimulating the dephosphorylation of myosin light-chain phosphate (Figure 12–3). 1. Cardiovascular—Smooth muscle relaxation by nitrates leads to an important degree of venodilation, which results in reduced cardiac size and cardiac output through reduced preload. Relaxation of arterial smooth muscle may increase flow through partially occluded epicardial coronary vessels. Reduced afterload, from arteriolar dilation of resistance vessels, may contribute to an increase in ejection and a further decrease Blood vessel lumen Capillary endothelial cells Nitrates Nitrites Arginine Nitric oxide (NO) Ca2+ Interstitium Ca2+ blockers – Nitrates Nitrites NO Ca 2+ + Vascular smooth muscle cell – GTP cGMP Sildenafil GMP + + Myosin light chains (myosin-LC) Myosin-LC Actin Contraction Relaxation FIGURE 12–3 Mechanisms of smooth muscle relaxation by calcium channel blockers and nitrates. Contraction results from phosphorylation of myosin light chains (MLC) by myosin light-chain kinase (MLCK). MLCK is activated by Ca2+, so calcium channel blockers reduce this step. Relaxation follows when the phosphorylated light chains are dephosphorylated, a process facilitated by cyclic guanosine monophosphate (cGMP). Nitrates and other sources of nitric oxide (NO) increase cGMP synthesis, and phosphodiesterase (PDE) inhibitors reduce cGMP metabolism. eNOS, endothelial nitric oxide synthase; GC, activated guanylyl cyclase; GTP, guanosine triphosphate. (Modified and reproduced, with permission, from Katzung BG, editor: Basic & Clinical Pharmacology, 12th ed. McGraw-Hill, 2012: Fig. 12–2.) CHAPTER 12 Drugs Used in the Treatment of Angina Pectoris 2. Other organs—Nitrates relax the smooth muscle of the bronchi, gastrointestinal tract, and genitourinary tract, but these effects are too small to be clinically significant. Intravenous nitroglycerin (sometimes used in unstable angina) reduces platelet aggregation. There are no clinically useful effects on other tissues. D. Clinical Uses As previously noted, nitroglycerin is available in several formulations (see Drug Summary Table). The standard form for treatment of acute anginal pain is the sublingual tablet or spray, which has a duration of action of 10–20 min. Sublingual isosorbide dinitrate is similar with a duration of 30 min. Oral (swallowed) normal-release formulations of nitroglycerin and isosorbide dinitrate have durations of action of 4–6 h. Sustainedrelease oral forms have a somewhat longer duration of action. Transdermal formulations (ointment or patch) can maintain blood levels for up to 24 h. Tolerance develops after 8–10 h, however, with rapidly diminishing effectiveness thereafter. It is therefore recommended that nitroglycerin patches be removed after 10–12 h. A new patch can be applied after 12 h of patchfree recovery. E. Toxicity of Nitrates and Nitrites The most common toxic effects of nitrates are the responses evoked by vasodilation. These include tachycardia (from the baroreceptor reflex), orthostatic hypotension (a direct extension of the venodilator effect), and throbbing headache from meningeal artery vasodilation. Nitrates interact with sildenafil and similar drugs promoted for erectile dysfunction. These agents inhibit a phosphodiesterase isoform (PDE5) that metabolizes cGMP in smooth muscle (Figure 12–4). The increased cGMP in erectile smooth muscle relaxes it, allowing for greater inflow of blood and more effective and prolonged erection. This relaxation also occurs in vascular smooth muscle. As a result, the combination of nitrates (through increased production of cGMP) and a PDE5 inhibitor (through decreased breakdown of cGMP) causes a synergistic relaxation of GTP Guanylyl cyclase + Nitrates Smooth muscle relaxation cGMP Phosphodiesterase 5 − in cardiac size. Some studies suggest that of the vascular beds, the veins are the most sensitive, arteries less so, and arterioles least sensitive. Venodilation leads to decreased diastolic heart size and fiber tension. Arteriolar dilation leads to reduced peripheral resistance and blood pressure. These changes contribute to an overall reduction in myocardial fiber tension, oxygen consumption, and the double product. Thus, the primary mechanism of therapeutic benefit in atherosclerotic angina is reduction of the oxygen requirement. A secondary mechanism—namely, an increase in coronary flow via collateral vessels in ischemic areas—has also been proposed. In vasospastic angina, reversal of coronary spasm and increased flow can be demonstrated. Nitrates have no direct effects on cardiac muscle, but significant reflex tachycardia and increased force of contraction are common results when nitroglycerin reduces the blood pressure. These compensatory effects result from the baroreceptor mechanism shown in Figure 6–4. 107 − Erectile tissue Blood vessels Sildenafil, vardenafil, tadalafil GMP FIGURE 12–4 Mechanism of the interaction between nitrates and drugs used in erectile dysfunction. Because these drug groups increase cyclic guanosine monophosphate (cGMP) by complementary mechanisms, they can have a synergistic effect on blood pressure resulting in dangerous hypotension. GTP, guanosine triphosphate. vascular smooth muscle with potentially dangerous hypotension and inadequate perfusion of critical organs. Nitrites are of significant toxicologic importance because they cause methemoglobinemia at high blood concentrations. This same effect has a potential antidotal action in cyanide poisoning (see later discussion). The nitrates do not cause methemoglobinemia. In the past, the nitrates were responsible for several occupational diseases in explosives factories in which workplace contamination by these volatile chemicals was severe. The most common of these diseases was “Monday disease,” that is, the alternating development of tolerance (during the work week) and loss of tolerance (over the weekend) for the vasodilating action and its associated tachycardia and resulting in headache (from cranial vasodilation), tachycardia, and dizziness (from orthostatic hypotension) every Monday. F. Nitrites in the Treatment of Cyanide Poisoning Cyanide ion rapidly complexes with the iron in cytochrome oxidase, resulting in a block of oxidative metabolism and cell death. Fortunately, the iron in methemoglobin has a higher affinity for cyanide than does the iron in cytochrome oxidase. Nitrites convert the ferrous iron in hemoglobin to the ferric form, yielding methemoglobin. Therefore, cyanide poisoning can be treated by a 3-step procedure: (1) immediate inhalation of amyl nitrite, followed by (2) intravenous administration of sodium nitrite, which rapidly increases the methemoglobin level to the degree necessary to remove a significant amount of cyanide from cytochrome oxidase. This is followed by (3) intravenous sodium thiosulfate, which converts cyanomethemoglobin resulting from step 2 to thiocyanate and methemoglobin. Thiocyanate is much less toxic than cyanide and is excreted by the kidney. (Note that excessive methemoglobinemia is fatal because methemoglobin is a very poor oxygen carrier.) Recently, hydroxocobalamin, a form of vitamin B12, has become the preferred method of treating cyanide poisoning (see Chapter 58). 108 PART III Cardiovascular Drugs CALCIUM CHANNEL-BLOCKING DRUGS A. Classification and Pharmacokinetics Several types of calcium channel blockers are approved for use in angina; these drugs are typified by nifedipine, a dihydropyridine, several other dihydropyridines, and the nondihydropyridines diltiazem and verapamil. Although calcium channel blockers differ markedly in structure, all are orally active and most have half-lives of 3–6 h. B. Mechanism of Action Calcium channel blockers block voltage-gated L-type calcium channels, the calcium channels most important in cardiac and smooth muscle, and reduce intracellular calcium concentration and muscle contractility (Figure 12–3). None of these channel blockers interferes with calcium-dependent neurotransmission or hormone release because these processes use different types of calcium channels that are not blocked by L-channel blockers. Nerve ending calcium channels are of the N-, P-, and R-types. Secretory cells use L-type channels, but these channels are less sensitive to the calcium blockers than are cardiac and smooth muscle L-type channels. C. Effects and Clinical Use Calcium blockers relax blood vessels and, to a lesser extent, the uterus, bronchi, and gut. The rate and contractility of the heart are reduced by diltiazem and verapamil. Because they block calcium-dependent conduction in the atrioventricular (AV) node, verapamil and diltiazem may be used to treat AV nodal arrhythmias (see Chapter 14). Nifedipine and other dihydropyridines evoke greater vasodilation, and the resulting sympathetic reflex prevents bradycardia and may actually increase heart rate. All the calcium channel blockers in sufficient dosage reduce blood pressure and reduce the double product in patients with angina. Calcium blockers are effective as prophylactic therapy in both effort and vasospastic angina; nifedipine has also been used to abort acute anginal attacks but use of the prompt-release form is discouraged (see Skill Keeper). In severe atherosclerotic angina, these drugs are particularly valuable when combined with nitrates (Table 12–1). In addition to well-established uses in angina, hypertension, and supraventricular tachycardia, some of these agents are used in migraine, preterm labor, stroke, and Raynaud’s phenomenon. SKILL KEEPER: NIFEDIPINE CARDIOTOXICITY (SEE CHAPTER 6) A pair of studies during the 1990s suggested that use of nifedipine was associated with an increased risk of myocardial infarction. What effects of nifedipine might lead to this result? The Skill Keeper Answer appears at the end of the chapter. D. Toxicity The calcium channel blockers cause constipation, pretibial edema, nausea, flushing, and dizziness. More serious adverse effects include heart failure, AV blockade, and sinus node depression; these are most common with verapamil and least common with the dihydropyridines. BETA-BLOCKING DRUGS A. Classification and Mechanism of Action These drugs are described in detail in Chapter 10. Because they reduce cardiac work (and oxygen demand), all β blockers are effective in the prophylaxis of atherosclerotic angina attacks. B. Effects and Clinical Use Actions include both beneficial antianginal effects (decreased heart rate, cardiac force, blood pressure) and detrimental effects (increased heart size, longer ejection period; Table 12–1). Like nitrates and calcium channel blockers, β blockers reduce cardiac work, the double product, and oxygen demand. Beta blockers are used only for prophylactic therapy of angina; they are of no value in an acute attack. They are effective in preventing exercise-induced angina but are ineffective against the vasospastic form. The combination of β blockers and nitrates is useful because the adverse undesirable compensatory effects evoked by the nitrates (tachycardia and increased cardiac force) are prevented or reduced by β blockade (Table 12–1). C. Toxicity See Chapter 10. TABLE 12–1 Effects of nitrates alone or with beta blockers or calcium channel blockers in angina pectoris.a a Nitrates Alone Beta Blockers or Calcium Channel Blockers Alone Combined Nitrates and a Blockers or Calcium Channel Blockers Heart rate Reflex increase Decrease Decrease Arterial pressure End-diastolic pressure Contractility Ejection time Decrease Decrease Reflex increase Reflex decrease Decrease Increase Decrease Increase Decrease Decrease No change or decrease No change Net myocardial oxygen requirement Decrease Decrease Decrease Undesirable effects (effects that increase oxygen requirement) are shown in italics; major beneficial effects are shown in bold. CHAPTER 12 Drugs Used in the Treatment of Angina Pectoris NEWER DRUGS Ranolazine appears to act mainly by reducing a late, prolonged sodium current in myocardial cells. The decrease in intracellular sodium causes an increase in calcium expulsion via the Na/Ca transporter (see Chapter 13) and a reduction in cardiac force and work. As noted previously, it may also alter cardiac metabolism. Ranolazine is moderately effective in angina prophylaxis. Ivabradine, an investigational drug, inhibits the If sodium current in the sinoatrial node. The reduction in this hyperpolarizationinduced inward pacemaker current results in decreased heart rate and consequently decreased cardiac work. NONPHARMACOLOGIC THERAPY Myocardial revascularization by coronary artery bypass grafting (CABG) and percutaneous transluminal coronary angioplasty (PTCA) are extremely important in the treatment of severe angina. These are the only methods capable of consistently increasing coronary flow in atherosclerotic angina and increasing the double product. QUESTIONS Questions 1–4. A 60-year-old man presents to his primary care physician with a complaint of severe chest pain when he walks uphill to his home in cold weather. The pain disappears when he rests. After evaluation and discussion of treatment options, a decision is made to treat him with nitroglycerin. 1. Which of the following is a common direct or reflex effect of nitroglycerin? (A) Decreased heart rate (B) Decreased venous capacitance (C) Increased afterload (D) Increased cardiac force (E) Increased diastolic myocardial fiber tension 2. In advising the patient about the adverse effects he may notice, you point out that nitroglycerin in moderate doses often produces certain symptoms. Which of the following effects might occur due to the mechanism listed? (A) Constipation (B) Dizziness due to reduced cardiac force of contraction (C) Diuresis due to sympathetic discharge (D) Headache due to meningeal vasodilation (E) Hypertension due to reflex tachycardia 3. One year later, the patient returns complaining that his nitroglycerin works well when he takes it for an acute attack but that he is now having more frequent attacks and would like something to prevent them. Useful drugs for the prophylaxis of angina of effort include (A) Amyl nitrite (B) Esmolol (C) Sublingual isosorbide dinitrate (D) Sublingual nitroglycerin (E) Verapamil 109 4. If a β blocker were to be used for prophylaxis in this patient, what is the most probable mechanism of action in angina? (A) Block of exercise-induced tachycardia (B) Decreased end-diastolic ventricular volume (C) Increased double product (D) Increased cardiac force (E) Decreased ventricular ejection time 5. A new 60-year-old patient presents to the medical clinic with hypertension and angina. He is 1.8 meters tall with a waist measurement of 1.1 m. Weight is 97 kg. Blood pressure is 150/95 and pulse 85. In considering adverse effects of possible drugs for these conditions, you note that an adverse effect that nitroglycerin and prazosin have in common is (A) Bradycardia (B) Impaired sexual function (C) Lupus erythematosus syndrome (D) Orthostatic hypotension (E) Weight gain 6. A man is admitted to the emergency department with a brownish cyanotic appearance, marked shortness of breath, and hypotension. Which of the following is most likely to cause methemoglobinemia? (A) Amyl nitrite (B) Isosorbide dinitrate (C) Isosorbide mononitrate (D) Nitroglycerin (E) Sodium cyanide 7. Another patient is admitted to the emergency department after a drug overdose. He is noted to have hypotension and severe bradycardia. He has been receiving therapy for hypertension and angina. Which of the following drugs in high doses causes bradycardia? (A) Amlodipine (B) Isosorbide dinitrate (C) Nitroglycerin (D) Prazosin (E) Verapamil 8. A 45-year-old woman with hyperlipidemia and frequent migraine headaches develops angina of effort. Which of the following is relatively contraindicated because of her migraines? (A) Amlodipine (B) Diltiazem (C) Metoprolol (D) Nitroglycerin (E) Verapamil 9. When nitrates are used in combination with other drugs for the treatment of angina, which one of the following combinations results in additive effects on the variable specified? (A) Beta blockers and nitrates on end-diastolic cardiac size (B) Beta blockers and nitrates on heart rate (C) Beta blockers and nitrates on venous tone (D) Calcium channel blockers and β blockers on cardiac force (E) Calcium channel blockers and nitrates on heart rate 110 PART III Cardiovascular Drugs 10. Certain drugs can cause severe hypotension when combined with nitrates. Which of the following interacts with nitroglycerin by inhibiting the metabolism of cGMP? (A) Atenolol (B) Hydralazine (C) Isosorbide mononitrate (D) Nifedipine (E) Ranolazine (F) Sildenafil (G) Terbutaline ANSWERS 1. Nitroglycerin increases heart rate and venous capacitance and decreases afterload and diastolic fiber tension. It increases cardiac contractile force because the decrease in blood pressure evokes a compensatory increase in sympathetic discharge. The answer is D. 2. The nitrates relax many types of smooth muscle, but the effect on motility in the colon is insignificant. Nitroglycerin causes hypotension as a result of arterial and venous dilation. Dilation of arteries in the meninges has no effect on central nervous system function but does cause headache. The answer is D. 3. The calcium channel blockers and the β blockers are generally effective in reducing the number of attacks of angina of effort, and most have durations of 4–8 h. Oral and transdermal nitrates have similar or longer durations. Amyl nitrite and the sublingual nitrates have short durations of action (a few minutes to 30 min). Esmolol (an intravenous β blocker) must be given intravenously and also has a very short duration of action. These drugs are of no value in prophylaxis. The answer is E. 4. Propranolol blocks tachycardia but has none of the other effects listed. Only revascularization increases double product; drugs that decrease cardiac work increase exercise time by decreasing double product. The answer is A. 5. Both drugs cause venodilation and reduce venous return sufficiently to cause some degree of postural hypotension. Bradycardia, lupus, weight gain, and urinary retention occur with neither of them, but prazosin has been used to relieve urinary retention in men with prostatic hyperplasia. The answer is D. 6. Read carefully! Nitrites, not nitrates, cause methemoglobinemia in adults. Methemoglobinemia is delibrately induced in one of the treatments of cyanide poisoning. The answer is A. 7. Isosorbide dinitrate (like all the nitrates) and prazosin can cause reflex tachycardia. Amlodipine, a dihydropyridine calcium channel blocker, causes much more vasodilation than cardiac depression and may also cause reflex tachycardia. Verapamil typically slows heart rate and high doses may cause severe bradycardia. The answer is E. 8. Acute migraine headache is associated with vasodilation of meningeal arteries. Of the drugs listed, only nitroglycerin is commonly associated with headache. In fact, calcium channel blockers and β blockers have been used with some success as prophylaxis for migraine. The answer is D. 9. The effects of β blockers (or calcium channel blockers) and nitrates on heart size, force, venous tone, and heart rate are opposite. The effects of β blockers and calcium channel blockers on the variables specified here are the same. The answer is D. 10. Sildenafil inhibits phosphodiesterase 5, an enzyme that inactivates cGMP. The nitrates (via nitric oxide) increase the synthesis of cGMP. This combination is synergistic. The answer is F. SKILL KEEPER ANSWER: NIFEDIPINE CARDIOTOXICITY (SEE CHAPTER 6) Several studies have suggested that patients receiving prompt-release nifedipine may have an increased risk of myocardial infarction. Slow-release formulations do not seem to impose this risk. These observations have been explained as follows: Rapid-acting vasodilators—such as nifedipine in its prompt-release formulation—cause significant and sudden reduction in blood pressure. The drop in blood pressure evokes increased sympathetic outflow to the cardiovascular system and increases heart rate and force of contraction by the mechanism shown in Figure 6–4. These changes can markedly increase cardiac oxygen requirement. If coronary blood flow does not increase sufficiently to match the increased requirement, ischemia and infarction can result. CHECKLIST When you complete this chapter, you should be able to: ❑ Describe the pathophysiology of effort angina and vasospastic angina and the major determinants of cardiac oxygen consumption. ❑ List the strategies and drug targets for relief of anginal pain. ❑ Contrast the therapeutic and adverse effects of nitrates, β blockers, and calcium channel blockers when used for angina. ❑ Explain why the combination of a nitrate with a β blocker or a calcium channel blocker may be more effective than either alone. ❑ Explain why the combination of a nitrate and sildenafil is potentially dangerous. ❑ Contrast the effects of medical therapy and surgical therapy of angina. CHAPTER 12 Drugs Used in the Treatment of Angina Pectoris 111 DRUG SUMMARY TABLE: Drugs Used in Angina Subclass Mechanism of Action Clinical Applications Pharmacokinetics Toxicities, Interactions Releases nitric oxide (NO), increases cGMP (cyclic guanosine monophosphate), and relaxes vascular smooth muscle Acute angina pectoris • acute coronary syndrome Rapid onset (1 min) • short duration (15 min) Tachycardia, orthostatic hypotension, headache Slow onset • Duration: 2–4 h Same as nitroglycerin SL Short-acting nitrate Nitroglycerin, sublingual (SL) Isosorbide dinitrate (SL): like nitroglycerin SL but slightly longer acting (20–30 min) Intermediate-acting nitrate Nitroglycerin, oral Like nitroglycerin SL • active metabolite dinitroglycerin Prophylaxis of angina Isosorbide dinitrate and mononitrate, oral: like nitroglycerin oral Pentaerythritol tetranitrate and other oral nitrates: like nitroglycerin oral Long-acting nitrate Transdermal nitroglycerin Like nitroglycerin oral Prophylaxis of angina Slow onset • long duration of absorption: 24 h • duration of effect: 10 h (tachyphylaxis) Same as nitroglycerin SL • loss of response is common after 10–12 h exposure to drug Same as nitroglycerin SL Obsolete for angina • some recreational use Volatile liquid, vapors are inhaled • onset seconds Duration: 1–5 min Same as nitroglycerine SL Blocks L-type Ca2+ channels in smooth muscle and heart • decreases intracellular Ca2+ Angina (both atherosclerotic and vasospastic), hypertension • AV-nodal arrhythmias; migraine Oral, parenteral Duration: 6–8 h Constipation, pretibial edema, flushing, dizziness • Higher doses: cardiac depression, hypotension Angina, hypertension Oral • slow-release form Duration: 6–8 h Like verapamil • less constipation, cardiac effect Oral, parenteral Duration: 6 h See Chapter 10 Ultrashort-acting nitrite Amyl nitrite Calcium channel blockers Verapamil Diltiazem: like verapamil; shorter half-life Nifedipine Dihydropyridine Ca2+ channel blocker; vascular > cardiac effect Amlodipine, felodipine, nicardipine, nisoldipine: like nifedipine but longer acting Beta blockers Propranolol Blocks sympathetic effects on heart and blood pressure • reduces renin release Angina, hypertension, arrhythmias, migraine, performance anxiety Atenolol, metoprolol, other β blockers: like propranolol; most have longer duration of action Other antianginal drugs Ranolazine Blocks late Na+ current in myocardium, reduces cardiac work Angina Oral Duration: 10–12 h QT prolongation on ECG • inhibits CYP3A and 2D6 Ivabradine Blocks pacemaker Na+ current (If) in sinoatrial node, reduces heart rate Investigational: angina, heart failure Oral, administered twice daily Unknown Erectile dysfunction in men Oral Duration: hours Interaction with nitrates • priapism Drugs for erectile dysfunction Sildenafil, tadalafil, vardenafil Block phosphodiesterase 5 • increase cGMP C H A P T E R 13 Drugs Used in Heart Failure Heart failure results when cardiac output is inadequate for the needs of the body. A defect in cardiac contractility is complicated by multiple compensatory processes that further weaken the failing heart. The drugs used in heart failure fall into 3 major groups with varying targets and actions. Drugs used in heart failure Positive inotropic drugs Cardiac glycosides (digoxin) Beta agonists (dobutamine) PDE inhibitors (milrinone) Vasodilators Miscellaneous drugs for chronic failure Nitroprusside Nitrates Hydralazine Loop diuretics ACE inhibitors Nesiritide PATHOPHYSIOLOGY Heart failure is an extremely serious cardiac condition associated with high mortality. The fundamental physiologic defect in heart failure is a decrease in cardiac output relative to the needs of the body, and the major manifestations are dyspnea and fatigue. The causes of heart failure are still not completely understood. In some cases, it can be ascribed to simple loss of functional myocardium, as in myocardial infarction. It is frequently associated with chronic hypertension, valvular disease, coronary artery disease, and a variety of cardiomyopathies. About one third of cases are due to a reduction of cardiac contractile force and ejection fraction (systolic failure). Another third is caused by stiffening or other changes of the ventricles that prevent adequate filling during diastole; ejection fraction may be normal (diastolic failure). The remainder of cases can be attributed to a combination of systolic and diastolic dysfunction. The natural history of heart failure is characterized by a slow deterioration of cardiac function, punctuated by episodes of acute cardiac decompensation that are often associated with pulmonary or peripheral edema or both (congestive heart failure). 112 Beta blockers Spironolactone The reduction in cardiac output is best shown by the ventricular function curve (Frank-Starling curve; Figure 13–1). The changes in the ventricular function curve reflect some compensatory responses of the body and demonstrate some of the responses to drugs. As ventricular ejection decreases, the end-diastolic fiber length increases, as shown by the shift from point A to point B in Figure 13–1. Operation at point B is intrinsically less efficient than operation at shorter fiber lengths because of the increase in myocardial oxygen requirement associated with increased fiber tension and length (see Figure 12–1). The homeostatic responses to depressed cardiac output are extremely important and are mediated mainly by the sympathetic nervous system and the renin-angiotensin-aldosterone system. They are summarized in Figure 13–2. Increased blood volume results in edema and pulmonary congestion and contributes to the increased end-diastolic fiber length. Cardiomegaly (enlargement and remodeling of the heart)—a slower compensatory response, mediated at least in part by sympathetic discharge and angiotensin II, is common. Although these compensatory responses can temporarily improve cardiac output (point C in Figure 13–1), they also increase the load on the heart, and the increased load contributes to further CHAPTER 13 Drugs Used in Heart Failure 113 High-Yield Terms to Learn End-diastolic fiber length The length of the ventricular fibers at the end of diastole; a determinant of the force of the following contraction and of oxygen requirement Heart failure A condition in which the cardiac output is insufficient for the needs of the body. Low-output failure may be due to decreased stroke volume with decreased ejection fraction (systolic failure) or decreased filling and preserved ejection fraction (diastolic failure) PDE inhibitor Phosphodiesterase inhibitor; a drug that inhibits one or more enzymes that degrade cAMP (and other cyclic nucleotides). Examples: high concentrations of theophylline, milrinone Premature ventricular beat An abnormal beat arising from a cell below the AV node—often from a Purkinje fiber, sometimes from a ventricular fiber Sodium pump (Na+/K+ ATPase) A transport molecule in the membranes of all vertebrate cells; responsible for the maintenance of normal low intracellular sodium and high intracellular potassium concentrations; it uses ATP to pump these ions against their concentration gradients Sodium-calcium exchanger A transport molecule in the membrane of many cells that pumps one calcium atom outward against its concentration gradient in exchange for three sodium ions moving inward down their concentration gradient Ventricular function curve The graph that relates cardiac output, stroke volume, etc, to filling pressure or end-diastolic fiber length; also known as the Frank-Starling curve Ventricular tachycardia An arrhythmia consisting entirely or largely of beats originating below the AV node Cardiac output Cardiac output or work Normal Carotid sinus firing Renal blood flow Sympathetic discharge Renin release Compensatory response or treatment A Heart failure C Angiotensin II B Force Rate Preload End-diastolic fiber length FIGURE 13–1 Ventricular function (Frank-Starling) curves. The abscissa can be any measure of preload: fiber length, filling pressure, pulmonary capillary wedge pressure, etc. The ordinate is a measure of useful external cardiac work: stroke volume, cardiac output, etc. In heart failure, output is reduced at all fiber lengths, and the heart expands because ejection fraction is decreased or filling pressure is increased (or both). As a result, the heart moves from point A to point B. Compensatory sympathetic discharge or effective treatment allows the heart to eject more blood, and the heart moves to point C on the middle curve. Afterload Remodeling Cardiac output (via compensation) FIGURE 13–2 Compensatory responses that occur in heart failure. These responses play an important role in the progression of the disease. Dashed arrows indicate interactions between the sympathetic and the renin-angiotensin systems. Increased force and rate, and remodeling, are cardiac responses. Increased preload and afterload are vascular and renal responses. (Modified and reproduced, with permission, from Katzung BG, editor: Basic & Clinical Pharmacology, 12th ed. McGraw-Hill, 2012: Fig. 13–2.) 114 PART III Cardiovascular Drugs long-term decline in cardiac function. Apoptosis is a later response, and results in a reduction in the number of functioning myocytes. Evidence suggests that catecholamines, angiotensin II, and aldosterone play a direct role in these changes. THERAPEUTIC STRATEGIES Pharmacologic therapies for heart failure include the removal of retained salt and water with diuretics; reduction of afterload and salt and water retention by means of angiotensin-converting enzyme (ACE) inhibitors; reduction of excessive sympathetic stimulation by means of β blockers; reduction of preload or afterload with vasodilators; and in systolic failure, direct augmentation of depressed cardiac contractility with positive inotropic drugs such as digitalis glycosides. Considerable evidence indicates that angiotensin antagonists, certain β-adrenoceptor blockers, and the aldosterone antagonists spironolactone and eplerenone also have long-term beneficial effects. These drug classes and targets are summarized in Table 13–1. The use of diuretics is discussed in Chapter 15. Current clinical evidence suggests that acute heart failure should be treated with a loop diuretic; if severe, a prompt-acting positive inotropic agent such as a a agonist or phosphodiesterase inhibitor, and vasodilators should be used as required to optimize filling pressures and blood pressure. Chronic failure is best treated with diuretics (often a loop agent plus spironolactone) plus an ACE inhibitor and, if tolerated, a a blocker. Digitalis may be helpful if systolic dysfunction is prominent. Nesiritide, a recombinant form of brain natriuretic peptide, has vasodilating and diuretic properties and has been heavily promoted for use in acute failure. CARDIAC GLYCOSIDES Digitalis glycosides are no longer considered first-line drugs in the treatment of heart failure. However, because they are not discussed elsewhere in this book, we begin our discussion with this group. A. Prototypes and Pharmacokinetics All cardiac glycosides are cardenolides (they include a steroid nucleus and a lactone ring); most also have one or more sugar residues, justifying the glycoside designation. The cardiac glycosides are often called “digitalis” because several come from the digitalis (foxglove) plant. Digoxin is the prototype agent and the only one commonly used in the United States. Digitoxin is a very similar but longer-acting molecule; it also comes from the foxglove plant but is no longer available in the United States. Digoxin has an oral bioavailability of 60–75%, and a half-life of 36–40 h. Elimination is by renal excretion (about 60%) and hepatic metabolism (40%). B. Mechanism of Action Inhibition of Na+/K+ ATPase (the “sodium pump”) of the cell membrane by digitalis is well documented and is considered to be the primary biochemical mechanism of action (Figure 13–3). Inhibition of Na+/K+ ATPase results in a small increase in intracellular sodium. The increased sodium alters the driving force for sodiumcalcium exchange by the exchanger, NCX, so that less calcium is removed from the cell. The increased intracellular calcium is stored in the sarcoplasmic reticulum and upon release increases contractile force. Other mechanisms of action for digitalis have been proposed, but they are probably not as important as inhibition of the ATPase. The consequences of Na+/K+ ATPase inhibition are seen in both the mechanical and the electrical function of the heart. Digitalis also modifies autonomic outflow, and this action has effects on the electrical properties of the heart. C. Cardiac Effects 1. Mechanical effects—The increase in contractility evoked by digitalis results in increased ventricular ejection, decreased end-systolic and end-diastolic size, increased cardiac output, and increased renal perfusion. These beneficial effects permit a decrease in the compensatory sympathetic and renal responses previously described. The decrease in sympathetic tone is especially TABLE 13–1 Drug targets and mechanisms in heart failure. Target or Drug Class Drug Examples Mechanisms Uses in Heart Failure Na /K ATPase inhibitors Digoxin Increases Cai, increases cardiac contractility Chronic failure Renal sodium transporter inhibitors Furosemide, spironolactone, other diuretics Reduce preload and afterload Acute and chronic failure ACE inhibitors Captopril, others Reduce preload and afterload, reduce remodeling, other Chronic failure Beta adrenoceptor antagonists Carvedilol, others Reduce afterload, reduce remodeling, other Chronic stable failure Beta adrenoceptor agonists Dobutamine, dopamine Increase Cai, increase contractility Acute failure Vasodilators Nitroprusside Reduce preload and afterload Acute failure Phosphodiesterase inhibitors Milrinone Vasodilation, increase contractility Acute failure Natriuretic peptide Nesiritide Vasodilation reduces preload and afterload; some diuretic effect Acute failure + + CHAPTER 13 Drugs Used in Heart Failure 115 Digoxin – Interstitium Cell membrane Na+/K+ Cav–L NCX – ATP + K Cytoplasm Ca2+channel blockers Na+ Ca2+ Trigger Ca2+ SERCA ATP CalS CalS Ca2+ Sarcoplasmic reticulum Ca2+ Ca2+ CalS CalS RyR CalS ATP Ca2+sensitizers Ca2+ Ca2+ + Actin-tropomyosin-troponin Myosin Z line Sarcomere FIGURE 13–3 Schematic diagram of a cardiac sarcomere with the cellular components involved in excitation-contraction coupling and the sites of action of several drugs. Factors involved in excitation-contraction coupling include Na+/K+ ATPase; Na+/Ca2+ exchanger, NCX; voltage-gated calcium channel (Cav-L); calcium transporter (SERCA) in the wall of the sarcoplasmic reticulum (SR); calcium release channel in the SR, RyR (ryanodine receptor); and the site of calcium interaction with the troponin-tropomyosin system. CalS, calsequestrin, a calcium-binding protein in the SR. (Modified and reproduced, with permission, from Katzung BG, editor: Basic & Clinical Pharmacology, 11th ed. McGraw-Hill, 2009: Fig. 13–1.) beneficial: reduced heart rate, preload, and afterload permit the heart to function more efficiently (point C in Figure 13–1 may approach point A as the function curve approaches normal). 2. Electrical effects—Electrical effects include early cardiac parasympathomimetic responses and later arrhythmogenic actions. They are summarized in Table 13–2. a. Early responses—Increased PR interval, caused by the decrease in atrioventricular (AV) conduction velocity, and flattening of the T wave are common electrocardiogram (ECG) effects. The effects on the atria and AV node are largely parasympathetic (mediated by the vagus nerve) and can be partially blocked by atropine. The increase in the AV nodal refractory period is particularly important when atrial flutter or fibrillation is present because the refractoriness of the AV node determines the ventricular rate in these arrhythmias. The effect of digitalis is to slow ventricular rate. Shortened QT interval, inversion of the T wave, and ST segment depression may occur later. 116 PART III Cardiovascular Drugs TABLE 13–2 Major actions of cardiac glycosides on cardiac electrical function. Tissue Variable Atrial Muscle AV Node Purkinje System, Ventricles Effective refractory period ↓ (PANS) ↑ (PANS) ↓ (Direct) Conduction velocity ↑ (PANS) ↓ (PANS) Negligible Automaticity ↑ (Direct) ↑ (Direct) ↑ (Direct) Electrocardiogram before arrhythmias Negligible ↑ PR interval ↓ QT interval; T-wave inversion; ST-segment depression Arrhythmias Atrial tachycardia, fibrillation AV nodal tachycardia, AV blockade Premature ventricular beats, bigeminy, ventricular tachycardia, ventricular fibrillation AV, atrioventricular; PANS, parasympathomimetic actions; direct, direct membrane actions. b. Toxic responses—Increased automaticity, caused by intracellular calcium overload, is the most important manifestation of digitalis toxicity. Intracellular calcium overload results in delayed afterdepolarizations, which may evoke extrasystoles, tachycardia, or fibrillation in any part of the heart. In the ventricles, the extrasystoles are recognized as premature ventricular beats (PVBs). When PVBs are coupled to normal beats in a 1:1 fashion, the rhythm is called bigeminy (Figure 13–4). D. Clinical Uses 1. Congestive heart failure—Digitalis is the traditional positive inotropic agent used in the treatment of chronic heart failure. However, careful clinical studies indicate that while digitalis may improve functional status (reducing symptoms), it does not prolong life. Other agents (diuretics, ACE inhibitors, vasodilators) may be equally effective and less toxic, and some of these alternative therapies do prolong life (see later discussion). Because the half-lives of cardiac glycosides are long, the drugs accumulate significantly in the body, and dosing regimens must be carefully designed and monitored. 2. Atrial fibrillation—In atrial flutter and fibrillation, it is desirable to reduce the conduction velocity or increase the refractory period of the AV node so that ventricular rate is controlled NSR PVB NSR PVB V6 ST FIGURE 13–4 Electrocardiographic record showing digitalisinduced bigeminy. The complexes marked NSR are normal sinus rhythm beats; an inverted T wave and depressed ST segment are present. The complexes marked PVB are premature ventricular beats. within a range compatible with efficient filling and ejection. The parasympathomimetic action of digitalis often accomplishes this therapeutic objective, although high doses may be required. Alternative drugs for rate control include β blockers and calcium channel blockers, but these drugs have negative inotropic effects. E. Interactions Quinidine causes a well-documented reduction in digoxin clearance and can increase the serum digoxin level if digoxin dosage is not adjusted. Several other drugs have the same effect (amiodarone, verapamil, others), but the interactions with these drugs are not clinically significant. Digitalis toxicity, especially arrhythmogenesis, is increased by hypokalemia, hypomagnesemia, and hypercalcemia. Loop diuretics and thiazides, which are always included in the treatment of heart failure, may significantly reduce serum potassium and thus precipitate digitalis toxicity. Digitalisinduced vomiting may deplete serum magnesium and similarly facilitate toxicity. These ion interactions are important when treating digitalis toxicity. F. Digitalis Toxicity The major signs of digitalis toxicity are arrhythmias, nausea, vomiting, and diarrhea. Rarely, confusion or hallucinations and visual or SKILL KEEPER: MAINTENANCE DOSE CALCULATIONS (SEE CHAPTER 3) Digoxin has a narrow therapeutic window, and its dosing must be carefully managed. The drug’s minimum effective concentration is about 1 ng/mL. About 60% is excreted in the urine; the rest is metabolized in the liver. The normal clearance of digoxin is 7 L/h/70 kg; volume of distribution is 500 L/70 kg; and bioavailability is 70%. If your 70-kg patient’s renal function is only 30% of normal, what daily oral maintenance dosage should be used to achieve a safe plasma concentration of 1 ng/mL? The Skill Keeper Answer appears at the end of the chapter. CHAPTER 13 Drugs Used in Heart Failure endocrine aberrations may occur. Arrhythmias are common and dangerous. Chronic intoxication is an extension of the therapeutic effect of the drug and is caused by excessive calcium accumulation in cardiac cells (calcium overload). This overload triggers abnormal automaticity and the arrhythmias noted in Table 13–2. Severe, acute intoxication caused by suicidal or accidental extreme overdose results in cardiac depression leading to cardiac arrest rather than tachycardia or fibrillation. Treatment of digitalis toxicity includes several steps, as follows. 1. Correction of potassium or magnesium deficiency—Correction of potassium deficiency (caused, eg, by diuretic use) is useful in chronic digitalis intoxication. Mild toxicity may often be managed by omitting 1 or 2 doses of digitalis and giving oral or parenteral K+ supplements. Severe acute intoxication (as in suicidal overdoses) usually causes marked hyperkalemia and should not be treated with supplemental potassium. 2. Antiarrhythmic drugs—Antiarrhythmic drugs may be useful if increased automaticity is prominent and does not respond to normalization of serum potassium. Agents that do not severely impair cardiac contractility (eg, lidocaine or phenytoin) are favored, but drugs such as propranolol have also been used successfully. Severe acute digitalis overdose usually causes marked inhibition of all cardiac pacemakers, and an electronic pacemaker may be required. Antiarrhythmic drugs are dangerous in such patients. 3. Digoxin antibodies—Digoxin antibodies (Fab fragments; Digibind) are extremely effective and should always be used if other therapies appear to be failing. They are effective for poisoning with several cardiac glycosides in addition to digoxin and may save patients who would otherwise die. OTHER DRUGS USED IN CONGESTIVE HEART FAILURE The other major agents used in heart failure include diuretics, ACE inhibitors, β1-selective sympathomimetics, β blockers, phosphodiesterase inhibitors, and vasodilators. A. Diuretics Diuretics are the first-line therapy for both systolic and diastolic failure and are used in heart failure before digitalis and other drugs are considered. Furosemide is a very useful agent for immediate reduction of the pulmonary congestion and severe edema associated with acute heart failure and for moderate or severe chronic failure. Thiazides such as hydrochlorothiazide are sometimes sufficient for mild chronic failure. Clinical studies suggest that, unlike other diuretics, spironolactone and eplerenone (aldosterone antagonist diuretics) have significant long-term benefits and can reduce mortality in chronic failure. Diuretics are discussed in Chapter 15. B. Angiotensin Antagonists These agents have been shown to reduce morbidity and mortality in chronic heart failure. Although they have no direct positive inotropic action, angiotensin antagonists reduce aldosterone secretion, salt and water retention, and vascular resistance (see Chapter 11). They are now considered, along with diuretics, to be first-line drugs 117 for chronic heart failure. The angiotensin receptor blockers (ARBs, eg, losartan) appear to have the same benefits as ACE inhibitors (eg, captopril), although experience with ARBs is not as extensive. C. Beta1-Adrenoceptor Agonists Dobutamine and dopamine are often useful in acute failure in which systolic function is markedly depressed (see Chapter 9). However, they are not appropriate for chronic failure because of tolerance, lack of oral efficacy, and significant arrhythmogenic effects. D. Beta-Adrenoceptor Antagonists Several β blockers (carvedilol, labetalol, metoprolol, Chapter 10) have been shown in long-term studies to slow progression of chronic heart failure. This benefit of β blockers had long been recognized in patients with hypertrophic cardiomyopathy but has also been shown to occur in patients without cardiomyopathy. Nebivolol, a β blocker with vasodilator effects approved for the treatment of hypertension, is investigational in heart failure. Beta blockers are of no value in acute failure and may be detrimental if systolic dysfunction is marked. E. Phosphodiesterase Inhibitors Milrinone is the major representative of this infrequently used group. Theophylline (in the form of its salt, aminophylline) was commonly used for acute failure in the past. These drugs increase cyclic adenosine monophosphate (cAMP) by inhibiting its breakdown by phosphodiesterase and cause an increase in cardiac intracellular calcium similar to that produced by β-adrenoceptor agonists. Phosphodiesterase inhibitors also cause vasodilation, which may be responsible for a major part of their beneficial effect. At sufficiently high concentrations, these agents may increase the sensitivity of the contractile protein system to calcium, but they also cause arrhythmias. These agents should not be used in chronic failure because they have been shown to increase morbidity and mortality. F. Vasodilators Vasodilator therapy with nitroprusside or nitroglycerin is often used for acute severe failure with congestion. The use of these vasodilator drugs is based on the reduction in cardiac size and improved efficiency that can be achieved with proper adjustment of venous return (preload) and reduction of impedance to ventricular ejection (afterload). Vasodilator therapy can be dramatically effective, especially in cases in which increased afterload is a major factor in causing the failure (eg, continuing hypertension in an individual who has just had an infarct). The natriuretic peptide nesiritide acts chiefly by causing vasodilation, although it does have natriuretic effects as well. It is given by IV infusion for acute failure only. Nesiritide has significant renal toxicity and renal function must be monitored. Chronic heart failure sometimes responds favorably to oral vasodilators such as hydralazine or isosorbide dinitrate (or both), and this combination has been shown to reduce mortality due to heart failure in African Americans. Calcium channel blockers (eg, verapamil) are of no value in heart failure. 118 PART III Cardiovascular Drugs G. Nonpharmacologic Therapy A variety of surgical procedures to remove nonfunctional regions of damaged myocardium have been attempted with mixed results. Resynchronization of right and left ventricular contraction by means of a pacemaker has been beneficial in patients with long QRS (indicating conduction abnormalities). Patients with coronary artery disease and heart failure may have improved systolic function after coronary revascularization. QUESTIONS Questions 1–2. A 73-year-old man with an inadequate response to other drugs is to receive digoxin for chronic heart failure. He is in normal sinus rhythm with a heart rate of 88 and blood pressure of 135/85 mm Hg. 1. Which of the following is the best-documented mechanism of beneficial action of cardiac glycosides? (A) A decrease in calcium uptake by the sarcoplasmic reticulum (B) An increase in ATP synthesis (C) A modification of the actin molecule (D) An increase in systolic cytoplasmic calcium levels (E) A block of cardiac β adrenoceptors 2. After your patient has been receiving digoxin for 3 wk, he presents to the emergency department with an arrhythmia. Which one of the following is most likely to contribute to the arrhythmogenic effect of digoxin? (A) Increased parasympathetic discharge (B) Increased intracellular calcium (C) Decreased sympathetic discharge (D) Decreased intracellular ATP (E) Increased extracellular potassium 3. A patient who has been taking digoxin for several years for atrial fibrillation and chronic heart failure is about to receive atropine for another condition. A common effect of digoxin (at therapeutic blood levels) that can be almost entirely blocked by atropine is (A) Decreased appetite (B) Headaches (C) Increased atrial contractility (D) Increased PR interval on ECG (E) Tachycardia 4. A 65-year-old woman has been admitted to the coronary care unit with a left ventricular myocardial infarction. She develops acute severe heart failure with marked pulmonary edema, but no evidence of peripheral edema. Which one of the following drugs would be most useful? (A) Digoxin (B) Furosemide (C) Minoxidil (D) Propranolol (E) Spironolactone 5. A 72-year-old woman has long-standing heart failure. Which one of the following drugs has been shown to reduce mortality in chronic heart failure? (A) Atenolol (B) Digoxin (C) Dobutamine (D) Furosemide (E) Spironolactone 6. Which row in the following table correctly shows the major effects of full therapeutic doses of digoxin on the AV node and the ECG? Row AV Refractory Period QT Interval T Wave (A) Increased Increased Upright (B) Increased Decreased Inverted (C) Decreased Increased Upright (D) Decreased Decreased Upright (E) Decreased Increased Inverted 7. Which one of the following drugs is associated with clinically useful or physiologically important positive inotropic effect? (A) Captopril (B) Dobutamine (C) Enalapril (D) Losartan (E) Nesiritide 8. A 68-year-old man with a history of chronic heart failure goes on vacation and abandons his low-salt diet. Three days later, he develops severe shortness of breath and is admitted to the local hospital emergency department with significant pulmonary edema. The first-line drug of choice in most cases of acute decompensation in patients with chronic heart failure is (A) Atenolol (B) Captopril (C) Carvedilol (D) Digoxin (E) Diltiazem (F) Dobutamine (G) Enalapril (H) Furosemide (I) Metoprolol (J) Spironolactone 9. Which of the following has been shown to prolong life in patients with chronic congestive failure in spite of having a negative inotropic effect on cardiac contractility? (A) Carvedilol (B) Digoxin (C) Dobutamine (D) Enalapril (E) Furosemide CHAPTER 13 Drugs Used in Heart Failure 10. A 5-year-old child was vomiting and was brought to the emergency department with sinus arrest and a ventricular rate of 35 bpm. An empty bottle of his uncle’s digoxin was found where he was playing. Which of the following is the drug of choice in treating a severe overdose of digoxin? (A) Digoxin antibodies (B) Lidocaine infusion (C) Magnesium infusion (D) Phenytoin by mouth (E) Potassium by mouth ANSWERS 1. Digitalis does not decrease calcium uptake by the sarcoplasmic reticulum or increase ATP synthesis; it does not modify actin. Cardiac adrenoceptors are not affected. The most accurate description of digitalis’s mechanism in this list is that it increases systolic cytoplasmic calcium indirectly by inhibiting Na+/K+ ATPase and altering Na/Ca exchange. The answer is D. 2. The effects of digitalis include increased vagal action on the heart (not arrhythmogenic) and increased intracellular calcium, including calcium overload, the most important cause of toxicity. Decreased sympathetic discharge and increased extracellular potassium and magnesium reduce digitalis arrhythmogenesis. The answer is B. 3. The parasympathomimetic effects of digitalis can be blocked by muscarinic blockers such as atropine. The only parasympathomimetic effect in the list provided is increased PR interval, a manifestation of slowed AV conduction. The answer is D. 4. Acute severe congestive failure with pulmonary edema often requires a vasodilator that reduces intravascular pressures in the lungs. Furosemide has such vasodilating actions in the context of acute failure. Pulmonary edema also involves a shift of fluid from the intravascular compartment to the lungs. Minoxidil would decrease arterial pressure and increase the heart rate excessively. Digoxin has a slow onset of action and lacks vasodilating effects. Spironolactone is useful in chronic failure but not in acute pulmonary edema. Pulmonary vasodilation and removal of edema fluid by diuresis are accomplished by furosemide. The answer is B. 5. Of the drugs listed, only spironolactone has been shown to reduce mortality in this highly lethal disease. Digoxin, dobutamine, and furosemide are used in the management of symptoms. The answer is E. 6. Digitalis increases the AV node refractory period—a parasympathomimetic action. Its effects on the ventricles include shortened action potential and QT interval, and a change in repolarization with flattening or inversion of the T wave. The answer is B. 7. Although they are extremely useful in heart failure, ACE inhibitors (eg, captopril, enalapril), and angiotensin receptor blockers (ARBs, eg, losartan) have no positive inotropic effect on the heart. Nesiritide is a vasodilator with diuretic effects and renal toxicity. Dobutamine is a β1-selective adrenoceptor agonist. The answer is B. 8. In both acute and chronic failure and systolic and diastolic heart failure, the initial treatment of choice is usually furosemide. The answer is H. 9. Several β blockers, including carvedilol, have been shown to prolong life in heart failure patients even though these drugs have a negative inotropic action on the heart. Their benefits presumably result from some other effect, and at least one β blocker has failed to show a mortality benefit. The answer is A. 10. The drug of choice in severe, massive overdose with any cardiac glycoside is digoxin antibody, Digibind. The other drugs listed are used in moderate overdosage associated with increased automaticity. The answer is A. SKILL KEEPER ANSWER: MAINTENANCE DOSE CALCULATIONS (SEE CHAPTER 3) Maintenance dosage is equal to CL × Cp ÷ F, so Maintenance dosage for a patient with normal renal function = 7 L/h × 1 ng/mL ÷ 0.7 = 7 L/h × 1 mcg/L ÷ 0.7 = 10 mcg/h × 24 h/d = 240 mcg/d = 0.24 mg/d But this patient has only 30% of normal renal function, so CL (total) = 0.3 × CL (renal [60% of total]) + CL (liver [40% of total]) CL (total) = 0.3 × 0.6 × 7 L/h + 0.4 × 7 L/h, and CL (total) = 1.26 L/h + 2.8 L/h = 4.06 L/h, and Maintenance dosage = 4.06 L/h × 1 mcg/L ÷ 0.7 = 5.8 mcg/h = 139 mcg/d = 0.14 mg/d CHECKLIST When you complete this chapter, you should be able to: ❑ Describe the strategies and list the major drug groups used in the treatment of acute heart failure and chronic failure. ❑ Describe the mechanism of action of digitalis and its major effects. Indicate why digitalis is no longer considered a first-line therapy for chronic heart failure. ❑ Describe the nature and mechanism of digitalis’s toxic effects on the heart. ❑ List positive inotropic drugs other than digitalis that have been used in heart failure. ❑ Explain the beneficial effects of diuretics, vasodilators, ACE inhibitors, and other drugs that lack positive inotropic effects in heart failure. 119 120 PART III Cardiovascular Drugs DRUG SUMMARY TABLE: Drugs Used in Heart Failure Subclass Mechanism of Action Clinical Applications Pharmacokinetics Toxicities, Interactions Furosemide, other loop diuretics Reduces preload, edema by powerful diuretic action on thick ascending limb in nephron • vasodilating effect on pulmonary vessels Acute and chronic heart failure, especially acute pulmonary edema • other edematous conditions, hypercalcemia (see Chapter 15) Oral, parenteral Duration: 2–4 h Ototoxicity • hypovolemia, hypokalemia Spironolactone Antagonist of aldosterone in kidney plus poorly understood reduction in mortality Chronic heart failure, aldosteronism Oral Duration: 24–48 h Hyperkalemia • gynecomastia Oral; short half-life but large doses used Duration: 12–24 h Cough, renal damage, hyperkalemia, contraindicated in pregnancy Diuretics Eplerenone: similar to spironolactone but lacks gynecomastia effect Angiotensin-converting enzyme (ACE) inhibitors and receptor blockers Captopril Blocks angiotensin-converting enzyme, reduces AII levels, decreases vascular tone and aldosterone secretion. Reduces mortality Heart failure, hypertension, diabetes Benazepril, enalapril, others: like captopril Losartan, candesartan, others: angiotensin receptor blockers (see Chapter 11); benefits not documented as well as those of ACE inhibitors Positive inotropic drugs Cardiac glycosides: digoxin Inhibits Na+/K+ ATPase sodium pump and increases intracellular Na+, decreasing Ca2+ expulsion and increasing cardiac contractility Chronic heart failure, nodal arrhythmias Oral, parenteral Duration: 40 h Arrhythmogenic! Nausea, vomiting, diarrhea, visual and endocrine changes (rare) Sympathomimetics: dobutamine Beta1-selective sympathomimetic, increases cAMP and force of contraction Acute heart failure Parenteral Duration: a few minutes Arrhythmias Poorly understood reduction of mortality, possibly by decreasing remodeling Chronic heart failure Oral Duration varies (see Chapter 10) Cardiac depression (see Chapter 10) Nitroprusside Rapid, powerful vasodilation reduces preload and afterload Acute severe decompensated failure IV infusion Duration: a few minutes Excessive hypotension • thiocyanate and cyanide toxicity Hydralazine + isosorbide dinitrate Poorly understood reduction in mortality Chronic failure in African Americans Oral Headache, tachycardia Nesiritide Atrial peptide vasodilator, diuretic Acute severe decompensated failure Parenteral Duration: a few minutes Renal damage, hypotension Beta blockers Carvedilol, metoprolol, bisoprolol Vasodilators cAMP, cyclic adenosine monophosphate. C A P T E R 14 Antiarrhythmic Drugs Cardiac arrhythmias are the most common cause of death in patients with a myocardial infarction or terminal heart failure. They are also the most serious manifestation of digitalis toxicity and are often associated with anesthetic procedures, hyperthyroidism, and electrolyte disorders. The drugs used H for arrhythmias fall into five major groups or classes, but most have very low therapeutic indices and when feasible, nondrug therapies (cardioversion, pacemakers, ablation, implanted defibrillators) are used. Drugs used in cardiac arrhythmias Group 1 Sodium channel blockers (procainamide) Group 2 β blockers (esmolol) Group 3 Potassium channel blockers (amiodarone, dofetilide) PATHOPHYSIOLOGY A. Nature of Arrhythmias Normal electrical cardiac function (normal sinus rhythm, NSR) is dependent on generation of an impulse in the normal sinoatrial (SA) node pacemaker and its conduction through the atrial muscle, through the atrioventricular (AV) node, through the Purkinje conduction system, to the ventricular muscle (Figure 14–1) where it is finally extinguished after activating all the myocytes. A new impulse must arise in the SA node for the next conducted action potential. Normal pacemaking and conduction require normal action potentials (dependent on sodium, calcium, and potassium channel activity) under appropriate autonomic control. Arrhythmias (also called dysrhythmias) are therefore defined by exclusion, that is, an arrhythmia is any cardiac rhythm that is not normal sinus rhythm. Abnormal automaticity and abnormal conduction are the 2 major mechanisms for arrhythmias. Abnormalities of conduction include reentrant conduction and less commonly, complete block. A few of the clinically important arrhythmias are atrial flutter, atrial fibrillation (AFib), atrioventricular nodal reentry (a Group 4 Calcium channel blockers (verapamil) Group 5 Miscellaneous group (adenosine, K+, Mg2+) common type of supraventricular tachycardia [SVT]), premature ventricular beats (PVBs), ventricular tachycardia (VT), and ventricular fibrillation (VF). Examples of electrocardiographic (ECG) recordings of normal sinus rhythm and some of these common arrhythmias are shown in Figure 14–2. Torsades de pointes is a ventricular arrhythmia of great pharmacologic importance because it is often induced by antiarrhythmic and other drugs that change the shape of the action potential and prolong the QT interval. It has the ECG morphology of a polymorphic ventricular tachycardia, often displaying waxing and waning QRS amplitude. Torsades is also associated with long QT syndrome, a heritable abnormal prolongation of the QT interval caused by mutations in the IK or INa channel proteins. B. Normal Electrical Activity in the Cardiac Cell The cellular action potentials shown in Figure 14–1 are the result of ion fluxes through voltage-gated channels and carrier mechanisms. These processes are diagrammed in Figure 14–3. In most parts of the heart, sodium channel current (INa) dominates the upstroke (phase 0) of the action potential (AP) and is the most 121 122 PART III Cardiovascular Drugs High-Yield Terms to Learn Abnormal automaticity Pacemaker activity that originates anywhere other than in the sinoatrial node Abnormal conduction Conduction of an impulse that does not follow the path defined in Figure 14–1 or reenters tissue previously excited Arrhythmias involving rapid reentry and chaotic movement of impulses through the tissue of the atria or ventricles. Ventricular, but not atrial, fibrillation is fatal if not terminated within a few minutes A method for classifying antiarrhythmic drugs, sometimes called the Singh-Vaughan Williams classification; based loosely on the channel or receptor affected Arrhythmias of abnormal conduction; they involve the repetitive movement of an impulse through tissue previously excited by the same impulse The time that must pass after the upstroke of a conducted impulse in a part of the heart before a new action potential can be propagated in that cell or tissue The ability of certain drugs to selectively depress areas of excitable membrane that are most susceptible, leaving other areas relatively unaffected A reentrant arrhythmia that travels through the AV node; it may also be conducted through atrial tissue as part of the reentrant circuit Atrial, ventricular fibrillation (AFib, VF) Group (class) 1, 2, 3, and 4 drugs Reentrant arrhythmias Effective refractory period Selective depression Supraventricular tachycardia (SVT) Ventricular tachycardia (VT) A very common arrhythmia, often associated with myocardial infarction; ventricular tachycardia may involve abnormal automaticity or abnormal conduction, usually impairs cardiac output, and may deteriorate into ventricular fibrillation; for these reasons it requires prompt management Superior vena cava Phase 0 3 4 SA node Atrium AV node Overshoot 1 2 0 Phase 0 mV 3 4 Purkinje –100 Tricuspid valve Resting potential Mitral valve Action potential phases 0: Upstroke 1: Early-fast repolarization 2: Plateau 3: Repolarization 4: Diastole Ventricle R T ECG P Q S PR QT 200 ms FIGURE 14–1 Schematic representation of the heart and normal cardiac electrical activity (intracellular recordings from areas indicated and ECG). The ECG is the body surface manifestation of the depolarization and repolarization waves of the heart. The P wave is generated by atrial depolarization, the QRS by ventricular muscle depolarization, and the T wave by ventricular repolarization. The PR interval is a measure of conduction time from atrium to ventricle through the atrioventricular (AV) node, and the QRS duration indicates the time required for all of the ventricular cells to be activated (ie, the intraventricular conduction time). The QT interval reflects the duration of the ventricular action potential. SA, sinoatrial. (Reproduced, with permission, from Katzung BG, editor: Basic & Clinical Pharmacology, 12th ed. McGraw-Hill, 2012: Fig. 14–1.) CHAPTER 14 Antiarrhythmic Drugs P R Panel 1: Normal sinus rhythm aVF Panel 2: Atrial flutter V2 T P' P' P' R T T P' P' P' V1 Panel 3: S S Atrial fibrillation V 1 Before digitalis S S S After digitalis R R QS 1. 2. 3. 4. Sodium channel blockers Beta-adrenoceptor blockers Potassium channel blockers Calcium channel blockers The miscellaneous group includes adenosine, potassium ion, and magnesium ion. R Panel 4: V1 Ventricular tachycardia (starting at arrow) Panel 5: Ventricular fibrillation (sodium pump and sodium–calcium exchanger) contribute little to the shape of the AP, but they are critical for the maintenance of the ion gradients on which the sodium, calcium, and potassium currents depend. Most antiarrhythmic drugs act on 1 or more of the 3 major currents (INa, ICa, IK) or on the β adrenoceptors that modulate these currents. C. Drug Classification The antiarrhythmic agents are usually classified using a system loosely based on the channel or receptor involved. As indicated by the overview figure on the first page of this chapter, this system specifies 4 groups or classes, usually denoted by the numerals 1 through 4, plus a miscellaneous group (see also Table 14–1 and Drug Summary Table). S T 123 GROUP 1 ANTIARRHYTHMICS (SODIUM CHANNEL BLOCKERS) QS T T T V4 FIGURE 14–2 Typical ECGs of normal sinus rhythm and some common arrhythmias. Major waves (P, Q, R, S, and T) are labeled in each electrocardiographic record except in panel 5, in which electrical activity is completely disorganized and none of these deflections are recognizable. (Modified and reproduced, with permission, from Goldman MJ: Principles of Clinical Electrocardiography, 11th ed. McGraw-Hill, 1982.) important determinant of its conduction velocity. After a very brief activation, most sodium channels enter a more prolonged period of inactivation. In the calcium-dependent AV node, calcium current (ICa) dominates the upstroke and the AP conduction velocity. The plateau of the AP (phase 2) is dominated by a depolarizing calcium current (ICa) and several repolarizing potassium currents (collectively referred to as IK). At the end of the plateau, IK causes rapid repolarization (phase 3). The refractory period of the sodium-dependent cardiac cells is a function of how rapidly sodium channels recover from inactivation. Recovery from inactivation depends on both the membrane potential, which varies with repolarization time and the extracellular potassium concentration, and on the actions of drugs that bind to the sodium channel (ie, sodium channel blockers). Similarly, in the calcium-dependent AV node, the duration of refractoriness is dependent on the rate of recovery from inactivation of the calcium channels. The carrier processes A. Prototypes and Mechanism of Action The group 1 drugs have local anesthetic actions and slow the upstroke of sodium-dependent action potentials and prolong QRS duration. They are further subdivided on the basis of their effects on AP duration (Figure 14–4). Group 1A agents (prototype procainamide) prolong the AP. Group 1B drugs (prototype lidocaine) shorten the AP in some cardiac tissues. Group 1C drugs (prototype flecainide) have no effect on AP duration. All group 1 drugs slow conduction in ischemic and depolarized cells and slow or abolish abnormal pacemakers wherever these processes depend on sodium channels. The most selective agents (those in group 1B) have significant effects on sodium channels in depressed ischemic tissue, but negligible effects on channels in normal cells. In contrast, less selective group 1 drugs (groups 1A and 1C) cause significant reduction of INa in depressed tissue and less blockade in normal cells. Useful sodium channel-blocking drugs bind to their receptors readily when the channel is open or inactivated and much less readily when it is fully repolarized and resting. Therefore, these antiarrhythmic drugs block channels in abnormal tissue more effectively than channels in normal tissue. They are use dependent or state dependent in their action (ie, they selectively depress tissue that is frequently depolarizing, eg, during a fast tachycardia; or tissue that is relatively depolarized during rest, eg, by ischemia). The effects of the major group 1 drugs are summarized in Table 14–1 and in Figure 14–4. 1. Drugs with group 1A action—Procainamide is a group 1A prototype. Other drugs with group 1A actions include quinidine and disopyramide. Amiodarone, often classified in group 3, also has typical group 1A actions. These drugs affect both atrial 124 PART III Cardiovascular Drugs Phase 2 (ICa and IK) 0 mV Phase 3 (IK) Phase 0 (INa) 100 ms Effective refractory period (ERP) Pacemaker Phase 4 (IK, also INa, ICa ) −85 mV Nonpacemaker Ca Na Na Outside Membrane Na ATP Inside K Sodium pump K Action potential currents Ca Na/Ca exchanger Na Ca K Diastolic currents FIGURE 14–3 Components of the membrane action potential (AP) in a typical Purkinje or ventricular cardiac cell. The deflections of the AP, designated as phases 0–3, are generated by several ionic currents. The actions of the sodium pump and sodium–calcium exchanger are mainly involved in maintaining ionic steady state during repetitive activity. Note that small but significant currents occur during diastole (phase 4) in addition to the pump and exchanger activity. In non-pacemaker cells, the outward potassium current during phase 4 is sufficient to maintain a stable negative resting potential as shown by the solid line at the right end of the tracing. In pacemaker cells, however, the potassium current is smaller and the depolarizing currents (sodium, calcium, or both) during phase 4 are large enough to gradually depolarize the cell during diastole (dashed line). ATP, adenosine triphosphate. and ventricular arrhythmias. They block INa and therefore slow conduction velocity in the atria, Purkinje fibers, and ventricular cells. At high doses they may slow AV conduction. These effects are summarized in Table 14–1. Amiodarone has similar effects on sodium current (INa block) and has the greatest AP-prolonging effect (IK block). 2. Drugs with group 1B actions—Lidocaine is the prototype 1B drug and is used exclusively by the IV or IM routes. Mexiletine is an orally active 1B agent. These drugs selectively affect ischemic or depolarized Purkinje and ventricular tissue and have little effect on atrial tissue; the drugs reduce AP duration in some cells, but because they slow recovery of sodium channels from inactivation, they do TABLE 14–1 Properties of the prototype antiarrhythmic drugs. Drug Group PR Interval QRS Duration QT Interval Procainamide, disopyramide, quinidine 1A ↑ or ↓a ↑↑ ↑↑ b Lidocaine, mexiletine 1B — — —, ↓c Flecainide 1C ↑ (slight) ↑↑ — Propranolol, esmolol 2 ↑↑ — — Amiodarone 3, 1A, 2, 4 ↑ ↑↑ ↑↑↑↑ Ibutilide, dofetilide 3 — — ↑↑↑ Sotalol 3, 2 ↑↑ — ↑↑↑ Verapamil 4 ↑↑ — — Adenosine Misc ↑↑↑ — — a PR interval may decrease owing to antimuscarinic action or increase owing to channel-blocking action. b Lidocaine, mexiletine, and some other group 1B drugs slow conduction through ischemic, depolarized ventricular cells but not in normal tissue. c Decreased QT in Purkinje cells. CHAPTER 14 Antiarrhythmic Drugs All group 1 drugs Group 1A Group 1B 0 mV Group 1C Phase 0 (INa) Phase 3 (IK) ERP −85 mV Outside Na Ca All group 1 drugs Membrane Inside All group 1 drugs K Action potential currents K Na Ca Diastolic currents FIGURE 14–4 Schematic diagram of the effects of group 1 agents. Note that all group 1 drugs reduce both phase 0 and phase 4 sodium currents (wavy lines) in susceptible cells. Group 1A drugs also reduce phase 3 potassium current (IK) and prolong the action potential (AP) duration. This results in significant prolongation of the effective refractory period (ERP). Group 1B and group 1C drugs have different (or no) effects on potassium current and shorten or have no effect on the AP duration. However, all group 1 drugs prolong the ERP by slowing recovery of sodium channels from inactivation. not shorten (and may even prolong) the effective refractory period. Because these agents have little effect on normal cardiac cells, they have little effect on the ECG (Table 14–1). Phenytoin, an anticonvulsant and not a true local anesthetic, is sometimes classified with the group 1B antiarrhythmic agents because it can be used to reverse digitalis-induced arrhythmias. It resembles lidocaine in lacking significant effects on the normal ECG. 3. Drugs with group 1C action—Flecainide is the prototype drug with group 1C actions. Other members of this group are used outside the United States and may be available in this country in special circumstances. These drugs have no effect on ventricular AP duration or the QT interval. They are powerful depressants of sodium current, however, and can markedly slow conduction velocity in atrial and ventricular cells. They increase the QRS duration of the ECG. B. Pharmacokinetics, Clinical Uses, and Toxicities Pharmacokinetics of the major drugs are listed in the Drug Summary Table at the end of the chapter. 1. Group 1A drugs—Procainamide can be used in all types of arrhythmias: atrial and ventricular arrhythmias are most responsive. Quinidine and disopyramide have similar effects but are used much less frequently. Procainamide is also commonly used in arrhythmias during the acute phase of myocardial infarction. Procainamide may cause hypotension and chronic use may cause a reversible syndrome similar to lupus erythematosus. Quinidine causes cinchonism (headache, vertigo, tinnitus); cardiac 125 depression; gastrointestinal upset; and autoimmune reactions (eg, thrombocytopenic purpura). As noted in Chapter 13, quinidine reduces the clearance of digoxin and may increase the serum concentration of the glycoside significantly. Disopyramide has marked antimuscarinic effects and may precipitate heart failure. All group 1A drugs may precipitate new arrhythmias. Torsades de pointes is particularly associated with quinidine and other drugs that prolong AP duration (except amiodarone). The toxicities of amiodarone are discussed in the following text. Hyperkalemia usually exacerbates the cardiac toxicity of group 1 drugs. Treatment of overdose with these agents is often carried out with sodium lactate (to reverse drug-induced arrhythmias) and pressor sympathomimetics (to reverse drug-induced hypotension) if indicated. 2. Group 1B drugs—Lidocaine is useful in acute ischemic ventricular arrhythmias, for example, after myocardial infarction. Atrial arrhythmias are not responsive unless caused by digitalis. Mexiletine has similar actions and is given orally for chronic arrhythmias and for certain types of neuropathic pain. Lidocaine is usually given intravenously, but intramuscular administration is also possible. It is never given orally because it has a very high first-pass effect and its metabolites are potentially cardiotoxic. Lidocaine and mexiletine occasionally cause typical local anesthetic toxicity (ie, central nervous system [CNS] stimulation, including convulsions); cardiovascular depression (usually minor); and allergy (usually rashes but may rarely extend to anaphylaxis). These drugs may also precipitate arrhythmias, but this is much less common than with group 1A drugs. Hyperkalemia increases cardiac toxicity. 3. Group 1C drugs—Flecainide is effective in both atrial and ventricular arrhythmias but is approved only for refractory ventricular tachycardias and for certain intractable supraventricular arrhythmias. Flecainide and its congeners are more likely than other antiarrhythmic drugs to exacerbate or precipitate arrhythmias (proarrhythmic effect). This toxicity was dramatically demonstrated by the Cardiac Arrhythmia Suppression Trial (CAST), a large clinical trial of the prophylactic use of group 1C drugs in myocardial infarction survivors. The trial results showed that group 1C drugs caused greater mortality than placebo. For this reason, the group 1C drugs are now restricted to use in persistent arrhythmias that fail to respond to other drugs. Group 1C drugs also cause local anesthetic-like CNS toxicity. Hyperkalemia increases the cardiac toxicity of these agents. GROUP 2 ANTIARRHYTHMICS (BETA BLOCKERS) A. Prototypes, Mechanisms, and Effects Beta blockers are discussed in more detail in Chapter 10. Propranolol and esmolol are prototypic antiarrhythmic β blockers. Their mechanism in arrhythmias is primarily cardiac β-adrenoceptor blockade and reduction in cAMP, which results in a modest reduction of both sodium and calcium currents and the 126 PART III Cardiovascular Drugs suppression of abnormal pacemakers. The AV node is particularly sensitive to β blockers and the PR interval is usually prolonged by group 2 drugs (Table 14–1). Under some conditions, these drugs may have some direct local anesthetic (sodium channel-blocking) effect in the heart, but this is probably rare at the concentrations achieved clinically. Sotalol and amiodarone, generally classified as group 3 drugs, also have group 2 β-blocking effects. Group 3 action 0 mV Phase 3 (IK) ERP B. Clinical Uses and Toxicities Esmolol, a very short-acting β blocker for intravenous administration, is used exclusively in acute arrhythmias. Propranolol, metoprolol, and timolol are commonly used as prophylactic drugs in patients who have had a myocardial infarction. −85 mV Na Describe the important subgroups of β blockers and their major pharmacokinetic and pharmacodynamic features. The Skill Keeper Answer appears at the end of the chapter. The toxicities of β blockers are the same in patients with arrhythmias as in patients with other conditions (Chapter 10 and Drug Summary Table). While patients with arrhythmias are often more prone to β-blocker-induced depression of cardiac output than are patients with normal hearts, it should be noted that judicious use of these drugs reduces progression of chronic heart failure (Chapter 13) and reduces the incidence of potentially fatal arrhythmias in this condition. GROUP 3 ANTIARRHYTHMICS (POTASSIUM IK CHANNEL BLOCKERS) A. Prototypes, Mechanisms, and Effects Dofetilide and ibutilide are typical group 3 drugs. Sotalol is a chiral compound (ie, it has 2 optical isomers). One isomer is an effective β blocker, and both isomers contribute to the antiarrhythmic action. The clinical preparation contains both isomers. Amiodarone is usually classified as a group 3 drug because it blocks the same K channels and markedly prolongs AP duration as well as blocking other channels and β receptors. Dronedarone is similar to amiodarone but less efficacious and less toxic. The hallmark of group 3 drugs is prolongation of the AP duration. This AP prolongation is caused by blockade of IK potassium channels, chiefly IKr, that are responsible for the repolarization of the AP (Figure 14–5). AP prolongation results in an increase in effective refractory period and reduces the ability of the heart to respond to rapid tachycardias. Sotalol, ibutilide, dofetilide, and amiodarone (and group 1A drugs; see prior discussion) produce this effect on most cardiac cells; the action of these drugs is, therefore, apparent in the ECG mainly as an increase in QT interval (Table 14–1). Group 3 action Membrane Inside SKILL KEEPER: CHARACTERISTICS OF a BLOCKERS (SEE CHAPTER 10) Ca Outside K Action potential currents K1 Na Ca Diastolic currents FIGURE 14–5 Schematic diagram of the effects of group 3 agents. All group 3 drugs prolong the AP duration in susceptible cardiac cells by reducing the outward (repolarizing) phase 3 potassium current (IK, wavy lines). The main effect is to prolong the effective refractory period (ERP). Note that the phase 4 diastolic potassium current (IK) is not affected by these drugs. B. Clinical Uses and Toxicities See the Drug Summary Table. C. Amiodarone: A Special Case Amiodarone is useful in most types of arrhythmias and is considered the most efficacious of all antiarrhythmic drugs. This may be because it has a broad spectrum of action: It blocks sodium, calcium, and potassium channels and β adrenoceptors. Because of its toxicities, however, amiodarone is approved for use mainly in arrhythmias that are resistant to other drugs. Nevertheless, it is used very extensively, off label, in a wide variety of arrhythmias because of its superior efficacy. Amiodarone causes microcrystalline deposits in the cornea and skin, thyroid dysfunction (hyper- or hypothyroidism), paresthesias, tremor, and pulmonary fibrosis. Amiodarone rarely causes new arrhythmias, perhaps because it blocks calcium channels and β receptors as well as sodium and potassium channels. Dronedarone, an amiodarone analog that may be less toxic, is also approved. Like amiodarone, it acts on sodium, potassium, and calcium channels, but at present it is approved only for the treatment of atrial fibrillation or flutter. GROUP 4 ANTIARRHYTHMICS (CALCIUM L-TYPE CHANNEL BLOCKERS) A. Prototypes, Mechanisms, and Effects Verapamil is the prototype. Diltiazem is also an effective antiarrhythmic drug. Nifedipine and the other dihydropyridines are not useful as antiarrhythmics, probably because they decrease arterial pressure CHAPTER 14 Antiarrhythmic Drugs 127 Phase 2 (lCa and IK) Group 4 action 0 mV Phase 0 Note ICa ERP −75 mV Ca Group 4 action Na Outside Membrane Inside K Action potential currents K Na Ca Diastolic currents FIGURE 14–6 Schematic diagram of the effects of group 4 drugs in a calcium-dependent cardiac cell in the AV node (note that the AP upstroke in this figure is due mainly to calcium current). Group 4 drugs reduce inward calcium current during the AP and during phase 4 (wavy lines). As a result, conduction velocity is slowed in the AV node and refractoriness is prolonged. Pacemaker depolarization during phase 4 is slowed as well if caused by excessive calcium current. ERP, effective refractory period. enough to evoke a compensatory sympathetic discharge to the heart. The latter effect facilitates rather than suppresses arrhythmias. Verapamil and diltiazem are effective in arrhythmias that must traverse calcium-dependent cardiac tissue such as the AV node. These agents cause a state- and use-dependent selective depression of calcium current (Figure 14–6). AV conduction velocity is decreased, and effective refractory period and PR interval are increased by these drugs (Table 14–1). B. Clinical Use and Toxicities Calcium channel blockers are effective for converting AV nodal reentry (also known as nodal tachycardia) to normal sinus rhythm. Their major use is in the prevention of these nodal arrhythmias in patients prone to recurrence. These drugs are available for oral and parenteral use (see Drug Summary Table). The most important toxicity of these drugs is excessive depression of cardiac contractility, AV conduction, and blood pressure. These agents should be avoided in ventricular tachycardias. See Chapter 12 for additional discussion of toxicity. Amiodarone has moderate calcium channelblocking activity. MISCELLANEOUS ANTIARRHYTHMIC DRUGS A. Adenosine Adenosine is a normal component of the body, but when given in high doses (6–12 mg) as an intravenous bolus, the drug markedly slows or completely blocks conduction in the atrioventricular node (Table 14–1), probably by hyperpolarizing this tissue (through increased IK) and by reducing calcium current. Adenosine is extremely effective in abolishing AV nodal arrhythmia, and because of its very low toxicity it has become the drug of choice for this arrhythmia. Adenosine has an extremely short duration of action (about 15 s). Toxicity includes flushing and hypotension, but because of their short duration these effects do not limit the use of the drug. Transient chest pain and dyspnea (probably due to bronchoconstriction) may also occur. B. Potassium Ion Potassium depresses ectopic pacemakers, including those caused by digitalis toxicity. Hypokalemia is associated with an increased incidence of arrhythmias, especially in patients receiving digitalis. Conversely, excessive potassium levels depress conduction and can cause reentry arrhythmias. Therefore, when treating arrhythmias, serum potassium should be measured and normalized if abnormal. C. Magnesium Ion Magnesium appears to have similar depressant effects as potassium on digitalis-induced arrhythmias. Magnesium also appears to be effective in some cases of torsades de pointes arrhythmia. D. Ranolazine and Ivabradine These newer agents were developed for use in angina and are discussed in Chapter 12. Their effects on cardiac ion currents are discussed in that chapter and they are under study for use in cardiac arrhythmias. 128 PART III Cardiovascular Drugs NONPHARMACOLOGIC TREATMENT OF ARRHYTHMIAS It should be noted that electrical methods of treatment of arrhythmias have become very important. These methods include (1) external defibrillation, (2) implanted defibrillators, (3) implanted pacemakers, and (4) radiofrequency ablation or cryoablation of arrhythmogenic foci via a catheter. QUESTIONS Questions 1 and 2. A 76-year-old patient with rheumatoid arthritis and chronic heart disease is being considered for treatment with procainamide. She is already receiving digoxin, hydrochlorothiazide, and potassium supplements for her cardiac condition. 1. In deciding on a treatment regimen with procainamide for this patient, which of the following statements is most correct? (A) A possible drug interaction with digoxin suggests that digoxin blood levels should be obtained before and after starting procainamide (B) Hyperkalemia should be avoided to reduce the likelihood of procainamide toxicity (C) Procainamide cannot be used if the patient has asthma because it has a β-blocking effect (D) Procainamide cannot be used if the patient has angina because it has a β-agonist effect (E) Procainamide is not active by the oral route 2. If this patient should take an overdose and manifest severe acute procainamide toxicity with markedly prolonged QRS, which of the following should be given immediately? (A) A calcium chelator such as EDTA (B) Digitalis (C) Nitroprusside (D) Potassium chloride (E) Sodium lactate 3. A 57-year-old man is admitted to the emergency department with chest pain and a fast irregular heart rhythm. The ECG shows an inferior myocardial infarction and ventricular tachycardia. Lidocaine is ordered. When used as an antiarrhythmic drug, lidocaine typically (A) Increases action potential duration (B) Increases contractility (C) Increases PR interval (D) Reduces abnormal automaticity (E) Reduces resting potential 4. A 36-year-old woman with a history of poorly controlled thyrotoxicosis has recurrent episodes of tachycardia with severe shortness of breath. When she is admitted to the emergency department with one of these episodes, which of the following drugs would be most suitable? (A) Amiodarone (B) Disopyramide (C) Esmolol (D) Quinidine (E) Verapamil 5. A 16-year-old girl has paroxysmal attacks of rapid heart rate with palpitations and shortness of breath. These episodes occasionally terminate spontaneously but often require a visit to the emergency department of the local hospital. Her ECG during these episodes reveals an AV nodal tachycardia. The antiarrhythmic of choice in most cases of acute AV nodal tachycardia is (A) Adenosine (B) Amiodarone (C) Flecainide (D) Propranolol (E) Verapamil 6. A 55-year-old man is admitted to the emergency department and is found to have an abnormal ECG. Overdose of an antiarrhythmic drug is considered. Which of the following drugs is correctly paired with its ECG effects? (A) Quinidine: Increased PR and decreased QT intervals (B) Flecainide: Increased PR, QRS, and QT intervals (C) Verapamil: Increased PR interval (D) Lidocaine: Decreased QRS and PR interval (E) Metoprolol: Increased QRS duration 7. A 60-year-old man comes to the emergency department with severe chest pain. ECG reveals ventricular tachycardia with occasional normal sinus beats, and ST-segment changes suggestive of ischemia. A diagnosis of myocardial infarction is made, and the man is admitted to the cardiac intensive care unit. His arrhythmia should be treated immediately with (A) Adenosine (B) Digoxin (C) Lidocaine (D) Quinidine (E) Verapamil 8. Which of the following drugs slows conduction through the AV node and has its primary action directly on L-type calcium channels? (A) Adenosine (B) Amiodarone (C) Diltiazem (D) Esmolol (E) Flecainide (F) Lidocaine (G) Mexiletine (H) Procainamide (I) Quinidine 9. When working in outlying areas, this 62-year-old rancher is away from his house for 12–14 h at a time. He has an arrhythmia that requires chronic therapy. Which of the following has the longest half-life of all antiarrhythmic drugs? (A) Adenosine (B) Amiodarone (C) Disopyramide (D) Esmolol (E) Flecainide (F) Lidocaine (G) Mexiletine (H) Procainamide (I) Quinidine (J) Verapamil CHAPTER 14 Antiarrhythmic Drugs 10. A drug was tested in the electrophysiology laboratory to determine its effects on the cardiac action potential in normal ventricular cells. The results are shown in the diagram. Control 0 mV Drug −80 mV Which of the following drugs does this agent most resemble? (A) Adenosine (B) Flecainide (C) Mexiletine (D) Procainamide (E) Verapamil ANSWERS 1. Hyperkalemia facilitates procainamide toxicity. Procainamide is active by the oral route and has a duration of action of 2–4 h (in the prompt-release form). Procainamide has no significant documented interaction with digoxin and no significant β-agonist or β-blocking action. The answer is B. 2. The most effective therapy for procainamide toxicity appears to be concentrated sodium lactate. This drug may (1) increase sodium current by increasing the sodium ion gradient and (2) reduce drug-receptor binding by alkalinizing the tissue. The answer is E. 3. Lidocaine reduces automaticity in the ventricles; the drug does not alter resting potential or AP duration and does not increase contractility. The answer is D. 4. Beta blockers are the most effective agents in acute thyrotoxic arrhythmias. Esmolol is a parenteral, rapid-acting β blocker (see Chapter 10). The answer is C. 5. Calcium channel blockers are effective in supraventricular AV nodal tachycardias. However, adenosine is just as effective in most acute nodal tachycardias and is less toxic because of its extremely short duration of action. The answer is A. 6. All the associations listed are incorrect except verapamil (see Table 14–1). Because calcium blockers slow AV conduction, group 4 drugs such as verapamil and diltiazem increase PR interval and have little effect on the other ECG variables. The answer is C. 129 7. Lidocaine has limited applications as an antiarrhythmic drug, but emergency treatment of myocardial infarction arrhythmias is one of the most important. Lidocaine is also useful in digoxin-induced arrhythmias. After recovery from the acute phase of a myocardial infarction, β blockers are used for 2 yr or more to prevent sudden death arrhythmias. The answer is C. 8. Diltiazem is the calcium channel blocker in this list. (Beta blockers also slow AV conduction but have much smaller effects on calcium channels.) The answer is C. 9. Amiodarone has the longest half-life of all the antiarrhythmics (weeks). The answer is B. 10. The drug effect shown in the diagram includes slowing of the upstroke of the AP but no significant change in repolarization or AP duration. This is most typical of group 1C drugs. The answer is B, flecainide. SKILL KEEPER ANSWER: CHARACTERISTICS OF a BLOCKERS (SEE CHAPTER 10) The major subgroups of β blockers and their pharmacologic features are conveniently listed in a table: a-Blocker Subgroup, Features Examples Nonselective Propranolol and timolol are typical; block both β1 and β2 β1-selective Atenolol, acebutolol, and metoprolol are typical; possibly less hazardous in asthmatic patients Acebutolol and pindolol are typical; possibly less hazardous in asthmatic patients Timolol is the prototype; important for use in glaucoma Atenolol is the prototype; may reduce CNS toxicity Esmolol (an ester) is the shortest acting and used only IV; nadolol is the longest acting Partial agonist Lacking local anesthetic effect Low lipid solubility Very short and long acting Combined β and α blockade Carvedilol, labetalol 130 PART III Cardiovascular Drugs CHECKLIST When you complete this chapter, you should be able to: ❑ Describe the distinguishing electrophysiologic action potential and ECG effects of the 4 major groups of antiarrhythmic drugs and adenosine. ❑ List 2 or 3 of the most important drugs in each of the 4 groups. ❑ List the major toxicities of those drugs. ❑ Describe the mechanism of selective depression by local anesthetic antiarrhythmic agents. ❑ Explain how hyperkalemia, hypokalemia, or an antiarrhythmic drug can cause an arrhythmia. DRUG SUMMARY TABLE: Antiarrhythmic Drugs Subclass Mechanism of Action Clinical Applications Pharmacokinetics Toxicities, Interactions Group 1A Procainamide Use- and state-dependent Atrial and ventricular Oral and parenteral Increased arrhythmias block of lNa channels arrhythmias, especially • oral slow-release forms including torsades, hypoten• some block of IK chanafter myocardial infarction available sion, lupus-like syndrome nels. Slowed conduction • Duration: 2–3 h velocity and pacemaker activity • prolonged action potential duration and refractory period Disopyramide: similar to procainamide but longer duration of action; toxicity includes antimuscarinic effects and heart failure Quinidine: similar to procainamide but greater toxicity, including cinchonism (tinnitus, vertigo, headache), gastrointestinal disturbance, and thrombocytopenia Group 1B Lidocaine Highly selective use- and state-dependent INa block; minimal effect in normal tissue; no effect on IK Ventricular arrhythmias post-myocardial infarction and digitalis-induced arrhythmias IV and IM Duration: 1–2 h Central nervous system (CNS) sedation or excitation Mexiletine: similar to lidocaine but oral activity and longer duration of action; also used in neuropathic pain Group 1C Flecainide Selective use- and statedependent block of lNa; slowed conduction velocity and pacemaker activity Refractory arrhythmias Oral Increased arrhythmias • CNS excitation Block of β receptors; slowed pacemaker activity Postmyocardial infarction as prophylaxis against sudden death ventricular fibrillation; thyrotoxicosis Oral, parenteral Duration: 4–6 h Bronchospasm • cardiac depression, atrioventricular (AV) block, hypotension (see Chapter 10) Group 2 Propranolol Metoprolol: similar to propranolol but β1-selective Esmolol: selective β1-receptor blockade; IV only, 10-min duration. Used in perioperative and thyrotoxicosis arrhythmias (Continued ) CHAPTER 14 Antiarrhythmic Drugs 131 DRUG SUMMARY TABLE: Antiarrhythmic Drugs (Continued ) Subclass Mechanism of Action Clinical Applications Pharmacokinetics Toxicities, Interactions Strong IK block produces marked prolongation of action potential and refractory period. Group 1 activity slows conduction velocity • groups 2 and 4 activity confer additional antiarrhythmic activity IK block and β-adrenoceptor block Selective IK block • prolonged action potential and QT interval Like ibutilide Refractory arrhythmias • used off-label in many arrhythmias (broad spectrum antiarrhythmic action) Oral, parenteral Half-life and duration of action: 1–10 wk Thyroid abnormalities, deposits in skin and cornea, pulmonary fibrosis, optic neuritis • torsades is rare with amiodarone Ventricular arrhythmias and atrial fibrillation Treatment of acute atrial fibrillation Oral Duration: 7 h Ibutilide is IV only Duration: 6 h Dose-related torsades de pointes • cardiac depression Torsades de pointes Treatment and prophylaxis of atrial fibrillation Oral Duration: 7 h Torsades de pointes Group 3 Amiodarone Sotalol Ibutilide Dofetilide Group 4 Verapamil State- and use-dependent ICa block slows conduction in AV node and pacemaker activity • PR interval prolongation AV nodal arrhythmias, especially in prophylaxis Oral, parenteral Duration: 7 h Cardiac depression, constipation, hypotension Diltiazem Like verapamil Rate control in atrial fibrillation Oral, parenteral Duration: 6 h Like verapamil Dihydropyridines: calcium channel blockers but not useful in arrhythmias; sometimes precipitate them Miscellaneous Adenosine Increase in diastolic IK of AV node that causes marked hyperpolarization and conduction block • reduced ICa Acute nodal tachycardias IV only Duration: 10–15 s Flushing, bronchospasm, chest pain, headache Potassium ion Increase in all K currents, decreased automaticity, decreased digitalis toxicity Digitalis toxicity and other arrhythmias if serum K is low Oral or IV Both hypokalemia and hyperkalemia are associated with arrhythmogenesis. Severe hyperkalemia causes cardiac arrest Magnesium ion Poorly understood, possible increase in Na+/K+ ATPase activity Digitalis arrhythmias and other arrhythmias if serum Mg is low IV Muscle weakness • severe hypermagnesemia can cause respiratory paralysis C A P T E R 15 Diuretics & Other Drugs That Act on the Kidney Drugs that act on the kidney have important applications in renal, cardiovascular, and endocrine disorders. These disorders mainly involve sodium and water homeostasis. Each segment of the nephron—proximal convoluted tubule (PCT), thick ascending limb of the loop of Henle (TAL), distal convoluted tubule (DCT), and cortical collecting tubule (CCT)—has a H different mechanism for reabsorbing sodium and other ions. The subgroups of the sodium-excreting diuretics are based on these sites and processes in the nephron. Several other drugs alter water excretion predominantly. The effects of the diuretic agents are predictable from knowledge of the function of the segment of the nephron in which they act. Drugs used in renal disorders Drugs that modify water excretion Drugs that modify salt excretion PCT TAL Loop diuretics (furosemide) Carbonic anhydrase inhibitors (acetazolamide) DCT CCT K+-sparing diuretics (spironolactone) Thiazides (hydrochlorothiazide) RENAL TRANSPORT MECHANISMS & DIURETIC DRUG GROUPS The kidney filters plasma water and solutes at the glomerulus at a very high rate (180 L/day) and must recover a significant percentage of most of these substances before excretion in the urine. The major transport mechanisms for the recovery of ions and water in the various segments of the nephron are shown in Figure 15–1. Because the mechanisms for reabsorption of salt and water differ in each of the 4 major tubular segments, the diuretics acting in these segments have differing mechanisms of 132 Osmotic diuretics (mannitol) ADH agonists (desmopressin) ADH antagonists (conivaptan) action. Most diuretics act from the luminal side of the membrane. An exception is the aldosterone receptor antagonist group (spironolactone and eplerenone); these drugs enter the collecting tubule cell from the basolateral side and bind to the cytoplasmic aldosterone receptor. The kidney contains numerous adenosine and prostaglandin receptors. Agonists and antagonists at these receptors can alter renal function directly and alter the response to the diuretic agents. Prostaglandins are important in maintaining glomerular filtration. When synthesis of prostaglandins is inhibited, for example, by nonsteroidal anti-inflammatory drugs (Chapter 36), the efficacy of most diuretics decreases. CHAPTER 15 Diuretics & Other Drugs That Act on the Kidney 133 High-Yield Terms to Learn Bicarbonate diuretic A diuretic that selectively increases sodium bicarbonate excretion. Example: a carbonic anhydrase inhibitor Diluting segment A segment of the nephron that removes solute without water; the thick ascending limb and the distal convoluted tubule are active salt-reabsorbing segments that are not permeable by water Hyperchloremic metabolic acidosis A shift in body electrolyte and pH balance involving elevated serum chloride, diminished bicarbonate concentration, and a decrease in pH in the blood. Typical result of bicarbonate diuresis Hypokalemic metabolic alkalosis A shift in body electrolyte balance and pH involving a decrease in serum potassium and an increase in blood pH. Typical result of loop and thiazide diuretic actions Nephrogenic diabetes insipidus Loss of urine-concentrating ability in the kidney caused by lack of responsiveness to antidiuretic hormone (ADH is normal or high) Pituitary diabetes insipidus Loss of urine-concentrating ability in the kidney caused by lack of antidiuretic hormone (ADH is low or absent) Potassium-sparing diuretic A diuretic that reduces the exchange of potassium for sodium in the collecting tubule; a drug that increases sodium and reduces potassium excretion. Example: aldosterone antagonists Uricosuric diuretic A diuretic that increases uric acid excretion, usually by inhibiting uric acid reabsorption in the proximal tubule. Example: ethacrynic acid NaHCO3 Proximal convoluted tubule NaCl NaCl Ca2+ (+PTH) Distal convoluted tubule 1 Proximal straight tubule 7 K+ 7 + K 2 Ca H2O Glomerulus Cortex 4 H+ 2+ Collecting tubule 7 Mg2+ Na+ ? 4 K Outer medulla + 5 3 NaCl (+aldosterone) 2Cl− K+ Diuretics 1 Acetazolamide 2 Osmotic agents (mannitol) 3 Loop agents (eg, furosemide) 4 Thiazides 5 Aldosterone antagonists 6 ADH antagonists 7 Adenosine H+ Thick ascending limb H2O (+ADH) Thin descending limb 2 H2O 7 6 2 Collecting duct Thin ascending limb Loop of Henle Inner medulla FIGURE 15–1 Tubule transport systems in the kidney and sites of action of diuretics. Circles with arrows denote known ion cotransporters that are targets of the diuretics indicated by the numerals. Question marks denote preliminary or incompletely documented suggestions for the location of certain drug effects. ADH, antidiuretic hormone; PTH, parathyroid hormone. (Modified and reproduced, with permission, from Katzung BG, editor: Basic & Clinical Pharmacology, 12th ed. McGraw-Hill, 2012: Fig. 15–1.) 134 PART III Cardiovascular Drugs Proximal convoluted tubule Lumenurine is the target of carbonic anhydrase inhibitor drugs. Active secretion and reabsorption of weak acids and bases also occurs in the proximal tubule. Most weak acid transport occurs in the straight S2 segment, distal to the convoluted part. Uric acid transport is especially important and is targeted by some of the drugs used in treating gout (Chapter 36). Weak bases are transported in the S1 and S2 segments. A glucose-sodium cotransporter (SGLT2) is responsible for the reabsorption of glucose in the proximal tubule, and inhibitors are now available that inhibit this transporter and reduce blood sugar in diabetics. Interstitiumblood Na+ NHE3 ATP Na+ K+ + HCO3– + H + H + HCO3 – Na+ H2CO3 H2CO3 + CA CA H2O + CO2 CARBONIC ANHYDRASE INHIBITORS CO2 + H2O – Cl Base– FIGURE 15–2 Mechanism of sodium bicarbonate reabsorption in the proximal tubule cell. NHE3, Na+/H+ exchanger 3; CA, carbonic anhydrase. (Reproduced, with permission, from Katzung BG, editor: Basic & Clinical Pharmacology, 12th ed. McGraw-Hill, 2012: Fig. 15–2.) PROXIMAL CONVOLUTED TUBULE (PCT) This segment carries out isosmotic reabsorption of amino acids, glucose, and numerous cations. It is the major site for sodium chloride and sodium bicarbonate reabsorption. The proximal tubule is responsible for 60–70% of the total reabsorption of sodium. No currently available drug directly acts on NaCl reabsorption in the PCT. The mechanism for bicarbonate reabsorption is shown in Figure 15–2. Bicarbonate itself is poorly reabsorbed through the luminal membrane, but conversion of bicarbonate to carbon dioxide via carbonic acid permits rapid reabsorption of the carbon dioxide. Bicarbonate can then be regenerated from carbon dioxide within the tubular cell and transported into the interstitium. Sodium is separately reabsorbed from the lumen in exchange for hydrogen ions (NHE3 transporter) and transported into the interstitial space by the sodium-potassium pump (Na+/K+ ATPase). Carbonic anhydrase, the enzyme required for the bicarbonate reabsorption process on the brush border and in the cytoplasm, A. Prototypes and Mechanism of Action Acetazolamide is the prototypic agent. These diuretics are sulfonamide derivatives. The mechanism of action is inhibition of carbonic anhydrase in the brush border and cytoplasm (Figure 15–2). Carbonic anhydrase is also found in other tissues and plays an important role in the secretion of cerebrospinal fluid and aqueous humor. Acetazolamide inhibits carbonic anhydrase in all tissues of the body. B. Effects The major renal effect is bicarbonate diuresis (ie, sodium bicarbonate is excreted); body bicarbonate is depleted, and metabolic acidosis results. As increased sodium is presented to the cortical collecting tubule, some of the excess sodium is reabsorbed and potassium is secreted, resulting in significant potassium wasting (Table 15–1). As a result of bicarbonate depletion, sodium bicarbonate excretion slows—even with continued diuretic administration—and the diuresis is self-limiting within 2–3 days. Secretion of bicarbonate into aqueous humor by the ciliary epithelium in the eye and into the cerebrospinal fluid by the choroid plexus is reduced. In the eye, a useful reduction in intraocular pressure can be achieved. In the central nervous system (CNS), acidosis of the cerebrospinal fluid results in hyperventilation, which can protect against high-altitude sickness. The ocular and cerebrospinal fluid effects are not self-limiting. C. Clinical Uses and Toxicity Acetazolamide is used parenterally in the treatment of severe acute glaucoma (see Table 10–2). Acetazolamide can also be administered orally, but topical analogs are available (dorzolamide, TABLE 15–1 Electrolyte changes produced by diuretic drugs. Amount in Urine Group NaHCO3 K+ Body pH a Carbonic anhydrase inhibitors ↑ ↑↑↑ ↑ Acidosisb Loop diuretics ↑↑↑↑ — ↑ Alkalosis Thiazides ↑↑ ↑,— ↑ Alkalosis K+-sparing diuretics ↑ — ↓ Acidosis a Self-limited (2–3 days). b NaCl Not self-limited. a a CHAPTER 15 Diuretics & Other Drugs That Act on the Kidney brinzolamide) for chronic use in the eye. Acetazolamide is also used to prevent acute mountain (high-altitude) sickness. It is used for the diuretic effect only if edema is accompanied by significant metabolic alkalosis. Drowsiness and paresthesia toxicities are commonly reported after oral therapy. Cross-allergenicity between these and all other sulfonamide derivatives (other sulfonamide diuretics, hypoglycemic agents, antibacterial sulfonamides) is uncommon but can occur. Alkalinization of the urine by these drugs may cause precipitation of calcium salts and formation of renal stones. Renal potassium wasting may be marked. Patients with hepatic impairment often excrete large amounts of ammonia in the urine in the form of ammonium ion. If they are given acetazolamide, alkalinization of the urine prevents conversion of ammonia to ammonium ion. As a result, they may develop hepatic encephalopathy because of increased ammonia reabsorption and hyperammonemia. THICK ASCENDING LIMB OF THE LOOP OF HENLE (TAL) This segment pumps sodium, potassium, and chloride out of the lumen into the interstitium of the kidney. It is also a major site of calcium and magnesium reabsorption, as shown in Figure 15–3. Reabsorption of sodium, potassium, and chloride are all accomplished by a Na+/K+/2Cl– carrier (NKCC2), which is the target of the loop diuretics. This cotransporter provides part of the concentration gradient for the countercurrent concentrating mechanism in the kidney and is responsible for the reabsorption of 20–30% of the sodium filtered at the glomerulus. Because Thick ascending limb Lumenurine Interstitiumblood NKCC2 K+ ATP – (+) Potential potassium is pumped into the cell from both the luminal and basal sides, an escape route must be provided; this occurs into the lumen via a potassium-selective channel. Because the potassium diffusing through these channels is not accompanied by an anion, a net positive charge is set up in the lumen. This positive potential drives the reabsorption of calcium and magnesium. LOOP DIURETICS A. Prototypes and Mechanism of Action Furosemide is the prototypical loop agent. Furosemide, bumetanide, and torsemide are sulfonamide derivatives. Ethacrynic acid is a phenoxyacetic acid derivative; it is not a sulfonamide but acts by the same mechanism. Loop diuretics inhibit the cotransport of sodium, potassium, and chloride (NKCC2, Figure 15–3). The loop diuretics are relatively short-acting (diuresis usually occurs over a 4-h period following a dose). B. Effects A full dose of a loop diuretic produces a massive sodium chloride diuresis if glomerular filtration is normal; blood volume may be significantly reduced. If tissue perfusion is adequate, edema fluid is rapidly excreted. The diluting ability of the nephron is reduced because the loop of Henle is the site of significant dilution of urine. Inhibition of the Na+/K+/2Cl– transporter also results in loss of the lumen-positive potential, which reduces reabsorption of divalent cations as well. As a result, calcium excretion is significantly increased. Ethacrynic acid is a moderately effective uricosuric drug if blood volume is maintained. The presentation of large amounts of sodium to the collecting tubule may result in significant potassium wasting and excretion of protons; hypokalemic alkalosis may result (Table 15–1). Loop diuretics also reduce pulmonary vascular pressures; the mechanism is not known. Na+ Na+ 2Cl 135 K+ K+ K+ Cl – Mg2+, Ca2+ FIGURE 15–3 Mechanism of sodium, potassium, and chloride reabsorption by the transporter NKCC2 in the thick ascending limb of the loop of Henle. Note that pumping of potassium into the cell from both the lumen and the interstitium would result in unphysiologically high intracellular K+ concentration. This is avoided by movement of K+ down its concentration gradient back into the lumen, carrying with it excess positive charge. This positive charge drives the reabsorption of calcium and magnesium. (Reproduced, with permission, from Katzung BG, editor: Basic & Clinical Pharmacology, 12th ed. McGraw-Hill, 2012: Fig. 15–3.) C. Clinical Use and Toxicities The major application of loop diuretics is in the treatment of edematous states (eg, heart failure, ascites, and acute pulmonary edema). They are sometimes used in hypertension if response to thiazides is inadequate, but the short duration of action of loop diuretics is a disadvantage in this condition. A less common but important application is in the treatment of severe hypercalcemia. This life-threatening condition can often be managed with large doses of furosemide together with parenteral volume and electrolyte (sodium and potassium chloride) replacement. It should be noted that diuresis without volume replacement results in hemoconcentration; serum calcium concentration then will not diminish and may even increase further. Loop diuretics usually induce hypokalemic metabolic alkalosis (Table 15–1). Because large amounts of sodium are presented to the collecting tubules, potassium wasting may be severe. Because they are so efficacious, loop diuretics can cause hypovolemia and cardiovascular complications. Ototoxicity is an important toxic effect of the loop agents. The sulfonamides in this group may rarely cause typical sulfonamide allergy, eg, rash. 136 PART III Cardiovascular Drugs DISTAL CONVOLUTED TUBULE (DCT) This segment actively pumps sodium and chloride out of the lumen of the nephron via the Na+/Cl– carrier (NCC) shown in Figure 15–4. This cotransporter is the target of the thiazide diuretics. The distal convoluted tubule is responsible for 5–8% of filtered sodium reabsorption. Calcium is also reabsorbed in this segment under the control of parathyroid hormone (PTH). Removal of the reabsorbed calcium back into the blood requires the sodium-calcium exchange process discussed in Chapter 13. THIAZIDE DIURETICS A. Prototypes and Mechanism of Action Hydrochlorothiazide, the prototypical agent, and all the other members of this group are sulfonamide derivatives. A few derivatives that lack the typical thiazide ring in their structure nevertheless have effects identical with those of thiazides and are therefore considered thiazide-like. The major action of thiazides is to inhibit sodium chloride transport in the early segment of the distal convoluted tubule (NCC, Figure 15–4). Thiazides are active by the oral route and have a duration of action of 6–12 h, considerably longer than most loop diuretics. Chlorothiazide is the only thiazide available for parenteral use. Distal convoluted tubule Lumenurine Interstitiumblood NCC Na+ Na+ ATP Cl– K+ + R PTH Ca2+ Ca2+ B. Effects In full doses, thiazides produce moderate but sustained sodium and chloride diuresis. Hypokalemic metabolic alkalosis may occur (Table 15–1). Reduction in the transport of sodium from the lumen into the tubular cell reduces intracellular sodium and promotes sodium-calcium exchange at the basolateral membrane. As a result, reabsorption of calcium from the urine is increased, and urine calcium content is decreased—the opposite of the effect of loop diuretics. Because they act in a diluting segment of the nephron, thiazides may reduce the excretion of water and cause dilutional hyponatremia. Thiazides also reduce blood pressure, and the maximal pressure-lowering effect occurs at doses lower than the maximal diuretic doses (see Chapter 11). Chlorthalidone is longer acting than hydrochlorothiazide and may be particularly valuable in hypertension. Inhibition of renal prostaglandin synthesis reduces the efficacy of the thiazides. When a thiazide is used with a loop diuretic, a synergistic effect occurs with marked diuresis. C. Clinical Use and Toxicities The major application of thiazides is in hypertension, for which their long duration and moderate intensity of action are particularly useful. Chronic therapy of edematous conditions such as mild heart failure is another application, although loop diuretics are usually preferred. Chronic renal calcium stone formation can sometimes be controlled with thiazides because they reduce urine calcium concentration. Thiazides are also used in the treatment of nephrogenic diabetes insipidus. Massive sodium diuresis with hyponatremia is an uncommon but dangerous early toxicity of thiazides. Chronic therapy is often associated with potassium wasting, since an increased sodium load is presented to the collecting tubules; the cortical collecting tubules compensate by reabsorbing sodium and excreting potassium. Diabetic patients may have significant hyperglycemia. Serum uric acid and lipid levels are also increased in some persons. Combination with loop agents may result in rapid development of severe hypovolemia and cardiovascular collapse. Thiazides are sulfonamides and share potential sulfonamide allergenicity. Na+ CORTICAL COLLECTING TUBULE (CCT) Ca2+ ATP H+ FIGURE 15–4 Mechanism of sodium and chloride reabsorption by the transporter NCC in the distal convoluted tubule. A separate reabsorptive mechanism, modulated by parathyroid hormone (PTH), is present for movement of calcium into the cell from the urine. This calcium must be transported via the sodium-calcium exchanger back into the blood. R, PTH receptor. (Reproduced, with permission, from Katzung BG, editor: Basic & Clinical Pharmacology, 12th ed. McGrawHill, 2012: Fig. 15–4.) The final segment of the nephron is the last tubular site of sodium reabsorption and is controlled by aldosterone (Figure 15–5), a steroid hormone secreted by the adrenal cortex. This segment is responsible for reabsorbing 2–5% of the total filtered sodium under normal circumstances; more if aldosterone is increased. The reabsorption of sodium occurs via channels (ENaC, not a transporter) and is accompanied by loss of potassium or hydrogen ions. The collecting tubule is thus the primary site of acidification of the urine and the last site of potassium excretion. The aldosterone receptor and the sodium channels are sites of action of the potassium-sparing diuretics. Reabsorption of water occurs in the medullary collecting tubule under the control of antidiuretic hormone (ADH). CHAPTER 15 Diuretics & Other Drugs That Act on the Kidney Lumenurine Interstitiumblood Collecting tubule Cl– Principal cell ENaC + Aldosterone R Na+ + K Na+ + ATP K+ Intercalated cell ATP Aldosteronism (eg, the elevated serum aldosterone levels that occur in cirrhosis) is an important indication for spironolactone. Aldosteronism is also a feature of heart failure, and spironolactone and eplerenone have been shown to have significant long-term benefits in this condition (Chapter 13). Some of this poorly understood effect may occur in the heart. The most important toxic effect of potassium-sparing diuretics is hyperkalemia. These drugs should never be given with potassium supplements. Other aldosterone antagonists (eg, angiotensin [ACE] inhibitors and angiotensin receptor blockers [ARBs]), if used at all, should be used with caution. Spironolactone can cause endocrine alterations including gynecomastia and antiandrogenic effects. Eplerenone has less affinity for gonadal steroid receptors. SKILL KEEPER: DIURETIC COMBINATIONS AND ELECTROLYTES (SEE CHAPTER 13) – H+ 137 HCO3 Cl– FIGURE 15–5 Mechanism of sodium, potassium, and hydrogen ion movement in the collecting tubule cells. Synthesis of Na+/K+ ATPase, and the epithelial sodium channels (ENaC) and potassium channels is under the control of aldosterone, which combines with an intracellular receptor, R, before entering the nucleus. (Reproduced, with permission, from Katzung BG, editor: Basic & Clinical Pharmacology, 12th ed. McGraw-Hill, 2012: Fig. 15–5.) POTASSIUM-SPARING DIURETICS A. Prototypes and Mechanism of Action Spironolactone and eplerenone are steroid derivatives and act as pharmacologic antagonists of aldosterone in the collecting tubules. By combining with and blocking the intracellular aldosterone receptor, these drugs reduce the expression of genes that code for the epithelial sodium ion channel (ENaC) and Na+/K+ ATPase. Amiloride and triamterene act by blocking the ENaC sodium channels (Figure 15–5). (These drugs do not block INa channels in excitable membranes.) Spironolactone and eplerenone have slow onsets and offsets of action (24–72 h). Amiloride and triamterene have durations of action of 12–24 h. B. Effects All drugs in this class cause an increase in sodium clearance and a decrease in potassium and hydrogen ion excretion and therefore qualify as potassium-sparing diuretics. They may cause hyperkalemic metabolic acidosis (Table 15–1). C. Clinical Uses and Toxicities Potassium wasting caused by chronic therapy with loop or thiazide diuretics, if not controlled by dietary potassium supplements, usually responds to these drugs. They are also available in combination with a thiazide in a single pill. Describe the possible interactions of cardiac glycosides (digoxin) with the major classes of diuretics. The Skill Keeper Answer appears at the end of the chapter. OSMOTIC DIURETICS A. Prototypes and Mechanism of Action Mannitol, the prototypical osmotic diuretic, is given intravenously. Other drugs often classified with mannitol (but rarely used) include glycerin, isosorbide (not isosorbide dinitrate), and urea. Because they are freely filtered at the glomerulus but poorly reabsorbed from the tubule, they remain in the lumen and “hold” water by virtue of their osmotic effect. The major location for this action is the proximal convoluted tubule. Reabsorption of water is also reduced in the descending limb of the loop of Henle and the collecting tubule. B. Effects The volume of urine is increased. Most filtered solutes are excreted in larger amounts unless they are actively reabsorbed. Sodium excretion is usually increased because the rate of urine flow through the tubule is greatly accelerated and sodium transporters cannot handle the volume rapidly enough. Mannitol can also reduce brain volume and intracranial pressure by osmotically extracting water from the tissue into the blood. A similar effect occurs in the eye. C. Clinical Use and Toxicities The osmotic drugs are used to maintain high urine flow (eg, when renal blood flow is reduced and in conditions of solute overload from severe hemolysis, rhabdomyolysis, or tumor lysis syndrome). Mannitol and several other osmotic agents are useful in reducing intraocular pressure in acute glaucoma and intracranial pressure in neurologic conditions. Movement of water from the intracellular compartment to the extracellular may cause hyponatremia and pulmonary edema. As the water is excreted, hypernatremia may follow. Headache, nausea, and vomiting are common. 138 PART III Cardiovascular Drugs SGLT2 ANTAGONISTS Dapagliflozin, canagliflozin, and empagliflozin are approved for the treatment of diabetes. They reduce the active reabsorption of filtered glucose in the proximal tubule and increase its excretion by 30–50%. Although they increase the volume of urine, they are not used as diuretics. High glucose concentration in the urine may result in urinary tract infections. ANTIDIURETIC HORMONE AGONISTS & ANTAGONISTS A. Prototypes and Mechanism of Action Antidiuretic hormone (ADH) and desmopressin are prototypical ADH agonists. They are peptides and must be given parenterally. Conivaptan and tolvaptan are ADH antagonists. Demeclocycline was previously used for this purpose. Lithium also has ADH-antagonist effects but is never used for this purpose. ADH facilitates water reabsorption from the collecting tubule by activation of V2 receptors, which stimulate adenylyl cyclase via Gs. The increased cyclic adenosine monophosphate (cAMP) causes the insertion of additional aquaporin AQP2 water channels into the luminal membrane in this part of the tubule (Figure 15–6). Conivaptan is an ADH inhibitor at V1a and V2 receptors. Lumenurine Interstitiumblood Collecting tubule AQP2 V2 H2O R H2O Tolvaptan is a more selective V2 blocker with little V1 affinity. Demeclocycline and lithium inhibit the action of ADH at some point distal to the generation of cAMP and presumably interfere with the insertion of water channels into the membrane. B. Effects and Clinical Uses 1. Agonists—ADH and desmopressin reduce urine volume and increase its concentration. ADH and desmopressin are useful in pituitary diabetes insipidus. They are of no value in the nephrogenic form of the disease, but salt restriction, water restriction, thiazides, and loop diuretics may be used. These therapies reduce blood volume, a very strong stimulus to proximal tubular reabsorption. The proximal tubule thus substitutes—in part—for the deficient concentrating function of the collecting tubule in nephrogenic diabetes insipidus. 2. Antagonists—ADH antagonists oppose the actions of ADH and other naturally occurring peptides that act on the same V2 receptor. Such peptides are produced by certain tumors (eg, small cell carcinoma of the lung) and can cause significant water retention and dangerous hyponatremia. This syndrome of inappropriate ADH secretion (SIADH syndrome) causes hyponatremia and can be treated with demeclocycline and conivaptan or tolvaptan. Lithium also works but has greater toxicity and is never used for this indication. Conivaptan and tolvaptan are also used off label in some patients with heart failure. C. Toxicity In the presence of ADH or desmopressin, a large water load may cause dangerous hyponatremia. Large doses of either peptide may cause hypertension in some persons. Conivaptan and tolvaptan may cause demyelination with serious neurologic consequences if hyponatremia is corrected too rapidly. Conivaptan may cause infusion site reactions. In children younger than 8 years, demeclocycline (like other tetracyclines) causes bone and teeth abnormalities. Lithium causes nephrogenic diabetes insipidus as a toxic effect; because of its other toxicities, the drug is never used to treat SIADH. V2 cAMP R ADH + H2O AQP2 H2O AQP3,4 H2O FIGURE 15–6 Mechanism of water reabsorption across the membranes of collecting duct cells. Aquaporins 3 and 4 (AQP3, 4) are normally present in the basolateral membranes, but the luminal water channel, AQP2, is inserted only in the presence of ADH or similar antidiuretic peptides acting on the vasopressin V2 receptor. (Reproduced, with permission, from Katzung BG, editor: Basic & Clinical Pharmacology, 12th ed. McGraw-Hill, 2012: Fig. 15–6.) QUESTIONS 1. A 70-year-old retired businessman is admitted with a history of recurrent heart failure and metabolic derangements. He has marked peripheral edema and metabolic alkalosis. Which of the following drugs is most appropriate for the treatment of his edema? (A) Acetazolamide (B) Digoxin (C) Dobutamine (D) Eplerenone (E) Hydrochlorothiazide CHAPTER 15 Diuretics & Other Drugs That Act on the Kidney 2. A 50-year-old man has a history of frequent episodes of renal colic with calcium-containing renal stones. A careful workup indicates that he has a defect in proximal tubular calcium reabsorption, which results in high concentrations of calcium salts in the tubular urine. The most useful diuretic agent in the treatment of recurrent calcium stones is (A) Chlorthalidone (B) Diazoxide (C) Ethacrynic acid (D) Mannitol (E) Spironolactone 3. Which of the following is an important effect of chronic therapy with loop diuretics? (A) Decreased urinary excretion of calcium (B) Elevation of blood pressure (C) Elevation of pulmonary vascular pressure (D) Metabolic alkalosis (E) Teratogenic action in pregnancy 4. Which drug is correctly associated with its actions in the following table? (+ indicates increase and – indicates decrease.) Choice Drug Urine Na+ Urine K+ Metabolic change A Acetazolamide +++ + Alkalosis B Furosemide ++ – Alkalosis C Hydrochlorothiazide + ++ Acidosis D Spironolactone + – Acidosis E Mannitol – ++ Alkalosis 5. Which of the following diuretics would be most useful in the acute treatment of a comatose patient with traumatic brain injury and cerebral edema? (A) Acetazolamide (B) Amiloride (C) Chlorthalidone (D) Furosemide (E) Mannitol 6. A 62-year-old man with advanced prostate cancer is admitted to the emergency department with mental obtundation. An electrolyte panel shows a serum calcium of 16.5 (normal 8.5–10.5 mg/dL). Which of the following therapies would be most useful in the management of severe hypercalcemia? (A) Acetazolamide plus saline infusion (B) Furosemide plus saline infusion (C) Hydrochlorothiazide plus saline infusion (D) Mannitol plus saline infusion (E) Spironolactone plus saline infusion 7. A 60-year-old patient complains of paresthesias and occasional nausea associated with one of her drugs. She is found to have hyperchloremic metabolic acidosis. She is probably taking (A) Acetazolamide for glaucoma (B) Amiloride for edema associated with aldosteronism (C) Furosemide for severe hypertension and heart failure (D) Hydrochlorothiazide for hypertension (E) Mannitol for cerebral edema 139 8. A 70-year-old woman is admitted to the emergency department because of a “fainting spell” at home. She appears to have suffered no trauma from her fall, but her blood pressure is 120/60 when lying down and 60/20 when she sits up. Neurologic examination and an ECG are within normal limits when she is lying down. Questioning reveals that she has recently started taking “water pills” (diuretics) for a heart condition. Which of the following drugs is the most likely cause of her fainting spell? (A) Acetazolamide (B) Amiloride (C) Furosemide (D) Hydrochlorothiazide (E) Spironolactone 9. A 58-year-old woman with lung cancer has abnormally low serum osmolality and hyponatremia. A drug that increases the formation of dilute urine and is used to treat SIADH is (A) Acetazolamide (B) Amiloride (C) Desmopressin (D) Ethacrynic acid (E) Furosemide (F) Hydrochlorothiazide (G) Mannitol (H) Spironolactone (I) Triamterene (J) Tolvaptan 10. A graduate student is planning to make a high-altitude climb in South America while on vacation. He will not have time to acclimate slowly to altitude. A drug that is useful in preventing high-altitude sickness is (A) Acetazolamide (B) Amiloride (C) Demeclocycline (D) Desmopressin (E) Ethacrynic acid ANSWERS 1. Although acetazolamide is rarely used in heart failure, carbonic anhydrase inhibitors are quite valuable in patients with edema and metabolic alkalosis. The high bicarbonate levels in these patients make them particularly susceptible to the action of carbonic anhydrase inhibitors. Digoxin is useful in chronic systolic failure but is not first-line therapy. Dobutamine is appropriate only when diuresis has already been accomplished in severe acute failure. Hydrochlorothiazide and spironolactone are not adequate for first-line therapy of edema in failure. The answer is A. 2. The thiazides are useful in the prevention of calcium stones because these drugs reduce tubular calcium concentration, probably by increasing passive proximal tubular and distal convoluted tubule reabsorption of calcium. In contrast, the loop agents (choice C) facilitate calcium excretion. Diazoxide is a thiazide-like vasodilator molecule but has no diuretic action; in fact, it may cause sodium retention. It is used in hypertension and insulinoma (see Chapter 11). The answer is A. 140 PART III Cardiovascular Drugs 3. Loop diuretics increase urinary calcium excretion and decrease blood pressure (in hypertension) and pulmonary vascular pressure (in congestive heart failure). They have no recognized teratogenic action. They cause metabolic alkalosis (Table 15–1). Loop diuretics also cause ototoxicity. The answer is D. 4. Acetazolamide causes metabolic acidosis. Furosemide causes a marked increase in sodium and a moderate increase in potassium excretion. Thiazides cause alkalosis and a greater increase in sodium than potassium excretion. Mannitol causes a small increase in both sodium and potassium excretion and no change in body pH. Spironolactone causes the changes indicated. The answer is D. 5. An osmotic agent is needed to remove water from the cells of the edematous brain and reduce intracranial pressure rapidly. The answer is E. 6. Diuretic therapy of hypercalcemia requires a reduction in calcium reabsorption in the thick ascending limb, an effect of loop diuretics. However, a loop diuretic alone would reduce blood volume around the remaining calcium so that serum calcium would not decrease appropriately. Therefore, saline infusion should accompany the loop diuretic. The answer is B. 7. Paresthesias and gastrointestinal distress are common adverse effects of acetazolamide, especially when it is taken chronically, as in glaucoma. The observation that the patient has metabolic acidosis also suggests the use of acetazolamide. The answer is A. 8. The case history suggests that the syncope (fainting) is associated with diuretic use. Complications of diuretics that can result in syncope include both postural hypotension (which this patient exhibits) due to excessive reduction of blood volume and arrhythmias due to excessive potassium loss. Potassium wasting is more common with thiazides (because of their long duration of action), but these drugs rarely cause reduction of blood volume sufficient to result in orthostatic hypotension. The answer is C, furosemide. 9. Retention of water with hyponatremia and inability to form dilute urine in the fully hydrated condition is characteristic of SIADH. Antagonists of ADH are needed to treat this condition. The answer is J, tolvaptan. 10. Carbonic anhydrase inhibitors are useful in the prevention of altitude sickness. The answer is A. SKILL KEEPER ANSWER: DIGITALIS AND DIURETICS (SEE CHAPTER 13) Digoxin toxicity is facilitated by hypokalemia. Therefore, potassium-wasting diuretics (eg, loop agents, thiazides), which are often needed in heart failure, can increase the risk of a fatal digitalis arrhythmia. Carbonic anhydrase inhibitors, though also potassium-wasting agents, are rarely used for their systemic and diuretic effects and are therefore less likely to be involved in digitalis toxicity. The potassium-sparing diuretics, in contrast to the other groups, can be useful in preventing such interactions with digitalis but may cause hyperkalemia, which can be arrhythmogenic. CHECKLIST When you complete this chapter, you should be able to: ❑ List 5 major types of diuretics and relate them to their sites of action. ❑ Describe 2 drugs that reduce potassium loss during sodium diuresis. ❑ Describe a therapy that reduces calcium excretion in patients who have recurrent urinary stones. ❑ Describe a treatment for severe acute hypercalcemia in a patient with advanced carcinoma. ❑ Describe a method for reducing urine volume in nephrogenic diabetes insipidus. ❑ Describe a method for increasing water excretion in SIADH secretion. ❑ List the major applications and the toxicities of acetazolamide, thiazides, loop diuretics, and potassium-sparing diuretics. CHAPTER 15 Diuretics & Other Drugs That Act on the Kidney 141 DRUG SUMMARY TABLE: Diuretic Agents Subclass Mechanism of Action Clinical Applications Pharmacokinetics Toxicities, Interactions Glaucoma, mountain sickness • edema with alkalosis Oral, parenteral Diuresis is self-limiting but effects in glaucoma and mountain sickness persist Metabolic acidosis; sedation, paresthesias. Hyperammonemia in cirrhosis Oral, parenteral Metabolic hypokalemic alkalosis • ototoxicity • hypovolemia • efficacy reduced by nonsteroidal anti-inflammatory drugs. Sulfonamide allergy (rare). Oral Metabolic hypokalemic alkalosis • early hyponatremia • increased serum glucose, lipids, uric acid • efficacy reduced by nonsteroidal anti-inflammatory drugs. Carbonic anhydrase inhibitors Acetazolamide Inhibits carbonic anhydrase. In proximal tubule, bicarbonate reabsorption is blocked and Na+ is excreted with HCO3–. In glaucoma, secretion of aqueous humor is reduced, and in mountain sickness, metabolic acidosis increases respiration Dorzolamide, brinzolamide: topical carbonic anhydrase inhibitors for glaucoma only Loop diuretics Furosemide, also bumetanide, torsemide Inhibit Na+/K+/2Cl– transporter in thick ascending limb of loop of Henle. Cause powerful diuresis and increased Ca2+ excretion Heart failure, pulmonary edema, severe hypertension; other forms of edema; hypercalcemia Ethacrynic acid: like furosemide but not a sulfonamide and has some uricosuric effect Thiazide diuretics Hydrochlorothiazide, chlorthalidone (thiazide-like); many other thiazides Inhibit Na+/Cl– transporter in distal convoluted tubule. Cause moderate diuresis and reduced excretion of calcium Hypertension, mild heart failure, hypercalciuria with stones • nephrogenic diabetes insipidus Sulfonamide allergy (rare) K+-sparing diuretics Spironolactone, eplerenone Steroid inhibitors of cytoplasmic aldosterone receptor in cortical collecting ducts • reduce K+ excretion Excessive K+ loss when using other diuretics • heart failure • aldosteronism Oral Hyperkalemia • gynecomastia (spironolactone only) Amiloride Inhibitor of ENaC epithelial sodium channels in cortical collecting duct, reduces Na+ reabsorption and K+ excretion Excessive K+ loss when using other diuretics • usually in combination with thiazides Oral Hyperkalemia Inhibitors of sodium-glucose cotransporter in the proximal tubule, markedly increase glucose excretion Diabetes Oral Urinary tract infections Osmotically retains water in tubule by reducing reabsorption in proximal tubule, descending limb of Henle’s loop, and collecting ducts • in the periphery, mannitol extracts water from cells Solute overload in rhabdomyolysis, hemolysis, tumor lysis syndrome • brain edema with coma • acute glaucoma Intravenous; short duration Hyponatremia followed by hypernatremia • headache, nausea, vomiting Triamterene: like amiloride but much less potent SGLT2 inhibitors Canagliflozin, dapagliflozin Osmotic diuretics Mannitol (Continued ) 142 PART III Cardiovascular Drugs DRUG SUMMARY TABLE: Diuretic Agents (Continued ) Subclass Mechanism of Action Clinical Applications Pharmacokinetics Toxicities, Interactions Agonists at V1 and V2 ADH receptors, activate insertion of aquaporin water channels in collecting tubule, reduce water excretion • vasoconstriction Pituitary diabetes insipidus Subcutaneous, nasal Hyponatremia • hypertension Antagonist at V1a, V2 receptors SIADH, hyponatremia Parenteral Infusion site reactions ADH agonists Desmopressin, vasopressin ADH antagonists Conivaptan Tolvaptan: like conivaptan, more selective for V2 receptors Demeclocycline: used in SIADH, mechanism unclear ADH, antidiuretic hormone; SIADH, syndrome of inappropriate antidiuretic hormone. PART IV DRUGS WITH IMPORTANT ACTIONS ON SMOOTH MUSCLE C Histamine receptor blockers H1 blockers First generation (diphenhydramine) P T E R drugs (not autacoids) that interact with serotonin receptors, dopamine receptors, and α receptors. They are included in this chapter because of their effects on serotonin receptors and on smooth muscle. Peptide and eicosanoid autacoids are discussed in Chapters 17 and 18. Nitric oxide is discussed in Chapter 19. Serotonin receptor agonists and antagonists Agonists, partial agonists H2 blockers (cimetidine) Second generation (cetirizine) A 16 Histamine, Serotonin, & the Ergot Alkaloids Autacoids are endogenous molecules that do not fall into traditional autonomic groups. They do not act on cholinoceptors or adrenoceptors but have powerful pharmacologic effects on smooth muscle and other tissues. Histamine and serotonin (5-hydroxytryptamine; 5-HT) are the most important amine autacoids. The ergot alkaloids are a heterogeneous group of H 5-HT1 agonists (sumatriptan) Antagonists 5-HT2 antagonists (ketanserin) 5-HT4 partial agonists (tegaserod) 5-HT3 antagonists (ondansetron) Ergot alkaloids CNS, pituitary (LSD, bromocriptine) Uterus (ergonovine) Vessels (ergotamine) 143 144 PART IV Drugs with Important Actions on Smooth Muscle High-Yield Terms to Learn Acid-peptic disease Disease of the upper digestive tract caused by acid and pepsin; includes gastroesophageal reflux, erosions, and ulcers Autacoids Endogenous substances with complex physiologic and pathophysiologic functions that have potent nonautonomic pharmacologic effects when administered as drugs; commonly understood to include histamine, serotonin, prostaglandins, and vasoactive peptides Carcinoid A neoplasm of the gastrointestinal tract or bronchi that may secrete serotonin and a variety of peptides Ergotism (“St. Anthony's fire”) Disease caused by excess ingestion of ergot alkaloids; classically an epidemic caused by consumption of grain (eg, in bread) that is contaminated by the ergot fungus Gastrinoma A tumor that produces large amounts of gastrin; associated with hypersecretion of gastric acid and pepsin leading to ulceration IgE-mediated immediate reaction An allergic response, for example, hay fever, angioedema, caused by interaction of an antigen with IgE antibodies on mast cells; results in the release of histamine and other mediators of allergy Oxytocic A drug that causes contraction of the uterus Zollinger-Ellison syndrome Syndrome of hypersecretion of gastric acid and pepsin, often caused by gastrinoma; it is associated with severe acid-peptic ulceration and diarrhea HISTAMINE Histamine is formed from the amino acid histidine and is stored in high concentrations in vesicles in mast cells, enterochromaffin cells in the gut, some neurons, and a few other cell types. Histamine is metabolized by the enzymes monoamine oxidase and diamine oxidase. Excess production of histamine in the body (eg, in systemic mastocytosis) can be detected by measurement of its major metabolite, imidazole acetic acid, in the urine. Because it is released from mast cells in response to IgE-mediated (immediate) allergic reactions, this autacoid plays a pathophysiologic role in seasonal rhinitis (hay fever), urticaria, and angioneurotic edema. (The peptide bradykinin also plays an important role in angioneurotic edema, see Chapter 17.) Histamine also plays a physiologic role in the control of acid secretion in the stomach and as a neurotransmitter. A. Receptors and Effects Two receptors for histamine, H1 and H2, mediate most of the peripheral actions; 2 others (H3, H4) have also been identified (Table 16–1). The triple response, a classic demonstration of histamine effect, is mediated mainly by H1 and H2 receptors. This response involves a small red spot at the center of an intradermal injection of histamine surrounded by an edematous wheal, which is surrounded by a red flare. TABLE 16–1 Some histamine and serotonin receptor subtypes.a Receptor Subtype Distribution Postreceptor Mechanisms Prototypic Antagonist H1 Smooth muscle Gq; ↑ IP3, DAG Diphenhydramine H2 Stomach, heart, mast cells Gs; ↑ cAMP Cimetidine H3 Nerve endings, CNS Gi; ↓ cAMP Clobenpropitb H4 Leukocytes Gi; ↓ cAMP — 5-HT1D/1B Brain Gi; ↓ cAMP — 5-HT2 Smooth muscle, platelets Gq; ↑ IP3, DAG Ketanserin 5-HT3 Area postrema (CNS), sensory and enteric nerves Ligand-gated cation channel Ondansetron 5-HT4 Presynaptic nerve terminals in the enteric nervous system Gs; ↑ cAMP Tegaserod (partial agonist) a Many other serotonin receptor subtypes are recognized in the CNS. They are discussed in Chapter 21. b Clobenpropit is investigational. cAMP, cyclic adenosine phosphate; CNS, central nervous system; DAG, diacylglycerol; IP3, inositol trisphosphate. CHAPTER 16 Histamine, Serotonin, & the Ergot Alkaloids 1. H1 receptor—This Gq-coupled receptor is important in smooth muscle effects, especially those caused by IgE-mediated responses. Inositol trisphosphate (IP3) and diacylglycerol (DAG) are the second messengers. Typical responses include pain and itching in the skin, bronchoconstriction, and vasodilation, the latter caused by histamine-evoked release of nitric oxide. Capillary endothelial cells, in addition to releasing nitric oxide (NO) and other vasodilating substances, also contract, opening gaps in the permeability barrier and leading to the formation of local edema. These effects occur in allergic reactions and in mastocytosis. 2. H2 receptor—This Gs-coupled receptor mediates gastric acid secretion by parietal cells in the stomach. It also has a cardiac stimulant effect. A third action is to reduce histamine release from mast cells—a negative feedback effect. These actions are mediated by activation of adenylyl cyclase, which increases intracellular cyclic adenosine monophosphate (cAMP). 3. H3 receptor—This Gi-coupled receptor appears to be involved mainly in presynaptic modulation of histaminergic neurotransmission in the central nervous system (CNS). Food intake and body weight increase in H3-receptor knockout animals. In the periphery, it appears to be a presynaptic heteroreceptor with modulatory effects on the release of other transmitters (see Chapter 6). 4. H4 receptor—The H4 receptor is located on leukocytes (especially eosinophils) and mast cells and is involved in chemotactic responses by these cells. Like H3, it is Gi coupled. B. Clinical Use Histamine has no therapeutic applications, but drugs that block its effects at H1 and at H2 receptors are very important in clinical medicine. No antagonists of H3 or H4 receptors are currently available for clinical use. HISTAMINE H1 ANTAGONISTS A. Classification and Prototypes A wide variety of antihistaminic H1 blockers are available from several different chemical families. Two major subgroups or “generations” have been developed. The older members of the first-generation agents, typified by diphenhydramine, are highly sedating agents with significant autonomic receptor-blocking effects. A newer subgroup of first-generation agents is less sedating and has much less autonomic effect. Chlorpheniramine and cyclizine may be considered prototypes. The second-generation H1 blockers, typified by cetirizine, fexofenadine, and loratadine, are far less lipid soluble than the first-generation agents and have greatly reduced sedating and autonomic effects. All H1 blockers are active by the oral route. Several are promoted for topical use in the eye or nose. Most are metabolized extensively in the liver. Half-lives of the older H1 blockers vary from 4 to 12 h. Secondgeneration agents have half-lives of 12–24 h. 145 B. Mechanism and Effects H1 blockers are competitive pharmacologic antagonists or inverse agonists at the H1 receptor; these drugs have no effect on histamine release from storage sites. They are more effective if given before histamine release occurs. Because their structure closely resembles that of muscarinic blockers and α-adrenoceptor blockers, many of the first-generation agents are potent pharmacologic antagonists at these autonomic receptors. A few also block serotonin receptors. As noted, most older first-generation agents are sedating, and some—not all—first-generation agents have anti-motion sickness effects. Many H1 blockers are potent local anesthetics. H1-blocking drugs have negligible effects at H2 receptors. C. Clinical Use H1 blockers have major applications in allergies of the immediate type (ie, those caused by antigens acting on IgE antibody-sensitized mast cells). These conditions include hay fever and urticaria. Diphenhydramine, dimenhydrinate, cyclizine, meclizine, and promethazine are used as anti-motion sickness drugs. Diphenhydramine is also used for management of chemotherapy-induced vomiting. Doxylamine, in combination with pyridoxine, is promoted for the prevention of morning sickness in pregnancy.∗ Adverse effects of the first-generation H1 blockers are sometimes exploited therapeutically (eg, in their use as hypnotics in over-the-counter sleep aids). D. Toxicity and Interactions Sedation is common, especially with diphenhydramine and promethazine and these drugs should not be consumed before operating machinery. It is much less common with second-generation agents, which do not enter the CNS readily. Antimuscarinic effects such as dry mouth and blurred vision occur with some firstgeneration drugs in some patients. Alpha-adrenoceptor blockade, which is significant with phenothiazine derivatives such as promethazine, may cause orthostatic hypotension. Interactions occur between older antihistamines and other drugs with sedative effects (eg, benzodiazepines and alcohol). Drugs that inhibit hepatic metabolism may result in dangerously high levels of certain antihistaminic drugs that are taken concurrently. For example, azole antifungal drugs and certain other CYP3A4 inhibitors interfere with the metabolism of astemizole and terfenadine, 2 second-generation agents that have been withdrawn from the US market because high plasma concentrations of either antihistamine can precipitate lethal arrhythmias. HISTAMINE H2 ANTAGONISTS A. Classification and Prototypes Four H2 blockers are available; cimetidine is the prototype. Ranitidine, famotidine, and nizatidine differ only in having ∗Doxylamine with pyridoxine was originally available as Bendectin but was withdrawn due to an unwarranted fear of teratogenic effects. It is again available in the USA as Diclegis. 146 PART IV Drugs with Important Actions on Smooth Muscle fewer adverse effects than cimetidine. These drugs do not resemble H1 blockers structurally. They are orally active, with half-lives of 1–3 h. Because they are all relatively nontoxic, they can be given in large doses, so that the duration of action of a single dose may be 12–24 h. All four agents are available in oral over-thecounter formulations. B. Mechanism and Effects H2 antagonists produce a surmountable pharmacologic blockade of histamine H2 receptors. They are relatively selective and have no significant blocking actions at H1 or autonomic receptors. The only therapeutic effect of clinical importance is the reduction of gastric acid secretion, but this is a very useful action. Blockade of cardiovascular and mast cell H2-receptor-mediated effects can be demonstrated but has no clinical significance. C. Clinical Use In acid-peptic disease, especially duodenal ulcer, these drugs reduce nocturnal acid secretion, accelerate healing, and prevent recurrences. Acute ulcer is usually treated with 2 or more doses per day, whereas recurrence of duodenal ulcers can often be prevented with a single bedtime dose. H2 blockers are also effective in accelerating healing and preventing recurrences of gastric peptic ulcers. Intravenous H2 blockers are useful in preventing gastric erosions and hemorrhage that occur in stressed patients in intensive care units. In Zollinger-Ellison syndrome, which is associated with gastrinoma and characterized by acid hypersecretion, severe recurrent peptic ulceration, gastrointestinal bleeding, and diarrhea, these drugs are helpful, but very large doses are required; proton pump inhibitors are preferred. Similarly, the H2 blockers have been used in gastroesophageal reflux disease (GERD), but they are not as effective as proton pump inhibitors (see Chapter 60). SKILL KEEPER: ANTIHISTAMINE ADVERSE EFFECTS (SEE CHAPTERS 8 AND 10) An elderly dental patient was given promethazine intravenously to reduce anxiety before undergoing an extraction in the dental office. Promethazine is an older first-generation antihistamine. Predict the CNS and autonomic effects of this drug when given intravenously. The Skill Keeper Answer appears at the end of the chapter. D. Toxicity Cimetidine is a potent inhibitor of hepatic drug-metabolizing enzymes (see Chapter 4) and may also reduce hepatic blood flow. Cimetidine also has significant antiandrogen effects in patients receiving high doses. Ranitidine has a weaker inhibitory effect on hepatic drug metabolism; neither it nor the other H2 blockers appear to have any endocrine effects. SEROTONIN (5-HYDROXYTRYPTAMINE; 5-HT) & RELATED AGONISTS Serotonin is produced from tryptophan and stored in vesicles in the enterochromaffin cells of the gut and neurons of the CNS and enteric nervous system. After release, it is metabolized by monoamine oxidase. Excess production in the body (eg, in carcinoid syndrome) can be detected by measuring its major metabolite, 5-hydroxyindole acetic acid (5-HIAA), in the urine. Serotonin plays a physiologic role as a neurotransmitter in both the CNS and the enteric nervous system and may have a role as a local hormone that modulates gastrointestinal activity. After release from neurons, it is partially taken back up into the nerve ending by a serotonin reuptake transporter (SERT). Serotonin is also stored (but synthesized to only a minimal extent) in platelets. In spite of the very large number of serotonin receptors (14 identified to date), most of the serotonin agonists in clinical use act at 5-HT1D/1B and 5-HT2C receptors. Serotonin antagonists in use or under investigation act at 5-HT2 and 5-HT3 receptors (see drug overview figure at the beginning of the chapter). A. Receptors and Effects 1. 5-HT1 receptors—5-HT1 receptors are most important in the brain and mediate synaptic inhibition via increased potassium conductance (Table 16–1). Peripheral 5-HT1 receptors mediate both excitatory and inhibitory effects in various smooth muscle tissues. 5-HT1 receptors are Gi-protein-coupled. 2. 5-HT2 receptors—5-HT2 receptors are important in both brain and peripheral tissues. These receptors mediate synaptic excitation in the CNS and smooth muscle contraction (gut, bronchi, uterus, some vessels) or relaxation (other vessels). Several mechanisms are involved, including (in different tissues) increased IP3, decreased potassium conductance, and decreased cAMP. This receptor probably mediates some of the vasodilation, diarrhea, and bronchoconstriction that occur as symptoms of carcinoid tumor, a neoplasm that releases serotonin and other substances. In the CNS, 5-HT2C receptors mediate a reduction in appetite that has been used in the treatment of obesity. 3. 5-HT3 receptors—5-HT3 receptors are found in the CNS, especially in the chemoreceptive area and vomiting center, and in peripheral sensory and enteric nerves. These receptors mediate excitation via a 5-HT-gated cation channel. Antagonists acting at this receptor are extremely useful antiemetic drugs. 4. 5-HT4 receptors—5-HT4 receptors are found in the gastrointestinal tract and play an important role in intestinal motility. B. Clinical Uses Serotonin has no clinical applications, but other more selective agonists are useful. CHAPTER 16 Histamine, Serotonin, & the Ergot Alkaloids 1. 5-HT1D/1B agonists—Sumatriptan is the prototype. Naratriptan and other “-triptans” are similar to sumatriptan (see Drug Summary Table). They are the first-line treatment for acute migraine and cluster headache attacks, an observation that strengthens the association of serotonin abnormalities with these headache syndromes. These drugs are active orally; sumatriptan is also available for nasal and parenteral administration. Ergot alkaloids, discussed later, are partial agonists at some 5-HT receptors. 2. 5-HT2C agonists—Lorcaserin has recently been approved for the treatment of obesity. It activates receptors in the CNS and appears to moderately reduce appetite. Older drugs, fenfluramine and dexfenfluramine, appear to act directly and by releasing neuronal 5-HT or inhibiting SERT, and thereby activating central 5-HT2C receptors. They were withdrawn in the USA because their use was associated with damage to cardiac valves. Dexfenfluramine was combined with phentermine, an amphetamine-like anorexiant, in a weight-loss product known as “dex-phen.” Because of toxicity, this combination product is also banned in the USA. 3. 5-HT4 Partial agonist—Tegaserod is a newer drug that acts as an agonist in the colon. It was approved and briefly marketed for use in chronic constipation, but because of cardiovascular toxicity, its use is now restricted. 4. Selective serotonin reuptake inhibitors (SSRI)—A number of important antidepressant drugs act to increase activity at central serotonergic synapses by inhibiting the serotonin reuptake transporter, SERT. These drugs are discussed in Chapter 30. C. Hyperthermic Syndromes Serotonin and drugs with 5-HT agonist effects are sometimes associated with drug reactions with high fever, skeletal muscle 147 effects, and cardiovascular abnormalities that can be life-threatening. These important syndromes are summarized in Table 16–2. SEROTONIN ANTAGONISTS A. Classification and Prototypes Ketanserin, phenoxybenzamine, and cyproheptadine are effective 5-HT2 blockers. Ondansetron, granisetron, dolasetron, and alosetron are 5-HT3 blockers. The ergot alkaloids are partial agonists (and therefore have some antagonist effects) at 5-HT and other receptors (see later discussion). B. Mechanisms and Effects Ketanserin and cyproheptadine are competitive pharmacologic 5-HT2 antagonists. Phenoxybenzamine (see Chapter 10) is an irreversible blocker at this receptor. Ketanserin, cyproheptadine, and phenoxybenzamine are poorly selective agents. In addition to inhibition of serotonin effects, other actions include α-blockade (ketanserin, phenoxybenzamine) or H1-blockade (cyproheptadine). Ondansetron, granisetron, and dolasetron are selective 5-HT3 receptor blockers and have important antiemetic actions in the area postrema of the medulla and also on peripheral sensory and enteric nerves. Although it acts at the 5-HT3 receptor, alosetron appears to lack these antiemetic effects. C. Clinical Uses Ketanserin is used as an antihypertensive drug outside the United States. Ketanserin, cyproheptadine, and phenoxybenzamine may be of value (separately or in combination) in the treatment of carcinoid tumor, a neoplasm that secretes large amounts of TABLE 16–2 Characteristics of serotonin syndrome and other hyperthermic syndromes. Syndrome Precipitating Drugs Clinical Presentation Therapya Serotonin syndrome SSRIs, second-generation antidepressants, MAOIs, linezolid, tramadol, meperidine, fentanyl, ondansetron, sumatriptan, MDMA, LSD, St. John's wort, ginseng Hyperthermia, hyperreflexia, tremor, clonus, hypertension, hyperactive bowel sounds, diarrhea, mydriasis, agitation, coma; onset within hours Sedation (benzodiazepines), paralysis, intubation and ventilationb; consider 5-HT2 block with cyproheptadine or chlorpromazine Neuroleptic malignant syndrome D2-blocking antipsychotic drugs Hyperthermia, acute severe parkinsonism; hypertension, normal or reduced bowel sounds, onset over 1–3 days Diphenhydramine (parenteral), cooling if temperature is very high, sedation with benzodiazepines Malignant hyperthermia Volatile anesthetics, succinylcholine Hyperthermia, muscle rigidity, hypertension, tachycardia; onset within minutes Dantrolene, cooling a Precipitating drugs should be discontinued immediately. b All first-line therapy is in bold font. MAOIs, monoamine oxidase inhibitors; MDMA, methylenedioxy-methamphetamine (ecstasy); SSRIs, selective serotonin reuptake inhibitors. Modified and reproduced, with permission, from Katzung BG, editor: Basic & Clinical Pharmacology, 12th ed. McGraw-Hill, 2012, p. 284. 148 PART IV Drugs with Important Actions on Smooth Muscle TABLE 16–3 Effects of some ergot alkaloids at several receptors. Serotonin Receptor (5-HT2) Uterine Smooth Muscle Stimulation +++ − 0 ++ − (PA) +++ ++ − (PA) 0 + (PA) +++ 0 +++ − −/++ in CNS + Ergot Alkaloid Alpha Receptor (`1) Bromocriptine − Ergonovine Ergotamine Lysergic acid diethylamide (LSD) Dopamine Receptor (D2) Agonist effects are indicated by +, antagonist by −, no effect by 0. Relative affinity for the receptor is indicated by the number of + or − signs. PA, partial agonist. Modified and reproduced, with permission, from Katzung BG, editor: Basic & Clinical Pharmacology, 12th ed. McGraw-Hill, 2012, p. 288. serotonin (and peptides) and causes diarrhea, bronchoconstriction, and flushing. Ondansetron and its congeners are extremely useful in the control of vomiting associated with cancer chemotherapy and postoperative vomiting. Alosetron is used in the treatment of women with irritable bowel syndrome associated with diarrhea. D. Toxicity Adverse effects of ketanserin are those of α blockade and H1 blockade. The toxicities of ondansetron, granisetron, and dolasetron include diarrhea and headache. Dolasetron has been associated with QRS and QTc prolongation in the ECG and should not be used in patients with heart disease. Alosetron causes significant constipation in some patients and has been associated with fatal bowel complications. ERGOT ALKALOIDS These complex molecules are produced by a fungus found in wet or spoiled grain. They are responsible for the epidemics of “St. Anthony’s fire” (ergotism) described during the Middle Ages and recurring to the present time. There are at least 20 naturally occurring members of the family, but only a few of these and a handful of semisynthetic derivatives are used as therapeutic agents. Most ergot alkaloids are partial agonists at α adrenoceptors and 5-HT receptors, and some are potent agonists at dopamine receptors. A. Classification and Effects The ergot alkaloids may be divided into 3 major subgroups on the basis of the organ or tissue in which they have their primary effects. The receptor effects of the ergot alkaloids are summarized in Table 16–3 and are most marked in the following tissues: 1. Vessels—Ergot alkaloids can produce marked and prolonged α-receptor–mediated vasoconstriction. Ergotamine is the prototype. An overdose can cause ischemia and gangrene of the limbs or bowel. Because they are partial agonists, the drugs may also block the α-agonist effects of sympathomimetics, and ergotamine can cause epinephrine reversal. 2. Uterus—Ergot alkaloids produce powerful contraction in this tissue, especially near term. Ergonovine is the prototype. In pregnancy, the uterine contraction is sufficient to cause abortion or miscarriage. Earlier in pregnancy (and in the nonpregnant uterus) much higher doses of ergot alkaloids are needed to cause contraction. 3. Brain—Hallucinations may be prominent with the naturally occurring ergots and with lysergic acid diethylamide (LSD), a semisynthetic prototypical hallucinogenic ergot derivative, but are uncommon with the therapeutic ergot derivatives. Although LSD is a potent 5-HT2 blocker in peripheral tissues, its actions in the CNS are thought to be due to agonist actions at dopamine receptors. In the pituitary, some ergot alkaloids are potent dopamine-like agonists and inhibit prolactin secretion. Bromocriptine and pergolide are among the most potent semisynthetic ergot derivatives. They act as dopamine D2 agonists in the pituitary and the basal ganglia (see Chapter 28). B. Clinical Uses 1. Migraine—Ergotamine has been a mainstay of treatment of acute attacks and is still used in combination with caffeine. Methysergide, dihydroergonovine, and ergonovine have been used for prophylaxis, but methysergide is no longer available in the United States. The triptan derivatives are now considered preferable to the ergots because of lower toxicity. 2. Obstetric bleeding—Ergonovine and ergotamine are effective agents for the reduction of postpartum bleeding. They produce a powerful and long-lasting contraction that reduces bleeding but must not be given before delivery of the placenta. 3. Hyperprolactinemia and parkinsonism—Bromocriptine, pergolide, and cabergoline have been used to reduce prolactin secretion (dopamine is the physiologic prolactin release inhibitor; Chapter 37). Pergolide has been withdrawn from the US market. Bromocriptine also appears to reduce the size of pituitary tumors of the prolactin-secreting cells. Bromocriptine and cabergoline are used in hyperprolactinemia and off label to treat acromegaly. These drugs have been used in the treatment of Parkinson’s disease, but other drugs are preferred (see Chapter 28). C. Toxicity The toxic effects of ergot alkaloids are quite important, both from a public health standpoint (epidemics of ergotism from spoiled CHAPTER 16 Histamine, Serotonin, & the Ergot Alkaloids grain) and from the toxicity resulting from overdose or abuse by individuals. Intoxication of grazing animals is sometimes reported by farmers and veterinarians. 1. Vascular effects—Severe prolonged vasoconstriction can result in ischemia and gangrene. The most consistently effective antidote is nitroprusside. When used for long periods, ergot derivatives may produce an unusual hyperplasia of connective tissue. This fibroplasia may be retroperitoneal, retropleural, or subendocardial and can cause hydronephrosis or cardiac valvular and conduction system malfunction. Similar lesions are found in some patients with carcinoid, suggesting that this action is probably mediated by agonist effects at serotonin receptors. 2. Gastrointestinal effects—Ergot alkaloids cause gastrointestinal upset (nausea, vomiting, diarrhea) in many persons. 3. Uterine effects—Marked uterine contractions may be produced. The uterus becomes progressively more sensitive to ergot alkaloids during pregnancy. Although abortion resulting from the use of ergot for migraine is rare, most obstetricians recommend avoidance or very conservative use of these drugs as pregnancy progresses. 4. CNS effects—Hallucinations resembling psychosis are common with LSD but less so with the other ergot alkaloids. Methysergide was occasionally used in the past as an LSD substitute by users of “recreational” drugs. QUESTIONS 1. Your 37-year-old patient has been diagnosed with a rare metastatic carcinoid tumor. This neoplasm is releasing serotonin, bradykinin, and several unknown peptides. The effects of serotonin in this patient are most likely to include (A) Constipation (B) Episodes of bronchospasm (C) Hypersecretion of gastric acid (D) Hypotension (E) Urinary retention 2. A 23-year-old woman suffers from recurrent episodes of angioneurotic edema with release of histamine and other mediators. Which of the following drugs is the most effective physiologic antagonist of histamine in smooth muscle? (A) Cetirizine (B) Epinephrine (C) Granisetron (D) Ranitidine (E) Sumatriptan 3. A 20-year-old woman is taking diphenhydramine for severe hay fever. Which of the following adverse effects is she most likely to report? (A) Muscarinic increase in bladder tone (B) Nausea (C) Nervousness, anxiety (D) Sedation (E) Vertigo 149 4. A laboratory study of new H2 blockers is planned. Which of the following will result from blockade of H2 receptors? (A) Increased cAMP (cyclic adenosine monophosphate) in cardiac muscle (B) Decreased channel opening in enteric nerves (C) Decreased cAMP in gastric mucosa (D) Increased IP3 (inositol trisphosphate) in platelets (E) Increased IP3 in smooth muscle 5. You are asked to consult on a series of cases of drug toxicities. Which of the following is a recognized adverse effect of cimetidine? (A) Blurred vision (B) Diarrhea (C) Orthostatic hypotension (D) P450 hepatic enzyme inhibition (E) Sedation 6. A 40-year-old patient is about to undergo cancer chemotherapy with a highly emetogenic (nausea- and vomiting-causing) drug combination. The antiemetic drug most likely to be included in her regimen is (A) Bromocriptine (B) Cetirizine (C) Cimetidine (D) Ketanserin (E) Ondansetron 7. The hospital Pharmacy Committee is preparing a formulary for staff use. Which of the following is a correct application of the drug mentioned? (A) Alosetron: for obstetric bleeding (B) Cetirizine: for hay fever (C) Ergonovine: for Alzheimer’s disease (D) Ondansetron: for acute migraine headache (E) Ranitidine: for Parkinson’s disease 8. A 26-year-old woman presents with amenorrhea and galactorrhea. Her prolactin level is grossly elevated (200 ng/mL vs normal 20 ng/mL). Which of the following is most useful in the treatment of hyperprolactinemia? (A) Bromocriptine (B) Cimetidine (C) Ergotamine (D) Ketanserin (E) LSD (F) Ondansetron (G) Sumatriptan 9. A 28-year-old office worker suffers from intense migraine headaches. Which of the following is a serotonin agonist useful for aborting an acute migraine headache? (A) Bromocriptine (B) Cimetidine (C) Ephedrine (D) Ketanserin (E) Loratadine (F) Ondansetron (G) Sumatriptan 150 PART IV Drugs with Important Actions on Smooth Muscle 10. A 33-year-old woman attempted to induce an abortion using ergotamine. She is admitted to the emergency department with severe pain in both legs. On examination, her legs are cold and pale with absent arterial pulses. Which of the following is the most useful antidote for reversing severe ergotinduced vasospasm? (A) Bromocriptine (B) Cimetidine (C) Ergotamine (D) Ketanserin (E) LSD (F) Nitroprusside (G) Sumatriptan (H) Ondansetron ANSWERS 1. Serotonin causes bronchospasm, but the other effects listed are not observed. Carcinoid is associated with diarrhea and hypertension. The answer is B. 2. The smooth muscle effects of histamine are mediated mainly by H1 receptors. Cetirizine is a pharmacologic antagonist of histamine at these receptors. Granisetron is a 5-HT3 antagonist. Sumatriptan is a 5-HT1D/1B agonist. Ranitidine is a histamine antagonist but blocks the H2 receptor in the stomach and the heart, not H1 receptors in smooth muscle. Epinephrine has a physiologic antagonist action that reverses histamine’s effects on smooth muscle. The answer is B. 3. H1 blockers do not activate muscarinic receptors, mediate vasoconstriction, or cause vertigo. Some relieve vertigo or motion sickness. They do not cause nervousness or anxiety. Diphenhydramine is a potent sedative. The answer is D. 4. H2 receptors are Gs-protein-coupled receptors, like β adrenoceptors. Blockade of this system will cause a decrease in cAMP. The answer is C. 5. The older H1 blockers, not H2 blockers, cause blurred vision, orthostatic hypotension, and sedation. Neither group typically causes diarrhea. Cimetidine (unlike other H2 blockers) is a potent CYP3A4 inhibitor. The answer is D. 6. Ondansetron and other 5-HT3 antagonists have significant antiemetic effects. Diphenhydramine and prednisone are also used for this purpose. The answer is E. 7. Alosetron is indicated in irritable bowel syndrome. Ergonovine is used in uterine bleeding. Ondansetron is useful for chemotherapy-induced emesis. Cetirizine, a second-generation H1 blocker, is used in the treatment of hay fever. The answer is B. 8. Bromocriptine is an effective dopamine agonist in the CNS with the advantage of oral activity. The drug inhibits prolactin secretion by activating pituitary dopamine receptors. The answer is A. 9. Sumatriptan, an agonist at 5-HT1D receptors, is indicated for prevention or treatment of migraine and cluster headaches. Ergotamine (not on the list) is also effective for acute migraine but is produced by the fungus Claviceps purpurea. The answer is G. 10. A very powerful vasodilator is necessary to reverse ergotinduced vasospasm; nitroprusside is such a drug (see Chapter 11). The answer is F. SKILL KEEPER ANSWER: ANTIHISTAMINE ADVERSE EFFECTS (SEE CHAPTERS 8 AND 10) Promethazine very effectively alleviated the anxiety of this elderly woman. However, when she attempted to get out of the dental chair after the procedure, she experienced severe orthostatic hypotension and fainted. In the horizontal position on the floor and later on a couch, she rapidly regained consciousness. Supine blood pressure was low normal, and heart rate was elevated. When she sat up, blood pressure dropped and heart rate increased. Promethazine and several other first-generation H1 antihistamines are effective α (and M3) blockers (Chapters 8 and 10). After 30 min supine, the patient was able to stand without fainting and experienced only a slight tachycardia. Older antihistaminic agents readily enter the CNS, causing sedation. This patient felt somewhat sleepy for 2 h but had no further signs or symptoms. If she had glaucoma, she might be at risk for an acute angle-closure episode, with markedly increased intraocular pressure as a result of the antimuscarinic action. An elderly man with prostatic hyperplasia might experience urinary retention. CHECKLIST When you complete this chapter, you should be able to: ❑ List the major organ system effects of histamine and serotonin. ❑ Describe the pharmacology of the 3 subgroups of H1 antihistamines; list prototypical agents for each subgroup. ❑ Describe the pharmacology of the H2 antihistamines; name 2 members of this group. ❑ Describe the action and indication for the use of sumatriptan. ❑ Describe one 5-HT2 and one 5-HT3 antagonist and their major applications. ❑ List the major organ system effects of the ergot alkaloids. ❑ Describe the major clinical applications and toxicities of the ergot drugs. CHAPTER 16 Histamine, Serotonin, & the Ergot Alkaloids 151 DRUG SUMMARY TABLE: Histamine, Serotonin, & the Ergot Alkaloids Subclass Mechanism of Action Clinical Applications Pharmacokinetics Toxicities, Interactions Hay fever, angioedema, motion sickness • used orally as OTC sleep aid; used parenterally for dystonias Oral, parenteral Duration: 6–8 h Sedation, autonomic block Rare CNS excitation H1 blockers, first generation Diphenhydramine, dimenhydrinate Competitive pharmacologic block of peripheral and CNS H1 receptors plus α- and M-receptor block. Anti-motion sickness effect Promethazine: H1 blocker with less anti-motion sickness action and more sedative and autonomic effects Cyclizine: H1 blocker with more anti-motion sickness action and less sedative and autonomic effect Chlorpheniramine: H1 blocker with negligible anti-motion sickness, sedative, and autonomic effects H1 blockers, second generation Cetirizine Competitive pharmacologic block of peripheral H1 receptors. No autonomic or anti-motion sickness effects Hay fever, angioedema Oral Duration: 12–24 h Minimal toxicities Fexofenadine, loratadine, desloratadine: very similar to cetirizine H2 blockers Cimetidine famotidine, ranitidine, nizatidine Competitive pharmacologic block. No H1, autonomic, or anti-motion effects Gastroesophageal reflux disease, stress ulcers Oral, parenteral Duration (large doses): 12–24 h Cimetidine: drug interactions; other H2 blockers much less 5-HT1D/1B agonist • causes vasoconstriction • modulates neurotransmitter release Migraine and cluster headache Oral, inhaled, parenteral Duration: 2–4 h Paresthesias, dizziness, chest pain • possible coronary vasospasm 5-HT1 agonists Sumatriptan Almotriptan, eletriptan, frovatriptan, naratriptan, rizatriptan, zolmitriptan: very similar to sumatriptan; injectable preparations not available; durations: 2–27 h 5-HT2 antagonists Ketanserin Competitive 5-HT2 and α1receptor block Hypertension, carcinoid tumor (not available in United States) Oral Duration: 12–24 h Hypotension Pharmacologic antagonist • blocks chemoreceptor trigger zone and enteric nervous system 5-HT3 receptors Chemotherapy and postoperative vomiting Oral, IV Duration: 3–6 h QT prolongation, possible arrhythmias 5-HT3 antagonists Ondansetron Granisetron, dolasetron, palonosetron: like ondansetron Alosetron: approved for treatment of diarrhea-predominant irritable bowel syndrome 5-HT4 partial agonist Tegaserod Partial agonist at 5-HT4 receptors Constipation-dominant irritable bowel syndrome (restricted use) Oral Duration: 12 h Diarrhea, ischemic colitis Ergotamine Partial agonist at 5-HT and α adrenoceptors, especially in vessels Migraine, cluster headache Oral Duration 10–12 h Nausea, vomiting, diarrhea, severe vasospasm Ergonovine Partial agonist at 5-HT and α adrenoceptors, especially in uterus Postpartum uterine bleeding Oral Duration 10–12 h Nausea, vomiting, diarrhea, severe vasospasm Lysergic acid diethylamide (LSD) Partial 5-HT2 agonist; CNS dopamine D2 agonist None (abused hallucinogen) Oral Duration hours ANS activation, cardiovascular instability (see Chapter 32) Bromocriptine Partial agonist at dopamine receptors Prolactinemia Oral Duration 10–20 h Hallucinations Ergot alkaloids OTC, over the counter. C A P T E R 17 Vasoactive Peptides Vasoactive peptides are autacoids with significant actions on vascular smooth muscle as well as other tissues. They include vasoconstrictors, vasodilators, and peptides with mixed effects. H Antagonists of these peptides or the enzymes that produce them have useful clinical properties. Vasoactive peptides Vasoconstrictors (angiotensin II, endothelins, neuropeptide Y) Mixed (substance P) Vasodilators (bradykinin, BNP, ANP, CGRP, VIP) Antagonists of peptides Renin (aliskiren) ACE (captopril) Angiotensin Vasopressin (losartan) (conivaptan) In addition to their actions on smooth muscle, many vasoactive peptides also function as neurotransmitters and local and systemic hormones. The most important vasoactive peptides include angiotensin, bradykinin, natriuretic peptides, calcitonin gene-related peptide (CGRP), endothelins, neuropeptide Y (NPY), substance P and vasoactive intestinal peptide (VIP) (discussed in this chapter), and vasopressin (Chapters 15 and 37). Many other endogenous peptides with very important actions (eg, insulin, glucagon, opioid peptides) have less or no direct vascular smooth muscle effects. Vasoactive peptides probably all act on cell surface receptors. Most act via G protein-coupled receptors and cause the production of wellknown second messengers (Table 17–1); a few may open ion channels. ANGIOTENSIN & ITS ANTAGONISTS A. Source and Disposition Angiotensin I is produced from circulating angiotensinogen by renin, an enzyme released from the juxtaglomerular apparatus of the 152 Endothelin (bosentan) Vasopeptidase (omapatrilat) Substance P (aprepitant) kidney. Angiotensin I is an inactive decapeptide, and is converted into angiotensin II (ANG II, also denoted AII), an active octapeptide, by angiotensin-converting enzyme (ACE), also known as peptidyl dipeptidase or kininase II (see Figure 11–3). Angiotensin II, the active form of the peptide, is rapidly degraded by peptidases (angiotensinases). B. Effects and Clinical Role ANG II is a potent arteriolar vasoconstrictor and stimulant of aldosterone release. ANG II directly increases peripheral vascular resistance and, through aldosterone, causes renal sodium retention. It also facilitates the release of norepinephrine from adrenergic nerve endings via presynaptic heteroreceptor action (see Chapter 6). All these effects are mediated by the angiotensin AT1 receptor, a Gq-coupled receptor. The AT2 receptor appears to mediate vasodilation via nitric oxide and is probably most important during fetal development. ANG II is also mitogenic and plays a role in cardiac remodeling. CHAPTER 17 Vasoactive Peptides 153 High-Yield Terms to Learn Kinins Family of vasoactive peptides associated with tissue injury and inflammation, for example, bradykinin Natriuretic peptides Family of peptides synthesized in brain, heart, and other tissues; have vasodilator as well as natriuretic effects Neuropeptides Peptides with prominent roles as neurotransmitters or modulators; many also have potent smooth muscle effects Peptidase Family of enzymes that activate or inactivate peptides by hydrolysis, for example, angiotensin-converting enzyme (dipeptidyl peptidase), neutral endopeptidase Tachykinins Group of 3 potent neuropeptides: substance P, neurokinin A, and neurokinin B ANG II is no longer used for clinical indications. Its major significance is as an endogenous pathophysiologic mediator in some cases of hypertension (high-renin hypertension) and in heart failure. Regardless of renin levels, ANG II antagonists have demonstrated clinical benefits in hypertension and heart failure. Therefore, ANG II antagonists are of considerable clinical importance. C. Angiotensin Antagonists As noted in Chapters 11 and 13, 2 types of antagonists are available. ACE inhibitors (eg, captopril, enalapril, others) are important orally active nonpeptide agents for the treatment of hypertension and heart failure. ANG II receptor blockers (ARBs, eg, losartan, valsartan, others) are inhibitors at the ANG II AT1 receptor and are also orally active nonpeptides. Block of angiotensin’s effects by either of these drug types is often accompanied by a compensatory increase in renin and angiotensin I. While ACE inhibitors increase the circulating levels of bradykinin, ARBs lack this property and are less likely to cause cough. Aliskiren, a newer orally active renin inhibitor, reduces angiotensin I as well as angiotensin II and is approved for use in hypertension. VASOPEPTIDASE INHIBITORS The vasopeptidase enzymes include neutral endopeptidase 24.11 and ACE. A class of drugs that block both enzymes is in clinical trials, and these drugs (eg, omapatrilat) show considerable efficacy in hypertension and heart failure. They reduce the concentration of ANG II and increase the concentration of natriuretic peptides (discussed below). Unfortunately, these drugs also cause angioedema in a significant number of patients and have not been approved for clinical use. TABLE 17–1 Some vasoactive peptides and their properties. Peptide Properties Angiotensin II (ANG II) ↑ IP3, DAG via AT1 G protein-coupled receptors. Constricts arterioles, increases aldosterone secretion Bradykinin ↑ IP3, DAG, cAMP, NO. Dilates arterioles, increases capillary permeability, stimulates sensory nerve endings Natriuretic peptides (ANP, BNP) ↑ cGMP via ANPA receptors. Dilate vessels, inhibit aldosterone secretion and effects, increase glomerular filtration Calcitonin gene-related peptide (CGRP) An extremely potent vasodilator; causes hypotension and reflex tachycardia Endothelins ↑ IP3, DAG via G protein-coupled ETA and ETB receptors. Synthesized in vascular endothelium. Constrict most vessels, may play a pathophysiologic role in pulmonary hypertension Neuropeptide Y Causes vasoconstriction and stimulates the heart. Effects mediated in part by IP3 Substance P, neurokinins Act on neurokinin receptors (NK1, NK2, NK3). Dilate arterioles, contract veins and intestinal and bronchial smooth muscle, cause diuresis; substance P is a transmitter in sensory pain neurons Vasoactive intestinal peptide (VIP) ↑ cAMP via G protein-coupled receptors VPAC1 and VPAC2. Dilates vessels, relaxes bronchi and intestinal smooth muscle ANP, atrial natriuretic peptide; BNP, brain natriuretic peptide; cAMP, cyclic adenosine monophosphate; cGMP, cyclic guanosine monophosphate; DAG, diacylglycerol; IP3, inositol trisphosphate. 154 PART IV Drugs with Important Actions on Smooth Muscle BRADYKININ ENDOTHELINS A. Source and Disposition Bradykinin is one of several vasodilator kinins produced from kininogen by a family of enzymes, the kallikreins. Bradykinin is rapidly degraded by various peptidases, including ACE. Endothelins are peptide vasoconstrictors formed in and released by endothelial cells in blood vessels. Endothelins appear to function as autocrine and paracrine hormones in the vasculature. Three endothelin peptides (ET-1, ET-2, and ET-3) with minor variations in amino acid sequence have been identified in humans. Two receptors, ETA and ETB, have been identified, both of which are G-protein-coupled to their effectors. The ETA receptor appears to be responsible for the vasoconstriction produced by endothelins. Endothelins are much more potent than norepinephrine as vasoconstrictors and have a relatively long-lasting effect. The peptides also stimulate the heart, increase natriuretic peptide release, and activate smooth muscle proliferation. The peptides may be involved in some forms of hypertension and other cardiovascular disorders. ETA antagonists available for the treatment of pulmonary hypertension include bosentan and ambrisentan. Macitentan, a newer dual inhibitor of both endothelin receptors, is also available for use in pulmonary hypertension. Riociguat is an oral activator of soluble guanylyl cyclase (not an ET receptor antagonist) that is also approved for use in pulmonary hypertension. B. Effects and Clinical Role Bradykinin acts through at least 2 receptors (B1 and B2) and causes the production of inositol 1,4,5-trisphosphate (IP3), diacylglycerol (DAG), cyclic adenosine monophosphate (cAMP), nitric oxide, and prostaglandins in tissues. Bradykinin is one of the most potent vasodilators known. The peptide is involved in inflammation and causes edema, vasodilation, and pain when released or injected into tissue. Bradykinin can be found in saliva and may play a role in stimulating its secretion. Although it has no therapeutic application, bradykinin may play a role in the antihypertensive action of ACE inhibitors, as previously noted (see Chapter 11; Figure 11–3). Bradykinin also plays a role in hereditary angioedema. Ecallantide, a parenteral kallikrein inhibitor, and icatibant, a parenteral bradykinin B2-receptor antagonist, are approved for use in angioedema. NATRIURETIC PEPTIDES A. Source and Disposition Natriuretic peptides (atrial natriuretic peptide [ANP] and brain natriuretic peptide [BNP]) are synthesized and stored in the cardiac atria of mammals. BNP has also been isolated from brain tissue. They are released from the atria in response to distention of the chambers. A similar peptide, C-type natriuretic peptide, has been isolated from other tissues. BNP appears to be the most important of these peptides. B. Effects and Clinical Role Natriuretic peptides activate guanylyl cyclase in many tissues via a membrane-spanning enzyme receptor. They act as vasodilators as well as natriuretic (sodium excretion-enhancing) agents. Their renal action includes increased glomerular filtration, decreased proximal tubular sodium reabsorption, and inhibitory effects on renin secretion. The peptides also inhibit the actions of ANG II and aldosterone. Although they lack positive inotropic action, endogenous natriuretic peptides may play an important compensatory role in congestive heart failure by limiting sodium retention. Blood levels of endogenous BNP have been shown to correlate with the severity of heart failure and can be used as a diagnostic marker. BNP administered as a drug has shown some benefit in the treatment of acute severe heart failure and is currently available for clinical use as nesiritide. This drug is approved for intravenous administration in acute severe heart failure (see Chapter 13) but has very significant toxicity. VIP, SUBSTANCE P, CGRP, & NPY VIP (vasoactive intestinal peptide) is an extremely potent vasodilator but is probably more important as a neurotransmitter. It is found in the central and peripheral nervous systems and in the gastrointestinal tract. No clinical application has been found for this peptide. The neurokinins, also known as tachykinins, include substance P, neurokinin A, and neurokinin B. They act at NK1, NK2, and NK3 receptors in the central nervous system (CNS) and the periphery. Substance P has mixed vascular effects. It is a potent arteriolar vasodilator and a potent stimulant of veins and intestinal and airway smooth muscle. The peptide may also function as a local hormone in the gastrointestinal tract. Highest concentrations of substance P are found in the parts of the nervous system that contain neurons subserving pain. Capsaicin, the “hot” component of chili peppers, releases substance P from its stores in nerve endings and depletes the peptide. Capsaicin has been approved for topical use on arthritic joints and for postherpetic neuralgia. Neurokinins appear to be involved in certain CNS conditions, including depression and nausea and vomiting. Aprepitant is an oral antagonist at NK1 receptors and is approved for use in chemotherapy-induced nausea and vomiting; fosaprepitant is a prodrug for aprepitant that is used parenterally. CGRP (calcitonin gene-related peptide) is found (along with calcitonin) in high concentrations in the thyroid but is also present in most smooth muscle tissues. It is a very potent vasodilator. The presence of CGRP in smooth muscle suggests a function as a cotransmitter in autonomic nerve endings. CGRP is the most potent hypotensive agent discovered to date and causes reflex CHAPTER 17 Vasoactive Peptides tachycardia. Some evidence suggests that CGRP is involved in migraine headache. Currently, there is no clinical application for this peptide. However, an oral CGRP antagonist, if available, would be of great interest for the treatment of migraine. NPY (neuropeptide Y) is a potent vasoconstrictor peptide that also stimulates the heart. NPY is found in the CNS and peripheral nerves; it is commonly localized as a cotransmitter in adrenergic nerve endings. In experimental animals, NPY administered in the CNS stimulates feeding and causes hypotension and hypothermia. Peripheral administration causes positive chronotropic and inotropic effects in the heart and hypertension. Several receptor subtypes have been identified, but neither agonists nor antagonists of this peptide have found clinical application. SKILL KEEPER: ANGIOTENSIN ANTAGONISTS (SEE CHAPTER 11) Discuss the differences between ACE inhibitors and AT1receptor blockers in the context of the peptides described in this chapter. The Skill Keeper Answer appears at the end of the chapter. QUESTIONS 1. Field workers exposed to a plant toxin develop painful fluidfilled blisters. Analysis of the blister fluid reveals high concentrations of a peptide. Which of the following is a peptide that causes increased capillary permeability and edema? (A) Angiotensin II (B) Bradykinin (C) Captopril (D) Histamine (E) Losartan 2. In a laboratory study of several peptides, one is found that decreases peripheral resistance but constricts veins. Which of the following causes arteriolar vasodilation and venoconstriction? (A) Angiotensin II (B) Bradykinin (C) Endothelin-1 (D) Substance P (E) Vasoactive intestinal peptide 3. Which of the following endogenous molecules is elevated in heart failure and when given as a drug is a vasodilator with significant renal toxicity? (A) Angiotensin I (B) Angiotensin II (C) Histamine (D) Nesiritide (E) Vasoactive intestinal peptide 155 4. A 45-year-old painter presents with respiratory symptoms and careful workup reveals idiopathic pulmonary hypertension. Which of the following binds endothelin receptors and is approved for use in pulmonary hypertension? (A) Aliskiren I (B) Bosentan (C) Capsaicin (D) Losartan (E) Nesiritide 5. A 60-year-old financial consultant presents with severe pain in a neuronal dermatome region of her chest. This area was previously affected by a herpes zoster rash. Which of the following might be of benefit in controlling this post-herpetic pain? (A) Aliskiren (B) Aprepitant (C) Bosentan (D) Capsaicin (E) Captopril (F) Losartan (G) Nesiritide 6. In a phase 2 clinical trial in hypertensive patients, an endogenous octapeptide vasoconstrictor was found to increase in the blood of patients treated with large doses of diuretics. Which of the following is the most likely endogenous peptide? (A) Angiotensin I (B) Angiotensin II (C) Atrial natriuretic peptide (D) Bradykinin (E) Calcitonin gene-related peptide (F) Endothelin (G) Neuropeptide Y (H) Renin (I) Substance P (J) Vasoactive intestinal peptide 7. Which of the following is a vasodilator that increases in the blood or tissues of patients treated with captopril? (A) Angiotensin II (B) Bradykinin (C) Brain natriuretic peptide (D) Calcitonin gene-related peptide (E) Endothelin (F) Neuropeptide Y (G) Renin 8. Which of the following is an antagonist at NK1 receptors and is used to prevent or reduce chemotherapy-induced nausea and vomiting? (A) Angiotensin I (B) Aprepitant (C) Bosentan (D) Bradykinin (E) Brain natriuretic peptide (F) Enalapril (G) Ondansetron 156 PART IV Drugs with Important Actions on Smooth Muscle ANSWERS 1. Histamine and bradykinin both cause a marked increase in capillary permeability that is often associated with edema, but histamine is not a peptide. The answer is B. 2. Substance P is a potent arterial vasodilator and venoconstrictor. The answer is D. 3. BNP is an atrial and brain peptide found in increased amounts in patients with heart failure. The commercial formulation (nesiritide) is approved for use in severe acute heart failure but has significant renal toxicity. The answer is D. 4. Aliskiren, captopril, and losartan are used in primary hypertension. Bosentan, an endothelin antagonist, is used in pulmonary hypertension. The answer is B. 5. Substance P is an important pain-mediating neurotransmitter peptide and appears to be involved in post-herpetic pain as well as arthritic pain. Capsaicin can be used topically to deplete substance P stores from sensory nerves. The answer is D. 6. Angiotensin II, an octapeptide, increases when blood volume decreases (a diuretic effect) because the compensatory response causes an increase in renin secretion. Its precursor, angiotensin I, would also increase, but it is a decapeptide. The answer is B. 7. Bradykinin increases because the enzyme inhibited by captopril, converting enzyme, degrades kinins in addition to synthesizing angiotensin II (see Figure 11–3). The answer is B. 8. Aprepitant and ondansetron are both used to reduce or prevent chemotherapy-induced nausea and vomiting. Ondansetron is an antagonist at 5-HT3 receptors. The answer is B. SKILL KEEPER ANSWER: ANGIOTENSIN ANTAGONISTS (SEE CHAPTER 11) Both ACE inhibitors (eg, captopril) and AT1-receptor blockers (eg, losartan) reduce the effects of the renin-angiotensinaldosterone system and thereby reduce blood pressure. Both result in a compensatory increase in the release of renin and angiotensin I. A major difference between the 2 types of drugs results from the fact that ACE inhibitors increase the circulating levels of bradykinin because bradykinin is normally inactivated by ACE. The increase in bradykinin contributes to the hypotensive action of ACE inhibitors but is probably also responsible for the high incidence of cough associated with ACE inhibitor use. The cough is believed to result from prostaglandins synthesized as a result of the increased bradykinin. AT1-receptor blockers have a lower incidence of cough. However, both groups of drugs interfere with renal development in the fetus and are contraindicated in pregnancy. CHECKLIST When you complete this chapter, you should be able to: ❑ Name an antagonist of angiotensin II at its receptor and at least 2 drugs that reduce the formation of ANG II. ❑ Outline the major effects of bradykinin and brain natriuretic peptide. ❑ Describe the functions of converting enzyme (peptidyl dipeptidase, kininase II). ❑ List 2 potent vasoconstrictor peptides. ❑ Describe the effects of vasoactive intestinal peptide and substance P. ❑ Describe the clinical applications of bosentan and aprepitant. CHAPTER 17 Vasoactive Peptides 157 DRUG SUMMARY TABLE: Vasoactive Peptides Subclass Mechanism of Action Clinical Applications Pharmacokinetics Toxicities, Interactions Renin-angiotensin antagonists Aliskiren Renin inhibitor • reduces angiotensin I and II and aldosterone secretion Hypertension Oral Duration: 12 h Angioedema, renal impairment Captopril, enalapril, others ACE inhibitor • reduces angiotensin II and aldosterone secretion • increases bradykinin Hypertension, heart failure Oral Half-life: ~2 h but large doses used for duration of effect ~12 h Cough, teratogenic, hyperkalemia Losartan, valsartan, other ARBs AT1 receptor inhibitor; reduces effects of angiotensin II Hypertension Oral Duration: 6–8 h Teratogenic, hyperkalemia Ecallantide Kallikrein inhibitor • reduces bradykinin levels Hereditary angioedema Subcutaneous Duration 2 h Hypersensitivity reactions Icatibant B2 bradykinin receptor blocker Hereditary angioedema Subcutaneous Duration 1 h Hepatic toxicity, hypersensitivity reactions BNP receptor agonist Acute heart failure Parenteral Half-life: 18 min Renal damage, hypotension ETA and ETB receptor antagonists Pulmonary hypertension Oral Half-life: 5 h Hepatic impairment; possible teratogen Kinin antagonists Natriuretic peptides Nesiritide Endothelin antagonists Bosentan, macitentan Ambrisentan: ET antagonist like bosentan, more selective for ETA receptor Neurokinin antagonists Aprepitant Tachykinin NK1 receptor antagonist Antiemetic for chemotherapy-induced vomiting Oral Half-life: 9–13 h Asthenia, hiccups Capsaicin Releases substance P from nerve endings Topical for painful conditions (joints, post-herpetic neuralgia) Topical Duration: 4–6 h Burning, stinging, erythema ACE, angiotensin-converting enzyme; ARB, angiotensin receptor blocker; BNP, brain natriuretic peptide. C Prostaglandins & Other Eicosanoids The eicosanoids are an important group of endogenous fatty acid autacoids that are synthesized from arachidonic acid, a 20-carbon fatty acid lipid in cell membranes. Major families H A P T E R 18 of eicosanoids of clinical importance include the straight-chain derivatives (leukotrienes) and cyclic derivatives (prostacyclin, prostaglandins, and thromboxane). Eicosanoids Leukotrienes (LTB4, LTC4, LTD4) Prostaglandins (PGE1, PGE2, PGF2), Prostacyclin (PGI2), Thromboxane (TXA2) Eicosanoid antagonists Leukotriene antagonists (zileuton, montelukast, zafirlukast) EICOSANOID AGONISTS A. Classification The principal eicosanoid subgroups are the leukotrienes and a group of cyclic molecules, including prostaglandins, prostacyclin, and thromboxane. The leukotrienes retain the straightchain configuration of the parent arachidonic acid. Prostacyclin, thromboxane, and other members of the prostaglandin group are cyclized derivatives of arachidonic acid. There are several series for most of the principal subgroups, based on different substituents (indicated by letters A, B, etc) and different numbers of double bonds (indicated by a subscript number) in the molecule. B. Synthesis Active eicosanoids are synthesized in response to a wide variety of stimuli (eg, physical injury, immune reactions). These stimuli activate phospholipases in the cell membrane or cytoplasm, and 158 Prostaglandin antagonists (corticosteroids, NSAIDs) arachidonic acid (a tetraenoic [4 double bonds] fatty acid) is released from membrane phospholipids (Figure 18–1). Arachidonic acid is then metabolized by several different enzymes. The 2 most important are lipoxygenase (LOX), which results in straight-chain leukotrienes, and cyclooxygenase (COX), which results in cyclization to prostacyclin, prostaglandins, or thromboxane. COX exists in at least 2 forms. COX-1 is found in many tissues; the prostaglandins produced by COX-1 appear to be important for a variety of normal physiologic processes (see later discussion). In contrast, COX-2 is found primarily in inflammatory cells; the products of its actions play a major role in tissue injury (eg, inflammation). In addition to these inflammatory functions, COX-2 is also responsible for synthesis of prostacyclin and of prostaglandins important in normal renal function. Thromboxane is preferentially synthesized in platelets, whereas prostacyclin is synthesized in the endothelial cells of vessels. Naturally occurring eicosanoids have very short half-lives (seconds to minutes) and are inactive when given orally. CHAPTER 18 Prostaglandins & Other Eicosanoids 159 High-Yield Terms to Learn Abortifacient A drug used to cause an abortion. Example: prostaglandin F2α Cyclooxygenase Enzyme that converts arachidonic acid to PGG and PGH, the precursors of the prostaglandins, including PGE, PGF, prostacyclin, and thromboxane Dysmenorrhea Painful uterine cramping caused by prostaglandins released during menstruation Great vessel transposition Congenital anomaly in which the pulmonary artery exits from the left ventricle and the aorta from the right ventricle. Incompatible with life after birth unless a large patent ductus or ventricular septal defect is present Lipoxygenase Enzyme that converts arachidonic acid to leukotriene precursors NSAID Nonsteroidal anti-inflammatory drug, for example, aspirin, ibuprofen, celecoxib. NSAIDs are cyclooxygenase inhibitors Oxytocic A substance that causes uterine contraction Patent ductus arteriosus Abnormal persistence after birth of the shunt between the pulmonary artery and the aorta; normal in the fetus Phospholipase A2 Enzyme in the cell membrane that generates arachidonic acid from membrane lipids Slow-reacting substance of anaphylaxis (SRS-A) Material originally identified by bioassay from tissues of animals in anaphylactic shock; now recognized as a mixture of leukotrienes, especially LTC4 and LTD4 Replacement of tetraenoic fatty acids in the diet with trienoic (3 double bonds) or pentaenoic (5 double bonds) precursors results in the synthesis of much less active prostaglandin and leukotriene products. Thus, dietary therapy with fatty oils from plant or cold-water fish sources can be useful in conditions involving pathogenic levels of eicosanoids. C. Mechanism of Action Most eicosanoid effects are brought about by activation of cell surface receptors (Table 18–1) that are coupled by the Gs protein to adenylyl cyclase (producing cyclic adenosine monophosphate [cAMP]) or by the Gq protein coupled to the phosphatidylinositol cascade (producing inositol 1,4,5-trisphosphate [IP3] and diacylglycerol [DAG] second messengers). D. Effects A vast array of effects are produced in smooth muscle, platelets, the central nervous system, and other tissues. Some of the most important effects are summarized in Table 18–1. Eicosanoids most directly involved in pathologic processes include prostaglandin Membrane lipid Phospholipase A2 − Corticosteroids − Protein synthesis Arachidonic acid Lipoxygenase Zileuton Zafirlukast − Cyclooxygenase (COX-1, COX-2) − NSAIDs Hydroperoxides (HPETEs) Endoperoxides (PGG, PGH) Leukotrienes (LTB, LTC, LTD) − Prostacyclin Thromboxane (PGI) (TXA) Prostaglandins (PGE, PGF) Receptors FIGURE 18–1 Synthesis of eicosanoid autacoids. Arachidonic acid is released from membrane lipids by phospholipase A2 and then converted into straight-chain derivatives by lipoxygenase or into cyclized derivatives by cyclooxygenase. Because many of the effects of these products are pathogenic, drugs that inhibit synthesis or prevent the actions of the products are clinically useful. 160 PART IV Drugs with Important Actions on Smooth Muscle TABLE 18–1 Effects of some important eicosanoids. Effect PGE2 PGF2α PGI2 TXA2 LTB4 LTC4 LTD4 Major receptors EP1-4 FPA,B IP TPα, β BLT1,2 CysLT2 CysLT1 Coupling protein Gs, Gq Gq Gs Gq Gq Gq Gq, Gi Vascular tone ↓ ↑ or ↓ ↓↓ ↑↑↑ ? ↑ or ↓ ↑ or ↓ Bronchial tone ↓↓ ↑↑ ↓ ↑↑↑ ? ↑↑↑↑ ↑↑↑↑ ↑↑↑ ↓ ↑↑ ? ? ? ↓↓↓ ↑↑↑ ? ? ? ? ? ↑↑↑↑ ↑↑ ↑↑ a Uterine tone ↑, ↓ Platelet aggregation ↑ or ↓ Leukocyte chemotaxis ? ? a Low concentrations cause contraction; higher concentrations cause relaxation. ?, unknown effect. F2α, thromboxane A2 (TXA2), and the leukotrienes LTC4 and LTD4. LTC4 and LTD4 are components of the important mediator of bronchoconstriction and shock, slow-reacting substance of anaphylaxis (SRS-A). Leukotriene LTB4 is a chemotactic factor important in inflammation. PGE2 and prostacyclin may act as endogenous vasodilators. PGE1 and its derivatives have significant protective effects on the gastric mucosa. The mechanism may involve increased secretion of bicarbonate and mucus, decreased acid secretion, or both. PGE1 and PGE2 relax vascular and other smooth muscle. PGE2 appears to be the natural vasodilator that maintains patency of the ductus arteriosus during fetal development. In the kidney, prostaglandins are important modulators of glomerular filtration and act on the afferent and efferent arterioles and mesangial cells. Suppression of prostaglandin production with nonsteroidal anti-inflammatory drugs (NSAIDs, see following text) can markedly reduce the efficacy of diuretic agents (see Chapter 15). PGE2 and PGF2α are released in large amounts from the endometrium during menstruation and can cause dysmenorrhea. PGE2 appears to be involved in the physiologic softening of the cervix at term; PGE2 and PGF2α may play a physiologic role in labor. Platelet aggregation is strongly activated by thromboxane. Topical PGF2α reduces intraocular pressure (see later discussion), but it is not known whether this is a physiologic effect of endogenous PGF2α. E. Clinical Uses 1. Obstetrics—PGE2 and PGF2α cause contraction of the uterus. PGE2 (as dinoprostone) is approved for use to soften the cervix at term before induction of labor with oxytocin. Both PGE2 and PGF2α have been used as abortifacients in the second trimester of pregnancy. Although effective in inducing labor at term, they produce more adverse effects (nausea, vomiting, diarrhea) than do other oxytocics (eg, oxytocin) used for this application. The PGE1 analog misoprostol has been used with the progesterone antagonist mifepristone (RU 486) as an extremely effective and safe abortifacient combination. Misoprostol has been used for this purpose in combination with either methotrexate or mifepristone in the United States. Misoprostol may cause diarrhea. 2. Pediatrics—PGE1 is given as an infusion to maintain patency of the ductus arteriosus in infants with transposition of the great vessels until surgical correction can be undertaken. 3. Pulmonary hypertension and dialysis—Prostacyclin (PGI2) is approved for use (as epoprostenol) in severe pulmonary hypertension and to prevent platelet aggregation in dialysis machines. 4. Peptic ulcer associated with NSAID use—Misoprostol is approved in the United States for the prevention of peptic ulcers in patients who must take high doses of NSAIDs for arthritis and who have a history of ulcer associated with this use. 5. Urology—PGE1 (as alprostadil) is used in the treatment of impotence by injection into the cavernosa or as a urethral suppository. 6. Ophthalmology—Latanoprost, a PGF2α derivative, is used extensively for the topical treatment of glaucoma. Bimatoprost, travoprost, and unoprostone are related drugs. These agents reduce intraocular pressure, apparently by increasing the outflow of aqueous humor. EICOSANOID ANTAGONISTS Phospholipase A2 and cyclooxygenase can be inhibited by drugs and some of these inhibitors are mainstays in the treatment of inflammation (Figure 18–1 and Chapter 36). Zileuton is a selective inhibitor of lipoxygenase; some cyclooxygenase inhibitors also exert a mild inhibitory effect on leukotriene synthesis via this enzyme. Inhibitors of the receptors for the prostaglandins and the leukotrienes are being actively sought. Montelukast and zafirlukast, inhibitors at CysLT1 (the LTD4 receptor), are currently available for the treatment of asthma (Chapter 20). CHAPTER 18 Prostaglandins & Other Eicosanoids A. Corticosteroids As indicated in Figure 18–1, corticosteroids inhibit the production of arachidonic acid by phospholipases in the membrane. This effect is mediated by intracellular steroid receptors that, when activated by an appropriate steroid, increase expression of specific proteins capable of inhibiting phospholipase. Steroids also inhibit the synthesis of COX-2. These effects are thought to be the major mechanisms of the important anti-inflammatory action of corticosteroids (see Chapter 39). B. NSAIDs Aspirin and other nonsteroidal anti-inflammatory drugs inhibit cyclooxygenase and the production of thromboxane, prostaglandin, and prostacyclin (see Figure 18–1). Most of the currently available NSAIDs, eg, ibuprofen and naproxen, nonselectively inhibit both COX-1 and COX-2. In fact, many inhibit COX-1 somewhat more effectively than COX-2, the isoform thought to be responsible for synthesis of inflammatory eicosanoids. Celecoxib is the most selective COX-2 inhibitor available in the United States; meloxicam is also slightly COX-2-selective. The highly COX-2-selective rofecoxib and valdecoxib were withdrawn from the US market because of reports of cardiovascular toxicity (see Chapter 36). Inhibition of cyclooxygenase by aspirin is irreversible, unlike the reversible inhibition produced by other NSAIDs. Aspirin allergy may result from diversion of arachidonic acid to the leukotriene pathway when the cyclooxygenasecatalyzed prostaglandin pathway is blocked. The resulting increase in leukotriene synthesis causes the bronchoconstriction that is typical of aspirin allergy. For unknown reasons, this form of aspirin allergy is more common in persons with nasal polyps. The antiplatelet action of aspirin results from the fact that the drug’s inhibition of thromboxane synthesis is essentially permanent in platelets; non-nucleated cells lack the machinery for new protein synthesis. In contrast, inhibition of prostacyclin synthesis in the vascular endothelium is temporary because these nucleated cells can synthesize new enzyme. Inhibition of prostaglandin synthesis also results in important antiinflammatory effects. Inhibition of synthesis of fever-inducing prostaglandins in the brain produces the antipyretic action of NSAIDs. Closure of a patent ductus arteriosus in an otherwise normal infant can be accelerated with an NSAID such as indomethacin or ibuprofen. C. Leukotriene Antagonists As noted, an inhibitor of lipoxygenase (zileuton) and LTD4 and LTE4 receptor antagonists (zafirlukast, montelukast) are available for clinical use. Currently, these agents are approved only for use in asthma (see Chapter 20). 161 QUESTIONS 1. A 50-year-old woman with moderately severe arthritis has been treated with nonsteroidal anti-inflammatory drugs for 6 mo. She now complains of heartburn and indigestion. You give her a prescription for a drug to be taken along with the anti-inflammatory agent, but 2 d later she calls the office complaining that your last prescription has caused severe diarrhea. Which of the following is most likely to be associated with increased gastrointestinal motility and diarrhea? (A) Aspirin (B) Famotidine (C) Leukotriene LTB4 (D) Misoprostol (E) Zileuton 2. Which of the following drugs inhibits thromboxane synthesis much more effectively than prostacyclin synthesis? (A) Aspirin (B) Hydrocortisone (C) Ibuprofen (D) Indomethacin (E) Zileuton 3. A 57-year-old man has severe pulmonary hypertension and right ventricular hypertrophy. Which of the following agents causes vasodilation and may be useful in pulmonary hypertension? (A) Angiotensin II (B) Ergotamine (C) Prostaglandin PGF2α (D) Prostacyclin (E) Thromboxane 4. A 19-year-old woman complains of severe dysmenorrhea. A uterine stimulant derived from membrane lipid in the endometrium is (A) Angiotensin II (B) Oxytocin (C) Prostacyclin (PGI2) (D) Prostaglandin PGF2α (E) Serotonin 5. Inflammation is a complex tissue reaction that includes the release of cytokines, leukotrienes, prostaglandins, and peptides. Prostaglandins involved in inflammatory processes are typically produced from arachidonic acid by which of the following enzymes? (A) Cyclooxygenase-1 (B) Cyclooxygenase-2 (C) Glutathione-S-transferase (D) Lipoxygenase (E) Phospholipase A2 6. A newborn infant is diagnosed with transposition of the great vessels, wherein the aorta exits from the right ventricle and the pulmonary artery from the left ventricle. Which of the following drugs is likely to be used in preparation for surgical correction of this anomaly? (A) Aspirin (B) Leukotriene LTC4 (C) Prednisone (D) Prostaglandin PGE1 (E) Prostaglandin PGF2α 162 PART IV Drugs with Important Actions on Smooth Muscle 7. A patient with a bleeding tendency presents in the hematology clinic. He is apparently taking large amounts of an unidentified drug that inhibits platelet activity. Which of the following is taken orally and directly and reversibly inhibits platelet cyclooxygenase? (A) Alprostadil (B) Aspirin (C) Ibuprofen (D) Leukotriene LTC4 (E) Misoprostol (F) Prednisone (G) Prostacyclin (H) Zafirlukast (I) Zileuton 8. Which of the following is a component of slow-reacting substance of anaphylaxis (SRS-A)? (A) Alprostadil (B) Aspirin (C) Leukotriene LTB4 (D) Leukotriene LTC4 (E) Misoprostol (F) Prednisone (G) Prostacyclin (H) Zafirlukast (I) Zileuton 9. A 17-year-old patient complains that he develops wheezing and severe shortness of breath whenever he takes aspirin for headache. Increased levels of which of the following may be responsible, in part, for some cases of aspirin hypersensitivity? (A) Alprostadil (B) Hydrocortisone (C) Ibuprofen (D) Leukotriene LTC4 (E) Misoprostol (F) PGE2 (G) Prostacyclin (H) Thromboxane (I) Zileuton 10. Which of the following is a leukotriene receptor blocker? (A) Alprostadil (B) Aspirin (C) Ibuprofen (D) Leukotriene LTC4 (E) Montelukast (F) Prednisone (G) Prostacyclin (H) Zileuton ANSWERS 1. Aspirin and zileuton rarely cause diarrhea. LTB4 is a chemotactic factor. Famotidine is an H2 blocker that does not cause diarrhea (Chapter 16). The answer is D. 2. Hydrocortisone and other corticosteroids inhibit phospholipase. Ibuprofen and indomethacin inhibit cyclooxygenase reversibly, whereas zileuton inhibits lipoxygenase. Because aspirin inhibits cyclooxygenase irreversibly, its action is more effective in platelets, which lack the ability to synthesize new enzyme, than in the endothelium. The answer is A. 3. Prostacyclin (PGI2) is a very potent vasodilator. All the other choices in the list are vasoconstrictors. The answer is D. 4. Although serotonin and, in some species, histamine may cause uterine stimulation, these substances are not derived from membrane lipid. Similarly, oxytocin causes uterine contraction, but it is a peptide hormone released from the posterior pituitary. Prostacyclin relaxes the uterus (Table 18–1). The answer is D. 5. See Figure 18–1. Phospholipase A2 converts membrane phospholipid to arachidonic acid. Cyclooxygenases convert arachidonic acid to prostaglandins. COX-1 products appear to be important in normal physiologic processes. COX-2 is the enzyme responsible for this reaction in inflammatory cells. The answer is B. 6. Infants with great vessel transposition pump venous blood to the aorta and oxygenated blood back to the lungs. Therefore, they require surgical correction as soon as they are strong enough to withstand the procedure. In the meantime, they are dependent on a patent ductus arteriosus to allow some oxygenated blood to flow from the left ventricle via the pulmonary artery and ductus to the aorta. The ductus can be prevented from closing by infusing the vasodilator PGE1. The answer is D. 7. Aspirin is a direct but irreversible inhibitor of cyclooxygenase. NSAIDs other than aspirin (such as ibuprofen) are reversible inhibitors of COX. Corticosteroids reduce the synthesis of cyclooxygenase. The answer is C. 8. The leukotriene C and D series are major components of SRS-A. Leukotriene LTB4 is a chemotactic eicosanoid. The answer is D. 9. When cyclooxygenase is blocked, leukotrienes may be produced in increased amounts by diversion of prostaglandin precursors into the lipoxygenase pathway (Figure 18–1). In patients with aspirin hypersensitivity, this might precipitate the bronchoconstriction often observed in this condition. The answer is D. 10. Zileuton blocks the synthesis of leukotrienes. Montelukast and zafirlukast block LTD4 receptors. The answer is E. CHAPTER 18 Prostaglandins & Other Eicosanoids 163 CHECKLIST When you complete this chapter, you should be able to: ❑ List the major effects of PGE1, PGE2, PGF2α, PGI2, LTB4, LTC4, and LTD4. ❑ List the cellular sites of synthesis and the effects of thromboxane and prostacyclin in the cardiovascular system. ❑ List the types of currently available antagonists of leukotrienes and prostaglandins and their targets (receptors or enzymes). ❑ Explain the different effects of aspirin on prostaglandin, thromboxane, and leukotriene synthesis. DRUG SUMMARY TABLE: Prostaglandins & Other Eicosanoids Subclass Toxicities, Interactions Mechanism of Action Clinical Applications Pharmacokinetics LTB4 Chemotactic factor in inflammation None Local release Duration: seconds Inflammatory mediator LTC4, LTD4 Bronchoconstrictors important in anaphylaxis, asthma • cause edema None Local release Duration: seconds Inflammatory mediators Lipoxygenase inhibitor: zileuton Blocks synthesis of leukotrienes Asthma prophylaxis Oral Duration: ~3 h Liver enzyme elevation Leukotriene receptor inhibitors: montelukast, zafirlukast Block CysLT1 receptor • reduce bronchoconstriction in asthma Asthma prophylaxis Oral Duration: ~3–10 h Liver enzyme elevation Activates TPα,β receptors, causes platelet aggregation, vasoconstriction None Local release Duration: seconds See Mechanism of Action Activates IP receptors, causes vasodilation, reduces platelet aggregation Vasodilator in pulmonary hypertension, antiplatelet agent in extracorporeal dialysis Infusion Duration: minutes Hypotension, flushing, headache Leukotrienes Leukotriene antagonists Thromboxane TXA2 Prostacyclin PGI2: epoprostenol PGI2 analog, treprostinil: parenteral or by inhalation for pulmonary hypertension (Continued ) 164 PART IV Drugs with Important Actions on Smooth Muscle DRUG SUMMARY TABLE: Prostaglandins & Other Eicosanoids (Continued ) Subclass Toxicities, Interactions Mechanism of Action Clinical Applications Pharmacokinetics Activates EP receptors, causes increased HCO3– and mucus secretion in stomach • uterine contraction Protective agent in peptic ulcer disease • abortifacient Oral Duration: minutes to hours Diarrhea, uterine cramping Prostaglandins PGE1 derivative: misoprostol PGE1 analog, alprostadil: injectable and suppository form for erectile dysfunction PGE1 Relaxes smooth muscle in ductus arteriosus Transposition of great vessels, to maintain patent ductus until surgery Infusion Duration: minutes Hypotension PGE2: dinoprostone Low concentrations contract, higher concentrations relax uterine and cervical smooth muscle Abortifacient, cervical ripening Vaginal Duration: 3–5 h Cramping, fetal trauma PGF2α derivative: latanoprost Increases outflow of aqueous humor, reduces intraocular pressure Glaucoma Topical Duration: 4–8 h Color change in iris Cyclooxygenase inhibitors (NSAIDs) Nonselective COX-1, COX-2 inhibitors: ibuprofen, indomethacin, naproxen, others Reversibly inhibit COX-1 and COX-2 • reduce synthesis of prostaglandins See Chapter 36 Aspirin Irreversibly inhibits COX-1 and COX-2 • reduces synthesis of prostaglandins See Chapter 36 Selective COX-2 inhibitor, celecoxib Selectively reversibly inhibits COX-2 See Chapter 36 Phospholipase A2 inhibitors Corticosteroids Reversibly inhibit phospholipase A2 and reduce synthesis of COX, LOX enzymes See Chapter 39 C A P T E R 19 Nitric Oxide, Donors, & Inhibitors Nitric oxide is a potent vasodilator autacoid produced from arginine in the body, and the active metabolite of drugs that release it (NO donors); it is also available as a drug in itself H (NO gas). It interacts with iron in hemoglobin and can be inhibited by hemoglobin. Agents related to nitric oxide (NO) Endogenous NOS activators (ACh, histamine, etc) Inhibitors (hemoglobin) Nitric oxide (NO) is a product of the metabolism of arginine in many tissues. It is thought to be an important paracrine vasodilator, and it may also play a role in cell death and in neurotransmission; it therefore qualifies as an autacoid. NO is also released from several important vasodilator drug molecules. ENDOGENOUS NO Endogenous NO is synthesized by a family of enzymes collectively called nitric oxide synthase (NOS), Figure 19–1. These cytoplasmic enzymes are activated by calcium influx or by cytokines. Arginine, the primary substrate, is converted by NOS to citrulline and NO. Three forms of NO synthase Exogenous NO donors (nitrates, nitroprusside) NO gas are known: isoform 1 (bNOS, cNOS, or nNOS, a constitutive form found in epithelial and neuronal cells); isoform 2 (iNOS or mNOS, an inducible form found in macrophages and smooth muscle cells); and isoform 3 (eNOS, a constitutive form found in endothelial cells). NOS can be inhibited by arginine analogs such as N G-monomethyl-l-arginine (l-NMMA). Under some circumstances (eg, ischemia), NO may be formed from endogenous nitrate ion. NO is not stored in cells because it is a gas at body temperature. NO very rapidly diffuses from its site of synthesis to surrounding tissues. Drugs that cause endogenous NO release do so by stimulating its synthesis by NOS. Such drugs include muscarinic agonists, histamine, and certain other vasodilators (bradykinin, hydralazine). High-Yield Terms to Learn Endothelium-derived relaxing factor, EDRF A mixture of nitric oxide and other vasodilator substances synthesized in vascular endothelium Nitric oxide donor A molecule from which nitric oxide can be released (eg, arginine, nitroprusside, nitroglycerin) cNOS, iNOS, eNOS Naturally occurring isoforms of nitric oxide synthase: respectively, constitutive (NOS-1), inducible (NOS-2), and endothelial (NOS-3) isoforms 165 166 PART IV Drugs with Important Actions on Smooth Muscle Arginine Nitric oxide synthase, NOS Nitrates, nitroprusside Citrulline + NO Nitration, nitrosylation + Guanylyl cyclase B. Cell Adhesion NO has effects on cell adhesion that result in reduced platelet aggregation and reduced neutrophil adhesion to vascular endothelium. The latter effect is probably due to reduced expression of adhesion molecules, for example, integrins, by endothelial cells. Guanylyl cyclase (activated) + GTP cGMP C. Inflammation Tissue injury causes NO synthesis, and NO appears to facilitate inflammation both directly and through the stimulation of prostaglandin synthesis by cyclooxygenase 2. FIGURE 19–1 The pathway for nitric oxide (NO) synthesis and release from NO-containing drugs and the mechanism of stimulation of cGMP (cyclic guanosine monophosphate) synthesis. The action of cGMP on smooth muscle relaxation is shown in Figure 12–3. D. Other Effects Some evidence suggests that NO may act as a neurotransmitter. NO also may be involved in some types of apoptosis and cell death and in host reactions to parasites. Excessive concentrations of NO (eg, from inhaled NO or from nitrites) convert hemoglobin to methemoglobin and may result in hypoxia. EXOGENOUS NO DONORS CLINICAL APPLICATIONS OF NO INHIBITORS & DONORS NO is released from several important drugs, including nitroprusside (Chapter 11), nitrates (Chapter 12), and nitrites. Release from nitroprusside occurs spontaneously in the blood in the presence of oxygen, whereas release from nitrates and nitrites is intracellular and requires the presence of the mitochondrial enzyme ALDH2 and thiol compounds such as cysteine (see Chapter 12). Tolerance may develop to nitrates and nitrites if endogenous thiol compounds are depleted. EFFECTS OF NO SKILL KEEPER: NONINNERVATED RECEPTORS (SEE CHAPTER 6) List some noninnervated receptors found in blood vessels and describe their second-messenger mechanisms of action. The Skill Keeper Answer appears at the end of the chapter. Although inhibitors of NO synthesis are of great research interest, none are currently in clinical use. NO can be inactivated by heme and hemoglobin, but application of this approach is investigational. In contrast, drugs that activate endogenous NO synthesis and donors of the molecule were in use long before NO was discovered and continue to be very important in clinical medicine. The cardiovascular applications of nitroprusside (Chapter 11) and the nitrates and nitrites (Chapter 12) have been discussed. The treatment of preeclampsia, pulmonary hypertension, and acute respiratory distress syndrome are currently under clinical investigation. Early results from pulmonary disease studies appear promising, and one preparation of NO gas (INOmax) is approved for use in neonates with hypoxic respiratory failure and adults with pulmonary hypertension. Preclinical studies suggest that chronic use of NO donor drugs or dietary supplementation with arginine may assist in slowing atherosclerosis, especially in grafted organs. In contrast, acute rejection of grafts may involve upregulation of NOS enzymes, and inhibition of these enzymes may prolong graft survival. QUESTIONS A. Smooth Muscle NO is a powerful vasodilator in all vascular beds and a potent relaxant in most other smooth muscle tissues, eg, erectile tissue. The mechanism of this effect involves activation of guanylyl cyclase (Figure 19–1) and the synthesis of cyclic guanosine monophosphate (cGMP). This cGMP, in turn, facilitates the dephosphorylation and inactivation of myosin light chains, which results in relaxation of smooth muscle (see Figure 12–3). NO plays a physiologic role in erectile tissue function, in which smooth muscle relaxation is required to bring about the influx of blood that causes erection. NO appears to be a pathophysiologic contributor to hypotension in septic shock. 1. Which one of the following is not a nitric oxide donor but causes it to be synthesized and released from endogenous precursors, resulting in vasodilation? (A) Acetylcholine (B) Arginine (C) Isosorbide mononitrate (D) Nitroglycerin (E) Nitroprusside 2. A molecule that releases nitric oxide in the blood is (A) Citrulline (B) Histamine (C) Isoproterenol (D) Nitroglycerin (E) Nitroprusside CHAPTER 19 Nitric Oxide, Donors, & Inhibitors 3. The inducible isoform of nitric oxide synthase (iNOS, isoform 2) is found primarily in which of the following? (A) Cartilage (B) Eosinophils (C) Macrophages (D) Platelets (E) Vascular endothelial cells 4. The primary endogenous substrate for the enzyme nitric oxide synthase (NOS) is (A) Acetylcholine (B) Angiotensinogen (C) Arginine (D) Citrulline (E) Heme 5. Which of the following is a recognized effect of nitric oxide (NO)? (A) Arrhythmia (B) Bronchoconstriction (C) Constipation (D) Inhibition of acute graft rejection (E) Pulmonary vasodilation 6. Which of the following is an endogenous inhibitor/inactivator of nitric oxide? (A) Arginine (B) Angiotensinogen (C) Arachidonic acid (D) Hemoglobin (E) Thromboxane ANSWERS 1. Nitroprusside and organic nitrites (eg, amyl nitrite) and nitrates (eg, nitroglycerin, isosorbide dinitrate, and isosorbide mononitrate) contain NO groups that can be released as NO. Arginine is the normal source of endogenous NO. Acetylcholine stimulates the synthesis of NO from arginine. The answer is A. 167 2. Nitroprusside is the only molecule in this list that spontaneously releases NO in the bloodstream. The answer is E. 3. The inducible form of NOS is associated with inflammation, and the enzyme is found in highest concentration in macrophages, cells that are particularly involved in inflammation. The answer is C. 4. Arginine is the substrate and citrulline and NO are the products of NOS. The answer is C. 5. NO does not cause arrhythmias or constipation. It causes bronchodilation and may hasten graft rejection. NO does cause pulmonary vasodilation. The answer is E. 6. Heme and hemoglobin inactivate NO. The answer is D. SKILL KEEPER ANSWER: NONINNERVATED RECEPTORS (SEE CHAPTER 6) Endothelial cells lining blood vessels have noninnervated muscarinic receptors. These M3 receptors use the Gq-coupling protein to activate phospholipase C, which releases inositol 1,4,5-trisphosphate and diacylglycerol from membrane lipids. eNOS is activated and NO is released, causing vasodilation. Histamine H1 receptors are also found in the vascular endothelium and similarly cause vasodilation through the synthesis and release of NO. Other noninnervated (or poorly innervated) receptors found in blood vessels include α2 and β2 receptors. The α2 receptors use Gi to inhibit adenylyl cyclase, reducing cyclic adenosine monophosphate (cAMP) and causing contraction in the vessel. (Recall that the blood pressurelowering action of α2 agonists is mediated by actions in the CNS, not in the vessels.) Conversely, β2 receptors activate adenylyl cyclase via Gs and increase cAMP, resulting in relaxation. In addition to these, receptors for many vasoactive peptides are found in vessels (see Chapter 17). CHECKLIST When you complete this chapter, you should be able to: ❑ Name the enzyme responsible for the synthesis of NO in tissues. ❑ List the major beneficial and toxic effects of endogenous NO. ❑ List 2 drugs that cause release of endogenous NO. ❑ List 2 drugs that spontaneously or enzymatically break down in the body to release NO. 168 PART IV Drugs with Important Actions on Smooth Muscle DRUG SUMMARY TABLE: Nitric Oxide, Donors, & Inhibitors Subclass Mechanism of Action Clinical Applications Pharmacokinetics Toxicities, Interactions Activates guanylyl cyclase, increases cGMP synthesis, causes smooth muscle relaxation Pulmonary hypertension Inhaled gas administered continuously Excessive hypotension, methemoglobinemia, conversion to nitrogen dioxide (a pulmonary irritant) Nitric oxide (NO) Nitric oxide gas Nitric oxide synthase (NOS) activators Acetylcholine, histamine, others Increased IP3 → ↑ intracellular Ca2+ → activates NOS, resulting in conversion of arginine to citrulline plus NO See Chapters 7 and 16 Release NO in smooth muscle (nitrates) or in blood (nitroprusside) • increase cGMP synthesis and cause relaxation in smooth muscle See Chapters 11 and 12 Nitric oxide donors Nitroglycerin, other nitrates, nitroprusside cGMP, cyclic guanosine monophosphate. C A P T E R 20 Drugs Used in Asthma & Chronic Obstructive Pulmonary Disease Asthma is a disease characterized by airway inflammation and episodic, reversible bronchospasm with severe shortness of breath. Drugs useful in asthma include bronchodilators (smooth muscle relaxants) and anti-inflammatory drugs. Bronchodilators include sympathomimetics, especially β2-selective agonists, muscarinic antagonists, methylxanthines, and leukotriene receptor H blockers. Anti-inflammatory drugs used in asthma include corticosteroids, mast cell stabilizers, and an anti-IgE antibody. Leukotriene antagonists play a dual role. Chronic obstructive pulmonary disease (COPD) is characterized by airflow limitation that is less reversible than in asthma and by a progressive course. However, many of the same drugs are used. Drugs used in asthma Bronchodilators Beta agonists Anti-inflammatory agents Lipoxygenase inhibitors Release inhibitors Steroids Muscarinic antagonists Leukotriene antagonists Receptor inhibitors Antibodies Slow anti-inflammatory drugs Methylxanthines Drugs used in chronic obstructive pulmonary disease Bronchodilators Anti-inflammatory agents Antibiotics Steroids PATHOPHYSIOLOGY OF ASTHMA AND COPD The immediate cause of asthmatic bronchoconstriction is the release of several mediators from IgE-sensitized mast cells and other cells involved in immunologic responses (Figure 20–1). These mediators include the leukotrienes LTC4 and LTD4. In addition, chemotactic mediators such as LTB4 attract inflammatory cells to the airways. Finally, several cytokines and some enzymes are released, leading to chronic inflammation. Chronic inflammation leads to marked bronchial hyperreactivity to various inhaled substances, including antigens, histamine, muscarinic agonists, and irritants such as sulfur 169 170 PART IV Drugs with Important Actions on Smooth Muscle High-Yield Terms to Learn Bronchial hyperreactivity Pathologic increase in the bronchoconstrictor response to antigens and irritants; caused by bronchial inflammation IgE-mediated disease Disease caused by excessive or misdirected immune response mediated by IgE antibodies. Example: asthma Mast cell degranulation Exocytosis of granules from mast cells with release of mediators of inflammation and bronchoconstriction Phosphodiesterase (PDE) Family of enzymes that degrade cyclic nucleotides to nucleotides, for example, cAMP (active) to AMP (inactive); various isoforms, some degrade cGMP to GMP Tachyphylaxis Rapid loss of responsiveness to a stimulus (eg, a drug) Antigen Peripheral lymphoid tissue IgE dioxide (SO2) and cold air. This reactivity is partially mediated by vagal reflexes. COPD is characterized by some degree of permanent structural damage to the airways and parenchyma; exacerbation of symptoms (wheezing, shortness of breath, cough) is often triggered by upper respiratory infection (like asthma) but occurs in older patients (usually long-term smokers) and is poorly reversible with bronchodilators. STRATEGIES OF ASTHMA THERAPY Sensitized mast cell IgE-antigen interaction Mediator release LTC4, D4 Acute bronchospasm must be treated promptly and effectively with bronchodilators (“reliever” drugs). Beta2 agonists, muscarinic antagonists, and theophylline and its derivatives are available for this indication. Long-term preventive treatment requires control of the inflammatory process in the airways (“controller” drugs). The most important anti-inflammatory drugs in the treatment of chronic asthma are the corticosteroids. Long-acting β2 agonists can improve the response to corticosteroids. Anti-IgE antibodies also appear promising for chronic therapy. The leukotriene antagonists have effects on both bronchoconstriction and inflammation but are used only for prophylaxis. Nasal oxygen is basic therapy for acute bronchospasm of any cause. PGD2 Histamine, tryptase BETA-ADRENOCEPTOR AGONISTS ECF-A FIGURE 20–1 Immunologic model for the pathogenesis of asthma. Exposure to antigen causes synthesis of IgE, which binds to and sensitizes mast cells and other inflammatory cells. When such sensitized cells are challenged with antigen, a variety of mediators are released that can account for most of the signs of the early bronchoconstrictor response in asthma. LTC4, D4, leukotrienes C4 and D4; ECF-A, eosinophil chemotactic factor-A; PGD2, prostaglandin D2. Modified and reproduced, with permission, from Gold WM: Cholinergic pharmacology in asthma. In: Asthma: Physiology, Immunopharmacology, and Treatment. Austen KF, Lichtenstein LM, editors. Academic Press, 1974. Copyright Elsevier. A. Prototypes and Pharmacokinetics The most important sympathomimetics used to reverse asthmatic bronchoconstriction are the direct-acting a2-selective agonists (see Chapter 9). Indirect-acting sympathomimetics, eg, ephedrine, were once used, but they are now obsolete for this application. Of the selective direct-acting agents, albuterol, terbutaline, and metaproterenol* are short-acting and are the most important in the United States. Salmeterol, formoterol, indacaterol, and vilanterol are long-acting β2 agonists (LABA), but indacaterol and vilanterol are currently approved only for COPD. Beta agonists are given almost exclusively by inhalation, usually from ∗Do not confuse metaproterenol, a β2 agonist, with metoprolol, a β blocker. CHAPTER 20 Drugs Used in Asthma & Chronic Obstructive Pulmonary Disease 171 ATP AC Relaxation + Beta agonists − Theophylline cAMP + Bronchial tone PDE AMP Acetylcholine + Leukotrienes + − Muscarinic antagonists − Leukotriene antagonists Adenosine + − Theophylline Constriction FIGURE 20–2 Possible mechanisms of β agonists, muscarinic antagonists, theophylline, and leukotriene antagonists in altering bronchial tone in asthma. AC, adenylyl cyclase; PDE, phosphodiesterase. pressurized aerosol canisters but occasionally by nebulizer. The inhalational route decreases the systemic dose (and adverse effects) while delivering an effective dose locally to the airway smooth muscle. The older drugs have durations of action of 6 h or less; salmeterol, formoterol, indacaterol, and vilanterol act for 12–24 h. B. Mechanism and Effects Beta-adrenoceptor agonists stimulate adenylyl cyclase (via the β2-adrenoceptor–Gs-coupling protein-adenylyl cyclase pathway) and increase cyclic adenosine monophosphate (cAMP) in smooth muscle cells (Figure 20–2). The increase in cAMP results in a powerful bronchodilator response. C. Clinical Use and Toxicity Sympathomimetics are first-line therapy in acute asthma. Shorter acting sympathomimetics (albuterol, metaproterenol, terbutaline) are the drugs of choice for acute episodes of bronchospasm. Their effects last for 4 h or less, and they are not effective for prophylaxis. The long-acting agents (salmeterol, formoterol) should be used for prophylaxis, in which their 12-h duration of action is useful. They should not be used for acute episodes because their onset of action is too slow. Furthermore, used alone, they increase asthma mortality, whereas in combination with corticosteroids, they improve control. In almost all patients, the shorter-acting β agonists are the most effective bronchodilators available and are life-saving for acute asthma. Many patients with chronic obstructive pulmonary disease (COPD) also benefit, although the risk of toxicity is increased in this condition. Skeletal muscle tremor is a common adverse β2 effect. Beta2 selectivity is relative. At high clinical dosage, these agents have significant β1 effects. Even when they are given by inhalation, some cardiac effect (tachycardia) is common. Other adverse effects are rare. When the agents are used excessively, arrhythmias and tremor SKILL KEEPER: SYMPATHOMIMETICS VS ANTIMUSCARINICS IN ASTHMA (SEE CHAPTERS 8 AND 9) The sympathomimetic bronchodilators are drugs of choice in acute asthma. Some patients benefit from muscarinic antagonists. Compare the properties of sympathomimetics and antimuscarinics relative to the therapeutic goals in asthma. Under what conditions might an antimuscarinic drug be preferable? The Skill Keeper Answers appear at the end of the chapter. may occur. Loss of responsiveness (tolerance, tachyphylaxis) is an unwanted effect of excessive use of the short-acting sympathomimetics. Patients with COPD often have concurrent cardiac disease and may have arrhythmias even at normal dosage. METHYLXANTHINES A. Prototypes and Pharmacokinetics The methylxanthines are purine derivatives. Three major methylxanthines are found in plants and provide the stimulant effects of 3 common beverages: caffeine (in coffee), theophylline (tea), and theobromine (cocoa). Theophylline is the only member of this group that is important in the treatment of asthma. This drug and several analogs are orally active and available as various salts and as the base. Theophylline is available in both promptrelease and slow-release forms. Theophylline is eliminated by P450 drug-metabolizing enzymes in the liver. Clearance varies with age (highest in young adolescents), smoking status (higher in smokers), and concurrent use of other drugs that inhibit or induce hepatic enzymes. 172 PART IV Drugs with Important Actions on Smooth Muscle B. Mechanism of Action and Effects The methylxanthines inhibit phosphodiesterase (PDE), the enzyme that degrades cAMP to AMP (Figure 20–2), and thus increase cAMP. This anti-PDE effect, however, requires high concentrations of the drug. Several isoforms of PDE have been identified; PDE3 appears to be the primary form responsible for methylxanthine-induced bronchodilation, while PDE4 may be responsible for inhibition of inflammatory cells. Methylxanthines also block adenosine receptors in the central nervous system (CNS) and elsewhere, but a relation between this action and the bronchodilating effect has not been clearly established. It is possible that bronchodilation is caused by a third as yet unrecognized action. In asthma, bronchodilation is the most important therapeutic action of theophylline. Increased strength of contraction of the diaphragm has been demonstrated in some patients, an effect particularly useful in COPD. Other effects of therapeutic doses include CNS stimulation, cardiac stimulation, vasodilation, a slight increase in blood pressure (probably caused by the release of norepinephrine from adrenergic nerves), diuresis, and increased gastrointestinal motility. C. Clinical Use and Toxicity The major clinical use of methylxanthines is asthma and COPD. Slow-release theophylline (for control of nocturnal asthma) is the most commonly used methylxanthine. Aminophylline is a salt of theophylline that is sometimes prescribed. Roflumilast, an oral, nonpurine, more selective PDE4 inhibitor, has been approved for use in COPD. Another methylxanthine derivative, pentoxifylline, is promoted as a remedy for intermittent claudication; this effect is said to result from decreased viscosity of the blood. Of course, the nonmedical use of the methylxanthines in coffee, tea, and cocoa is far greater, in total quantities consumed, than the medical uses of the drugs. Two cups of strong coffee are said to contain enough methylxanthine drug to produce measurable bronchodilation. The common adverse effects of methylxanthines include gastrointestinal distress, tremor, and insomnia. Severe nausea and vomiting, hypotension, cardiac arrhythmias, and seizures may result from overdosage. Very large overdoses (eg, in suicide attempts) are potentially lethal because of arrhythmias and seizures. Beta blockers are useful in reversing severe cardiovascular toxicity from theophylline. MUSCARINIC ANTAGONISTS A. Prototypes and Pharmacokinetics Atropine and other naturally occurring belladonna alkaloids were used for many years in the treatment of asthma but have been replaced by ipratropium, a quaternary antimuscarinic agent designed for aerosol use (see Chapter 8). This drug is delivered to the airways by pressurized aerosol and has little systemic action. Tiotropium and aclidinium are longer-acting analogs approved for use in COPD. B. Mechanism of Action and Effects When given by aerosol, these drugs competitively block muscarinic receptors in the airways and effectively prevent bronchoconstriction mediated by vagal discharge. If given systemically (not an approved use), these drugs are indistinguishable from other short-acting muscarinic blockers. Muscarinic antagonists reverse bronchoconstriction in some asthma patients (especially children) and in many patients with COPD. They have no effect on the chronic inflammatory aspects of asthma. C. Clinical Use and Toxicity Inhaled antimuscarinic agents are useful in one third to two thirds of asthmatic patients; β2 agonists are effective in almost all. For acute bronchospasm, therefore, the β agonists are usually preferred. However, in COPD, which is often associated with acute episodes of bronchospasm, the antimuscarinic agents may be more effective and less toxic than β agonists. Because these agents are delivered directly to the airway and are minimally absorbed, systemic effects are small. When given in excessive dosage, minor atropine-like toxic effects may occur (see Chapter 8). In contrast to the β2 agonists, muscarinic antagonists do not cause tremor or arrhythmias. CORTICOSTEROIDS A. Prototypes and Pharmacokinetics All the corticosteroids are potentially beneficial in severe asthma (see Chapter 39). However, because of their toxicity, systemic (oral) corticosteroids (usually prednisone) are used chronically only when other therapies are unsuccessful. In contrast, local aerosol administration of surface-active corticosteroids (eg, beclomethasone, budesonide, dexamethasone, flunisolide, fluticasone, mometasone) is relatively safe, and inhaled corticosteroids have become common first-line therapy for individuals with moderate to severe asthma. Important intravenous corticosteroids for status asthmaticus include prednisolone (the active metabolite of prednisone) and hydrocortisone (cortisol). B. Mechanism of Action and Effects Corticosteroids reduce the synthesis of arachidonic acid by phospholipase A2 and inhibit the expression of COX-2, the inducible form of cyclooxygenase (see Chapter 18). Concentrations of prostaglandins and leukotrienes are reduced. It has also been suggested that the glucocorticoid corticosteroids increase the responsiveness of β adrenoceptors in the airway and they probably act by other mechanisms as well. Glucocorticoids bind to intracellular receptors and activate glucocorticoid response elements (GREs) in the nucleus, resulting in synthesis of substances that prevent the full expression of inflammation and allergy. See Chapter 39 for details. Reduced activity of phospholipase A2 is thought to be particularly important in asthma because the leukotrienes that result from phospholipase-stimulated eicosanoid synthesis are extremely potent CHAPTER 20 Drugs Used in Asthma & Chronic Obstructive Pulmonary Disease Exposure to antigen (eg, dust, pollen) Avoidance − Antigen and IgE on mast cells Cromolyn, steroids, zileuton, antibody − Mediators (eg, leukotrienes, cytokines) Beta agonists, theophylline, muscarinic antagonists, leukotriene antagonists − − Steroids, cromolyn, leukotriene antagonists Early response: bronchoconstriction Late response: inflammation Acute symptoms Bronchial hyperreactivity FIGURE 20–3 Summary of treatment strategies in asthma. (Data from Cockcroft DW: The bronchial late response in the pathogenesis of asthma and its modulation by therapy. Allergy Asthma Immunol 1985;55:857.) bronchoconstrictors and may also participate in the late inflammatory response (Figure 20–3). C. Clinical Use and Toxicity Inhaled glucocorticoids are now considered appropriate (even for children) in most cases of moderate asthma that are not fully responsive to aerosol β agonists. It is believed that such early use may prevent the severe, progressive inflammatory changes characteristic of long-standing asthma. This is a shift from the earlier belief that steroids should be used only in severe refractory asthma. In such cases of severe asthma, patients are usually hospitalized and stabilized on daily systemic prednisone and then switched to inhaled or alternate-day oral therapy before discharge. In status asthmaticus, parenteral steroids are lifesaving and apparently act more promptly than in ordinary asthma. Patients with COPD tend to be more resistant to the beneficial effects of steroids. Their mechanism of action in these conditions is not fully understood. (See Chapter 39 for other uses.) Frequent aerosol administration of glucocorticoids can occasionally result in a small degree of adrenal suppression, but this is rarely significant. More commonly, deposition of inhaled drug droplets in the pharynx causes changes in oropharyngeal flora that result in candidiasis. If oral therapy is required, adrenal 173 suppression can be reduced by using alternate-day therapy (ie, giving the drug in slightly higher dosage every other day rather than smaller doses every day). The major systemic toxicities of the glucocorticoids described in Chapter 39 are much more likely to occur when systemic treatment is required for more than 2 weeks, as in severe refractory asthma. Regular use of inhaled steroids does cause mild growth retardation in children, but these children eventually reach full predicted adult stature. LEUKOTRIENE ANTAGONISTS These drugs interfere with the synthesis or the action of the leukotrienes (see also Chapter 18). Although their value has been established, they are not as effective as corticosteroids in severe asthma. A. Leukotriene Receptor Blockers Montelukast and zafirlukast are antagonists at the LTD4 leukotriene receptor (see Table 18–1). The LTE4 receptor is also blocked. These drugs are orally active and have been shown to be effective in preventing exercise-, antigen-, and aspirininduced bronchospasm. They are not recommended for acute episodes of asthma. Toxicity is generally low. Rare reports of Churg-Strauss syndrome, allergic granulomatous angiitis, have appeared, but an association with these drugs has not been established. B. Lipoxygenase Inhibitor Zileuton is an orally active drug that selectively inhibits 5-lipoxygenase, a key enzyme in the conversion of arachidonic acid to leukotrienes. The drug is effective in preventing both exercise- and antigen-induced bronchospasm. It is also effective against “aspirin allergy,” the bronchospasm that results from ingestion of aspirin by individuals who apparently divert all eicosanoid production to leukotrienes when the cyclooxygenase pathway is blocked (Chapter 18). The toxicity of zileuton includes occasional elevation of liver enzymes, and this drug is therefore less popular than the receptor blockers. CROMOLYN & NEDOCROMIL A. Prototypes and Pharmacokinetics Cromolyn (disodium cromoglycate) and nedocromil are unusually insoluble chemicals, so even massive doses given orally or by aerosol result in minimal systemic blood levels. They are given by aerosol for asthma but are now rarely used in the United States. Cromolyn is the prototype of this group. B. Mechanism of Action and Effects The mechanism of action of these drugs is poorly understood but may involve a decrease in the release of mediators (such as leukotrienes and histamine) from mast cells. The drugs have no bronchodilator action but can prevent bronchoconstriction 174 PART IV Drugs with Important Actions on Smooth Muscle caused by a challenge with antigen to which the patient is allergic. Cromolyn and nedocromil are capable of preventing both early and late responses to challenge (Figure 20–3). Because they are not absorbed from the site of administration, cromolyn and nedocromil have only local effects. When administered orally, cromolyn has some efficacy in preventing food allergy. Similar actions have been demonstrated after local application in the conjunctiva and the nasopharynx for allergic IgE-mediated reactions in these tissues. C. Clinical Uses and Toxicity Asthma (especially in children) was the most important use for cromolyn and nedocromil. Nasal and eyedrop formulations of cromolyn are available for hay fever, and an oral formulation is used for food allergy. Cromolyn and nedocromil may cause cough and irritation of the airway when given by aerosol. Rare instances of drug allergy have been reported. ANTI-IgE ANTIBODY Omalizumab is a humanized murine monoclonal antibody to human IgE. It binds to the IgE on sensitized mast cells and prevents activation by asthma trigger antigens and subsequent release of inflammatory mediators. Although approved in 2003 for the prophylactic management of severe asthma, experience with this drug is limited because it is very expensive and must be administered parenterally. QUESTIONS 1. One effect that theophylline, nitroglycerin, isoproterenol, and histamine have in common is (A) Direct stimulation of cardiac contractile force (B) Tachycardia (C) Bronchodilation (D) Postural hypotension (E) Throbbing headache 2. A 23-year-old woman is using an albuterol inhaler for frequent acute episodes of asthma and complains of symptoms that she ascribes to the albuterol. Which of the following is not a recognized action of albuterol? (A) Diuretic effect (B) Positive inotropic effect (C) Skeletal muscle tremor (D) Smooth muscle relaxation (E) Tachycardia 3. A 10-year-old child has severe asthma and was hospitalized 5 times between the ages of 7 and 9. He is now receiving outpatient medications that have greatly reduced the frequency of severe attacks. Which of the following is most likely to have adverse effects when used daily over long periods for severe asthma? (A) Albuterol by aerosol (B) Beclomethasone by aerosol (C) Ipratropium by inhaler (D) Prednisone by mouth (E) Theophylline in long-acting oral form 4–5. A 16-year-old patient is in the emergency department receiving nasal oxygen. She has a heart rate of 125 bpm, a respiratory rate of 40 breaths/min, and a peak expiratory flow <50% of the predicted value. Wheezing and rales are audible without a stethoscope. 4. Which of the following drugs does not have a direct bronchodilator effect? (A) Epinephrine (B) Terbutaline (C) Prednisone (D) Theophylline (E) Ipratropium 5. After successful treatment of the acute attack, the patient was referred to the outpatient clinic for follow-up treatment for asthma. Which of the following is not an established prophylactic strategy for asthma? (A) Avoidance of antigen exposure (B) Blockade of histamine receptors (C) Blockade of leukotriene receptors (D) IgE antibody blockade (E) Inhibition of phospholipase A2 6. Mr Green is a 60-year-old former smoker with cardiac disease and severe chronic obstructive pulmonary disease (COPD) associated with frequent episodes of bronchospasm. Which of the following is a bronchodilator useful in COPD and least likely to cause cardiac arrhythmia? (A) Aminophylline (B) Cromolyn (C) Epinephrine (D) Ipratropium (E) Metaproterenol (F) Metoprolol (G) Prednisone (H) Salmeterol (I) Zafirlukast (J) Zileuton 7. A 22-year-old man is brought to the emergency department after suffering seizures resulting from an overdose of a drug he has been taking. His friends state that he took the drug orally and sometimes had insomnia after taking it. Which of the following is a direct bronchodilator that is most often used in asthma by the oral route and is capable of causing insomnia and seizures? (A) Cromolyn (B) Epinephrine (C) Ipratropium (D) Metaproterenol (E) Metoprolol (F) Prednisone (G) Salmeterol (H) Theophylline (I) Zileuton CHAPTER 20 Drugs Used in Asthma & Chronic Obstructive Pulmonary Disease 8. Which of the following in its parenteral form is life-saving in severe status asthmaticus and acts, at least in part, by inhibiting phospholipase A2? (A) Aminophylline (B) Cromolyn (C) Epinephrine (D) Ipratropium (E) Metaproterenol (F) Metoprolol (G) Prednisone (H) Salmeterol (I) Zafirlukast (J) Zileuton 9. Which of the following has a slow onset but long duration of action and is always used in combination with a corticosteroid by inhalation? (A) Aminophylline (B) Cromolyn (C) Epinephrine (D) Ipratropium (E) Metaproterenol (F) Metoprolol (G) Prednisone/prednisolone (H) Salmeterol (I) Zafirlukast (J) Zileuton 10. Oral medications are popular for the treatment of asthma in children because young children may have difficulty with the proper use of aerosol inhalers. Which of the following is an orally active inhibitor of leukotriene receptors? (A) Albuterol (B) Aminophylline (C) Ipratropium (D) Montelukast (E) Zileuton 175 4. Although extremely important in severe chronic asthma and status asthmaticus, corticosteroids do not have a demonstrable direct bronchodilator action. The answer is C. 5. Histamine does not appear to play a significant role in asthma, and antihistaminic drugs, even in high doses, are of little or no value. Antigen avoidance is well established. Blockade of leukotriene receptors with montelukast; inhibition of phospholipase with corticosteroids; and inhibition of mediator release with the IgE antibody are also useful. The answer is B. 6. Ipratropium or a similar antimuscarinic agent is the bronchodilator that is most likely to be useful in COPD without causing arrhythmias. The answer is D. 7. Theophylline is a bronchodilator that is active by the oral route. It causes insomnia in therapeutic doses and seizures in overdosage. The answer is H. 8. Parenteral corticosteroids such as prednisolone (the active metabolite of prednisone) are lifesaving in status asthmaticus. They probably act by reducing production of leukotrienes (see Chapter 18). The answer is G. 9. Salmeterol is a β2-selective agonist that has a slow onset and long duration of action. Used alone, it increases asthma mortality, but in combination with inhaled corticosteroids prophylactic use improves asthma control. The answer is H. 10. Zileuton is an inhibitor of the lipoxygenase enzyme involved in the synthesis of leukotrienes. Montelukast and zafirlukast are leukotriene antagonists at the leukotriene receptor. The answer is D. ANSWERS 1. Theophylline does not ordinarily cause headache or postural hypotension. Nitroglycerin does not cause direct cardiac stimulation but does evoke a compensatory sympathetic reflex. Histamine does not cause bronchodilation. The answer is B. 2. Albuterol is a β2-selective receptor agonist, but in moderate to high doses it produces β1 cardiac effects as well as β2-mediated smooth and skeletal muscle effects. It does not cause diuresis. The answer is A. 3. Chronic systemic corticosteroids have important toxicities (see Chapter 39). If oral corticosteroids must be used, alternate-day therapy is preferred because it interferes less with normal growth and metabolism in children. The answer is D. SKILL KEEPER ANSWERS: SYMPATHOMIMETICS VS ANTIMUSCARINICS IN ASTHMA (SEE CHAPTERS 8 AND 9) Direct-acting sympathomimetics are usually rapid in onset and short acting (eg, epinephrine, albuterol; exceptions: salmeterol, formoterol, indacaterol, vilanterol). They are extremely efficacious and actively relax the bronchioles. Antimuscarinic drugs are somewhat slower in onset of action and are therefore used more often as “controller” therapy in COPD rather than “reliever” therapy in asthma. Not all asthma patients have vagal reflex output to the bronchi as a major contributor to the bronchospasm, and these patients will not respond well to an antimuscarinic. On the other hand, a patient with severe cardiac disease may be very sensitive to the arrhythmogenic effects of β agonists and therefore tolerate these agents poorly, while antimuscarinic agents rarely cause arrhythmias. CHECKLIST When you complete this chapter, you should be able to: ❑ Describe the strategies of drug treatment of asthma and COPD. ❑ List the major classes of drugs used in asthma and COPD. ❑ Describe the mechanisms of action of these drug groups. ❑ List the major adverse effects of the prototype drugs used in airways disease. 176 PART IV Drugs with Important Actions on Smooth Muscle DRUG SUMMARY TABLE: Bronchodilators & Other Drugs Used in Asthma & COPD Subclass Mechanism of Action Clinical Applications Pharmacokinetics Toxicities, Interactions Beta2-selective agonist • bronchodilation Asthma acute attack relief drug of choice (not for prophylaxis) Inhalation (aerosol) Duration: 2–4 h Tremor, tachycardia Short-acting a agonists Albuterol Metaproterenol, terbutaline: similar to albuterol; terbutaline also available as oral and parenteral formulations Long-acting a agonists Salmeterol, formoterol, indacaterol, vilanterol Beta2-selective agonists; bronchodilation; potentiation of corticosteroid action Asthma prophylaxis (not for acute relief) • indacaterol and vilanterol for COPD Inhalation (aerosol) Duration: 12–24 h Tremor, tachycardia, cardiovascular events Asthma (obsolete) Inhalation (aerosol, nebulizer) Duration: 1–2 h Excess sympathomimetic effect (Chapter 9) Releases stored catecholamines • causes nonselective sympathetic effects Asthma (obsolete) Oral Duration: 6–8 h Insomnia, tremor, anorexia, arrhythmias Phosphodiesterase inhibition, adenosine receptor antagonist • other effects poorly understood Asthma, especially prophylaxis against nocturnal attacks Oral slow-release Duration: 12 h Insomnia, tremor, anorexia, seizures, arrhythmias Nonselective sympathomimetics Epinephrine, isoproterenol Nonselective β activation • epinephrine also an α agonist Indirect-acting sympathomimetic Ephedrine Methylxanthines Theophylline Roflumilast: a nonpurine molecule with effects similar to theophylline but more selective for PDE4; approved for COPD Caffeine: similar to theophylline with increased CNS effect, not used in asthma or COPD Theobromine: similar to theophylline with increased cardiac effect, not used in asthma or COPD Antimuscarinic agents Ipratropium, tiotropium, aclidinium Competitive pharmacologic muscarinic antagonists Asthma and chronic obstructive pulmonary disease Inhalation (aerosol) Duration: several hours Dry mouth, cough Rarely used prophylaxis of asthma; cromolyn also used for ophthalmic, nasopharyngeal, and gastrointestinal allergy Inhaled aerosol for asthma • cromolyn local application for other applications Duration: 3–6 h Cough Unknown mechanism, possibly mast cell stabilizers Cromolyn, nedocromil Reduce release of inflammatory and bronchoconstrictor mediators from sensitized mast cells (Continued ) CHAPTER 20 Drugs Used in Asthma & Chronic Obstructive Pulmonary Disease 177 DRUG SUMMARY TABLE: Bronchodilators & Other Drugs Used in Asthma & COPD (Continued ) Subclass Mechanism of Action Clinical Applications Pharmacokinetics Toxicities, Interactions Montelukast, zafirlukast Pharmacologic antagonists at LTD4 receptors Prophylaxis of asthma Oral Duration: 12–24 h Minimal Zileuton Inhibitor of lipoxygenase • reduces synthesis of leukotrienes Prophylaxis of asthma Oral Duration: 12 h Elevation of liver enzymes Inhibition of phospholipase A2 • reduces expression of cyclooxygenase Prophylaxis of asthma: drugs of choice Inhalation Duration: 10–12 h Pharyngeal candidiasis • minimal systemic steroid toxicity (eg, adrenal suppression) Like inhaled corticosteroids Treatment of severe refractory chronic asthma Oral Duration: 12–24 h See Chapter 39 Parenteral • administered as several courses of injections Extremely expensive • long-term toxicity not yet well documented Leukotriene antagonists Corticosteroids Inhaled Beclomethasone, others Systemic Prednisone, others Prednisolone: parenteral for status asthmaticus; similar to prednisone Antibodies Omalizumab Binds IgE antibodies on mast cells; reduces reaction to inhaled antigen Prophylaxis of severe, refractory asthma not responsive to all other drugs This page intentionally left blank PART V DRUGS THAT ACT IN THE CENTRAL NERVOUS SYSTEM C Introduction to CNS Pharmacology TARGETS OF CNS DRUG ACTION Most drugs that act on the central nervous system (CNS) appear to do so by changing ion flow through transmembrane channels of nerve cells. A. Types of Ion Channels Ion channels of neuronal membranes are of 2 major types: voltage gated and ligand gated (Figure 21–1). Voltage-gated ion channels respond to changes in membrane potential. They are concentrated on the axons of nerve cells and include the sodium channels responsible for action potential propagation. Cell bodies and dendrites also have voltage-sensitive ion channels for potassium and calcium. Ligand-gated ion channels, also called ionotropic receptors, respond to chemical neurotransmitters that bind to receptor subunits present in their macromolecular structure. Neurotransmitters also bind to G protein-coupled receptors (metabotropic receptors) that can modulate voltage-gated ion channels. Neurotransmitter-coupled ion channels are found on cell bodies and on both the presynaptic and postsynaptic sides of synapses. B. Types of Receptor-Channel Coupling In the case of ligand-gated ion channels, activation (or inactivation) is initiated by the interaction between chemical neurotransmitters and their receptors (Figure 21–1). Coupling may be through a receptor that acts directly on the channel protein (panel B), through a receptor that is coupled to the ion channel through a G protein (C), or through a receptor coupled to a G protein that modulates the formation of diffusible second messengers, including cyclic adenosine monophosphate (cAMP), inositol H A P T E R 21 trisphosphate (IP3), and diacylglycerol (DAG), which secondarily modulate ion channels (D). C. Role of the Ion Current Carried by the Channel Excitatory postsynaptic potentials (EPSPs) are usually generated by the opening of sodium or calcium channels. In some synapses, similar depolarizing potentials result from the closing of potassium channels. Inhibitory postsynaptic potentials (IPSPs) are usually generated by the opening of potassium or chloride channels. For example, activation of postsynaptic metabotropic receptors increases the efflux of potassium. Presynaptic inhibition can occur via a decrease in calcium influx elicited by activation of metabotropic receptors. SITES & MECHANISMS OF DRUG ACTION A small number of neuropharmacologic agents exert their effects through direct interactions with molecular components of ion channels on axons. Examples include certain anticonvulsants (eg, carbamazepine, phenytoin), local anesthetics, and some drugs used in general anesthesia. However, the effects of most therapeutically important CNS drugs are exerted mainly at synapses. Possible mechanisms are indicated in Figure 21–2. Thus, drugs may act presynaptically to alter the synthesis, storage, release, reuptake, or metabolism of transmitter chemicals. Other drugs can activate or block both pre- and postsynaptic receptors for specific transmitters or can interfere with the actions of second messengers. The selectivity of CNS drug action is largely based on the fact 179 180 PART V Drugs That Act in the Central Nervous System High-Yield Terms to Learn Voltage-gated ion channels Transmembrane ion channels regulated by changes in membrane potential Ligand-gated ion channels Transmembrane ion channels that are regulated by interactions between neurotransmitters and their receptors (also called ionotropic receptors) Metabotropic receptors G protein-coupled receptors that respond to neurotransmitters either by a direct action of G proteins on ion channels or by G protein-enzyme activation that leads to formation of diffusible second messengers EPSP Excitatory postsynaptic potential; a depolarizing membrane potential change IPSP Inhibitory postsynaptic potential; a hyperpolarizing membrane potential change Synaptic mimicry Ability of an administered chemical to mimic the actions of the natural neurotransmitter: a criterion for identification of a putative neurotransmitter A B Voltage-gated Ligand-gated ion channel (ionotropic) ++ –– C Membrane-delimited metabotropic ion channel ++ –– α βγ G protein Receptor D Diffusible second messenger metabotropic ion channel ++ –– α Receptor βγ G protein Enzyme Diffusible messenger FIGURE 21–1 Types of ion channels and neurotransmitter receptors in the CNS: A shows a voltage-gated ion channel in which the voltage sensor controls the gating (broken arrow). B shows a ligand-gated ion channel in which binding of the neurotransmitter to the ionotropic channel receptor controls the gating. C shows a metabotropic receptor coupled to a G protein that can interact directly with an ion channel. D shows a receptor coupled to a G protein that activates an enzyme; the activated enzyme generates a diffusible second messenger, for example, cAMP, which interacts to modulate an ion channel. (Reproduced, with permission, from Katzung BG, editor: Basic & Clinical Pharmacology, 12th ed. McGraw-Hill, 2012: Fig. 21–2.) CHAPTER 21 Introduction to CNS Pharmacology B. Diffuse Systems Diffuse or nonspecific systems are broadly distributed, with single cells frequently sending branches to many different areas. The axons are fine and branch repeatedly to form synapses with many cells. Axons commonly have periodic enlargements (varicosities) that contain transmitter vesicles. The transmitters in diffuse systems are often amines (norepinephrine, dopamine, serotonin) or peptides that commonly exert actions on metabotropic receptors. Drugs that affect these systems often have marked effects on such CNS functions as attention, appetite, and emotional states. 1 Synthesis Metabolism 4 2 181 3 Storage TRANSMITTERS AT CENTRAL SYNAPSES 6 5 Reuptake 7 Release Degradation 8 Receptor Ionic conductance 9 FIGURE 21–2 Sites of CNS drug action. Drugs may alter (1) the action potential in the presynaptic fiber; (2) synthesis of transmitter; (3) storage; (4) metabolism; (5) release; (6) reuptake; (7) degradation; (8) receptor for the transmitter; or (9) receptor-induced decrease or increase in ionic conduction. (Reproduced, with permission, from Katzung BG, editor: Basic & Clinical Pharmacology, 12th ed. McGraw-Hill, 2012: Fig. 21–5.) that different groups of neurons use different neurotransmitters and that they are segregated into networks that subserve different CNS functions. A few neurotoxic substances damage or kill nerve cells. For example, 1-methyl-4-phenyl-1,2,3,6-tetrahydropyridine (MPTP) is cytotoxic to neurons of the nigrostriatal dopaminergic pathway. ROLE OF CNS ORGANIZATION The CNS contains 2 types of neuronal systems: hierarchical and diffuse. A. Hierarchical Systems These systems are delimited in their anatomic distribution and generally contain large myelinated, rapidly conducting fibers. Hierarchical systems control major sensory and motor functions. The major excitatory transmitters in these systems are aspartate and glutamate. These systems also include numerous small inhibitory interneurons, which use γ-aminobutyric acid (GABA) or glycine as transmitters. Drugs that affect hierarchical systems often have profound effects on the overall excitability of the CNS. A. Criteria for Transmitter Status To be accepted as a neurotransmitter, a candidate chemical must (1) be present in higher concentration in the synaptic area than in other areas (ie, must be localized in appropriate areas), (2) be released by electrical or chemical stimulation via a calcium-dependent mechanism, and (3) produce the same sort of postsynaptic response that is seen with physiologic activation of the synapse (ie, must exhibit synaptic mimicry). Table 21–1 lists the most important chemicals currently accepted as neurotransmitters in the CNS. B. Acetylcholine Approximately 5% of brain neurons have receptors for acetylcholine (ACh). Most CNS responses to ACh are mediated by a large family of G protein-coupled muscarinic M1 receptors that lead to slow excitation when activated. The ionic mechanism of slow excitation involves a decrease in membrane permeability to potassium. Of the nicotinic receptors present in the CNS (they are less common than muscarinic receptors), those on the Renshaw cells activated by motor axon collaterals in the spinal cord are the best characterized. Drugs affecting the activity of cholinergic systems in the brain include the acetylcholinesterase inhibitors used in Alzheimer’s disease (eg, rivastigmine) and the muscarinic blocking agents used in parkinsonism (eg, benztropine). C. Dopamine Dopamine exerts slow inhibitory actions at synapses in specific neuronal systems, commonly via G protein-coupled activation of potassium channels (postsynaptic) or inhibition of calcium channels (presynaptic). The D2 receptor is the main dopamine subtype in basal ganglia neurons, and it is widely distributed at the supraspinal level. Dopaminergic pathways include the nigrostriatal, mesolimbic, and tuberoinfundibular tracts. In addition to the 2 receptors listed in Table 21–1, 3 other dopamine receptor subtypes have been identified (D3, D4, and D5). Drugs that block the activity of dopaminergic pathways include older antipsychotics (eg, chlorpromazine, haloperidol), which may cause parkinsonian symptoms. Drugs that increase brain dopaminergic activity include CNS stimulants (eg, amphetamine), and commonly used antiparkinsonism drugs (eg, levodopa). 182 PART V Drugs That Act in the Central Nervous System TABLE 21–1 Neurotransmitter pharmacology in the CNS. Transmitter Anatomical Distribution Receptor Subtypes Receptor Mechanisms Acetylcholine Cell bodies at all levels, short and long axons Muscarinic, M1; blocked by pirenzepine and atropine Excitatory; ↓ K+ conductance; ↑ IP3 and DAG Inhibitory; ↑ K+ conductance; ↓ cAMP Motoneuron-Renshaw cell synapse Muscarinic, M2; blocked by atropine Nicotinic, N D1; blocked by phenothiazines Inhibitory; ↑cAMP D2; blocked by phenothiazines and haloperidol Inhibitory (presynaptic); ↓ Ca2+ conductance; Dopamine Cell bodies at all levels, short, medium, and long axons Excitatory; ↑ cation conductance Inhibitory (postsynaptic); ↑ K+ conductance; cAMP Norepinephrine Cell bodies in pons and brain stem project to all levels Alpha1; blocked by prazosin Excitatory; ↓ K+ conductance; ↑ IP3 and DAG Alpha2; activated by clonidine Inhibitory (presynaptic); ↓ Ca2+ conductance Inhibitory (postsynaptic); ↑ K+ conductance; cAMP Serotonin (5-hydroxytryptamine) GABA Cell bodies in midbrain and pons project to all levels Supraspinal interneurons; spinal interneurons involved in presynaptic inhibition Beta1; blocked by propranolol Excitatory; ↓ K+ conductance; ↑ cAMP Beta2; blocked by propranolol Inhibitory; ↑ electrogenic sodium pump 5-HT1A; buspirone is a partial agonist 5-HT2A; blocked by clozapine, risperidone, and olanzapine 5-HT3; blocked by ondansetron Inhibitory; ↑ K+ conductance 5-HT4 Excitatory; ↓ K+ conductance; ↑ cAMP GABAA; facilitated by benzodiazepines and zolpidem Inhibitory; ↑ Cl– conductance GABAB; activated by baclofen Inhibitory (presynaptic); ↓ Ca2+ conductance Excitatory; ↓ K+ conductance; ↑ IP3 and DAG Excitatory; ↑ cation conductance Inhibitory (postsynaptic); ↑ K+ conductance Glutamate, aspartate Relay neurons at all levels Four subtypes; NMDA subtype blocked by phencyclidine, ketamine, and memantine Excitatory; ↑ Ca2+ or cation conductance Metabotropic subtypes Inhibitory (presynaptic); ↓ Ca2+ conductance; ↓ cAMP Excitatory (postsynaptic); ↓ K+ conductance; ↑ IP3 and DAG Glycine Interneurons in spinal cord and brain stem Single subtype; blocked by strychnine Inhibitory; ↑ Cl– conductance Opioid peptides Cell bodies at all levels Three major subtypes: µ, δ, κ Inhibitory (presynaptic); ↓ Ca2+ conductance; ↓cAMP Inhibitory (postsynaptic); ↑ K+ conductance; ↓cAMP Adapted, with permission, from Katzung BG, editor: Basic & Clinical Pharmacology, 12th ed. McGraw-Hill, 2012. CHAPTER 21 Introduction to CNS Pharmacology D. Norepinephrine Noradrenergic neuron cell bodies are mainly located in the brain stem and the lateral tegmental area of the pons. These neurons fan out broadly to provide most regions of the CNS with diffuse noradrenergic input. Excitatory effects are produced by activation of α1 and β1 receptors. Inhibitory effects are caused by activation of α2 and β2 receptors. CNS stimulants (eg, amphetamines, cocaine), monoamine oxidase inhibitors (eg, phenelzine), and tricyclic antidepressants (eg, amitriptyline) are examples of drugs that enhance the activity of noradrenergic pathways. E. Serotonin Most serotonin (5-hydroxytryptamine; 5-HT) pathways originate from cell bodies in the raphe or midline regions of the pons and upper brain stem; these pathways innervate most regions of the CNS. Multiple 5-HT receptor subtypes have been identified and, with the exception of the 5-HT3 subtype, all are metabotropic. 5-HT1A receptors and GABAB receptors share the same potassium channel. Serotonin can cause excitation or inhibition of CNS neurons depending on the receptor subtype activated. Both excitatory and inhibitory actions can occur on the same neuron if appropriate receptors are present. Most of the agents used in the treatment of major depressive disorders affect serotonergic pathways (eg, tricyclic antidepressants, selective serotonin reuptake inhibitors). The actions of some CNS stimulants and newer antipsychotic drugs (eg, olanzapine) also appear to be mediated via effects on serotonergic transmission. Reserpine, which may cause severe depression of mood, depletes vesicular stores of both serotonin and norepinephrine in CNS neurons. F. Glutamic Acid Most neurons in the brain are excited by glutamic acid. High concentrations of glutamic acid in synaptic vesicles are achieved by the vesicular glutamate transporter (VGLUT). Both ionotropic and metabotropic receptors have been characterized. Subtypes of glutamate receptors include the N-methyl-d-aspartate (NMDA) receptor, which is blocked by phencyclidine (PCP) and ketamine. NMDA receptors appear to play a role in synaptic plasticity related to learning and memory. Memantine is an NMDA antagonist introduced for treatment of Alzheimer’s dementia. Excessive activation of NMDA receptors after neuronal injury may be responsible for cell death. Glutamate metabotropic receptor activation can result in G protein-coupled activation of phospholipase C or inhibition of adenylyl cyclase. G. GABA and Glycine GABA is the primary neurotransmitter mediating IPSPs in neurons in the brain; it is also important in the spinal cord. GABAA receptor activation opens chloride ion channels. GABAB receptors (activated by baclofen, a centrally acting muscle relaxant) are coupled to G proteins that either open potassium channels or close calcium channels. Fast IPSPs are blocked by GABAA receptor antagonists, and slow IPSPs are blocked by GABAB receptor antagonists. Drugs that influence GABAA receptor systems include sedative-hypnotics (eg, barbiturates, benzodiazepines, zolpidem) 183 and some anticonvulsants (eg, gabapentin, tiagabine, vigabatrin). Glycine receptors, which are more numerous in the cord than in the brain, are blocked by strychnine, a spinal convulsant. H. Peptide Transmitters Many peptides have been identified in the CNS, and some meet most or all of the criteria for acceptance as neurotransmitters. The best-defined peptides are the opioid peptides (beta-endorphin, met- and leu-enkephalin, and dynorphin), which are distributed at all levels of the neuraxis. Some of the important therapeutic actions of opioid analgesics (eg, morphine) are mediated via activation of receptors for these endogenous peptides. Another peptide, substance P, is a mediator of slow EPSPs in neurons involved in nociceptive sensory pathways in the spinal cord and brain stem. Peptide transmitters differ from nonpeptide transmitters in that (1) the peptides are synthesized in the cell body and transported to the nerve ending via axonal transport, and (2) no reuptake or specific enzyme mechanisms have been identified for terminating their actions. I. Endocannabinoids These are widely distributed brain lipid derivatives (eg, 2-arachidonyl-glycerol) that bind to receptors for cannabinoids found in marijuana. They are synthesized and released postsynaptically after membrane depolarization but travel backward acting presynaptically (retrograde) to decrease transmitter release, via their interaction with a specific cannabinoid receptor. SKILL KEEPER: BIODISPOSITION OF CNS DRUGS (SEE CHAPTER 1) 1. What characteristics of drug molecules afford access to the CNS? 2. What concerns do you have regarding CNS drug use in the pregnant patient? 3. How are most CNS drugs usually eliminated from the body? The Skill Keeper Answers appear at the end of the chapter. QUESTIONS 1. Which of the following chemicals does not satisfy the criteria for a neurotransmitter role in the CNS? (A) Acetylcholine (B) Cyclic AMP (C) Dopamine (D) Glycine (E) Substance P 2. Neurotransmitters may (A) Increase chloride conductance to cause inhibition (B) Increase potassium conductance to cause inhibition (C) Increase sodium conductance to cause excitation (D) Increase calcium conductance to cause excitation (E) Exert all of the above actions 184 PART V Drugs That Act in the Central Nervous System 3. All of the listed neurotransmitters change membrane excitability by decreasing K+ conductance EXCEPT (A) Acetylcholine (B) Dopamine (C) Glutamic acid (D) Norepinephrine (E) Serotonin 4. Which of the following receptors shares the same potassium channel as the 5-HT1A receptor? (A) Dopamine D2 receptor (B) GABAB receptor (C) Mu opioid receptor (D) Muscarinic M1 receptor (E) Substance P receptor 5. Which of the following chemicals is most likely to function as a neurotransmitter in hierarchical systems? (A) GABA (B) Glutamate (C) Met-enkephalin (D) Nitric oxide (E) Norepinephrine 6. Activation of metabotropic receptors located presynaptically causes inhibition by decreasing the inward flux of (A) Calcium (B) Chloride (C) Potassium (D) Sodium (E) None of the above 7. This transmitter is mostly located in diffuse neuronal systems in the CNS, with cell bodies particularly in the raphe nuclei. It appears to play a major role in the expression of mood states, and many antidepressant drugs are thought to increase its functional activity. (A) Acetylcholine (B) Dopamine (C) GABA (D) Glutamate (E) Serotonin 8. Cyclic adenosine monophosphate (cAMP) functions as a diffusible second messenger after activation of (A) Acetylcholine M1 receptors (B) Beta1 adrenoceptors (C) 5-HT3 receptors (D) GABAA receptors (E) Glutamate NMDA receptors 9. One of the first neurotransmitter receptors to be identified in the CNS is located on the Renshaw cell in the spinal cord. Activation of this receptor results in excitation via an increase in cation (Na+, K+ ) conductance independently of G proteincoupled mechanisms. Which of the following compounds is most likely to activate this receptor? (A) Dopamine (B) Glycine (C) GABA (D) Nicotine (E) Serotonin 10. This neurotransmitter, found in high concentrations in cell bodies in the pons and brain stem, can exert both excitatory and inhibitory actions. Multiple receptor subtypes have been identified, some of which are targets for drugs that can exert both CNS and peripheral actions. (A) Acetylcholine (B) Beta-endorphin (C) Glycine (D) Glutamate (E) Norepinephrine ANSWERS 1. Cyclic AMP (cAMP) is a mediator in many receptor mechanisms in the CNS, including those for acetylcholine (M2), and norepinephrine (β1). However, the characteristics of cAMP do not satisfy the criteria for a neurotransmitter role (see A. Criteria for Transmitter Status). The answer is B. 2. Activation of chloride or potassium ion channels commonly generates inhibitory postsynaptic potentials (IPSPs). Activation of sodium and calcium channels (and inhibition of potassium ion channels) generate excitatory postsynaptic potentials (EPSPs). The answer is E. 3. A decrease in K+ conductance is associated with neuronal excitation. With the exception of dopamine, all of the neurotransmitters listed are able to cause excitation by this mechanism via their activation of specific receptors: acetylcholine (M1), glutamate (metabotropic), norepinephrine (α1 and β1), and serotonin (5-HT2A). The answer is B. 4. GABAB receptors and 5-HT1A receptors share the same potassium ion channel, with a G protein involved in the coupling mechanism. The spasmolytic drug baclofen is an activator of GABAB receptors in the spinal cord. The anxiolytic drug buspirone may act as a partial agonist at brain 5-HT1A receptors. The answer is B. 5. Catecholamines (dopamine, norepinephrine), opioid peptides, and serotonin act as neurotransmitters in nonspecific or diffuse neuronal systems. Glutamate is the primary excitatory transmitter in hierarchical neuronal systems. These systems also contain numerous inhibitory neurons, which use GABA and glycine. Nitric oxide, though present in many brain regions, does not meet the critera for a CNS neurotransmitter. The answer is B. 6. Activation of metabotropic receptors located presynaptically results in the inhibition of calcium influx with a resultant decrease in the release of neurotransmitter from nerve endings. This type of presynaptic inhibition occurs after activation of dopamine D2, norepinephrine α2, glutamate, and mu opioid peptide receptors. The answer is A. 7. Amine transmitters thought to be involved in the control of mood states include norepinephrine and serotonin. Cell bodies of serotonergic neurons are found in the raphe nuclei. Many of the drugs used for the treatment of major depressive disorders act to increase serotonergic activity in the CNS. The answer is E. CHAPTER 21 Introduction to CNS Pharmacology 8. Metabotropic receptors modulate voltage-gated ion channels directly (membrane-delimited action) and also by the formation of diffusible second messengers through G proteinmediated effects on enzymes involved in their synthesis. An example of the latter type of action is provided by the β1 adrenoceptor, which generates cAMP via the activation of adenylyl cyclase. The answer is B. 9. Nicotinic receptors on the Renshaw cell are activated by the release of ACh from motor neuron collaterals. This results in the release of glycine, which, via interaction with its receptors on the motor neuron, causes membrane hyperpolarization, an example of feedback inhibition. The receptors were so named because of their activation by nicotine. The answer is D. 10. The brief description might apply to several CNS neurotransmitters, including serotonin and possibly dopamine (neither of which is listed). Cell bodies of noradrenergic neurons located in the pons and brain stem project to all levels of the CNS. Most of the subclasses of adrenergic receptors that occur in peripheral tissues are present in the CNS. Agents that activate presynaptic α2 receptors on such neurons (eg, clonidine, methyldopa) decrease central noradrenergic activity, an action thought to result in decreased vasomotor outflow. The answer is E. 185 SKILL KEEPER ANSWERS: BIODISPOSITION OF CNS DRUGS (SEE CHAPTER 1) 1. Lipid solubility is an important characteristic of most CNS drugs in terms of their ability to cross the blood-brain barrier. Access to the CNS of water-soluble (polar) molecules is limited to those of low molecular weight such as lithium ion and ethanol. 2. CNS drugs readily cross the placental barrier and enter the fetal circulation. Concerns during pregnancy include possible effects on fetal development and the potential for drug effects on the neonate if CNS drugs are used near the time of delivery. 3. With the notable exception of lithium, almost all CNS drugs require metabolism to more water-soluble (polar) metabolites for their elimination. Thus, drugs that modify the activities of drug-metabolizing enzymes may have an impact on the clearance of CNS drugs, possibly affecting the intensity or duration of their effects. CHECKLIST When you complete this chapter, you should be able to: ❑ Explain the difference between voltage-gated and ligand-gated ion channels. ❑ List the criteria for accepting a chemical as a neurotransmitter. ❑ Identify the major excitatory and inhibitory CNS neurotransmitters in the CNS. ❑ Identify the sites of drug action at synapses and the mechanisms by which drugs modulate synaptic transmission. ❑ Give an example of a CNS drug that influences neurotransmitter functions at the level of (a) synthesis, (b) metabolism, (c) release, (d) reuptake, and (e) receptor. C H A P T E R 22 Sedative-Hypnotic Drugs The sedative-hypnotics belong to a chemically heterogeneous class of drugs almost all of which produce dose-dependent CNS depressant effects. A major subgroup is the benzodiazepines, but representatives of other subgroups, including barbiturates, and miscellaneous agents (carbamates, alcohols, and cyclic ethers) are still in use. Newer drugs with distinctive characteristics include the anxiolytic buspirone, several widely used hypnotics (zolpidem, zaleplon, eszopiclone), and melatonin agonists and orexin antagonists, novel drugs used in sleep disorders. Sedative-hypnotics Benzodiazepines Short action (triazolam) Barbiturates Ultra-short action (thiopental) Intermediate action (alprazolam) Long action (flurazepam) PHARMACOKINETICS A. Absorption and Distribution Most sedative-hypnotic drugs are lipid-soluble and are absorbed well from the gastrointestinal tract, with good distribution to the brain. Drugs with the highest lipid solubility (eg, thiopental) enter the CNS rapidly and can be used as induction agents in anesthesia. The CNS effects of thiopental are terminated by rapid redistribution of the drug from brain to other highly perfused tissues, including skeletal muscle. Other drugs with a rapid onset of CNS action include eszopiclone, zaleplon, and zolpidem. B. Metabolism and Excretion Sedative-hypnotics are metabolized before elimination from the body, mainly by hepatic enzymes. Metabolic rates and pathways vary among different drugs. Many benzodiazepines are converted 186 Short action (secobarbital) Long action (phenobarbital) Miscellaneous agents Buspirone Chloral hydrate Eszopiclone Ramelteon Zaleplon Zolpidem initially to active metabolites with long half-lives. After several days of therapy with some drugs (eg, diazepam, flurazepam), accumulation of active metabolites can lead to excessive sedation. Lorazepam and oxazepam undergo extrahepatic conjugation and do not form active metabolites. With the exception of phenobarbital, which is excreted partly unchanged in the urine, the barbiturates are extensively metabolized. Chloral hydrate is oxidized to trichloroethanol, an active metabolite. Rapid metabolism by liver enzymes is responsible for the short duration of action of zolpidem. A biphasic release form of zolpidem extends its plasma half-life. Zaleplon undergoes even more rapid hepatic metabolism by aldehyde oxidase and cytochrome P450. Eszopiclone is also metabolized by cytochrome P450 with a half-life of 6 h. The duration of CNS actions of sedative-hypnotic drugs ranges from just a few hours (eg, zaleplon < zolpidem = triazolam = eszopiclone < chloral hydrate) to more than 30 h (eg, chlordiazepoxide, clorazepate, diazepam, phenobarbital). CHAPTER 22 Sedative-Hypnotic Drugs 187 High-Yield Terms to Learn Addiction The state of response to a drug whereby the drug taker feels compelled to use the drug and suffers anxiety when separated from it Anesthesia Loss of consciousness associated with absence of response to pain Anxiolytic A drug that reduces anxiety, a sedative Dependence The state of response to a drug whereby removal of the drug evokes unpleasant, possibly life-threatening symptoms, often the opposite of the drug’s effects Hypnosis Induction of sleep REM sleep Phase of sleep associated with rapid eye movements; most dreaming takes place during REM sleep Sedation Reduction of anxiety Tolerance Reduction in drug effect requiring an increase in dosage to maintain the same response MECHANISMS OF ACTION No single mechanism of action for sedative-hypnotics has been identified, and the different chemical subgroups may have different actions. Certain drugs (eg, benzodiazepines) facilitate neuronal membrane inhibition by actions at specific receptors. C. Other Drugs The hypnotics zolpidem, zaleplon, and eszopiclone are not benzodiazepines but appear to exert their CNS effects via interaction with certain benzodiazepine receptors, classified as BZ1 or ω1 Cl– GABA A. Benzodiazepines Receptors for benzodiazepines (BZ receptors) are present in many brain regions, including the thalamus, limbic structures, and the cerebral cortex. The BZ receptors form part of a GABAA receptorchloride ion channel macromolecular complex, a pentameric structure assembled from 5 subunits each with 4 transmembrane domains. A major isoform of the GABAA receptor consists of 2 α1, 2 β2, and 1 γ 2 subunits. In this isoform, the binding site for benzodiazepines is between an α1 and the γ 2 subunit. However, benzodiazepines also bind to other GABAA receptor isoforms that contain α2, α3, and α5 subunits. Binding of benzodiazepines facilitates the inhibitory actions of GABA, which are exerted through increased chloride ion conductance (Figure 22–1). Benzodiazepines increase the frequency of GABA-mediated chloride ion channel opening. Flumazenil reverses the CNS effects of benzodiazepines and is classified as an antagonist at BZ receptors. Certain β-carbolines have a high affinity for BZ receptors and can elicit anxiogenic and convulsant effects. These drugs are classified as inverse agonists. B. Barbiturates Barbiturates depress neuronal activity in the midbrain reticular formation, facilitating and prolonging the inhibitory effects of GABA and glycine. Barbiturates also bind to multiple isoforms of the GABAA receptor but at different sites from those with which benzodiazepines interact. Their actions are not antagonized by flumazenil. Barbiturates increase the duration of GABA-mediated chloride ion channel opening. They may also block the excitatory transmitter glutamic acid, and, at high concentration, sodium channels. GABA β β α α Benzodiazepines Flumazenil Zolpidem Extracellular γ Barbiturates Intracellular Ion channel FIGURE 22–1 A model of the GABAA receptor-chloride ion channel macromolecular complex. A hetero-oligomeric glycoprotein, the complex consists of 5 or more membrane–spanning subunits. Multiple forms of α, β, and γ subunits are arranged in various pentameric combinations so that GABAA receptors exhibit molecular heterogeneity. GABA appears to interact at two sites between α and β subunits, triggering chloride channel opening with resulting membrane hyperpolarization. Binding of benzodiazepines and the newer hypnotic drugs such as zolpidem occurs at a single site between α and γ subunits, facilitating the process of chloride ion channel opening. The benzodiazepine antagonist flumazenil also binds at this site and can reverse the hypnotic effects of zolpidem. Note that these binding sites are distinct from those of the barbiturates. (Reproduced, with permission, from Katzung BG, editor: Basic & Clinical Pharmacology, 12th ed. McGraw-Hill, 2012: Fig. 22–6.) 188 PART V Drugs That Act in the Central Nervous System subtypes. In contrast to benzodiazepines, these drugs bind more selectively, interacting only with GABAA receptor isoforms that contain α1 subunits. Their CNS depressant effects can be antagonized by flumazenil. PHARMACODYNAMICS The CNS effects of most sedative-hypnotics depend on dose, as shown in Figure 22–2. These effects range from sedation and relief of anxiety (anxiolysis), through hypnosis (facilitation of sleep), to anesthesia and coma. Depressant effects are additive when 2 or more drugs are given together. The steepness of the dose–response curve varies among drug groups; those with flatter curves, such as benzodiazepines and the newer hypnotics (eg, zolpidem), are safer for clinical use. A. Sedation Sedative actions, with relief of anxiety, occur with all drugs in this class. Anxiolysis is usually accompanied by some impairment of psychomotor functions, and behavioral disinhibition may also occur. In animals, most conventional sedative-hypnotics release punishment-suppressed behavior. B. Hypnosis Sedative-hypnotics can promote sleep onset and increase the duration of the sleep state. Rapid eye movement (REM) sleep duration is usually decreased at high doses; a rebound increase in REM sleep may occur on withdrawal from chronic drug use. Effects on sleep patterns occur infrequently with newer hypnotics such as zaleplon and zolpidem. Coma Central nervous system effects (Barbiturates) Medullary depression Anesthesia D. Anticonvulsant Actions Suppression of seizure activity occurs with high doses of most of the barbiturates and some of the benzodiazepines, but this is usually at the cost of marked sedation. Selective anticonvulsant action (ie, suppression of convulsions at doses that do not cause severe sedation) occurs with only a few of these drugs (eg, phenobarbital, clonazepam). High doses of intravenous diazepam, lorazepam, or phenobarbital are used in status epilepticus. In this condition, heavy sedation is desirable. E. Muscle Relaxation Relaxation of skeletal muscle occurs only with high doses of most sedative-hypnotics. However, diazepam is effective at sedative dose levels for specific spasticity states, including cerebral palsy. Meprobamate also has some selectivity as a muscle relaxant. F. Medullary Depression High doses of conventional sedative-hypnotics, especially alcohols and barbiturates, can cause depression of medullary neurons, leading to respiratory arrest, hypotension, and cardiovascular collapse. These effects are the cause of death in suicidal overdose. SKILL KEEPER: LOADING DOSE (SEE CHAPTER 3) Three hours after ingestion of an unknown quantity of diazepam, a patient was hospitalized and the drug concentration in the plasma was found to be 2 mg/L. Assume that in this patient the pharmacokinetic parameters for diazepam are as follows: oral bioavailability, 100%; Vd, 80 L; CL, 38 L/day; halflife, 2 days. Estimate the dose of diazepam ingested. The Skill Keeper Answer appears at the end of the chapter. (Benzodiazepines) Hypnosis Sedation, disinhibition, anxiolysis C. Anesthesia At high doses of most older sedative-hypnotics, loss of consciousness may occur, with amnesia and suppression of reflexes. Anterograde amnesia is more likely with benzodiazepines than with other sedative-hypnotics. Anesthesia can be produced by most barbiturates (eg, thiopental) and certain benzodiazepines (eg, midazolam). Possible selective anticonvulsant and muscle-relaxing activity Increasing sedative-hypnotic dose FIGURE 22–2 Relationships between dose of benzodiazepines and barbiturates and their CNS effects. G. Tolerance and Dependence Tolerance—a decrease in responsiveness—occurs when sedativehypnotics are used chronically or in high dosage. Cross-tolerance may occur among different chemical subgroups. Psychological dependence occurs frequently with most sedative-hypnotics and is manifested by the compulsive use of these drugs to reduce anxiety. Physiologic dependence constitutes an altered state that leads to an abstinence syndrome (withdrawal state) when the drug is discontinued. Withdrawal signs, which may include anxiety, tremors, hyperreflexia, and seizures, occur more commonly with shorter-acting drugs. The dependence liability of zolpidem, zaleplon, and eszopiclone may be less than that of the CHAPTER 22 Sedative-Hypnotic Drugs benzodiazepines since withdrawal symptoms are minimal after their abrupt discontinuance. CLINICAL USES Most of these uses can be predicted from the pharmacodynamic effects outlined previously. A. Anxiety States Benzodiazepines are favored in the drug treatment of acute anxiety states and for rapid control of panic attacks. Although it is difficult to demonstrate the superiority of one drug over another, alprazolam and clonazepam have greater efficacy than other benzodiazepines in the longer term treatment of panic and phobic disorders. Note the increasing use of newer antidepressants in the treatment of chronic anxiety states (see Chapter 30). B. Sleep Disorders Benzodiazepines, including estazolam, flurazepam, and triazolam, have been widely used in primary insomnia and for the management of certain other sleep disorders. Lower doses should be used in elderly patients who are more sensitive to their CNS depressant effects. More recently there has been increasing use of zolpidem, zaleplon, and eszopiclone in insomnia, since they have rapid onset with minimal effects on sleep patterns and cause less daytime cognitive impairment than benzodiazepines. Note that sedativehypnotic drugs are not recommended for breathing-related sleep disorders, eg, sleep apnea. C. Other Uses Thiopental is commonly used for the induction of anesthesia, and certain benzodiazepines (eg, diazepam, midazolam) are used as components of anesthesia protocols including those used in day surgery. Special uses include the management of seizure disorders (eg, clonazepam, phenobarbital) and bipolar disorder (eg, clonazepam) and treatment of muscle spasticity (eg, diazepam). Longer acting benzodiazepines (eg, chlordiazepoxide, diazepam) are used in the management of withdrawal states in persons physiologically dependent on ethanol and other sedative-hypnotics. TOXICITY A. Psychomotor Dysfunction This includes cognitive impairment, decreased psychomotor skills, and unwanted daytime sedation. These adverse effects are more common with benzodiazepines that have active metabolites with long half-lives (eg, diazepam, flurazepam), but can also occur after a single dose of a short-acting benzodiazepine such as triazolam. The dosage of a sedative-hypnotic should be reduced in elderly patients, who are more susceptible to drugs that cause psychomotor dysfunction. In such patients excessive daytime sedation has been shown to increase the risk of falls and fractures. Anterograde amnesia may also occur with benzodiazepines, 189 especially when used at high dosage, an action that forms the basis for their criminal use in cases of “date rape.” Zolpidem and the newer hypnotics cause modest day-after psychomotor depression with few amnestic effects. However, all prescription drugs used as sleep aids may cause functional impairment, including “sleep driving,” defined as “driving while not fully awake after ingestion of a sedative-hypnotic product, with no memory of the event.” B. Additive CNS Depression This occurs when sedative-hypnotics are used with other drugs in the class as well as with alcoholic beverages, antihistamines, antipsychotic drugs, opioid analgesics, and tricyclic antidepressants. This is the most common type of drug interaction involving sedative-hypnotics. C. Overdosage Overdosage of sedative-hypnotic drugs causes severe respiratory and cardiovascular depression; these potentially lethal effects are more likely to occur with alcohols, barbiturates, and carbamates than with benzodiazepines or the newer hypnotics such as zolpidem. Management of intoxication requires maintenance of a patent airway and ventilatory support. Flumazenil may reverse CNS depressant effects of benzodiazepines, eszopiclone, zolpidem, and zaleplon but has no beneficial actions in overdosage with other sedative-hypnotics. D. Other Adverse Effects Barbiturates and carbamates (but not benzodiazepines, eszopiclone, zolpidem, or zaleplon) induce the formation of the liver microsomal enzymes that metabolize drugs. This enzyme induction may lead to multiple drug interactions. Barbiturates may also precipitate acute intermittent porphyria in susceptible patients. Chloral hydrate may displace coumarins from plasma protein binding sites and increase anticoagulant effects. ATYPICAL SEDATIVE-HYPNOTICS A. Buspirone Buspirone is a selective anxiolytic, with minimal CNS depressant effects (it does not affect driving skills) and has no anticonvulsant or muscle relaxant properties. The drug interacts with the 5-HT1A subclass of brain serotonin receptors as a partial agonist, but the precise mechanism of its anxiolytic effect is unknown. Buspirone has a slow onset of action (>1 week) and is used in generalized anxiety disorders, but is less effective in panic disorders. Tolerance development is minimal with chronic use, and there is little rebound anxiety or withdrawal symptoms on discontinuance. Buspirone is metabolized by CYP3A4, and its plasma levels are markedly increased by drugs such as erythromycin and ketoconazole. Side effects of buspirone include tachycardia, paresthesias, pupillary constriction, and gastrointestinal distress. Buspirone has minimal abuse liability and is not a schedule-controlled drug. The drug appears to be safe in pregnancy. 190 PART V Drugs That Act in the Central Nervous System B. Ramelteon This novel hypnotic drug activates melatonin receptors in the suprachiasmatic nuclei of the CNS and decreases the latency of sleep onset with minimal rebound insomnia or withdrawal symptoms. Ramelteon has no direct effects on GABA-ergic neurotransmission in the CNS. Unlike conventional hypnotics ramelteon appears to have minimal abuse liability, and it is not a controlled substance. The drug is metabolized by hepatic cytochrome P450, forming an active metabolite. The P450 inducer rifampin markedly reduces plasma levels of ramelteon and its metabolite. Conversely, inhibitors of CYP1A2 (eg, fluvoxamine) or CYP2C9 (eg, fluconazole) increase plasma levels of ramelteon. The adverse effects of the drug include dizziness, fatigue, and endocrine changes including decreased testosterone and increased prolactin. Tasimelteon, a similar melatonin receptor agonist, has recently been approved. C. Orexin Antagonists Orexin is a peptide found in the hypothalamus and is involved in wakefulness. Suvorexant, a recently approved antagonist at orexin receptors, has hypnotic properties. QUESTIONS 1. A 43-year-old very overweight man complains of not sleeping well and feeling tired during the day. He says that his wife is the cause of the problem because she wakes him up several times during the night because of his loud snores. This appears to be a breathing-related sleep disorder, so you should probably write a prescription for (A) Clorazepate (B) Diazepam (C) Flurazepam (D) Pentobarbital (E) None of the above 2. Which statement concerning the barbiturates is accurate? (A) Abstinence syndromes are more severe during withdrawal from phenobarbital than from secobarbital (B) Alkalinization of the urine accelerates the elimination of phenobarbital (C) Barbiturates may increase the half-lives of drugs metabolized by the liver (D) Compared with barbiturates, the benzodiazepines exhibit a steeper dose-response relationship (E) Respiratory depression caused by barbiturate overdosage can be reversed by flumazenil 3. A 24-year-old stockbroker has developed a “nervous disposition.” He is easily startled, worries about inconsequential matters, and sometimes complains of stomach cramps. At night he grinds his teeth in his sleep. There is no history of drug abuse. Diagnosed as suffering from generalized anxiety disorder, he is prescribed buspirone. The patient should be informed to anticipate (A) A need to continually increase drug dosage because of tolerance (B) A significant effect of the drug on memory (C) Additive CNS depression with alcoholic beverages (D) That the drug is likely to take a week or more to begin working (E) That if he stops taking the drug abruptly, he will experience withdrawal signs 4. Which of the following best describes the mechanism of action of benzodiazepines? (A) Activate GABAB receptors in the spinal cord (B) Block glutamate receptors in hierarchical neuronal pathways in the brain (C) Increase frequency of opening of chloride ion channels coupled to GABAA receptors (D) Inhibit GABA transaminase to increase brain levels of GABA (E) Stimulate release of GABA from nerve endings in the brain 5. An 82-year-old woman, otherwise healthy for her age, has difficulty sleeping. Triazolam is prescribed for her at one half of the conventional adult dose. Which statement about the use of triazolam in this elderly patient is accurate? (A) Ambulatory dysfunction is unlikely to occur in elderly patients taking one half of the conventional adult dose (B) Hypertension is a common adverse effect of benzodiazepines in elderly patients (C) Over-the-counter cold medications may antagonize the hypnotic effects of the drug (D) The patient may experience amnesia, especially if she also consumes alcoholic beverages (E) Triazolam does not cause rebound insomnia on abrupt discontinuance 6. The most likely explanation for the increased sensitivity of elderly patients after a single dose of a benzodiazepine is (A) Age-dependent changes in brain function (B) Decreases in plasma protein binding (C) Decreased metabolism of lipid-soluble drugs (D) Decreases in renal function (E) Increased cerebral blood flow 7. A 40-year-old woman has sporadic attacks of intense anxiety with marked physical symptoms, including hyperventilation, tachycardia, and sweating. If she is diagnosed as suffering from a panic disorder, the most appropriate drug to use is (A) Alprazolam (B) Eszopiclone (C) Flurazepam (D) Propranolol (E) Ramelteon 8. Which drug used in the maintenance treatment of patients with tonic-clonic or partial seizure states increases the hepatic metabolism of many drugs including both phenytoin and warfarin? (A) Buspirone (B) Clonazepam (C) Eszopiclone (D) Phenobarbital (E) Triazolam 9. A patient with liver dysfunction is scheduled for a surgical procedure. Lorazepam or oxazepam can be used for preanesthetic sedation in this patient without special concern regarding excessive CNS depression because these drugs are (A) Actively secreted in the renal proximal tubule (B) Conjugated extrahepatically (C) Eliminated via the lungs (D) Reversible by administration of naloxone (E) Selective anxiolytics like buspirone CHAPTER 22 Sedative-Hypnotic Drugs 10. This drug used in the management of insomnia facilitates the inhibitory actions of GABA, but it lacks anticonvulsant or muscle-relaxing properties and has minimal effect on sleep architecture. Its actions are antagonized by flumazenil. (A) Buspirone (B) Chlordiazepoxide (C) Eszopiclone (D) Ramelteon (E) Phenobarbital ANSWERS 1. Benzodiazepines and barbiturates are contraindicated in breathing-related sleep disorders because they further compromise ventilation. In obstructive sleep apnea (pickwickian syndrome), obesity is a major risk factor. The best prescription you can give this patient is to lose weight. The answer is E. 2. Withdrawal symptoms from use of the shorter-acting barbiturate secobarbital are more severe than with phenobarbital. The dose-response curve for benzodiazepines is flatter than that for barbiturates. Induction of liver drug-metabolizing enzymes occurs with barbiturates and may lead to decreases in half-life of other drugs. Flumazenil is an antagonist at BZ receptors and is used to reverse CNS depressant effects of benzodiazepines. As a weak acid (pKa 7), phenobarbital will be more ionized (nonprotonated) in the urine at alkaline pH and less reabsorbed in the renal tubule. The answer is B. 3. Buspirone is a selective anxiolytic with pharmacologic characteristics different from those of sedative-hypnotics. Buspirone has minimal effects on cognition or memory; it is not additive with ethanol in terms of CNS depression; tolerance is minimal; and it has no dependence liability. Buspirone is not effective in acute anxiety because it has a slow onset of action. The answer is D. 4. Benzodiazepines exert most of their CNS effects by increasing the inhibitory effects of GABA, interacting with components of the GABAA receptor-chloride ion channel macromolecular complex to increase the frequency of chloride ion channel opening. Benzodiazepines do not affect GABA metabolism or release, and they are not GABA receptor agonists because they do not interact directly with the binding site for GABA. The answer is C. 5. In elderly patients taking benzodiazepines, hypotension is far more likely than an increase in blood pressure. Elderly patients are more prone to the CNS depressant effects of hypnotics; a dose reduction of 50% may still cause excessive sedation with possible ambulatory impairment. Additive CNS depression occurs commonly with drugs used in overthe-counter cold medications, and rebound insomnia can occur with abrupt discontinuance of benzodiazepines used as sleeping pills. Alcohol enhances psychomotor depression and the amnestic effects of the benzodiazepines. The answer is D. 191 6. Decreased blood flow to vital organs, including the liver and kidney, occurs during the aging process. These changes may contribute to cumulative effects of sedative-hypnotic drugs. However, this does not explain the enhanced sensitivity of the elderly patient to a single dose of a central depressant, which appears to be due to changes in brain function that accompany aging. The answer is A. 7. Alprazolam and clonazepam (not listed) are the most effective of the benzodiazepines for the treatment of panic disorders. Eszopiclone and flumazenil are hypnotics. Propranolol is commonly used to attenuate excessive sympathomimetic activity in persons who suffer from performance anxiety (“stage fright”). The answer is A. 8. Clonazepam and phenobarbital are both used in seizure disorders. Chronic administration of phenobarbital (but not clonazepam) increases the activity of hepatic drug-metabolizing enzymes, including several cytochrome P450 isozymes. This can increase the rate of metabolism of drugs administered concomitantly, resulting in decreases in the intensity and duration of their effects. The answer is D. 9. The elimination of most benzodiazepines involves their metabolism by liver enzymes, including cytochrome P450 isozymes. In a patient with liver dysfunction, lorazepam and oxazepam, which are metabolized extrahepatically, are less likely to cause excessive CNS depression. Benzodiazepines are not eliminated via the kidneys or lungs. Flumazenil is used to reverse excessive CNS depression caused by benzodiazepines. The answer is B. 10. Only two of the drugs listed are used for insomnia, eszopiclone and ramelteon. Eszopiclone, zaleplon, and zolpidem are related hypnotics that, though structurally different from benzodiazepines, appear to have a similar mechanism of action. However, unlike benzodiazepines, these drugs are not used in seizures or in muscle spasticity states. Compared with benzodiazepines, the newer hypnotics are less likely to alter sleep patterns. Ramelteon activates melatonin receptors in the suprachiasmatic nuclei. Buspirone is not a hypnotic! The answer is C. SKILL KEEPER ANSWER: LOADING DOSE (SEE CHAPTER 3) Because the half-life of diazepam is 2 days, one may assume that the plasma concentration 3 h after drug ingestion is similar to the peak plasma level. If so, and assuming 100% bioavailability, then Dose ingested = Plasma concentration × Vd = 2 mg/L × 80 L = 160 mg 192 PART V Drugs That Act in the Central Nervous System CHECKLIST When you complete this chapter, you should be able to: ❑ Identify major drugs in each sedative-hypnotic subgroup. ❑ Recall the significant pharmacokinetic features of the sedative-hypnotic drugs commonly used for treatment of anxiety and sleep disorders. ❑ Describe the proposed mechanisms of action of benzodiazepines, barbiturates, and zolpidem. ❑ List the pharmacodynamic actions of major sedative-hypnotics in terms of their clinical uses and their adverse effects. ❑ Identify the distinctive properties of buspirone, eszopiclone, ramelteon, zaleplon, and zolpidem. ❑ Describe the symptoms and management of overdose of sedative-hypnotics and withdrawal from physiologic dependence. DRUG SUMMARY TABLE: Sedative-Hypnotics Subclass Mechanism of Action Clinical Applications Pharmacokinetics and Drug Interactions Bind GABAA receptor subunits to facilitate chloride channel opening and increase frequency • membrane hyperpolarization Acute anxiety states, panic attacks, generalized anxiety disorder, insomnia; skeletal muscle relaxation • seizure disorders Hepatic metabolism • active metabolites. Additive CNS depression with many drugs Half-lives: 2–4 h Extension of CNS depressant actions • tolerance • dependence liability Antagonist at benzodiazepine sites on GABAA receptor Management of benzodiazepine overdose IV form Short half-life Agitation, confusion • possible withdrawal syndrome Bind to GABAA receptor sites (distinct from benzodiazepines) • facilitate chloride channel opening and increase duration Anesthesia (thiopental) • insomnia and sedation (secobarbital) • seizure disorders (phenobarbital) Oral activity • hepatic metabolism; induction of metabolism of many drugs Half-lives: 4–60 h Extension of CNS depressant actions • tolerance • dependence liability > benzodiazepines Bind to GABAA receptor sites (close to benzodiazepine site) • facilitate chloride channel opening Sleep disorders, esp when sleep onset is delayed Oral activity, P450 substrates Additive CNS depression with ethanol and other depressants Short half-lives Extension of CNS depressant effects • dependence liability Toxicities Benzodiazepines Alprazolam Chlordiazepoxide Clorazepate Clonazepam Diazepam Flurazepam Lorazepam Midazolam, etc Benzodiazepine antagonist Flumazenil Barbiturates Amobarbital Butabarbital Pentobarbital Phenobarbital Secobarbital Thiopental Newer hypnotics Eszopiclone Zaleplon Zolpidem (Continued ) CHAPTER 22 Sedative-Hypnotic Drugs 193 DRUG SUMMARY TABLE: Sedative-Hypnotics (Continued ) Subclass Mechanism of Action Clinical Applications Pharmacokinetics and Drug Interactions Activates MT1 and MT2 receptors in suprachiasmatic nucleus Sleep disorders, esp when sleep onset is delayed Not a controlled substance Oral activity; forms active metabolite via CYP1A2 • fluvoxamine inhibits metabolism Dizziness, fatigue, endocrine changes Partial agonist at 5-HT receptors and possibly D2 receptors Generalized anxiety states Oral activity • forms active metabolite • interactions with CYP3A4 inducers and inhibitors; short half-life GI distress, tachycardia • paresthesias Toxicities Melatonin receptor agonist Ramelteon 5-HT agonist Buspirone C H A P T E R 23 Alcohols Ethanol, a sedative-hypnotic drug, is the most important alcohol of pharmacologic interest. It has few medical applications, but its abuse causes major medical and socioeconomic problems. Other alcohols of toxicologic importance are methanol and ethylene glycol. Several important drugs discussed in this chapter are used to prevent the potentially life-threatening ethanol withdrawal syndrome, to treat chronic alcoholism, or to treat acute methanol and ethylene glycol poisoning. Clinically important alcohols and their antagonists Drugs to treat alcohol withdrawal Alcohols Ethanol Thiamine Methanol Sedativehypnotics (diazepam) Ethylene glycol ETHANOL A. Pharmacokinetics After ingestion, ethanol is rapidly and completely absorbed; the drug is then distributed to most body tissues, and its volume of distribution is equivalent to that of total body water (0.5–0.7 L/kg). Two enzyme systems metabolize ethanol to acetaldehyde (Figure 23–1). 1. Alcohol dehydrogenase (ADH)—This family of cytosolic, NAD+-dependent enzymes, found mainly in the liver and gut, accounts for the metabolism of low to moderate doses of ethanol. Because of the limited supply of the coenzyme NAD+, the reaction has zero-order kinetics, resulting in a fixed capacity for ethanol metabolism of 7–10 g/h. Gastrointestinal metabolism of ethanol is lower in women than in men. Genetic variation in ADH affects the rate of ethanol metabolism and vulnerability to alcohol-use disorders. 194 Drugs to treat alcohol dependence Disulfiram Drugs to treat acute methanol or ethylene glycol intoxication Ethanol Fomepizole Naltrexone Acamprosate 2. Microsomal ethanol-oxidizing system (MEOS)—At blood ethanol levels higher than 100 mg/dL, the liver microsomal mixed function oxidase system that catalyzes most phase I drugmetabolizing reactions (see Chapter 2) contributes significantly to ethanol metabolism (Figure 23–1). Chronic ethanol consumption induces cytochrome P450 enzyme synthesis and MEOS activity; this is partially responsible for the development of tolerance to ethanol. The primary isoform of cytochrome P450 induced by ethanol—2E1 (see Table 4–3)—converts acetaminophen to a hepatotoxic metabolite. Acetaldehyde formed from the oxidation of ethanol by either ADH or MEOS is rapidly metabolized to acetate by aldehyde dehydrogenase, a mitochondrial enzyme found in the liver and many other tissues. Aldehyde dehydrogenase is inhibited by disulfiram and other drugs, including metronidazole, oral hypoglycemics, and some cephalosporins. Some individuals, primarily of CHAPTER 23 Alcohols 195 High-Yield Terms to Learn Alcohol abuse An alcohol-use disorder characterized by compulsive use of ethanol in dangerous situations (eg, driving, combined with other CNS depressants) or despite adverse consequences directly related to the drinking Alcohol dependence An alcohol-use disorder characterized by alcohol abuse plus physical dependence on ethanol Alcohol withdrawal syndrome The characteristic syndrome of insomnia, tremor, agitation, seizures, and autonomic instability engendered by deprivation in an individual who is physically dependent on ethanol Delirium tremens (DTs) Severe form of alcohol withdrawal whose main symptoms are sweating, tremor, confusion, and hallucinations Fetal alcohol syndrome A syndrome of craniofacial dysmorphia, heart defects, and mental retardation caused by the teratogenic effects of ethanol consumption during pregnancy Wernicke-Korsakoff syndrome A syndrome of ataxia, confusion, and paralysis of the extraocular muscles that is associated with chronic alcoholism and thiamine deficiency Asian descent, have genetic deficiency of aldehyde dehydrogenase. After consumption of even small quantities of ethanol, these individuals experience nausea and a flushing reaction from accumulation of acetaldehyde. B. Acute Effects 1. CNS—The major acute effects of ethanol on the CNS are sedation, loss of inhibition, impaired judgment, slurred speech, and ataxia. In nontolerant persons, impairment of driving ability is thought to occur at ethanol blood levels between 60 and 80 mg/dL. Blood levels of 120 to 160 mg/dL are usually associated NAD+ Ethanol CH3CH2OH Alcohol dehydrogenase – NADH MEOS Acetaldehyde CH3CHO Fomepizole NADPH + O2 NADP+ + H2O NAD+ Aldehyde dehydrogenase NADH – Acetate CH3COO– Disulfiram FIGURE 23–1 Metabolism of ethanol by alcohol dehydrogenase (ADH) and the microsomal ethanol-oxidizing system (MEOS). Alcohol dehydrogenase and aldehyde dehydrogenase are inhibited by fomepizole and disulfiram, respectively. (Reproduced, with permission, from Katzung BG, Masters SB, Trevor AT, editors: Basic & Clinical Pharmacology, 12th ed. McGraw-Hill, 2012: Fig. 23–1.) with gross drunkenness. Levels greater than 300 mg/dL may lead to loss of consciousness, anesthesia, and coma sometimes with fatal respiratory and cardiovascular depression. Blood levels higher than 500 mg/dL are usually lethal. Individuals with alcohol dependence who are tolerant to the effects of ethanol can function almost normally at much higher blood concentrations than occasional drinkers. Additive CNS depression occurs with concomitant ingestion of ethanol and a wide variety of CNS depressants, including sedative-hypnotics, opioid agonists, and many drugs that block muscarinic and H1 histamine receptors. The molecular mechanisms underlying the complex CNS effects of ethanol are not fully understood. Specific receptors for ethanol have not been identified. Rather, ethanol appears to modulate the function of a number of signaling proteins. It facilitates the action of GABA at GABAA receptors, inhibits the ability of glutamate to activate NMDA (N-methyl-d-aspartate) receptors, and modifies the activities of adenylyl cyclase, phospholipase C, and ion channels. 2. Other organ systems—Ethanol, even at relatively low blood concentrations, significantly depresses the heart. Vascular smooth muscle is relaxed, which leads to vasodilation, sometimes with marked hypothermia. C. Chronic Effects 1. Tolerance and dependence—Tolerance occurs mainly as a result of CNS adaptation and to a lesser extent by an increased rate of ethanol metabolism. There is cross-tolerance to sedativehypnotic drugs that facilitate GABA activity (eg, benzodiazepines and barbiturates). Both psychological and physical dependence are marked. 2. Liver—Liver disease is the most common medical complication of chronic alcohol abuse. Progressive loss of liver function occurs with reversible fatty liver progressing to irreversible hepatitis, cirrhosis, and liver failure. Hepatic dysfunction is often 196 PART V Drugs That Act in the Central Nervous System more severe in women than in men and in both men and women infected with hepatitis B or C virus. 3. Gastrointestinal system—Irritation, inflammation, bleeding, and scarring of the gut wall occur after chronic heavy use of ethanol and may cause absorption defects and exacerbate nutritional deficiencies. Chronic alcohol abuse greatly increases the risk of pancreatitis. 4. CNS—Peripheral neuropathy is the most common neurologic abnormality in alcohol abuse. More rarely, thiamine deficiency, along with alcohol abuse, leads to Wernicke-Korsakoff syndrome, which is characterized by ataxia, confusion, and paralysis of the extraocular muscles. Prompt treatment with parenteral thiamine is essential to prevent a permanent memory disorder known as Korsakoff’s psychosis. 5. Endocrine system—Gynecomastia, testicular atrophy, and salt retention can occur, partly because of altered steroid metabolism in the cirrhotic liver. 6. Cardiovascular system—Excessive chronic ethanol use is associated with an increased incidence of hypertension, anemia, and dilated cardiomyopathy. Acute drinking for several days (“binge” drinking) can cause arrhythmias. However, the ingestion of modest quantities of ethanol (10–15 g/day) raises serum levels of high-density lipoprotein (HDL) cholesterol and may protect against coronary heart disease. 7. Fetal alcohol syndrome—Ethanol use in pregnancy is associated with teratogenic effects that include mental retardation (most common), growth deficiencies, microcephaly, and a characteristic underdevelopment of the midface region. 8. Neoplasia—Ethanol is not a primary carcinogen, but its chronic use is associated with an increased incidence of neoplastic diseases in the gastrointestinal tract and a small increase in the risk of breast cancer. 9. Immune system—Chronic alcohol abuse has complex effects on immune functions because it enhances inflammation in the liver and pancreas and inhibits immune function in other tissues. Heavy use predisposes to infectious pneumonia. SKILL KEEPER: ELIMINATION HALF LIFE (SEE CHAPTER 1) Search “high and low” through drug information resources and you will not find data on the elimination half-life of ethanol! Can you explain why this is the case? The Skill Keeper Answer appears at the end of the chapter. D. Treatment of Acute and Chronic Alcoholism 1. Excessive CNS depression—Acute ethanol intoxication is managed by maintenance of vital signs and prevention of aspiration after vomiting. Intravenous dextrose is standard. Thiamine administration is used to protect against Wernicke-Korsakoff syndrome, and correction of electrolyte imbalance may be required. 2. Alcohol withdrawal syndrome—In individuals physically dependent on ethanol, discontinuance can lead to a withdrawal syndrome characterized by insomnia, tremor, anxiety, and, in severe cases, life-threatening seizures and delirium tremens (DTs). Peripheral effects include nausea, vomiting, diarrhea, and arrhythmias. The withdrawal syndrome is managed by correction of electrolyte imbalance and administration of thiamine and a sedative-hypnotic. A long-acting benzodiazepine (eg, diazepam, chlordiazepoxide) is preferred unless the patient has compromised liver function, in which case a short-acting benzodiazepine with less complex metabolism (eg, lorazepam) is preferred. 3. Treatment of alcoholism—Alcoholism is a complex sociomedical problem, characterized by a high relapse rate. Several CNS neurotransmitter systems appear to be targets for drugs that reduce the craving for alcohol. The opioid receptor antagonist naltrexone has proved to be useful in some patients, presumably through its ability to decrease the effects of endogenous opioid peptides in the brain (see Chapters 31 and 32). Acamprosate, an NMDA glutamate receptor antagonist, is also FDA approved for treatment of alcoholism. The aldehyde dehydrogenase inhibitor disulfiram is used adjunctively in some treatment programs. If ethanol is consumed by a patient who has taken disulfiram, acetaldehyde accumulation leads to nausea, headache, flushing, and hypotension (Figure 23–1). OTHER ALCOHOLS A. Methanol Methanol (wood alcohol), a constituent of windshield cleaners and “canned heat,” is sometimes ingested intentionally. Intoxication causes visual dysfunction, gastrointestinal distress, shortness of breath, loss of consciousness, and coma. Methanol is metabolized to formaldehyde and formic acid, which causes severe acidosis, retinal damage, and blindness. The formation of formaldehyde is reduced by prompt intravenous administration of fomepizole, an inhibitor of alcohol dehydrogenase, or ethanol, which competitively inhibits alcohol dehydrogenase oxidation of methanol (Figure 23–2). B. Ethylene Glycol Industrial exposure to ethylene glycol (by inhalation or skin absorption) or self-administration (eg, by drinking antifreeze CHAPTER 23 Alcohols Fomepizole – Ethylene glycol Oxalic acid Acidosis, nephrotoxicity Alcohol dehydrogenase Formaldehyde, formic acid Methanol Severe acidosis, retinal damage – Ethanol Aldehyde FIGURE 23–2 The oxidation of ethylene glycol and methanol by alcohol dehydrogenase (ADH) creates metabolites that cause serious toxicity. Fomepizole, an inhibitor of alcohol dehydrogenase, is used in methanol or ethylene glycol poisoning to slow the rate of formation of toxic metabolites. Ethanol, a substrate with higher affinity for ADH than ethylene glycol or methanol, also slows the formation of toxic metabolites and is an alternative to fomepizole. products) leads to severe acidosis and renal damage from the metabolism of ethylene glycol to oxalic acid. Prompt treatment with intravenous fomepizole or ethanol may slow or prevent formation of this toxic metabolite (Figure 23–2). QUESTIONS 1. A 45-year-old moderately obese man has been drinking heavily for 72 h. This level of drinking is much higher than his regular habit of drinking 1 alcoholic drink per day. His only significant medical problem is mild hypertension, which is adequately controlled by metoprolol. With this history, this man is at significant risk for (A) Bacterial pneumonia (B) Cardiac arrhythmias (C) Hyperthermia (D) Tonic-clonic seizures (E) Wernicke-Korsakoff syndrome 2. A 42-year-old man with a history of alcoholism is brought to the emergency department in a confused and delirious state. He has truncal ataxia and ophthalmoplegia. The most appropriate immediate course of action is to administer diazepam plus (A) Chlordiazepoxide (B) Disulfiram (C) Folic acid (D) Glucosamine (E) Thiamine 197 3. The cytochrome P450-dependent microsomal ethanol oxidizing system (MEOS) pathway of ethanol metabolism is most likely to be maximally activated under the condition of low concentrations of (A) Acetaldehyde (B) Ethanol (C) NAD+ (D) NADPH (E) Oxygen 4. A freshman student (weight 70 kg) attends a college party where he rapidly consumes a quantity of an alcoholic beverage that results in a blood level of 500 mg/dL. Assuming that this young man has not had an opportunity to develop tolerance to ethanol, his present condition is best characterized as (A) Able to walk, but not in a straight line (B) Alert and competent to drive a car (C) Comatose and near death (D) Sedated with increased reaction times (E) Slightly inebriated Questions 5 and 6. A homeless middle-aged male patient presents in the emergency department in a state of intoxication. You note that he is behaviorally disinhibited and rowdy. He tells you that he has recently consumed about a pint of a red-colored liquid that his friends were using to “get high.” He complains that his vision is blurred and that it is “like being in a snowstorm.” His breath smells a bit like formaldehyde. He is acidotic. 5. Which of the following is the most likely cause of this patient’s intoxicated state? (A) Ethanol (B) Ethylene glycol (C) Isopropanol (D) Hexane (E) Methanol 6. After assessing and stabilizing the patient’s airway, respiration, and circulatory status, fomepizole was administered intravenously. Which of the following most accurately describes the therapeutic purpose of the fomepizole administration? (A) Accelerate the rate of elimination of the toxic liquid that he consumed (B) Combat acidosis (C) Inhibit the metabolic production of toxic metabolites (D) Prevent alcohol withdrawal seizures (E) Sedate the patient 7. The regular ingestion of moderate or heavy amounts of alcohol predisposes to hepatic damage after overdose of acetaminophen because chronic ethanol ingestion (A) Blocks acetaminophen metabolism (B) Causes thiamine deficiency (C) Displaces acetaminophen from plasma proteins (D) Induces hepatic drug-metabolizing enzymes (E) Inhibits renal clearance of acetaminophen 198 PART V Drugs That Act in the Central Nervous System 8. A 23-year-old pregnant woman with alcoholism presented to the emergency department in the early stages of labor. She had consumed large amounts of alcohol throughout her pregnancy. This patient’s infant is at high risk of a syndrome that includes (A) Ambiguous genitalia in a male fetus and normal genitalia in a female fetus (B) Failure of closure of the atrial septum or ventricular septum (C) Limb or digit malformation (D) Mental retardation and craniofacial abnormalities (E) Underdevelopment of the lungs 9. The combination of ethanol and disulfiram results in nausea and hypotension as a result of the accumulation of (A) Acetaldehyde (B) Acetate (C) Methanol (D) NADH (E) Pyruvate 10. The intense craving experienced by those who are trying to recover from chronic alcohol abuse can be ameliorated by a drug that is an (A) Agonist of α1 adrenoceptors (B) Agonist of serotonin receptors (C) Antagonist of β2 adrenoceptors (D) Antagonist of opioid receptors (E) Inhibitor of cyclooxygenase ANSWERS 1. This man’s regular rate of alcohol consumption is not high enough to put him at risk of long-term consequences such as Wernicke-Korsakoff syndrome, increased susceptibility to bacterial pneumonia, or alcohol withdrawal seizures. This pattern of “binge drinking” does put him at increased risk of cardiac arrhythmia. The answer is B. 2. This patient has symptoms of Wernicke’s encephalopathy, including delirium, gait disturbances, and paralysis of the external eye muscles. The condition results from thiamine deficiency but is rarely seen in the absence of alcoholism. The diazepam is administered to prevent the alcohol withdrawal syndrome. Glucosamine is primarily used for pain associated with arthritis. The answer is E. 3. The microsomal ethanol-oxidizing system (MEOS) contributes most to ethanol metabolism at relatively high blood alcohol concentrations (>100 mg/dL), when the alcohol dehydrogenase pathway is saturated due to depletion of NAD+. So, the MEOS system contributes most when the NAD+ concentration is low. NADPH and oxygen are cofactors for MEOS reactions. The concentration of acetaldehyde does not appear to affect the rate of either the ADH or the MEOS reactions. The answer is C. 4. The blood level of ethanol achieved in this individual is extremely high and likely to result in coma and possibly death due to respiratory arrest in a person who lacks tolerance to ethanol. The answer is C. 5. Behavioral disinhibition is a feature of early intoxication from ethanol and most other alcohols but not the solvent, hexane. Ocular dysfunction, including horizontal nystagmus and diplopia, is also a common finding in poisoning with alcohols, but the complaint of “flickering white spots before the eyes” or “being in a snowstorm” is highly suggestive of methanol intoxication. In some cases, the odor of formaldehyde may be present on the breath. In this patient, blood methanol levels should be determined as soon as possible. The answer is E. 6. In patients with suspected methanol intoxication, fomepizole is given intravenously to inhibit the ADH-catalyzed formation of toxic metabolites. The answer is C. 7. Chronic use of ethanol induces a CYP2E1 isozyme that converts acetaminophen to a cytotoxic metabolite. This appears to be the explanation for the increased susceptibility to acetaminophen-induced hepatotoxicity found in individuals who regularly ingest alcohol. The answer is D. 8. This woman’s infant is at risk for fetal alcohol syndrome, a syndrome associated with mental retardation, abnormalities of the head and face, and growth deficiency. This syndrome is a leading cause of mental retardation. The answer is D. 9. The nausea, hypotension, and ill feeling that result from drinking ethanol while also taking disulfiram stems from acetaldehyde accumulation. Disulfiram inhibits acetaldehyde dehydrogenase, the enzyme that converts acetaldehyde to acetate. The answer is A. 10. Naltrexone, a competitive inhibitor of opioid receptors, decreases the craving for alcohol in patients who are recovering from alcoholism. The answer is D. SKILL KEEPER ANSWER: ELIMINATION HALF-LIFE (SEE CHAPTER 1) Drug information resources do not provide data on the elimination half-life of ethanol because, in the case of this drug, it is not constant. Ethanol elimination follows zero-order kinetics because the drug is metabolized at a constant rate irrespective of its concentration in the blood (see Chapter 3). The pharmacokinetic relationship between elimination halflife, volume of distribution, and clearance, given by t1/2 = 0.693 × Vd CL is not applicable to ethanol. Its rate of metabolism is constant, but its clearance decreases with an increase in blood level. The arithmetic plot of ethanol blood level versus time follows a straight line (not exponential decay). CHAPTER 23 Alcohols 199 CHECKLIST When you complete this chapter, you should be able to: ❑ Sketch the biochemical pathways for ethanol metabolism and indicate where fomepizole and disulfiram act. ❑ Summarize characteristic pharmacodynamic and pharmacokinetic properties of ethanol. ❑ Relate blood alcohol levels in a nontolerant person to CNS depressant effects of acute alcohol ingestion. ❑ Identify the toxic effects of chronic ethanol ingestion. ❑ Describe the fetal alcohol syndrome. ❑ Describe the treatment of ethanol overdosage. ❑ Outline the pharmacotherapy of (1) the alcohol withdrawal syndrome and (2) alcohol-use disorders. ❑ Describe the toxicity and treatment of acute poisoning with (1) methanol and (2) ethylene glycol. DRUG SUMMARY TABLE: Alcohols Subclass Mechanism of Action Clinical Applications Pharmacokinetics Toxicities, Interactions Multiple effects on neurotransmitter receptors, ion channels, and signaling pathways Antidote in methanol and ethylene glycol poisoning Zero-order metabolism, duration depends on dose Toxicity: Acute, CNS depression and respiratory failure. Chronic, damage to many systems, including liver, pancreas, gastrointestinal tract, and central and peripheral nervous systems. Interactions: Induction of CYP2E1 • increased conversion of acetaminophen to toxic metabolite Alcohols Ethanol Methanol: poisoning result in toxic levels of formate, which causes characteristic visual disturbance plus coma, seizures, acidosis, and death due to respiratory failure Ethylene glycol: poisoning creates toxic aldehydes and oxalate, which causes kidney damage and severe acidosis Drugs used in acute ethanol withdrawal Diazepam BDZ receptor agonist that facilitates GABA-mediated activation of GABAA receptors Prevention and treatment of acute ethanol withdrawal syndrome • see Chapter 22 See Chapter 22 See Chapter 22 Other long-acting benzodiazepines and barbiturates are also effective (see Chapter 22) Thiamine (vitamin B1) Essential vitamin required for synthesis of the coenzyme thiamine pyrophosphate Administered to patients suspected of alcohol dependence to prevent the Wernicke-Korsakoff syndrome Parenteral administration None (Continued ) 200 PART V Drugs That Act in the Central Nervous System DRUG SUMMARY TABLE: Alcohols (Continued ) Subclass Mechanism of Action Clinical Applications Pharmacokinetics Toxicities, Interactions Drugs used in chronic alcoholism Naltrexone Nonselective competitive antagonist of opioid receptors Reduced risk of relapse in individuals with alcoholuse disorders Available as an oral or long-acting parenteral formulation (see Chapters 31 and 32) Gastrointestinal effects and liver toxicity • rapid antagonism of all opioid actions Acamprosate Poorly understood NMDA receptor antagonist and GABAA agonist effects Reduced risk of relapse in individuals with alcoholuse disorders Oral administration Gastrointestinal effects and rash Disulfiram Inhibits aldehyde dehydrogenase • causes aldehyde accumulation during ethanol ingestion Deterrent to relapse in individuals with alcoholuse disorders Oral administration Little effect on its own but severe flushing, headache, nausea, vomiting, and hypotension when combined with ethanol Parenteral administration Headache, nausea, dizziness, rare allergic reactions Drugs used in acute methanol or ethylene glycol toxicity Fomepizole Inhibits alcohol dehydrogenase • prevents conversion of methanol and ethylene glycol to toxic metabolites Methanol and ethylene glycol poisoning Ethanol: higher affinity for alcohol dehydrogenase; used to reduce metabolism to toxic products C H A P T E R 24 Antiseizure Drugs Epilepsy comprises a group of chronic syndromes that involve the recurrence of seizures (ie, limited periods of abnormal discharge of cerebral neurons). Effective antiseizure drugs have, to varying degrees, selective depressant actions on such abnormal neuronal activity. However, they vary in terms of their mechanisms of action and in their effectiveness in specific seizure disorders. Antiseizure drugs Tonic-clonic & partial seizures Absence seizures Myoclonic seizures Carbamazepine Lamotrigine Phenytoin Valproic acid Clonazepam Ethosuximide Valproic acid Clonazepam Lamotrigine Valproic acid PHARMACOKINETICS Antiseizure drugs are commonly used for long periods of time, and consideration of their pharmacokinetic properties is important for avoiding toxicity and drug interactions. For some of these drugs (eg, phenytoin), determination of plasma levels and clearance in individual patients may be necessary for optimum therapy. In general, antiseizure drugs are well absorbed orally and have good bioavailability. Most antiseizure drugs are metabolized by hepatic enzymes (exceptions include gabapentin and vigabatrin), and in some cases active metabolites are formed. Resistance to antiseizure drugs may involve increased expression of drug transporters at the level of the blood-brain barrier. Pharmacokinetic drug interactions are common in this drug group. In the presence of drugs that inhibit antiseizure drug metabolism or displace anticonvulsants from plasma protein binding sites, plasma concentrations of the antiseizure agents may Back-up & adjunctive drugs Felbamate Gabapentin Lamotrigine Levetiracetam Phenobarbital Tiagabine Topiramate Vigabatrin Zonisamide reach toxic levels. On the other hand, drugs that induce hepatic drug-metabolizing enzymes (eg, rifampin) may result in plasma levels of the antiseizure agents that are inadequate for seizure control. Several antiseizure drugs are themselves capable of inducing hepatic drug metabolism, especially carbamazepine and phenytoin. A. Phenytoin The oral bioavailability of phenytoin is variable because of individual differences in first-pass metabolism. Rapid-onset and extended-release forms are available. Phenytoin metabolism is nonlinear; elimination kinetics shift from first-order to zeroorder at moderate to high dose levels. The drug binds extensively to plasma proteins (97–98%), and free (unbound) phenytoin levels in plasma are increased transiently by drugs that compete for binding (eg, carbamazepine, sulfonamides, valproic acid). The metabolism of phenytoin is enhanced in the presence of 201 202 PART V Drugs That Act in the Central Nervous System High-Yield Terms to Learn Seizures Finite episodes of brain dysfunction resulting from abnormal discharge of cerebral neurons Partial seizures, simple Consciousness preserved; manifested variously as convulsive jerking, paresthesias, psychic symptoms (altered sensory perception, illusions, hallucinations, affect changes), and autonomic dysfunction Partial seizures, complex Impaired consciousness that is preceded, accompanied, or followed by psychological symptoms Tonic-clonic seizures, generalized Tonic phase (less than 1 min) involves abrupt loss of consciousness, muscle rigidity, and respiration arrest; clonic phase (2–3 min) involves jerking of body muscles, with lip or tongue biting, and fecal and urinary incontinence; formerly called grand mal Absence seizures, generalized Impaired consciousness (often abrupt onset and brief), sometimes with automatisms, loss of postural tone, or enuresis; begin in childhood (formerly, petit mal) and usually cease by age 20 yrs Myoclonic seizures Single or multiple myoclonic muscle jerks Status epilepticus A series of seizures (usually tonic-clonic) without recovery of consciousness between attacks; it is a life-threatening emergency inducers of liver metabolism (eg, phenobarbital, rifampin) and inhibited by other drugs (eg, cimetidine, isoniazid). Phenytoin itself induces hepatic drug metabolism, decreasing the effects of other antiepileptic drugs including carbamazepine, clonazepam, and lamotrigine. Fosphenytoin is a water-soluble prodrug form of phenytoin that is used parenterally. B. Carbamazepine Carbamazepine induces formation of liver drug-metabolizing enzymes that increase metabolism of the drug itself and may increase the clearance of many other anticonvulsant drugs including clonazepam, lamotrigine, and valproic acid. Carbamazepine metabolism can be inhibited by other drugs (eg, propoxyphene, valproic acid). A related drug, oxcarbazepine, is less likely to be involved in drug interactions. C. Valproic Acid In addition to competing for phenytoin plasma protein binding sites, valproic acid inhibits the metabolism of carbamazepine, ethosuximide, phenytoin, phenobarbital, and lamotrigine. Hepatic biotransformation of valproic acid leads to formation of a toxic metabolite that has been implicated in the hepatotoxicity of the drug. D. Other Drugs Gabapentin, pregabalin, levetiracetam, and vigabatrin are unusual in that they are eliminated by the kidney, largely in unchanged form. These agents have virtually no drug-drug interactions. Tiagabine, topiramate, and zonisamide undergo both hepatic metabolism and renal elimination of intact drug. Lamotrigine is eliminated via hepatic glucuronidation. MECHANISMS OF ACTION The general effect of antiseizure drugs is to suppress repetitive action potentials in epileptic foci in the brain. Many different mechanisms are involved in achieving this effect. In some cases, several mechanisms may contribute to the antiseizure activity of an individual drug. Some of the recognized mechanisms are described next. A. Sodium Channel Blockade At therapeutic concentrations, phenytoin, carbamazepine, lamotrigine, and zonisamide block voltage-gated sodium channels in neuronal membranes. This action is rate-dependent (ie, dependent on the frequency of neuronal discharge) and results in prolongation of the inactivated state of the Na+ channel and the refractory period of the neuron. Phenobarbital and valproic acid may exert similar effects at high doses. B. GABA-Related Targets As described in Chapter 22, benzodiazepines interact with specific receptors on the GABAA receptor–chloride ion channel macromolecular complex. In the presence of benzodiazepines, the frequency of chloride ion channel opening is increased; these drugs facilitate the inhibitory effects of GABA. Phenobarbital and other barbiturates also enhance the inhibitory actions of GABA but interact with a different receptor site on chloride ion channels that results in an increased duration of chloride ion channel opening. GABA aminotransaminase (GABA-T) is an important enzyme in the termination of action of GABA. The enzyme is irreversibly inactivated by vigabatrin at therapeutic plasma levels and can also be inhibited by valproic acid at very high concentrations. Tiagabine inhibits a GABA transporter (GAT-1) in neurons and glia prolonging the action of the neurotransmitter. Gabapentin is a structural analog of GABA, but it does not activate GABA receptors directly. Other drugs that may facilitate the inhibitory actions of GABA include felbamate, topiramate, and valproic acid. C. Calcium Channel Blockade Ethosuximide inhibits low-threshold (T type) Ca2+ currents, especially in thalamic neurons that act as pacemakers to generate rhythmic cortical discharge. A similar action is reported for CHAPTER 24 Antiseizure Drugs valproic acid, as well as for both gabapentin and pregabalin, and it may be the primary action of the latter drugs. D. Other Mechanisms In addition to its action on calcium channels, valproic acid causes neuronal membrane hyperpolarization, possibly by enhancing K+ channel permeability. Although phenobarbital acts on both sodium channels and GABA-chloride channels, it also acts as an antagonist at some glutamate receptors. Felbamate blocks glutamate NMDA receptors. Topiramate blocks sodium channels and potentiates the actions of GABA and may also block glutamate receptors. SKILL KEEPER: ANTIARRHYTHMIC DRUG ACTIONS (SEE CHAPTER 14) 1. Which of the mechanisms of action of antiseizure drugs have theoretical implications regarding their activity in cardiac arrhythmias? 2. Recall any clinical uses of antiseizure drugs in the management of cardiac arrhythmias? The Skill Keeper Answers appear at the end of the chapter. CLINICAL USES Diagnosis of a specific seizure type is important for prescribing the most appropriate antiseizure drug (or combination of drugs). Drug choice is usually made on the basis of established efficacy in the specific seizure state that has been diagnosed, the prior responsiveness of the patient, and the anticipated toxicity of the drug. Treatment may involve combinations of drugs, following the principle of adding known effective agents if the preceding drugs are not sufficient. A. Generalized Tonic-Clonic Seizures Valproic acid, carbamazepine, and phenytoin are the drugs of choice for generalized tonic-clonic (grand mal) seizures. Phenobarbital (or primidone) is now considered to be an alternative agent in adults but continues to be a primary drug in infants. Lamotrigine and topiramate are also approved drugs for this indication, and several others may be used adjunctively in refractory cases. B. Partial Seizures The drugs of first choice are carbamazepine (or oxcarbazepine) or lamotrigine or phenytoin. Alternatives include felbamate, phenolbarbital, topiramate, and valproic acid. Many of the newer anticonvulsants can be used adjunctively, including gabapentin and pregabalin, a structural congener. C. Absence Seizures Ethosuximide or valproic acid are the preferred drugs because they cause minimal sedation. Ethosuximide is often used in uncomplicated 203 absence seizures if patients can tolerate its gastrointestinal side effects. Valproic acid is particularly useful in patients who have concomitant generalized tonic-clonic or myoclonic seizures. Clonazepam is effective as an alternative drug but has the disadvantages of causing sedation and tolerance. Lamotrigine, levetiracetam, and zonisamide are also effective in absence seizures. D. Myoclonic and Atypical Absence Syndromes Myoclonic seizure syndromes are usually treated with valproic acid; lamotrigine is approved for adjunctive use, but is commonly used as monotherapy. Clonazepam can be effective, but the high doses required cause drowsiness. Levetiracetam, topiramate, and zonisamide are also used as backup drugs in myoclonic syndromes. Felbamate has been used adjunctively with the primary drugs but has both hematotoxic and hepatotoxic potential. E. Status Epilepticus Intravenous diazepam or lorazepam is usually effective in terminating attacks and providing short-term control. For prolonged therapy, intravenous phenytoin has often been used because it is highly effective and less sedating than benzodiazepines or barbiturates. However, phenytoin may cause cardiotoxicity (perhaps because of its solvent, propylene glycol), and fosphenytoin (watersoluble) is a safer parenteral agent. Phenobarbital has also been used in status epilepticus, especially in children. In very severe status epilepticus that does not respond to these measures, general anesthesia may be used. F. Other Clinical Uses Several antiseizure drugs are effective in the management of bipolar affective disorders, especially valproic acid, which is now often used as a first-line drug in the treatment of mania. Carbamazepine and lamotrigine have also been used successfully in bipolar disorder. Carbamazepine is the drug of choice for trigeminal neuralgia, and its congener oxcarbazepine may provide similar analgesia with fewer adverse effects. Gabapentin has efficacy in pain of neuropathic origin, including postherpetic neuralgia, and, like phenytoin, may have some value in migraine. Topiramate is also used in the treatment of migraine. Pregabalin is also approved for neuropathic pain. TOXICITY Chronic therapy with antiseizure drugs is associated with specific toxic effects, the most important of which are listed in Table 24–1. A. Teratogenicity Children born of mothers taking anticonvulsant drugs have an increased risk of congenital malformations. Neural tube defects (eg, spina bifida) are associated with the use of valproic acid; carbamazepine has been implicated as a cause of craniofacial anomalies and spina bifida; and a fetal hydantoin syndrome has been described after phenytoin use by pregnant women. 204 PART V Drugs That Act in the Central Nervous System TABLE 24–1 Adverse effects and complications of antiepileptic drugs. Antiepileptic Drug Adverse Effects Benzodiazepines Sedation, tolerance, dependence Carbamazepine Diplopia, cognitive dysfunction, drowsiness, ataxia; rare occurrence of severe blood dyscrasias and Stevens-Johnson syndrome; induces hepatic drug metabolism; teratogenic potential Ethosuximide Gastrointestinal distress, lethargy, headache, behavioral changes Felbamate Aplastic anemia, hepatic failure Gabapentin Dizziness, sedation, ataxia, nystagmus; does not affect drug metabolism (pregabalin is similar) Lamotrigine Dizziness, ataxia, nausea, rash, rare Stevens-Johnson syndrome Levetiracetam Dizziness, sedation, weakness, irritability, hallucinations, and psychosis have occurred Oxcarbazepine Similar to carbamazepine, but hyponatremia is more common; unlike carbamazepine, does not induce drug metabolism Phenobarbital Sedation, cognitive dysfunction, tolerance, dependence, induction of hepatic drug metabolism; primidone is similar Phenytoin Nystagmus, diplopia, sedation, gingival hyperplasia, hirsutism, anemias, peripheral neuropathy, osteoporosis, induction of hepatic drug metabolism Tiagabine Abdominal pain, nausea, dizziness, tremor, asthenia; drug metabolism is not induced Topiramate Drowsiness, dizziness, ataxia, psychomotor slowing and memory impairment; paresthesias, weight loss, acute myopia Valproic acid Drowsiness, nausea, tremor, hair loss, weight gain, hepatotoxicity (infants), inhibition of hepatic drug metabolism Vigabatrin Sedation, dizziness, weight gain; visual field defects with long-term use, which may not be reversible Zonisamide Dizziness, confusion, agitation, diarrhea, weight loss, rash, Stevens-Johnson syndrome B. Overdosage Toxicity Most of the commonly used anticonvulsants are CNS depressants, and respiratory depression may occur with overdosage. Management is primarily supportive (airway management, mechanical ventilation), and flumazenil may be used in benzodiazepine overdose. C. Life-Threatening Toxicity Fatal hepatotoxicity has occurred with valproic acid, with greatest risk to children younger than 2 yr and patients taking multiple anticonvulsant drugs. Lamotrigine has caused skin rashes and lifethreatening Stevens-Johnson syndrome or toxic epidermal necrolysis. Children are at higher risk (1–2% incidence), especially if they are also taking valproic acid. Zonisamide may also cause severe skin reactions. Reports of aplastic anemia and acute hepatic failure have limited the use of felbamate to severe, refractory seizure states. D. Withdrawal Withdrawal from antiseizure drugs should be accomplished gradually to avoid increased seizure frequency and severity. In general, withdrawal from anti-absence drugs is more easily accomplished than withdrawal from drugs used in partial or generalized tonicclonic seizure states. QUESTIONS 1. A 9-year-old child is having learning difficulties at school. He has brief lapses of awareness with eyelid fluttering that occur every 5–10 min. Electroencephalogram (EEG) studies reveal brief 3-Hz spike and wave discharges appearing synchronously in all leads. Which drug would be effective in this child without the disadvantages of excessive sedation or tolerance development? (A) Clonazepam (B) Diazepam (C) Ethosuximide (D) Gabapentin (E) Phenobarbital 2. Which statement concerning the proposed mechanisms of action of anticonvulsant drugs is inaccurate? (A) Benzodiazepines facilitate GABA-mediated inhibitory actions (B) Ethosuximide selectively blocks potassium ion (K+) channels in thalamic neurons (C) Phenobarbital has multiple actions, including enhancement of the effects of GABA, antagonism of glutamate receptors, and blockade of sodium ion (Na+) channels (D) Phenytoin prolongs the inactivated state of the Na+ channel (E) Zonisamide blocks voltage-gated Na+ channels CHAPTER 24 Antiseizure Drugs 205 3. Which drug used in management of seizure disorders is most likely to elevate the plasma concentration of other drugs administered concomitantly? (A) Carbamazepine (B) Clonazepam (C) Phenobarbital (D) Phenytoin (E) Valproic acid 9. Which statement about phenytoin is accurate? (A) Displaces sulfonamides from plasma proteins (B) Drug of choice in myoclonic seizures (C) Half-life is increased if used with phenobarbital (D) Isoniazid (INH) decreases steady-state blood levels of phenytoin (E) Toxic effects may occur with only small increments in dose 4. A young female patient suffers from absence seizures. Which of the following statements about her proposed drug management is NOT accurate? (A) Ethosuximide and valproic acid are preferred drugs (B) Gastrointestinal side effects are common with ethosuximide (C) The patient should be examined every 2 or 3 mo for deep tendon reflex activity (D) The use of valproic acid in pregnancy may cause congenital malformations (E) Weight gain is common in patients on valproic acid 10. A young male patient suffers from a seizure disorder characterized by tonic rigidity of the extremities followed in 15–30 s of tremor progressing to massive jerking of the body. This clonic phase lasts for 1 or 2 min, leaving the patient in a stuporous state. Of the following drugs, which is most suitable for longterm management of this patient? (A) Clonazepam (B) Ethosuximide (C) Felbamate (D) Phenytoin (E) Pregabalin 5. Which statement concerning the pharmacokinetics of antiseizure drugs is accurate? (A) Administration of phenytoin to patients in methadone maintenance programs has led to symptoms of opioid overdose, including respiratory depression (B) To reduce gastrointestinal toxicity, ethosuximide is usually taken twice a day (C) At high doses, phenytoin elimination follows first-order kinetics (D) The administration of phenytoin to patients in methadone maintenance programs has led to symptoms of opioid overdose, including respiratory depression (E) Treatment with vigabatrin reduces the effectiveness of oral contraceptives (F) Valproic acid may increase the activity of hepatic ALA synthase and the synthesis of porphyrins 6. With chronic use in seizure states, the adverse effects of this drug include coarsening of facial features, hirsutism, and gingival hyperplasia. (A) Carbamazepine (B) Ethosuximide (C) Phenytoin (D) Tiagabine (E) Zonisamide 7. Abrupt withdrawal of antiseizure drugs can result in increases in seizure frequency and severity. Withdrawal is most easily accomplished if the patient is treated with (A) Carbamazepine (B) Clonazepam (C) Ethosuximide (D) Phenobarbital (E) Phenytoin 8. The mechanism of antiseizure activity of carbamazepine is (A) Block of sodium ion channels (B) Block of calcium ion channels (C) Facilitation of GABA actions on chloride ion channels (D) Glutamate receptor antagonism (E) Inhibition of GABA transaminase ANSWERS 1. This child suffers from absence seizures, and 2 of the drugs listed are effective in this seizure disorder. Clonazepam is effective but exerts troublesome CNS-depressant effects, and tolerance develops with chronic use. Ethosuximide is not excessively sedating, and tolerance does not develop to its antiseizure activity. Valproic acid (not listed) is also used in absence seizures. The answer is C. 2. The mechanism of action of phenylsuccinimides such as ethosuximide involves blockade of T-type Ca2+ channels in thalamic neurons. Ethosuximide does not block K+ channels, which in any case would be likely to result in an increase (rather than a decrease) in neuronal excitability. The answer is B. 3. With chronic use, carbamazepine, phenobarbital, and phenytoin can induce the synthesis of hepatic drug-metabolizing enzymes. This action may lead to a decrease in the plasma concentration of other drugs used concomitantly. Valproic acid, an inhibitor of drug metabolism, can increase the plasma levels of many drugs, including those used in seizure disorders such as carbamazepine, lamotrigine, phenobarbital, and phenytoin. Benzodiazepines (including clonazepam and diazepam) as well as gabapentin and vigabatrin have no major effects on the metabolism of other drugs. The answer is E. 4. Ethosuximide and valproic acid are preferred drugs in absence seizures because they cause minimal sedation. However, valproic acid causes gastrointestinal distress and weight gain and is potentially hepatotoxic. In addition, its use in pregnancy has been associated with teratogenicity (neural tube defects). Peripheral neuropathy, including diminished deep tendon reflexes in the lower extremities, occurs with the chronic use of phenytoin, not valproic acid. The answer is C. 206 PART V Drugs That Act in the Central Nervous System 5. The enzyme-inducing activity of phenytoin has led to symptoms of opioid withdrawal, presumably because of an increase in the rate of metabolism of methadone. Monitoring of plasma concentration of phenytoin may be critical is establishing and effective dosage because of nonlinear elimination kinetics at high doses. Valproic acid has no effect on porphyrin synthesis. Vigabatrin does not affect the metabolism of oral contraceptives. Twice-daily dosage of ethosuximide reduces the severity of adverse gastrointestinal effects. The answer is B. 6. Common adverse effects of phenytoin include nystagmus, diplopia, and ataxia. With chronic use, abnormalities of vitamin D metabolism, coarsening of facial features, gingival overgrowth and hirsutism may also occur. A major adverse effect of tiagabine and zonisamide is CNS depression. The answer is C. 7. Dose tapering is an important principle in antiseizure drug withdrawal. As a rule, withdrawal from drugs used for absence seizures such as ethosuximide is easier than withdrawal from drugs used for partial and tonic-clonic seizures. Withdrawal is most difficult in patients who have been treated with barbiturates and benzodiazepines. The answer is C. 8. The mechanism of action of carbamazepine is similar to that of phenytoin, blocking sodium ion channels. Ethosuximide blocks calcium channels; benzodiazepines and barbiturates facilitate the inhibitory actions of GABA; topiramate may block glutamate receptors; and vigabatrin inhibits GABA metabolism. The answer is A. 9. Sulfonamides can displace phenytoin from its binding sites, increasing the plasma-free fraction of the drug. Induction of liver drug-metabolizing enzymes by phenobarbital results in a decreased half-life of phenytoin, and isoniazid increases plasma levels of phenytoin by inhibiting its metabolism. Because of the dose-dependent elimination kinetics of phenytoin, some toxicity may occur with only small increments in dose. The answer is E. 10. This patient is suffering from generalized tonic-clonic seizures. For many years, the drugs of choice in this seizure disorder have been carbamazepine or phenytoin or valproic acid. However, many newer drugs are also effective, including gabapentin, lamotrigine, levetiracetem, topiramate, and zonisamide. Clonazepam and ethosuximide are not effective in this type of seizure disorder. Pregabalin is approved for use only in partial seizures. The answer is D. SKILL KEEPER ANSWERS: ANTIARRHYTHMIC DRUG ACTIONS (SEE CHAPTER 14) 1. Close similarities of structure and function exist between voltage-gated sodium channels in neurons and in cardiac cells. Drugs that exert antiseizure actions via their blockade of sodium channels in the CNS have the potential for a similar action in the heart. Delayed recovery of sodium channels from their inactivated state subsequently slows the rising phase of the action potential in Na+-dependent fibers and is characteristic of group I antiarrhythmic drugs. In theory, antiseizure drugs that block calcium ion channels might also have properties akin to those of group IV antiarrhythmic drugs, although neuronal calcium channels differ from those in the heart. 2. In practice, the only antiseizure drug that has been used in cardiac arrhythmias is phenytoin, which has characteristics similar to those of group IB antiarrhythmic drugs. Phenytoin has been used for arrhythmias resulting from cardiac glycoside overdose and for ventricular arrhythmias unresponsive to lidocaine. CHECKLIST When you complete this chapter, you should be able to: ❑ List the drugs of choice for partial seizures, generalized tonic-clonic seizures, absence and myoclonic seizures, and status epilepticus. ❑ Identify the mechanisms of antiseizure drug action at the levels of specific ion channels or neurotransmitter systems. ❑ Describe the main pharmacokinetic features, and list the adverse effects of carbamazepine, phenytoin, and valproic acid. ❑ Identify the distinctive toxicities of felbamate, lamotrigine, and topiramate.. ❑ Indicate why benzodiazepines are rarely used in the chronic therapy of seizure states but are valuable in status epilepticus. CHAPTER 24 Antiseizure Drugs 207 DRUG SUMMARY TABLE: Antiseizure Drugs Subclass Pharmacokinetics and Interactions Mechanism of Action Clinical Applications Toxicities Phenytoin Blocks voltage-gated Na+ channels Generalized tonic-clonic and partial seizures Variable absorption, dose-dependent elimination; protein binding; many drug interactions Ataxia, diplopia, gingival hyperplasia, hirsutism, neuropathy Phenobarbital Enhances GABAA receptor responses Same as above Long half-life, inducer of P450 • many interactions Sedation, ataxia Ethosuximide Decreases Ca2+ currents (T-type) Absence seizures Long half-life GI distress, dizziness, headache Blocks voltage-gated Na+ channels and decreases glutamate release Generalized tonic-clonic and partial seizures Well absorbed, active metabolite • many drug interactions Ataxia, diplopia, headache, nausea Enhance GABAA receptor responses Status epilepticus See Chapter 22 Sedation Absence and myoclonic seizures, infantile spasms See Chapter 22 Similar to above Generalized tonic-clonic and partial seizures Variable bioavailability • renal elimination Ataxia, dizziness, somnolence Cyclic ureides Tricyclics Carbamazepine Benzodiazepines Diazepam Clonazepam GABA derivatives Gabapentin Blocks Ca2+ channels Pregabalin Same as above Partial seizures Renal elimination Same as above Vigabatrin Inhibits GABA transaminase Partial seizures Renal elimination Drowsiness, dizziness, psychosis, ocular effects Valproate Blocks high-frequency firing Generalized tonic-clonic, partial, and myoclonic seizures Extensive protein binding and metabolism; many drug interactions Nausea, alopecia, weight gain, teratogenic Lamotrigine Blocks Na+ and Ca2+ channels, decreases neuronal glutamate release Generalized tonic-clonic, partial, myoclonic, and absence seizures Not protein-bound, extensive metabolism • many drug interactions Dizziness, diplopia, headache, rash Levetiracetam Binds synaptic protein, modifies GABA and glutamate release Generalized tonic-clonic and partial seizures Well absorbed, extensive metabolism • some drug interactions Dizziness, nervousness, depression, seizures Tiagabine Blocks GABA reuptake Partial seizures Extensive protein binding and metabolism • some drug interactions Dizziness, nervousness, depression, seizures Topiramate May block Na+ and Ca2+ channels; also increases GABA effects Generalized tonic-clonic, absence, and partial seizures, migraine Both hepatic and renal clearance Sleepiness, cognitive slowing, confusion, paresthesias Zonisamide Blocks Na+ channels Generalized tonic-clonic, partial, and myoclonic seizures Both hepatic and renal clearance Sleepiness, cognitive slowing, poor concentration, paresthesias Miscellaneous C A P T E R 25 General Anesthetics General anesthesia is a state characterized by unconsciousness, analgesia, amnesia, skeletal muscle relaxation, and loss of reflexes. Drugs used as general anesthetics are CNS depressants H with actions that can be induced and terminated more rapidly than those of conventional sedative-hypnotics. General anesthetics Inhaled Gas (nitrous oxide) Intravenous Volatile liquids (halothane) Barbiturates (thiopental) Benzodiazepines (midazolam) Dissociative (ketamine) Opioids (fentanyl) Miscellaneous (etomidate, propofol) STAGES OF ANESTHESIA Modern anesthetics act very rapidly and achieve deep anesthesia quickly. With older and more slowly acting anesthetics, the progressively greater depth of central depression associated with increasing dose or time of exposure is traditionally described as stages of anesthesia. A. Stage 1: Analgesia In stage 1, the patient has decreased awareness of pain, sometimes with amnesia. Consciousness may be impaired but is not lost. B. Stage 2: Disinhibition In stage 2, the patient appears to be delirious and excited. Amnesia occurs, reflexes are enhanced, and respiration is typically irregular; retching and incontinence may occur. 208 C. Stage 3: Surgical Anesthesia In stage 3, the patient is unconscious and has no pain reflexes; respiration is very regular, and blood pressure is maintained. D. Stage 4: Medullary Depression In stage 4, the patient develops severe respiratory and cardiovascular depression that requires mechanical and pharmacologic support. ANESTHESIA PROTOCOLS Anesthesia protocols vary according to the proposed type of diagnostic, therapeutic, or surgical intervention. For minor procedures, conscious sedation techniques that combine intravenous agents with local anesthetics (see Chapter 26) are often used. CHAPTER 25 General Anesthetics 209 High-Yield Terms to Learn Balanced anesthesia Anesthesia produced by a mixture of drugs, often including both inhaled and intravenous agents Inhalation anesthesia Anesthesia induced by inhalation of drug Minimum alveolar anesthetic concentration (MAC) The alveolar concentration of an inhaled anesthetic that is required to prevent a response to a standardized painful stimulus in 50% of patients Analgesia A state of decreased awareness of pain, sometimes with amnesia General anesthesia A state of unconsciousness, analgesia, and amnesia, with skeletal muscle relaxation and loss of reflexes These can provide profound analgesia, with retention of the patient's ability to maintain a patent airway and respond to verbal commands. For more extensive surgical procedures, anesthesia protocols commonly include intravenous drugs to induce the anesthetic state, inhaled anesthetics (with or without intravenous agents) to maintain an anesthetic state, and neuromuscular blocking agents to effect muscle relaxation (see Chapter 27). Vital sign monitoring remains the standard method of assessing depth of anesthesia during surgery. Cerebral monitoring, automated techniques based on quantification of anesthetic effects on the electroencephalograph (EEG), is also useful. MECHANISMS OF ACTION The mechanisms of action of general anesthetics are varied. As CNS depressants, these drugs usually increase the threshold for firing of CNS neurons. The potency of inhaled anesthetics is roughly proportional to their lipid solubility. Mechanisms of action include effects on ion channels by interactions of anesthetic drugs with membrane lipids or proteins with subsequent effects on central neurotransmitter mechanisms. Inhaled anesthetics, barbiturates, benzodiazepines, etomidate, and propofol facilitate γ-aminobutyric acid (GABA)-mediated inhibition at GABAA receptors. These receptors are sensitive to clinically relevant concentrations of the anesthetic agents and exhibit the appropriate stereospecific effects in the case of enantiomeric drugs. Ketamine does not produce its effects via facilitation of GABAA receptor functions, but possibly via its antagonism of the action of the excitatory neurotransmitter glutamic acid on the N-methyl-daspartate (NMDA) receptor. Most inhaled anesthetics also inhibit nicotinic acetylcholine (ACh) receptor isoforms at moderate to high concentrations. The strychnine-sensitive glycine receptor is another ligand-gated ion channel that may function as a target for certain inhaled anesthetics. CNS neurons in different regions of the brain have different sensitivities to general anesthetics; inhibition of neurons involved in pain pathways occurs before inhibition of neurons in the midbrain reticular formation. INHALED ANESTHETICS A. Classification and Pharmacokinetics The agents currently used in inhalation anesthesia are nitrous oxide (a gas) and several easily vaporized liquid halogenated hydrocarbons, including halothane, desflurane, enflurane, isoflurane, sevoflurane, and methoxyflurane. They are administered as gases; their partial pressure, or “tension,” in the inhaled air or in blood or other tissue is a measure of their concentration. Because the standard pressure of the total inhaled mixture is atmospheric pressure (760 mm Hg at sea level), the partial pressure may also be expressed as a percentage. Thus, 50% nitrous oxide in the inhaled air would have a partial pressure of 380 mm Hg. The speed of induction of anesthetic effects depends on several factors, discussed next. 1. Solubility—The more rapidly a drug equilibrates with the blood, the more quickly the drug passes into the brain to produce anesthetic effects. Drugs with a low blood:gas partition coefficient (eg, nitrous oxide) equilibrate more rapidly than those with a higher blood solubility (eg, halothane), as illustrated in Figure 25–1. Partition coefficients for inhalation anesthetics are shown in Table 25–1. 2. Inspired gas partial pressure—A high partial pressure of the gas in the lungs results in more rapid achievement of anesthetic levels in the blood. This effect can be taken advantage of by the initial administration of gas concentrations higher than those required for maintenance of anesthesia. 3. Ventilation rate—The greater the ventilation, the more rapid is the rise in alveolar and blood partial pressure of the agent and the onset of anesthesia (Figure 25–2). This effect is taken advantage of in the induction of the anesthetic state. 4. Pulmonary blood flow—At high pulmonary blood flows, the gas partial pressure rises at a slower rate; thus, the speed of onset of anesthesia is reduced. At low flow rates, onset is faster. In circulatory shock, this effect may accelerate the rate of onset of anesthesia with agents of high blood solubility. 5. Arteriovenous concentration gradient—Uptake of soluble anesthetics into highly perfused tissues may decrease gas tension in mixed venous blood. This can influence the rate of onset of anesthesia because achievement of equilibrium is dependent on the difference in anesthetic tension between arterial and venous blood. B. Elimination Inhaled anesthesia is terminated by redistribution of the drug from the brain to the blood and elimination of the drug through the lungs. 210 PART V Drugs That Act in the Central Nervous System Alveoli Airway Brain Blood Nitrous oxide Brain Blood Alveoli Airway Halothane FIGURE 25–1 Why induction of anesthesia is slower with more soluble anesthetic gases and faster with less soluble ones. In this schematic diagram, solubility is represented by the size of the blood compartment (the more soluble the gas, the larger is the compartment). For a given concentration or partial pressure of the 2 anesthetic gases in the inspired air, it will take much longer with halothane than with nitrous oxide for the blood partial pressure to rise to the same partial pressure as in the alveoli. Because the concentration in the brain can rise no faster than the concentration in the blood, the onset of anesthesia will be much slower with halothane than with nitrous oxide. (Reproduced, with permission, from Katzung BG, editor: Basic & Clinical Pharmacology, 12th ed. McGraw-Hill, 2012: Fig. 25–3.) The rate of recovery from anesthesia using agents with low blood:gas partition coefficients is faster than that of anesthetics with high blood solubility. This important property has led to the introduction of several newer inhaled anesthetics (eg, desflurane, sevoflurane), which, because of their low blood solubility, are characterized by recovery times that are considerably shorter than is the case with older agents. Halothane and methoxyflurane are metabolized by liver enzymes to a significant extent (Table 25–1). Metabolism of halothane and methoxyflurane has only a minor influence on the speed of recovery from their anesthetic effect but does play a role in potential toxicity of these anesthetics. C. Minimum Alveolar Anesthetic Concentration The potency of inhaled anesthetics is best measured by the minimum alveolar anesthetic concentration (MAC), defined as the alveolar concentration required to eliminate the response to a standardized painful stimulus in 50% of patients. Each anesthetic has a defined MAC (Table 25–1), but this value may vary among patients depending on age, cardiovascular status, and use of adjuvant drugs. Estimations of MAC value suggest a relatively “steep” dose–response relationship for inhaled anesthetics. MACs for infants and elderly patients are lower than those for adolescents and young adults. When several anesthetic agents are used simultaneously, their MAC values are additive. D. Effects of Inhaled Anesthetics 1. CNS effects—Inhaled anesthetics decrease brain metabolic rate. They reduce vascular resistance and thus increase cerebral blood flow. This may lead to an increase in intracranial pressure. High concentrations of enflurane may cause spike-and-wave activity and muscle twitching, but this effect is unique to this drug. Although nitrous oxide has low anesthetic potency (ie, a high MAC), it exerts marked analgesic and amnestic actions. TABLE 25–1 Properties of inhalation anesthetics. Anesthetic Blood:Gas Partition Coefficient Minimum Alveolar Concentration (%)a Nitrous oxide 0.47 Desflurane 0.42 6.5 <0.1% Sevoflurane 0.69 2.0 2–5% (fluoride) Isoflurane 1.40 1.4 <2% Enflurane 1.80 1.7 8% Halothane 2.30 0.75 >40% 0.16 >70% (fluoride) Methoxyflurane 12 >100 Metabolism a None Minimum alveolar concentration (MAC) is the anesthetic concentration that eliminates the response in 50% of patients exposed to a standardized painful stimulus. In this table, MAC is expressed as a percentage of the inspired gas mixture. Modified and reproduced, with permission, from Katzung BG, editor: Basic & Clinical Pharmacology, 10th ed. McGraw-Hill, 2007. CHAPTER 25 General Anesthetics 1.0 anesthetics are used together with neuromuscular blockers (especially succinylcholine). This rare condition is thought in some cases to be due to mutations in the gene loci corresponding to the ryanodine receptor (RyR1). Other chromosomal loci for malignant hyperthermia include mutant alleles of the gene-encoding skeletal muscle L-type calcium channels. The uncontrolled release of calcium by the sarcoplasmic reticulum of skeletal muscle leads to muscle spasm, hyperthermia, and autonomic lability (Table 16-2). Dantrolene is indicated for the treatment of this life-threatening condition, with supportive management. Ventilation (L/min) 8 Nitrous oxide 2 FA /FI Halothane 0.5 211 8 2 SKILL KEEPER: SIGNALING MECHANISMS (SEE CHAPTER 2) 0 10 20 30 40 50 Time (min) FIGURE 25–2 Ventilation rate and arterial anesthetic tensions. Increased ventilation (8 versus 2 L/min) has a much greater effect on equilibration of halothane than nitrous oxide. FA/FI, ratio of alveolar drug concentration to inhaled concentration. (Reproduced, with permission, from Katzung BG, editor: Basic & Clinical Pharmacology, 12th ed. McGraw-Hill, 2012: Fig. 25–5.) 2. Cardiovascular effects—Most inhaled anesthetics decrease arterial blood pressure moderately. Enflurane and halothane are myocardial depressants that decrease cardiac output, whereas isoflurane, desflurane and sevoflurane cause peripheral vasodilation. Nitrous oxide is less likely to lower blood pressure than are other inhaled anesthetics. Blood flow to the liver and kidney is decreased by most inhaled agents. Inhaled anesthetics depress myocardial function—nitrous oxide least. Halothane, and to a lesser degree isoflurane, may sensitize the myocardium to the arrhythmogenic effects of catecholamines. 3. Respiratory effects—Although the rate of respiration may be increased, all inhaled anesthetics cause a dose-dependent decrease in tidal volume and minute ventilation, leading to an increase in arterial CO2 tension. Inhaled anesthetics decrease ventilatory response to hypoxia even at subanesthetic concentrations (eg, during recovery). Nitrous oxide has the smallest effect on respiration. Most inhaled anesthetics are bronchodilators, but desflurane is a pulmonary irritant and may cause bronchospasm. The pungency of enflurane causes breath-holding, which limits its use in anesthesia induction. 4. Toxicity—Postoperative hepatitis has occurred (rarely) after halothane anesthesia in patients experiencing hypovolemic shock or other severe stress. The mechanism of hepatotoxicity is unclear but may involve formation of reactive metabolites that cause direct toxicity or initiate immune-mediated responses. Fluoride released by metabolism of methoxyflurane (and possibly enflurane and sevoflurane) may cause renal insufficiency after prolonged anesthesia. Prolonged exposure to nitrous oxide decreases methionine synthase activity and may lead to megaloblastic anemia. Susceptible patients may develop malignant hyperthermia when Like most drugs, general anesthetics appear to act via interactions with specific receptor molecules involved in cell signaling. For review purposes, list the major types of signaling mechanisms relevant to the actions of drugs that act via receptors. The Skill Keeper Answers appear at the end of the chapter. INTRAVENOUS ANESTHETICS A. Propofol Propofol produces anesthesia as rapidly as the intravenous barbiturates, and recovery is more rapid. Propofol has antiemetic actions, and recovery is not delayed after prolonged infusion. The drug is very commonly used as a component of balanced anesthesia and as an anesthetic in outpatient surgery. Propofol is also effective in producing prolonged sedation in patients in critical care settings. Propofol may cause marked hypotension during induction of anesthesia, primarily through decreased peripheral resistance. Total body clearance of propofol is greater than hepatic blood flow, suggesting that its elimination includes other mechanisms in addition to metabolism by liver enzymes. Fospropofol, a water-soluble prodrug form, is broken down in the body by alkaline phosphatase to form propofol. However, onset and recovery are both slower than propofol. Although fospropofol appears to cause less pain at injection sites than the standard form of the drug, many patients experience paresthesias. B. Barbiturates Thiopental and methohexital have high lipid solubility, which promotes rapid entry into the brain and results in surgical anesthesia in one circulation time (<1 min). These drugs are used for induction of anesthesia and for short surgical procedures. The anesthetic effects of thiopental are terminated by redistribution from the brain to other highly perfused tissues (Figure 25–3), but hepatic metabolism is required for elimination from the body. Barbiturates are respiratory and circulatory depressants; because they depress cerebral blood flow, they can also decrease intracranial pressure. 212 PART V Drugs That Act in the Central Nervous System Lean tissues Brain and viscera duration of action. The drug is not analgesic, and its primary advantage is in anesthesia for patients with limited cardiac or respiratory reserve. Etomidate may cause pain and myoclonus on injection and nausea postoperatively. Prolonged administration may cause adrenal suppression. Fat 0.5 1 G. Dexmedetomidine This centrally acting α2-adrenergic agonist has analgesic and hypnotic actions when used intravenously. Its characteristics include rapid clearance resulting in a short elimination half-life. Dexmedetomidine is mainly used for short-term sedation in an ICU setting. When used in general anesthesia, the drug decreases dosage requirements for both inhaled and intravenous anesthetics. 100 Blood Percent of dose 80 60 40 20 0 0.125 4 16 64 256 Time (min) FIGURE 25–3 Redistribution of thiopental after intravenous bolus administration. Note that the time axis is not linear. (Reproduced, with permission, from Katzung BG, editor: Basic & Clinical Pharmacology, 12th ed. McGraw-Hill, 2012: Fig. 25–7.) C. Benzodiazepines Midazolam is widely used adjunctively with inhaled anesthetics and intravenous opioids. The onset of its CNS effects is slower than that of thiopental, and it has a longer duration of action. Cases of severe postoperative respiratory depression have occurred. The benzodiazepine receptor antagonist, flumazenil, accelerates recovery from midazolam and other benzodiazepines. D. Ketamine This drug produces a state of “dissociative anesthesia” in which the patient remains conscious but has marked catatonia, analgesia, and amnesia. Ketamine is a chemical congener of the psychotomimetic agent, phencyclidine (PCP), and inhibits NMDA glutamate transmission. The drug is a cardiovascular stimulant, and this action may lead to an increase in intracranial pressure. Emergence reactions, including disorientation, excitation, and hallucinations, which occur during recovery from ketamine anesthesia, can be reduced by the preoperative use of benzodiazepines. E. Opioids Morphine and fentanyl are used with other CNS depressants (nitrous oxide, benzodiazepines) in anesthesia regimens and are especially valuable in high-risk patients who might not survive a full general anesthetic. Intravenous opioids may cause chest wall rigidity, which can impair ventilation. Respiratory depression with these drugs may be reversed postoperatively with naloxone. Neuroleptanesthesia is a state of analgesia and amnesia is produced when fentanyl is used with droperidol and nitrous oxide. Newer opioids related to fentanyl have been introduced for intravenous anesthesia. Alfentanil and remifentanil have been used for induction of anesthesia. Recovery from the actions of remifentanil is faster than recovery from other opioids used in anesthesia because of its rapid metabolism by blood and tissue esterases. F. Etomidate This imidazole derivative affords rapid induction with minimal change in cardiac function or respiratory rate and has a short QUESTIONS 1. A new halogenated gas anesthetic has a blood:gas partition coefficient of 0.5 and a MAC value of 1%. Which prediction about this agent is most accurate? (Refer to Table 25–1 for comparison of agents.) (A) Equilibrium between arterial and venous gas tension will be achieved very slowly (B) It will be metabolized by the liver to release fluoride ions (C) It will be more soluble in the blood than isoflurane (D) Speed of onset will be similar to that of nitrous oxide (E) The new agent will be more potent than halothane 2. Which statement concerning the effects of anesthetic agents is false? (A) Bronchiolar smooth muscle relaxation occurs during halothane anesthesia (B) Chest muscle rigidity often follows the administration of fentanyl (C) Mild, generalized muscle twitching occurs at high doses of enflurane (D) Severe hepatitis has been reported after the use of methoxyflurane (E) The use of midazolam with inhalation anesthetics may prolong postanesthesia recovery 3. A 23-year-old man has a pheochromocytoma, blood pressure of 190/120 mm Hg, and hematocrit of 50%. Pulmonary function and renal function are normal. His catecholamines are elevated, and he has a well-defined abdominal tumor on MRI. He has been scheduled for surgery. Which one of the following agents should be avoided in the anesthesia protocol? (A) Desflurane (B) Fentanyl (C) Isoflurane (D) Midazolam (E) Sevoflurane 4. Which statement concerning nitrous oxide is accurate? (A) A useful component of anesthesia protocols because it lacks cardiovascular depression (B) Anemia is a common adverse effect in patients exposed to nitrous oxide for periods longer than 2 h (C) It is the most potent of the inhaled anesthetics (D) There is a direct association between the use of nitrous oxide and malignant hyperthermia (E) Up to 50% of nitrous oxide is eliminated via hepatic metabolism CHAPTER 25 General Anesthetics 5. Which statement concerning anesthetic MAC (minimum anesthetic concentration) value is accurate? (A) Anesthetics with low MAC value have low potency (B) MAC values increase in elderly patients (C) MAC values give information about the slope of the dose–response curve (D) Methoxyflurane has an extremely low MAC value (E) Simultaneous use of opioid analgesics increases the MAC for inhaled anesthetics 6. Total intravenous anesthesia with fentanyl has been selected for a frail elderly woman about to undergo cardiac surgery. Which statement about this anesthesia protocol is accurate? (A) Fentanyl will control the hypertensive response to surgical stimulation (B) Marked relaxation of skeletal muscles is anticipated (C) Opioids such as fentanyl provide useful cardiostimulatory effects (D) Patient awareness may occur during surgery, with recall after recovery (E) The patient is likely to experience pain during surgery Questions 7 and 8. A 20-year-old male patient scheduled for hernia surgery was anesthetized with halothane and nitrous oxide; tubocurarine was provided for skeletal muscle relaxation. The patient rapidly developed tachycardia and became hypertensive. Generalized skeletal muscle rigidity was accompanied by marked hyperthermia. Laboratory values revealed hyperkalemia and acidosis. 7. This unusual complication of anesthesia is most likely to be caused by (A) Acetylcholine release from somatic nerve endings at skeletal muscle (B) Activation of brain dopamine receptors by halothane (C) Antagonism of autonomic ganglia by tubocurarine (D) Calcium released within skeletal muscle (E) Toxic metabolites of nitrous oxide 8. The patient should be treated immediately with (A) Atropine (B) Baclofen (C) Dantrolene (D) Edrophonium (E) Flumazenil 9. If ketamine is used as the sole anesthetic in the attempted reduction of a dislocated shoulder joint, its actions will include (A) Analgesia (B) Bradycardia (C) Hypotension (D) Muscle rigidity (E) Respiratory depression 10. Postoperative vomiting is uncommon with this intravenous agent, and patients are often able to ambulate sooner than those who receive other anesthetics. (A) Enflurane (B) Etomidate (C) Midazolam (D) Propofol (E) Thiopental 213 ANSWERS 1. The partition coefficient of an inhaled anesthetic is a determinant of its kinetic characteristics. Agents with low blood:gas solubility have a fast onset of action and a short duration of recovery. The new agent described here resembles nitrous oxide but is more potent, as indicated by its low MAC value. Not all halogenated anesthetics undergo significant hepatic metabolism or release fluoride ions. The answer is D. 2. Hepatitis after general anesthesia has been linked to use of halothane, although the incidence is very low (1 in 20,00–35,000). Hepatotoxicity has not been reported after administration of methoxyflurane or other inhaled anesthetics. However, fluoride release from prolonged use of methoxyflurane has caused renal insufficiency. The answer is D. 3. Isoflurane sensitizes the myocardium to catecholamines, as does halothane (not listed). Arrhythmias may occur in patients with cardiac disease who have high circulating levels of epinephrine and norepinephrine (eg, patients with pheochromocytoma). Other newer inhaled anesthetics are considerably less arrhythmogenic. The answer is C. 4. Anemia has not been reported in patients exposed to nitrous oxide anesthesia for periods as long as 6 h. Nitrous oxide is the least potent of the inhaled anesthetics, and the compound has not been implicated in malignant hyperthermia. More than 98% of the gas is eliminated via exhalation. The answer is A. 5. MAC value is inversely related to potency; a low MAC means high potency. MAC gives no information about the slope of the dose–response curve. Use of opioid analgesics or other CNS depressants with inhaled anesthetics lowers the MAC value. As with most CNS depressants, the elderly patient is more sensitive, so MAC values are lower. Methoxyflurane has the lowest MAC value of the inhaled anesthetics. The answer is D. 6. Intravenous opioids (eg, fentanyl) are widely used in anesthesia for cardiac surgery because they provide full analgesia and cause less cardiac depression than inhaled anesthetic agents. The opioids are not cardiac stimulants, and fentanyl is more likely to cause skeletal muscle rigidity than relaxation. Disadvantages of this technique are patient recall (which can be decreased by concomitant use of a benzodiazepine) and the occurrence of hypertensive responses to surgical stimulation. The addition of vasodilators (eg, nitroprusside) or a β blocker (eg, esmolol) may be needed to prevent intraoperative hypertension. The answer is D. 7. Malignant hyperthermia is a rare but life-threatening reaction that may occur during general anesthesia with halogenated anesthetics and skeletal muscle relaxants, particularly succinylcholine and tubocurarine. Release of calcium from skeletal sarcoplasmic reticulum leads to muscle spasms, hyperthermia and autonomic instability. Predisposing genetic factors include clinical myopathy associated with mutations in the gene loci for the skeletal muscle ryanodine receptor or L-type calcium receptors. Nitrous oxide is not metabolized! The answer is D. 214 PART V Drugs That Act in the Central Nervous System 8. The drug of choice in malignant hyperthermia is dantrolene, which prevents release of calcium from the sarcoplasmic reticulum of skeletal muscle cells. Appropriate measures must be taken to lower body temperature, control hypertension, and restore acid-base and electrolyte balance. The answer is C. 9. Ketamine is a cardiovascular stimulant, increasing heart rate and blood pressure. This results in part from central sympathetic stimulation and from inhibition of norepinephrine reuptake at sympathetic nerve endings. Analgesia and amnesia occur, with preservation of muscle tone and minimal depression of respiration. The answer is A. 10. Propofol is used extensively in anesthesia protocols, including those for day surgery. The favorable properties of the drug include an antiemetic effect and recovery more rapid than that after use of other intravenous drugs. Propofol does not cause cumulative effects, possibly because of its short half-life (2–8 min) in the body. The drug is also used for prolonged sedation in critical care settings. The answer is D. SKILL KEEPER ANSWER: SIGNALING MECHANISMS (SEE CHAPTER 2) 1. Receptors that modify gene transcription: adrenal and gonadal steroids 2. Receptors on membrane-spanning enzymes: insulin 3. Receptors activating Janus kinases that modulate STAT molecules: cytokines 4. Receptors directly coupled to ion channels: nicotinic (ACh), GABA, glycine 5. Receptors coupled to enzymes via G proteins: many endogenous compounds (eg, ACh, NE, serotonin) and drugs 6. Receptors that are enzymes or transporters: acetylcholinesterase, angiotensin-converting enzyme, carbonic anhydrase, H+/K+ antiporter, etc CHECKLIST When you complete this chapter, you should be able to: ❑ Name the major inhalation anesthetic agents and identify their pharmacodynamic and pharmacokinetic properties. ❑ Describe what is meant by the terms (1) blood:gas partition coefficient and (2) minimum alveolar anesthetic concentration. ❑ Identify proposed molecular targets for the actions of anesthetic drugs. ❑ Describe how the blood:gas partition coefficient of an inhalation anesthetic influences its speed of onset of anesthesia and its recovery time. ❑ Identify the commonly used intravenous anesthetics and list their main pharmacokinetic and pharmacodynamic characteristics. CHAPTER 25 General Anesthetics 215 DRUG SUMMARY TABLE: General Anesthetics Subclass Possible Mechanism Pharmacologic Effects Pharmacokinetics Toxicities and Interactions Facilitate GABA-mediated inhibition • block brain NMDA and ACh-N receptors Increase cerebral blood flow • enflurane and halothane decrease cardiac output. Others cause vasodilation • all decrease respiratory functions— lung irritation (desflurane) Rate of onset and recovery vary by blood:gas partition coefficient • recovery mainly due to redistribution from brain to other tissues Toxicity: extensions of effects on brain, heart/vasculature, lungs Drug interactions: additive CNS depression with many agents, especially opioids and sedative-hypnotics Barbiturates, benzodiazepines, etomidate, and propofol facilitate GABAmediated inhibition at GABAA receptors Circulatory and respiratory depression • decrease intracranial pressure High lipid solubility—fast onset and short duration due to redistribution Extensions of CNS depressant actions • additive CNS depression with many drugs Less depressant than barbiturates Slower onset, but longer duration than barbiturates Postoperative respiratory depression reversed by flumazenil Analgesia, amnesia and catatonia but consciousness retained • cardiovascular (CV) stimulation! Moderate duration of action—hepatic metabolism Increased intracranial pressure • emergence reactions Minimal effects on CV and respiratory functions Short duration due to redistribution No analgesia, pain on injection (may need opioid), myoclonus, nausea, and vomiting Interact with µ, κ, and δ opioid receptors Marked analgesia, respiratory depression (see Chapter 31) Alfentanil and remifentanil fast onset (induction) Respiratory depression— reversed by naloxone Uncertain Vasodilation and hypotension • negative inotropy. Fospropofol water-soluble Fast onset and fast recovery due to inactivation Hypotension (during induction), cardiovascular depression Inhaled anesthetics Desflurane Enflurane Halothane Isoflurane Sevoflurane Nitrous oxide Intravenous anesthetics Barbiturates Thiopental, Thioamylal, Methohexital Benzodiazepines Midazolam Dissociative Ketamine Blocks excitation by glutamate at NMDA receptors Imidazole Etomidate Opioids Fentanyl Alfentanil Remifentanil Morphine Phenols Propofol, Fospropofol ACh, acetylcholine; NMDA, N-methyl-D-aspartate. C H A P T E R 26 Local Anesthetics Local anesthesia is the condition that results when sensory transmission from a local area of the body to the CNS is blocked. The local anesthetics constitute a group of chemically similar agents (esters and amides) that block the sodium channels of excitable membranes. Because these drugs can be administered by injection in the target area, or by topical application in some cases, the anesthetic effect can be restricted to a localized area (eg, the cornea or an arm). When given intravenously, local anesthetics have effects on other tissues. Local anesthetics Esters Long action (tetracaine) Short action (procaine) Amides Surface action (benzocaine, cocaine) CHEMISTRY Most local anesthetic drugs are esters or amides of simple benzene derivatives. Subgroups within the local anesthetics are based on this chemical characteristic and on duration of action. The commonly used local anesthetics are weak bases with at least 1 ionizable amine function that can become charged through the gain of a proton (H+). As discussed in Chapter 1, the degree of ionization is a function of the pKa of the drug and the pH of the medium. Because the pH of tissue may differ from the physiologic 7.4 (eg, it may be as low as 6.4 in infected tissue), the degree of ionization of the drug will vary. Because the pKa of most local anesthetics is between 8.0 and 9.0 (benzocaine is an exception), variations in pH associated with infection can have significant effects on the proportion of ionized to nonionized drug. The question of the active form of the drug (ionized versus nonionized) is discussed later. PHARMACOKINETICS Many shorter-acting local anesthetics are readily absorbed into the blood from the injection site after administration. The duration 216 Long action (bupivacaine, ropivacaine) Medium action (lidocaine) of local action is therefore limited unless blood flow to the area is reduced. This can be accomplished by administration of a vasoconstrictor (usually an α-agonist sympathomimetic) with the local anesthetic agent. Cocaine is an important exception because it has intrinsic sympathomimetic action due to its inhibition of norepinephrine reuptake into nerve terminals. The longer-acting agents (eg, bupivacaine, ropivacaine, tetracain) are also less dependent on the coadministration of vasoconstrictors. Surface activity (ability to reach superficial nerves when applied to the surface of mucous membranes) is a property of certain local anesthetics, especially cocaine and benzocaine (both only available as topical forms), lidocaine, and tetracaine. Metabolism of ester local anesthetics is carried out by plasma cholinesterases (pseudocholinesterases) and is very rapid for procaine (half-life, 1–2 min), slower for cocaine, and very slow for tetracaine). The amides are metabolized in the liver, in part by cytochrome P450 isozymes. The half-lives of lidocaine and prilocaine are approximately 1.5 h. Bupivacaine and ropivacaine are the longest-acting amide local anesthetics with half-lives of 3.5 and 4.2 h, respectively. Liver dysfunction may increase the elimination half-life of amide local anesthetics (and increase the risk of toxicity). CHAPTER 26 Local Anesthetics Acidification of the urine promotes ionization of local anesthetics; the charged forms of such drugs are more rapidly excreted than nonionized forms. rate, and anatomic location (Table 26–1). In general, smaller fibers are blocked more easily than larger fibers, and myelinated fibers are blocked more easily than unmyelinated fibers. Activated pain fibers fire rapidly; thus, pain sensation appears to be selectively blocked by local anesthetics. Fibers located in the periphery of a thick nerve bundle are blocked sooner than those in the core because they are exposed earlier to higher concentrations of the anesthetic. MECHANISM OF ACTION Local anesthetics block voltage-dependent sodium channels and reduce the influx of sodium ions, thereby preventing depolarization of the membrane and blocking conduction of the action potential. Local anesthetics gain access to their receptors from the cytoplasm or the membrane (Figure 26–1). Because the drug molecule must cross the lipid membrane to reach the cytoplasm, the more lipidsoluble (nonionized, uncharged) form reaches effective intracellular concentrations more rapidly than does the ionized form. On the other hand, once inside the axon, the ionized (charged) form of the drug is the more effective blocking entity. Thus, both the nonionized and the ionized forms of the drug play important roles—the first in reaching the receptor site and the second in causing the effect. The affinity of the receptor site within the sodium channel for the local anesthetic is a function of the state of the channel, whether it is resting, open, or inactivated, and therefore follows the same rules of use dependence and voltage dependence that were described for the sodium channel-blocking antiarrhythmic drugs (see Chapter 14). In particular, if other factors are equal, rapidly firing fibers are usually blocked before slowly firing fibers. High concentrations of extracellular K+ may enhance local anesthetic activity, whereas elevated extracellular Ca2+ may antagonize it. B. Other Tissues The effects of these drugs on the heart are discussed in Chapter 14 (see group 1 antiarrhythmic agents). Most local anesthetics also have weak blocking effects on skeletal muscle neuromuscular transmission, but these actions have no clinical application. The mood elevation induced by cocaine reflects actions on dopamine or other amine-mediated synaptic transmission in the CNS rather than a local anesthetic action on membranes. CLINICAL USE The local anesthetics are commonly used for minor surgical procedures often in combination with vasoconstrictors such as epinephrine. Onset of action may be accelerated by the addition of sodium bicarbonate, which enhances intracellular access of these weakly basic compounds. Articaine has the fastest onset of action. Local anesthetics are also used in spinal anesthesia and to produce autonomic blockade in ischemic conditions. Slow epidural infusion at low concentrations has been used successfully for postoperative analgesia (in the same way as epidural opioid infusion; Chapter 31). Repeated epidural injection in anesthetic doses may lead to tachyphylaxis, however. Intravenous local anesthetics may be used for reducing pain in the perioperative period. Oral and parenteral forms of local anesthetics are sometimes used adjunctively in neuropathic pain states. PHARMACOLOGIC EFFECTS A. Nerves Differential sensitivity of various types of nerve fibers to local anesthetics depends on fiber diameter, myelination, physiologic firing + Drug −H+ + +H 217 Na+ channel Drug Outside + Na Membrane diffusion Receptor Drug Membrane Na+ Drug+ Inside Cytoplasmic diffusion Drug +H+ −H+ Drug + FIGURE 26–1 Schematic diagram of the sodium channel in an excitable membrane (eg, an axon) and the pathways by which a local anesthetic molecule (Drug) may reach its receptor. Sodium ions are not able to pass through the channel when the drug is bound to the receptor. The local anesthetic diffuses within the membrane in its uncharged form. In the aqueous extracellular and intracellular spaces, the charged form (Drug+) is also present. 218 PART V Drugs That Act in the Central Nervous System TABLE 26–1 Susceptibility to block of types of nerve fibers. Diameter (μm) Myelination Conduction Velocity (m/s) Sensitivity to Block Proprioception, motor 12–20 Heavy 70–120 + Fiber Type Function Type A Alpha Beta Touch, pressure 5–12 Heavy 30–70 ++ Gamma Muscle spindles 3–6 Heavy 15–30 ++ Delta Pain, temperature 2–5 Heavy 12–30 +++ Type B Preganglionic, autonomic <3 Light 3–15 ++++ Type C Dorsal root Pain 0.4–1.2 None 0.5–2.3 ++++ Postganglionic 0.3–1.3 None 0.7–2.3 ++++ Sympathetic Reproduced, with permission, from Katzung BG, editor: Basic & Clinical Pharmacology, 12th ed. McGraw-Hill, 2012. TOXICITY A. CNS Effects The important toxic effects of most local anesthetics are in the CNS. All local anesthetics are capable of producing a spectrum of central effects, including light-headedness or sedation, restlessness, nystagmus, and tonic-clonic convulsions. Severe convulsions may be followed by coma with respiratory and cardiovascular depression. B. Cardiovascular Effects With the exception of cocaine, all local anesthetics are vasodilators. Patients with preexisting cardiovascular disease may develop heart block and other disturbances of cardiac electrical function at high plasma levels of local anesthetics. Bupivacaine, a racemic mixture of two isomers may produce severe cardiovascular toxicity, including arrhythmias and hypotension. The (S) isomer, levobupivacaine, is less cardiotoxic. Cardiotoxicity has also been reported for ropivacaine when used for peripheral nerve block. The ability of cocaine to block norepinephrine reuptake at sympathetic neuroeffector junctions and the drug's vasoconstricting actions contribute to cardiovascular toxicity. When cocaine is used as a drug of abuse, its cardiovascular toxicity includes severe hypertension with cerebral hemorrhage, cardiac arrhythmias, and myocardial infarction. C. Other Toxic Effects Prilocaine is metabolized to products that include o-toluidine, an agent capable of converting hemoglobin to methemoglobin. Though tolerated in healthy persons, even moderate methemoglobinemia can cause decompensation in patients with cardiac or pulmonary disease. The ester-type local anesthetics are metabolized to products that can cause antibody formation in some patients. Allergic responses to local anesthetics are rare and can usually be prevented by using an agent from the amide subclass. In high concentrations, local anesthetics may cause a local neurotoxic action (especially important in the spinal cord) that includes histologic damage and permanent impairment of function. SKILL KEEPER: CARDIAC TOXICITY OF LOCAL ANESTHETICS (SEE CHAPTER 14) Explain how hyperkalemia facilitates the cardiac toxicity of local anesthetics. The Skill Keeper Answer appears at the end of the chapter. D. Treatment of Toxicity Severe toxicity is treated symptomatically; there are no antidotes. Convulsions are usually managed with intravenous diazepam or a short-acting barbiturate such as thiopental. Hyperventilation with oxygen is helpful. Occasionally, a neuromuscular blocking drug may be used to control violent convulsive activity. The cardiovascular toxicity of bupivacaine overdose is difficult to treat and has caused fatalities in young adults; intravenous administration of lipid has been reported to be of benefit. QUESTIONS 1. Characteristic properties of local anesthetics include all of the following EXCEPT (A) An increase in membrane refractory period (B) Blockade of voltage-dependent sodium channels (C) Effects on vascular tone (D) Preferential binding to resting channels (E) Slowing of axonal impulse conduction CHAPTER 26 Local Anesthetics 2. The pKa of lidocaine is 7.7. In infected tissue, which can be acidic, for example, at pH 6.7, the percentage of the drug in the nonionized form will be (A) 1% (B) 10% (C) 50% (D) 90% (E) 99% 3. Which statement about the speed of onset of nerve blockade with local anesthetics is correct? (A) Faster in hypercalcemia (B) Faster in myelinated fibers (C) Faster in tissues that are infected (D) Slower in hyperkalemia (E) Slower in the periphery of a nerve bundle than in the center of a bundle 4. The most important effect of inadvertent intravenous administration of a large dose of lidocaine is (A) Bronchoconstriction (B) Methemoglobinemia (C) Renal failure (D) Seizures (E) Tachycardia 5. All of the following factors influence the action of local anesthetics EXCEPT (A) Acetylcholinesterase activity in the region of the injection site (B) Blood flow through the tissue in which the injection is made (C) Dose of local anesthetic injected (D) The use of vasoconstrictors (E) Tissue pH 6. You have a vial containing 10 mL of a 2% solution of lidocaine. How much lidocaine is present in 1 mL? (A) 2 mg (B) 5 mg (C) 10 mg (D) 20 mg (E) 50 mg 7. Which statement about the toxicity of local anesthetics is correct? (A) Bupivacaine is the safest local anesthetic to use in patients at risk for cardiac arrhythmias. (B) In overdosage, hyperventilation (with oxygen) is helpful to correct acidosis and lower extracellular potassium (C) Intravenous injection of local anesthetics may stimulate ectopic cardiac pacemaker activity (D) Most local anesthetics cause vasoconstriction (E) Serious cardiovascular reactions are more likely to occur with tetracaine than with bupivacaine 8. A vasoconstrictor added to a solution of lidocaine for a peripheral nerve block will (A) Decrease the risk of a seizure (B) Increase the duration of anesthetic action of the local anesthetic (C) Both A and B (D) Neither A nor B 219 9. A child requires multiple minor surgical procedures involving the nasopharynx. Which drug has high surface local anesthetic activity and intrinsic vasoconstrictor actions that reduce bleeding in mucous membranes? (A) Bupivacaine (B) Cocaine (C) Lidocaine (D) Mepivacaine (E) Tetracaine 10. Prilocaine is relatively contraindicated in patients with cardiovascular or pulmonary disease because the drug (A) Acts as an agonist at β adrenoceptors in the heart and the lung (B) Causes decompensation through formation of methemoglobin (C) Inhibits cyclooxygenase in cardiac and pulmonary cells (D) Is a potent bronchoconstrictor (E) None of the above ANSWERS 1. Local anesthetics bind preferentially to sodium channels in the open and inactivated states. Recovery from drug-induced block is 10–1000 times slower than recovery of channels from normal inactivation. Resting channels have a lower affinity for local anesthetics. The answer is D. 2. Because the drug is a weak base, it is more ionized (protonated) at pH values lower than its pKa. Because the pH given is 1 log unit lower (more acid) than the pKa, the ratio of ionized to nonionized drug will be approximately 90:10. The answer is B. (Recall from Chapter 1 that at a pH equal to pKa, the ratio is 1:1; at 1 log unit difference, the ratio is approximately 90:10; at 2 log units difference, 99:1; and so on.) 3. Myelinated nerve fibers are blocked by local anesthetics more readily than unmyelinated ones. See the Skill Keeper answer for an explanation of the effects of hypocalcemia and hyperkalemia on nerve blockade by local anesthetics. The answer is B. 4. Of the effects listed, the most important in local anesthetic overdose (of both amide and ester types) concern the CNS. Such effects can include sedation or restlessness, nystagmus, coma, respiratory depression, and seizures. Intravenous diazepam is commonly used for seizures caused by local anesthetics. Methemoglobinemia is caused by a prilocaine metabolite. The answer is D. 5. Local anesthetics are poor substrates for acetylcholinesterase, and the activity of this enzyme does not play a part in terminating the actions of local anesthetics. Ester-type local anesthetics are hydrolyzed by plasma (and tissue) pseudocholinesterases. Persons with genetically based defects in pseudocholinesterase activity are unusually sensitive to procaine and other esters. The answer is A. 6. The fact that you have 10 mL of the solution of lidocaine is irrelevant. A 2% solution of any drug contains 2 g/100 mL. The amount of lidocaine in 1 mL of a 2% solution is thus 0.02 g, or 20 mg. The answer is D. 220 PART V Drugs That Act in the Central Nervous System 7. Acidosis resulting from tissue hypoxia favors local anesthetic toxicity because these drugs bind more avidly (or dissociate more slowly) from the sodium channel binding site when they are in the charged state. (Note that onset of therapeutic effect may be slower because charged local anesthetics penetrate the membrane less rapidly; see text.) Hyperkalemia depolarizes the membrane, which also favors local anesthetic binding. Oxygenation reduces both acidosis and hyperkalemia. Bupivacaine may cause severe cardiotoxicity including arrhythmias. The answer is B. 8. Epinephrine increases the duration of a nerve block when it is administered with short- and medium-duration local anesthetics. As a result of the vasoconstriction that prolongs the duration of this block, less local anesthetic is required, so the risk of toxicity (eg, a seizure) is reduced. The answer is C. 9. Cocaine is the only local anesthetic with intrinsic vasoconstrictor activity owing to its action to block the reuptake of norepinephrine released from sympathetic nerve endings (Chapter 9). Cocaine also has significant surface local anesthetic activity and is favored for head, neck, and pharyngeal surgery. The answer is B. 10. Large doses of prilocaine may cause accumulation of o-toluidine, a metabolite that converts hemoglobin to methemoglobin. Patients may become cyanotic with blood “chocolate colored.” High blood levels of methemoglobin have resulted in decompensation in patients who have cardiac or pulmonary diseases. The answer is B. SKILL KEEPER ANSWER: CARDIAC TOXICITY OF LOCAL ANESTHETICS (SEE CHAPTER 14) Sodium channel blockers (eg, local anesthetics) bind more readily to open (activated) or inactivated sodium channels. Hyperkalemia depolarizes the resting membrane potential, so more sodium channels are in the inactivated state. Conversely, hypercalcemia tends to hyperpolarize the resting potential and reduces the block of sodium channels. CHECKLIST When you complete this chapter, you should be able to: ❑ Describe the mechanism of action of local anesthetics. ❑ Know what is meant by the terms “use-dependent blockade” and “state-dependent blockade.” ❑ Explain the relationship among tissue pH, drug pKa, and the rate of onset of local anesthetic action. ❑ List 4 factors that determine the susceptibility of nerve fibers to local anesthetic blockade. ❑ Describe the major toxic effects of the local anesthetics. DRUG SUMMARY TABLE: Drugs Used for Local Anesthesia Subclass Mechanism of Action Pharmacokinetics Clinical Applications Toxicities Blockade of Na+ channels slows, then prevents action potential propagation Hepatic metabolism via CYP450 in part • Half-lives: lidocaine, prilocaine < 2 h, others 3–4 h Analgesia via topical use, or injection (perineural, epidural, subarachnoid) • rarely IV CNS: excitation, seizures • CV: vasodilation, hypotension, arrhythmias (bupivacaine) As above, plus cocaine has intrinsic sympathomimetic actions Rapid metabolism via plasma esterases • short half-lives Analgesia, topical only for cocaine and benzocaine As above re CNS actions • Cocaine vasoconstricts • When abused cocaine has caused hypertension, seizures, and cardiac arrhythmias Amides Articaine Bupivacaine Levobupivacaine Lidocainea Mepivacaine Prilocaine Ropivacaine Esters Benzocainea Cocainea Procaine Tetracainea a Topical fomulations available. C A P T E R 27 Skeletal Muscle Relaxants The drugs in this chapter are divided into 2 dissimilar groups. The neuromuscular blocking drugs, which act at the skeletal myoneural junction, are used to produce muscle paralysis to H facilitate surgery or assisted ventilation. The spasmolytic drugs, most of which act in the CNS, are used to reduce abnormally elevated tone caused by neurologic or muscle end plate disease. Skeletal muscle relaxants Neuromuscular blockers Nondepolarizing Long action (tubocurarine) Spasmolytics Depolarizing (succinylcholine) Intermediate action (rocuronium) NEUROMUSCULAR BLOCKING DRUGS A. Classification and Prototypes Skeletal muscle contraction is evoked by a nicotinic cholinergic transmission process. Blockade of transmission at the end plate (the postsynaptic structure bearing the nicotinic receptors) is clinically useful in producing muscle relaxation, a requirement for surgical relaxation, tracheal intubation, and control of ventilation. The neuromuscular blockers are quaternary amines structurally related to acetylcholine (ACh). Most are antagonists (nondepolarizing type), and the prototype is tubocurarine. One neuromuscular blocker used clinically, succinylcholine, is an agonist at the nicotinic end plate receptor (depolarizing type). Chronic use CNS action (baclofen, diazepam, tizanidine) Acute use (cyclobenzaprine) Muscle action (dantrolene) B. Nondepolarizing Neuromuscular Blocking Drugs 1. Pharmacokinetics—All agents are given parenterally. They are highly polar drugs and do not cross the blood-brain barrier. Drugs that are metabolized (eg, mivacurium, withdrawn in the USA) or eliminated in the bile (eg, rocuronium) have shorter durations of action (10–20 min) than those eliminated by the kidney (eg, metocurine, pancuronium, pipecuronium, and tubocurarine) which usually have durations of action of 35–60 min. In addition to hepatic metabolism, atracurium clearance involves rapid spontaneous breakdown (Hofmann elimination) to form laudanosine and other products. At high blood levels, laudanosine may cause seizures. Cisatracurium, a stereoisomer of atracurium, is also inactivated spontaneously but forms less laudanosine and 221 222 PART V Drugs That Act in the Central Nervous System High Yield Terms to Learn Depolarizing blockade Neuromuscular paralysis that results from persistent depolarization of the end plate (eg, by succinylcholine) Desensitization A phase of blockade by a depolarizing blocker during which the end plate repolarizes but is less than normally responsive to agonists (acetylcholine or succinylcholine) Malignant hyperthermia Hyperthermia that results from massive release of calcium from the sarcoplasmic reticulum, leading to uncontrolled contraction and stimulation of metabolism in skeletal muscle Nondepolarizing blockade Neuromuscular paralysis that results from pharmacologic antagonism at the acetylcholine receptor of the end plate (eg, by tubocurarine) Spasmolytic A drug that reduces abnormally elevated muscle tone (spasm) without paralysis (eg, baclofen, dantrolene) Stabilizing blockade Synonym for nonpolarizing blockade currently is one of the most commonly used muscle relaxants in clinical practice. 2. Mechanism of action—Nondepolarizing drugs prevent the action of ACh at the skeletal muscle end plate (Figure 27–1). They act as surmountable blockers. (That is, the blockade can be overcome by increasing the amount of agonist [ACh] in the synaptic cleft.) They behave as though they compete with ACh at the receptor, and their effect is reversed by cholinesterase inhibitors. Some drugs in this group may also act directly to plug the ion channel operated by the ACh receptor. Post-tetanic potentiation is preserved in the presence of these agents, but tension during the tetanus fades rapidly. See Table 27–1 for additional details. Larger muscles (eg, abdominal, diaphragm) are more resistant to neuromuscular blockade, but they recover more rapidly than smaller muscles (eg, facial, hand). Of the available nondepolarizing drugs, rocuronium (60–120 s) has the most rapid onset time. C. Depolarizing Neuromuscular Blocking Drugs 1. Pharmacokinetics—Succinylcholine is composed of 2 ACh molecules linked end to end. Succinylcholine is metabolized by a cholinesterase (butyrylcholinesterase or pseudocholinesterase) Agonist Closed normal Nondepolarizing blocker Closed blocked Open normal Depolarizing blocker Open blocked FIGURE 27–1 Drug interactions with the acetylcholine (ACh) receptor on the skeletal muscle end plate. Top: ACh, the normal agonist, opens the sodium channel. Bottom left: Nondepolarizing blockers bind to the receptor to prevent opening of the channel. Bottom right: Succinylcholine causes initial depolarization (fasciculation) and then persistent depolarization of the channel, which leads to muscle relaxation. (Reproduced, with permission, from Katzung BG, editor: Basic & Clinical Pharmacology, 12th ed. McGraw-Hill, 2012: Fig. 27–6.) CHAPTER 27 Skeletal Muscle Relaxants 223 TABLE 27–1 Comparison of a typical nondepolarizing neuromuscular blocker (rocuronium) and a depolarizing blocker (succinylcholine). Succinylcholine Process Rocuronium Phase I Phase II Administration of tubocurarine Additive Antagonistic Augmenteda Administration of succinylcholine Antagonistic Additive Augmenteda Effect of neostigmine Antagonistic Augmenteda Antagonistic Initial excitatory effect on skeletal muscle None Fasciculations None b Response to tetanic stimulus Unsustained (“fade”) Sustained Unsustained Post-tetanic facilitation Yes No Yes a It is not known whether this interaction is additive or synergistic (superadditive). b The amplitude is decreased, but the response is sustained. Adapted, with permission, from Katzung BG, editor: Basic & Clinical Pharmacology, 13th ed. McGraw-Hill, 2014. in the liver and plasma. It has a duration of action of only a few minutes if given as a single dose. Blockade may be prolonged in patients with genetic variants of plasma cholinesterase that metabolize succinylcholine very slowly. Such variant cholinesterases are resistant to the inhibitory action of dibucaine. Succinylcholine is not rapidly hydrolyzed by acetylcholinesterase. 2. Mechanism of action—Succinylcholine acts like a nicotinic agonist and depolarizes the neuromuscular end plate (Figure 27–1). The initial depolarization is often accompanied by twitching and fasciculations (prevented by pretreatment with small doses of a nondepolarizing blocker). Because tension cannot be maintained in skeletal muscle without periodic repolarization and depolarization of the end plate, continuous depolarization results in muscle relaxation and paralysis. Succinylcholine may also plug the end plate channels. When given by continuous infusion, the effect of succinylcholine changes from continuous depolarization (phase I) to gradual repolarization with resistance to depolarization (phase II) (ie, a curare-like block; see Table 27–1). D. Reversal of Blockade The action of nondepolarizing blockers is readily reversed by increasing the concentration of normal transmitter at the receptors. This is best accomplished by administration of cholinesterase inhibitors such as neostigmine or pyridostigmine. In contrast, the paralysis produced by the depolarizing blocker succinylcholine is increased by cholinesterase inhibitors during phase I. During phase II, the block produced by succinylcholine is usually reversible by cholinesterase inhibitors. Sugammadex, approved in Europe, is a novel chemical antagonist of rocuronium. E. Toxicity 1. Respiratory paralysis—The action of full doses of neuromuscular blockers leads directly to respiratory paralysis. If mechanical ventilation is not provided, the patient will asphyxiate. 2. Autonomic effects and histamine release—Autonomic ganglia are stimulated by succinylcholine and weakly blocked by tubocurarine. Succinylcholine activates cardiac muscarinic receptors, whereas pancuronium is a moderate blocking agent and causes tachycardia. Tubocurarine and mivacurium are the most likely of these agents to cause histamine release, but it may also occur to a slight extent with atracurium and succinylcholine. Vecuronium and several newer nondepolarizing drugs (cisatracurium, doxacurium, pipecuronium, rocuronium) have no significant effects on autonomic functions or histamine release. A summary of the autonomic effects of neuromuscular drugs is shown in Table 27–2. 3. Specific effects of succinylcholine—Muscle pain is a common postoperative complaint, and muscle damage may occur. Succinylcholine may cause hyperkalemia, especially in patients with burn or spinal cord injury, peripheral nerve dysfunction, or muscular dystrophy. Increases in intragastric pressure caused by fasciculations may promote regurgitation with possible aspiration of gastric contents. 4. Drug interactions—Inhaled anesthetics, especially isoflurane, strongly potentiate and prolong neuromuscular blockade. A rare interaction of succinylcholine with inhaled anesthetics can result in malignant hyperthermia (see Table 16-2). A very early sign of this potentially life-threatening condition is contraction of the jaw muscles (trismus). Aminoglycoside antibiotics and antiarrhythmic drugs may potentiate and prolong the relaxant action of neuromuscular blockers to a lesser degree. 5. Effects of aging and diseases—Older patients (>75 years) and those with myasthenia gravis are more sensitive to the actions of the nondepolarizing blockers, and doses should be reduced in these patients. Conversely, patients with severe burns or who suffer from upper motor neuron disease are less responsive to these agents, probably as a result of proliferation of extrajunctional nicotinic receptors. 224 PART V Drugs That Act in the Central Nervous System TABLE 27–2 Autonomic effects of neuromuscular drugs. Drug Effect on Autonomic Ganglia Effect on Cardiac Muscarinic Receptors Ability to Release Histamine Nondepolarizing Atracurium Cisatracurium Rocuronium Pancuronium Tubocurarine Vecuronium None None None None Weak block None None None Slight block Moderate block None None Slight None None None Moderate None Depolarizing Succinylcholine Stimulation Stimulation Slight Modified and reproduced with permission from Katzung BG, editor: Basic & Clinical Pharmacology, 12th ed. McGraw-Hill, 2012. SKILL KEEPER: AUTONOMIC CONTROL OF HEART RATE (SEE CHAPTER 6) Tubocurarine can block bradycardia caused by phenylephrine but has no effect on bradycardia caused by neostigmine. Explain! The Skill Keeper Answer appears at the end of the chapter. SPASMOLYTIC DRUGS Certain chronic diseases of the CNS (eg, cerebral palsy, multiple sclerosis, stroke) are associated with abnormally high reflex activity in the neuronal pathways that control skeletal muscle; the result is painful spasm. Bladder control and anal sphincter control are also affected in most cases and may require autonomic drugs for management. In other circumstances, acute injury or inflammation of muscle leads to spasm and pain. Such temporary spasm can sometimes be reduced with appropriate drug therapy. The goal of spasmolytic therapy in both chronic and acute conditions is reduction of excessive skeletal muscle tone without reduction of strength. Reduced spasm results in reduction of pain and improved mobility. A. Drugs for Chronic Spasm 1. Classification—The spasmolytic drugs do not resemble ACh in structure or effect. They act in the CNS and in one case in the skeletal muscle cell rather than at the neuromuscular end plate. The spasmolytic drugs used in treatment of the chronic conditions mentioned previously include diazepam, a benzodiazepine (see Chapter 22); baclofen, a γ-aminobutyric acid (GABA) agonist; tizanidine, a congener of clonidine; and dantrolene, an agent that acts on the sarcoplasmic reticulum of skeletal muscle. These agents are usually administered by the oral route. Refractory cases may respond to chronic intrathecal administration of baclofen. Botulinum toxin injected into selected muscles can reduce pain caused by severe spasm (see Chapter 6) and also has application for ophthalmic purposes and in more generalized spastic disorders (eg, cerebral palsy). Gabapentin and pregabalin, antiseizure drugs, have been shown to be effective spasmolytics in patients with multiple sclerosis. 2. Mechanisms of action—The spasmolytic drugs act by several mechanisms. Three of the drugs (baclofen, diazepam, and tizanidine) act in the spinal cord (Figure 27–2). Baclofen acts as a GABAB agonist at both presynaptic and postsynaptic receptors, causing membrane hyperpolarization. Presynaptically, baclofen, by reducing calcium influx, decreases the release of the excitatory transmitter glutamic acid; at postsynaptic receptors, baclofen facilitates the inhibitory action of GABA. Diazepam facilitates GABA-mediated inhibition via its interaction with GABAA receptors (see Chapter 22). Tizanidine, an imidazoline related to clonidine with significant α2 agonist activity, reinforces presynaptic inhibition in the spinal cord. All 3 drugs reduce the tonic output of the primary spinal motoneurons. Dantrolene acts in the skeletal muscle cell to reduce the release of activator calcium from the sarcoplasmic reticulum via interaction with the ryanodine receptor (RyR1) channel. Cardiac muscle and smooth muscle are minimally depressed. Dantrolene is also effective in the treatment of malignant hyperthermia, a disorder characterized by massive calcium release from the sarcoplasmic reticulum of skeletal muscle. Though rare, malignant hyperthermia can be triggered by general anesthesia protocols that include succinylcholine or tubocurarine (see Chapter 25). In this emergency condition, dantrolene is given intravenously to block calcium release (see Table 16-2). 3. Toxicity—The sedation produced by diazepam is significant but milder than that produced by other sedative-hypnotic drugs at doses that induce equivalent muscle relaxation. Baclofen causes somewhat less sedation than diazepam, and tolerance occurs with chronic use—withdrawal should be accomplished slowly. Tizanidine may cause asthenia, drowsiness, dry mouth, and hypotension. Dantrolene causes significant muscle weakness but less sedation than either diazepam or baclofen. CHAPTER 27 Skeletal Muscle Relaxants 225 Inhibitory interneuron Tizanidine Corticospinal pathway Baclofen α2 – – GABAB Glu GABA Motor neuron GABAB AMPA – α2 – Muscle GABAA – Dantrolene Benzodiazepines Action potentials FIGURE 27–2 Sites of spasmolytic action of benzodiazepines (GABAA), baclofen (GABAB), tizanidine (α2) in the spinal cord and dantrolene (skeletal muscle). AMPA, amino-hydroxyl-methyl-isosoxazole-proprionic acid, a ligand for a glutamate receptor subtype; Glu, glutamatergic neuron. (Reproduced, with permission, from Katzung BG, editor: Basic & Clinical Pharmacology, 12th ed. McGraw-Hill, 2012: Fig. 27–11). B. Drugs for Acute Muscle Spasm Many drugs (eg, cyclobenzaprine, metaxalone, methocarbamol, orphenadrine) are promoted for the treatment of acute spasm resulting from muscle injury. Most of these drugs are sedatives or act in the brain stem. Cyclobenzaprine, a typical member of this group, is believed to act in the brain stem, possibly by interfering with polysynaptic reflexes that maintain skeletal muscle tone. The drug is active by the oral route and has marked sedative and antimuscarinic actions. Cyclobenzaprine may cause confusion and visual hallucinations in some patients. None of these drugs used for acute spasm is effective in muscle spasm resulting from cerebral palsy or spinal cord injury. Patients with renal failure often have decreased levels of plasma cholinesterase, thus prolonging the duration of action of succinylcholine. QUESTIONS 1. Characteristics of phase I depolarizing neuromuscular blockade due to succinylcholine include (A) Easy reversibility with nicotinic receptor antagonists (B) Marked muscarinic blockade (C) Muscle fasciculations only in the later stages of block (D) Reversibility by acetylcholinesterase (AChE) inhibitors (E) Sustained tension during a period of tetanic stimulation Questions 2 and 3. A patient underwent a surgical procedure of 2 h. Anesthesia was provided by isoflurane, supplemented by intravenous midazolam and a nondepolarizing muscle relaxant. At the end of the procedure, a low dose of atropine was administered followed by pyridostigmine. 2. The main reason for administering atropine was to (A) Block cardiac muscarinic receptors (B) Enhance the action of pyridostigmine (C) Prevent spasm of gastrointestinal smooth muscle (D) Provide postoperative analgesia (E) Reverse the effects of the muscle relaxant 226 PART V Drugs That Act in the Central Nervous System 3. A muscarinic receptor antagonist would probably not be needed for reversal of the skeletal muscle relaxant actions of a nondepolarizing drug if the agent used was (A) Cisatracurium (B) Mivacurium (C) Pancuronium (D) Tubocurarine (E) Vecuronium 4. Which of the following drugs is the most effective in the emergency management of malignant hyperthermia? (A) Atropine (B) Dantrolene (C) Haloperidol (D) Succinylcholine (E) Vecuronium 5. The clinical use of succinylcholine, especially in patients with diabetes, is associated with (A) Antagonism by pyridostigmine during the early phase of blockade (B) Aspiration of gastric contents (C) Decreased intragastric pressure (D) Histamine release in a genetically determined population (E) Metabolism at the neuromuscular junction by acetylcholinesterase 6. Which drug (related to clonidine) is most often associated with hypotension? (A) Baclofen (B) Pancuronium (C) Succinylcholine (D) Tizanidine (E) Vecuronium 7. Regarding the spasmolytic drugs, which of the following statements is not accurate? (A) Baclofen acts on GABA receptors in the spinal cord to increase chloride ion conductance (B) Cyclobenzaprine decreases both oropharyngeal secretions and gut motility (C) Dantrolene has no significant effect on the release of calcium from cardiac muscle (D) Diazepam causes sedation at doses commonly used to reduce muscle spasms (E) Intrathecal use of baclofen is effective in some refractory cases of muscle spasticity 8. Which drug is most likely to cause hyperkalemia leading to cardiac arrest in patients with spinal cord injuries? (A) Baclofen (B) Dantrolene (C) Pancuronium (D) Succinylcholine (E) Vecuronium 9. Which drug has spasmolytic activity and could also be used in the management of seizures caused by overdose of a local anesthetic? (A) Baclofen (B) Cyclobenzaprine (C) Diazepam (D) Gabapentin (E) Tizanidine 10. Myalgias are a common postoperative complaint of patients who receive large doses of succinylcholine, possibly the result of muscle fasciculations caused by depolarization. Which drug administered in the operating room can be used to prevent postoperative pain caused by succinylcholine? (A) Atracurium (B) Baclofen (C) Dantrolene (D) Diazepam (E) Lidocaine ANSWERS 1. Phase I depolarizing blockade caused by succinylcholine is not associated with antagonism at muscarinic receptors, nor is it reversible with cholinesterase inhibitors. Muscle fasciculations occur at the start of the action of succinylcholine. The answer is E. 2. Acetylcholinesterase inhibitors used for reversing the effects of nondepolarizing muscle relaxants cause increases in ACh at all sites where it acts as a neurotransmitter. To offset the resulting side effects, including bradycardia, a muscarinic blocking agent is used concomitantly. Although atropine is effective, glycopyrollate is usually preferred because it lacks CNS effects. The answer is A. 3. One of the distinctive characteristics of pancuronium is that it can block muscarinic receptors, especially those in the heart. It has sometimes caused tachycardia and hypertension and may cause dysrhythmias in predisposed individuals. The answer is C. 4. Prompt treatment is essential in malignant hyperthermia to control body temperature, correct acidosis, and prevent calcium release. Dantrolene interacts with the RyR1 channel to block the release of activator calcium from the sarcoplasmic reticulum, which prevents the tension-generating interaction of actin with myosin. The answer is B. 5. Fasciculations associated with succinylcholine may increase intragastric pressure with possible complications of regurgitation and aspiration of gastric contents. The complication is more likely in patients with delayed gastric emptying such as those with esophageal dysfunction or diabetes. Histamine release resulting from succinylcholine is not genetically determined. The answer is B. 6. Tizanidine causes hypotension via α2-adrenoceptor activation, like its congener clonidine. Hypotension may occur with tubocurarine (not listed) due partly to histamine release and to ganglionic blockade. The answer is D. 7. Baclofen activates GABAB receptors in the spinal cord. However, these receptors are coupled to K+ channels (see Chapter 21). GABAA receptors in the CNS modulate chloride ion channels, an action facilitated by diazepam and other benzodiazepines. The answer is A. 8. Skeletal muscle depolarization by succinylcholine releases potassium from the cells, and the ensuing hyperkalemia can be life-threatening in terms of cardiac arrest. Patients most susceptible include those with extensive burns, spinal cord injuries, neurologic dysfunction, or intra-abdominal infection. The answer is D. CHAPTER 27 Skeletal Muscle Relaxants 9. Diazepam is both an effective antiseizure drug and a spasmolytic. The spasmolytic action of diazepam is thought to be exerted partly in the spinal cord because it reduces spasm of skeletal muscle in patients with cord transection. Cyclobenzaprine is used for acute local spasm and has no antiseizure activity. The answer is C. 10. The depolarizing action of succinylcholine at the skeletal muscle end plate can be antagonized by small doses of nondepolarizing blockers. To prevent skeletal muscle fasciculations and the resulting postoperative pain caused by succinylcholine, a small nonparalyzing dose of a nondepolarizing drug (eg, atracurium) is often given immediately before succinylcholine. The answer is A. SKILL KEEPER ANSWER: AUTONOMIC CONTROL OF HEART RATE (SEE CHAPTER 6) Reflex changes in heart rate involve ganglionic transmission. Activation of α1 receptors on blood vessels by phenylephrine elicits a reflex bradycardia because mean blood pressure is increased. One of the characteristic effects of tubocurarine is its block of autonomic ganglia; this action can interfere with reflex changes in heart rate. Tubocurarine would not prevent bradycardia resulting from neostigmine (an inhibitor of acetylcholinesterase) because this occurs via stimulation by ACh of cardiac muscarinic receptors. CHECKLIST When you complete this chapter, you should be able to: ❑ Describe the transmission process at the skeletal neuromuscular end plate and the points at which drugs can modify this process. ❑ Identify the major nondepolarizing neuromuscular blockers and 1 depolarizing neuromuscular blocker; compare their pharmacokinetics. ❑ Describe the differences between depolarizing and nondepolarizing blockers from the standpoint of tetanic and post-tetanic twitch strength. ❑ Describe the method of reversal of nondepolarizing blockade. ❑ List drugs for treatment of skeletal muscle spasticity and identify their sites of action and their adverse effects. 227 228 PART V Drugs That Act in the Central Nervous System DRUG SUMMARY TABLE: Skeletal Muscle Relaxants Subclass Mechanism of Action Receptor Interactions Pharmacokinetics Adverse Effects Agonist at ACh-N receptors causing initial twitch then persistent depolarization Stimulates ANS ganglia and M receptors Parenteral: short action, inactivated by plasma esterases Muscle pain, hyperkalemia, increased intragastric and intraocular pressure Competitive antagonists at skeletal muscle ACh-N receptors ANS ganglion block (tubocurarine) • Cardiac M block (pancuronium) Parenteral use, variable disposition • Spontaneous inactivation (atracurium, cisatracurium) • Plasma ChE (mivacurium) • Hepatic metabolism (rocuronium, vecuronium) • Renal elimination (doxacurium, pancuronium, tubocurarine) Histamine release (mivacurium, tubocurarine) • Laudanosine formation (atracurium) Muscle relaxation is potentiated by inhaled anesthetics, aminoglycosides and possibly quinidine Baclofen Facilitates spinal inhibition of motor neurons GABAB receptor activation: pre- and postsynaptic Oral; intrathecal for severe spasticity Sedation, muscle weakness Cyclobenzaprine (many others; see text) Inhibition of spinal stretch reflex Mechanism unknown Oral for acute muscle spasm due to injury or inflammation M block, sedation, confusion, and ocular effects Diazepam Facilitates GABA-ergic transmission in CNS GABAA receptor activation: postsynaptic Oral and parenteral for acute and chronic spasms Sedation, additive with other CNS depressants • abuse potential Tizanidine Pre- and postsynaptic inhibition α2 Agonist in spinal cord Oral for acute and chronic spasms Muscle weakness, sedation, hypotension Weakens muscle contraction by reducing myosin-actin interaction Blocks RyR1 Ca2+ channels in skeletal muscle Oral for acute and chronic spasms • IV for malignant hyperthermia Muscle weakness Depolarizing Succinylcholine Nondepolarizing d-Tubocurarine Atracurium Cisatracurium Mivacuriuma Rocuronium Vecuronium Centrally acting Direct-acting Dantrolene ACh, acetylcholine; ANS, autonomic nervous system; ChE, cholinesterase; M, muscarinic receptor; N, nicotinic receptor a Mivacurium is no longer available in the USA. C A P T E R 28 Drugs Used in Parkinsonism & Other Movement Disorders Movement disorders constitute a number of heterogeneous neurologic conditions with very different therapies. They include parkinsonism, Huntington’s disease, Wilson’s disease, and Gilles de la Tourette’s syndrome. Movement disorders, H including athetosis, chorea, dyskinesia, dystonia, tics, and tremor, can be caused by a variety of general medical conditions, neurologic dysfunction, and drugs. Drugs used in parkinsonism MAO Dopamine Dopamine inhibitors precursor agonists (levodopa) (bromocriptine, (selegiline) pramipexole) COMT inhibitors (entacapone) Muscarinic antagonists (benztropine) Drugs for other movement disorders Tremor (propranolol) Huntington’s & Tourette’s (haloperidol, tetrabenazine) PARKINSONISM A. Pathophysiology Parkinsonism (paralysis agitans) is a common movement disorder that involves dysfunction in the basal ganglia and associated brain structures. Signs include rigidity of skeletal muscles, akinesia (or bradykinesia), flat facies, and tremor at rest (mnemonic RAFT). 1. Naturally occurring parkinsonism—The naturally occurring disease is of uncertain origin and occurs with increasing frequency during aging from the fifth or sixth decade of life onward. Pathologic characteristics include a decrease in the levels of striatal dopamine Wilson’s disease (penicillamine) and the degeneration of dopaminergic neurons in the nigrostriatal tract that normally inhibit the activity of striatal GABAergic neurons (Figure 28–1). Most of the postsynaptic dopamine receptors on GABAergic neurons are of the D2 subclass (negatively coupled to adenylyl cyclase). The reduction of normal dopaminergic neurotransmission leads to excessive excitatory actions of cholinergic neurons on striatal GABAergic neurons; thus, dopamine and acetylcholine activities are out of balance in parkinsonism (Figure 28–1). 2. Drug-induced parkinsonism—Many drugs can cause parkinsonian symptoms; these effects are usually reversible. The most important drugs are the butyrophenone and phenothiazine 229 230 PART V Drugs That Act in the Central Nervous System High-Yield Terms to Learn Athetosis Involuntary slow writhing movements, especially severe in the hands; “mobile spasm” Chorea Irregular, unpredictable, involuntary muscle jerks that impair voluntary activity Dystonia Prolonged muscle contractions with twisting and repetitive movements or abnormal posture; may occur in the form of rhythmic jerks Huntington disease An inherited adult-onset neurologic disease characterized by dementia and bizarre involuntary movements Parkinsonism A progressive neurologic disease characterized by shufflinq gait, stooped posture, resting tremor, speech impediments, movement difficulties, and an eventual slowing of mental processes and dementia Tics Sudden coordinated abnormal movements, usually repetitive, especially about the face and head Tourette’s syndrome A neurologic disease of unknown cause that presents with multiple tics associated with snorting, sniffing, and involuntary vocalizations (often obscene) Wilson’s disease An inherited (autosomal recessive) disorder of copper accumulation in liver, brain, kidneys, and eyes; symptoms include jaundice, vomiting, tremors, muscle weakness, stiff movements, liver failure, and dementia antipsychotic drugs, which block brain dopamine receptors. At high doses, reserpine causes similar symptoms, presumably by depleting brain dopamine. MPTP (1-methyl-4-phenyl-1,2,3,6tetrahydropyridine), a by-product of the attempted synthesis of an illicit meperidine analog, causes irreversible parkinsonism through destruction of dopaminergic neurons in the nigrostriatal tract. Treatment with type B monoamine oxidase inhibitors (MAOIs) protects against MPTP neurotoxicity in animals. Normal Substantia nigra Corpus striatum Dopamine Acetylcholine GABA Parkinsonism Dopamine agonists + − Huntington’s disease Antimuscarinic drugs FIGURE 28–1 Schematic representation of the sequence of neurons involved in parkinsonism and Huntington’s chorea. Top: Neurons in the normal brain. Middle: Neurons in parkinsonism. The dopaminergic neuron is lost. Bottom: Neurons in Huntington’s disease. The GABAergic neuron is lost. (Reproduced, with permission, from Katzung BG, editor: Basic & Clinical Pharmacology, 9th ed. McGraw-Hill, 2004: Fig. 28–1). DRUG THERAPY OF PARKINSONISM Strategies of drug treatment of parkinsonism involve increasing dopamine activity in the brain, decreasing muscarinic cholinergic activity in the brain, or both. Although several dopamine receptor subtypes are present in the substantia nigra, the benefits of most antiparkinson drugs appear to depend on activation of the D2 receptor subtype. A. Levodopa 1. Mechanisms—Because dopamine has low bioavailability and does not readily cross the blood-brain barrier, its precursor, l-dopa (levodopa), is used. This amino acid enters the brain via an l-amino acid transporter (LAT) and is converted to dopamine by the enzyme aromatic l-amino acid decarboxylase (dopa decarboxylase), which is present in many body tissues, including the brain. Levodopa is usually given with carbidopa, a drug that does not cross the blood-brain barrier but inhibits dopa decarboxylase in peripheral tissues (Figure 28–2). With this combination, the plasma half-life is prolonged, lower doses of levodopa are effective, and there are fewer peripheral side effects. 2. Pharmacologic effects—Levodopa ameliorates the signs of parkinsonism, particularly bradykinesia; moreover, the mortality rate is decreased. However, the drug does not cure parkinsonism, and responsiveness fluctuates and gradually decreases with time, which may reflect progression of the disease. Clinical response fluctuations may, in some cases, be related to the timing of levodopa dosing. In other cases, unrelated to dosing, off-periods of akinesia may alternate over a few hours with on-periods of improved mobility but often with dyskinesias (on-off phenomena). In some case, off-periods may respond to apomorphine. Although drug holidays sometimes reduce toxic effects, they rarely affect response fluctuations. However, catechol-O-methyltransferase (COMT) inhibitors used adjunctively may improve fluctuations in levodopa responses in some patients (see below). CHAPTER 28 Drugs Used in Parkinsonism & Other Movement Disorders Pramipexole, ropinirole + Selegiline, rasagiline Bromocriptine, pergolide Dopamine receptors Tolcapone + – – MAO-B DOPAC + Dopamine COMT 3-MT DOPA decarboxylase L-DOPA L-amino acid transporter Brain Blood-brain barrier Periphery 3-OMD L-DOPA COMT Dopamine DOPA decarboxylase – – Entacapone, tolcapone Carbidopa Adverse effects 231 Bromocriptine has been used as an individual drug, in combinations with levodopa (and with anticholinergic drugs), and in patients who are refractory to or cannot tolerate levodopa. Common adverse effects include anorexia, nausea and vomiting, dyskinesias, and postural hypotension. Behavioral effects, which occur more commonly with bromocriptine than with newer dopamine agonists, include confusion, hallucinations, and delusions. Ergot-related effects include erythromelalgia and pulmonary infiltrates. Use of bromocriptine in patients with Parkinson’s disease has declined with the introduction of non-ergot dopamine receptor agonists. 2. Pramipexole—This non-ergot has high affinity for the dopamine D3 receptor. It is effective as monotherapy in mild parkinsonism and can be used together with levodopa in more advanced disease. Pramipexole is administered orally 3 times daily and is excreted largely unchanged in the urine. The dose of pramipexole may need to be reduced in renal dysfunction. Adverse effects include anorexia, nausea and vomiting, postural hypotension, and dyskinesias. Mental disturbances (confusion, delusions, hallucinations, impulsivity) are more common with pramipexole than with levodopa. In rare cases, an uncontrollable tendency to fall asleep may occur. The drug is contraindicated in patients with active peptic ulcer disease, psychotic illness, or recent myocardial infarction. Pramipexole may be neuroprotective because it is reported to act as a scavenger for hydrogen peroxide. FIGURE 28–2 Pharmacologic strategies for dopaminergic therapy of Parkinson’s disease. The actions of the drugs are described in the text. MAO, monoamine oxidase; COMT, catecholO-methyltransferase; DOPAC, dihydroxyphenylacetic acid; L-DOPA, levodopa; 3-OMD, 3-O-methyldopa. (Reproduced, with permission, from Katzung BG, editor: Basic & Clinical Pharmacology, 12th ed. McGraw-Hill, 2012: Fig. 28–5.) 3. Ropinirole—Another non-ergot, this drug has high affinity for the dopamine D2 receptor. It is effective as monotherapy and can be used with levodopa to smooth out response fluctuations. The standard form is given 3 times daily, but a prolonged release form can be taken once daily. Ropinirole is metabolized by hepatic CYP1A2, and other drugs metabolized by this isoform (eg, caffeine, warfarin) may reduce its clearance. Adverse effects and contraindications are similar to those of pramipexole. 3. Toxicity—Most adverse effects are dose dependent. Gastrointestinal effects include anorexia, nausea, and emesis and can be reduced by taking the drug in divided doses. Tolerance to the emetic action of levodopa usually occurs after several months. Postural hypotension is common, especially in the early stage of treatment. Other cardiac effects include tachycardia, asystole, and cardiac arrhythmias (rare). Dyskinesias occur in up to 80% of patients, with choreoathetosis of the face and distal extremities occurring most often. Some patients may exhibit chorea, ballismus, myoclonus, tics, and tremor. Behavioral effects may include anxiety, agitation, confusion, delusions, hallucinations, and depression. Levodopa is contraindicated in patients with a history of psychosis. 4. Apomorphine—A potent dopamine receptor agonist, apomorphine injected subcutaneously may provide rapid (within 10 min) but temporary relief (1–2 h) of “off-periods” of akinesia in patients on optimized dopaminergic therapy. Because of severe nausea, pretreatment for 3 days with antiemetics (eg, trimethobenzamide) is necessary. Other side effects of apomorphine include dyskinesias, hypotension, drowsiness, and sweating. B. Dopamine Agonists 1. Bromocriptine—An ergot alkaloid, bromocriptine acts as a partial agonist at dopamine D2 receptors in the brain. The drug increases the functional activity of dopamine neurotransmitter pathways, including those involved in extrapyramidal functions (Figure 28–2). Pergolide is similar. C. Monoamine Oxidase Inhibitors 1. Mechanism—Selegiline and rasagiline are selective inhibitors of monoamine oxidase type B, the form of the enzyme that metabolizes dopamine (Figure 28–2). Hepatic metabolism of selegiline results in the formation of desmethylselegiline (possibly neuroprotective) and amphetamine. 2. Clinical use—Selegiline has minimal efficacy in parkinsonism if given alone but can be used adjunctively with levodopa. Rasagiline is more potent and has been used as monotherapy in early symptomatic parkinsonism as well as in combinations with levodopa. 232 PART V Drugs That Act in the Central Nervous System 3. Toxicity and drug interactions—Adverse effects and interactions of monoamine oxidase inhibitors include insomnia, mood changes, dyskinesias, gastrointestinal distress, and hypotension. Combinations of these drugs with meperidine have resulted in agitation, delirium, and mortality. Selegiline has been implicated in the serotonin syndrome when used with serotonin selective reuptake inhibitors (SSRIs). D. Catechol-O-methyltransferase (COMT) Inhibitors 1. Mechanism of action—Entacapone and tolcapone are inhibitors of COMT, the enzyme in both the CNS and peripheral tissues (Figure 28–2) that converts levodopa to 3-O-methyldopa (3OMD). Increased plasma levels of 3OMD are associated with poor response to levodopa partly because the compound competes with levodopa for active transport into the CNS. Entacapone acts only in the periphery. 2. Clinical uses—The drugs are used as adjuncts to levodopacarbidopa, decreasing fluctuations, improving response, and prolonging “on-time.” Tolcapone is taken 3 times daily, entacapone 5 times daily. A formulation combining levodopa, carbidopa, and entacapone is available, simplifying the drug regimen. 3. Toxicity—Adverse effects related partly to increased levels of levodopa include dyskinesias, gastrointestinal distress, and postural hypotension. Levodopa dose reductions may be needed for the first few days of COMT inhibitor use. Other side effects include sleep disturbances and orange discoloration of the urine. Tolcapone increases liver enzymes and has caused acute hepatic failure, necessitating routine monitoring of liver function tests and signed patient consent for use in the United States. E. Amantadine 1. Mechanism of action—Amantadine enhances dopaminergic neurotransmission by unknown mechanisms that may involve increasing synthesis or release of dopamine or inhibition of dopamine reuptake. The drug also has muscarinic blocking actions. 2. Pharmacologic effects—Amantadine may improve bradykinesia, rigidity, and tremor but is usually effective for only a few weeks. Amantadine also has antiviral effects. 3. Toxicity—Behavioral effects include restlessness, agitation, insomnia, confusion, hallucinations, and acute toxic psychosis. Dermatologic reactions include livedo reticularis. Miscellaneous effects may include gastrointestinal disturbances, urinary retention, and postural hypotension. Amantadine also causes peripheral edema, which responds to diuretics. F. Acetylcholine-Blocking (Antimuscarinic) Drugs 1. Mechanism of action—The drugs (eg, benztropine, biperiden, orphenadrine) decrease the excitatory actions of cholinergic neurons on cells in the striatum by blocking muscarinic receptors. 2. Pharmacologic effects—These drugs may improve the tremor and rigidity of parkinsonism but have little effect on bradykinesia. They are used adjunctively in parkinsonism and also alleviate the reversible extrapyramidal symptoms caused by antipsychotic drugs. 3. Toxicity—CNS toxicity includes drowsiness, inattention, confusion, delusions, and hallucinations. Peripheral adverse effects are typical of atropine-like drugs. These agents exacerbate tardive dyskinesias that result from prolonged use of antipsychotic drugs. SKILL KEEPER: AUTONOMIC DRUG SIDE EFFECTS (SEE CHAPTERS 8 AND 9) Based on your understanding of the receptors affected by drugs used in Parkinson’s disease, what types of autonomic side effects can you anticipate? The Skill Keeper Answers appear at the end of the chapter. DRUG THERAPY OF OTHER MOVEMENT DISORDERS A. Physiologic and Essential Tremor Physiologic and essential tremor are clinically similar conditions characterized by postural tremor. The conditions may be accentuated by anxiety, fatigue, and certain drugs, including bronchodilators, tricyclic antidepressants, and lithium. They may be alleviated by β-blocking drugs including propranolol. Beta blockers should be used with caution in patients with heart failure, asthma, diabetes, or hypoglycemia. Metoprolol, a β1-selective antagonist, is also effective, and its use is preferred in patients with concomitant pulmonary disease. Antiepileptic drugs including gabapentin, primidone, and topiramate, as well as intramuscular injection of botulinum toxin, have also been used to treat essential tremor. B. Huntington’s Disease and Tourette’s Syndrome Huntington’s disease, an inherited disorder, results from a brain neurotransmitter imbalance such that GABA functions are diminished and dopaminergic functions are enhanced (Figure 28–1). There may also be a cholinergic deficit because choline acetyltransferase is decreased in the basal ganglia of patients with this disease. However, pharmacologic attempts to enhance brain GABA and acetylcholine activities have not been successful in patients with this disease. Drug therapy usually involves the use of amine-depleting drugs (eg, reserpine, tetrabenazine), the latter having less troublesome adverse effects. Dopamine receptor antagonists (eg, haloperidol, perphenazine) are also sometimes effective and olanzapine is also used. Tourette’s syndrome is a disorder of unknown cause that frequently responds to haloperidol and other dopamine D2 receptor blockers, including pimozide. Though less effective overall, carbamazepine, clonazepam, and clonidine have also been used. CHAPTER 28 Drugs Used in Parkinsonism & Other Movement Disorders C. Drug-Induced Dyskinesias Parkinsonism symptoms caused by antipsychotic agents (see Chapter 29) are usually reversible by lowering drug dosage, changing the therapy to a drug that is less toxic to extrapyramidal function, or treating with a muscarinic blocker. In acute dystonias, parenteral administration of benztropine or diphenhydramine is helpful. Levodopa and bromocriptine are not useful because dopamine receptors are blocked by the antipsychotic drugs. Tardive dyskinesias that develop from therapy with older antipsychotic drugs are possibly a form of denervation supersensitivity. They are not readily reversed; no specific drug therapy is available. D. Wilson’s Disease This recessively inherited disorder of copper metabolism results in deposition of copper salts in the liver and other tissues. Hepatic and neurologic damage may be severe or fatal. Treatment involves use of the chelating agent penicillamine (dimethylcysteine), which removes excess copper. Toxic effects of penicillamine include gastrointestinal distress, myasthenia, optic neuropathy, and blood dyscrasias. Trientine and tetrathiomolybdate have also been used. E. Restless Legs Syndrome This syndrome, of unknown cause, is characterized by an unpleasant creeping discomfort in the limbs that occurs particularly when the patient is at rest. The disorder is more common in pregnant women and in uremic and diabetic patients. Dopaminergic therapy is the preferred treatment, and both pramipexole and ropinirole are approved for this condition. Opioid analgesics, benzodiazepines, and certain anticonvulsants (eg, gabapentin) are also used. QUESTIONS Questions 1 and 2. Bradykinesia has made drug treatment necessary in a 60-year-old male patient with Parkinson’s disease, and therapy is to be initiated with levodopa. 1. Regarding the anticipated actions of levodopa, the patient would not be informed that (A) Dizziness may occur, especially when standing (B) He should take the drug in divided doses to avoid nausea (C) Livedo reticularis is a possible side effect (D) The drug will probably improve his symptoms for a period of time but not indefinitely (E) Uncontrollable muscle jerks may occur 2. The prescribing physician will (or should) know that levodopa (A) Causes fewer CNS side effects if given together with a drug that inhibits hepatic dopa decarboxylase (B) Fluctuates in its effectiveness with increasing frequency as treatment continues (C) Prevents extrapyramidal adverse effects of antipsychotic drugs (D) Protects against cancer in patients with melanoma (E) Has toxic effects, which include pulmonary infiltrates 233 3. Which statement about pramipexole is accurate? (A) Effectiveness in Parkinson’s disease requires its metabolic conversion to an active metabolite (B) It should not be administered to patients taking antimuscarinic drugs (C) Pramipexole causes less mental disturbances than levodopa (D) The drug selectively activates the dopamine D3 receptor subtype (E) Warfarin may enhance the actions of pramipexole 4. A patient with parkinsonism is being treated with levodopa. He suffers from irregular, involuntary muscle jerks that affect the proximal muscles of the limbs. Which of the following statements about these symptoms is accurate? (A) Coadministration of muscarinic blockers prevents the occurrence of dyskinesias during treatment with levodopa (B) Drugs that activate dopamine receptors can exacerbate dyskinesias in a patient taking levodopa (C) Dyskinesias are less likely to occur if levodopa is administered with carbidopa (D) Symptoms are likely to be alleviated by continued treatment with levodopa (E) The symptoms are usually reduced if the dose of levodopa is increased 5. A 51-year-old patient with parkinsonism is being maintained on levodopa-carbidopa with adjunctive use of low doses of tolcapone but continues to have off-periods of alkinesia. The most appropriate drug to “rescue” the patient but that will only provide temporary relief is (A) Apomorphine (B) Benztropine (C) Carbidopa (D) Pramipexole (E) Selegiline 6. Concerning the drugs used in parkinsonism, which statement is accurate? (A) Dopamine receptor agonists should never be used in Parkinson’s disease before a trial of levodopa (B) Levodopa causes mydriasis and may precipitate an acute attack of glaucoma (C) Selegiline is a selective inhibitor of COMT (D) The primary benefit of antimuscarinic drugs in parkinsonism is their ability to relieve bradykinesia (E) Therapeutic effects of amantadine continue for several years 7. A previously healthy 40-year-old woman begins to suffer from slowed mentation, lack of coordination, and brief writhing movements of her hands that are not rhythmic. In addition, she has delusions of being persecuted. The woman has no history of psychiatric or neurologic disorders. Although further diagnostic assessment should be made, it is very likely that the most appropriate drug for treatment will be (A) Amantadine (B) Bromocriptine (C) Diazepam (D) Haloperidol (E) Levodopa 234 PART V Drugs That Act in the Central Nervous System 8. With respect to pramipexole, which of the following is accurate? (A) Activates brain dopamine D3 receptors (B) Effective as monotherapy in mild parkinsonism (C) May cause postural hypotension (D) Not an ergot derivative (E) All of the above 9. Tolcapone may be of value in patients being treated with levodopa-carbidopa because it (A) Activates COMT (B) Decreases the formation of 3-O-methyldopa (C) Inhibits monoamine oxidase type A (D) Inhibits neuronal reuptake of dopamine (E) Releases dopamine from nerve endings 10. Which of the following drugs is most suitable for management of essential tremor in a patient who has pulmonary disease? (A) Diazepam (B) Levodopa (C) Metoprolol (D) Propranolol (E) Terbutaline ANSWERS 1. In prescribing levodopa, the patient should be informed about adverse effects, including gastrointestinal distress, postural hypotension, and dyskinesias. It is reasonable to advise the patient that therapeutic benefits cannot be expected to continue indefinitely. Livedo reticularis (a netlike rash) is an adverse effect of treatment with amantadine. The answer is C. 2. Levodopa causes less peripheral toxicity but more CNS or behavioral side effects when its conversion to dopamine is inhibited outside the CNS. The drug is not effective in antagonizing the akinesia, rigidity, and tremor caused by treatment with antipsychotic agents. Levodopa is a precursor of melanin and may activate malignant melanoma. Use of levodopa is not associated with pulmonary dysfunction. The answer is B. 3. Pramipexole is a dopamine D3 receptor activator and does not require bioactivation. It is excreted in unchanged form. Confusion, delusions, and hallucinations occur more frequently with dopamine receptor activators than with levodopa. The use of dopaminergic agents in combination with antimuscarinic drugs is common in the treatment of parkinsonism. Warfarin may enhance the action of ropinirole, another dopamine receptor agonist. The answer is D. 4. The form and severity of dyskinesias resulting from levodopa may vary widely in individual patients. Dyskinesias occur in up to 80% of patients receiving levodopa for long periods. With continued treatment, dyskinesias may develop at a dose of levodopa that was previously well tolerated. Muscarinic receptor blockers do not prevent their occurrence. They occur more commonly in patients treated with levodopa in combination with carbidopa or with other dopamine receptor agonists. The answer is B. 5. Apomorphine, via subcutaneous injection, is used for temporary relief of off-periods of akinesia (rescue) in parkinsonian patients on dopaminergic drug therapy. Pretreatment with the antiemetic trimethobenzamide for 3 days is essential to prevent severe nausea. The answer is A. 6. The non-ergot dopamine agonists (pramipexole, ropinirole) are commonly used prior to levodopa in mild parkinsonism. The mydriatic action of levodopa may increase intraocular pressure; the drug should be used cautiously in patients with open-angle glaucoma and is contraindicated in those with angle-closure glaucoma. Antimuscarinic drugs may improve the tremor and rigidity of parkinsonism but have little effect on bradykinesia. Selegiline is a selective inhibitor of MAO type B. Amantadine is effective for only a few weeks. The answer is B. 7. Although further diagnosis is desirable, choreoathetosis with decreased mental abilities and psychosis (paranoia) suggests that this patient has the symptoms of Huntington’s disease. Drugs that are partly ameliorative include agents that deplete dopamine (eg, tetrabenazine) or that block dopaminergic receptors (eg, haloperidol). The answer is D. 8. Pramipexole is a non-ergot agonist at dopamine receptors and has greater selectivity for D3 receptors in the striatum. Pramipexole (or the D2 receptor antagonist ropinirole) is often chosen for monotherapy of mild parkinsonism, and these drugs sometimes have value in patients who have become refractory to levodopa. Adverse effects of these drugs include dyskinesias, postural hypotension, and somnolence. The answer is E. 9. Tolcapone and entacapone are inhibitors of COMT used adjunctively in patients treated with levodopa-carbidopa. The drugs decrease the formation of 3-O-methyldopa (3-OMD) from levodopa. This improves patient response by increasing levodopa levels and by decreasing competition between 3-OMD and levodopa for active transport into the brain by l-amino acid carrier mechanism. The answer is B. 10. Increased activation of β adrenoceptors has been implicated in essential tremor, and management commonly involves administration of propranolol. However, the more selective β1 blocker metoprolol is equally effective and is more suitable in a patient with pulmonary disease. The answer is C. SKILL KEEPER ANSWER: AUTONOMIC DRUG SIDE EFFECTS (SEE CHAPTERS 8 AND 9) Pharmacologic strategy in Parkinson’s disease involves attempts to enhance dopamine functions or antagonize acetylcholine at muscarinic receptors. Thus, peripheral adverse effects must be anticipated. 1. Adverse effects referable to activation of peripheral dopamine (or adrenoceptors in the case of levodopa) include postural hypotension, tachycardia (possible arrhythmias), mydriasis, and emetic responses. 2. Adverse effects referable to antagonism of peripheral muscarinic receptors include dry mouth, mydriasis, urinary retention, and cardiac arrhythmias. CHAPTER 28 Drugs Used in Parkinsonism & Other Movement Disorders 235 CHECKLIST When you complete this chapter, you should be able to: ❑ Describe the neurochemical imbalance underlying the symptoms of Parkinson’s disease. ❑ Identify the mechanisms by which levodopa, dopamine receptor agonists, selegiline, tolcapone, and muscarinic blocking drugs alleviate parkinsonism. ❑ Describe the therapeutic and toxic effects of the major antiparkinsonism agents. ❑ Identify the compounds that inhibit dopa decarboxylase and COMT and describe their use in parkinsonism. ❑ Identify the chemical agents and drugs that cause parkinsonism symptoms. ❑ Identify the most important drugs used in the management of essential tremor, Huntington’s disease, drug-induced dyskinesias, restless legs syndrome, and Wilson’s disease. DRUG SUMMARY TABLE: Drugs Used for Movement Disorders Subclass Mechanism of Action Clinical Applications Pharmacokinetics Toxicities Levodopa (+/– carbidopa) Precursor of dopamine Carbidopa inhibits peripheral metabolism via dopa decarboxylase Primary drug used in Parkinson’s disease (PD) Oral COMT and MAO type B inhibitors diminish doses and prolong actions Duration of effects: 6–8 h GI upsets, dyskinesias, behavioral effects • on-off phenomena D2 agonists (apomorphine bromocriptine, and ropinirole) • D3 agonist (pramipexole) Pramipexole and ropinirole used as sole agents in early Parkinson’s disease and adjunct to L-dopa • apomorphine rescue therapy Oral • pramipexole: short half-life (tid dosing), renal elimination • Ropinirole, CYP1A2 metabolism • drug interactions possible Anorexia, nausea, constipation postural hypotension, dyskinesias, mental disturbances Inhibit MAO type B Rasagiline for early PD • Both drugs adjunctive with L-dopa Oral • half-lives permit bid dosing Serotonin syndrome with meperidine and possibly SSRIs and TCAs Block L-dopa metabolism in periphery (both) and CNS dopamine (tolcapone) Prolong L-dopa actions Oral Relates to increased levels of L-dopa Block muscarinic receptors Improve tremor and rigidity not bradykinesia Oral: once daily Typical atropine-like side effects Reduce symptom (eg, chorea) severity Oral (see Chapter 11) Depression, hypotension, sedation Oral (see Chapter 29) Extrapyramidal dysfunction Oral Extrapyramidal dysfunction Dopamine agonists Pramipexole Ropinirole Apomorphine Bromocriptine (rarely used) MAO inhibitors Rasagiline Selegiline COMT inhibitors Entacapone Tolcapone Antimuscarinic agents Benztropine, and others Drugs for Huntington’s disease Tetrabenazine, reserpine Deplete amines Haloperidol D2 antagonist Drugs for Tourette’s syndrome Haloperidol D2 receptor blocker Clonidine α2 blocker Reduce vocal and motor tic frequency and severity Oral COMT, catechol-O-methyltransferase; MAO, monoamine oxidase; SSRIs, selective serotonin reuptake inhibitors; TCAs, tricyclic antidepressants. C A P T E R 29 Antipsychotic Agents & Lithium The antipsychotic drugs (neuroleptics) are used in schizophrenia and are also effective in the treatment of other psychoses and agitated states. Older drugs have high affinity for dopamine D2 receptors, whereas newer antipsychotic drugs have greater affinity for serotonin 5-HT2 receptors. Although schizophrenia is not cured by drug therapy, the symptoms, including thought disorder, emotional withdrawal, and hallucinations or H delusions, may be ameliorated by antipsychotic drugs. Unfortunately, protracted therapy (years) is often needed and can result in severe toxicity in some patients. In bipolar affective disorder, although lithium has been the mainstay of treatment for many years, the use of newer antipsychotic agents and of several antiseizure drugs is increasing. Drugs for psychoses & bipolar disorders Antipsychotics Classic drugs Newer agents (D2 receptor affinity) (5HT2 receptor affinity) chlorpromazine fluphenazine haloperidol thioridazine trifluoperazine clozapine olanzapine quetiapine risperidone ziprasidone Bipolar drugs Classic drug lithium Newer agents carbamazepine clonazepam olanzapine valproic acid ANTIPSYCHOTIC DRUGS However, they are much more costly than the older drugs, most of which are prescribed generically. A. Classification The major chemical subgroups of older antipsychotic drugs are the phenothiazines (eg, chlorpromazine, thioridazine, fluphenazine), the thioxanthenes (eg, thiothixene), and the butyrophenones (eg, haloperidol). Newer second generation drugs of varied heterocyclic structure are also effective in schizophrenia, including clozapine, loxapine, olanzapine, risperidone, quetiapine, ziprasidone, and aripiprazole. In some cases, these atypical antipsychotic drugs may be somewhat more effective and less toxic than the older drugs. B. Pharmacokinetics The antipsychotic drugs are well absorbed when given orally, and because they are lipid soluble, they readily enter the central nervous system (CNS) and most other body tissues. Many are bound extensively to plasma proteins. These drugs require metabolism by liver enzymes before elimination and have long plasma half-lives that permit once-daily dosing. In some cases, other drugs that inhibit cytochrome P450 enzymes can prolong the half-lives of antipsychotic agents. Parenteral forms of some 236 CHAPTER 29 Antipsychotic Agents & Lithium agents (eg, fluphenazine, haloperidol) are available for both rapid initiation of therapy and depot treatment. 237 receptor-blocking actions, has virtually no affinity for D2 receptors. Most of the newer atypical drugs (eg, olanzapine, quetiapine, and risperidone) also have high affinity for 5-HT2A receptors, although they may also interact with D2 and other receptors. Ziprasidone is an antagonist at the D2, 5-HT2A, and 5-HT1D receptors and an agonist at the 5-HT1A receptor. The newer antipsychotic agent aripiprazole is a partial agonist at D2 and 5-HT1A receptors but is a strong antagonist at 5-HT2A receptors. The receptor-binding characteristics of the newer antipsychotic drugs have led to a serotonin hypothesis as an alternative to the dopamine hypothesis of the nature of schizophrenia. Most of the atypical drugs cause less extrapyramidal dysfunction than standard drugs. With the exception of haloperidol, all antipsychotic drugs block H1 receptors to some degree. C. Mechanism of Action 1. Dopamine hypothesis—The dopamine hypothesis of schizophrenia proposes that the disorder is caused by a relative excess of functional activity of the neurotransmitter dopamine in specific neuronal tracts in the brain. This hypothesis is based on several observations. First, many antipsychotic drugs block brain dopamine receptors (especially D2 receptors). Second, dopamine agonist drugs (eg, amphetamine, levodopa) exacerbate schizophrenia. Third, an increased density of dopamine receptors has been detected in certain brain regions of untreated schizophrenics. The dopamine hypothesis of schizophrenia is not fully satisfactory because antipsychotic drugs are only partly effective in most patients and many effective drugs have a higher affinity for other receptors, than for D2 receptors. Phencyclidine (PCP) causes a psychotic syndrome but has no effect on dopamine receptors. D. Effects Dopamine receptor blockade is the major effect that correlates with therapeutic benefit for older antipsychotic drugs. Dopaminergic tracts in the brain include the mesocortical-mesolimbic pathways (regulating mentation and mood), nigrostriatal tract (extrapyramidal function), tuberoinfundibular pathways (control of prolactin release), and chemoreceptor trigger zone (emesis). Mesocortical-mesolimbic dopamine receptor blockade presumably underlies antipsychotic effects, and a similar action on the chemoreceptor trigger zone leads to the useful antiemetic properties of some antipsychotic drugs. Adverse effects resulting from receptor blockade in the other dopaminergic tracts, a major problem with older antipsychotic drugs, include extrapyramidal dysfunction and hyperprolactinemia (see later discussion). Note that almost all antipsychotic agents block both α1 and histamine H1 receptors to some extent. The relative receptor-blocking actions of various antipsychotic drugs are shown in Table 29–1. 2. Dopamine receptors—Five different dopamine receptors (D1–D5) have been characterized. Each is G protein-coupled and contains 7 transmembrane domains. The D2 receptor, found in the caudate putamen, nucleus accumbens, cerebral cortex, and hypothalamus, is negatively coupled to adenylyl cyclase. The therapeutic efficacy of the older antipsychotic drugs correlates with their relative affinity for the D2 receptor. Unfortunately, there is also a correlation between blockade of D2 receptors and extrapyramidal dysfunction. 3. Other receptors—Most of the newer atypical antipsychotic agents have higher affinities for other receptors than for the D2 receptor. For example, α adrenoceptor-blocking action correlates well with antipsychotic effect for many of the drugs (Table 29–1). Clozapine, a drug with significant D4 and 5-HT2 TABLE 29–1 Relative receptor-blocking actions of neuroleptic drugs. Drug D2 Block D4 Block Alpha1 Block 5-HT2 Block M Block H1 Block Most phenothiazines and thioxanthines ++ − ++ + + + Thioridazine ++ − ++ + +++ + Haloperidol +++ − + − − − Clozapine − ++ ++ ++ ++ + Molindone ++ − + − + + Olanzapine + − + ++ + + Quetiapine + − + ++ + + Risperidone ++ − + ++ + + ++ − ++ ++ − + + + + ++ − + Ziprasidone a Aripiprazole a Partial agonist at D2 and 5-HT1A receptors and antagonist activity at 5-HT2A receptors. +, blockade; −, no effect. The number of plus signs indicates the intensity of receptor blockade. 238 PART V Drugs That Act in the Central Nervous System E. Clinical Use 1. Treatment of schizophrenia—Antipsychotic drugs reduce some of the positive symptoms of schizophrenia, including hyperactivity, bizarre ideation, hallucinations, and delusions. Consequently, they can facilitate functioning in both inpatient and outpatient environments. Beneficial effects may take several weeks to develop. Overall efficacy of the antipsychotic drugs is, for the most part, equivalent in terms of the management of the floridly psychotic forms of the illness, although individual patients may respond best to a specific drug. However, clozapine is effective in some schizophrenic patients resistant to treatment with other antipsychotic drugs. Older drugs are still commonly used, partly because of their low cost compared with newer agents. However, none of the traditional drugs has much effect on negative symptoms of schizophrenia. Newer atypical drugs are reported to improve some of the negative symptoms of schizophrenia, including emotional blunting, social withdrawal, and lack of motivation. 2. Other psychiatric and neurologic indications—The newer antipsychotic drugs are often used with lithium in the initial treatment of mania. Several second-generation drugs are approved for treatment of acute mania; two of these (aripiprazole and olanzapine) are approved for maintenance treatment of bipolar disorder. The antipsychotic drugs are also used in the management of psychotic symptoms of schizoaffective disorders, in Gilles de la Tourette syndrome, and for management of toxic psychoses caused by overdosage of certain CNS stimulants. Molindone is used mainly in Tourette’s syndrome; it is rarely used in schizophrenia. The newer atypical antipsychotics have also been used to allay psychotic symptoms in patients with Alzheimer’s disease and in parkinsonism. 3. Nonpsychiatric indications—With the exception of thioridazine, most phenothiazines have antiemetic actions; prochlorperazine is promoted solely for this indication. H1-receptor blockade, most often present in short side-chain phenothiazines, provides the basis for their use as antipruritics and sedatives and contributes to their antiemetic effects. F. Toxicity 1. Reversible neurologic effects—Dose-dependent extrapyramidal effects include a Parkinson-like syndrome with bradykinesia, rigidity, and tremor. This toxicity may be reversed by a decrease in dose and may be antagonized by concomitant use of muscarinic blocking agents. Extrapyramidal toxicity occurs most frequently with haloperidol and the more potent piperazine side-chain phenothiazines (eg, fluphenazine, trifluoperazine). Parkinsonism occurs infrequently with clozapine and is much less common with the newer drugs. Other reversible neurologic dysfunctions that occur more frequently with older agents include akathisia and dystonias; these usually respond to treatment with diphenhydramine or muscarinic blocking agents. 2. Tardive dyskinesias—This important toxicity includes choreoathetoid movements of the muscles of the lips and buccal cavity and may be irreversible. Tardive dyskinesias tend to develop after several years of antipsychotic drug therapy but have appeared as early as 6 mo. Antimuscarinic drugs that usually ameliorate other extrapyramidal effects generally increase the severity of tardive dyskinesia symptoms. There is no effective drug treatment for tardive dyskinesia. Switching to clozapine does not exacerbate the condition. Tardive dyskinesia may be attenuated temporarily by increasing neuroleptic dosage; this suggests that tardive dyskinesia may be caused by dopamine receptor sensitization. 3. Autonomic effects—Autonomic effects result from blockade of peripheral muscarinic receptors and α adrenoceptors and are more difficult to manage in elderly patients. Tolerance to some of the autonomic effects occurs with continued therapy. Of the older antipsychotic agents, thioridazine has the strongest autonomic effects and haloperidol the weakest. Clozapine and most of the atypical drugs have intermediate autonomic effects. Atropine-like effects (dry mouth, constipation, urinary retention, and visual problems) are often pronounced with the use of thioridazine and phenothiazines with aliphatic side chains (eg, chlorpromazine). These effects also occur with clozapine and most of the atypical drugs but not with ziprasidone or aripiprazole. CNS effects from block of M receptors may include a toxic confusional state similar to that produced by atropine and the tricyclic antidepressants. Postural hypotension caused by α blockade is a common manifestation of many of the older drugs, especially phenothiazines. In the elderly, measures must be taken to avoid falls resulting from postural fainting. The atypical drugs, especially clozapine and ziprasidone, also block α receptors and can cause orthostatic hypotension. Failure to ejaculate is common in men treated with the phenothiazines. 4. Endocrine and metabolic effects—Endocrine and metabolic effects include hyperprolactinemia, gynecomastia, the amenorrhea-galactorrhea syndrome, and infertility. Most of these adverse effects are predictable manifestations of dopamine D2 receptor blockade in the pituitary; dopamine is the normal inhibitory regulator of prolactin secretion. Elevated prolactin is prominent with risperidone. Significant weight gain and hyperglycemia due to a diabetogenic action occur with several of the atypical agents, especially clozapine and olanzapine. These effects may be especially problematic in pregnancy. Aripiprazole and ziprasidone have little or no tendency to cause hyperglycemia, hyperprolactinemia, or weight gain. 5. Neuroleptic malignant syndrome—Patients who are particularly sensitive to the extrapyramidal effects of antipsychotic drugs may develop a malignant hyperthermic syndrome. The symptoms include muscle rigidity, impairment of sweating, hyperpyrexia, and autonomic instability, which may be life threatening. Drug treatment involves the prompt use of dantrolene, diazepam, and dopamine agonists (see also Table 16-2). CHAPTER 29 Antipsychotic Agents & Lithium 239 TABLE 29–2 Adverse pharmacologic effects antipsychotic drugs. Type Manifestations Mechanism Autonomic nervous system Loss of accommodation, dry mouth, difficulty urinating, constipation, orthostatic hypotension, impotence, failure to ejaculate Muscarinic cholinoceptor blockade, α adrenoceptor (that’s an alpha) blockade Central nervous system Parkinson’s syndrome, akathisia, dystonias, tardive dyskinesia, toxic-confusional state Dopamine-receptor blockade, supersensitivity of dopamine receptors, muscarinic blockade Endocrine system Amenorrhea-galactorrhea, infertility, impotence Dopamine-receptor blockade resulting in hyperprolactinemia Other Weight gain Possibly combined H1 and 5-HT2 blockade Reproduced, with permission, from Katzung BG, editor: Basic & Clinical Pharmacology, 12th ed. McGraw-Hill, 2012. 6. Sedation—This is more marked with phenothiazines (especially chlorpromazine) than with other antipsychotics; this effect is usually perceived as unpleasant by nonpsychotic persons. Fluphenazine and haloperidol are the least sedating of the older drugs; aripiprazole appears to be the least sedating of the newer agents. 7. Miscellaneous toxicities—Visual impairment caused by retinal deposits has occurred with thioridazine; at high doses, this drug may also cause severe conduction defects in the heart resulting in fatal ventricular arrhythmias. Most of the atypicals, especially quetiapine and ziprasidone, prolong the QT interval of the electrocardiogram (ECG). Clozapine causes a small but important (1–2%) incidence of agranulocytosis and blood counts must be monitored; at high doses the drug has caused seizures. 8. Overdosage toxicity—Poisoning with antipsychotics other than thioridazine is not usually fatal, although the FDA has warned of an increased risk of death in elderly patients with dementia. Hypotension often responds to fluid replacement. Most neuroleptics lower the convulsive threshold and may cause seizures, which are usually managed with diazepam or phenytoin. Thioridazine (and possibly ziprasidone) overdose, because of cardiotoxicity, is more difficult to treat. safe dosage regimen. For acute symptoms, the target therapeutic plasma concentration is 0.8–1.2 mEq/L and for maintenance 0.4– 0.7 mEq/L. Plasma levels of the drug may be altered by changes in body water. Dehydration, or treatment with thiazides, nonsteroidal anti-inflammatory drugs (NSAIDs), angiotensin-converting enzyme inhibitors (ACEIs), and loop diuretics, may result in an increase of lithium in the blood to toxic levels. Caffeine and theophylline increase the renal clearance of lithium. B. Mechanism of Action The mechanism of action of lithium is not well defined. The drug inhibits several enzymes involved in the recycling of neuronal membrane phosphoinositides. This action may result in depletion of the second messenger source, phosphatidylinositol bisphosphate (PIP2), which, in turn, would decrease generation of inositol trisphosphate (IP3) and diacylglycerol (DAG). These second messengers are important in amine neurotransmission, including that mediated by central adrenoceptors and muscarinic receptors (Figure 29–1). Receptor PIP PIP2 G PLC PI LITHIUM & OTHER DRUGS USED IN BIPOLAR (MANIC-DEPRESSIVE) DISORDER Inositol IP3 IP1 IP2 − − Lithium is effective in treatment of the manic phase of bipolar disorder and continues to be used for acute-phase illness and for prevention of recurrent manic and depressive episodes. A. Pharmacokinetics Lithium is absorbed rapidly and completely from the gut. The drug is distributed throughout the body water and cleared by the kidneys at a rate one fifth that of creatinine. The half-life of lithium is about 20 h. Plasma levels should be monitored, especially during the first weeks of therapy, to establish an effective and DAG Effects Lithium FIGURE 29–1 Postulated effect of lithium on the inositol trisphosphate (IP3) and diacylglycerol (DAG) second messenger system. The schematic diagram shows the synaptic membrane of a neuron in the brain. PLC, phospholipase C; G, coupling protein; PI, PIP, PIP2, IP2, IP1, intermediates in the production of IP3. By interfering with this cycle, lithium may cause a use-dependent reduction of synaptic transmission. (Reproduced, with permission, from Katzung BG, editor: Basic & Clinical Pharmacology, 12th ed. McGraw-Hill, 2012: Fig. 29–4.) 240 PART V Drugs That Act in the Central Nervous System C. Clinical Use Lithium carbonate continues to be used for the treatment of bipolar disorder (manic-depressive disease) although other drugs including valproic acid and carbamazepine are equally effective (see text that follows). Maintenance therapy with lithium decreases manic behavior and reduces both the frequency and the magnitude of mood swings. Antipsychotic agents and/or benzodiazepines are commonly required at the initiation of treatment because both lithium and valproic acid have a slow onset of action. Olanzapine and quetiapine are both approved as monotherapy for acute mania. Although lithium has protective effects against suicide and self-harm, antidepressant drugs are often used concurrently during maintenance. Note that monotherapy with antidepressants can precipitate mania in bipolar patients. D. Toxicity Adverse neurologic effects of lithium include tremor, sedation, ataxia, and aphasia. Thyroid enlargement may occur, but hypothyroidism is rare. Reversible nephrogenic diabetes insipidus occurs commonly at therapeutic drug levels. Edema is a common adverse effect of lithium therapy; acneiform skin eruptions occur; and leukocytosis is always present. The issue of dysmorphogenesis is not settled. The use of lithium during pregnancy is thought to increase the incidence of congenital cardiac anomalies (Ebstein’s anomaly). Recent analyses suggest that the teratogenic risk is low, but in pregnancy it appears to contribute to low Apgar scores in the neonate. Consequently, lithium should be withheld 24–48 h before delivery, and its use is contraindicated in nursing mothers. E. Other Drugs Used in Bipolar Disorder The manic phase in bipolar disorder can be treated with antipsychotic drugs, and both olanzapine and quetiapine are approved as monotherapy for this indication. Several antiseizure drugs are used in bipolar disorder. Valproic acid has antimanic effects equivalent to those of lithium and is now widely used in the Unites States for this indication, often as a first choice in acute illness. Valproic acid may be effective in patients who fail to respond to lithium, and in some instances it has been used in combination with lithium. The antiseizure drugs carbamazepine and lamotrigine are also used both in acute mania and for prophylaxis in the depressive phase. For more information on antiseizure drugs, see Chapter 24. QUESTIONS 1. Which statement about the pathophysiologic basis of schizophrenia is most accurate? (A) All clinically effective antipsychotic drugs have high affinity for dopamine D2 receptors (B) Dopamine receptor-blocking drugs are used to alleviate psychotic symptoms in parkinsonism (C) Drug-induced psychosis can occur without activation of brain dopamine receptors (D) Serotonin receptors are present at lower than normal levels in the brains of untreated schizophrenics (E) The clinical potency of olanzapine correlates well with its dopamine receptor-blocking activity 2. Trifluoperazine was prescribed for a young male patient diagnosed as suffering from schizophrenia. He complains about the side effects of his medication. Which of the following is not likely to be on his list? (A) Constipation (B) Decreased libido (C) Excessive salivation (D) Postural hypotension 3. Which statement concerning the adverse effects of antipsychotic drugs is accurate? (A) Acute dystonic reactions occur commonly with olanzapine (B) Akathisias due to antipsychotic drugs are managed by increasing the drug dose (C) Blurring of vision and urinary retention are common adverse effects of haloperidol (D) Retinal pigmentation is a dose-dependent toxic effect of thioridazine (E) The late-occurring choreoathetoid movements caused by conventional antipsychotic drugs are alleviated by atropine 4. Haloperidol is not an appropriate drug for management of (A) Acute mania (B) Amenorrhea-galactorrhea syndrome (C) Phencyclidine intoxication (D) Schizoaffective disorders (E) Tourette’s syndrome 5. Which statement concerning the use of lithium in the treatment of bipolar affective disorder is accurate? (A) Ingestion of foods with high salt content enhances the toxicity of lithium (B) Lithium usually alleviates the manic phase of bipolar disorder within 12 h (C) Lithium dosage may need to be decreased in patients taking thiazides (D) Since lithium does not cross the placental barrier, it is safe in pregnancy (E) The elimination rate of lithium is equivalent to that of creatinine 6. A 30-year-old male patient is on drug therapy for a psychiatric problem. He complains that he feels “flat” and that he gets confused at times. He has been gaining weight and has lost his sex drive. As he moves his hands, you notice a slight tremor. He tells you that since he has been on medication he is always thirsty and frequently has to urinate. The drug he is most likely to be taking is (A) Carbamazepine (B) Clozapine (C) Lithium (D) Risperidone (E) Valproic acid 7. A young male patient recently diagnosed as schizophrenic develops severe muscle cramps with torticollis a short time after drug therapy is initiated with haloperidol. The best course of action would be to (A) Add risperidone to the drug regimen (B) Discontinue haloperidol and observe the patient (C) Give oral diphenhydramine (D) Inject benztropine (E) Switch the patient to fluphenazine CHAPTER 29 Antipsychotic Agents & Lithium 8. Which of the following drugs is established to be both effective and safe to use in a pregnant patient suffering from bipolar disorder? (A) Carbamazepine (B) Fluphenazine (C) Lithium (D) Olanzapine (E) Valproic acid 9. In comparing the characteristics of thioridazine with other older antipsychotic drugs, which of the following statements is accurate? (A) Most likely to cause extrapyramidal dysfunction (B) Least likely to cause urinary retention (C) Most likely to be safe in patients with history of cardiac arrhythmias (D) Most likely to cause ocular dysfunction (E) The safest antipsychotic drug in overdose 10. Which of the following drugs has a high affinity for 5-HT2 receptors in the brain, does not cause extrapyramidal dysfunction or hematotoxicity, but is reported to increase the risk of significant QT prolongation? (A) Clozapine (B) Haloperidol (C) Olanzapine (D) Valproic acid (E) Ziprasidone SKILL KEEPER : RECEPTOR MECHANISMS (SEE CHAPTERS 2, 6, AND 21) Antipsychotic drugs to varying degrees act as antagonists at several receptor types, including those for acetylcholine, dopamine, norepinephrine, and serotonin. What are the second-messenger systems for each of the following receptor subtypes that are blocked by antipsychotic drugs? 1. D2 2. M3 3. Alpha1 4. 5-HT2A The Skill Keeper Answers appear at the end of the chapter. 241 ANSWERS 1. Although most older antipsychotic drugs block D2 receptors, this action is not a requirement for antipsychotic action. Aripiprazole, clozapine, and most newer second-generation drugs have a very low affinity for such receptors, but a high affinity for serotonin 5-HT2 receptors. There are no reports of decreased serotonin receptors in the brains of schizophrenics. The CNS effects of phencyclidine (PCP) closely parallel an acute schizophrenic episode, but PCP has no actions on brain dopamine receptors. Dopamine receptor blockers cause extrapyramidal dysfunction. The answer is C. 2. Phenothiazines such as trifluoperazine cause sedation and are antagonists at muscarinic and α adrenoceptors. Postural hypotension, blurring of vision, and dry mouth are common autonomic adverse effects, as is constipation. Effects on the male libido may result from increased prolactin or from increased peripheral conversion of androgens to estrogens. The answer is C. 3. Olanzapine has minimal dopamine receptor–blocking action and is unlikely to cause acute dystonias. Muscarinic blockers such as atropine exacerbate tardive dyskinesias. Akathisias (uncontrollable restlessness) resulting from antipsychotic drugs may be relieved by a reduction in dosage. Retinal pigmentation may occur from treatment with thioridazine. The answer is D. 4. In addition to its use in schizophrenia and acute mania, haloperidol has been used in the management of intoxication due to phencyclidine (PCP) and in Tourette’s syndrome. Hyperprolactinemia and the amenorrhea-galactorrhea syndrome may occur as adverse effects during treatment with antipsychotic drugs, especially those like haloperidol that strongly antagonize dopamine receptors in the tuberoinfundibular tract. The answer is B. 5. Clinical effects of lithium are slow in onset and may not be apparent before 1 or 2 weeks of daily treatment. High urinary levels of sodium inhibit renal tubular reabsorption of lithium, thus decreasing its plasma levels. Lithium clearance is decreased by distal tubule diuretics (eg, thiazides) because natriuresis stimulates a reflex increase in the proximal tubule reabsorption of both lithium and sodium. Any drug that can cross the blood-brain barrier can cross the placental barrier! Teratogenic risk is low, but use of lithium during pregnancy may contribute to low Apgar score in the neonate. The answer is C. 6. Confusion, mood changes, decreased sexual interest, and weight gain are symptoms that may be unrelated to drug administration. On the other hand, psychiatric drugs are often responsible for such symptoms. Tremor and symptoms of nephrogenic diabetes insipidus are characteristic adverse effects of lithium that may occur at blood levels within the therapeutic range. The answer is C. 7. Acute dystonic reactions are usually very painful and should be treated immediately with parenteral administration of a drug that blocks muscarinic receptors such as benztropine. Adding risperidone is not protective, and fluphenazine is as likely as haloperidol to cause acute dystonia. Oral administration of diphenhydramine is a possibility, but the patient may find it difficult to swallow and it would take a longer time to act. The answer is D. 242 PART V Drugs That Act in the Central Nervous System 8. Carbamazepine and valproic acid are effective in bipolar disorder but are contraindicated in the pregnant patient because of possible effects on fetal development. Although the potential for dysmorphogenesis due to lithium is probably low, the most conservative approach would be to treat the patient with quetiapine or olanzapine. Fluphenazine has no proven efficacy in bipolar disorder. The answer is D. 9. Atropine-like side effects are more prominent with thioridazine than with other phenothiazines, but the drug is less likely to cause extrapyramidal dysfunction. The drug has quinidine-like actions on the heart and, in overdose, may cause arrhythmias and cardiac conduction block with fatality. At high doses, thioridazine causes retinal deposits, which in advanced cases resemble retinitis pigmentosa. The patient may complain of browning of vision. The answer is D. 10. Many of the newer antipsychotic drugs have a greater affinity for 5-HT2 receptors than dopamine receptors. However, because clozapine is hematotoxic, the choice comes down to olanzapine and ziprasidone, both of which block 5-HT receptors. Of the currently available atypical antipsychotic drugs, ziprasidone carries the greatest risk of QT prolongation. The answer is E. SKILL KEEPER ANSWERS: RECEPTOR MECHANISMS (SEE CHAPTERS 2, 6, AND 21) 1. 2. 3. 4. D2: Gi linked ↓ cAMP M3: Gq linked ↑ IP3 and DAG Alpha1: Gq linked ↑ IP3 and DAG 5-HT2A: Gq linked ↑ IP3 and DAG CHECKLIST When you complete this chapter, you should be able to: ❑ Describe the “dopamine hypothesis” of schizophrenia. ❑ Identify 4 receptors blocked by various antipsychotic drugs and name drugs that block each. ❑ Identify the established toxicities of each of the following drugs: chlorpromazine, clozapine, haloperidol, thioridazine, ziprasidone. ❑ Describe tardive dyskinesia and the neuroleptic malignant syndrome. ❑ Identify the distinctive pharmacokinetic features of lithium, and list its adverse effects and toxicities. ❑ List the alternative drugs used in bipolar disorder CHAPTER 29 Antipsychotic Agents & Lithium 243 DRUG SUMMARY TABLE: Antipsychotics and Lithium Subclass Mechanism of Action Effects Clinical Applications Pharmacokinetics and Interactions Toxicities Phenothiazines Chlorpromazine Fluphenazine Thioridazine Block of D2 receptors >> 5-HT2 receptors Block α, M, and H1 receptors • sedation, decreased seizure threshold Schizophrenia • bipolar disorder (manic phase), antiemesis, preop sedation Oral and parenteral forms, hepatic metabolism, long half-life Extensions of α- and M receptor-blocking actions • extrapyramidal dysfunction, tardive dyskinesias, hyperprolactinemia Block of D2 receptors >> 5-HT2 receptors Some α block • less M block and sedation than phenothiazines Schizophrenia; bipolar disorder (manic phase), Huntington’s chorea, Tourette’s syndrome Oral and parenteral forms • hepatic metabolism Extrapyramidal dysfunction (major) Block of 5-HT2 receptors >> D2 receptors Some α block (clozapine, risperidone, ziprasidone) and M block (clozapine, olanzapine), variable H1 block Schizophrenia (positive and negative symptoms) • bipolar disorder (olanzapine, risperidone), major depression (aripiprazole), agitation in Alzheimer’s and Parkinson’s Oral and parenteral forms • hepatic metabolism Agranulocytosis (clozapine) • diabetes and weight gain (clozapine, olanzapine), hyperprolactinemia (risperidone) • QT prolongation (ziprasidone) Uncertain, suppresses IP3 and DAG signaling No specific actions on ANS receptors or specific CNS receptors • no sedation Bipolar affective disorder • prevents mood swings (prophylaxis) Renal elimination, half-life 20 h • narrow therapeutic window— monitor blood levels • clearance decreased by thiazides and NSAIDs Tremor, edema, hypothyroidism, renal dysfunction • pregnancy category D Valproic acid competes with lithium as first choice in bipolar disorder, acute phase • others also used in acute phase and for prophylaxis in depressive phase Carbamazepine forms active metabolite (phase I); lamotrigine and valproic acid form conjugates (phase II) Hematotoxicity and induction of drug metabolism (carbamazepine) • rash (lamotrigine) • hepatic dysfunction, weight gain, and inhibition of drug metabolism (valproic acid) Thioxanthene Thiothixene Butyrophenone Haloperidol Atypicals Aripiprazole Clozapine Olanzapine Quetiapine Risperidone Ziprasidone Lithium Alternative drugs for bipolar affective disorder Carbamazepine Lamotrigine Valproic acid Unclear actions in bipolar disorder • see Chapter 24 for antiepileptic drug mechanism Ataxia and diplopia (carbamazepine) • nausea, dizziness, and headache (lamotrigine) • gastrointestinal distress, weight gain, alopecia (valproic acid) ANS, autonomic nervous system; DAG, diacylglycerol; 5-HT 2, serotonin type 2; IP 3, inositol trisphosphate NSAIDs, nonsteroidal anti-inflammatory drugs. C H A P T E R 30 Antidepressants Major depressive disorder, or endogenous depression, is a depression of mood without any obvious medical or situational causes, manifested by an inability to cope with ordinary events or experience pleasure. The drugs used in major depressive disorder are of varied chemical structures; many have effects that enhance the CNS actions of norepinephrine, serotonin, or both. Antidepressants MAO inhibitors (phenelzine, selegiline, tranylcypromine) Tricyclic antidepressants (amitriptyline, clomipramine, imipramine) Heterocyclic antidepressants (amoxapine, bupropion, mirtazapine) THE AMINE HYPOTHESIS OF MOOD The amine hypothesis of mood postulates that brain amines, particularly norepinephrine (NE) and serotonin (5-HT), are neurotransmitters in pathways that function in the expression of mood. According to the hypothesis, a functional decrease in the activity of such amines is thought to result in depression; a functional increase of activity results in mood elevation. The amine hypothesis is largely based on studies showing that many drugs capable of alleviating symptoms of major depressive disorders enhance the actions of the central nervous system (CNS) neurotransmitters 5-HT and NE. Difficulties with this hypothesis include the facts that (1) postmortem studies of patients do not reveal decreases in the brain levels of NE or 5-HT; (2) although antidepressant drugs may cause changes in brain amine activity within hours, clinical response requires weeks; (3) most antidepressants ultimately cause a downregulation of amine receptors; (4) bupropion has minimal effects on brain NE or 5-HT; (5) Brain-derived neurotrophic factor (BDNF) is depressed in the brains of depressed patients. 244 5-HT-NE reuptake inhibitors (duloxetine, venlafaxine) 5-HT antagonists (nefazodone, trazodone) Selective serotonin reuptake inhibitors (escitalopram, fluoxetine, fluvoxamine, paroxetine, sertraline) DRUG CLASSIFICATION & PHARMACOKINETICS A. Tricyclic Antidepressants Tricyclic antidepressants (TCAs; eg, imipramine, amitriptyline) are structurally related to the phenothiazine antipsychotics and share certain of their pharmacologic effects. The TCAs are well absorbed orally but may undergo first-pass metabolism. They have high volumes of distribution and are not readily dialyzable. Extensive hepatic metabolism is required before their elimination; plasma half-lives of 8–36 h usually permit once-daily dosing. Both amitriptyline and imipramine form active metabolites, nortriptyline and desipramine, respectively. B. Selective Serotonin Reuptake Inhibitors Fluoxetine is the prototype of a group of drugs that are selective serotonin reuptake inhibitors (SSRIs). All of them require hepatic metabolism and have half-lives of 18–24 h. However, fluoxetine forms an active metabolite with a half-life of several days (the basis CHAPTER 30 Antidepressants 245 High-Yield Terms to Learn Amine hypothesis of mood The hypothesis that major depressive disorders result from a functional deficiency of norepinephrine or serotonin at synapses in the CNS MAO inhibitors (MAOIs) Drugs inhibiting monoamine oxidases that metabolize norepinephrine and serotonin (MAO type A) and dopamine (MAO type B) Tricyclic antidepressants (TCAs) Structurally related drugs that block reuptake transporters of both norepinephrine (NE) and serotonin (5-HT) Selective serotonin reuptake inhibitors (SSRIs) Drugs that selectively inhibit serotonin (5-HT) transporters with only modest effects on other neurotransmitters Serotonin-norepinephrine reuptake inhibitors (SNRIs) Heterocyclic drugs that block NE and 5-HT transporters, but lack the alpha blocking, anticholinergic and antihistaminic actions of TCAs 5-HT2 receptor antagonists Structurally related drugs that block this subgroup of serotonin receptors with only minor effects on amine transporters Heterocyclics Term used for antidepressants of varying chemical structures, the characteristics of which do not strictly conform to any of the above designations for a once-weekly formulation). Other members of this group (eg, citalopram, escitalopram, fluvoxamine, paroxetine, and sertraline) do not form long-acting metabolites. C. Heterocyclics These drugs have varied structures and include the serotonin-norepinephrine reuptake inhibitors (SNRIs, duloxetine, venlafaxine, levomilnacipran), 5-HT2 receptor antagonists (nefazodone, trazodone) and miscellaneous other heterocyclic agents including amoxapine, bupropion, maprotiline, and mirtazapine. The pharmacokinetics of most of these agents are similar to those of the TCAs. Nefazodone and trazodone are exceptions; their half-lives are short and usually require administration 2 or 3 times daily. D. Monoamine Oxidase Inhibitors Monoamine oxidase inhibitors (MAOIs; eg, phenelzine, tranylcypromine) are structurally related to amphetamines and are orally active. The older, standard drugs inhibit both MAO-A (monoamine oxidase type A), which metabolizes NE, 5-HT, and tyramine, and MAO-B (monoamine oxidase type A), which metabolizes dopamine. Tranylcypromine is the fastest in onset of effect but has a shorter duration of action (about 1 week) than other MAOIs (2–3 weeks). In spite of these prolonged actions, the MAOIs are given daily. They are inhibitors of hepatic drug-metabolizing enzymes and cause drug interactions. Selegiline, a selective inhibitor of MAO type B, was recently approved for treatment of depression. MECHANISMS OF ANTIDEPRESSANT ACTION Potential sites of action of antidepressants at CNS synapses are shown in Figure 30–1. By means of several mechanisms, most antidepressants cause potentiation of the neurotransmitter actions of NE, 5-HT, or both. However, nefazodone and trazodone are weak inhibitors of NE and 5-HT transporters, and their main action appears to be antagonism of the 5-HT2A receptor. Long-term use of tricyclics and MAOIs, but not SSRIs, leads to downregulation of β receptors. A. TCAs The acute effect of tricyclic drugs is to inhibit the reuptake mechanisms (transporters) responsible for the termination of the synaptic actions of both NE and 5-HT in the brain. This presumably results in potentiation of their neurotransmitter actions at postsynaptic receptors. B. SSRIs The acute effect of SSRIs is a highly selective action on the serotonin transporter (SERT). SSRIs allosterically inhibit the transporter, binding at a site other than that of serotonin. They have minimal inhibitory effects on the NE transporter, or blocking actions on adrenergic and cholinergic receptors. C. SNRIs SNRIs bind to transporters for both serotonin and NE, presumably enhancing the actions of both neurotransmitters. Venlafaxine has less affinity for the NE transporter than desvenlafaxine or duloxetine. The SNRIs differ from the TCAs in lacking significant blocking effects on peripheral receptors including histamine H1, muscarinic, or α-adrenergic receptors. D. Serotonin 5-HT2 Receptor Antagonists The major antidepressant actions of nefazodone and trazodone appear to result from block of the 5-HT2A receptor, a G protein-coupled receptor located in several CNS regions including the neocortex. Antagonism of this receptor is associated with both the antianxiety and antidepressant actions of these drugs. 246 PART V Drugs That Act in the Central Nervous System Noradrenergic neuron Serotonergic neuron MAO inhibitors − − MAO Metabolites MAO Metabolites α2 receptor − Mirtazapine NE reuptake − Desipramine, maprotiline 5-HT reuptake NE receptor − 5-HT receptor Postsynaptic neuron Fluoxetine, trazodone FIGURE 30–1 Possible sites of action of antidepressant drugs. Inhibition of neuronal uptake of norepinephrine (NE) and serotonin (5-HT) increases the synaptic activities of these neurotransmitters. Inhibition of monoamine oxidase increases the presynaptic stores of both NE and 5-HT, which leads to increased neurotransmitter effects. Blockade of the presynaptic α2 autoreceptor prevents feedback inhibition of the release of NE. Note: These are acute actions of antidepressants. E. Other Heterocyclic Antidepressants Mirtazapine has a unique action to increase amine release from nerve endings by antagonism of presynaptic α2 adrenoceptors involved in feedback inhibition. The drug is also an antagonist at serotonin 5-HT2 receptors. The mechanism of antidepressant action of bupropion is unknown—the drug has no effect on either 5-HT or NE receptors or on amine transporters. F. MAOIs The MAOIs increase brain amine levels by interfering with their metabolism in the nerve endings, resulting in an increase in the vesicular stores of NE and 5-HT. When neuronal activity discharges the vesicles, increased amounts of the amines are released, presumably enhancing the actions of these neurotransmitters. PHARMACOLOGIC EFFECTS latter commonly prescribed for this purpose and as a sleeping aid. MAOIs, SSRIs, and bupropion are more likely to cause CNSstimulating effects. C. Muscarinic Receptor Blockade Antagonism of muscarinic receptors occurs with all tricyclics and is particularly marked with amitriptyline and doxepin (Table 30–1). Atropine-like adverse effects may also occur with nefazodone, amoxapine, and maprotiline. Atropine-like effects are minimal with the other heterocyclics, the SSRIs, and bupropion. D. Cardiovascular Effects Cardiovascular effects occur most commonly with tricyclics and include hypotension from α-adrenoceptor blockade and depression of cardiac conduction. The latter effect may lead to arrhythmias. There have been reports of cardiotoxicity with overdose of venlafaxine. A. Amine Uptake Blockade The drugs that block NE transporters in the CNS (eg, tricyclics, maprotiline, venlafaxine) also inhibit the reuptake of NE at nerve endings in the autonomic nervous system. Likewise, MAOIs increase NE in sympathetic nerve terminals. In both cases, this can lead to peripheral autonomic sympathomimetic effects. However, long-term use of MAOIs can decrease blood pressure. E. Seizures Because the convulsive threshold is lowered by TCAs and MAOIs, seizures may occur with overdoses of these agents. Overdoses of maprotiline and the SSRIs have also caused seizures. B. Sedation Sedation is a common CNS effect of tricyclic drugs and some heterocyclic agents, especially mirtazapine and the 5-HT2 receptor antagonists nefazodone and trazodone (Table 30–1), the A. Major Depressive Disorders Major depression is the primary clinical indication for antidepressant drugs. Patients typically vary in their responsiveness to individual agents. Because of more tolerable side effects and safety CLINICAL USES CHAPTER 30 Antidepressants 247 TABLE 30–1 Pharmacodynamic characteristics of selected antidepressants. Drug Sedation Muscarinic Receptor Block NE Reuptake Block 5-HT Reuptake Block Tricyclics Amitriptylinea +++ +++ + ++ Desipramine + + +++ + Doxepina + ++ +++ + Imipramine ++ ++ + ++ Nortriptyline ++ + ++ + SSRIs Citalopram, etc 0 0 0 +++ Heterocyclics—SNRIs Duloxetine 0 0 ++ +++ 0 0 + +++ + 0 0/+ 0 + ++ ++ 0 ++ 0 ++ 0 + + ++ + 0 0 ++ ++ + 0 Venlafaxine Heterocyclics—5-HT2 antagonists Nefazodone ++ Trazodone + Heterocyclics—other Amoxapine Bupropion Maprotiline Mirtazapineb SNRI, serotonin-norepinephrine reuptake inhibitor; SSRI, selective serotonin reuptake inhibitor. a Significant α1 antagonism. b Significant H1 and α2 antagonism. 0/+, minimal activity; +, mild activity; + +, moderate activity; + + +, high activity. in overdose (see later discussion), the newer drugs (SSRIs, SNRIs, 5-HT antagonists, and certain heterocyclics) are now the most widely prescribed agents. However, none of the newer antidepressants has been shown to be more effective overall than tricyclic drugs. As alternative agents, tricyclic drugs continue to be most useful in patients with psychomotor retardation, sleep disturbances, poor appetite, and weight loss. MAOIs are thought to be most useful in patients with significant anxiety, phobic features, and hypochondriasis. Selegiline, the MAO type B inhibitor used in parkinsonism (see Chapter 28), is now available in a skin-patch formulation for treatment of depression. SSRIs may decrease appetite; overweight patients often lose weight on these drugs, at least during the first 6–12 months of treatment. Concerns have been expressed that SSRIs, SNRIs, and newer heterocyclics may increase suicide risk in children and adolescents. B. Other Clinical Uses TCAs are also used in the treatment of bipolar affective disorders, acute panic attacks, phobic disorders (compare with alprazolam; Chapter 22), enuresis, attention deficit hyperkinetic disorder, and chronic pain states. The SNRIs (eg, duloxetine, venlafaxine) are effective in patients with neuropathic pain and fibromyalgia; duloxetine is also approved for the pain of diabetic neuropathy. Clomipramine and the SSRIs are effective in obsessive-compulsive disorders. SSRIs are approved for patients who suffer from generalized anxiety disorders, panic attacks, social phobias, post-traumatic stress disorder, bulimia, and premenstrual dysphoric disorder, and they may also be useful in the treatment of alcohol dependence. Bupropion is used for management of patients attempting to withdraw from nicotine dependence. TOXICITY & DRUG INTERACTIONS A. TCAs The adverse effects of TCAs are largely predictable from their pharmacodynamic actions. These include (1) excessive sedation, lassitude, fatigue, and, occasionally, confusion; (2) sympathomimetic effects, including tachycardia, agitation, sweating, and insomnia; (3) atropine-like effects; (4) orthostatic hypotension, electrocardiogram (ECG) abnormalities, and cardiomyopathies; (5) tremor and paresthesias; and (6) weight gain. Overdosage with tricyclics is extremely hazardous, and the ingestion of as little as a 2-week supply has been lethal. Manifestations include (1) agitation, delirium, neuromuscular irritability, convulsions, and coma; (2) respiratory depression and circulatory collapse; (3) hyperpyrexia; and (4) cardiac conduction defects and severe 248 PART V Drugs That Act in the Central Nervous System TABLE 30–2 Drug interactions involving antidepressants. Antidepressant Taken With Consequence Fluoxetine Lithium, TCAs, warfarin Increased blood levels of second drug Fluvoxamine Alprazolam, theophylline, TCAs, warfarin Increased blood levels of second drug MAO inhibitors SSRIs, sympathomimetics, tyramine-containing foods Hypertensive crisis, serotonin syndrome Nefazodone Alprazolam, triazolam Increased blood levels of second drug Paroxetine Theophylline, TCAs, warfarin Increased blood levels of second drug Sertraline TCAs, warfarin Increased effects of second drug TCAs Ethanol, sedative hypnotics Increased CNS depression MAO, monoamine oxidase; SSRIs, selective serotonin reuptake inhibitors; TCAs, tricyclic antidepressants. arrhythmias. The “3 Cs”—coma, convulsions, and cardiotoxicity— are characteristic. Tricyclic drug interactions (Table 30–2) include additive depression of the CNS with other central depressants, including ethanol, barbiturates, benzodiazepines, and opioids. Tricyclics may also cause reversal of the antihypertensive action of guanethidine by blocking its transport into sympathetic nerve endings. Less commonly, tricyclics may interfere with the antihypertensive actions of methylnorepinephrine (the active metabolite of methyldopa) and clonidine. B. SSRI Toxicity Fluoxetine and the other SSRIs may cause nausea, headache, anxiety, agitation, insomnia, and sexual dysfunction. Jitteriness can be alleviated by starting with low doses or by adjunctive use of benzodiazepines. Extrapyramidal effects early in treatment may include akathisia, dyskinesias, and dystonic reactions. Seizures are a consequence of gross overdosage. Cardiac effects of citalopram overdose include QT prolongation. A withdrawal syndrome has been described for SSRIs, which includes nausea, dizziness, anxiety, tremor, and palpitations. Certain SSRIs are inhibitors of hepatic cytochrome P450 isozymes, an action that has led to increased activity of other drugs, including TCAs and warfarin (Table 30–2). Fluoxetine inhibits CYP2D6 and to a lesser extent CYP3A4 isoforms; fluvoxamine inhibits CYP1A2 and paroxetine CYP2D6. Through its inhibition of CYP2D6, fluoxetine can increase plasma levels of several drugs including dextromethorphan, propranolol, tamoxifen, and the TCAs. Citalopram causes fewer drug interactions than other SSRIs. A serotonin syndrome was first described for an interaction between fluoxetine and an MAOI (see later discussion). This life-threatening syndrome includes severe muscle rigidity, myoclonus, hyperthermia, cardiovascular instability, and marked CNS stimulatory effects, including seizures. Drugs implicated include MAOIs, TCAs, dextromethorphan, meperidine, St John’s wort, and possibly illicit recreational drugs such as MDMA (“ecstasy”). Antiseizure drugs, muscle relaxants, and blockers of 5-HT receptors (eg, cyproheptadine) have been used in the management of the syndrome. C. Toxicity of SNRIs, 5-HT2 Antagonists, and Heterocyclic Drugs Mirtazapine causes weight gain and is markedly sedating, as is trazodone. Amoxapine, maprotiline, mirtazapine, and trazodone cause some autonomic effects. Amoxapine is also a dopamine receptor blocker and may cause akathisia, parkinsonism, and the amenorrhea-galactorrhea syndrome. Adverse effects of bupropion include anxiety, agitation, dizziness, dry mouth, aggravation of psychosis, and, at high doses, seizures. Seizures and cardiotoxicity are prominent features of overdosage with amoxapine and maprotiline. Venlafaxine causes a dose-dependent increase in blood pressure and has CNS stimulant effects similar to those of the SSRIs. Severe withdrawal symptoms can occur, even after missing a single dose of venlafaxine. Both nefazodone and venlafaxine are inhibitors of cytochrome P450 isozymes. Through its inhibitory action on CYP3A4, nefazodone enhances the actions of several drugs including carbamazepine, clozapine, HMG-CoA reductase inhibitors (“statins”), and TCAs. Though rare, nefazodone has caused life-threatening hepatotoxicity requiring liver transplantation. Duloxetine is also reported to cause liver dysfunction. D. MAOI Toxicity Adverse effects of the traditional MAOIs include hypertensive reactions in response to indirectly acting sympathomimetics, hyperthermia, and CNS stimulation leading to agitation and convulsions. Hypertensive crisis may occur in patients taking MAOIs who consume food that contains high concentrations of the indirect sympathomimetic tyramine. In the absence of indirect sympathomimetics, MAOIs typically lower blood pressure; overdosage with these drugs may result in shock, hyperthermia, and seizures. MAOIs administered together with SSRIs have resulted in the serotonin syndrome. CHAPTER 30 Antidepressants QUESTIONS 1. A 36-year-old woman presents with symptoms of major depression that are unrelated to a general medical condition, bereavement, or substance abuse. She is not currently taking any prescription or over-the-counter medications. Drug treatment is to be initiated with sertraline. In your information to the patient, you would tell her that (A) Sertraline may take 2 wk or more to become effective (B) It is preferable that she take the drug in the morning (C) Muscle cramps and twitches can occur (D) She should notify you if she anticipates using other prescription drugs (E) All of the above 2. Concerning the proposed mechanisms of action of antidepressant drugs, which statement is accurate? (A) Bupropion inhibits NE reuptake into nerve endings in the CNS (B) Chronic treatment with tricyclic antidepressants leads to downregulation of adrenoceptors in the CNS (C) Decreased levels of NE and 5-HT in cerebrospinal fluid is a characteristic of depressed patients before drug therapy (D) Nefazodone activates 5-HT receptors in the CNS (E) Selegiline selectively decreases the metabolism of serotonin 3. A 34-year-old male patient who was prescribed citalopram for depression has decided he wants to stop taking the drug. When questioned, he said that it was affecting his sexual performance. You ascertain that he is also trying to overcome his dependency on tobacco products. If you decide to reinstitute drug therapy in this patient, the best choice would be (A) Amitriptyline (B) Bupropion (C) Fluoxetine (D) Imipramine (E) Venlafaxine 4. Regarding the clinical use of antidepressant drugs, which statement is accurate? (A) Chronic use of serotonin-norepinephrine reuptake inhibitors (SNRIs) increases the activity of hepatic drug-metabolizing enzymes (B) In the treatment of major depressive disorders, citalopram is usually more effective than paroxetine (C) Tricyclics are highly effective in depressions with attendant anxiety, phobic features, and hypochondriasis (D) Weight gain often occurs during the first few months in patients taking SSRIs (E) When selecting an appropriate drug for treatment of depression, the history of patient response to specific drugs is a valuable guide 5. A patient under treatment for a major depressive disorder is brought to the emergency department after ingesting 30 times the normal daily therapeutic dose of imipramine. Which of the following would be least useful? (A) Administer bicarbonate and potassium chloride (to correct acidosis and hypokalemia) (B) Administer lidocaine (to control cardiac arrhythmias) (C) Initiate hemodialysis (to hasten drug elimination) (D) Maintain heart rhythm by electrical pacing (E) Use intravenous diazepam to control seizures 249 6. Which drug is an antagonist at 5-HT2 receptors and widely used for the management of insomnia? (A) Estazolam (B) Flurazepam (C) Trazodone (D) Triazolam (E) Zolpidem 7. A recently widowed 76-year-old female patient was treated with a benzodiazepine for several weeks after the death of her husband, but she did not like the daytime sedation it caused even at low dosage. Living independently, she has no major medical problems but appears rather infirm for her age and has poor eyesight. Because her depressive symptoms are not abating, you decide on a trial of an antidepressant medication. Which of the following drugs would be the most appropriate choice for this patient? (A) Amitriptyline (B) Citalopram (C) Mirtazapine (D) Phenelzine (E) Trazodone 8. SSRIs are much less effective than tricyclic antidepressants in the management of (A) Bulimia (B) Chronic pain of neuropathic origin (C) Generalized anxiety disorder (D) Obsessive-compulsive disorder (E) Premenstrual dysphoric disorder 9. Which of the following drugs is most likely to be of value in obsessive-compulsive disorders? (A) Amitriptyline (B) Bupropion (C) Clomipramine (D) Trazodone (E) Venlafaxine 10. To be effective in breast cancer, tamoxifen must be converted to an active form by CYP2D6. Cases of inadequate treatment of breast cancer have occurred when tamoxifen was administered to patients who were being treated with (A) Amitriptyline (B) Bupropion (C) Fluoxetine (D) Mirtazapine (E) Phenelzine ANSWERS 1. All the statements are appropriate regarding the initiation of treatment with sertraline or other SSRI in a depressed patient. The SSRIs have CNS-stimulating effects and may cause agitation, anxiety, “the jitters,” and insomnia, especially early in treatment. Consequently, the evening is not the best time to take SSRI drugs. The answer is E. 250 PART V Drugs That Act in the Central Nervous System 2. The mechanism of action of bupropion is unknown, but the drug does not inhibit either NE or 5-HT transporters. Levels of NE and 5-HT metabolites in the cerebrospinal fluid of depressed patients before drug treatment are not higher than normal. Selegiline is a selective inhibitor of MAO-B, the enzyme form that metabolizes dopamine (see Chapter 28). Nefazodone is a highly selective antagonist at the 5-HT2 receptor subtype. Downregulation of adrenoceptors appears to be a common feature of chronic treatment of depression with tricyclic drugs such as amitriptyline. The answer is B. 3. The SSRIs (eg, fluoxetine) and venlafaxine (an SNRI) can cause sexual dysfunction with decreased libido, erectile dysfunction, and anorgasmia. TCAs may also decrease libido or prevent ejaculation. Bupropion is the least likely antidepressant to affect sexual performance. The drug is also purportedly useful in withdrawal from nicotine dependence, which could be helpful in this patient. The answer is B. 4. No antidepressant has been shown to increase hepatic drug metabolism. MAO inhibitors (not TCAs), though now used infrequently, are the drugs most likely to be effective in depression with attendant anxiety, phobic features, and hypochondriasis. SSRIs are usually associated with weight loss, at least during the first 6 months of treatment. There is no evidence that any SSRI is more effective than another, or more effective overall than a tricyclic drug, in treatment of major depressive disorder. The answer is E. 5. Overdose with imipramine or any other tricyclic antidepressant drug is a medical emergency. The “3 Cs”—coma, convulsions, and cardiac problems—are the most common causes of death. Widening of the QRS complex on the ECG is a major diagnostic feature of cardiac toxicity. Arrhythmias resulting from cardiac toxicity require the use of drugs with the least effect on cardiac conductivity (eg, lidocaine). Hemodialysis does not increase the rate of elimination of tricyclic antidepressants in overdose. The answer is C. 6. All of the drugs listed are effective hypnotic drugs, but only trazodone is an antagonist at 5-HT2 receptors. Trazodone has wide use as a sleeping aid, especially in patients with symptoms of affective disorder. The answer is C. 7. Older patients are more likely to be sensitive to antidepressant drugs that cause sedation, atropine-like adverse effects, or postural hypotension. Tricyclics and MAO inhibitors cause many autonomic side effects; mirtazapine and trazodone are highly sedating. Citalopram (or another SSRI) is often the best choice in such patients. The answer is B. 8. The SSRIs are not effective in chronic pain of neuropathic origin. All the other uses of SSRIs are approved indications with clinical effectiveness equivalent or superior to that of tricyclic drugs. In addition to treatment of chronic pain states and depression the tricyclics are also used to treat enuresis and attention deficit hyperkinetic disorder. The answer is B. 9. Clomipramine, a tricyclic agent, is a more selective inhibitor of 5-HT reuptake than other drugs in its class. This activity appears to be important in the treatment of obsessive-compulsive disorder. However, the SSRIs have now become the drugs of choice for this disorder because they are safer in overdose than tricyclics. The answer is C. 10. Fluoxetine is an inhibitor of hepatic cytochrome P450s especially CYP2D6, and to a lesser extent CYP3A4. Dosages of several drugs may need to be reduced if given concomitantly with fluoxetine. In the case of tamoxifen, however, its antineoplastic action is dependent on its conversion to an active metabolite by CYP2D6. The answer is C. CHECKLIST When you complete this chapter, you should be able to: ❑ Describe the probable mechanisms of action and the major characteristics of TCAs, including receptor interactions, adverse effects (from chronic use and in overdose), drug interactions, and clinical uses. ❑ Identify the drugs classified as SSRIs and SNRIs, and describe their characteristics, including clinical uses, adverse effects and toxicity, and potential drug interactions. ❑ Identify drugs thought to act via block of serotonin receptors, and describe their characteristics including clinical uses, adverse effects and toxicity, and potential drug interactions. ❑ What are the major toxicities of MAO inhibitors? CHAPTER 30 Antidepressants 251 DRUG SUMMARY TABLE: Antidepressants Subclass Pharmacokinetics & Drug Interactions Mechanism of Action Clinical Applications Toxicities Block norepinephrine (NE) and 5-HT transporters Major depression (backup), chronic pain, obsessivecompulsive disorder (OCD)—clomipramine CYP substrates: interactions with inducers and inhibitors • Long half-lives α block, M block, sedation, weight gain • overdose: arrhythmias, seizures Major depression, anxiety disorders, OCD, PMDD, PTSD, bulimia, etc CYP 2D6 and 3A4 inhibition (fluoxetine, paroxetine) • 1A2 (fluvoxamine) Half-lives: 15+ h Sexual dysfunction Block NE and 5-HT transporters Major depression, chronic pain, fibromyalgia, menopausal symptoms Half-lives: 10 + h Anticholinergic, sedation, hypertension (venlafaxine) Block 5-HT2 receptors Major depression, hypnosis (trazodone) Usually require bid dosing • CYP3A4 inhibition (nefazodone) • Short half-lives Sedation • modest α and H1 blockade (trazodone) Mirtazepine blocks presynaptic α2 receptors • mechanism of action of others uncertain Major depression, smoking cessation (bupropion), sedation (mirtazepine) Extensive hepatic metabolism • CYP2D6 inhibition (bupropion) Lowers seizure threshold (amoxapine, bupropion) • sedation and weight gain (mirtazepine) Major depression unresponsive to other drugs Hypertension with tyramine and sympathomimetics • Serotonin syndrome with SSRIs • Very long half-lives Hypotension, insomnia Tricyclic antidepressants Amitriptyline, clomipramine, imipramine, etc Selective serotonin reuptake inhibitors (SSRIs) Citalopram, fluoxetine, paroxetine, sertraline, etc Block 5-HT transporters Serotonin-norepinephrine reuptake inhibitors (SNRIs) Venlafaxine, desvenlafaxine, duloxetine 5-HT2 antagonists Nefazodone, trazodone Other heterocyclics Amoxapine, bupropion, maprotiline, mirtazepine Monoamine oxidase inhibitors (MAOIs) Isocarboxazid, phenelzine, selegiline Inhibit MAO-A and MAO-B • selegiline more active vs MAO-B MAO-A, monoamine oxidase type A; MAO-B, monoamine oxidase type B; OD, overdose; PMDD, premenstrual dysphoric disorder; PTSD, post-traumatic stress disorder. C H A P T E R 31 Opioid Analgesics & Antagonists The opioids include natural opiates and semisynthetic alkaloids derived from the opium poppy, pharmacologically similar synthetic surrogates, and endogenous peptides. On the basis of their interaction with opioid receptors the drugs are classified as agonists, mixed agonist-antagonists, and antagonists. Opioid peptides released from nerve endings modulate transmission in the brain and spinal cord and in primary afferents via their interaction with specific receptors. Many of the pharmacologic actions of opiates and synthetic opioid drugs are effected via their interactions with endogenous opioid peptide receptors. Opioids Agonists Strong (morphine, methadone, meperidine) Mixed agonist-antagonists (buprenorphine, nalbuphine) Moderate (codeine, oxycodone) CLASSIFICATION The opioid analgesics and related drugs are derived from several chemical subgroups and may be classified in several ways. A. Spectrum of Clinical Uses Opioid drugs can be subdivided on the basis of their major therapeutic uses (eg, analgesics, antitussives, and antidiarrheal drugs). B. Strength of Analgesia On the basis of their relative abilities to relieve pain, the analgesic opioids may be classified as strong, moderate, and weak agonists. Partial agonists are opioids that exert less analgesia than morphine, the prototype of a strong analgesic, or full agonist. 252 Antagonists (naloxone, naltrexone) Weak (propoxyphene) C. Ratio of Agonist to Antagonist Effects Opioid drugs may be classified as agonists (full or partial receptor activators), antagonists (receptor blockers), or mixed agonistantagonists, which are capable of activating one opioid receptor subtype and blocking another subtype. PHARMACOKINETICS A. Absorption and Distribution Most drugs in this class are well absorbed when taken orally, but morphine, hydromorphone, and oxymorphone undergo extensive first-pass metabolism. In most cases, opioids can be given parenterally, and sustained-release forms of some drugs are now available, including morphine and oxycodone. Fentanyl is available as a transdermal patch. Opioid drugs are widely distributed to CHAPTER 31 Opioid Analgesics & Antagonists 253 High-Yield Terms to Learn Opiate Opioid Opioid peptides Opioid agonist Partial agonist Opioid antagonist Mixed agonist-antagonist A drug derived from alkaloids of the opium poppy The class of drugs that includes opiates, opiopeptins, and all synthetic and semisynthetic drugs that mimic the actions of the opiates Endogenous peptides that act on opioid receptors A drug that activates some or all opioid receptor subtypes and does not block any A drug that can activate an opioid receptor to effect a submaximal response A drug that blocks some or all opioid receptor subtypes A drug that activates some opioid receptor subtypes and blocks other opioid receptor subtypes body tissues. They cross the placental barrier and exert effects on the fetus that can result in both respiratory depression and, with continuous exposure, physical dependence in neonates. B. Metabolism With few exceptions, the opioids are metabolized by hepatic enzymes, usually to inactive glucuronide conjugates, before their elimination by the kidney. However, morphine-6-glucuronide has analgesic activity equivalent to that of morphine, and morphine-3-glucuronide (the primary metabolite) is neuroexcitatory. Codeine, oxycodone, and hydrocodone are metabolized by cytochrome CYP2D6, an isozyme exhibiting genotypic variability. In the case of codeine, this may be responsible for variability in analgesic response because the drug is demethylated by CYP2D6 to form the active metabolite, morphine. The ingestion of alcohol causes major increases in the peak serum levels of several opioids including hydromorphone and oxymorphone. Meperidine is metabolized to normeperidine, which may cause seizures at high plasma levels. Depending on the specific drug, the duration of their analgesic effects ranges from 1–2 h (eg, fentanyl) to 6–8 h (eg, buprenorphine). However, long-acting formulations of some drugs may provide analgesia for 24 h or more. The elimination half-life of opioids increases in patients with liver disease. Remifentanil, a congener of fentanyl, is metabolized by plasma and tissue esterases and has a very short half-life. MECHANISMS OF ACTION A. Receptors Many of the effects of opioid analgesics have been interpreted in terms of their interactions with specific receptors for endogenous peptides in the CNS and peripheral tissues. Certain opioid receptors are located on primary afferents and spinal cord pain transmission neurons (ascending pathways) and on neurons in the midbrain and medulla (descending pathways) that function in pain modulation (Figure 31–1). Other opioid receptors that may be involved in altering reactivity to pain are located on neurons in the basal ganglia, the hypothalamus, the limbic structures, and the cerebral cortex. Three major opioid receptor subtypes have been extensively characterized pharmacologically: µ, δ, and κ receptors. All 3 receptor subtypes appear to be involved in antinociceptive and analgesic mechanisms at both spinal and supraspinal levels. The µ-receptor activation plays a major role in the respiratory depressant actions of opioids and together with κ-receptor activation slows gastrointestinal transit; κ-receptor activation also appears to be involved in sedative actions; δ-receptor activation may play a role in the development of tolerance. B. Opioid Peptides Opioid receptors are thought to be activated by endogenous peptides under physiologic conditions. These peptides, which include endorphins such as a-endorphin, enkephalins, and dynorphins, are synthesized in the cell body and are transported to the nerve endings where they accumulate in synaptic vesicles. On release from nerve endings, they bind to opioid receptors and can be displaced from binding by opioid antagonists. Endorphins have highest affinity for µ receptors, enkephalins for δ receptors, and dynorphins for κ receptors. Although it remains unclear whether these peptides function as classic neurotransmitters, they appear to modulate transmission at many sites in the brain and spinal cord and in primary afferents. Opioid peptides are also found in the adrenal medulla and neural plexus of the gut. C. Ionic Mechanisms Opioid analgesics inhibit synaptic activity partly through direct activation of opioid receptors and partly through release of the endogenous opioid peptides, which are themselves inhibitory to neurons. All 3 major opioid receptors are coupled to their effectors by G proteins and activate phospholipase C or inhibit adenylyl cyclase. At the postsynaptic level, activation of these receptors can open potassium ion channels to cause membrane hyperpolarization (inhibitory postsynaptic potentials). At the presynaptic level, opioid receptor activation can close voltage-gated calcium ion channels to inhibit neurotransmitter release (Figure 31–2). Presynaptic actions result in the inhibition of release of multiple neurotransmitters, including acetylcholine (ACh), norepinephrine, serotonin, glutamate, and substance P. ACUTE EFFECTS A. Analgesia The opioids are the most powerful drugs available for the relief of pain. They attenuate both emotional and sensory aspects of the 254 PART V Drugs That Act in the Central Nervous System Transmission Modulation L Cortex SS Ventral C caudal thalamus Midbrain Amygdala D Periaqueductal gray Medulla/Pons Parabrachial nuclei E Rostral ventral medulla Dorsal horn B Spinal cord A FIGURE 31–1 Putative sites of action of opioid analgesics. On the left, sites of action on the pain transmission pathway from the periphery to the higher centers are shown. (A) Direct action of opioids on inflamed or damaged peripheral tissues. (B) Inhibition also occurs in the spinal cord. (C) Possible sites of action in the thalamus. Different thalamic regions project to somatosensory (SS) or limbic (L) cortex. Parabrachial nuclei (medulla/pons) project to the amygdala. On the right, actions of opioids on pain-modulating neurons in the midbrain (D), rostral ventral medulla (E), and the locus coeruleus indirectly control pain transmission pathways by enhancing descending inhibition to the dorsal horn. (Adapted, with permission, from Katzung BG, editor: Basic & Clinical Pharmacology, 12th ed. McGraw-Hill, 2012.) Primary afferent κ δ µ (Presynaptic)↓ Ca2+ influx, ↓ transmitter release pain experience. Strong agonists (ie, those with the highest analgesic efficacy, full agonists) include morphine, methadone, meperidine, fentanyl, levorphanol, and heroin. Drugs with mixed agonistantagonist actions (eg, buprenorphine, see below) may antagonize the analgesic actions of full agonists and should not be used concomitantly. Codeine, hydrocodone, and oxycodone are partial agonists with mild to moderate analgesic efficacy. They are commonly available in combinations with acetaminophen and nonsteroidal anti-inflammatory drugs (NSAIDs). Propoxyphene, a very weak agonist drug, is also available combined with acetaminophen. µ (Postsynaptic) ↑ K+ conductance, →IPSP Spinal paintransmission neuron FIGURE 31–2 Spinal sites of opioid action. The µ, κ, and δ agonists reduce excitatory transmitter release from presynaptic terminals of nociceptive primary afferents. The µ agonists also hyperpolarize second-order pain transmission neurons by increasing K+ conductance, evoking an inhibitory postsynaptic potential (IPSP). (Reproduced, with permission, from Katzung BG, editor: Basic & Clinical Pharmacology, 10th ed. McGraw-Hill, 2007.) B. Sedation and Euphoria These effects may occur at doses lower than those required for maximum analgesia. The sedation is additive with other CNS depressants, but there is little amnesia. Some patients experience dysphoric effects from opioid drugs. At higher doses, the drugs may cause mental clouding and result in a stuporous, or even a comatose, state. C. Respiratory Depression Opioid actions in the medulla lead to inhibition of the respiratory center, with decreased response to carbon dioxide challenge. CHAPTER 31 Opioid Analgesics & Antagonists With full agonists, respiratory depression may be seen at conventional analgesic doses. Increased Pco2 may cause cerebrovascular dilation, resulting in increased blood flow and increased intracranial pressure. Opioid analgesics are relatively contraindicated in patients with head injuries. D. Antitussive Actions Suppression of the cough reflex by unknown mechanisms is the basis for the clinical use of opioids as antitussives. This action can be obtained with the use of doses lower than those needed for analgesia. E. Nausea and Vomiting Nausea and vomiting are caused by opioid activation of the chemoreceptor trigger zone and are increased by ambulation. F. Gastrointestinal Effects Constipation occurs through decreased intestinal peristalsis, which is probably mediated by effects on opioid receptors in the enteric nervous system. This powerful action is the basis for the clinical use of these drugs as antidiarrheal agents. G. Smooth Muscle Opioids (with the exception of meperidine) cause contraction of biliary tract smooth muscle, which can result in biliary colic or spasm, increased ureteral and bladder sphincter tone, and a reduction in uterine tone, which may contribute to prolongation of labor. H. Miosis Pupillary constriction is a characteristic effect of all opioids except meperidine, which has a muscarinic blocking action. Little or no tolerance occurs. Miosis is blocked by the opioid antagonist naloxone and by atropine. I. Miscellaneous Opioid analgesics, especially morphine, can cause flushing and pruritus through histamine release. They cause release of antidiuretic hormone (ADH) and prolactin but may inhibit the release of luteinizing hormone (LH). Exaggerated responses to opioid analgesics may occur in patients with adrenal insufficiency or hypothyroidism. SKILL KEEPER: OPIOID PEPTIDES AND SUBSTANCE P (SEE CHAPTERS 6 AND 17) These peptides are relevant to understanding the analgesic actions of opioid-analgesic drugs in terms of CNS function. What are the roles of these peptides in peripheral tissues? The Skill Keeper Answers appear at the end of the chapter. 255 The mechanism of opioid tolerance development may involve receptor uncoupling. Antagonists of glutamate N-methyl-daspartate (NMDA) receptors (eg, ketamine), as well as δ-receptor antagonists, are reported to block opioid tolerance. Although there is cross-tolerance between different opioid agonists, it is not complete. This provides the basis for “opioid rotation,” whereby analgesia is maintained (eg, in cancer patients) by changing from one drug to another. B. Dependence Physical dependence is an anticipated physiologic response to chronic therapy with drugs in this group, particularly the strong agonists. Physical dependence is revealed on abrupt discontinuance as an abstinence syndrome, which includes rhinorrhea, lacrimation, chills, gooseflesh, muscle aches, diarrhea, yawning, anxiety, and hostility. A more intense state of precipitated withdrawal results when an opioid antagonist is administered to a physically dependent individual. CLINICAL USES A. Analgesia Treatment of relatively constant moderate to severe pain is the major indication. Although oral formulations are most commonly used, buccal and suppository forms of some drugs are available. In the acute setting, strong agonists are usually given parenterally. Prolonged analgesia, with some reduction in adverse effects, can be achieved with epidural administration of certain strong agonist drugs (eg, fentanyl and morphine). Fentanyl has also been used by the transdermal route providing analgesia for up to 72 h. For less severe pain and in the chronic setting, moderate agonists are given by the oral route, sometimes in combinations with acetaminophen or NSAIDs. B. Cough Suppression Useful oral antitussive drugs include codeine and dextromethorphan. The latter, an over-the-counter drug, has recently been the subject of FDA warnings regarding its abuse potential. Large doses of dextromethorphan may cause hallucinations, confusion, excitation, increased or decreased pupil size, nystagmus, seizures, coma, and decreased breathing. C. Treatment of Diarrhea Selective antidiarrheal opioids include diphenoxylate and loperamide. They are given orally. CHRONIC EFFECTS D. Management of Acute Pulmonary Edema Morphine (parenteral) may be useful in acute pulmonary edema because of its hemodynamic actions; its calming effects probably also contribute to relief of the pulmonary symptoms. A. Tolerance Marked tolerance can develop to the just-mentioned acute pharmacologic effects, with the exception of miosis and constipation. E. Anesthesia Opioids are used as preoperative medications and as intraoperative adjunctive agents in balanced anesthesia protocols. High-dose 256 PART V Drugs That Act in the Central Nervous System intravenous opioids (eg, morphine, fentanyl) are often the major component of anesthesia for cardiac surgery. F. Opioid Dependence Methadone, one of the longer acting opioids, is used in the management of opioid withdrawal states and in maintenance programs for addicts. In withdrawal states, methadone permits a slow tapering of opioid effect that diminishes the intensity of abstinence symptoms. Buprenorphine (see later discussion) has an even longer duration of action and is sometimes used in withdrawal states. In maintenance programs, the prolonged action of methadone blocks the euphoria-inducing effects of doses of shorter acting opioids (eg, heroin, morphine). TOXICITY Most of the adverse effects of the opioid analgesics (eg, nausea, constipation, respiratory depression) are predictable extensions of their pharmacologic effects. In addition, overdose and drug interaction toxicities are very important. A. Overdose A triad of pupillary constriction, comatose state, and respiratory depression is characteristic; the latter is responsible for most fatalities. Diagnosis of overdosage is confirmed if intravenous injection of naloxone, an antagonist drug, results in prompt signs of recovery. Treatment of overdose involves the use of antagonists such as naloxone and other therapeutic measures, especially ventilatory support. B. Drug Interactions The most important drug interactions involving opioid analgesics are additive CNS depression with ethanol, sedative-hypnotics, anesthetics, antipsychotic drugs, tricyclic antidepressants, and antihistamines. Concomitant use of certain opioids (eg, meperidine) with monoamine oxidase inhibitors increases the incidence of hyperpyrexic coma. Meperidine has also been implicated in the serotonin syndrome when used with selective serotonin reuptake inhibitors. Buprenorphine is a µ-receptor partial agonist with weak antagonist effects at κ and δ receptors. These characteristics can lead to decreased analgesia, or even precipitate withdrawal symptoms, when such drugs are used in patients taking conventional full µ-receptor agonists. Buprenorphine has a long duration of effect, binding strongly to µ receptors. Although prolonged activity of buprenorphine may be clinically useful (eg, to suppress withdrawal signs in dependency states), this property renders its effects resistant to naloxone reversal, since the antagonist drug has a short half-life. In overdose, respiratory depression caused by nalbuphine may also be resistant to naloxone reversal. Naloxone is included in some formulations of these agonist-antagonist drugs to discourage abuse. C. Effects The mixed agonist-antagonist drugs often cause sedation at analgesic doses. Dizziness, sweating, and nausea may also occur, and anxiety, hallucinations, and nightmares are possible adverse effects. Respiratory depression may be less intense than with pure agonists but is not predictably reversed by naloxone. Tolerance develops with chronic use but is less than the tolerance that develops to the full agonists, and there is minimal cross-tolerance. Physical dependence occurs, but the abuse liability of mixed agonist-antagonist drugs is less than that of the full agonists. D. Miscellaneous Tramadol is a weak µ-receptor agonist only partially antagonized by naloxone. Its analgesic activity is mainly based on blockade of the reuptake of serotonin; it is a weak norepinephrine reuptake blocker. Tramadol is effective in treatment of moderate pain and has been used as an adjunct to opioid analgesics in chronic pain syndromes. The drug is relatively contraindicated in patients with a history of seizure disorders, and there is risk of the serotonin syndrome if it is co-administered with SSRIs. No significant effects on cardiovascular functions or respiration have been reported. Tapentadol has strong norepinephrine reuptake-inhibiting activity (blocked by α antagonists) and only modest µ-opioid receptor affinity. It is less effective than oxycodone in the treatment of moderate to severe pain but causes less gastrointestinal distress and nausea. Tapentadol has been implicated in the serotonin syndrome and should be used with caution in seizure disorders. AGONIST-ANTAGONIST DRUGS A. Analgesic Activity The analgesic activity of mixed agonist-antagonists varies with the individual drug but is somewhat less than that of strong full agonists like morphine. Buprenorphine, butorphanol, and nalbuphine afford greater analgesia than pentazocine, which is similar to codeine in analgesic efficacy. B. Receptors Butorphanol, nalbuphine, and pentazocine are κ agonists, with weak µ-receptor antagonist activity. Butorphanol may act as a partial agonist or antagonist at the µ receptor. OPIOID ANTAGONISTS Naloxone, nalmefene, and naltrexone are pure opioid receptor antagonists that have few other effects at doses that produce marked antagonism of agonist effects. These drugs have greater affinity for µ receptors than for other opioid receptors. A major clinical use of the opioid antagonists is in the management of acute opioid overdose. Naloxone and nalmefene are given intravenously. Because naloxone has a short duration of action (1–2 h), multiple doses may be required in opioid analgesic overdose. Nalmefene has a duration of action of 8–12 h. Naltrexone has a long elimination half-life, blocking the actions of strong CHAPTER 31 Opioid Analgesics & Antagonists agonists (eg, heroin) for up to 48 h after oral use. Naltrexone decreases the craving for ethanol and is approved for adjunctive use in alcohol dependency programs. Unlike the older drugs, two new antagonists, methylnaltrexone and alvimopan, do not cross the blood-brain barrier. These agents block adverse effects of strong opioids on peripheral µ receptors, including those in the gastrointestinal tract responsible for constipation, with minimal effects on analgesic actions and without precipitating an abstinence syndrome. QUESTIONS Questions 1 and 2. A 63-year-old man is undergoing radiation treatment as an outpatient for metastatic bone cancer. His pain has been treated with a fixed combination of oxycodone plus acetaminophen taken orally. Despite increasing doses of the analgesic combination, the pain is getting worse. 1. The most appropriate oral medication for his increasing pain is (A) Buprenorphine (B) Codeine plus aspirin (C) Hydromorphone (D) Pentazocine (E) Tramadol 2. It is possible that this patient will have to increase the dose of the analgesic as his condition progresses as a result of developing tolerance. However, tolerance will not develop to a significant extent with respect to (A) Biliary smooth muscle (B) Emesis (C) Pupillary constriction (D) Sedation (E) Urinary retention 3. You are on your way to take an examination and you suddenly get an attack of diarrhea. If you stop at a nearby drugstore for an over-the-counter opioid with antidiarrheal action, you will be asking for (A) Codeine (B) Dextromethorphan (C) Diphenoxylate (D) Loperamide (E) Nalbuphine 4. An emergency department patient with severe pain thought to be of gastrointestinal origin received 80 mg of meperidine. He subsequently developed a severe reaction characterized by tachycardia, hypertension, hyperpyrexia, and seizures. Questioning revealed that the patient had been taking a drug for a psychiatric condition. Which drug is most likely to be responsible for this untoward interaction with meperidine? (A) Alprazolam (B) Bupropion (C) Lithium (D) Phenelzine (E) Mirtazapine 257 5. Genetic polymorphisms in certain hepatic enzymes involved in drug metabolism are established to be responsible for variations in analgesic response to (A) Buprenorphine (B) Codeine (C) Fentanyl (D) Methadone (E) Tramadol Questions 6 and 7. A young male patient is brought to the emergency department in an anxious and agitated state. He informs the attending physician that he uses “street drugs” and that he gave himself an intravenous “fix” approximately 12 h ago. He now has chills and muscle aches and has also been vomiting. His symptoms include hyperventilation and hyperthermia. The attending physician notes that his pupil size is larger than normal. 6. What is the most likely cause of these signs and symptoms? (A) The patient had injected dextroamphetamine (B) The patient has hepatitis B (C) The patient has overdosed with an opioid (D) The signs and symptoms are those of the opioid abstinence syndrome (E) These are early signs of toxicity due to contaminants in “street heroin” 7. Which drug will be most effective in alleviating the symptoms experienced by this patient? (A) Buprenorphine (B) Codeine (C) Methadone (D) Naltrexone (E) Tramadol 8. Which statement about nalbuphine is accurate? (A) Activates µ receptors (B) Does not cause respiratory depression (C) Is a nonsedating opioid (D) Pain-relieving action is not superior to that of codeine (E) Response to naloxone in overdose may be unreliable 9. Which drug does not activate opioid receptors, has been proposed as a maintenance drug in treatment programs for opioid addicts, and with a single oral dose, will block the effects of injected heroin for up to 48 h? (A) Fentanyl (B) Nalbuphine (C) Naloxone (D) Naltrexone (E) Propoxyphene 10. Which drug is a full agonist at opioid receptors with analgesic activity equivalent to morphine, a longer duration of action, and fewer withdrawal signs on abrupt discontinuance than morphine? (A) Fentanyl (B) Hydromorphone (C) Methadone (D) Nalbuphine (E) Oxycodone 258 PART V Drugs That Act in the Central Nervous System ANSWERS 1. In most situations, pain associated with metastatic carcinoma ultimately necessitates the use of an opioid analgesic that is equivalent in strength to morphine, so hydromorphone, oxymorphone, or levorphanol would be indicated. Pentazocine or the combination of codeine plus salicylate would not be as effective as the original drug combination. Propoxyphene is even less active than codeine alone. Buprenorphine, a mixed agonist-antagonist, is not usually recommended for cancerassociated pain because it has a limited maximum analgesic effect (“ceiling”) and because of possible dysphoric and psychotomimetic effects. The answer is C. 2. Chronic use of strong opioid analgesics leads to the development of tolerance to their analgesic, euphoric, and sedative actions. Tolerance also develops to their emetic effects and to effects on some smooth muscle, including the biliary and the urethral sphincter muscles. However, tolerance does not develop significantly to the constipating effects or the miotic actions of the opioid analgesics. The answer is C. 3. Codeine and nalbuphine could decrease gastrointestinal peristalsis, but not without marked side effects (and a prescription). Dextromethorphan is a cough suppressant. The other 2 drugs listed are opioids with antidiarrheal actions. Diphenoxylate is not available over the counter because it is a constituent of a proprietary combination that includes atropine sulfate (Lomotil). The answer is D. 4. Concomitant administration of meperidine and monoamine oxidase inhibitors such as isocarboxazid or phenelzine has resulted in life-threatening hyperpyrexic reactions that may culminate in seizures or coma. Such reactions have occurred even when the MAO inhibitor was administered more than a week after a patient had been treated with meperidine. Note that concomitant use of selective serotonin reuptake inhibitors and meperidine has resulted in the serotonin syndrome, another life-threatening drug interaction (see Chapter 16). The answer is D. 5. Codeine, hydrocodone, and oxycodone are metabolized by the cytochrome P450 isoform CYP2D6, and variations in analgesic response to these drugs have been attributed to genotypic polymorphisms in this isozyme. In the case of codeine, this may be especially important since the drug is demethylated by CYP2D6 to form the active metabolite, morphine (see Chapter 5). The answer is B. 6. The signs and symptoms are those of withdrawal in a patient physically dependent on an opioid agonist. They usually start within 6–10 h after the last dose; their intensity depends on the degree of physical dependence, and peak effects usually occur at 36–48 h. Mydriasis is a prominent feature of the abstinence syndrome; other symptoms include rhinorrhea, lacrimation, piloerection, muscle jerks, and yawning. The answer is D. 7. Prevention of signs and symptoms of withdrawal after chronic use of a strong opiate like heroin requires replacement with another strong opioid analgesic drug. Methadone is most commonly used, but other strong µ-receptor agonists would also be effective. Acetaminophen and codeine will not be effective. Beneficial effects of diazepam are restricted to relief of anxiety and agitation. The antagonist drug naltrexone may exacerbate withdrawal symptoms. The answer is C. 8. Nalbuphine and butorphanol are κ agonists, with weak µ-receptor antagonist activity. They have analgesic efficacy superior to that of codeine, but it is not equivalent to that of strong opioid receptor agonists. Although these mixed agonist-antagonist drugs are less likely to cause respiratory depression than strong µ activators, if depression does occur, reversal with opioid antagonists such as naloxone is unpredictable. Sedation is common. The answer is E. 9. The opioid antagonist naltrexone has a much longer half-life than naloxone, and its effects may last 2 d. A high degree of client compliance would be required for naltrexone to be of value in opioid dependence treatment programs. The same reservation is applicable to the use of naltrexone in alcoholism. The answer is D. 10. Fentanyl, hydromorphone, and methadone are full agonists with analgesic efficacy similar to that of morphine. When given intravenously, fentanyl has a duration of action of just 60–90 min. Hydromorphone has poor oral bioavailability. Methadone has the greatest bioavailability of the drugs used orally, and its effects are more prolonged. Tolerance and physical dependence develop, and dissipate, more slowly with methadone than with morphine. These properties underlie the use of methadone for detoxification and maintenance programs. The answer is C. SKILL KEEPER ANSWERS: OPIOID PEPTIDES AND SUBSTANCE P (SEE CHAPTERS 6 AND 17) 1. Precursor molecules that release opioid peptides are found at various peripheral sites, including the adrenal medulla and the pituitary gland and in some secretomotor neurons and interneurons in the enteric nervous system. In the gut these peptides appear to inhibit the release of ACh, presumably from parasympathetic nerve endings, and thereby inhibit peristalsis. In other tissues, opioid peptides may stimulate the release of transmitters or act as neurohormones. 2. Substance P, an undecapeptide, is a member of the tachykinin peptide group. It is an important sensory neuron transmitter in the enteric nervous system and in primary afferents involved in nociception. Substance P contracts intestinal and bronchiolar smooth muscle but is an arteriolar vasodilator (possibly via nitric oxide release). It may also play a role in renal and salivary gland functions. CHAPTER 31 Opioid Analgesics & Antagonists 259 CHECKLIST When you complete this chapter, you should be able to: ❑ Identify 3 opioid receptor subtypes and describe 2 ionic mechanisms that result from such activation. ❑ Name the major opioid agonists, rank them in terms of analgesic efficacy, and identify specific dynamic or kinetic characteristics. ❑ Describe the cardinal signs and treatment of opioid drug overdose and of the withdrawal syndrome. ❑ List acute and chronic adverse effects of opioid analgesics. ❑ Identify an opioid receptor antagonist and a mixed agonist-antagonist. ❑ Identify opioids used for antitussive effects and for antidiarrheal effects. DRUG SUMMARY TABLE: Opioids, Opioid Substitutes, & Opioid Antagonists Subclass Strong agonists Fentanyl, hydromorphone, meperidine, morphine, methadone, oxymorphone Partial agonists Codeine, hydrocodone Mechanism of Action (Receptors) Antagonists Naloxone, naltrexone, nalmefene Antitussives Codeine, dextromethorphan Tramadol Pharmacokinetics & Interactions Toxicities Strong µ agonists • variable δ and κ agonists Severe pain, anesthesia (adjunctive) • dependence maintenance (methadone) Hepatic metabolism • duration: 1–4 h (methadone 4–6 h) Respiratory depression, constipation, addiction liability As above, but lower affinity Mild-to-moderate pain; cough (codeine) • analgesic combinations with NSAIDs and acetaminophen Genetic variations in metabolism As above, but weaker Moderate-to-severe pain • dependence maintenance, reduces craving for alcohol (buprenorphine) Buprenorphine (long duration) • Nalbuphine (parenteral only) Like strong agonists but can antagonize their effects Antagonists at all opioid receptors Opioid overdose • dependence maintenance (naltrexone) Duration: naloxone 2 h • naltrexone and nalmefene >10 h Rapid antagonism of all opioid actions Mechanism uncertain • Weak µ agonist • inhibits norepinephrine and 5-HT transporters Acute debilitating cough Duration: 0.5–1 h Reduce cough reflex • toxic in overdose Weak µ agonist, blocks serotonin reuptake Moderate pain • adjunctive to opioids in chronic pain states Duration: 4–6 h Toxic in overdose (seizures) Mixed agonist-antagonist Buprenorphine Partial µ agonist and κ antagonist Nalbuphine Clinical Applications κ agonist and µ antagonist NSAIDs, nonsteroidal anti-inflammatory drugs. C Drugs of Abuse Drug abuse is usually taken to mean the use of an illicit drug or the excessive or nonmedical use of a licit drug. It also denotes the deliberate use of chemicals that generally are not considered drugs by the lay public but may be harmful to the user. A primary motivation for drug abuse appears to be the anticipated feeling of pleasure derived from the CNS effects of the drug. The older term “physical (physiologic) dependence” is now generally denoted as dependence, whereas “psychological dependence” is more simply called addiction. THE DOPAMINE HYPOTHESIS OF ADDICTION Dopamine in the ventral tegmental area and the nucleus accumbens of the mesolimbic system appears to play a primary role in the expression of “reward,” and excessive dopaminergic stimulation may lead to reinforcement such that the rewarded behavior may become compulsive—a common feature of addiction. Though not the only neurochemical characteristic of drugs of abuse, it appears that most addictive drugs have actions that include facilitation of the effects of dopamine in the CNS. SEDATIVE-HYPNOTICS The sedative-hypnotic drugs are responsible for many cases of drug abuse. The group includes ethanol, barbiturates, and benzodiazepines. Benzodiazepines are commonly prescribed drugs for anxiety and, as Schedule IV drugs, are judged by the US government to have low abuse liability (Table 32–1). Short-acting barbiturates (eg, secobarbital) have high addiction potential. Ethanol is not listed in schedules of controlled substances with abuse liability although it is clearly a heavily abused drug. A. Effects Sedative-hypnotics reduce inhibitions, suppress anxiety, and produce relaxation. All of these actions are thought to encourage repetitive use. Although the primary actions of sedative-hypnotics involve facilitation of the effects of GABA or antagonism at 260 H A P T E R 32 cholinergic nicotinic receptors, these drugs also enhance brain dopaminergic pathways, the latter action possibly related to the development of addiction. The drugs are CNS depressants, and their depressant effects are enhanced by concomitant use of opioid analgesics, antipsychotic agents, marijuana, and any other drug with sedative properties. Acute overdoses commonly result in death through depression of the medullary respiratory and cardiovascular centers (Table 32–2). Management of overdose includes maintenance of a patent airway plus ventilatory support. Flumazenil can be used to reverse the CNS depressant effects of benzodiazepines, but there is no antidote for barbiturates or ethanol. Flunitrazepam (Rohypnol), a potent rapid-onset benzodiazepine with marked amnestic properties, has been used in “date rape.” Added to alcoholic beverages, chloral hydrate or f-hydroxybutyrate (GHB; sodium oxybate) also renders the victim incapable of resisting rape. The latter compound, a minor metabolite of GABA, binds to GABAB receptors in the CNS. When used as a “club drug,” GHB causes euphoria, enhanced sensory perception, and amnesia. B. Withdrawal Physiologic dependence occurs with continued use of sedativehypnotics; the signs and symptoms of the withdrawal (abstinence) syndrome are most pronounced with drugs that have a half-life of less than 24 h (eg, ethanol, secobarbital, methaqualone). However, physiologic dependence may occur with any sedativehypnotic, including the longer acting benzodiazepines. The most important signs of withdrawal derive from excessive CNS stimulation and include anxiety, tremor, nausea and vomiting, delirium, and hallucinations (Table 32–2). Seizures are not uncommon and may be life-threatening. Treatment of sedative-hypnotic withdrawal involves administration of a long acting sedative-hypnotic (eg, chlordiazepoxide or diazepam) to suppress the acute withdrawal syndrome, followed by gradual dose reduction. Clonidine or propranolol may also be of value to suppress sympathetic overactivity. The opioid receptor antagonist naltrexone, and acamprosate, an antagonist at N-methyl-d-aspartate (NMDA) glutamate receptors, are both used in the treatment of alcoholism (see Chapter 23). A syndrome of therapeutic withdrawal has occurred on discontinuance of sedative-hypnotics after long-term therapeutic CHAPTER 32 Drugs of Abuse 261 High-Yield Terms to Learn Abstinence syndrome The signs and symptoms that occur on withdrawal of a drug in a dependent person Addiction Compulsive drug-using behavior in which the person uses the drug for personal satisfaction, often in the face of known risks to health; formerly termed psychological dependence Controlled substance A drug deemed to have abuse liability that is listed on governmental Schedules of Controlled Substances.a Such schedules categorize illicit drugs, control prescribing practices, and mandate penalties for illegal possession, manufacture, and sale of listed drugs. Controlled substance schedules are presumed to reflect current attitudes toward substance abuse; therefore, which drugs are regulated depends on a social judgment Dependence A state characterized by signs and symptoms, frequently the opposite of those caused by a drug, when it is withdrawn from chronic use or when the dose is abruptly lowered; formerly termed physical or physiologic dependence Designer drug A synthetic derivative of a drug, with slightly modified structure but no major change in pharmacodynamic action. Circumvention of the Schedules of Controlled Drugs is a motivation for the illicit synthesis of designer drugs Tolerance A decreased response to a drug, necessitating larger doses to achieve the same effect. This can result from increased disposition of the drug (metabolic tolerance), an ability to compensate for the effects of a drug (behavioral tolerance), or changes in receptor or effector systems involved in drug actions (functional tolerance) a An example of such a schedule promulgated by the US Drug Enforcement Agency is shown in Table 32–1. Note that the criteria given by the agency do not always reflect the actual pharmacologic properties of the drugs. administration. In addition to the symptoms of classic withdrawal presented in Table 32–2, this syndrome includes weight loss, paresthesias, and headache. (See Chapters 22 and 23 for additional details.) OPIOID ANALGESICS A. Effects As described in Chapter 31, the primary targets underlying the actions of the opioid analgesics are the µ, κ, and δ receptors. However, the opioids have other actions including disinhibition in dopaminergic pathways in the CNS. The most commonly abused drugs in this group are heroin, morphine, codeine, oxycodone, and among health professionals, meperidine and fentanyl. The effects of intravenous heroin are described by abusers as a “rush” or orgasmic feeling followed by euphoria and then sedation. Intravenous administration of opioids is associated with rapid development of tolerance, dependence, and addiction. Oral administration or smoking of opioids causes milder effects, with a slower onset of tolerance and dependence. Overdose of opioids leads to respiratory depression progressing to coma and death (Table 32–2). Overdose is managed with intravenous naloxone or nalmefene and ventilatory support. B. Withdrawal Deprivation of opioids in physiologically dependent individuals leads to an abstinence syndrome that includes lacrimation, rhinorrhea, yawning, sweating, weakness, gooseflesh (“cold turkey”), nausea and vomiting, tremor, muscle jerks (“kicking the habit”), and hyperpnea (Table 32–2). Although extremely unpleasant, TABLE 32–1 Schedules of controlled drugs.a Schedule Criteria Examples I No medical use; high addiction potential Flunitrazepam, heroin, LSD, mescaline, PCP, MDA, MDMA, STP II Medical use; high addiction potential Amphetamines, cocaine, methylphenidate, short acting barbiturates, strong opioids III Medical use; moderate abuse potential Anabolic steroids, barbiturates, dronabinol, ketamine, moderate opioid agonists IV Medical use; low abuse potential Benzodiazepines, chloral hydrate, mild stimulants (eg, phentermine, sibutramine), most hypnotics (eg, zaleplon, zolpidem), weak opioids a Adapted, with permission, from Katzung BG, editor: Basic & Clinical Pharmacology, 11th ed, McGraw-Hill, 2009. LSD, lysergic acid diethylamide; MDA, methylene dioxyamphetamine; MDMA, methylene dioxymethamphetamine; PCP, phencyclidine; STP (DOM), 2,5-dimethoxy4-methylamphetamine. 262 PART V Drugs That Act in the Central Nervous System TABLE 32–2 Signs and symptoms of overdose and withdrawal from selected drugs of abuse. Drug Overdose Effects Withdrawal Symptoms Amphetamines, methylphenidate, cocainea Agitation, hypertension, tachycardia, delusions, hallucinations, hyperthermia, seizures, death Apathy, irritability, increased sleep time, disorientation, depression Barbiturates, benzodiazepines, ethanolb Slurred speech, drunken behavior, dilated pupils, weak and rapid pulse, clammy skin, shallow respiration, coma, death Anxiety, insomnia, delirium, tremors, seizures, death Heroin, other strong opioids Constricted pupils, clammy skin, nausea, drowsiness, respiratory depression, coma, death Nausea, chills, cramps, lacrimation, rhinorrhea, yawning, hyperpnea, tremor a Cardiac arrhythmias, myocardial infarction, and stroke occur more frequently in cocaine overdose. b Ethanol withdrawal includes the excited hallucinatory state of delirium tremens. withdrawal from opioids is rarely fatal (unlike withdrawal from sedative-hypnotics). Treatment involves replacement of the illicit drug with a pharmacologically equivalent agent (eg, methadone), followed by slow dose reduction. Buprenorphine, a partial agonist at µ opioid receptors and a longer acting opioid (half-life >40 h), is also used to suppress withdrawal symptoms and as substitution therapy for opioid addicts. The administration of naloxone to a person who is using strong opioids (but not overdosing) may cause more rapid and more intense symptoms of withdrawal (precipitated withdrawal). Neonates born to mothers physiologically dependent on opioids require special management of withdrawal symptoms. STIMULANTS A. Caffeine and Nicotine 1. Effects—Caffeine (in beverages) and nicotine (in tobacco products) are legal in most Western cultures even though they have adverse medical effects. In the United States, cigarette smoking is a major preventable cause of death; tobacco use is associated with a high incidence of cardiovascular, respiratory, and neoplastic disease. Addiction (psychological dependence) to caffeine and nicotine has been recognized for some time. More recently, demonstration of abstinence signs and symptoms has provided evidence of dependence. 2. Withdrawal—Withdrawal from caffeine is accompanied by lethargy, irritability, and headache. The anxiety and mental discomfort experienced from discontinuing nicotine are major impediments to quitting the habit. Varenicline, a partial agonist at the α4β2 subtype nicotinic receptors, which occludes the rewarding effects of nicotine, is used for smoking cessation. Rimonabant, an agonist at cannabinoid receptors, approved for use in obesity, is also used off-label in smoking cessation. 3. Toxicity—Acute toxicity from overdosage of caffeine or nicotine includes excessive CNS stimulation with tremor, insomnia, and nervousness; cardiac stimulation and arrhythmias; and, in the case of nicotine, respiratory paralysis (Chapters 6 and 7). Severe toxicity has been reported in small children who ingest discarded nicotine gum or nicotine patches, which are used as substitutes for tobacco products. B. Amphetamines 1. Effects—Amphetamines alter transporters of CNS amines including dopamine, norepinephrine, and serotonin, and increase their release (Chapter 9). They cause a feeling of euphoria and self-confidence that contributes to the rapid development of addiction. Drugs in this class include dextroamphetamine and methamphetamine (“speed”), a crystal form of which (“ice”) can be smoked. Chronic high-dose abuse leads to a psychotic state (with delusions and paranoia) that is difficult to differentiate from schizophrenia. Symptoms of overdose include agitation, restlessness, tachycardia, hyperthermia, hyperreflexia, and possibly seizures (Table 32–2). There is no specific antidote, and supportive measures are directed toward control of body temperature and protection against cardiac arrhythmias and seizures. Chronic abuse of amphetamines is associated with the development of necrotizing arteritis, leading to cerebral hemorrhage and renal failure. 2. Tolerance and withdrawal—Tolerance can be marked, and an abstinence syndrome, characterized by increased appetite, sleepiness, exhaustion, and mental depression, can occur on withdrawal. Antidepressant drugs may be indicated. 3. Congeners of amphetamines—Several chemical congeners of amphetamines have hallucinogenic properties. These include 2,5-dimethoxy-4-methylamphetamine (DOM [STP]), methylene dioxyamphetamine (MDA), and methylene dioxymethamphetamine (MDMA; “ecstasy”). MDMA has a more selective action than amphetamine on the serotonin transporter in the CNS. The drug is purported to facilitate interpersonal communication and act as a sexual enhancer. Positron emission tomography studies of the brains of regular users of MDMA show a depletion of neurons in serotonergic tracts. Overdose toxicity includes hyperthermia, symptoms of the serotonin syndrome (see Chapter 30), and seizures. A withdrawal syndrome with protracted depression has been described in chronic users of MDMA. CHAPTER 32 Drugs of Abuse C. Cocaine 1. Effects—Cocaine, an inhibitor of the CNS transporters of dopamine, norepinephrine, and serotonin, has marked amphetamine-like effects (“super-speed”). Its abuse continues to be widespread in the United States partly because of the availability of a free-base form (“crack”) that can be smoked. The euphoria, self-confidence, and mental alertness produced by cocaine are short-lasting and positively reinforce its continued use. Overdoses with cocaine commonly result in fatalities from arrhythmias, seizures, or respiratory depression (see Table 32–2). Cardiac toxicity is partly due to blockade of norepinephrine reuptake by cocaine; its local anesthetic action contributes to the production of seizures. In addition, the powerful vasoconstrictive action of cocaine may lead to severe hypertensive episodes, resulting in myocardial infarcts and strokes. No specific antidote is available. Cocaine abuse during pregnancy is associated with increased fetal morbidity and mortality. 2. Withdrawal—The abstinence syndrome after withdrawal from cocaine is similar to that after amphetamine discontinuance. Severe depression of mood is common and strongly reinforces the compulsion to use the drug. Antidepressant drugs may be indicated. Infants born to mothers who abuse cocaine (or amphetamines) have possible teratogenic abnormalities (cystic cortical lesions) and increased morbidity and mortality and may be cocaine dependent. The signs and symptoms of CNS stimulant overdose and withdrawal are listed in Table 32–2. HALLUCINOGENS A. Phencyclidine The arylcyclohexylamine drugs include phencyclidine (PCP; “angel dust”) and ketamine (“special K”), which are antagonists at the glutamate NMDA receptor (Chapter 21). Unlike most drugs of abuse, they have no actions on dopaminergic neurons in the CNS. PCP is probably the most dangerous of the hallucinogenic agents. Psychotic reactions are common with PCP, and impaired judgment often leads to reckless behavior. This drug should be classified as a psychotomimetic. Effects of overdosage with PCP include both horizontal and vertical nystagmus, marked hypertension, and seizures, which may be fatal. Parenteral benzodiazepines (eg, diazepam, lorazepam) are used to curb excitation and protect against seizures. B. Miscellaneous Hallucinogenic Agents Several drugs with hallucinogenic effects have been classified as having abuse liability, including lysergic acid diethylamide (LSD), mescaline, and psilocybin. Hallucinogenic effects may also occur with scopolamine and other antimuscarinic agents. None of these drugs has actions on dopaminergic pathways in the CNS and, interestingly, they do not cause dependence. Terms that have been used to describe the CNS effects of such drugs include “psychedelic” and “mind revealing.” The perceptual and psychological effects of such drugs are usually accompanied by marked somatic effects, particularly nausea, weakness, and paresthesias. Panic reactions (“bad trips”) may also occur. 263 MARIJUANA A. Classification Marijuana (“grass”) is a collective term for the psychoactive constituents in crude extracts of the plant Cannabis sativa (hemp), the active principles of which include the cannabinoid compounds tetrahydrocannabinol (THC), cannabidiol (CBD), and cannabinol (CBN). Hashish is a partially purified material that is more potent. B. Cannabinoids Endogenous cannabinoids in the CNS, which include anandamide and 2-arachidonyl glycerol, are released postsynaptically and act as retrograde messengers to inhibit presynaptic release of conventional transmitters including dopamine. The receptors for these compounds are thought to be the targets for exogenous cannabinoids present in marijuana. C. Effects CNS effects of marijuana include a feeling of being “high,” with euphoria, disinhibition, uncontrollable laughter, changes in perception, and achievement of a dream-like state. Mental concentration may be difficult. Vasodilation occurs, and the pulse rate is increased. Habitual users show a reddened conjunctiva. A withdrawal state has been noted only in heavy users of marijuana. The dangers of marijuana use concern its impairment of judgment and reflexes, effects that are potentiated by concomitant use of sedative-hypnotics, including ethanol. Potential therapeutic effects of marijuana include its ability to decrease intraocular pressure and its antiemetic actions. Dronabinol (a controlledsubstance formulation of THC) is used to combat severe nausea. Rimonabant, an inverse agonist that acts as an antagonist at cannabinoid receptors, is approved for use in the treatment of obesity. INHALANTS Certain gases or volatile liquids are abused because they provide a feeling of euphoria or disinhibition. A. Anesthetics This group includes nitrous oxide, chloroform, and diethylether. Such agents are hazardous because they affect judgment and induce loss of consciousness. Inhalation of nitrous oxide as the pure gas (with no oxygen) has caused asphyxia and death. Ether is highly flammable. B. Industrial Solvents Solvents and a wide range of volatile compounds are present in commercial products such as gasoline, paint thinners, aerosol propellants, glues, rubber cements, and shoe polish. Because of their ready availability, these substances are most frequently abused by children in early adolescence. Active ingredients that have been identified include benzene, hexane, methylethylketone, toluene, and trichloroethylene. Many of these are toxic to the liver, 264 PART V Drugs That Act in the Central Nervous System kidneys, lungs, bone marrow, and peripheral nerves and cause brain damage in animals. C. Organic Nitrites Amyl nitrite, isobutyl nitrite, and other organic nitrites are referred to as “poppers” and are mainly used as sexual intercourse enhancers. Inhalation of the nitrites causes dizziness, tachycardia, hypotension, and flushing. With the exception of methemoglobinemia, few serious adverse effects have been reported. STEROIDS In many countries, including the United States, anabolic steroids are controlled substances based on their potential for abuse. Effects sought by abusers are increase in muscle mass and strength rather than euphoria. However, excessive use can have adverse behavioral, cardiovascular, and musculoskeletal effects. Acne (sometimes severe), premature closure of the epiphyses, and masculinization in females are anticipated androgenic adverse effects. Hepatic dysfunction has been reported, and the anabolic steroids may pose an increased risk of myocardial infarct. Behavioral manifestations include increases in libido and aggression (“roid rage”). A withdrawal syndrome has been described with fatigue and depression of mood. SKILL KEEPER: DRUG OF ABUSE OVERDOSE SIGNS AND SYMPTOMS (SEE CHAPTERS 22 AND 31) In an emergency situation, behavioral manifestations of the toxicity of drugs of abuse can be of assistance in diagnosis. What other readily detectable markers will also be helpful? The Skill Keeper Answer appears at the end of the chapter. QUESTIONS Questions 1 and 2. A 42-year-old homemaker suffers from anxiety with phobic symptoms and occasional panic attacks. She uses over-the-counter antihistamines for allergic rhinitis and claims that ethanol use is “just 1 or 2 glasses of wine with dinner.” Alprazolam, a benzodiazepine, is prescribed, and the patient is maintained on the drug for 3 yr, with several dose increments over that time period. Her family notices that she does not seem to be improving and that her speech is often slurred in the evenings. She is finally hospitalized with severe withdrawal signs on one weekend while attempting to end her dependence on drugs. 1. Which statement about the use of alprazolam is accurate? (A) Abrupt discontinuance of alprazolam after 4 wk of treatment may elicit withdrawal signs (B) Additive CNS depression occurs with ethanol (C) Benzodiazepines are Schedule IV-controlled drugs (D) Tolerance can occur with chronic use of any benzodiazepine (E) All of the above statements are accurate 2. The main reason for hospitalization of this patient was to be able to effectively control (A) Cardiac arrhythmias (B) Delirium (C) Hepatic dysfunction (D) Seizures (E) None of the above 3. Which drug, a partial agonist at nicotinic acetycholine receptors, is used in smoking cessation programs but may cause seizures in overdose? (A) Acamprosate (B) Buprenorphine (C) Nalbuphine (D) Rimonabant (E) Varenicline 4. Which statement about abuse of the opioid analgesics is false? (A) Lacrimation, rhinorrhea, yawning, and sweating are early signs of withdrawal from opioid analgesics (B) In withdrawal from opioids, clonidine may be useful in reducing symptoms caused by sympathetic overactivity (C) Methadone alleviates most of the symptoms of heroin withdrawal (D) Most patients experiencing withdrawal from heroin are free of the symptoms of abstinence in 6–8 d (E) Naloxone may precipitate a severe withdrawal state in abusers of opioid analgesics with symptoms starting in less than 15–30 min 5. A young male patient is brought to the emergency department suffering from an overdose of cocaine after its intravenous administration. His symptoms are not likely to include (A) Agitation (B) Bradycardia (C) Hyperthermia (D) Myocardial infarct (E) Seizures 6. Which statement about hallucinogens is accurate? (A) Dilated pupils and tachycardia are characteristic effects of scopolamine (B) LSD is unique among hallucinogens in that animals will self-administer it (C) Mescaline and psilocybin exert their CNS actions through dopaminergic systems in the brain (D) Phencyclidine is a known teratogen (E) Withdrawal signs characteristic of dependence occur with abrupt discontinuance of ketamine 7. Which statement about inhalants is accurate? (A) Euphoria, numbness, and tingling sensations with visual and auditory disturbances occur in most persons who inhale organic nitrites (B) Methemoglobinemia is a common toxicologic problem after repetitive inhalation of industrial solvents (C) Nitrous oxide is the most commonly abused drug by medical personnel working in hospitals (D) Solvent inhalation is mainly a drug abuse problem in petroleum industry workers (E) The inhalation of isobutyl nitrite is likely to cause headache, hypotension, and flushing CHAPTER 32 Drugs of Abuse 8. Which sign or symptom is likely to occur with marijuana? (A) Bradycardia (B) Conjunctival reddening (C) Hypertension (D) Increased psychomotor performance (E) Mydriasis Questions 9 and 10. A college student is brought to the emergency department by friends. The physician is informed that the student had taken a drug and then “went crazy.” The patient is agitated and delirious. Several persons are required to hold him down. His skin is warm and sweaty, and his pupils are dilated. Bowel sounds are normal. Signs and symptoms include tachycardia, marked hypertension, hyperthermia, increased muscle tone, and both horizontal and vertical nystagmus. 9. The most likely cause of these signs and symptoms is intoxication from (A) Hashish (B) LSD (C) Mescaline (D) Methamphetamine (E) Phencyclidine 10. The management of this patient is likely to include (A) Administration of epinephrine (B) Alkalinization of the urine to increase drug elimination (C) Amitriptyline if psychosis ensues (D) Atropine to control hyperthermia (E) None of the above ANSWERS 1. Therapeutic doses of benzodiazepines may lead to dependence with withdrawal symptoms including anxiety and agitation observable on abrupt discontinuance after a few weeks of treatment. Like most sedative-hypnotics, benzodiazepines are schedule-controlled, exhibiting dependence liability and the development of tolerance. Additive depression occurs with ethanol and many other CNS drugs. The answer is E. 2. This patient is probably withdrawing from dependence on both alprazolam and alcohol use. In addition to the symptoms described previously, abrupt withdrawal from sedativehypnotic dependence may include hyperreflexia progressing to seizures, with ensuing coma and possibly death. The risk of a seizure is increased if the patient abruptly withdraws from ethanol use at the same time. Depending on severity of symptoms, initial management may require parenteral diazepam or lorazepam, with the latter drug often favored in hepatic dysfunction. The answer is D. 3. Acamprosate is an antagonist of NMDA glutamate receptors used together with counseling in alcohol treatment programs. Varenicline blocks the rewarding effects of nicotine and is used in smoking cessation programs. However, the drug may cause psychiatric changes and in overdose has caused seizures. The answer is E. 4. Symptoms of opioid withdrawal usually begin within 6–8 h, and the acute course may last 6–8 d. However, a secondary phase of heroin withdrawal, characterized by bradycardia, hypotension, hypothermia, and mydriasis, may last 26–30 wk. Methadone is commonly used in detoxification of the heroin addict because it is a strong agonist, has high oral bioavailability, and has a relatively long half-life. The answer is D. 265 5. Overdoses with amphetamines or cocaine have many signs and symptoms in common. However, the ability of cocaine to block the reuptake of norepinephrine at sympathetic nerve terminals results in greater cardiotoxicity. Tachycardia is the rule, with the possibility of an arrhythmia, infarct, or stroke. The answer is B. 6. Psilocybin, mescaline, and LSD have similar central (via serotonergic systems) and peripheral (sympathomimetic) effects, but no actions on dopaminergic receptors in the CNS. None of the hallucinogenic drugs have been shown to have teratogenic potential. Unlike most hallucinogens, PCP (not LSD) acts as a positive reinforcer of self-administration in animals. Emergence reactions can occur after use of ketamine, but they are not signs of withdrawal. Scopolamine blocks muscarinic receptors. The answer is A. 7. Male preteens are most likely to “experiment” with solvent inhalation. This can result in central and peripheral neurotoxicity, liver and kidney damage, and pulmonary disease. Opioids, including fentanyl and meperidine, are the most widely abused by medical personnel working in hospitals. Industrial solvents rarely cause methemoglobinemia, but this (and headaches, flushing, and hypotension) may occur after excessive use of nitrites. The answer is E. 8. Two of the most characteristic signs of marijuana use are increased pulse rate and reddening of the conjunctiva. Decreases in blood pressure and in psychomotor performance occur. Pupil size is not changed by marijuana. The answer is B. 9. The signs and symptoms point to PCP intoxication. The presence of both horizontal and vertical nystagmus is pathognomonic. The answer is E. 10. Management of phencyclidine (PCP) overdose involves ventilatory support and control of seizures (with a benzodiazepine), hypertension, and hyperthermia. Antipsychotic drugs (eg, haloperidol) may also be useful for psychosis. None of the drugs listed are of value. Atropine may cause hyperthermia! Phencyclidine is a weak base, and its renal elimination may be accelerated by urinary acidification, not alkalinization! A large percentage of phencyclidine is secreted into the stomach, so removal of the drug may be hastened by activated charcoal or nasogastric suction. The answer is E. SKILL KEEPER ANSWER: DRUG OF ABUSE OVERDOSE SIGNS AND SYMPTOMS (SEE CHAPTERS 22 AND 31) Readily detectable markers that may assist in diagnosis of the cause of drug overdose toxicity include changes in heart rate, blood pressure, respiration, body temperature, sweating, bowel signs, and pupillary responses. For example, tachycardia, hypertension, increased body temperature, decreased bowel signs, and mydriasis are common characteristics of overdose of CNS stimulants, including amphetamines, cocaine, and most hallucinogens. 266 PART V Drugs That Act in the Central Nervous System CHECKLIST When you complete this chapter, you should be able to: ❑ Identify the major drugs that are commonly abused. ❑ Describe the signs and symptoms of overdose with, and withdrawal from, CNS stimulants, opioid analgesics, and sedative-hypnotics, including ethanol. ❑ Describe the general principles of the management of overdose of commonly abused drugs. ❑ Identify the most likely causes of death from commonly abused drugs. DRUG SUMMARY TABLE: Drugs Used to Treat Dependence & Addiction Subclass Mechanism of Action Effects Clinical Applications Pharmacokinetics, Toxicities, Interactions Antagonists of opioid receptors Reverse or block effects of opioids Opioid overdose Short half-life (1–2 h) Treatment of alcoholism Half-life like morphine (4 h) Acute effects like morphine Substitution therapy for opioid addicts Variable half-life Toxicity: Like morphine re acute and chronic effects including withdrawal Attenuates acute effects of morphine and other strong opioids Substitution therapy for opioid addicts Long half-life (>40 h) • formulated with nalorphine to avoid illicit IV use Agonist at ACh-N receptor subtype Blocks rewarding effects of nicotine Smoking cessation Nausea and vomiting, psychiatric changes, seizures in high dose Modulators of GABAA receptors Enhance GABA functions in CNS Attenuate withdrawal symptoms including seizures from alcohol and other sedative-hypnotics Half-life 4–15 h; lorazepam kinetics not affected by liver dysfunction May block synaptic plasticity Treatment of alcoholism (in combination with counseling) Allergies, arrhythmias, variable BP effects, headaches, and impotence • hallucinations in elderly Decrease GABA and glutamate release in CNS Treatment of obesity • off-label use for smoking cessation Major depression • increased suicide risk Opioid antagonists Naloxone Naltrexone Synthetic opioid Methadone Slow-acting agonist at µ opioid receptors Partial l-receptor agonist Buprenorphine Partial agonist at µ opioid receptors N-receptor partial agonist Varenicline Benzodiazepines Oxazepam, lorazepam NMDA receptor antagonist Acamprosate Antagonist at glutamate NMDA receptors Cannabinoid receptor agonist Rimonabant Inverse agonist at CB1 receptors ACh, acetylcholine; NMDA, N-methyl-D-aspartate. PART VI DRUGS WITH IMPORTANT ACTIONS ON BLOOD, INFLAMMATION, & GOUT C Agents Used in Cytopenias; Hematopoietic Growth Factors Blood cells play essential roles in oxygenation of tissues, coagulation, protection against infectious agents, and tissue repair. Blood cell deficiency is a relatively common occurrence that can have profound repercussions. The most common cause of erythrocyte deficiency, or anemia, is insufficient supply of iron, vitamin B12 or folic acid substances required for normal production H A P T E R 33 of erythrocytes. Pharmacologic treatment of these types of anemia usually involves replacement of the missing substance. An alternative therapy for certain types of anemia and for deficiency in other types of blood cells is administration of recombinant hematopoietic growth factors, which stimulate the production of various lineages of blood cells and regulate blood cell function. Hematopoietic factors Erythrocyte factors Vitamins (B12, folate) Iron Platelet factor Erythropoiesisstimulating agents (ESAs; erythropoietin) Oprelvekin (IL-11) Granulocyte factors Filgrastim (G-CSF) Sargramostim (GM-CSF) 267 268 PART VI Drugs with Important Actions on Blood, Inflammation, & Gout High-Yield Terms to Learn Cobalamin Vitamin B12 ESAs Erythropoiesis-stimulating agents dTMP synthesis A set of biochemical reactions that produce deoxythymidylate (dTMP), an essential constituent of DNA synthesis. The cycle depends on the conversion of dihydrofolate to tetrahydrofolate by dihydrofolate reductase G-CSF Granulocyte colony-stimulating factor, a hematopoietic growth factor that regulates production and function of neutrophils GM-CSF Granulocyte-macrophage colony-stimulating factor, a hematopoietic growth factor that regulates production of granulocytes (basophils, eosinophils, and neutrophils), and other myeloid cells Hemochromatosis A condition of chronic excess total body iron caused either by an inherited abnormality of iron absorption or by frequent transfusions to treat certain types of hemolytic disorders (eg, thalassemia major) Megaloblastic anemia A deficiency in serum hemoglobin and erythrocytes in which the erythrocytes are abnormally large. Results from either folate or vitamin B12 deficiency Microcytic anemia A deficiency in serum hemoglobin and erythrocytes in which the erythrocytes are abnormally small. Often caused by iron deficiency Neutropenia An abnormally low number of neutrophils in the blood; patients with neutropenia are susceptible to serious infection Pernicious anemia A form of megaloblastic anemia resulting from deficiency of intrinsic factor, a protein produced by gastric mucosal cells and required for intestinal absorption of vitamin B12 Thrombocytopenia An abnormally low number of platelets in the blood; patients with thrombocytopenia are susceptible to hemorrhage BLOOD CELL DEFICIENCIES A. Iron and Vitamin Deficiency Anemias Microcytic hypochromic anemia, caused by iron deficiency, is the most common type of anemia. Megaloblastic anemias are caused by a deficiency of vitamin B12 or folic acid, cofactors required for the normal maturation of red blood cells. Pernicious anemia, the most common type of vitamin B12 deficiency anemia, is caused by a defect in the synthesis of intrinsic factor, a protein required for efficient absorption of dietary vitamin B12, or by surgical removal of that part of the stomach that secretes intrinsic factor. B. Other Blood Cell Deficiencies Deficiency in the concentration of the various lineages of blood cells can be a manifestation of a disease or a side effect of radiation or cancer chemotherapy. Recombinant DNA-directed synthesis of hematopoietic growth factors now makes possible the treatment of more patients with deficiencies in erythrocytes, neutrophils, and platelets. Some of these growth factors also play an important role in hematopoietic stem cell transplantation. IRON A. Role of Iron Iron is the essential metallic component of heme, the molecule responsible for the bulk of oxygen transport in the blood. Although most of the iron in the body is contained in hemoglobin, an important fraction is bound to transferrin, a transport protein, and ferritin, a storage protein. Deficiency of iron occurs most often in women because of menstrual blood loss and in vegetarians or malnourished persons because of inadequate dietary iron intake. Children and pregnant women have increased requirements for iron. B. Regulation of Iron Stores Although iron is an essential ion, excessive amounts are highly toxic. As a result, a complex system has evolved for the absorption, transport, and storage of free iron (Figure 33–1). Since there is no mechanism for the efficient excretion of iron, regulation of body iron content occurs through modulation of intestinal absorption. 1. Absorption—Dietary iron in the form of heme and the ferrous ion (Fe2+) are taken up by specialized transporters on the luminal surface of intestinal epithelial cells (Figure 33–1). Intestinal cell iron is either stored as ferritin or the ferrous iron is transported across the basolateral membrane by ferroportin and oxidized to ferric iron (Fe3+) by a ferroxidase (Figure 33–1). 2. Transport and storage—Ferric iron is transported in a complex with transferrin (Figure 33–1). Excess iron is stored in the protein-bound form in gastrointestinal epithelial cells, macrophages, and hepatocytes, and in cases of gross overload, in parenchymal cells of the skin, heart, and other organs. CHAPTER 33 Agents Used in Cytopenias; Hematopoietic Growth Factors 269 4 Spleen, other tissues macrophage Blood Senescent RBC 1 Gut lumen Intestinal epithelial cells Hgb Hgb HCP1 FO F Hgb Tf AF FP Fe3+ F FR FP AF Fe2+ FP TfR DMT1 TfR F TfR AF Hgb Fe 2 Bone marrow erythrocyte precursor TfR 3 Hepatocyte FIGURE 33–1 Absorption, transport, and storage of iron. Intestinal epithelial cells actively absorb inorganic iron via the divalent metal transporter 1 (DMT1) and heme iron via the heme carrier protein 1 (HCP1). Iron that is absorbed or released from absorbed heme iron in the intestine (section 1) is actively transported into the blood by ferroportin (FP) or complexed with apoferritin (AF) and stored as ferritin (F). In the blood, iron is transported by transferrin (Tf) to erythroid precursors in the bone marrow for synthesis of hemoglobin (Hgb) (section 2) or to hepatocytes for storage as ferritin (section 3). The transferrin-iron complex binds to transferrin receptors (TfR) in erythroid precursors and hepatocytes and is internalized. After release of iron, the TfR-Tf complex is recycled to the plasma membrane and Tf is released. Macrophages that phagocytize senescent erythrocytes (RBC) reclaim the iron from the RBC hemoglobin and either export it or store it as ferritin (section 4). Hepatocytes use several mechanisms to take up iron and store the iron as ferritin. FO, ferroxidase. (Reproduced, with permission, from Katzung BG, editor: Basic & Clinical Pharmacology, 13th ed. McGraw-Hill, 2014: Fig. 33–1.) 3. Elimination—Minimal amounts of iron are lost from the body with sweat and saliva and in exfoliated skin and intestinal mucosal cells. C. Clinical Use Prevention or treatment of iron deficiency anemia is the only indication for iron administration. Iron deficiency can be diagnosed from red blood cell changes (microcytic cell size due to diminished hemoglobin content) and from measurements of serum and bone marrow iron stores. The disease is treated by dietary ferrous iron supplementation with ferrous sulfate, ferrous gluconate, or ferrous fumarate. In special cases, treatment is by parenteral administration of a colloid containing a core of iron oxyhydroxide surrounded by a shell of carbohydrate. Parenteral iron preparations include iron dextran, sodium ferric gluconate complex, and iron sucrose. Iron should not be given in hemolytic anemia because iron stores are elevated, not depressed, in this type of anemia. Ferumoxytol is a super-paramagnetic iron oxide nanoparticle coated with carbohydrate. Ferumoxytol may interfere with magnetic resonance imaging (MRI) studies. Thus, MRI should be performed prior to ferumoxytol therapy. D. Toxicity of Iron (See Also Chapter 57) 1. Signs and symptoms—Acute iron intoxication is most common in children and usually occurs as a result of accidental ingestion of iron supplementation tablets. Depending on the dose of iron, necrotizing gastroenteritis, shock, metabolic acidosis, coma, and death may result. Chronic iron overload, known as hemochromatosis, damages the organs that store excess iron (heart, liver, pancreas). Hemochromatosis occurs most often in individuals with an inherited abnormality of iron absorption and those who receive frequent transfusions for treatment of hemolytic disorders (eg, thalassemia major). 270 PART VI Drugs with Important Actions on Blood, Inflammation, & Gout 2. Treatment of acute iron intoxication—Immediate treatment is necessary and usually consists of removal of unabsorbed tablets from the gut, correction of acid-base and electrolyte abnormalities, and parenteral administration of deferoxamine, which chelates circulating iron. Activated charcoal does not bind iron in the gut and thus is ineffective. 3. Treatment of chronic iron toxicity—Treatment of the genetic form of hemochromatosis is usually by phlebotomy. Hemochromatosis that is due to frequent transfusions is treated with parenteral deferoxamine or with the newer oral iron chelator deferasirox. VITAMIN B12 A. Role of Vitamin B12 Vitamin B12 (cobalamin), a cobalt-containing molecule, is, along with folic acid, a cofactor in the transfer of 1-carbon units, a step necessary for the synthesis of DNA. Impairment of DNA synthesis affects all cells, but because red blood cells must be produced continuously, deficiency of either vitamin B12 or folic acid usually manifests first as anemia. In addition, vitamin B12 deficiency can cause neurologic defects, which may become irreversible if not treated promptly. B. Pharmacokinetics Vitamin B12 is produced only by bacteria; this vitamin cannot be synthesized by multicellular organisms. It is found in many foods and absorbed from the gastrointestinal tract in the presence of intrinsic factor, a product of the parietal cells of the stomach. Plasma transport is accomplished by binding to transcobalamin II. Vitamin B12 is stored in the liver in large amounts; a normal individual has enough to last 5 yr. The 2 available forms of vitamin B12, cyanocobalamin and hydroxocobalamin, have similar pharmacokinetics, but hydroxocobalamin has a longer circulating half-life. C. Pharmacodynamics Vitamin B12 is essential in 2 reactions: conversion of methylmalonyl-coenzyme A (CoA) to succinyl-CoA and conversion of homocysteine to methionine. The second reaction is linked to folic acid metabolism and synthesis of deoxythymidylate (dTMP; Figure 33–2, section 2), a precursor required for DNA synthesis. In vitamin B12 deficiency, folates accumulate as N 5-methyltetrahydrofolate; the supply of tetrahydrofolate is depleted; and the production of red blood cells slows. Administration of folic acid to patients with vitamin B12 deficiency helps refill the tetrahydrofolate pool (Figure 33–2, section 3) and partially or fully corrects Purines N 5, N10-Methylenetetrahydrofolate dUMP 2 Thymidylate synthase dTMP Glycine Serine transhydroxymethylase DNA synthesis Serine Tetrahydrofolate Dihydrofolate reductase Dihydrofolate 3 Methylcobalamin Methionine Dihydrofolate reductase Folic acid 1 Cobalamin Homocysteine N 5-Methyltetrahydrofolate Dietary folates FIGURE 33–2 Enzymatic reactions that use folates. Section 1 shows the vitamin B12-dependent reaction that allows most dietary folates to enter the tetrahydrofolate cofactor pool and becomes the “folate trap” in vitamin B12 deficiency. Section 2 shows the dTMP cycle. Section 3 shows the pathway by which folate enters the tetrahydrofolate cofactor pool. Double arrows indicate pathways with more than 1 intermediate step. (Reproduced, with permission, from Katzung BG, editor: Basic & Clinical Pharmacology, 12th ed. McGraw-Hill, 2012: Fig. 33–3.) CHAPTER 33 Agents Used in Cytopenias; Hematopoietic Growth Factors the anemia. However, the exogenous folic acid does not correct the neurologic defects of vitamin B12 deficiency. D. Clinical Use and Toxicity The 2 available forms of vitamin B12—hydroxocobalamin and cyanocobalamin—have equivalent effects. The major application is in the treatment of naturally occurring pernicious anemia and anemia caused by gastric resection. Because vitamin B12 deficiency anemia is almost always caused by inadequate absorption, therapy should be by replacement of vitamin B12, using parenteral therapy. Neither form of vitamin B12 has significant toxicity. FOLIC ACID A. Role of Folic Acid Like vitamin B12, folic acid is required for normal DNA synthesis, and its deficiency usually presents as megaloblastic anemia. In addition, deficiency of folic acid during pregnancy increases the risk of neural tube defects in the fetus. B. Pharmacokinetics Folic acid is readily absorbed from the gastrointestinal tract. Only modest amounts are stored in the body, so a decrease in dietary intake is followed by anemia within a few months. C. Pharmacodynamics Folic acid is converted to tetrahydrofolate by the action of dihydrofolate reductase (Figure 33–2, section 3). One important set of reactions involving tetrahydrofolate and dihydrofolate constitutes the dTMP cycle (Figure 33–2, section 2), which supplies the dTMP required for DNA synthesis. Rapidly dividing cells are highly sensitive to folic acid deficiency. For this reason, antifolate drugs are useful in the treatment of various infections and cancers. D. Clinical Use and Toxicity Folic acid deficiency is most often caused by dietary insufficiency or malabsorption. Anemia resulting from folic acid deficiency is readily treated by oral folic acid supplementation. Because maternal folic acid deficiency is associated with increased risk of neural tube defects in the fetus, folic acid supplementation is recommended before and during pregnancy. Folic acid supplements correct the anemia but not the neurologic deficits of vitamin B12 deficiency. Therefore, vitamin B12 deficiency must be ruled out before one selects folic acid as the sole therapeutic agent in the treatment of a patient with megaloblastic anemia. Folic acid has no recognized toxicity. HEMATOPOIETIC GROWTH FACTORS More than a dozen glycoprotein hormones that regulate the differentiation and maturation of stem cells within the bone marrow have been identified. Several growth factors, produced by recombinant DNA technology, have FDA approval for the treatment of patients with blood cell deficiencies. 271 SKILL KEEPER: ROUTES OF ADMINISTRATION (SEE CHAPTER 1) All of the recombinant hematopoietic growth factors approved for clinical use are administered by injection. Why can these growth factors not be given orally? Which 3 routes of administration require drug injection? How do these 3 routes compare with regard to onset and duration of drug action and risk of adverse effects? The Skill Keeper Answers appear at the end of the chapter. A. Erythropoiesis-Stimulating Agents (ESAs) Erythropoietin is produced by the kidney; reduction in its synthesis underlies the anemia of renal failure. Through activation of receptors on erythroid progenitors in the bone marrow, erythropoietin stimulates the production of red cells and increases their release from the bone marrow. Erythropoiesis-stimulating agents (ESAs) are routinely used for the anemia associated with renal failure and are sometimes effective for patients with other forms of anemia (eg, primary bone marrow disorders or anemias secondary to cancer chemotherapy or HIV treatment, bone marrow transplantation, AIDS, or cancer). As an alternative to recombinant human erythropoietin (epoetin alfa), darbepoetin alfa, a glycosylated form of erythropoietin, has a much longer half-life. Methoxy polyethylene glycol-epoetin beta is a long-lasting form of erythropoietin that can be administered once or twice a month. The most common complications of ESA therapy are hypertension and thrombosis. The serum hemoglobin concentration of patients treated with an ESA should not exceed 12 g/dL because hemoglobin concentrations above this target have been linked to an increased rate of mortality and cardiovascular events. B. Myeloid Growth Factors Filgrastim (granulocyte colony-stimulating factor; G-CSF) and sargramostim (granulocyte-macrophage colony-stimulating factor; GM-CSF) stimulate the production and function of neutrophils. GM-CSF also stimulates the production of other myeloid and megakaryocyte progenitors. G-CSF and, to a lesser degree, GM-CSF mobilize hematopoietic stem cells (ie, increase their concentration in peripheral blood). Both growth factors are used to accelerate the recovery of neutrophils after cancer chemotherapy and to treat other forms of secondary and primary neutropenia (eg, aplastic anemia, congenital neutropenia). When given to patients soon after autologous stem cell transplantation, G-CSF reduces the time to engraftment and the duration of neutropenia. In patients with multiple myeloma or non-Hodgkin’s lymphoma who respond poorly to G-CSF alone, G-CSF may be combined with the novel hematopoietic stem cell mobilizer plerixafor, an inhibitor of the CXC chemokine receptor 4 (CXCR4). G-CSF is also used to mobilize peripheral blood stem cells in preparation for autologous and allogeneic stem cell transplantation. The toxicity of G-CSF is minimal, although the drug sometimes causes bone pain. GM-CSF can cause more 272 PART VI Drugs with Important Actions on Blood, Inflammation, & Gout severe effects, including fever, arthralgias, and capillary damage with edema. Allergic reactions are rare. Pegfilgrastim, a covalent conjugation product of filgrastim and a form of polyethylene glycol, has a much longer serum half-life than recombinant G-CSF. Lenograstim, used widely in Europe, is a glycosylated form of recombinant G-CSF. C. Megakaryocyte Growth Factors Oprelvekin (interleukin-11 [IL-11]) stimulates the growth of primitive megakaryocytic progenitors and increases the number of peripheral platelets. IL-11 is used for the treatment of patients who have had a prior episode of thrombocytopenia after a cycle of cancer chemotherapy. In such patients, it reduces the need for platelet transfusions. The most common adverse effects of IL-11 are fatigue, headache, dizziness, and fluid retention. Romiplostim, a thrombopoietin receptor agonist with a novel peptide structure, is used subcutaneously in patients with chronic idiopathic thrombocytopenia who have failed to respond to conventional treatment. Eltrombopag is an oral agonist of the thrombopoietin receptor that is also used for patients with chronic idiopathic thrombocytopenia that is refractory to other agents. The risk of hepatotoxicity and hemorrhage has restricted eltrombopag use to registered physicians and patients. QUESTIONS Questions 1–4. A 23-year-old pregnant woman is referred by her obstetrician for evaluation of anemia. She is in her fourth month of pregnancy and has no history of anemia; her grandfather had pernicious anemia. Her hemoglobin is 10 g/dL (normal, 12–16 g/dL). 1. If this woman has macrocytic anemia, an increased serum concentration of transferrin, and a normal serum concentration of vitamin B12, the most likely cause of her anemia is deficiency of which of the following? (A) Cobalamin (B) Erythropoietin (C) Folic acid (D) Intrinsic factor (E) Iron 2. The laboratory data for your pregnant patient indicate that she does not have macrocytic anemia but rather microcytic anemia. Optimal treatment of normocytic or mild microcytic anemia associated with pregnancy uses which of the following? (A) A high-fiber diet (B) Erythropoietin injections (C) Ferrous sulfate tablets (D) Folic acid supplements (E) Hydroxocobalamin injections 3. If this patient has a young child at home and is taking ironcontaining prenatal supplements, she should be warned that they are a common source of accidental poisoning in young children and advised to make a special effort to keep these pills out of her child’s reach. Toxicity associated with acute iron poisoning usually includes which of the following? (A) Dizziness, hypertension, and cerebral hemorrhage (B) Hyperthermia, delirium, and coma (C) Hypotension, cardiac arrhythmias, and seizures (D) Necrotizing gastroenteritis, shock, and metabolic acidosis (E) Severe hepatic injury, encephalitis, and coma 4. The child in the previous question did ingest the iron-containing supplements. What immediate treatment is necessary? Correction of acid-base and electrolyte abnormalities and (A) Activated charcoal (B) Oral deferasirox (C) Parenteral deferoxamine (D) Parenteral dantrolene 5. A 45-year-old male stomach cancer patient underwent tumor removal surgery. After surgery, he developed megaloblastic anemia. His anemia is caused by a deficiency of X and can be treated with Y. (A) X = intrinsic factor; Y = folic acid. (B) X = intrinsic factor; Y = vitamin B12 (C) X = extrinsic factor; Y = parenteral iron (D) X = extrinsic factor; Y = sargramostim 6. Which of the following is most likely to be required by a 5-year-old boy with chronic renal insufficiency? (A) Cyanocobalamin (B) Deferoxamine (C) Erythropoietin (D) Filgrastim (G-CSF) (E) Oprelvekin (IL-11) 7. In a patient who requires filgrastim (G-CSF) after being treated with anticancer drugs, the therapeutic objective is to prevent which of the following? (A) Allergic reactions (B) Cancer recurrence (C) Excessive bleeding (D) Hypoxia (E) Systemic infection 8. The megaloblastic anemia that results from vitamin B12 deficiency is due to inadequate supplies of which of the following? (A) Cobalamin (B) dTMP (C) Folic acid (D) Homocysteine (E) N 5-methyltetrahydrofolate CHAPTER 33 Agents Used in Cytopenias; Hematopoietic Growth Factors Questions 9 and 10. After undergoing surgery for breast cancer, a 53-year-old woman is scheduled to receive 4 cycles of cancer chemotherapy. The cycles are to be administered every 3–5 wk. Her first cycle was complicated by severe chemotherapy-induced thrombocytopenia. 9. During the second cycle of chemotherapy, it would be appropriate to consider treating this patient with which of the following? (A) Darbepoetin alpha (B) Filgrastim (G-CSF) (C) Iron dextran (D) Oprelvekin (IL-11) (E) Vitamin B12 10. Twenty months after finishing her chemotherapy, the woman had a relapse of breast cancer. The cancer was now unresponsive to standard doses of chemotherapy. The decision was made to treat the patient with high-dose chemotherapy followed by autologous stem cell transplantation. Which of the following drugs is most likely to be used to mobilize the peripheral blood stem cells needed for the patient’s autologous stem cell transplantation? (A) Erythropoietin (B) Filgrastim (G-CSF) (C) Folic acid (D) Intrinsic factor (E) Oprelvekin (interleukin-11) ANSWERS 1. Deficiencies of folic acid or vitamin B12 are the most common causes of megaloblastic anemia. If a patient with this type of anemia has a normal serum vitamin B12 concentration, folate deficiency is the most likely cause of the anemia. The answer is C. 2. Iron deficiency microcytic anemia is the anemia that is most commonly associated with pregnancy. In this condition, oral iron supplementation is indicated. The answer is C. 3. Acute iron poisoning often causes severe gastrointestinal damage resulting from direct corrosive effects, shock from fluid loss in the gastrointestinal tract, and metabolic acidosis from cellular dysfunction. The answer is D. 4. Activated charcoal does not bind iron and thus is ineffective. Oral deferasirox is effective for chronic iron toxicity. Dantrolene inhibits Ca2+ release from the sarcoplasmic reticulum and is an antidote for malignant hyperthermia induced by inhaled anesthetics. The answer is C. 5. Resection of the stomach does lead to loss of intrinsic factor and the patient will be deficient in vitamin B12. Prevention or treatment of iron deficiency anemia (microcytic cell size) is the only indication for iron administration. Sargramostim is a GM-CSF and is used to stimulate the production of neutrophils and other myeloid and megakaryocyte progenitors. The answer is B. 273 6. The kidney produces erythropoietin; patients with chronic renal insufficiency often require exogenous erythropoietin to avoid chronic anemia. The answer is C. 7. Filgrastim (G-CSF) stimulates the production and function of neutrophils, important cellular mediators of the innate immune system that serve as the first line of defense against infection. The answer is E. 8. Deficiency of vitamin B12 (cobalamin) leads to a deficiency in tetrahydrofolate and subsequently a deficiency of the dTMP required for DNA synthesis. Homocysteine and N 5-methyltetrahydrofolate accumulate. The answer is B. 9. Oprelvekin (IL-11) stimulates platelet production and decreases the number of platelet transfusions required by patients undergoing bone marrow suppression therapy for cancer. The answer is D. 10. The success of transplantation with peripheral blood stem cells depends on infusion of adequate numbers of hematopoietic stem cells. Administration of G-CSF to the donor (in the case of autologous transplantation, the patient who also will be the recipient of the transplantation) greatly increases the number of hematopoietic stem cells harvested from the donor’s blood. The answer is B. SKILL KEEPER ANSWERS: ROUTES OF ADMINISTRATION (SEE CHAPTER 1) All of the hematopoietic growth factors are proteins with molecular weights greater than 15,000. Like other proteinaceous drugs, the growth factors cannot be administered orally because they have very poor bioavailability. Their peptide bonds are destroyed by stomach acid and digestive enzymes. Injections are required for intravenous, intramuscular, and subcutaneous administration. The intravenous route offers the fastest onset of drug action and shortest duration of drug action. Because intravenous administration can produce high blood levels, this route of administration has the greatest risk of producing concentration-dependent drug toxicity. Intramuscular injection has a quicker onset of action than subcutaneous injection, and larger volumes of injected fluid can be given. Because protective barriers can be breached by the needle or tubing used for drug injection, all 3 of these routes of administration carry a greater risk of infection than does oral drug administration. 274 PART VI Drugs with Important Actions on Blood, Inflammation, & Gout CHECKLIST When you complete this chapter, you should be able to: ❑ Name the 2 most common types of nutritional anemia, and, for each, describe the most likely biochemical causes. ❑ Diagram the normal pathways of absorption, transport, and storage of iron in the human body. ❑ Name the anemias for which iron supplementation is indicated and those for which it is contraindicated. ❑ List the acute and chronic toxicities of iron. ❑ Sketch the dTMP cycle and show how deficiency of folic acid or deficiency of vitamin B12 affects the normal cycle. ❑ Explain the major hazard involved in the use of folic acid as sole therapy for megaloblastic anemia and indicate on a sketch of the dTMP cycle the biochemical basis of the hazard. ❑ Name 3–5 major hematopoietic growth factors that are used clinically and describe the clinical uses and toxicity of each. ❑ Explain the advantage of covalently attaching polyethylene glycol to filgrastim. DRUG SUMMARY TABLE: Drugs for Cytopenias; Hematopoietic Growth Factors Subclass Mechanism of Action Clinical Applications Pharmacokinetics Toxicities, Interactions Required for biosynthesis of heme and hemecontaining proteins, including hemoglobin and myoglobin Iron deficiency, which manifests as microcytic anemia Complicated endogenous system for absorbing, storing, and transporting iron • no mechanism for iron excretion other than cell and blood loss Acute overdose results in necrotizing gastroenteritis, abdominal pain, bloody diarrhea, shock, lethargy, and dyspnea • chronic iron overload results in hemochromatosis, with damage to the heart, liver, and pancreas Iron Ferrous sulfate Ferrous gluconate and ferrous fumarate: oral iron preparations Iron dextran, iron sucrose complex, sodium ferric gluconate complex and ferumoxytol: parenteral preparations; can cause pain, hypersensitivity reactions. Ferumoxytol may interfere with MRI studies. Iron chelators (see also Chapters 57 and 58) Deferoxamine Chelates excess iron Acute iron poisoning • inherited or acquired hemochromatosis Preferred routes of administration: intramuscular or subcutaneous Rapid IV administration may cause hypotension • neurotoxicity and increased susceptibility to certain infections has occurred with long-term use Deferasirox: oral iron chelator for treatment of hemochromatosis (Continued ) CHAPTER 33 Agents Used in Cytopenias; Hematopoietic Growth Factors 275 DRUG SUMMARY TABLE: Drugs for Cytopenias; Hematopoietic Growth Factors (Continued ) Subclass Mechanism of Action Clinical Applications Pharmacokinetics Toxicities, Interactions Cofactor required for essential enzymatic reactions that form tetrahydrofolate, convert homocysteine to methionine, and metabolize L-methylmalonyl-CoA Vitamin B12 deficiency, which manifests as megaloblastic anemia and is the basis of pernicious anemia Parenteral vitamin B12 is required for pernicious anemia and other malabsorption syndromes No toxicity associated with excess vitamin B12 Precursor of an essential donor of methyl groups used for synthesis of amino acids, purines, and deoxynucleotides Folic acid deficiency, which manifests as megaloblastic anemia • prevention of congenital neural tube defects Oral is well absorbed; need for parenteral administration is rare Not toxic in overdose, but large amounts can mask vitamin B12 deficiency Anemia, especially associated with chronic renal failure, HIV infection, cancer, and prematurity • prevention of need for transfusion in patients undergoing certain types of elective surgery Intravenous or subcutaneous administration 1–3 × per week Hypertension, thrombotic complications, and, very rarely, pure red cell aplasia • to reduce the risk of serious cardiovascular events, hemoglobin levels should be maintained <12 g/dL Vitamin B12 Cyanocobalamin, hydroxocobalamin Folic acid Folacin (pteroylglutamic acid) Erythropoiesis-stimulating agents (ESAs) Epoetin alfa Agonist of erythropoietin receptors expressed by red cell progenitors Darbepoetin alfa: long-acting glycosylated form administered weekly Methoxy polyethylene glycol-epoetin beta: long-acting form administered 1–2 × per month Myeloid growth factors G-CSF (filgrastim) Stimulates G-CSF receptors expressed on mature neutrophils and their progenitors Neutropenia associated with congenital neutropenia, cyclic neutropenia, myelodysplasia, and aplastic anemia • secondary prevention of neutropenia in patients undergoing cytotoxic chemotherapy • mobilization of peripheral blood cells in preparation for autologous and allogenic stem cell transplantation Daily subcutaneous administration Bone pain • rarely, splenic rupture Pegfilgrastim: long-acting form of filgrastim that is covalently linked to a type of polyethylene glycol GM-CSF (sargramostim): myeloid growth factor that acts through a distinct GM-CSF receptor to stimulate proliferation and differentiation of early and late granulocytic progenitor cells, and erythroid and megakaryocyte progenitors. Clinical uses are similar to those of G-CSF, although it is more likely than G-CSF to cause fever, arthralgia, myalgia, and a capillary leak syndrome Plerixafor: antagonist of CXCR4 receptor used in combination with G-CSF for mobilization of peripheral blood cells prior to autologous transplantation in patients with multiple myeloma or non-Hodgkin’s lymphoma who responded suboptimally to G-CSF alone Megakaryocyte growth factors Oprelvekin (interleukin-11; IL-11) Recombinant form of an endogenous cytokine • activates IL-11 receptors Secondary prevention of thrombocytopenia in patients undergoing cytotoxic chemotherapy for nonmyeloid cancers Daily subcutaneous administration Fatigue, headache, dizziness, anemia, fluid accumulation in the lungs, and transient atrial arrhythmias Romiplostim: genetically engineered protein in which the Fc components of a human antibody are fused to multiple copies of a peptide that stimulates the thrombopoietin receptors; approved for treatment of idiopathic thrombocytopenic purpura (ITP) Eltrombopag: orally active agonist of thrombopoietin receptor; restricted use because of risk of hepatotoxicity and hemorrhage C A P T E R 34 Drugs Used in Coagulation Disorders The drugs used in clotting and bleeding disorders fall into 2 major groups: (1) drugs used to decrease clotting or dissolve clots already present in patients at risk for vascular occlusion and (2) drugs used to increase clotting in patients with clotting deficiencies. The first group, the anticlotting drugs, includes some of the most commonly used drugs in the United States. Anticlotting drugs are H used in the treatment and prevention of myocardial infarction and other acute coronary syndromes, atrial fibrillation, ischemic stroke, and deep vein thrombosis (DVT). Within the anticlotting group, the anticoagulant and thrombolytic drugs are effective in treatment of both venous and arterial thrombosis, whereas antiplatelet drugs are used primarily for treatment of arterial disease. Drugs used in clotting disorders Heparins Direct thrombin inhibitors Anticoagulants Direct factor Xa inhibitors Warfarin Anticlotting drugs t-PA derivatives Thrombolytics Streptokinase Aspirin Antiplatelet drugs Glycoprotein IIb/IIla inhibitors ADP inhibitors (clopidogrel) PDE/adenosine uptake inhibitors Replacement factors Drugs that facilitate clotting Vitamin K Antiplasmin drugs ANTICOAGULANTS A. Classification Anticoagulants inhibit the formation of fibrin clots. Three major types of anticoagulants are available: heparin and related products, which must be used parenterally; direct thrombin and factor X inhibitors, which are used parenterally or orally; and the orally 276 active coumarin derivatives (eg, warfarin). Comparative properties of the heparins and warfarin are shown in Table 34–1. B. Heparin 1. Chemistry—Heparin is a large sulfated polysaccharide polymer obtained from animal sources. Each batch contains molecules of varying size, with an average molecular weight of CHAPTER 34 Drugs Used in Coagulation Disorders 277 High-Yield Terms to Learn Activated partial thromboplastin time (aPTT) test Laboratory test used to monitor the anticoagulant effect of unfractionated heparin and direct thrombin inhibitors; prolonged when drug effect is adequate Antithrombin III An endogenous anticlotting protein that irreversibly inactivates thrombin and factor Xa. Its enzymatic action is markedly accelerated by the heparins Clotting cascade System of serine proteases and substrates in the blood that provides rapid generation of clotting factors resulting in a fibrin clot, in response to blood vessel damage Glycoprotein IIb/IIIa (GPIIb/IIIa) A protein complex on the surface of platelets. When activated, it aggregates platelets primarily by binding to fibrin. Endogenous factors including thromboxane A2, ADP, and serotonin initiate a signaling cascade that activates GPIIb/IIIa Heparin-induced thrombocytopenia (HIT) A hypercoagulable state plus thrombocytopenia that occurs in a small number of individuals treated with unfractionated heparin LMW heparins Fractionated preparations of heparin of molecular weight 2000–6000. Unfractionated heparin has a molecular weight range of 5000–30,000 Prothrombin time (PT) test Laboratory test used to monitor the anticoagulant effect of warfarin; prolonged when drug effect is adequate 15,000–20,000. Heparin is highly acidic and can be neutralized by basic molecules (eg, protamine). Heparin is given intravenously or subcutaneously to avoid the risk of hematoma associated with intramuscular injection. Low-molecular-weight (LMW) fractions of heparin (eg, enoxaparin) have molecular weights of 2000–6000. LMW heparins have greater bioavailability and longer durations of action than unfractionated heparin; thus, doses can be given less frequently (eg, once or twice a day). They are given subcutaneously. Fondaparinux is a small synthetic drug that contains the biologically active pentasaccharide present in unfractionated and LMW heparins. It is administered subcutaneously once daily. 2. Mechanism and effects—Unfractionated heparin binds to endogenous antithrombin III (ATIII) via a key pentasaccharide sequence. The heparin–ATIII complex combines with and irreversibly inactivates thrombin and several other factors, particularly factor Xa (Figure 34–1). In the presence of heparin, ATIII proteolyzes thrombin and factor Xa approximately 1000-fold faster than in its absence. Because it acts on preformed blood components, heparin provides anticoagulation immediately after administration. The action of heparin is monitored with the activated partial thromboplastin time (aPTT) laboratory test. LMW heparins and fondaparinux, like unfractionated heparin, bind ATIII. These complexes have the same inhibitory effect on TABLE 34–1 Properties of heparins and warfarin. Property Heparins Warfarin Structure Large acidic polysaccharide polymers Small lipid-soluble molecule Route of administration Parenteral Oral Site of action Blood Liver Onset of action Rapid (minutes) Slow (days); limited by half-lives of preexisting normal factors Mechanism of action Activate antithrombin III, which inactivates coagulation factors including thrombin and factor Xa Impairs post-translational modification of factors II, VII, IX and X Monitoring aPTT for unfractionated heparin but not LMW heparins Prothrombin time Antidote Protamine for unfractionated heparin; protamine reversal of LMW heparins is incomplete Vitamin K1, plasma, prothrombin complex concentrates Use Mostly acute, over days Chronic, over weeks to months Use in pregnancy Yes No aPTT, activated partial thromboplastin time; LMW, low molecular weight. 278 PART VI Drugs with Important Actions on Blood, Inflammation, & Gout TFPI Protein C Thrombomodulin Endothelial cells VIIa − TE VII + TF Protein Cact XIa IX IXa VIIIa X Xa Va Inhibited by heparin Inhibited by oral anticoagulant drugs Down-regulated by protein Cact Prothrombin II IIa I Fibrinogen Thrombin Ia Fibrin clot FIGURE 34–1 A model of the coagulation cascade, including its inhibition by the activated form of protein C. Tissue factor (TF) is important in initiating the cascade. Tissue factor pathway inhibitor (TFPI) inhibits the action of the VIIa–TF complex. (Reproduced, with permission, from Katzung BG, editor: Basic & Clinical Pharmacology, 11th ed. McGraw-Hill, 2009: Fig. 34–2.) factor Xa as the unfractionated heparin–ATIII complex. However, the short-chain heparin–ATIII and fondaparinux–ATIII complexes provide a more selective action because they fail to affect thrombin. The aPTT test does not reliably measure the anticoagulant effect of the LMW heparins and fondaparinux; this is a potential problem, especially in renal failure, in which their clearance may be decreased. 3. Clinical use—Because of its rapid effect, heparin is used when anticoagulation is needed immediately (eg, when starting therapy). Common uses include treatment of DVT, pulmonary embolism, and acute myocardial infarction. Heparin is used in combination with thrombolytics for revascularization and in combination with glycoprotein IIb/IIIa inhibitors during angioplasty and placement of coronary stents. Because it does not cross the placental barrier, heparin is the drug of choice when an anticoagulant must be used in pregnancy. LMW heparins and fondaparinux have similar clinical applications. 4. Toxicity—Increased bleeding is the most common adverse effect of heparin and related molecules; the bleeding may result in hemorrhagic stroke. Protamine can lessen the risk of serious bleeding that can result from excessive unfractionated heparin. Protamine only partially reverses the effects of LMW heparins and does not affect the action of fondaparinux. Unfractionated heparin causes moderate transient thrombocytopenia in many patients and severe thrombocytopenia and thrombosis (heparin-induced thrombocytopenia or HIT) in a small percentage of patients who produce an antibody that binds to a complex of heparin and platelet factor 4. LMW heparins and fondaparinux are less likely to cause this immune-mediated thrombocytopenia. Prolonged use of unfractionated heparin is associated with osteoporosis. C. Direct Thrombin Inhibitors 1. Chemistry and pharmacokinetics—Direct thrombin inhibitors are based on proteins made by Hirudo medicinalis, the medicinal leech. Lepirudin is the recombinant form of the leech protein hirudin, while desirudin and bivalirudin are modified forms of hirudin. Argatroban is a small molecule with a short half-life. All 4 drugs are administered parenterally. Dabigatran is an orally active direct thrombin inhibitor. 2. Mechanism and effects—The protein analogs of lepirudin bind simultaneously to the active site of thrombin and to thrombin substrates. Argatroban binds solely to the thrombin-active site. Unlike the heparins, these drugs inhibit both soluble thrombin and the thrombin enmeshed within developing clots. Bivalirudin also inhibits platelet activation. 3. Clinical use—Direct thrombin inhibitors are used as alternatives to heparin primarily in patients with heparin-induced thrombocytopenia. Bivalirudin also is used in combination with aspirin during percutaneous coronary angioplasty. Like unfractionated heparin, the action of these drugs is monitored with the aPTT laboratory test. Advantages of oral direct thrombin inhibitors include predictable pharmacokinetics, which allows for fixed dosing, as well as a predictable immediate anticoagulant response CHAPTER 34 Drugs Used in Coagulation Disorders that makes routine monitoring or overlap with other anticoagulants unnecessary. In addition, these agents do not interact with P450-interacting drugs. Dabigatran is approved for prevention of stroke and systemic embolism in nonvalvular atrial fibrillation. 4. Toxicity—Like other anticoagulants, the direct thrombin inhibitors can cause bleeding. No reversal agents exist. Prolonged infusion of lepirudin can induce antibodies that form a complex with lepirudin and prolong its action, and it can induce anaphylactic reactions. Lepirudin production was discontinued in 2012. D. Direct Oral Factor Xa inhibitors 1. Chemistry and pharmacokinetics—Oral Xa inhibitors, including the small molecules rivaroxaban, apixaban, and edoxaban, have a rapid onset of action and shorter half-lives than warfarin. These drugs are given as fixed oral doses and do not require monitoring. They undergo cytochrome P450-dependent and cytochrome P450-independent elimination. 2. Mechanism and effects—These small molecules directly bind to and inhibit both free factor Xa and factor Xa bound in the clotting complex. 3. Clinical use—Rivaroxaban is approved for prevention and treatment of venous thromboembolism following hip or knee surgery and for prevention of stroke in patients with atrial fibrillation, without valvular heart disease. Apixaban is approved for prevention of embolic stroke in patients with nonvalvular atrial fibrillation. 4. Toxicity—Like other anticoagulants, the factor Xa inhibitors can cause bleeding. No reversal agents exist. E. Warfarin and Other Coumarin Anticoagulants 1. Chemistry and pharmacokinetics—Warfarin and other coumarin anticoagulants are small, lipid-soluble molecules that are readily absorbed after oral administration. Warfarin is highly bound to plasma proteins (>99%), and its elimination depends on metabolism by cytochrome P450 enzymes. 2. Mechanism and effects—Warfarin and other coumarins interfere with the normal post-translational modification of clotting factors in the liver, a process that depends on an adequate supply of reduced vitamin K. The drugs inhibit vitamin K epoxide reductase (VKOR), which normally converts vitamin K epoxide to reduced vitamin K. The vitamin K-dependent factors include thrombin and factors VII, IX, and X (Figure 34–1). Because the clotting factors have half-lives of 8–60 h in the plasma, an anticoagulant effect is observed only after sufficient time has passed for elimination of the normal preformed factors. The action of warfarin can be reversed with vitamin K, but recovery requires the synthesis of new normal clotting factors and is, therefore, slow (6–24 h). More rapid reversal can be achieved by transfusion with fresh or frozen plasma that contains normal clotting factors. The effect of warfarin is monitored by the prothrombin time (PT) test. 279 SKILL KEEPER: TREATMENT OF ATRIAL FIBRILLATION (SEE CHAPTERS 13 AND 14) Patients with chronic atrial fibrillation routinely receive warfarin to prevent the formation of blood clots in the poorly contracting atrium and to decrease the risk of embolism of such clots to the brain or other tissues. Such patients are also often treated with antiarrhythmic drugs. The primary goals of antiarrhythmic treatment are to slow the atrial rate and, most importantly, control the ventricular rate. 1. Which antiarrhythmic drugs are most appropriate for treating chronic atrial fibrillation? 2. Do any of these drugs have significant interactions with warfarin? The Skill Keeper Answers appear at the end of the chapter. 3. Clinical use—Warfarin is used for chronic anticoagulation in all of the clinical situations described previously for heparin, except in pregnant women. 4. Toxicity—Bleeding is the most important adverse effect of warfarin. Early in therapy, a period of hypercoagulability with subsequent dermal vascular necrosis can occur. This is due to deficiency of protein C, an endogenous vitamin K-dependent anticoagulant with a short half-life. Warfarin can cause bone defects and hemorrhage in the developing fetus and, therefore, is contraindicated in pregnancy. Because warfarin has a narrow therapeutic window, its involvement in drug interactions is of major concern. Cytochrome P450-inducing drugs (eg, carbamazepine, phenytoin, rifampin, barbiturates) increase warfarin’s clearance and reduce the anticoagulant effect of a given dose. Cytochrome P450 inhibitors (eg, amiodarone, selective serotonin reuptake inhibitors, cimetidine) reduce warfarin’s clearance and increase the anticoagulant effect of a given dose. Genetic variability in cytochrome P450 2C9 and VKOR affect responses to warfarin. Algorithms to determine initial warfarin dose based on cytochrome P450 2C9 and VKOR, age, body size, and concomitant medications are being tested. THROMBOLYTIC AGENTS A. Classification and Prototypes The thrombolytic drugs used most commonly are either forms of the endogenous tissue plasminogen activator (t-PA; eg, alteplase, tenecteplase, and reteplase) or a protein synthesized by streptococci (streptokinase). All are given intravenously. B. Mechanism of Action Plasmin is an endogenous fibrinolytic enzyme that degrades clots by splitting fibrin into fragments (Figure 34–2). The thrombolytic enzymes catalyze the conversion of the inactive precursor, plasminogen, to plasmin. 280 PART VI Drugs with Important Actions on Blood, Inflammation, & Gout Plasminogen Antiplasmin drugs Fibrinolytics t-PA analogs (eg, alteplase) + Aminocaproic acid, tranexamic acid − Streptokinase + Plasminogen + Plasmin + + Thrombin Degradation products Fibrinogen Fibrin Fibrin split products FIGURE 34–2 Diagram of the fibrinolytic system. The useful thrombolytic drugs are shown on the left. These drugs increase the formation of plasmin, the major fibrinolytic enzyme. Antiplasmin drugs are shown on the right. Aminocaproic acid and tranexamic acid inhibit plasmin formation. 1. Tissue plasminogen activator—t-PA is an enzyme that directly converts plasminogen to plasmin (Figure 34–2). It has little activity unless it is bound to fibrin, which, in theory, should make it selective for the plasminogen that has already bound to fibrin (ie, in a clot) and should result in less danger of widespread production of plasmin and spontaneous bleeding. In fact, t-PA’s selectivity appears to be quite limited. Alteplase is normal human plasminogen activator. Reteplase is a mutated form of human t-PA with similar effects but a slightly faster onset of action and longer duration of action. Tenecteplase is another mutated form of t-PA with a longer half-life. 2. Streptokinase—Streptokinase is obtained from bacterial cultures. Although not itself an enzyme, streptokinase forms a complex with endogenous plasminogen; the plasminogen in this complex undergoes a conformational change that allows it to rapidly convert free plasminogen into plasmin. Unlike the forms of t-PA, streptokinase does not show selectivity for fibrin-bound plasminogen. C. Clinical Use The major application of the thrombolytic agents is as an alternative to percutaneous coronary angioplasty in the emergency treatment of coronary artery thrombosis. Under ideal conditions (ie, treatment within 6 h), these agents can promptly recanalize the occluded coronary vessel. Very prompt use (ie, within 3 h of the first symptoms) of t-PA in patients with ischemic stroke is associated with a significantly better clinical outcome. Cerebral hemorrhage must be positively ruled out before such use. The thrombolytic agents are also used in cases of severe pulmonary embolism. D. Toxicity Bleeding is the most important hazard and has about the same frequency with all the thrombolytic drugs. Cerebral hemorrhage is the most serious manifestation. Streptokinase, a bacterial protein, can evoke the production of antibodies that cause it to lose its effectiveness or induce severe allergic reactions on subsequent therapy. Patients who have had streptococcal infections may have preformed antibodies to the drug. Because they are human proteins, the recombinant forms of t-PA are not subject to this problem. However, they are much more expensive than streptokinase and not much more effective. ANTIPLATELET DRUGS Platelet aggregation contributes to the clotting process (Figure 34–3) and is especially important in clots that form in the arterial circulation. Platelets appear to play a central role in pathologic coronary and cerebral artery occlusion. Platelet aggregation is triggered by a variety of endogenous mediators that include the prostaglandin thromboxane, adenosine diphosphate (ADP), thrombin, and fibrin. Substances that increase intracellular cyclic adenosine monophosphate (cAMP; eg, the prostaglandin prostacyclin, adenosine) inhibit platelet aggregation. CHAPTER 34 Drugs Used in Coagulation Disorders 281 Wall defect vWF C GPIa Degranulation Adenosine EC GPIb Dipyridamole, cilostazol AA TXA 2 COX Platelets Aspirin cAMP AMP PDE − Fibrinogen − − − + + Adenosine − ADP TXA 2 GP IIb/ IIIa − GPIIb/ IIIa − GPIIb/ IIIa − Abciximab, eptifibatide, tirofiban GPIIb/ IIIa Dipyridamole, cilostazol − − Clopidogrel, ticlopidine FIGURE 34–3 Thrombus formation at the site of the damaged vascular wall (EC, endothelials cell) and the role of platelets and clotting factors. Platelet membrane receptors include the glycoprotein (GP) Ia receptor, binding to collagen (C); GP Ib receptor, binding von Willebrand factor (vWF); and GP IIb/IIIa, which binds fibrinogen and other macromolecules. Antiplatelet prostacyclin (PGI2) is released from the endothelium. Aggregating substances released from the degranulating platelet include adenosine diphosphate (ADP), thromboxane A2 (TXA2) and serotonin (5-HT). PDE, phosphodiesterase. (Modified and reproduced, with permission, from Katzung BG, editor: Basic & Clinical Pharmacology, 12th ed. McGraw-Hill, 2012: Fig. 34–1.) A. Classification and Prototypes Antiplatelet drugs include aspirin and other nonsteroidal anti-inflammatory drugs (NSAIDs), glycoprotein IIb/IIIa receptor inhibitors (abciximab, tirofiban, and eptifibatide), antagonists of ADP receptors (clopidogrel, prasugrel, and ticlopidine), and inhibitors of phosphodiesterase 3 (dipyridamole and cilostazol). B. Mechanism of Action Aspirin and other NSAIDs inhibit thromboxane synthesis by blocking the enzyme cyclooxygenase (COX; Chapter 18). Thromboxane A2 is a potent stimulator of platelet aggregation. Aspirin, an irreversible COX inhibitor, is particularly effective. Because platelets lack the machinery for synthesis of new protein, inhibition by aspirin persists for several days until new platelets are formed. Other NSAIDs, which cause a less persistent antiplatelet effect (hours), are not used as antiplatelet drugs and, in fact, can interfere with the antiplatelet effect of aspirin when used in combination with aspirin. Abciximab is a monoclonal antibody that reversibly inhibits the binding of fibrin and other ligands to the platelet glycoprotein IIb/IIIa receptor, a cell surface protein involved in platelet cross-linking. Eptifibatide and tirofiban also reversibly block the glycoprotein IIb/IIIa receptor. Clopidogrel, prasugrel, and the older drug ticlopidine are converted in the liver to active metabolites that irreversibly inhibit the platelet ADP receptor and thereby prevent ADP-mediated platelet aggregation. Ticagrelor is a newer drug that does not require activation and reversibly inhibits the platelet ADP receptor. Dipyridamole and the newer cilostazol appear to have a dual mechanism of action. They prolong the platelet-inhibiting action of intracellular cAMP by inhibiting phosphodiesterase enzymes that degrade cyclic nucleotides, including cAMP, an inhibitor of platelet aggregation, and cyclic guanosine monophosphate (cGMP), a vasodilator (see Chapter 19). They also inhibit the uptake of adenosine by endothelial cells and erythrocytes and thereby increase the plasma concentration of adenosine. Adenosine acts through platelet adenosine A2 receptors to increase platelet cAMP and inhibit aggregation. C. Clinical Use Aspirin is used to prevent further infarcts in persons who have had 1 or more myocardial infarcts and may also reduce the incidence of first infarcts. The drug is used extensively to prevent 282 PART VI Drugs with Important Actions on Blood, Inflammation, & Gout transient ischemic attacks (TIAs), ischemic stroke, and other thrombotic events. The glycoprotein IIb/IIIa inhibitors prevent restenosis after coronary angioplasty and are used in acute coronary syndromes (eg, unstable angina and non-Q-wave acute myocardial infarction). Clopidogrel and ticlopidine are effective in preventing TIAs and ischemic strokes, especially in patients who cannot tolerate aspirin. Clopidogrel is routinely used to prevent thrombosis in patients who have received a coronary artery stent. Dipyridamole is approved as an adjunct to warfarin in the prevention of thrombosis in those with cardiac valve replacement and has been used in combination with aspirin for secondary prevention of ischemic stroke. Cilostazol is used to treat intermittent claudication, a manifestation of peripheral arterial disease. D. Toxicity Aspirin and other NSAIDs cause gastrointestinal and CNS effects (Chapter 36). All antiplatelet drugs significantly enhance the effects of other anticlotting agents. The major toxicities of the glycoprotein IIb/IIIa receptor-blocking drugs are bleeding and, with chronic use, thrombocytopenia. Ticlopidine is used rarely because it causes bleeding in up to 5% of patients, severe neutropenia in about 1%, and very rarely thrombotic thrombocytopenic purpura (TTP), a syndrome characterized by the disseminated formation of small thrombi, platelet consumption, and thrombocytopenia. Clopidogrel is less hematotoxic. The most common adverse effects of dipyridamole and cilostazol are headaches and palpitations. Cilostazol is contraindicated in patients with congestive heart failure because of evidence of reduced survival. DRUGS USED IN BLEEDING DISORDERS Inadequate blood clotting can result from vitamin K deficiency, genetically determined errors of clotting factor synthesis (eg, hemophilia), a variety of drug-induced conditions, and thrombocytopenia. Treatment involves administration of vitamin K, preformed clotting factors, or antiplasmin drugs. Thrombocytopenia can be treated by administration of platelets or oprelvekin, the recombinant form of the megakaryocyte growth factor interleukin-11 (see Chapter 33). A. Vitamin K Deficiency of vitamin K, a fat-soluble vitamin, is most common in older persons with abnormalities of fat absorption and in newborns, who are at risk of bleeding due to vitamin K deficiency. The deficiency is readily treated with oral or parenteral phytonadione (vitamin K1). In the United States, all newborns receive an injection of phytonadione. Large doses of vitamin K1 are used to reverse the anticoagulant effect of excess warfarin. B. Clotting Factors and Desmopressin The most important agents used to treat hemophilia are fresh plasma and purified human blood clotting factors, especially factor VIII (for hemophilia A) and factor IX (for hemophilia B), which are either purified from blood products or produced by recombinant DNA technology. These products are expensive and carry a risk of immunologic reactions and, in the case of factors purified from blood products, infection (although most known blood-borne pathogens are removed by chemical treatment of the plasma extracts.) The vasopressin V2 receptor agonist desmopressin acetate (see Chapter 37) increases the plasma concentration of von Willebrand factor and factor VIII. It is used to prepare patients with mild hemophilia A or von Willebrand disease for elective surgery. C. Antiplasmin Agents Antiplasmin agents are valuable for the prevention or management of acute bleeding episodes in patients with hemophilia and others with a high risk of bleeding disorders. Aminocaproic acid and tranexamic acid are orally active agents that inhibit fibrinolysis by inhibiting plasminogen activation (Figure 34–2). Adverse effects include thrombosis, hypotension, myopathy, and diarrhea. QUESTIONS Questions 1–3. A 55-year-old lawyer is brought to the emergency department 2 h after the onset of severe chest pain during a stressful meeting. He has a history of poorly controlled mild hypertension and elevated blood cholesterol but does not smoke. ECG changes (ST elevation) and cardiac enzymes confirm the diagnosis of myocardial infarction. The decision is made to attempt to open his occluded artery. 1. Which of the following drugs accelerates the conversion of plasminogen to plasmin? (A) Aminocaproic acid (B) Heparin (C) Argatroban (D) Reteplase (E) Warfarin 2. If a fibrinolytic drug is used for treatment of this man’s acute myocardial infarction, which of the following adverse drug effects is most likely to occur? (A) Acute renal failure (B) Development of antiplatelet antibodies (C) Encephalitis secondary to liver dysfunction (D) Hemorrhagic stroke (E) Neutropenia 3. If this patient undergoes a percutaneous coronary angiography procedure and placement of a stent in a coronary blood vessel, he will need to be on dual antiplatelet therapy. eg, aspirin and clopidogrel for at least a year. Which of the following most accurately describes the mechanism of action of clopidogrel? (A) Clopidogrel directly binds to the platelet ADP receptors (B) Clopidogrel irreversibly inhibits cyclooxygenase (C) Clopidogrel facilitates the action of antithrombin III (D) The active metabolite of clopidogrel binds to the platelet ADP receptors (E) The active metabolite of clopidogrel binds to the platelet glycoprotein IIb/IIIa receptors 283 Free warfarin plasma concentration CHAPTER 34 Drugs Used in Coagulation Disorders 0 1 2 3 4 5 6 7 8 9 10 Weeks Drug C Drug B Warfarin Drugs 4. The above graph shows the plasma concentration of free warfarin as a function of time for a patient who was treated with 2 other agents, drugs B and C, on a daily basis at constant dosage starting at the times shown. Which of the following is the most likely explanation for the observed changes in warfarin concentration? (A) Drug B displaces warfarin from plasma proteins; drug C displaces warfarin from tissue-binding sites (B) Drug B inhibits hepatic metabolism of warfarin; drug C displaces drug B from tissue-binding sites (C) Drug B stimulates hepatic metabolism of warfarin; drug C displaces warfarin from plasma protein (D) Drug B increases renal clearance of warfarin; drug C inhibits hepatic metabolism of drug B Questions 5–7. A 58-year-old woman with chronic hypertension and diabetes mellitus was recently admitted to the hospital for congestive heart failure and new onset atrial fibrillation. She is now seeing you after discharge and, though feeling better, is still in atrial fibrillation. An echocardiogram shows an ejection fraction of 40%; there are no valvular abnormalities. An ECG reveals only atrial fibrillation. You calculate her risk using the CHADS(2) system and the score indicates that she requires anticoagulation rather than antiplatelet therapy. 5. You are discussing the risks and benefits of anticoagulation therapy with her, including the option of using direct thrombin inhibitors. Which of the following anticoagulants is a direct inhibitor of thrombin? (A) Abciximab (B) Dabigatran (C) Rivaroxaban (D) Warfarin 6. She tells you that her main reason for not wanting oral anticoagulation is that she does not want to come to clinic for frequent blood draws. You agree on an oral alternative and start her on apixaban. You counsel her extensively on the importance of taking the medication each day, as suddenly stopping can lead to (A) Anaphylaxis (B) Excess bleeding (C) Increase in INR (D) Stroke (E) Thrombocytopenia 7. She is excited about not having to come in for blood tests but wonders if there is a test, just in case the doctors need to know. Which of the following tests would provide accurate information about the coagulation status of a patient taking apixaban? (A) aPTT (B) Factor X test (C) INR (D) PT test Questions 8 and 9. A 67-year-old woman presents with pain in her left thigh muscle. Duplex ultrasonography indicates the presence of deep vein thrombosis (DVT) in the affected limb. 8. The decision was made to treat this woman with enoxaparin. Relative to unfractionated heparin, enoxaparin (A) Can be used without monitoring the patient’s aPTT (B) Has a shorter duration of action (C) Is less likely to have a teratogenic effect (D) Is more likely to be given intravenously (E) Is more likely to cause thrombosis and thrombocytopenia 9. During the next week, the patient was started on warfarin and her enoxaparin was discontinued. Two months later, she returned after a severe nosebleed. Laboratory analysis revealed an INR (international normalized ratio) of 7.0 (INR value in such a warfarin-treated patient should be 2.0–3.0). To prevent severe hemorrhage, the warfarin should be discontinued and this patient should be treated immediately with which of the following? (A) Aminocaproic acid (B) Desmopressin (C) Factor VIII (D) Protamine (E) Vitamin K1 10. A patient develops severe thrombocytopenia in response to treatment with unfractionated heparin and still requires parenteral anticoagulation. The patient is most likely to be treated with which of the following? (A) Abciximab (B) Bivalirudin (C) Tirofiban (D) Plasminogen (E) Vitamin K1 284 PART VI Drugs with Important Actions on Blood, Inflammation, & Gout ANSWERS 1. Reteplase is the only thrombolytic drug listed. Heparin and warfarin are anticoagulants. Argatroban is a direct inhibitor of thrombin, and aminocaproic acid is an inhibitor, not an activator, of the conversion of plasminogen to plasmin. The answer is D. 2. The most common serious adverse effect of the fibrinolytics is bleeding, especially in the cerebral circulation. The fibrinolytics do not usually have serious effects on the renal, hepatic, or hematologic systems. Unlike heparin, they do not induce antiplatelet antibodies. The answer is D. 3. Clopidogrel is a prodrug that is activated by CYP2C9 and CYP2C19. It irreversibly binds to the ADP receptor on the surface of platelets that serves as a key role in platelet aggregation. Aspirin and clopidogrel help prevent platelet-induced occlusion of coronary stents. The answer is D. 4. A drug that increases metabolism (clearance) of the anticoagulant lowers the steady-state plasma concentration (both free and bound forms), whereas one that displaces the anticoagulant increases the plasma level of the free form only until elimination of the drug has again lowered it to the steadystate level. The answer is C. 5. Abciximab is an antiplatelet agent that binds to and inhibits GPIIb/IIIa. Rivaroxaban is an oral factor X inhibitor and warfarin inhibits vitamin K epoxide reductase (VKOR). The answer is B. 6. Due to the shorter half-life of the oral factor X and thrombin inhibitors, the anticoagulant status of the patient changes rapidly. Sudden cessation of short-acting oral anticoagulants can lead to stroke. Excess bleeding is associated with taking any of the anticoagulants not with stopping them. An increase in INR reflects increased anticoagulation by warfarin. Thrombocytopenia is a risk associated with heparin. The answer is D. 7. INR (measured as PT test) reflects changes due to warfarin and to some extent the thrombin inhibitors. Factor X inhibition is not reliably measured by the aPTT (used for unfractionated heparin) or PT test. The answer is B. 8. Enoxaparin is an LMW heparin. LMW heparins have a longer half-life than standard heparin and a more consistent relationship between dose and therapeutic effect. Enoxaparin is given subcutaneously, not intravenously. It is less, not more, likely to cause thrombosis and thrombocytopenia. Neither LMW heparins nor standard heparin are teratogenic. The aPTT is not useful for monitoring the effects of LMW heparins. The answer is A. 9. The elevated INR indicates excessive anticoagulation with a high risk of hemorrhage. Warfarin should be discontinued and vitamin K1 administered to accelerate formation of vitamin K-dependent factors. The answer is E. 10. Direct thrombin inhibitors such as bivalirudin and argatroban provide parenteral anticoagulation similar to that achieved with heparin, but the direct thrombin inhibitors do not induce formation of antiplatelet antibodies. The answer is B. SKILL KEEPER ANSWERS: TREATMENT OF ATRIAL FIBRILLATION (SEE CHAPTERS 13 AND 14) 1. The β-adrenoceptor-blocking drugs (class II; eg, propranolol, acebutolol) and calcium channel-blocking drugs (class IV; eg, verapamil, diltiazem) are useful for atrial fibrillation because they slow atrioventricular (AV) nodal conduction and thereby help control ventricular rate. Though rarely used, digoxin can be effective by increasing the effective refractory period in AV nodal tissue and decreasing AV nodal conduction velocity. If symptoms persist in spite of effective rate control, ablation therapy or class I or class III antiarrhythmic drugs (eg, amiodarone, procainamide, sotalol) can be used in an attempt to provide rhythm control. 2. With warfarin, one is always concerned about pharmacodynamic and pharmacokinetic drug interactions. A metabolite of amiodarone inhibits the metabolism of warfarin and can increase the anticoagulant effect of warfarin. None of the other antiarrhythmic drugs mentioned appears to have significant interactions with warfarin. CHECKLIST When you complete this chapter, you should be able to: ❑ List the 3 major classes of anticlotting drugs and compare their usefulness in venous and arterial thromboses. ❑ Name 3 types of anticoagulants and describe their mechanisms of action. ❑ Explain why the onset of warfarin’s action is relatively slow. ❑ Compare the oral anticoagulants, standard heparin, and LMW heparins with respect to pharmacokinetics, mechanisms, and toxicity. ❑ Give several examples of warfarin’s role in pharmacokinetic and pharmacodynamic drug interactions. ❑ Diagram the role of activated platelets at the site of a damaged blood vessel wall and show where the 4 major classes of antiplatelet drugs act. ❑ Compare the pharmacokinetics, clinical uses, and toxicities of the major antiplatelet drugs. ❑ List 3 drugs used to treat disorders of excessive bleeding. CHAPTER 34 Drugs Used in Coagulation Disorders 285 DRUG SUMMARY TABLE: Drugs Used for Anticoagulation & for Bleeding Disorders Subclass Mechanism of Action Clinical Applications Pharmacokinetics Complexes with antithrombin III • irreversibly inactivates the coagulation factors thrombin and factor Xa Venous thrombosis, pulmonary embolism, myocardial infarction, unstable angina, adjuvant to percutaneous coronary intervention (PCI) and thrombolytics Parenteral administration Toxicities, Drug Interactions Anticoagulants Heparins Unfractionated heparin Bleeding (monitor with aPTT, protamine is reversal agent) • thrombocytopenia • osteoporosis with chronic use LMW heparins (enoxaparin, dalteparin, tinzaparin): more selective anti-factor X activity, more reliable pharmacokinetics with renal elimination, protamine reversal only partially effective, less risk of thrombocytopenia Fondaparinux: effects similar to those of LMW heparins Direct factor X inhibitors Rivaroxaban Venous thrombosis, pulmonary embolism, prevention of stroke in patients with nonvalvular atrial fibrillation Oral administration • fixed dose, no routine monitoring (factor Xa test) Bleeding • no specific reversal agent Bind to thrombin’s active site and inhibit its enzymatic action Anticoagulation in patients with heparininduced thrombocytopenia (HIT) Bivalirudin and argatroban: IV administration Dabigatran: oral administration Both: Bleeding (monitor with aPTT) Inhibits vitamin K epoxide reductase and thereby interferes with production of functional vitamin K-dependent clotting and anticlotting factors Venous thrombosis, pulmonary embolism, prevention of thromboembolic complications of atrial fibrillation or cardiac valve replacement Oral administration • delayed onset and offset of anticoagulant activity • many drug interactions Bleeding (monitor with PT, vitamin K1 is a reversal agent) • thrombosis early in therapy due to protein C deficiency • teratogen Converts plasminogen to plasmin, which degrades the fibrin in thrombi Coronary artery thrombosis, ischemic stroke, pulmonary embolism Parenteral administration Bleeding, especially cerebral hemorrhage Binds to the active site of factor Xa and inhibits its enzymatic action Apixaban and edoxaban: similar to rivaroxaban Direct thrombin inhibitors Buvalirudin, argatroban, and dabigatran Coumadin anticoagulant Warfarin Thrombolytic drugs Alteplase, recombinant human tissue plasminogen activator (t-PA) Reteplase, tenecteplase: similar to alteplase but with a longer half-life Streptokinase: bacterial protein that forms a complex with plasminogen that rapidly converts plasminogen to plasmin. Subject to inactivating antibodies and allergic reactions (Continued ) 286 PART VI Drugs with Important Actions on Blood, Inflammation, & Gout DRUG SUMMARY TABLE: Drugs Used for Anticoagulation & for Bleeding Disorders (Continued ) Subclass Mechanism of Action Clinical Applications Pharmacokinetics Toxicities, Drug Interactions Antiplatelet drugs COX inhibitor Aspirin Nonselective, irreversible COX inhibitor • reduces platelet production of thromboxane A2, a potent stimulator of platelet aggregation Prevention and treatment of arterial thrombosis Dose required for antithrombotic effect is lower than anti-inflammatory dose (see Chapter 36) • duration of activity is longer than pharmacokinetic half-life due to irreversible action Gastrointestinal toxicity, nephrotoxicity • hypersensitivity reaction due to increased leukotrienes; tinnitus, hyperventilation metabolic acidosis, hyperthermia, coma in overdose Used during PCI to prevent restenosis • acute coronary syndrome Parenteral administration Bleeding, thrombocytopenia with prolonged use Oral administration Bleeding, gastrointestinal disturbances, hematologic abnormalities Glycoprotein IIb/IIIa inhibitor (GP IIb/IIIa) Abciximab Inhibits platelet aggregation by interfering with GPIIb/IIIa binding to fibrinogen and other ligands Eptifibatide, tirofiban: Reversible GP IIb/IIIa inhibitors of smaller size than abciximab ADP receptor antagonists Clopidogrel Prodrug: active metabolite by CYP2C9 and CYP2C19 irreversibly inhibits platelet ADP receptor Acute coronary syndrome, prevention of restenosis after PCI, prevention and treatment of arterial thrombosis Ticlopidine: older ADP receptor antagonist with more toxicity, particularly leukopenia and thrombotic thrombocytopenic purpura Prasugrel: newer drug, similar to clopidogrel with less variable kinetics, activation primarily by CYP3A4 Ticagrelor: reversible ADP receptor antagonist that does not require activation Dipyridamole Dipyridamole Inhibits adenosine uptake and inhibits phosphodiesterase enzymes that degrade cyclic nucleotides (cAMP, cGMP) Prevention of thromboembolic complications of cardiac valve replacement • combined with aspirin for secondary prevention of ischemic stroke Oral administration Headache, palpitations, contraindicated in congestive heart failure Vitamin K deficiency, reversal of excessive warfarin anticlotting activity Oral or parenteral administration Severe infusion reaction when given IV or IM Cilostazol: similar to dipyridamole Drugs used in bleeding disorders Reversal agents Vitamin K1 (phytonadione) Increases supply of reduced vitamin K, which is required for synthesis of functional vitamin K-dependent clotting and anticlotting factors Protamine: Cationic form is acidic protein administered parenterally to reverse excessive anticlotting activity of unfractionated heparin (Continued ) CHAPTER 34 Drugs Used in Coagulation Disorders 287 DRUG SUMMARY TABLE: Drugs Used for Anticoagulation & for Bleeding Disorders (Continued ) Subclass Mechanism of Action Clinical Applications Key factor in the clotting cascade Hemophilia A Pharmacokinetics Toxicities, Drug Interactions Clotting factors Factor VIII Parenteral administration Infusion reaction, hypersensitivity reaction Plasma and purified human clotting factors: available to treat other forms of hemophilia Desmopressin: vasopressin V2 receptor agonist increases concentrations of von Willebrand factor and factor VIII (see Chapter 37) Antiplasmin drugs Aminocaproic acid Competitively inhibits plasminogen activation Excessive fibrinolysis Oral or parenteral administration Thrombosis, hypotension, myopathy, diarrhea Tranexamic acid: analog of aminocaproic acid aPTT, activated partial thromboplastin time; cAMP, cyclic adenosine monophosphate; cGMP, cyclic guanosine monophosphate; COX, cyclooxygenase; GP, glycoprotein; PCI, percutaneous coronary intervention. C H A P T E R 35 Agents Used in Dyslipidemia Atherosclerosis is the leading cause of death in the Western world. Drugs discussed in this chapter prevent the sequelae of atherosclerosis (heart attacks, angina, peripheral arterial disease, ischemic stroke) and decrease mortality in patients with a history of cardiovascular disease and hyperlipidemia. Although the drugs are generally safe and effective, they can cause problems, including drug-drug interactions and toxic reactions in skeletal muscle and the liver. Lipid-lowering drugs HMG-CoA reductase inhibitors (eg, lovastatin) Resins Ezetimibe HYPERLIPOPROTEINEMIA A. Pathogenesis Premature or accelerated development of atherosclerosis is strongly associated with elevated concentrations of certain plasma lipoproteins, especially the low-density lipoproteins (LDLs) that participate in cholesterol transport. A depressed level of high-density lipoproteins (HDLs) is also associated with increased risk of atherosclerosis. In some families, hypertriglyceridemia is similarly correlated with atherosclerosis. Chylomicronemia, the occurrence of chylomicrons in the serum while fasting, is a recessive trait that is correlated with a high incidence of acute pancreatitis and managed by restriction of total fat intake (Table 35–1). Regulation of plasma lipoprotein levels involves a complex interplay of dietary fat intake, hepatic processing, and utilization in peripheral tissues (Figure 35–1). Primary disturbances in regulation occur in a number of genetic conditions involving mutations in apolipoproteins, their receptors, transport mechanisms, and lipid-metabolizing enzymes. Secondary disturbances 288 Niacin Fibrates (gemfibrozil) are associated with a Western diet, many endocrine conditions, and diseases of the liver or kidneys. B. Treatment Strategies 1. Diet—Cholesterol and saturated fats are the primary dietary factors that contribute to elevated levels of plasma lipoproteins. Dietary measures designed to reduce the total intake of these substances constitute the first method of management and may be sufficient to reduce lipoprotein levels to a safe range. Because alcohol raises triglyceride and very-low-density lipoprotein (VLDL) levels, it should be avoided by patients with hypertriglyceridemia. 2. Drugs—For an individual patient, the choice of drug treatment is based on the lipid abnormality. The drugs that are most effective at lowering LDL cholesterol include the HMG-CoA reductase inhibitors, resins, ezetimibe, and niacin. The fibric acid derivatives (eg, gemfibrozil), niacin, and marine omega-3 fatty acids are most effective at lowering triglyceride and VLDL concentrations and raising HDL cholesterol concentrations (Table 35–2). CHAPTER 35 Agents Used in Dyslipidemia 289 High-Yield Terms to Learn Lipoproteins Macromolecular complexes in the blood that transport lipids Apolipoproteins Proteins on the surface of lipoproteins; they play critical roles in the regulation of lipoprotein metabolism and uptake into cells Low-density lipoprotein (LDL) Cholesterol-rich lipoprotein whose regulated uptake by hepatocytes and other cells requires functional LDL receptors; an elevated LDL concentration is associated with atherosclerosis High-density lipoprotein (HDL) Cholesterol-rich lipoprotein that transports cholesterol from the tissues to the liver; a low concentration is associated with atherosclerosis Very-low-density lipoprotein (VLDL) Triglyceride- and cholesterol-rich lipoprotein secreted by the liver that transports triglycerides to the periphery; precursor of LDL HMG-CoA reductase 3-Hydroxy-3-methylglutaryl-coenzyme A reductase; the enzyme that catalyzes the rate-limiting step in cholesterol biosynthesis Lipoprotein lipase (LPL) An enzyme found primarily on the surface of endothelial cells that releases free fatty acids from triglycerides in lipoproteins; the free fatty acids are taken up into cells Proliferator-activated receptor-alpha (PPAR-`) Member of a family of nuclear transcription regulators that participate in the regulation of metabolic processes; target of the fibrate drugs and omega-3 fatty acids HMG-CoA REDUCTASE INHIBITORS A. Mechanism and Effects The rate-limiting step in hepatic cholesterol synthesis is conversion of hydroxymethylglutaryl coenzyme A (HMG-CoA) to mevalonate by HMG-CoA reductase. The statins are structural analogs of HMG-CoA that competitively inhibit the enzyme (Figure 35–2). Lovastatin and simvastatin are prodrugs, whereas the other HMG-CoA reductase inhibitors (atorvastatin, fluvastatin, pravastatin, and rosuvastatin) are active as given. Although the inhibition of hepatic cholesterol synthesis contributes a small amount to the total serum cholesterol-lowering effect of these drugs, a much greater effect derives from the response to a reduction in a tightly regulated hepatic pool of cholesterol. The liver compensates by increasing the number of high-affinity LDL receptors, which clear LDL and VLDL TABLE 35–1 Primary hyperlipoproteinemias and their drug treatment. Condition/Cause Manifestations, Cause Single Drug Drug Combination Primary chylomicronemia Chylomicrons, VLDL increased; deficiency in LPL or apoC-II Dietary management (omega-3 fatty acids, niacin, or fibrate) Niacin plus fibratea VLDL, chylomicrons increased; decreased clearance of VLDL VLDL increased, chylomicrons may be increased; increased production of VLDL Omega-3 fatty acids, niacin or fibrate Omega-3 fatty acids, niacin or fibrate Niacin plus fibrate Omega-3 fatty acids, niacin, fibrate, statin Niacin, statin, ezetimibe Two or 3 of the individual drugs Two or 3 of the individual drugs Statin plus niacin or fibrate Familial hypertriglyceridemia Severe Moderate Familial combined hyperlipoproteinemia Increased hepatic apoB and VLDL production VLDL increased LDL increased Niacin plus fibrate VLDL, LDL increased Omega-3 fatty acids, niacin, statin Familial dysbetalipoproteinemia VLDL remnants, chylomicron remnants increased; deficiency in apoE Omega-3 fatty acids, fibrate, statin, or niacin Fibrate plus niacin, or either plus statin Familial hypercholesterolemia Heterozygous LDL increased; defect in LDL receptors Statin, resin, niacin, ezetimibe Two or 3 of the individual drugs Niacin, atorvastatin, rosuvastatin, ezetimibe, mipomersen, or lomitapide Niacin plus statin plus ezetimibe Homozygous a Single-drug therapy with marine omega-3 dietary supplement should be evaluated before drug combinations are used. Modified and reproduced, with permission, from Katzung BG, editor: Basic & Clinical Pharmacology, 13th ed. McGraw-Hill, 2014. 290 PART VI Drugs with Important Actions on Blood, Inflammation, & Gout Hepatocyte Golgi vesicle B-100 Blood ApoE Capillary endothelium ApoC RER HDL Lipoprotein lipase VLDL ApoB, ApoE, ApoC LDL receptor Cholesterol VLDL remnant Lysosome * Mevalonic acid FFA * HDL LDL HMG-CoA reductase Peripheral cell AcetylCoA Cholesterol biosynthetic pathway Cholesterol Lysosome Cholesteryl esters FIGURE 35–1 Metabolism of lipoproteins of hepatic origin. The heavy arrows show the primary pathways. Nascent VLDL are secreted via the Golgi apparatus. They acquire additional apoC lipoproteins and apoE from HDL. VLDL is converted to VLDL remnants by lipolysis via lipoprotein lipase associated with capillaries in peripheral tissue supplies. In the process, C apolipoproteins and a portion of apoE are given back to HDL. Some of the VLDL remnants are converted to LDL by further loss of triglycerides and loss of apoE. A major pathway for LDL degradation involves the endocytosis of LDL by LDL receptors in the liver and the peripheral tissues, for which apoB-100 is the ligand. Dark color denotes cholesteryl esters; light color, triglycerides; the asterisk denotes a functional ligand for LDL receptors; triangles indicate apoE; circles and squares represent C apolipoproteins. FFA, free fatty acid; RER, rough endoplasmic reticulum. (Reproduced, with permission, from Katzung BG, editor: Basic & Clinical Pharmacology, 12th ed. McGraw-Hill, 2012: Fig. 35–1.) TABLE 35−2 Lipid-modifying effects of antihyperlipidemic drugs. Drug or Drug Group LDL Cholesterol HDL Cholesterol Triglycerides Statins Atorvastatin, rosuvastatin, simvastatin Lovastatin, pravastatin Fluvastatin −25 to −50% −25 to −40% −20 to −30% +5 to +15% +5 to +10% +5 to +10% ↓↓ ↓ ↓ Resins −15 to −25% +5 to +10% ±a −20% +5% ± −15 to −25% +25 to +35% ↓↓ b +15 to +20% ↓↓ Ezetimibe Niacin Gemfibrozil −10 to −15% LDL, low-density lipoprotein; HDL, high-density lipoprotein; ±, variable, if any. a Resins can increase triglycerides in some patients with combined hyperlipidemia. b Gemfibrozil and other fibrates can increase LDL cholesterol in patients with combined hyperlipidemia. Modified and reproduced, with permission, from McPhee SJ, Papadakis MA, Tierney LM, editors: Current Medical Diagnosis & Treatment, 46th ed. McGraw-Hill, 2006. CHAPTER 35 Agents Used in Dyslipidemia Blood Hepatocyte Gut Acetyl-CoA LDL B-100 HMG-CoA R HMG-CoA reductase inhibitors Cholesterol Ezetimibe VLDL B-100 Niacin Bile acids Resins FIGURE 35–2 Sites of action of HMG-coA reductase inhibitors, niacin, ezetimibe, and bile acid-binding resins. Low-density lipoprotein (LDL) receptor synthesis is increased by treatment with drugs that reduce the hepatocyte reserve of cholesterol. (Reproduced, with permission, from Katzung BG, editor: Basic & Clinical Pharmacology, 12th ed. McGraw-Hill, 2012: Fig. 35–2.) remnants from the blood (Figure 35–1). Note that functional LDL receptors are required to achieve a therapeutic LDL-lowering effect with reductase inhibitors. HMG-CoA reductase inhibitors also have direct anti-atherosclerotic effects and anti-inflammatory effects and have been shown to prevent bone loss. B. Clinical Use Statins can reduce LDL cholesterol levels dramatically (Table 35–2), especially when used in combination with other cholesterol-lowering drugs (Table 35–1). These drugs are used commonly because they are effective and well tolerated. Large clinical trials have shown that they reduce the risk of coronary events and mortality in patients with ischemic heart disease, and they also reduce the risk of ischemic stroke. Rosuvastatin, atorvastatin, and simvastatin have greater maximal efficacy than the other HMG-CoA reductase inhibitors. These drugs also reduce triglycerides and increase HDL cholesterol in patients with triglycerides levels that are higher than 250 mg/dL and with reduced HDL cholesterol levels. Fluvastatin has less maximal efficacy than the other drugs in this group. C. Toxicity Mild elevations of serum aminotransferases are common but are not often associated with hepatic damage. Patients with preexisting liver 291 disease may have more severe reactions. An increase in creatine kinase (released from skeletal muscle) is noted in about 10% of patients; in a few, severe muscle pain and even rhabdomyolysis may occur. HGMCoA reductase inhibitors are metabolized by the cytochrome P450 system; drugs or foods (eg, grapefruit juice) that inhibit cytochrome P450 activity increase the risk of hepatotoxicity and myopathy. Because of evidence that the HMG-CoA reductase inhibitors are teratogenic, these drugs should be avoided in pregnancy. SKILL KEEPER: ANGINA (SEE CHAPTER 12) The antihyperlipidemic drugs, especially the HMG-CoA reductase inhibitors, are commonly used to treat patients with ischemic heart disease. One of the most common manifestations of ischemic heart disease and coronary atherosclerosis is angina. 1. What are the 3 major forms of angina? 2. Name the 3 major drug groups used to treat angina and specify which form of angina each is useful for. The Skill Keeper Answers appear at the end of the chapter. RESINS A. Mechanism and Effects Normally, over 90% of bile acids, metabolites of cholesterol, are reabsorbed in the gastrointestinal tract and returned to the liver for reuse. Bile acid-binding resins (cholestyramine, colestipol, and colesevelam) are large nonabsorbable polymers that bind bile acids and similar steroids in the intestine and prevent their absorption (Figure 35–2). By preventing the recycling of bile acids, bile acid-binding resins divert hepatic cholesterol to synthesis of new bile acids, thereby reducing the amount of cholesterol in a tightly regulated pool. A compensatory increase in the synthesis of high-affinity LDL receptors increases the removal of LDL lipoproteins from the blood. The resins cause a modest reduction in LDL cholesterol (Table 35–2) but have little effect on HDL cholesterol or triglycerides. In some patients with a genetic condition that predisposes them to hypertriglyceridemia and hypercholesterolemia (familial combined hyperlipidemia), resins increase triglycerides and VLDL. B. Clinical Use The resins are used in patients with hypercholesterolemia (Table 35–1). They have also been used to reduce pruritus in patients with cholestasis and bile salt accumulation. C. Toxicity Adverse effects from resins include bloating, constipation, and an unpleasant gritty taste. Absorption of vitamins (eg, vitamin K, dietary folates) and drugs (eg, thiazide diuretics, warfarin, pravastatin, fluvastatin) is impaired by the resins. 292 PART VI Drugs with Important Actions on Blood, Inflammation, & Gout EZETIMIBE A. Mechanism and Effects Ezetimibe is a prodrug that is converted in the liver to the active glucuronide form. This active metabolite inhibits a transporter that mediates gastrointestinal uptake of cholesterol and phytosterols (plant sterols that normally enter gastrointestinal epithelial cell but then are immediately transported back into the intestinal lumen). By preventing absorption of dietary cholesterol and cholesterol that is excreted in bile, ezetimibe reduces the cholesterol in the tightly regulated hepatic pool. A compensatory increase in the synthesis of high-affinity LDL receptors increases the removal of LDL lipoproteins from the blood. As monotherapy, ezetimibe reduces LDL cholesterol by about 20% (Table 35–2). When combined with an HMG-CoA reductase inhibitor, it is even more effective. B. Clinical Use Ezetimibe is used for treatment of hypercholesterolemia and phytosterolemia, a rare genetic disorder that results from impaired export of phytosterols. C. Toxicity Ezetimibe is well tolerated. When combined with HMG-CoA reductase inhibitors, it may increase the risk of hepatic toxicity. Serum concentrations of the glucuronide form are increased by fibrates and reduced by cholestyramine. NIACIN (NICOTINIC ACID) A. Mechanism and Effects Through multiple actions, niacin (but not nicotinamide) reduces LDL cholesterol, triglycerides, and VLDL and also often increases HDL cholesterol. In the liver, niacin reduces VLDL synthesis, which in turn reduces LDL levels (Figures 35–1 and 35–2). In adipose tissue, niacin appears to activate a signaling pathway that reduces hormone-sensitive lipase activity and thus decreases plasma fatty acid and triglyceride levels. Consequently, LDL formation is reduced, and there is a decrease in LDL cholesterol. Increased clearance of VLDL by the lipoprotein lipase associated with capillary endothelial cells has also been demonstrated and probably accounts for the reduction in plasma triglyceride concentrations. Niacin reduces the catabolic rate for HDL. Finally, niacin decreases circulating fibrinogen and increases tissue plasminogen activator. B. Clinical Use Because it lowers serum LDL cholesterol and triglyceride concentrations and increases HDL cholesterol concentrations, niacin has wide clinical usefulness in the treatment of hypercholesterolemia, hypertriglyceridemia, and low levels of HDL cholesterol. C. Toxicity Cutaneous flushing is a common adverse effect of niacin. Pretreatment with aspirin or other nonsteroidal anti-inflammatory drugs (NSAIDs) reduces the intensity of this flushing, suggesting that it is mediated by prostaglandin release. Tolerance to the flushing reaction usually develops within a few days. Dose-dependent nausea and abdominal discomfort often occur. Pruritus and other skin conditions are reported. Moderate elevations of liver enzymes and even severe hepatotoxicity may occur. Severe liver dysfunction has been associated with an extended-release preparation, which is not the same as the sustained-release formulation. Hyperuricemia occurs in about 20% of patients, and carbohydrate tolerance may be moderately impaired. FIBRIC ACID DERIVATIVES A. Mechanism and Effects Fibric acid derivatives (eg, gemfibrozil, fenofibrate) are ligands for the peroxisome proliferator-activated receptor-alpha (PPAR-α) protein, a receptor that regulates transcription of genes involved in lipid metabolism. This interaction with PPAR-α results in increased synthesis by adipose tissue of lipoprotein lipase, which associates with capillary endothelial cells and enhances clearance of triglyceride-rich lipoproteins (Figure 35–1). In the liver, fibrates stimulate fatty acid oxidation, which limits the supply of triglycerides and decreases VLDL synthesis. They also decrease expression of apoC-III, which impedes the clearance of VLDL, and increases the expression of apoA-I and apoA-II, which in turn increases HDL levels. In most patients, fibrates have little or no effect on LDL concentrations. However, fibrates can increase LDL cholesterol in patients with a genetic condition called familial combined hyperlipoproteinemia, which is associated with a combined increase in VLDL and LDL. B. Clinical Use Gemfibrozil and other fibrates are used to treat hypertriglyceridemia. Because these drugs have only a modest ability to reduce LDL cholesterol and can increase LDL cholesterol in some patients, they often are combined with other cholesterol-lowering drugs for treatment of patients with elevated concentrations of both LDL and VLDL. C. Toxicity Nausea is the most common adverse effect with all members of the fibric acid derivatives subgroup. Skin rashes are common with gemfibrozil. A few patients show decreases in white blood count or hematocrit, and these drugs can potentiate the action of anticoagulants. There is an increased risk of cholesterol gallstones; these drugs should be used with caution in patients with a history of cholelithiasis. When used in combination with reductase inhibitors, the fibrates significantly increase the risk of myopathy. COMBINATION THERAPY All patients with hyperlipidemia are treated first with dietary modification, but this is often insufficient and drugs must be added. Drug combinations are often required to achieve the CHAPTER 35 Agents Used in Dyslipidemia maximum lowering possible with minimum toxicity and to achieve the desired effect on the various lipoproteins (LDL, VLDL, and HDL). Certain drug combinations provide advantages (Table 35–1), whereas others present specific challenges. Because resins interfere with the absorption of certain HMG-CoA reductase inhibitors (pravastatin, cerivastatin, atorvastatin, and fluvastatin), these must be given at least 1 h before or 4 h after the resins. The combination of reductase inhibitors with either fibrates or niacin increases the risk of myopathy. DRUGS RESTRICTED TO PATIENTS WITH HOMOZYGOUS FAMILIAL HYPERCHOLESTEROLEMIA Lomitapide is a microsomal triglyceride transfer protein (MTP) inhibitor. MTP plays an essential role in the accretion of triglycerides to nascent VLDL in liver and to chylomicrons in the intestine. Its inhibition decreases VLDL secretion and consequently the accumulation of LDL in plasma. An adverse effect is that it can cause accumulation of triglycerides in the liver and elevations in transaminases. Mipomersen is an antisense oligonucleotide that targets apoB100, mainly in the liver. Mild to moderate injection site reactions and flu-like symptoms can occur. QUESTIONS 1. PJ is a 4.5-year-old boy. At his checkup, the pediatrician notices cutaneous xanthomas and orders a lipid panel. Repeated measures confirm that the patient’s serum cholesterol levels are high (936 mg/dL). Further testing confirms a diagnosis of homozygous familial hypercholesterolemia. Which of the following interventions will be least effective in this patient? (A) Atorvastatin (B) Ezetimibe (C) Lomitapide (D) Mipomersen (E) Niacin 2. A 46-year-old woman with a history of hyperlipidemia was treated with a drug. The chart below shows the results of the patient’s fasting lipid panel before treatment and 6 mo after initiating drug therapy. Normal values are also shown. Which of the following drugs is most likely to be the one that this patient received? (A) Colestipol (B) Ezetimibe (C) Gemfibrozil (D) Lovastatin (E) Niacin Time of Lipid Measurement Before treatment Six months after starting treatment Normal values 293 Questions 3–6. A 35-year-old woman appears to have familial combined hyperlipidemia. Her serum concentrations of total cholesterol, LDL cholesterol, and triglyceride are elevated. Her serum concentration of HDL cholesterol is somewhat reduced. 3. Which of the following drugs is most likely to increase this patient’s triglyceride and VLDL cholesterol concentrations when used as monotherapy? (A) Atorvastatin (B) Cholestyramine (C) Ezetimibe (D) Gemfibrozil (E) Niacin 4. If this patient is pregnant, which of the following drugs should be avoided because of a risk of harming the fetus? (A) Cholestyramine (B) Ezetimibe (C) Fenofibrate (D) Niacin (E) Pravastatin 5. The patient is started on gemfibrozil. Which of the following is a major mechanism of gemfibrozil’s action? (A) Increased excretion of bile acid salts (B) Increased expression of high-affinity LDL receptors (C) Increased secretion of VLDL by the liver (D) Increased triglyceride hydrolysis by lipoprotein lipase (E) Reduced uptake of dietary cholesterol 6. Which of the following is a major toxicity associated with gemfibrozil therapy? (A) Bloating and constipation (B) Cholelithiasis (C) Hyperuricemia (D) Liver damage (E) Severe cardiac arrhythmia Questions 7–10. A 43-year-old man has heterozygous familial hypercholesterolemia. His serum concentrations of total cholesterol and LDL are markedly elevated. His serum concentration of HDL cholesterol, VLDL cholesterol, and triglycerides are normal or slightly elevated. The patient’s mother and older brother died of myocardial infarctions before the age of 50. This patient recently experienced mild chest pain when walking upstairs and has been diagnosed as having angina of effort. The patient is somewhat overweight. He drinks alcohol most evenings and smokes about 1 pack of cigarettes per week. 7. Consumption of alcohol is associated with which of the following changes in serum lipid concentrations? (A) Decreased chylomicrons (B) Decreased HDL cholesterol (C) Decreased VLDL cholesterol (D) Increased LDL cholesterol (E) Increased triglyceride Triglyceride Total Cholesterol LDL Cholesterol VLDL Cholesterol HDL Cholesterol 1000 640 120 500 20 300 275 90 150 40 <150 <200 <130 <30 >35 294 PART VI Drugs with Important Actions on Blood, Inflammation, & Gout 8. If the patient has a history of gout, which of the following drugs is most likely to exacerbate this condition? (A) Colestipol (B) Ezetimibe (C) Gemfibrozil (D) Niacin (E) Simvastatin 9. After being counseled about lifestyle and dietary changes, the patient was started on atorvastatin. During his treatment with atorvastatin, it is important to routinely monitor serum concentrations of which of the following? (A) Blood urea nitrogen (B) Alanine and aspartate aminotransferase (C) Platelets (D) Red blood cells (E) Uric acid 10. Six months after beginning atorvastatin, the patient’s total and LDL cholesterol concentrations remained above normal, and he continued to have anginal attacks despite good adherence to his antianginal medications. His physician decided to add ezetimibe. Which of the following is the most accurate description of ezetimibe’s mechanism of an action? (A) Decreased lipid synthesis in adipose tissue (B) Decreased secretion of VLDL by the liver (C) Decreased gastrointestinal absorption of cholesterol (D) Increased endocytosis of HDL by the liver (E) Increased lipid hydrolysis by lipoprotein lipase 4. The HMG-CoA reductase inhibitors are contraindicated in pregnancy because of the risk of teratogenic effects. The answer is E. 5. A major mechanism recognized for gemfibrozil is increased activity of the lipoprotein lipase associated with capillary endothelial cells. Gemfibrozil and other fibrates decrease VLDL secretion, presumably by stimulating hepatic fatty acid oxidation. The answer is D. 6. A major toxicity of the fibrates is increased risk of gallstone formation, which may be due to enhanced biliary excretion of cholesterol. The answer is B. 7. Chronic ethanol ingestion can increase serum concentrations of VLDL and triglycerides. This is one of the factors that places patients with alcoholism at risk of pancreatitis. Chronic ethanol ingestion also has the possibly beneficial effect of raising, not decreasing, serum HDL concentrations. The answer is E. 8. Niacin can exacerbate both hyperuricemia and glucose intolerance. The answer is D. 9. The 2 primary adverse effects of the HMG-CoA reductase inhibitors are hepatotoxicity and myopathy. Patients taking these drugs should have liver function tests performed before starting therapy, and at regular intervals as needed during therapy. Serum concentrations of alanine and aspartate aminotransferase are used as markers of hepatocellular toxicity. The answer is B. 10. The major recognized effect of ezetimibe is inhibition of absorption of cholesterol in the intestine. The answer is C. ANSWERS 1. Homozygous familial hypercholesterolemia is caused by mutations leading to dysfunctional LDL receptors incapable of taking up LDL from the bloodstream. Options B–E would have a cholesterol-lowering effect. Lomitapide and mipomersen are specifically indicated for patients with familial hypercholesterolemia. Reductase inhibitors such as atorvastatin rely on functional LDL receptors to achieve a LDL-lowering effect and thus will not work in patients with homozygous familial hypercholesterolemia. The answer is A. 2. This patient presents with striking hypertriglyceridemia, elevated VLDL cholesterol, and depressed HDL cholesterol. Six months after drug treatment was initiated, her triglyceride and VLDL cholesterol have dropped dramatically and her HDL cholesterol level has doubled. The drug that is most likely to have achieved all of these desirable changes, particularly the large increase in HDL cholesterol, is niacin. Although gemfibrozil lowers triglyceride and VLDL concentrations, it does not cause such large increases in HDL cholesterol and decreases in LDL cholesterol. The answer is E. 3. In some patients with familial combined hyperlipidemia and elevated VLDL, the resins increase VLDL and triglyceride concentrations even though they also lower LDL cholesterol. The answer is B. SKILL KEEPER ANSWERS: ANGINA (SEE CHAPTER 12) 1. The 3 major forms of angina are (1) angina of effort, which is associated with a fixed plaque that partially occludes 1 or more coronary arteries; (2) vasospastic angina, which involves unpredictably timed, reversible coronary spasm; and (3) unstable angina, which often immediately precedes a myocardial infarction and requires emergency treatment. 2. The 3 major drug groups used in angina are nitrates, calcium channel blockers, and β blockers. Nitrates are used in all 3 types of angina. Calcium channel blockers are useful for treatment of angina of effort and vasospastic angina. They can be added to β blockers and nitroglycerin in patients with refractory unstable angina. β blockers are not useful in vasospastic angina or for an acute attack of angina of effort. They are primarily used for prophylaxis of angina of effort and also in emergency treatment of acute coronary syndromes. CHAPTER 35 Agents Used in Dyslipidemia 295 CHECKLIST When you complete this chapter, you should be able to: ❑ Describe the proposed role of lipoproteins in the formation of atherosclerotic plaques. ❑ Describe the dietary management of hyperlipidemia. ❑ List the 5 main classes of drugs used to treat hyperlipidemia. For each, describe the mechanism of action, effects on serum lipid concentrations, and adverse effects. ❑ On the basis of a set of baseline serum lipid values, propose a rational drug treatment regimen. ❑ Argue the merits of combined drug therapy for some diseases, and list 3 rational drug combinations. DRUG SUMMARY TABLE: Drugs for the Treatment of Hyperlipidemias Subclass Toxicities, Drug Interactions Mechanism of Action Clinical Applications Pharmacokinetics Inhibit HMG-CoA reductase Atherosclerotic vascular disease (primary and secondary prevention) • acute coronary syndromes Oral administration • P450-dependent metabolism (CYP3A4, CYP2C9) interacts with P450 inhibitors/competitors Myopathy, hepatic dysfunction, teratogen Statins Atorvastatin, simvastatin, rosuvastatin Fluvastatin, pravastatin, lovastatin: similar but somewhat less efficacious Fibrates Gemfibrozil, fenofibrate PPAR-α agonistsa Hypertriglyceridemia, low HDL cholesterol Oral administration Myopathy, hepatic dysfunction, cholestasis Prevents reabsorption of bile acids from the gastrointestinal tract Elevated LDL cholesterol, pruritus Oral administration • interferes with absorption of some drugs and vitamins Constipation, bloating Reduces intestinal uptake of cholesterol by inhibiting sterol transporter NPC1L1 Elevated LDL cholesterol, phytosterolemia Oral administration Rarely, hepatic dysfunction, myositis Decreases VLDL synthesis and LDL cholesterol concentrations • increases HDL cholesterol Low HDL cholesterol, elevated VLDL and LDL Oral administration Gastrointestinal irritation, flushing, hepatic toxicity, hyperuricemia, may reduce glucose tolerance Bile acid-binding resins Colestipol Cholestyramine, colesevelam: similar to colestipol Sterol absorption inhibitor Ezetimibe Niacin a PPAR-α, peroxisome proliferator-activated receptor-alpha. Also responsible for TG-lowering effect of omega-3 fatty acids. C NSAIDs, Acetaminophen, & Drugs Used in Rheumatoid Arthritis & Gout Inflammation is a complex response to cell injury that primarily occurs in vascularized connective tissue and often involves the immune response. The mediators of inflammation function to eliminate the cause of cell injury and clear away debris, in preparation for tissue repair. Unfortunately, inflammation also causes pain and, in instances in which the cause of cell injury is not eliminated, can result in a chronic condition of pain and tissue damage such as that seen in rheumatoid arthritis. The nonsteroidal anti-inflammatory drugs (NSAIDs) and H A P T E R 36 acetaminophen are often effective in controlling inflammatory pain. Other treatment strategies applied to the reduction of inflammation are targeted at immune processes. These include glucocorticoids and disease-modifying antirheumatic drugs (DMARDs). Gout is a metabolic disease associated with precipitation of uric acid crystals in joints. Treatment of acute episodes targets inflammation, whereas treatment of chronic gout targets both inflammatory processes and the production and elimination of uric acid. Anti-inflammatory drugs, acetaminophen, drugs used in gout Anti-inflammatory drugs NSAIDs Aspirin 296 Other nonselective NSAIDs Acetaminophen DMARDs COX-2 inhibitors (celecoxib) NSAIDs Drugs used in gout Acute Glucocorticoids Chronic Colchicine Uricosurics (probenecid) Xanthine oxidase inhibitors (allopurinol, febuxostat) CHAPTER 36 NSAIDs, Acetaminophen, & Drugs Used in Rheumatoid Arthritis & Gout 297 High-Yield Terms to Learn Antipyretic A drug that reduces fever (eg, aspirin, other NSAIDs, acetaminophen) Cyclooxygenase (COX), lipoxygenase (LOX) The enzymes responsible for prostaglandin (COX) and leukotriene (LOX) synthesis (Figure 36–2) Cytotoxic drug Drugs that interfere with essential processes, especially DNA maintenance and replication and cell division. Such drugs generally kill rapidly dividing cells and are used for cancer chemotherapy and immunosuppression (Chapters 54 and 55) Disease-modifying antirheumatic drugs (DMARDs) Diverse group of drugs that modify the inflammatory processes underlying rheumatoid arthritis and similar autoimmune conditions; they have a slow (weeks to months) onset of clinical effects Nonsteroidal anti-inflammatory drugs (NSAIDs) Inhibitors of cyclooxygenase; the term nonsteroidal differentiates them from corticosteroid drugs (eg, cortisol; Chapter 39) Reye’s syndrome A rare syndrome of rapid liver degeneration and encephalopathy in children treated with aspirin during a viral infection Tumor necrosis factor-` (TNF-`) A cytokine that plays a central role in inflammation Uricosuric agent A drug that increases the renal excretion of uric acid Xanthine oxidase A key enzyme in the purine metabolism pathway that converts hypoxanthine to xanthine and xanthine to uric acid ASPIRIN & OTHER NONSELECTIVE NSAIDs TABLE 36–1 Selected NSAIDs. Drug Half-life (hr) A. Classification and Prototypes Aspirin (acetylsalicylic acid) is the prototype of the salicylates and other NSAIDs (Table 36–1). The other older nonselective NSAIDs (ibuprofen, indomethacin, many others) vary primarily in their potency, analgesic and anti-inflammatory effectiveness, and duration of action. Ibuprofen and naproxen have moderate effectiveness; indomethacin has greater anti-inflammatory effectiveness; and ketorolac has greater analgesic effectiveness. Celecoxib was the first member of a newer NSAID subgroup, the cyclooxygenase-2 (COX-2)-selective inhibitors, which were developed in an attempt to lessen the gastrointestinal toxicity associated with COX inhibition while preserving efficacy. Unfortunately, clinical trials involving some of the highly selective COX-2 inhibitors have shown a higher incidence of cardiovascular thrombotic events than the nonselective drugs. Aspirin Nabumetone 26 B. Mechanism of Action As noted in Chapter 18, cyclooxygenase is the enzyme that converts arachidonic acid into the endoperoxide precursors of prostaglandins, important mediators of inflammation (Figure 36–1). Cyclooxygenase has at least 2 isoforms: COX-1 and COX-2. COX-1 is primarily expressed in noninflammatory cells, whereas COX-2 is expressed in activated lymphocytes, polymorphonuclear cells, and other inflammatory cells. Aspirin and nonselective NSAIDs inhibit both cyclooxygenase isoforms and thereby decrease prostaglandin and thromboxane synthesis throughout the body. Release of prostaglandins necessary Naproxen 14 Oxaprozin 58 Piroxicam 57 0.25 Celecoxib 11 Diclofenac 1.1 Diflunisal 13 Etodolac 6.5 Fenoprofen 2.5 Flurbiprofen 3.8 Ibuprofen 2 Indomethacin 4–5 Ketoprofen 1.8 Meloxicam 20 a b Salicylate 2–19 Sulindac 8 Tolmetin 1 a Nabumetone is a prodrug; the half-life is for its active metabolite. b Major anti-inflammatory metabolite of aspirin. Salicylate is usually given in the form of aspirin. (Modified and reproduced, with permission, from Katzung BG, editors: Basic & Clinical Pharmacology, 13th ed. McGraw-Hill, 2014.) 298 PART VI Drugs with Important Actions on Blood, Inflammation, & Gout Stimulus Disturbance of cell membranes Phospholipids Phospholipase inhibitors Corticosteroids − Phospholipase A2 Arachidonic acid − Lipoxygenase inhibitors Lipoxygenase Cyclooxygenase − Fatty acid substitution (diet) NSAID, ASA Leukotrienes LTC4/D4/E4 LTB4 Phagocyte attraction, activation Alteration of vascular permeability, bronchial constriction, increased secretion − Colchicine Inflammation Thromboxane Prostacyclin − Receptor antagonists Prostaglandins Leukocyte modulation Inflammation Bronchospasm, congestion, mucous plugging FIGURE 36–1 Prostanoid mediators derived from arachidonic acid and sites of drug action. ASA, acetylsalicylic acid (aspirin); LT, leukotriene; NSAID, nonsteroidal anti-inflammatory drug. (Reproduced, with permission, from Katzung BG, editor: Basic & Clinical Pharmacology, 12th ed. McGraw-Hill, 2012: Fig. 36–2.) for homeostatic function is disrupted, as is release of prostaglandins involved in inflammation. The COX-2-selective inhibitors have less effect on the prostaglandins involved in homeostatic function, particularly those in the gastrointestinal tract. The major difference between the mechanisms of action of aspirin and other NSAIDs is that aspirin (but not its active metabolite, salicylate) acetylates and thereby irreversibly inhibits cyclooxygenase, whereas the inhibition produced by other NSAIDs is reversible. The irreversible action of aspirin results in a longer duration of its antiplatelet effect and is the basis for its use as an antiplatelet drug (Chapter 34). C. Effects Arachidonic acid derivatives are important mediators of inflammation; cyclooxygenase inhibitors reduce the manifestations of inflammation, although they have no effect on underlying tissue damage or immunologic reactions. These inhibitors also suppress the prostaglandin synthesis in the CNS that is stimulated by pyrogens and thereby reduce fever (antipyretic action). The analgesic mechanism of these agents is less well understood. Activation of peripheral pain sensors may be diminished as a result of reduced production of prostaglandins in injured tissue; in addition, a central mechanism is operative. Cyclooxygenase inhibitors also interfere with the homeostatic function of prostaglandins. Most important, they reduce prostaglandin-mediated cytoprotection in the gastrointestinal tract and autoregulation of renal function. D. Pharmacokinetics and Clinical Use 1. Aspirin—Aspirin has 3 therapeutic dose ranges: The low range (<300 mg/d) is effective in reducing platelet aggregation; intermediate doses (300–2400 mg/d) have antipyretic and analgesic effects; and high doses (2400–4000 mg/d) are used for an anti-inflammatory effect. Aspirin is readily absorbed and is hydrolyzed in blood and tissues to acetate and salicylic acid. Salicylate is a reversible nonselective inhibitor of cyclooxygenase. Elimination of salicylate is first order at low doses, with a half-life of 3–5 h. At high (anti-inflammatory) doses, half-life increases to 15 h or more and elimination becomes zero order. Excretion is via the kidney. CHAPTER 36 NSAIDs, Acetaminophen, & Drugs Used in Rheumatoid Arthritis & Gout 2. Other NSAIDs—The other NSAIDs are well absorbed after oral administration. Ibuprofen has a half-life of about 2 h, is relatively safe, and is the least expensive of the older, nonselective NSAIDs. Naproxen and piroxicam are noteworthy because of their longer half-lives (Table 36–1), which permit less frequent dosing. These other NSAIDs are used for the treatment of mild to moderate pain, especially the pain of musculoskeletal inflammation such as that seen in arthritis and gout. They are also used to treat many other conditions, including dysmenorrhea, headache, and patent ductus arteriosus in premature infants. Ketorolac is notable as a drug used mainly as a systemic analgesic, not as an anti-inflammatory (although it has typical nonselective NSAID properties). It is the only NSAID available in a parenteral formulation. Nonselective NSAIDs reduce polyp formation in patients with primary familial adenomatous polyposis. Long-term use of NSAIDs reduces the risk of colon cancer. E. Toxicity 1. Aspirin—The most common adverse effect from therapeutic anti-inflammatory doses of aspirin is gastric upset. Chronic use can result in gastric ulceration, upper gastrointestinal bleeding, and renal effects, including acute failure and interstitial nephritis. Aspirin increases the bleeding time (Chapter 34). When prostaglandin synthesis is inhibited by even small doses of aspirin, persons with aspirin hypersensitivity (especially associated with nasal polyps) can experience asthma from the increased synthesis of leukotrienes. This type of hypersensitivity to aspirin precludes treatment with any NSAID. At higher doses of aspirin, tinnitus, vertigo, hyperventilation, and respiratory alkalosis are observed. At very high doses, the drug causes metabolic acidosis, dehydration, hyperthermia, collapse, coma, and death. Children with viral infections who are treated with aspirin have an increased risk for developing Reye's syndrome, a rare but serious syndrome of rapid liver degeneration and encephalopathy. There is no specific antidote for aspirin. 2. Nonselective NSAIDs—Like aspirin, these agents are associated with significant gastrointestinal disturbance, but the incidence is lower than with aspirin. There is a risk of renal damage with any of the NSAIDs, especially in patients with preexisting renal disease. Because these drugs are cleared by the kidney, renal damage results in higher, more toxic serum concentrations. Use of parenteral ketorolac is generally restricted to 72 h because of the risk of gastrointestinal and renal damage with longer administration. Serious hematologic reactions have been noted with indomethacin. 3. COX-2-selective inhibitors—The COX-2-selective inhibitors (celecoxib, rofecoxib, valdecoxib) have a reduced risk of gastrointestinal effects, including gastric ulcers and serious gastrointestinal bleeding. The COX-2 inhibitors carry the same risk of renal damage as nonselective COX inhibitors, presumably because COX-2 contributes to homeostatic renal effects. 299 Clinical trial data suggest that highly selective COX-2 inhibitors such as rofecoxib and valdecoxib carry an increased risk of myocardial infarction and stroke. The increased risk of arterial thrombosis is believed to be due to the COX-2 inhibitors having a greater inhibitory effect on endothelial prostacyclin (PGI2) formation than on platelet TXA2 formation. Prostacyclin promotes vasodilation and inhibits platelet aggregation, whereas TXA2 has the opposite effects. Several COX-2 inhibitors have been removed from the market, and the others are now labeled with warnings about the increased risk of thrombosis. ACETAMINOPHEN A. Classification and Prototype Acetaminophen is the only over-the-counter non-anti-inflammatory analgesic commonly available in the United States. Phenacetin, a toxic prodrug that is metabolized to acetaminophen, is still available in some other countries. B. Mechanism of Action The mechanism of analgesic action of acetaminophen is unclear. The drug is only a weak COX-1 and COX-2 inhibitor in peripheral tissues, which accounts for its lack of anti-inflammatory effect. Evidence suggests that acetaminophen may inhibit a third enzyme, COX-3, in the CNS. C. Effects Acetaminophen is an analgesic and antipyretic agent; it lacks antiinflammatory or antiplatelet effects. D. Pharmacokinetics and Clinical Use Acetaminophen is effective for the same indications as intermediatedose aspirin. Acetaminophen is therefore useful as an aspirin substitute, especially in children with viral infections and in those with any type of aspirin intolerance. Acetaminophen is well absorbed orally and metabolized in the liver. Its half-life, which is 2–3 h in persons with normal hepatic function, is unaffected by renal disease. E. Toxicity In therapeutic dosages, acetaminophen has negligible toxicity in most persons. However, when taken in overdose or by patients with severe liver impairment, the drug is a dangerous hepatotoxin. The mechanism of toxicity involves oxidation to cytotoxic intermediates by phase I cytochrome P450 enzymes. This occurs if substrates for phase II conjugation reactions (acetate and glucuronide) are lacking (Chapter 4). Prompt administration of acetylcysteine, a sulfhydryl donor, may be lifesaving after an overdose. People who regularly consume 3 or more alcoholic drinks per day are at increased risk of acetaminophen-induced hepatotoxicity (Chapters 4 and 23). 300 PART VI Drugs with Important Actions on Blood, Inflammation, & Gout SKILL KEEPER: OPIOID ANALGESICS AND ANTAGONISTS (SEE CHAPTER 31) Although NSAIDs and acetaminophen are extremely useful for the treatment of mild to moderate pain, adequate control of more intense pain often requires treatment with an opioid. 1. Name 1 strong, 1 moderate, and 1 weak opioid drug. 2. Briefly describe the most common adverse effects of strong and moderate opioids. 3. What drug should be administered in the event of an opioid overdose? The Skill Keeper Answers appear at the end of the chapter. DISEASE-MODIFYING ANTIRHEUMATIC DRUGS (DMARDs) A. Classification This heterogeneous group of agents (Table 36–2) has antiinflammatory actions in several connective tissue diseases. They are called disease-modifying drugs because some evidence shows slowing or even reversal of joint damage, an effect never seen with NSAIDs. They are also called slow-acting antirheumatic drugs because it may take 6 wk to 6 mo for their benefits to become apparent. Corticosteroids can be considered anti-inflammatory drugs with an intermediate rate of action (ie, slower than NSAIDs but faster than other DMARDs). However, the corticosteroids are too toxic for routine chronic use (Chapter 39) and are reserved for temporary control of severe exacerbations and long-term use in patients with severe disease not controlled by other agents. B. Mechanisms of Action and Effects The mechanisms of action of most DMARDs in treating rheumatoid arthritis are complex. Cytotoxic drugs (eg, methotrexate) probably act by reducing the number of immune cells available to maintain the inflammatory response; many of these drugs are also used in the treatment of cancer (Chapter 54). Other drugs appear to interfere with the activity of T lymphocytes (eg, sulfasalazine, hydroxychloroquine, cyclosporine, leflunomide, mycophenolate mofetil, abatacept), B lymphocytes (rituximab), or macrophages (gold compounds). Biologic agents that inhibit the action of tumor necrosis factor-α (TNF-α), including infliximab, adalimumab, and etanercept, have also shown efficacy in rheumatoid arthritis, as has the recombinant human interleukin-1 TABLE 36–2 Some Disease-Modifying Antirheumatic Drugs (DMARDs). Drug Other Clinical Uses Toxicity When Used for Rheumatoid Arthritis Abatacept (T-cell modulator) Infection, exacerbation of COPD, hypersensitivity reactions Anti-IL-1 drugs (anakinra, rilonacept, and canakinumab) Injection-site reaction, infection, neutropenia Anti-IL-6 drugs (tocilizumab) Upper respiratory tract infections, headache, hypertension, and elevated liver enzymes Anti-TNF-α drugs (infliximab, etanercept, adalimumab, golimumab, certolizumab) Inflammatory bowel disease, other rheumatic disorders Infection, lymphoma, hepatoxicity, hematologic effects, hypersensitivity reactions, cardiovascular toxicity Belimumab (inhibits B-lymphocyte stimulator [BLyS]) Systemic lupus erythematosus Nausea, diarrhea, and respiratory tract infection Cyclosporine Tissue transplantation Nephrotoxicity, hypertension, liver toxicity Gold compounds Hydroxychloroquine, chloroquine Many adverse effects, including diarrhea, dermatitis, hematologic abnormalities Antimalarial Leflunomide Rash, gastrointestinal disturbance, myopathy, neuropathy, ocular toxicity Teratogen, hepatotoxicity, gastrointestinal disturbance, skin reactions Methotrexate Anticancer Nausea, mucosal ulcers, hematotoxicity, hepatotoxicity, teratogenicity Penicillamine Chelating agent Many adverse effects, including proteinuria, dermatitis, gastrointestinal disturbance, hematologic abnormalities Rituximab Non-Hodgkin’s lymphoma Infusion reaction, rash, infection, cardiac toxicity Sulfasalazine Inflammatory bowel disease Rash, gastrointestinal disturbance, dizziness, headache, leukopenia Tofacitinib (Janus kinase inhibitor) Infection, neutropenia, anemia, and increases in LDL and HDL CHAPTER 36 NSAIDs, Acetaminophen, & Drugs Used in Rheumatoid Arthritis & Gout receptor antagonist anakinra. The immunosuppressant effects of these drugs are discussed in more detail in Chapter 55. C. Pharmacokinetics and Clinical Use Sulfasalazine, hydroxychloroquine, methotrexate, cyclosporine, penicillamine, and leflunomide are given orally. Anti-TNF-α drugs are given by injection. Gold compounds are available for parenteral use (gold sodium thiomalate and aurothioglucose) and for oral administration (auranofin) but are rarely used. Increasingly, DMARDs, particularly low doses of methotrexate, are initiated fairly early in patients with moderate to severe rheumatoid arthritis in an attempt to ameliorate disease progression. Some of these drugs are also used in other rheumatic diseases such as lupus erythematosus, arthritis associated with Sjögren’s syndrome, juvenile rheumatoid arthritis, ankylosing spondylitis, and in other immunologic disorders (Chapter 55). D. Toxicity All DMARDs can cause severe or fatal toxicities. Careful monitoring of patients who take these drugs is mandatory. Their major adverse effects are listed in Table 36–2. DRUGS USED IN GOUT A. Classification and Prototypes Gout is associated with increased serum concentrations of uric acid. Acute attacks involve joint inflammation initiated by precipitation of uric acid crystals. Treatment strategies include (1) reducing inflammation during acute attacks (with colchicine, NSAIDs, or glucocorticoids; Figure 36–2); (2) accelerating renal excretion of uric acid with uricosuric drugs (probenecid or sulfinpyrazone); and (3) Synoviocytes Colchicine Urate crystal − LTB 4 PG Enzymes IL-1 B. Anti-Inflammatory Drugs Used for Gout 1. Mechanisms—NSAIDs such as indomethacin are effective in inhibiting the inflammation of acute gouty arthritis. These agents act through the reduction of prostaglandin formation and the inhibition of crystal phagocytosis by macrophages (Figure 36–2). Colchicine, a selective inhibitor of microtubule assembly, reduces leukocyte migration and phagocytosis; the drug may also reduce production of leukotriene B4 and decrease free radical formation. 2. Effects—NSAIDs and glucocorticoids reduce the synthesis of inflammatory mediators in the gouty joint. Because it reacts with tubulin and interferes with microtubule assembly, colchicine is a general mitotic poison. Tubulin is necessary for normal cell division, motility, and many other processes. 3. Pharmacokinetics and clinical use—An NSAID or a glucocorticoid is preferred for the treatment of acute gouty arthritis. Although colchicine can be used for acute attacks, the doses required cause significant gastrointestinal disturbance, particularly diarrhea. Lower doses of colchicine are used to prevent attacks of gout in patients with a history of multiple acute attacks. Colchicine is also of value in the management of familial Mediterranean fever, a disease of unknown cause characterized by fever, hepatitis, peritonitis, pleuritis, arthritis, and, occasionally, amyloidosis. Indomethacin, some glucocorticoids, and colchicine are used orally; parenteral preparations of glucocorticoids and colchicine are also available. PG MNP − − IL-1 reducing (with allopurinol or febuxostat) the conversion of purines to uric acid by xanthine oxidase (Figure 36–3). 4. Toxicity—NSAIDs can cause renal damage, and indomethacin can additionally cause bone marrow depression. Short courses of glucocorticoids can cause behavioral changes and impaired glucose control. Because colchicine can severely damage the liver and kidney, dosage must be carefully limited and monitored. Overdose is often fatal. PMN PG 301 Indomethacin, phenylbutazone FIGURE 36–2 Sites of action of some anti-inflammatory drugs in a gouty joint. Synoviocytes damaged by uric acid crystals release prostaglandins (PG), interleukins (ILs), and other mediators of inflammation. Polymorphonuclear leukocytes (PMN), macrophages, and other inflammatory cells enter the joint and also release inflammatory substances, including leukotrienes (eg, LTB4), that attract additional inflammatory cells. Colchicine acts on microtubules in the inflammatory cells. NSAIDs act on cyclooxygenase-2 (COX II) and inhibit PG formation in all of the cells of the joint. MNP, mononuclear phagocytes. (Reproduced, with permission, from Katzung BG, editor: Basic & Clinical Pharmacology, 12th ed. McGraw-Hill, 2012: Fig. 36–5.) C. Uricosuric Agents 1. Mechanism—Normally, over 90% of the uric acid filtered by the kidney is reabsorbed in the proximal tubules. Uricosuric agents (probenecid, sulfinpyrazone) are weak acids that compete with uric acid for reabsorption by the weak acid transport mechanism in the proximal tubules and thereby increase uric acid excretion. At low doses, these agents may also compete with uric acid for secretion by the tubule and occasionally can elevate, rather than reduce, serum uric acid concentration. Elevation of uric acid levels by this mechanism occurs with aspirin (another weak acid) over much of its dose range. 2. Effects—Uricosuric drugs inhibit the secretion of a large number of other weak acids (eg, penicillin, methotrexate) in addition to inhibiting the reabsorption of uric acid. 302 PART VI Drugs with Important Actions on Blood, Inflammation, & Gout Alloxanthine, febuxostat − O HN 1 6 5 2 3 N 4 N 7 Xanthine oxidase − O N HN O Xanthine oxidase H N HN OH 8 9 N H Hypoxanthine O N H N H Xanthine O N H N H Uric acid FIGURE 36–3 The action of xanthine oxidase in uric acid synthesis. (Modified and reproduced, with permission, from Katzung BG, editor: Basic & Clinical Pharmacology, 12th ed. McGraw-Hill, 2012: Fig. 36–7.) 3. Pharmacokinetics and clinical use—Uricosuric drugs are used orally to treat chronic gout, caused by under-excretion of uric acid. These drugs are of no value in acute episodes. depend on xanthine oxidase for elimination. Febuxostat can cause liver function abnormalities, headache, and gastrointestinal upset. 4. Toxicity—Uricosuric drugs can precipitate an attack of acute gout during the early phase of their action. This can be avoided by simultaneously administering colchicine or indomethacin. Because they are sulfonamides, the uricosuric drugs may share allergenicity with other classes of sulfonamide drugs (diuretics, antimicrobials, oral hypoglycemic drugs). QUESTIONS D. Xanthine Oxidase Inhibitors 1. Mechanism—The production of uric acid can be reduced by inhibition of xanthine oxidase, the enzyme that converts hypoxanthine to xanthine and xanthine to uric acid (Figure 36–3). Allopurinol is converted to oxypurinol (alloxanthine) by xanthine oxidase; alloxanthine is an irreversible suicide inhibitor of the enzyme. The newer drug febuxostat is a nonpurine inhibitor of xanthine oxidase that is more selective than allopurinol and alloxanthine, which inhibit other enzymes involved in purine and pyrimidine metabolism. 2. Effects—Inhibition of xanthine oxidase increases the concentrations of the more soluble hypoxanthine and xanthine and decreases the concentration of the less soluble uric acid. As a result, there is less likelihood of precipitation of uric acid crystals in joints and tissues. Clinical trials suggest that febuxostat is more effective than allopurinol in lowering serum uric acid. 3. Pharmacokinetics and clinical use—The xanthine oxidase inhibitors are given orally in the management of chronic gout. Like uricosuric agents, these drugs are usually withheld for 1–2 wk after an acute episode of gouty arthritis and are administered in combination with colchicine or an NSAID to avoid an acute attack. Allopurinol is also used as an adjunct to cancer chemotherapy to slow the formation of uric acid from purines released by the death of large numbers of neoplastic cells. 4. Toxicity and drug interactions—Allopurinol causes gastrointestinal upset, rash, and rarely, peripheral neuritis, vasculitis, or bone marrow dysfunction, including aplastic anemia. It inhibits the metabolism of mercaptopurine and azathioprine, drugs that 1. Among NSAIDs, aspirin is unique because it (A) Irreversibly inhibits its target enzyme (B) Prevents episodes of gouty arthritis with long-term use (C) Reduces fever (D) Reduces the risk of colon cancer (E) Selectively inhibits the COX-2 enzyme 2. Which of the following is an analgesic and antipyretic drug that lacks an anti-inflammatory action? (A) Acetaminophen (B) Celecoxib (C) Colchicine (D) Indomethacin (E) Probenecid 3. A 16-year-old girl comes to the emergency department suffering from the effects of an aspirin overdose. Which of the following syndromes is this patient most likely to exhibit as a result of this drug overdose? (A) Bone marrow suppression and possibly aplastic anemia (B) Fever, hepatic dysfunction, and encephalopathy (C) Hyperthermia, metabolic acidosis, and coma (D) Rapid, fulminant hepatic failure (E) Rash, interstitial nephritis, and acute renal failure 4. Which of the following drugs is most likely to increase serum concentrations of conventional doses of methotrexate, a weak acid that is primarily cleared in the urine? (A) Acetaminophen (B) Allopurinol (C) Colchicine (D) Hydroxychloroquine (E) Probenecid 5. The main advantage of ketorolac over aspirin is that ketorolac (A) Can be combined more safely with an opioid such as codeine (B) Can be obtained as an over-the-counter agent (C) Does not prolong the bleeding time (D) Is available in a parenteral formulation that can be injected intramuscularly or intravenously (E) Is less likely to cause acute renal failure in patients with some preexisting degree of renal impairment CHAPTER 36 NSAIDs, Acetaminophen, & Drugs Used in Rheumatoid Arthritis & Gout 6. An 18-month-old boy dies from an accidental overdose of acetaminophen. Which of the following is the most likely cause of this patient’s death? (A) Arrhythmia (B) Hemorrhagic stroke (C) Liver failure (D) Noncardiogenic pulmonary edema (E) Ventilatory failure Questions 7 and 8. A 52-year-old woman presented with intense pain, warmth, and redness in the first toe on her left foot. Examination of fluid withdrawn from the inflamed joint revealed crystals of uric acid. 7. In the treatment of this woman’s acute attack of gout, a high dose of colchicine will reduce the pain and inflammation. However, many physicians prefer to treat acute gout with a corticosteroid or indomethacin because high doses of colchicine are likely to cause (A) Behavioral changes that include psychosis (B) High blood pressure (C) Rash (D) Severe diarrhea (E) Sudden gastrointestinal bleeding 8. Over the next 7 mo, the patient had 2 more attacks of acute gout. Her serum concentration of uric acid was elevated. The decision was made to put her on chronic drug therapy to try to prevent subsequent attacks. Which of the following drugs could be used to decrease this woman’s rate of production of uric acid? (A) Allopurinol (B) Aspirin (C) Colchicine (D) Hydroxychloroquine (E) Probenecid Questions 9 and 10. A 54-year-old woman presented with signs and symptoms consistent with an early stage of rheumatoid arthritis. The decision was made to initiate NSAID therapy. 9. Which of the following patient characteristics is the most compelling reason for avoiding celecoxib in the treatment of her arthritis? (A) History of alcohol abuse (B) History of gout (C) History of myocardial infarction (D) History of osteoporosis (E) History of peptic ulcer disease 10. Although the patient’s disease was adequately controlled with an NSAID and methotrexate for some time, her symptoms began to worsen and radiologic studies of her hands indicated progressive destruction in the joints of several fingers. Treatment with another second-line agent for rheumatoid arthritis was considered. Which of the following is a parenterally administered DMARD whose mechanism of antiinflammatory action is antagonism of tumor necrosis factor? (A) Cyclosporine (B) Etanercept (C) Penicillamine (D) Phenylbutazone (E) Sulfasalazine 303 ANSWERS 1. Aspirin differs from other NSAIDs by irreversibly inhibiting cyclooxygenase. The answer is A. 2. Acetaminophen is the only drug that fits this description. Indomethacin is a nonselective COX inhibitor and celecoxib is a COX-2 inhibitor; both have analgesic, antipyretic, and anti-inflammatory effects. Colchicine is a drug used for gout that also has an anti-inflammatory action. Probenecid is a uricosuric drug that promotes the excretion of uric acid. The answer is A. 3. Salicylate intoxication is associated with metabolic acidosis, dehydration, and hyperthermia. If these problems are not corrected, coma and death ensue. The answer is C. 4. Like other weak acids, methotrexate depends on active tubular excretion in the proximal tubule for efficient elimination. Probenecid competes with methotrexate for binding to the proximal tubule transporter and thereby decreases the rate of clearance of methotrexate. The answer is E. 5. Ketorolac exerts typical NSAID effects. It prolongs the bleeding time and can impair renal function, especially in a patient with preexisting renal disease. Its primary use is as a parenteral agent for pain management, especially for treatment of postoperative patients. The answer is D. 6. In overdose, acetaminophen causes fulminant liver failure as a result of its conversion by hepatic cytochrome P450 enzymes to a highly reactive metabolite. The answer is C. 7. At doses needed to treat acute gout, colchicine frequently causes significant diarrhea. Such gastrointestinal effects are less likely with the lower doses used in chronic gout. The answer is D. 8. Allopurinol is the only drug listed that decreases production of uric acid. Probenecid increases uric acid excretion. Colchicine and hydroxychloroquine do not affect uric acid metabolism. Aspirin actually slows renal secretion of uric acid and raises uric acid blood levels. It should not be used in gout. The answer is A. 9. Celecoxib is a COX-2-selective inhibitor. Although the COX-2 inhibitors have the advantage over nonselective NSAIDs of reduced gastrointestinal toxicity, clinical data suggest that they are more likely to cause arterial thrombotic events. A history of myocardial infarction would be a compelling reason to avoid a COX-2 inhibitor. The answer is C. 10. Etanercept is a recombinant protein that binds to tumor necrosis factor and prevents its inflammatory effects. The answer is B. 304 PART VI Drugs with Important Actions on Blood, Inflammation, & Gout SKILL KEEPER ANSWERS: OPIOIDS (SEE CHAPTER 31) 1. Morphine is the prototype strong opioid. Fentanyl is a strong agent with a rapid onset that is commonly used in the hospital. Methadone is a strong agonist used in maintenance programs for patients addicted to opioids. Codeine, oxycodone, and hydrocodone are moderate agonists, whereas propoxyphene is a weak agonist. 2. Constipation and sedation occur with therapeutic doses; constipation should be managed with stool softeners. In overdose, opioids cause a triad of pinpoint pupils, coma, and respiratory depression. 3. Naloxone, a nonselective opioid receptor antagonist, is an antidote for opioid overdose. CHECKLIST When you complete this chapter, you should be able to: ❑ Describe the effects of NSAIDs on prostaglandin synthesis. ❑ Contrast the functions of COX-1 and COX-2. ❑ Compare the actions and toxicity of aspirin, the older nonselective NSAIDs, and the COX-2-selective drugs. ❑ Explain why several of the highly selective COX-2 inhibitors have been withdrawn from the market. ❑ Describe the toxic effects of aspirin. ❑ Describe the effects and the major toxicity of acetaminophen. ❑ Name 5 disease-modifying antirheumatic drugs (DMARDs) and describe their toxicity. ❑ Contrast the pharmacologic treatment of acute and chronic gout. ❑ Describe the mechanisms of action and toxicity of 3 different drug groups used in gout. CHAPTER 36 NSAIDs, Acetaminophen, & Drugs Used in Rheumatoid Arthritis & Gout 305 DRUG SUMMARY TABLE: NSAIDs, Acetaminophen, & Drugs for Rheumatoid Arthritis & Gout Subclass Mechanism of Action Clinical Applications Pharmacokinetics Toxicities, Drug Interactions Acetylation of COX-1 and COX-2 results in decreased prostaglandin synthesis Analgesia, antipyretic, anti-inflammatory, and antithrombotic • prevention of colon cancer Duration of activity is longer than pharmacokinetic half-life of drug due to irreversible COX inhibition Gastrointestinal (GI) toxicity, nephrotoxicity, and increased bleeding time at therapeutic levels • hypersensitivity reaction due to increased leukotrienes • tinnitus, hyperventilation, metabolic acidosis, hyperthermia, coma in overdose Reversible inhibition of COX-1 and COX-2 results in decreased prostaglandin synthesis Analgesiaa, antipyretic, and anti-inflammatory • closure of patent ductus arteriosus Rapid metabolism and renal elimination GI toxicity, nephrotoxicity • hypersensitivity due to increased leukotrienes • interference with aspirin’s antithrombotic action Salicylates Aspirin Nonselective NSAIDs Ibuprofen Many nonselective nonsteroidal anti-inflammatory drugs (NSAIDs) available for clinical use. See Table 36–1 COX-2 inhibitor Celecoxib Selective, reversible inhibition of COX-2 results in decreased prostaglandin synthesis Analgesia, antipyretic, and anti-inflammatory Hepatic metabolism Nephrotoxicity • hypersensitivity due to increased leukotrienes • less risk of GI toxicity than nonselective NSAIDs • greater risk of thrombosis than nonselective NSAIDs Mechanism unknown, weak COX inhibitor Analgesia, antipyretic Hepatic conjugation Hepatotoxicity in overdose (antidote is acetylcysteine) • hepatotoxicity more likely with chronic alcohol consumption, which induces P450 enzymes Renal elimination Nausea, mucosal ulcers, hematotoxicity, hepatotoxicity, teratogenicity Other analgesic Acetaminophen Disease-modifying antirheumatic drugs (DMARDs) Methotrexate Cytotoxic to rapidly dividing immune cells due to inhibition of dihydrofolate reductase Anticancer, rheumatic disorders Diverse array of DMARDs available for clinical use. See Table 36–2 Microtubule assembly inhibitor Colchicine Inhibition of microtubule assembly decreases macrophage migration and phagocytosis Chronic and acute gout, familial Mediterranean fever Oral drug Diarrhea, severe liver and kidney damage in overdose Inhibition of renal reuptake of uric acid Chronic gout, prolongation of antimicrobial drug action Oral drug Exacerbation of acute gout, hypersensitivity reactions, inhibits renal tubular secretion of weak acids such as methotrexate Chronic gout, adjunct to cancer chemotherapy Activated by xanthine oxidase • oral drug GI upset, hypersensitivity reactions, bone marrow suppression Uricosurics Probenecid Sulfinpyrazone: similar to probenecid Xanthine oxidase inhibitors Allopurinol Active metabolite irreversibly inhibits xanthine oxidase and lowers production of uric acid Febuxostat: reversible inhibitor of xanthine oxidase a Ketorolac is used as pure analgesic (not for anti-inflammatory effect). This page intentionally left blank PART VII ENDOCRINE DRUGS C A P T E R 37 Hypothalamic & Pituitary Hormones The hormones produced by the hypothalamus and pituitary gland are key regulators of metabolism, growth, and reproduction. Preparations of these hormones, including products H made by recombinant DNA technology and drugs that mimic or block their effects, are used in the treatment of a variety of endocrine disorders. Drugs that mimic or block the effects of hypothalamic & pituitary hormones Anterior pituitary Growth hormone Agonist action Somatropin Gonadotropins Prolactin Agonist action Antagonist action Antagonist action Octreotide Mecasermin Pegvisomant Mixed LH & FSH LH Menotropins Lutropin FSH hCG Oxytocin GnRH Gonadorelin Follitropin Posterior pituitary Hypothalamus Agonist action D2 dopamine agonists (bromocriptine) Antagonist action GnRH receptor agonist (leuprolide) GnRH receptor antagonist (ganirelix) Vasopressin Agonist action Antagonist action Agonist action Antagonist action Oxytocin Atosiban Desmopressin Conivaptan 307 308 PART VII Endocrine Drugs High-Yield Terms to Learn Acromegaly A rare syndrome of growth hormone (GH) excess in adults characterized by abnormal growth of tissues (particularly connective tissue), metabolic abnormalities, and cardiac dysfunction Central diabetes insipidus A syndrome of polyuria, polydipsia, and hypernatremia caused by inadequate production of vasopressin Gigantism A syndrome of GH excess in children and adolescents with open long bone epiphyses that results in excessive height Gonadotropins The 2 anterior pituitary hormones (luteinizing hormone [LH] and follicle-stimulating hormone [FSH]) that regulate reproduction in males and females Insulin-like growth factor-1 (IGF-1) A growth factor that is the primary mediator of GH effects Prolactinoma Pituitary tumor that secretes excessive amounts of prolactin and is associated with a syndrome of infertility and galactorrhea Tocolytic Drug used to inhibit preterm labor (eg, the oxytocin receptor antagonist atosiban; magnesium sulfate; nifedipine; β2 agonists) ANTERIOR PITUITARY HORMONES & THEIR HYPOTHALAMIC REGULATORS The hypothalamic and pituitary hormones and their antagonists are often grouped according to the anatomic site of release of the hormone that they mimic or block—the hypothalamus for gonadotropin-releasing hormone (GnRH); the anterior pituitary for growth hormone (GH), the 2 gonadotropins, luteinizing hormone (LH) and follicle-stimulating hormone (FSH), and prolactin; or the posterior pituitary for oxytocin and vasopressin (antidiuretic hormone [ADH]). This chapter focuses on the agents used commonly; refer to the full text Basic and Clinical Pharmacology for hormones that are either not used clinically or are used solely for specialized diagnostic testing (thyrotropin-releasing hormone [TRH], thyroid-stimulating hormone [TSH], corticotropin-releasing hormone [CRH], adrenocorticotropic hormone [ACTH], and growth hormone-releasing hormone [GHRH]). Hormones of the anterior pituitary are central links in the hypothalamic-pituitary endocrine system (or axis; Figure 37–1). All the anterior pituitary hormones are under the control of a hypothalamic hormone, and with the exception of prolactin, all mediate their ultimate effects by regulating the production by peripheral tissues of other hormones (Table 37–1). Four anterior pituitary hormones (TSH, LH, FSH, and ACTH) and their hypothalamic regulators are subject to feedback regulation by the hormones whose production they control. The complex systems that regulate hormones of the anterior pituitary provide multiple avenues of pharmacologic intervention. A. Growth Hormone and Mecasermin 1. GH—Growth hormone is required for normal growth during childhood and adolescence and is an important regulator throughout life of lipid and carbohydrate metabolism and lean body mass. Its effects are primarily mediated by regulating the production in peripheral tissues of insulin-like growth factor 1 (IGF-1). Somatropin, the recombinant form of human GH, is used for GH deficiency in children and adults and in the treatment of children with genetic diseases associated with short stature (eg, Turner syndrome, Noonan syndrome, Prader-Willi syndrome). GH treatment also improves growth in children with failure to thrive due to chronic renal failure or the small-for-gestational-age condition. The most controversial use of GH is for children with idiopathic short stature who are not GH deficient. In this group of children, multiple years of GH therapy at great cost and some risk of toxicity results in a small (1.5–3 inches) average increase in final adult height. In adults, GH has efficacy in treatment of AIDS-associated wasting and GH deficiency, and it may improve gastrointestinal function in patients who have undergone intestinal resection and have subsequently developed a malabsorption syndrome. GH is a popular component of antiaging programs even though studies in model animal systems have consistently found that analogs of GH and IGF-1 shorten lifespan. GH is also used by athletes for a purported increase in muscle mass and athletic performance and is one of the drugs banned by the Olympic Committee and professional sports associations. Recombinant bovine GH is used in dairy cattle to increase milk production. Rare but serious adverse effects of GH in children include pseudotumor cerebri, slipped capital femoral epiphysis, progression of scoliosis, edema, and hyperglycemia. Children with GH deficiency should be monitored periodically for concurrent deficiency of other anterior pituitary hormones. Adults generally tolerate GH less well than children. Adverse effects include peripheral edema, myalgia, and arthralgia. 2. Mecasermin—A small group of children with growth failure unresponsive to GH therapy are deficient in IGF-1. Mecasermin, recombinant human IGF-1, is administered parenterally to children with IGF-1 deficiency. Its most important toxicity is hypoglycemia, which can be prevented by consumption of a snack or meal shortly before mecasermin administration. In some countries, children are treated with mecasermin rinfabate, a combination of recombinant human IGF-1 and human insulin-like growth factor-binding protein-3 (rhIGFBP-3), which increases the half-life of IGF-1. CHAPTER 37 Hypothalamic & Pituitary Hormones 1. Somatostatin analogs—Somatostatin, a 14-amino-acid peptide, inhibits the release of GH, glucagon, insulin, and gastrin. Octreotide and lanreotide, long-acting synthetic analogs of somatostatin, are used to treat acromegaly, carcinoid, gastrinoma, glucagonoma, and other endocrine tumors. Regular octreotide must be administered subcutaneously 2–4 times daily, whereas a slow-release intramuscular formulation of octreotide or lanreotide is administered every 4 weeks for long-term therapy. Octreotide and lanreotide cause significant gastrointestinal disturbances, gallstones, and cardiac conduction abnormalities. Hypothalamus GHRH TRH CRH GnRH DA SST – + Portal venous system Posterior pituitary Anterior pituitary GH TSH ACTH LH FSH PRL 309 Oxytocin ADH Endocrine glands, liver, bone & other tissues Target tissues FIGURE 37–1 The hypothalamic-pituitary endocrine system. Except for prolactin, hormones released from the anterior pituitary stimulate the production of hormones by a peripheral endocrine gland, the liver, or other tissues. Prolactin and the hormones released from the posterior pituitary (vasopressin and oxytocin) act directly on target tissues. Hypothalamic factors regulate the release of anterior pituitary hormones. ACTH, adrenocorticotropin; ADH, antidiuretic hormone [vasopressin]; CRH, corticotropin-releasing hormone; DA, dopamine; FSH, follicle-stimulating hormone; GH, growth hormone; GHRH, growth hormone-releasing hormone; GnRH, gonadotropinreleasing hormone; LH, luteinizing hormone; PRL, prolactin; SST, somatostatin; TRH, thyrotropin-releasing hormone; TSH, thyroidstimulating hormone. (Reproduced, with permission, Katzung BG, editor: Basic & Clinical Pharmacology, 12th ed. McGraw-Hill, 2012: Fig. 37–1.) B. Growth Hormone Antagonists Growth hormone-secreting pituitary adenomas cause acromegaly in adults and, rarely, gigantism in children and adolescents who have not completed their growth phase. Pharmacologic treatment of GH excess seeks to inhibit GH secretion or interfere with GH effects. 2. Dopamine D2 receptor agonists—Dopamine D2 receptor agonists such as bromocriptine are more effective at inhibiting prolactin release than inhibiting GH release (see following text). However, high doses of D2 receptor agonists have some efficacy in the treatment of small GH-secreting tumors. 3. Pegvisomant—Pegvisomant is a GH receptor antagonist approved for treatment of acromegaly. Normally, GH, which has 2 distinct receptor binding sites, initiates cellular signaling cascades by dimerizing 2 GH receptors. Pegvisomant is a long-acting derivative of a mutant GH that is able to cross-link GH receptors but is incapable of inducing the conformational changes required for receptor activation. C. Follicle-Stimulating Hormone (FSH), Luteinizing Hormone (LH), and Their Analogs In women, FSH directs follicle development, whereas FSH and LH collaborate in the regulation of ovarian steroidogenesis. In men, FSH is the primary regulator of spermatogenesis, whereas LH is the main stimulus for testicular androgen production. The gonadotropins or their analogs are used in combination to stimulate spermatogenesis in infertile men and to induce ovulation in women with anovulation that is not responsive to less complicated treatments (see Chapter 40). In men, the treatment of infertility due to hypogonadism requires months of administration of a mixture of drugs with LH and FSH activity. Ovulation induction protocols are increasingly complex. They require close monitoring to ensure successful insemination or retrieval of mature oocytes and to prevent the 2 most serious complications of ovulation induction—multiple pregnancies and the ovarian hyperstimulation syndrome, a syndrome of ovarian enlargement, ascites, hypovolemia, and possibly shock. All ovulation induction protocols that use gonadotropins have 3 basic steps. First, endogenous gonadotropin production is inhibited by administration of a GnRH agonist or antagonist (see text that follows). Second, follicle development is driven by daily injections of a preparation with FSH activity (menotropins, FSH, or an FSH analog). Last, the final stage of oocyte maturation is induced with an injection of LH or the LH analog human chorionic gonadotropin (hCG). A variety of gonadotropin preparations are available. All are administered parenterally. 1. Menotropins—These gonadotropins consist of a mixture of FSH and LH purified from the urine of postmenopausal women (who produce high levels of FSH and LH owing to the 310 PART VII Endocrine Drugs TABLE 37–1 Links between hypothalamic, anterior pituitary, and target organ hormones or mediators.a Primary Target Organ Hormone(s) or Mediator(s) Anterior Pituitary Hormone Hypothalamic Hormone Target Organ Growth hormone (GH, somatotropin) Growth hormone-releasing hormone (GHRH) (+) Somatostatin (–) Liver, bone, muscle, kidney, and others Insulin-like growth factor-1 (IGF-1) Thyroid-stimulating hormone (TSH) Thyrotropin-releasing hormone (TRH) (+) Thyroid Thyroxine, triiodothyronine Adrenocorticotropin (ACTH) Corticotropin-releasing hormone (CRH) (+) Adrenal cortex Cortisol Follicle-stimulating hormone (FSH) Luteinizing hormone (LH) Gonadotropin-releasing hormone (GnRH) (+)b Gonads Estrogen, progesterone, testosterone Prolactin (PRL) Dopamine (–) Breast — (+), stimulant; (–), inhibitor. a All of these hormones act through G protein-coupled receptors except GH and prolactin, which act through JAK/STAT receptors. b Endogenous GnRH, which is released in pulses, stimulates LH and FSH release. When administered continuously as a drug, GnRH and its analogs inhibit LH and FSH release. Reproduced, with permission, from Katzung BG, editor: Basic & Clinical Pharmacology, 12th ed. McGraw-Hill, 2012. disinhibition of pituitary gonadotropin production that results from cessation of ovarian steroidogenesis). 2. FSH and its analogs—Three forms of FSH are available. Urofollitropin is a purified preparation extracted from the urine of postmenopausal women. The 2 recombinant forms of human FSH—follitropin alpha and follitropin beta—differ in the composition of their carbohydrate side chains. 3. LH and its analogs—Human chorionic gonadotropin (hCG), the placental protein that supports the corpus luteum during the early stages of pregnancy, has a structure that is nearly identical to LH and mediates its effects through activation of LH receptors. hCG purified from human urine or recombinant hCG is used commonly for LH action. Lutropin, a recombinant form of human LH, is also available. D. Gonadotropin-Releasing Hormone (GnRH) and Its Analogs GnRH is a decapeptide that stimulates gonadotropin release when it is secreted in a pulsatile pattern by the hypothalamus. Leuprolide was the first of a set of synthetic peptides with long-acting GnRH agonist activity. Other long-acting GnRH agonists include goserelin, histrelin, nafarelin, and triptorelin. In men and women, steady dosing with these GnRH agonists inhibits gonadotropin release by downregulating GnRH receptors in the pituitary cells that normally release gonadotropins. Continuous GnRH agonist treatment is used to suppress endogenous gonadotropin secretion in women undergoing ovulation induction with gonadotropins, in women with gynecologic disorders that benefit from ovarian suppression (eg, endometriosis, uterine leiomyomata), in men with advanced prostate cancer, in early pubertal transgender adolescents (to block endogenous puberty prior to treatment with cross-gender gonadal hormones), and in children with central precocious puberty. In women, continuous treatment with a GnRH agonist causes the typical symptoms of menopause (hot flushes, sweats, headache). Long-term treatment is avoided because of the risk of bone loss and osteoporosis. In men treated continuously with a GnRH agonist, adverse effects include hot flushes, sweats, gynecomastia, reduced libido, decreased hematocrit, and reduced bone density. In men with prostate cancer and children with central precocious puberty, the first few weeks of therapy can temporarily exacerbate the condition. E. Gonadotropin-Releasing Hormone (GnRH) Antagonists Ganirelix, cetrorelix, and degarelix are GnRH antagonists. Ganirelix and cetrorelix can be used during ovulation induction in place of GnRH agonists to suppress endogenous gonadotropin production. Degarelix is approved for the treatment of advanced prostate cancer. The adverse effects of GnRH antagonists are similar to those associated with continuous treatment with a GnRH agonist except that they do not cause a tumor flare when used for treatment of advanced prostate cancer and they may be less likely to cause the ovarian hyperstimulation syndrome when used for ovulation induction. F. Prolactin Antagonists (Dopamine D2 Receptor Agonists) The anterior pituitary hormone prolactin regulates lactation. In women and men, hyperprolactinemia and an associated syndrome of infertility and galactorrhea can result from prolactin-secreting adenomas. Dopamine is the physiologic inhibitor of prolactin release (Figure 37–1). Prolactin-secreting adenomas usually retain their sensitivity to dopamine. In hyperprolactinemia, bromocriptine and other orally active D2 dopamine receptor agonists (eg, cabergoline, pergolide; see Chapter 16) are effective in reducing serum prolactin concentrations and restoring fertility. As previously mentioned, high doses of a dopamine agonist can also be used in the treatment of acromegaly. CHAPTER 37 Hypothalamic & Pituitary Hormones SKILL KEEPER: DRUGS THAT CAUSE HYPERPROLACTINEMIA (SEE CHAPTER 29) As many as 25% of infertile women have hyperprolactinemia. In women, hyperprolactinemia causes galactorrhea, oligomenorrhea, or amenorrhea as well as infertility (the amenorrhea-galactorrhea syndrome). Although prolactin-secreting tumors are the most common cause of hyperprolactinemia, the condition can also be precipitated by drugs that interfere with the control of prolactin release. 1. What types of pharmacologic actions are most likely to cause hyperprolactinemia? 2. Name several drugs with this type of pharmacologic action. The Skill Keeper Answers appear at the end of the chapter. POSTERIOR PITUITARY HORMONES A. Oxytocin Oxytocin is a nonapeptide synthesized in cell bodies in the paraventricular nuclei of the hypothalamus and transported through the axons of these cells to the posterior pituitary (Figure 37–1). Oxytocin is an effective stimulant of uterine contraction and is used intravenously to induce or reinforce labor. Atosiban is an antagonist of the oxytocin receptor that is used in some countries as a tocolytic, a drug used to suppress preterm labor. B. Vasopressin (Antidiuretic Hormone [ADH]) Vasopressin is synthesized in neuronal cell bodies in the hypothalamus and released from nerve terminals in the posterior pituitary (Figure 37–1). As discussed in Chapter 15, vasopressin acts through V2 receptors to increase the insertion of water channels in the apical membranes of collecting duct cells in the kidney and to thereby provide an antidiuretic effect. Extrarenal V2-like receptors regulate the release of coagulation factor VIII and von Willebrand factor (see Chapter 34). Desmopressin, a selective agonist of V2 receptors, is administered orally, nasally, or parenterally in patients with pituitary diabetes insipidus and in patients with mild hemophilia A or von Willebrand disease. Vasopressin also contracts vascular smooth muscle by activating V1 receptors. Because of this vasoconstrictor effect, vasopressin is sometimes used to treat patients with bleeding from esophageal varices or colon diverticula. Several antagonists of vasopressin receptors (eg, conivaptan, tolvaptan) have been developed to offset the fluid retention that results from the excessive production of vasopressin associated with hyponatremia or acute heart failure (see Chapter 15). 311 QUESTIONS 1. A young couple (25-year-old male, 23-year-old female) wants to start a family. They have not conceived after 1 yr of unprotected intercourse. Infertility evaluation revealed no abnormalities in the female partner and low sperm count in the male. Which of the following is a drug that is purified from the urine of postmenopausal women and is used to promote spermatogenesis in infertile men? (A) Desmopressin (B) Gonadorelin (C) Goserelin (D) Somatropin (E) Urofollitropin 2. A 29-year-old woman in her 41st wk of gestation had been in labor for 12 h. Although her uterine contractions had been strong and regular initially, they had diminished in force during the past hour. Which of the following agents would be used to facilitate this woman’s labor and delivery? (A) Dopamine (B) Leuprolide (C) Oxytocin (D) Prolactin (E) Vasopressin 3. A 3-year-old boy with failure to thrive and metabolic disturbances was found to have an inactivating mutation in the gene that encodes the growth hormone receptor. Which of the following drugs is most likely to improve his metabolic function and promote his growth? (A) Atosiban (B) Bromocriptine (C) Mecasermin (D) Octreotide (E) Somatropin 4. An important difference between leuprolide and ganirelix is that ganirelix (A) Can be administered as an oral formulation (B) Can be used alone to restore fertility to hypogonadal men and women (C) Immediately reduces gonadotropin secretion (D) Initially stimulates pituitary production of LH and FSH (E) Must be administered in a pulsatile fashion 5. A 27-year-old woman with amenorrhea, infertility, and galactorrhea was treated with a drug that successfully restored ovulation and menstruation. Before being given the drug, the woman was carefully questioned about previous mental health problems, which she did not have. She was advised to take the drug orally. Which of the following is most likely to be the drug that was used to treat this patient? (A) Bromocriptine (B) Desmopressin (C) Human gonadotropin hormone (D) Leuprolide (E) Octreotide 312 PART VII Endocrine Drugs 6. A 3-year-old girl was referred to the genetic counselor by her pediatrician. She presents with short stature (height is 85 cm, –3 standard deviations) and appears to have loose skin on her neck. Cytogenetic testing reveals an XO karyotype. Which of the following drugs will allow her to achieve a higher adult height? (A) Adrenocorticotropin (ACTH) (B) Corticotropin-releasing hormone (CRH) (C) Growth hormone-releasing hormone (GHRH) (D) Gonadotropin-releasing hormone (GnRH) (E) Somatropin 7. A 3-year-old girl presented with hirsutism, breast enlargement, and a height and bone age that was consistent with an age of 9. Diagnostic testing revealed precocious puberty. Which of the following is the most appropriate drug for treatment of this patient’s precocious puberty? (A) Atosiban (B) Follitropin (C) Leuprolide (D) Octreotide (E) Pegvisomant 8. A 47-year-old man exhibited signs and symptoms of acromegaly. Radiologic studies indicated the presence of a large pituitary tumor. Surgical treatment of the tumor was only partially effective in controlling his disease. At this point, which of the following drugs is most likely to be used as pharmacologic therapy? (A) Cosyntropin (B) Desmopressin (C) Leuprolide (D) Octreotide (E) Somatropin 9. A 37-year-old woman with infertility due to obstructed fallopian tubes was undergoing ovulation induction in preparation for in vitro fertilization. After 10 d of treatment with leuprolide, the next step in the procedure is most likely to involve 10–14 d of treatment with which of the following? (A) Bromocriptine (B) Follitropin (C) Gonadorelin (D) hCG (E) Pergolide 10. A 7-year-old boy underwent successful chemotherapy and cranial radiation for treatment of acute lymphocytic leukemia. One month after the completion of therapy, the patient presented with excessive thirst and urination plus hypernatremia. Laboratory testing revealed pituitary diabetes insipidus. To correct these problems, this patient is likely to be treated with which of the following? (A) Corticotropin (B) Desmopressin (C) hCG (D) Menotropins (E) Thyrotropin ANSWERS 1. Spermatogenesis in males requires the action of FSH and LH. Urofollitropin, which is purified from the urine of postmenopausal women, is used clinically to provide FSH activity. The answer is E. 2. Oxytocin is an effective stimulant of uterine contraction that is routinely used to augment labor. The answer is C. 3. This child’s condition is due to the inability of GH to stimulate the production of insulin-like growth factors, the ultimate mediators of GH effects. Mecasermin, a combination of recombinant IGF-1 and the binding protein that protects IGF-1 from immediate destruction, will help correct the IGF deficiency. Because of the inactive GH receptors, somatropin will not be effective. The answer is C. 4. Leuprolide is an agonist of GnRH receptors, whereas ganirelix is an antagonist. Although both drugs can be used to inhibit gonadotropin release, ganirelix does so immediately, whereas leuprolide does so only after about 1 wk of sustained activity. The answer is C. 5. Bromocriptine, a dopamine receptor agonist, is used to treat the amenorrhea-galactorrhea syndrome, which is a consequence of hyperprolactinemia. Because of its central dopaminergic effects, the drug should not be used in patients with a history of schizophrenia or other forms of psychotic illness. The answer is A. 6. Adrenocorticotropin (ACTH) is used diagnostically in suspected adrenal insufficiency. Corticotropin-releasing hormone (CRH) is used to distinguish Cushing’s syndrome from ectopic ACTH secretion. GHRH is rarely used as treatment. Its main use is as a diagnostic tool. GnRH can be used to treat infertility. Somatropin, recombinant human GH, promotes growth in children with Turner’s syndrome (an XO genetic genotype) or chronic renal failure. It also helps combat the AIDS-associated wasting syndrome. The answer is E. 7. In precocious puberty, the hypothalamic-pituitary-gonadal axis becomes prematurely active for reasons that are not understood. Treatment involves suppressing gonadotropin secretion with continuous administration of a long-acting GnRH agonist such as leuprolide. The answer is C. 8. Octreotide, a somatostatin analog, has some efficacy in reducing the excess GH production that causes acromegaly. The answer is D. 9. Once the patient’s endogenous gonadotropin production has been inhibited through continuous administration of the GnRH agonist leuprolide, the next step in ovulation induction is the administration of a drug with FSH activity to stimulate follicle maturation. Follitropin is recombinant FSH. The only other drug listed that is used in ovulation induction is hCG, but this is an LH analog. The answer is B. 10. Pituitary diabetes insipidus results from deficiency in vasopressin. It is treated with desmopressin, a peptide agonist of vasopressin V2 receptors. The answer is B. SKILL KEEPER ANSWERS: DRUGS THAT CAUSE HYPERPROLACTINEMIA (SEE CHAPTER 29) 1. Drugs that block dopamine D2 receptors cause hyperprolactinemia by blocking the inhibitory effects of endogenous dopamine on the pituitary cells that release prolactin. 2. The older antipsychotic drugs (eg, phenothiazines, haloperidol), with their strong dopamine D2 receptor-blocking activity, are most likely to be the pharmacologic cause of hyperprolactinemia (see Chapter 29). This adverse effect is less likely with atypical antipsychotic drugs (eg, olanzapine). Drugs or drug groups that cause hyperprolactinemia through mechanisms that are not well characterized include methyldopa (an antihypertensive), amphetamines, tricyclic and other types of antidepressants, and opioids. CHAPTER 37 Hypothalamic & Pituitary Hormones 313 CHECKLIST When you complete this chapter, you should be able to: ❑ Describe the drugs used as substitutes for the natural pituitary hormones, and list their clinical uses. ❑ List the gonadotropin analogs and GnRH agonists and antagonists, and describe their clinical use in treating male and female infertility, endometriosis, and prostate cancer. ❑ Describe the drugs used for treatment of acromegaly and hyperprolactinemia. DRUG SUMMARY TABLE: Drugs that Mimic or Inhibit Hypothalamic & Pituitary Hormones Subclass Mechanism of Action Clinical Applications Pharmacokinetics Toxicities, Interactions Recombinant human GH • acts through GH receptors to increase the production of IGF-1 Replacement in GH deficiency • increased final adult height in children with certain conditions associated with short stature • wasting in HIV infection • short bowel syndrome Subcutaneous (SC) injection In children, pseudotumor cerebri, slipped capital femoral epiphysis, progression of scoliosis, edema, and hyperglycemia • in adults, peripheral edema, myalgia, and arthralgia Recombinant IGF-1 Replacement in IGF-1 deficiency that is not responsive to exogenous GH SC injection Hypoglycemia, intracranial hypertension, increased liver enzymes Somatostatin receptor agonist Acromegaly and several other hormone-secreting tumors • acute control of bleeding from esophageal varices SC injection • long-acting formulation injected intramuscularly (IM) GI disturbances, gallstones, bradycardia, cardiac conduction anomalies SC injection Increased liver enzymes SC injection Ovarian hyperstimulation syndrome and multiple pregnancies in women • gynecomastia in men • headache, depression, edema in both sexes Growth hormone (GH) Somatropin IGF-1 agonist Mecasermin Somatostatin analogs Octreotide Lanreotide: similar to octreotide; available as a long-acting formulation for acromegaly Growth hormone receptor antagonist Pegvisomant Blocks GH receptor signaling Acromegaly Gonadotropins: Follicle-stimulating hormone (FSH) analogs Follitropin alfa Follicle-stimulating hormone (FSH) receptor agonist Controlled ovulation hyperstimulation in women • infertility due to hypogonadotropic hypogonadism in men Follitropin beta: recombinant product with the same peptide sequence as follitropin alfa but differs in its carbohydrate side chains Urofollitropin: human FSH purified from the urine of postmenopausal women Menotropins (hMG): extract of the urine of postmenopausal women; contains both FSH and LH activity (Continued ) 314 PART VII Endocrine Drugs DRUG SUMMARY TABLE: Drugs that Mimic or Inhibit Hypothalamic & Pituitary Hormones (Continued ) Subclass Mechanism of Action Clinical Applications Pharmacokinetics Toxicities, Interactions Initiation of final oocyte maturation and ovulation during controlled ovarian stimulation • male hypogonadotropic hypogonadism IM injection Ovarian hyperstimulation syndrome and multiple pregnancies in women • gynecomastia in men • headache, depression, edema in both sexes Gonadotropins: Luteinizing hormone (LH) analogs Human chorionic gonadotropin (hCG) LH receptor agonist Choriogonadotropin alfa: recombinant form of hCG Lutropin: recombinant form of human LH Menotropins (hMG): extract of the urine of postmenopausal women; contains both FSH and LH activity Gonadotropin-releasing hormone (GnRH) analogs Leuprolide GnRH receptor agonist Administered IV, SC, IM, or intranasally • depot formulations are available Headache, light-headedness, nausea, injection site reactions • with continuous treatment symptoms of hypogonadism SC injection Nausea, headache Hyperprolactinemia, Parkinson’s disease (see Chapter 28) Administered orally or, for hyperprolactinemia, vaginally Gastrointestinal disturbances, orthostatic hypotension, headache, psychiatric disturbances, vasospasm and pulmonary infiltrates in high doses Induction and augmentation of labor • control of uterine hemorrhage after delivery IV infusion Fetal distress, placental abruption, uterine rupture, fluid retention, hypotension Tocolysis for preterm labor IV infusion Concern about rates of infant death • not FDA approved Ovarian suppression • controlled ovarian stimulation • central precocious puberty • block of endogenous puberty in some transgender early pubertal adolescents • advanced prostate cancer Gonadorelin: synthetic human GnRH Other GnRH analogs: goserelin, buserelin, histrelin, nafarelin, and triptorelin GnRH receptor antagonists Ganirelix Antagonist of GnRH receptors Prevention of premature LH surges during controlled ovarian stimulation Cetrorelix: similar to ganirelix, approved for controlled ovarian hyperstimulation Degarelix, abarelix: approved for advanced prostate cancer Dopamine agonists Bromocriptine Dopamine D2 receptor agonist Cabergoline: another ergot derivative with similar effects Oxytocin Oxytocin Oxytocin receptor agonist Oxytocin receptor antagonist Atosiban Antagonist of oxytocin receptor (Continued ) CHAPTER 37 Hypothalamic & Pituitary Hormones 315 DRUG SUMMARY TABLE: Drugs that Mimic or Inhibit Hypothalamic & Pituitary Hormones (Continued ) Subclass Mechanism of Action Clinical Applications Pharmacokinetics Toxicities, Interactions Pituitary diabetes insipidus • hemophilia A and von Willebrand disease Oral, IV, SC, or intranasal administration GI disturbances, headache, hyponatremia, allergic reactions Vasopressin receptor agonists Desmopressin Agonist of vasopressin V2 receptors Vasopressin: treatment of diabetes insipidus and sometimes used to control bleeding from esophageal varices Vasopressin receptor antagonist Conivaptan Antagonist of vasopressin V1a and V2 receptors Hyponatremia in hospitalized patients Administered as an IV infusion Infusion site reactions Tolvaptan: similar but more selective for vasopressin V2 receptors; oral administration limited to 30 day treatment due to hepatotoxicity C A P T E R 38 Thyroid & Antithyroid Drugs The thyroid secretes 2 types of hormones: iodine-containing amino acids (thyroxine and triiodothyronine) and a peptide (calcitonin). Thyroxine and triiodothyronine have broad effects on growth, development, and metabolism. Calcitonin is important H in calcium metabolism and is discussed in Chapter 42. This chapter describes the drugs used in the treatment of hypothyroidism and hyperthyroidism. Drugs used in thyroid disease Hyperthyroidism Hypothyroidism Levothyroxine (T4) Liothyronine (T3) Thioamides (propythiouracil) THYROID HORMONES A. Synthesis and Transport of Thyroid Hormones The thyroid secretes 2 iodine-containing hormones: thyroxine (T4) and triiodothyronine (T3). The iodine necessary for the synthesis of these molecules comes from food or iodide supplements. Iodide ion is actively taken up by and highly concentrated in the thyroid gland, where it is converted to elemental iodine by thyroidal peroxidase (Figure 38–1). The protein thyroglobulin serves as a scaffold for thyroid hormone synthesis. Tyrosine residues in thyroglobulin are iodinated to form monoiodotyrosine (MIT) or diiodotyrosine (DIT) in a process known as iodine organification. Within thyroglobulin, 2 molecules of DIT combine to form T4, while 1 molecule each of MIT and DIT combine to form T3. Proteolysis of thyroglobulin liberates the T4 and T3, which are then released from the thyroid. After release from the gland, T4 and T3 are transported in the blood by thyroxine-binding globulin, a protein synthesized in the liver. 316 Iodide (Lugol solution) Beta blockers (propranolol) 131 I Thyroid function is controlled by the pituitary through the release of thyrotropin (thyroid-stimulating hormone [TSH]) (see Figure 37–1) and by the availability of iodide. Thyrotropin stimulates the uptake of iodide as well as synthesis and release of thyroid hormone. It also has a growth-promoting effect that causes thyroid cell hyperplasia and an enlarged gland (goiter). High levels of thyroid hormones inhibit the release of TSH, providing an effective negative feedback control mechanism. In Graves’ disease, an autoimmune disorder, B lymphocytes produce an antibody that activates the TSH receptor and can cause a syndrome of hyperthyroidism called thyrotoxicosis. Because these lymphocytes are not susceptible to negative feedback, patients with Graves’ disease can have very high blood concentrations of thyroid hormone at the same time that their blood concentrations of TSH are very low. B. Mechanisms of Action of T4 and T3 T3 is about 10 times more potent than T4. Because T4 is converted to T3 in target cells, the liver, and the kidneys, most of the effect of circulating T4 is probably due to T3. Thyroid hormones bind to CHAPTER 38 Thyroid & Antithyroid Drugs 317 High-Yield Terms to Learn Cretinism Irreversible mental retardation and dwarfism caused by congenital hypothyroidism Myxedema Severe hypothyroidism Goiter Enlargement of the thyroid gland Graves’ disease Autoimmune disorder that results in hyperthyroidism during the early phase and can progress to hypothyroidism if there is destruction of the gland in later phases Thyroglobulin A protein synthesized in the thyroid gland; its tyrosine residues are used to synthesize thyroid hormones Thyroid-stimulating hormone (TSH) The anterior pituitary hormone that regulates thyroid gland growth, uptake of iodine and synthesis of thyroid hormone Thyroid storm Severe thyrotoxicosis Thyrotoxicosis Medical syndrome caused by an excess of thyroid hormone (Table 38–1) Thyroxine-binding globulin (TBG) Protein synthesized in the liver that transports thyroid hormone in the blood intracellular receptors that control the expression of genes responsible for many metabolic processes. The proteins synthesized under T3 control differ depending on the tissue involved; these proteins include, for example, Na+/K+ ATPase, specific contractile proteins in smooth muscle and the heart, enzymes involved in lipid metabolism, and important developmental components in the brain. T3 may also have a separate membrane receptormediated effect in some tissues. 1. Effects of thyroid hormone—The organ-level actions of the thyroid hormones include normal growth and development of the nervous, skeletal, and reproductive systems and control of Thyroid gland Transport − I − I Thyroglobulin Peroxidase I° MIT-DIT- T3-T4 Iodides – – Proteolysis – Iodides, thioamides SCN–, ClO4 – T4, T3 Peripheral tissues Blood T4, T3 – Radiocontrast media, β-blockers, corticosteroids, amiodarone T3 Sites of action of some antithyroid drugs. I–, iodide ion; I°, elemental iodine. Not shown: radioactive iodine (131I), which destroys the gland through radiation. (Reproduced, with permission, from Katzung BG, editor: Basic & Clinical Pharmacology, 12th ed. McGraw-Hill, 2012: Fig. 38–1.) FIGURE 38–1 318 PART VII Endocrine Drugs TABLE 38–1 Key features of thyrotoxicosis and hypothyroidism. Thyrotoxicosis Hypothyroidism Warm, moist skin Pale, cool, puffy yellowish skin, face and hands. Brittle hair and nails Sweating, heat intolerance Sensation of being cold Tachycardia, increased stroke volume, cardiac output, and pulse pressure Bradycardia, decreased stroke volume, cardiac output, and pulse pressure Dyspnea Pleural effusions, hypoventilation, and CO2 retention Increased appetite Reduced appetite Nervousness, hyperkinesia, tremor Lethargy, general slowing of mental processes Weakness, increased deep tendon reflexes Stiffness, decreased deep tendon reflexes Menstrual irregularity, decreased fertility Infertility, decreased libido, impotence, oligospermia Weight loss Weight gain Retraction of upper lid with wide stare, exophthalmos (Graves’ disease) Drooping of eyelids metabolism of fats, carbohydrates, proteins, and vitamins. The key features of excess thyroid activity (thyrotoxicosis) and hypothyroidism are listed in Table 38–1. 2. Clinical use—Thyroid hormone therapy can be accomplished with either T4 or T3. Synthetic levothyroxine (T4) is usually the form of choice. T3 (liothyronine) is faster acting but has a shorter half-life and is more expensive. 3. Toxicity—Toxicity is that of thyrotoxicosis (Table 38–1). Older patients, those with cardiovascular disease, and those with longstanding myxedema are highly sensitive to the stimulatory effects of T4 on the heart. Such patients should receive lower initial doses of T4. SKILL KEEPER: THE CYCLIC AMP SECONDMESSENGER SYSTEM (CHAPTER 2) Like many neurotransmitters and hormones, TSH mediates its effects in thyroid cells by activating the cAMP (cyclic adenosine monophosphate) second-messenger system. Draw a diagram that shows the key events in this pathway, beginning with the binding of an agonist to its receptor and ending with cellular responses. The Skill Keeper Answer appears at the end of the chapter. ANTITHYROID DRUGS A. Thioamides Methimazole and propylthiouracil (PTU) are small sulfurcontaining thioamides that inhibit thyroid hormone synthesis by blocking peroxidase-catalyzed reactions, iodination of the tyrosine residues of thyroglobulin, and coupling of DIT and MIT (Figure 38–1). Propylthiouracil and, to a much lesser extent, methimazole inhibit peripheral conversion of T4 to T3. Because the thioamides do not inhibit the release of preformed thyroid hormone, their onset of activity is usually slow, often requiring 3–4 wk for full effect. The thioamides can be used by the oral route and are effective in young patients with small glands and mild disease. Methimazole is generally preferred because it can be administered once per day. However, PTU is preferred in pregnancy because it is less likely than methimazole to cross the placenta and enter breast milk. Toxic effects include skin rash (common) and severe reactions (rare) such as vasculitis, agranulocytosis, hypoprothrombinemia, and liver dysfunction. These effects are usually reversible. B. Iodide Salts and Iodine Iodide salts inhibit iodination of tyrosine and thyroid hormone release (Figure 38–1); these salts also decrease the size and vascularity of the hyperplastic thyroid gland. Because iodide salts inhibit release as well as synthesis of the hormones, their onset of action occurs rapidly, within 2–7 d. However, the effects are transient; the thyroid gland “escapes” from the iodide block after several weeks of treatment. Iodide salts are used in the management of thyroid storm and to prepare patients for surgical resection of a hyperactive thyroid. The usual forms of this drug are Lugol’s solution (iodine and potassium iodide) and saturated solution of potassium iodide. Adverse effects include rash, drug fever, metallic taste, bleeding disorders, and, rarely, anaphylactic reactions. C. Radioactive Iodine Radioactive iodine (131I) is taken up and concentrated in the thyroid gland so avidly that a dose large enough to severely damage the gland can be given without endangering other tissues. Unlike the thioamides and iodide salts, an effective dose of 131I can produce a permanent cure of thyrotoxicosis without surgery. 131 I should not be used in pregnant or nursing women. D. Anion Inhibitors Anions such as thiocyanate (SCN–) and perchlorate (ClO4–) block the uptake of iodide by the thyroid gland through competitive CHAPTER 38 Thyroid & Antithyroid Drugs inhibition of the iodide transporter. Their effectiveness is unpredictable and ClO4– can cause aplastic anemia, so these drugs are rarely used clinically. E. Other Drugs An important class of drugs for the treatment of thyrotoxicosis is the β blockers. These agents are particularly useful in controlling the tachycardia and other cardiac abnormalities of severe thyrotoxicosis. Propranolol also inhibits the peripheral conversion of T4 to T3. The iodine-containing antiarrhythmic drug amiodarone (Chapter 14) can cause hypothyroidism through its ability to block the peripheral conversion of T4 to T3. It also can cause hyperthyroidism either through an iodine-induced mechanism in persons with an underlying thyroid disease such as multinodular goiter or through an inflammatory mechanism that causes leakage of thyroid hormone into the circulation. Amiodarone-associated hypothyroidism is treated with thyroid hormone. Iodine-associated hyperthyroidism caused by amiodarone is treated with thioamides, whereas the inflammatory version is best treated with corticosteroids. Iodinated radiocontrast media (eg, oral diatrizoate and intravenous iohexol) rapidly suppress the conversion of T4 to T3 in the liver, kidney, and other peripheral tissues. QUESTIONS Questions 1–3. A 24-year-old woman was found to have mild hyperthyroidism due to Graves’ disease. She appears to be in good health otherwise. 1. In Graves’ disease, the cause of the hyperthyroidism is the production of an antibody that does which of the following? (A) Activates the pituitary thyrotropin-releasing hormone (TRH) receptor and stimulates TSH release (B) Activates the thyroid gland TSH receptor and stimulates thyroid hormone synthesis and release (C) Activates thyroid hormone receptors in peripheral tissues (D) Binds to thyroid gland thyroglobulin and accelerates its proteolysis and the release of its supply of T4 and T3 (E) Binds to thyroid-binding globulin (TBG) and displaces bound T4 and T3 2. The decision is made to begin treatment with methimazole. Methimazole reduces serum concentration of T3 primarily by which of the following mechanisms? (A) Accelerating the peripheral metabolism of T3 (B) Inhibiting the proteolysis of thyroid-binding globulin (C) Inhibiting the secretion of TSH (D) Inhibiting the uptake of iodide by cells in the thyroid (E) Preventing the addition of iodine to tyrosine residues on thyroglobulin 3. Though rare, a serious toxicity associated with the thioamides is which of the following? (A) Agranulocytosis (B) Lupus erythematosus-like syndrome (C) Myopathy (D) Torsades de pointes arrhythmia (E) Thrombotic thrombocytic purpura (TTP) 319 4. A 56-year-old woman presented to the emergency department with tachycardia, shortness of breath, and chest pain. She had had shortness of breath and diarrhea for the last 2 d and was sweating and anxious. A relative reported that the patient had run out of methimazole 2 wk earlier. A TSH measurement revealed a value of <0.01 mIU/L (normal 0.4–4.0 mIU/L). The diagnosis of thyroid storm was made. Which of the following is a drug that is a useful adjuvant in the treatment of thyroid storm? (A) Amiodarone (B) Betamethasone (C) Epinephrine (D) Propranolol (E) Radioactive iodine 5. A 65-year-old man with multinodular goiter is scheduled for a near-total thyroidectomy. Which of the following drugs will be administered for 10–14 d before surgery to reduce the vascularity of his thyroid gland? (A) Levothyroxine (B) Liothyronine (C) Lugol’s solution (D) Prednisone (E) Radioactive iodine 6. Which of the following is a sign or symptom that would be expected to occur in the event of chronic overdose with exogenous T4? (A) Bradycardia (B) Dry, puffy skin (C) Large tongue and drooping of the eyelids (D) Lethargy, sleepiness (E) Weight loss 7. When initiating T4 therapy for an elderly patient with longstanding hypothyroidism, it is important to begin with small doses to avoid which of the following? (A) A flare-up of exophthalmos (B) Acute renal failure (C) Hemolysis (D) Overstimulation of the heart (E) Seizures 8. A 27-year-old woman underwent near total thyroidectomy. She was started on levothyroxine. What hormone is produced in the peripheral tissues when levothyroxine is administered? (A) Methimazole (B) T3 (C) T4 (D) TSH (E) FSH 9. A 62-year-old woman presents with complaints of fatigue, sluggishness, and weight gain. She needs to nap several times a day, which is unusual for her. She has been taking T4 for the past 15 yr without significant problems regarding her energy level. Her recent history is significant for diagnosis of arrhythmia, and she is currently taking an antiarrhythmic drug. What is the most likely cause of her current condition? (A) Amiodarone (B) Lidocaine (C) Procainamide (D) Sotalol (E) Verapamil 320 PART VII Endocrine Drugs 10. A 25-year-old woman presents with insomnia and fears she may have “something wrong with her heart.” She describes “her heart jumping out of her chest.” She feels healthy otherwise and reports she has lots of energy. Lab tests confirm hyperthyroidism. Which of the following is a drug that produces a permanent reduction in thyroid activity? (A) 131I (B) Methimazole (C) Propylthiouracil (D) Thiocyanate (SCN–) (E) Thyroglobulin ANSWERS 1. The antibodies produced in Graves’ disease activate thyroid gland TSH receptors. Their effects mimic those of TSH. The answer is B. 2. The thioamides (methimazole and propylthiouracil) act in thyroid cells to prevent conversion of tyrosine residues in thyroglobulin to MIT or DIT. The answer is E. 3. Rarely, the thioamides cause severe adverse reactions that include agranulocytosis, vasculitis, hepatic damage, and hypoprothrombinemia. The answer is A. 4. In thyroid storm, β blockers such as propranolol are useful in controlling the tachycardia and other cardiac abnormalities, and propranolol also inhibits peripheral conversion of T4 to T3. The answer is D. 5. Iodides inhibit the synthesis and release of thyroid hormone and decrease the size and vascularity of the hyperplastic gland. Lugol’s solution contains a mixture of potassium iodide and iodine. The answer is C. 6. In hyperthyroidism, the metabolic rate increases, and even though there is increased appetite, weight loss often occurs. The other choices are symptoms seen in hypothyroidism. The answer is E. 7. Patients with longstanding hypothyroidism, especially those who are elderly, are highly sensitive to the stimulatory effects of T4 on cardiac function. Administration of regular doses can cause overstimulation of the heart and cardiac collapse. The answer is D. 8. The thioamides (methimazole and propylthiouracil) act in thyroid cells to prevent conversion of tyrosine residues in thyroglobulin to MIT or DIT. Levothyroxine (T4) is converted into T3 in the periphery. FSH is follicle-stimulating hormone. The answer is B. 9. Amiodarone is an iodine-containing antiarrhythmic drug with complex effects on the thyroid gland and thyroid hormones. One of its actions is to inhibit peripheral conversion of T4 to T3. Note that propranolol also reduces conversion of T4 to T3. Procainamide (class 1A), lidocaine (class 1B), sotalol (class III), and verapamil (class IV) are antiarrhythmics and have no effect on T4 conversion. The answer is A. 10. Propylthiouracil and, to a much lesser extent, methimazole inhibit peripheral conversion of T4 to T3. Thyroglobulin is not a drug. Radioactive iodine is the only medical therapy that produces a permanent reduction of thyroid activity. Anions such as thiocyanate (SCN–) and perchlorate (ClO4–) block the uptake of iodide by the thyroid gland through competitive inhibition of the iodide transporter. Their effectiveness is unpredictable and ClO4– can cause aplastic anemia, so these drugs are rarely used. The answer is A. SKILL KEEPER ANSWER: THE CYCLIC AMP SECOND-MESSENGER SYSTEM (CHAPTER 2) Your drawing should show that receptor (Rec) stimulation acts through the G protein Gs to activate the enzyme adenylyl cyclase (AC). Adenylyl cyclase converts ATP to cAMP, which binds to the regulatory subunit (R) of cAMP-dependent protein kinases and thereby frees the catalytic subunit (C) of the kinase so it can transfer phosphate from ATP to substrate proteins (S) that mediate the ultimate cellular responses. These responses are varied and include immediately apparent effects that stem from phosphorylation of substrates such as enzymes and ion channels as well as delayed effects that follow changes in gene transcription. “Brakes” are applied to the pathway by phosphodiesterases (PDE) that hydrolyze cAMP and phosphatases (P’ase) that dephosphorylate substrates. Agonist Rec Gs ATP Membrane AC cAMP 5′-AMP PDE R2 • cAMP4 R2C2 2C∗ ADP ATP S~P S Pi P’ase Response (Reproduced, with permission, from Katzung BG, editor: Basic & Clinical Pharmacology, 11th ed. McGraw-Hill, 2009: Fig. 2–13.) CHAPTER 38 Thyroid & Antithyroid Drugs 321 CHECKLIST When you complete this chapter, you should be able to: ❑ Sketch the biochemical pathway for thyroid hormone synthesis and release and indicate the sites of action of antithyroid drugs. ❑ List the principal drugs for the treatment of hypothyroidism. ❑ List the principal drugs for the treatment of hyperthyroidism and compare the onset and duration of their action. ❑ Describe the major toxicities of thyroxine and the antithyroid drugs. DRUG SUMMARY TABLE: Thyroid & Antithyroid Drugs Subclass Toxicities, Drug Interactions Mechanism of Action Clinical Applications Pharmacokinetics Activation of nuclear receptors results in gene expression with RNA formulation and protein synthesis Hypothyroidism T4 is converted to T3 in target cells, the liver, and the kidneys • T3 is 10× more potent than T4 See Table 38–1 for symptoms of thyroid excess Inhibit thyroid peroxidase reactions, iodine organification, and peripheral conversion of T4 to T3 Hyperthyroidism Oral administration, delayed onset of activity Nausea, gastrointestinal disturbances, rash, agranulocytosis, hepatitis, hypothyroidism Lugol’s solution, potassium iodide Inhibit iodine organification and hormone release • reduce size and vascularity of thyroid gland Preparation for surgical thyroidectomy Oral administration, acute onset of activity within 2–7 d Rare Radioactive iodine (131I) Radiation-induced destruction of thyroid parenchyma Hyperthyroidism Oral administration Sore throat, hypothyroidism Inhibition of β receptors; inhibition of conversion of T4 to T3 Thyroid storm Rapid onset of activity Asthma, AV blockade, hypotension, bradycardia Thyroid preparations Levothyroxine (T4) Liothyronine (T3) Thioamides Propylthiouracil (PTU) Methimazole Iodides Beta blockers Propranolol C H A P T E R 39 Corticosteroids & Antagonists The corticosteroids are steroid hormones produced by the adrenal cortex. They consist of 2 major physiologic and pharmacologic groups: (1) glucocorticoids, which have important effects on intermediary metabolism, catabolism, immune responses, and inflammation; and (2) mineralocorticoids, which regulate sodium and potassium reabsorption in the collecting tubules of the kidney. This chapter reviews the glucocorticoids, the mineralocorticoids, and the corticosteroid antagonists. Corticosteroid Agonists & Antagonists Agonists Glucocorticoids (prednisone) Antagonists Mineralocorticoids (fludrocortisone) Receptor antagonists Glucocorticoid antagonists (mifepristone) GLUCOCORTICOIDS A. Mechanism of Action Corticosteroids enter the cell and bind to cytosolic receptors that transport the steroid into the nucleus. The steroid-receptor complex alters gene expression by binding to glucocorticoid response elements (GREs) or mineralocorticoid-specific elements (Figure 39–1). Tissuespecific responses to steroids are made possible by the presence in each tissue of different protein regulators that control the interaction between the hormone-receptor complex and particular DNA response elements. B. Organ and Tissue Effects 1. Metabolic effects—Glucocorticoids stimulate gluconeogenesis. As a result, blood glucose rises, muscle protein is catabolized, and insulin secretion is stimulated. Both lipolysis and lipogenesis 322 Synthesis inhibitors (ketoconazole) Mineralocorticoid antagonists (spironolactone) are stimulated, with a net increase of fat deposition in certain areas (eg, the face and the shoulders and back) 2. Catabolic effects—Glucocorticoids cause muscle protein catabolism. In addition, lymphoid and connective tissue, fat, and skin undergo wasting under the influence of high concentrations of these steroids. Catabolic effects on bone can lead to osteoporosis. In children, growth is inhibited. 3. Immunosuppressive effects—Glucocorticoids inhibit cellmediated immunologic functions, especially those dependent on lymphocytes. These agents are actively lymphotoxic and, as such, are important in the treatment of hematologic cancers. The drugs do not interfere with the development of normal acquired immunity but delay rejection reactions in patients with organ transplants. CHAPTER 39 Corticosteroids & Antagonists 323 High-Yield Terms to Learn Addison’s disease Partial or complete loss of adrenocortical function, including loss of glucocorticoid and mineralocorticoid function Adrenal suppression A suppression of the ability of the adrenal cortex to produce corticosteroids. Most commonly is an iatrogenic effect of prolonged exogenous glucocorticoid treatment Cushing’s syndrome A metabolic disorder caused by excess secretion of adrenocorticoid steroids, which is most commonly due to increased amounts of ACTH Glucocorticoid A substance, usually a steroid, that activates glucocorticoid receptors (eg, cortisol) Mineralocorticoid A substance, usually a steroid, that activates mineralocorticoid receptors (eg, aldosterone) 4. Anti-inflammatory effects—Glucocorticoids have a dramatic suppressant effect on numerous inflammatory processes. These drugs increase neutrophils and decrease lymphocytes, eosinophils, basophils, and monocytes. The migration of leukocytes is also inhibited. The biochemical mechanisms underlying these cellular effects include the induced synthesis of an inhibitor of phospholipase A2 (Chapter 18), decreased mRNA for cyclooxygenase 2 (COX-2), decreases in interleukin-2 (IL-2) and IL-3, and decreases in platelet activating factor (PAF), an inflammatory cytokine. R Hsp90 Hsp90 x 5. Other effects—Glucocorticoids such as cortisol are required for normal renal excretion of water loads. The glucocorticoids also have effects on the CNS. When given in large doses, these drugs may cause profound behavioral changes. Large doses also stimulate gastric acid secretion and decrease resistance to ulcer formation. C. Important Glucocorticoids 1. Cortisol—The major natural glucocorticoid is cortisol (hydrocortisone; Figure 39–2). The physiologic secretion of cortisol is regulated (Unstable) S R S S CBG S S S Steroid-receptor R* R* dimer (activated) DNA Response S S R* R* GRE Protein mRNA pre(Editing) mRNA Cytoplasm Transcription machinery (RNA polymerase, etc) Nucleus FIGURE 39–1 Mechanism of glucocorticoid action. This figure models the interaction of a steroid (S; eg, cortisol), with its receptor (R) and the subsequent events in a target cell. The steroid is present in the blood bound to corticosteroid-binding globulin (CBG) but enters the cell as the free molecule. The intracellular receptor is bound to stabilizing proteins, including heat shock protein 90 (Hsp90) and several others (X). When the complex binds a molecule of steroid, the Hsp90 and associated molecules are released. The steroid-receptor complex enters the nucleus as a dimer, binds to the glucocorticoid response element (GRE) on the gene, and regulates gene transcription. The resulting mRNA is edited and exported to the cytoplasm for the production of protein that brings about the final hormone response. (Reproduced, with permission, from Katzung BG, editor: Basic & Clinical Pharmacology, 12th ed. McGraw-Hill, 2012: Fig. 39–4.) 324 PART VII Endocrine Drugs Acetate Cholesterol CH3 17α-Hydroxylase (P450c17) CH3 17, 20-Lyase C O C O O OH (ACTH?) NADPH O2 HO HO CH3 3β-Dehydrogenase ∆5, ∆4-Isomerase NAD+ C Dehydroepi__________ androsterone ___________ 17-Hydroxypregnenolone Pregnenolone HO CH3 C O O O OH O O CH2OH 21α-Hydroxylase (P450c21) C ∆4-Androstene3,17-dione 17-Hydroxyprogesterone Progesterone O CH2OH C O O OH Testosterone 11-Deoxycorticosterone O 11β-Hydroxylase (P450c11) 11β-Deoxycortisol O CH2OH C CH2OH C O HO O Estradiol OH HO CH2OH C CHO HO Cortisol ______ Corticosterone O O O Aldosterone __________ O Mineralocorticoid pathway Glucocorticoid pathway Androgen and estrogen pathway FIGURE 39–2 Outline of major pathways in adrenocortical hormone biosynthesis. The names of major adrenal secretory products are underlined. The enzymes and cofactors for the reactions progressing down each column are shown on the left and across columns at the top of the figure. When a particular enzyme is deficient, hormone production is blocked at points indicated by the shaded bars. (Modified and reproduced, with permission, from Ganong WF: Review of Medical Physiology, 22nd ed. McGraw-Hill, 2005: Fig. 20–8.) by adrenocorticotropin (ACTH) and varies during the day (circadian rhythm); the peak occurs in the morning and the trough occurs about midnight. In the plasma, cortisol is 95% bound to corticosteroidbinding globulin (CBG). Given orally, cortisol is well absorbed from the gastrointestinal tract, is cleared by the liver, and has a short duration of action compared with its synthetic congeners (Table 39–1). Although it diffuses poorly across normal skin, cortisol is readily absorbed across inflamed skin and mucous membranes. TABLE 39–1 Properties of representative corticosteroids. Agent Primarily glucocorticoid Cortisol Prednisone Triamcinolone Dexamethasone Primarily mineralocorticoid Aldosterone Fludrocortisone a Relative to cortisol. Duration of Action (hours) 8–12 12–24 15–24 24–36 1–2 8–12 Anti-inflammatory Activitya 1 4 5 30 0.3 10 Salt-retaining Activitya Topical Activity 1 0.3 0 0 0 (+) +++ +++++ 3000 125–250 0 0 CHAPTER 39 Corticosteroids & Antagonists The cortisol molecule also has a small but significant salt-retaining (mineralocorticoid) effect (Table 39–1). This is an important cause of hypertension in patients with a cortisol-secreting adrenal tumor or a pituitary ACTH-secreting tumor (Cushing’s syndrome). 2. Synthetic glucocorticoids—The mechanism of action of these agents is identical with that of cortisol. A large number of synthetic glucocorticoids are available for use; prednisone and its active metabolite, prednisolone, dexamethasone, and triamcinolone are representative. Their properties (compared with cortisol) include longer half-life and duration of action, reduced salt-retaining effect, and better penetration of lipid barriers for topical activity (Table 39–1). Special glucocorticoids have been developed for use in asthma (see Chapter 20) and other conditions in which good surface activity on mucous membranes or skin is needed and systemic effects are to be avoided. Beclomethasone and budesonide readily penetrate the airway mucosa but have very short half-lives after they enter the blood, so that systemic effects and toxicity are greatly reduced. D. Clinical Uses 1. Adrenal disorders—Glucocorticoids are essential to preserve life in patients with chronic adrenal cortical insufficiency (Addison’s disease) and are necessary in acute adrenal insufficiency associated with life-threatening shock, infection, or trauma. Glucocorticoids are also used in certain types of congenital adrenal hyperplasia, in which synthesis of abnormal forms of corticosteroids are stimulated by ACTH. In these conditions, administration of a potent synthetic glucocorticoid suppresses ACTH secretion sufficiently to reduce the synthesis of the abnormal steroids. 2. Nonadrenal disorders—Many disorders respond to corticosteroid therapy. Some of these are inflammatory or immunologic in nature (eg, asthma, organ transplant rejection, collagen diseases, rheumatic disorders). Other applications include the treatment of hematopoietic cancers, neurologic disorders, chemotherapyinduced vomiting, hypercalcemia, and mountain sickness. Betamethasone, a glucocorticoid with a low degree of protein binding, is given to pregnant women in premature labor to hasten maturation of the fetal lungs. The degree of benefit differs considerably in different disorders, and the toxicity of corticosteroids given chronically limits their use. E. Toxicity Most of the toxic effects of the glucocorticoids are predictable from the effects already described. Some are life threatening and include metabolic effects (growth inhibition, diabetes, muscle wasting, osteoporosis), salt retention, and psychosis. Methods for minimizing these toxicities include local application (eg, aerosols for asthma), alternate-day therapy (to reduce pituitary suppression), and tapering the dose soon after achieving a therapeutic response. To avoid adrenal insufficiency in patients who have had long-term therapy, additional “stress doses” may need to be given during serious illness or before major surgery. Patients who are being withdrawn from glucocorticoids after protracted use should have their doses tapered slowly, over the course of several months, to allow recovery of normal adrenal function. 325 MINERALOCORTICOIDS A. Aldosterone The major natural mineralocorticoid in humans is aldosterone, which is discussed in connection with hypertension (see Chapter 11) and with control of its secretion by angiotensin II (see Chapter 17). The secretion of aldosterone is regulated by ACTH and by the renin-angiotensin system and is very important in the regulation of blood volume and blood pressure (see Figure 6–4). Aldosterone has a short half-life and little glucocorticoid activity (Table 39–1). Its mechanism of action is the same as that of the glucocorticoids. B. Other Mineralocorticoids Other mineralocorticoids include deoxycorticosterone, the naturally occurring precursor of aldosterone, and fludrocortisone, which also has significant glucocorticoid activity. Because of its long duration of action (Table 39–1), fludrocortisone is favored for replacement therapy after adrenalectomy and in other conditions in which mineralocorticoid therapy is needed. CORTICOSTEROID ANTAGONISTS A. Receptor Antagonists Spironolactone and eplerenone, antagonists of aldosterone at its receptor, are discussed in connection with the diuretics (see Chapter 15). Mifepristone (RU-486) is a competitive inhibitor of glucocorticoid receptors as well as progesterone receptors (see Chapter 40) and has been used in the treatment of Cushing’s syndrome. SKILL KEEPER: ALDOSTERONE ANTAGONISTS AND CONGESTIVE HEART FAILURE (CHAPTERS 13 AND 15) Clinical trials have shown that the aldosterone receptor antagonists spironolactone and eplerenone decrease morbidity and mortality in patients who are taking other standard therapies. 1. Why is aldosterone elevated in patients with congestive heart failure? 2. How does the increase in aldosterone contribute to the signs and symptoms of heart failure? 3. What happens to serum potassium concentrations in patients who are treated with aldosterone antagonists? The Skill Keeper Answers appear at the end of the chapter. B. Synthesis Inhibitors Several drugs inhibit adrenal steroid synthesis. The most important of these drugs are ketoconazole, aminoglutethimide, and metyrapone. Ketoconazole (an antifungal drug) inhibits the cytochrome P450 enzymes necessary for the synthesis of all steroids and is used in a number of conditions in which reduced 326 PART VII Endocrine Drugs steroid levels are desirable (eg, adrenal carcinoma, hirsutism, breast and prostate cancer). Aminoglutethimide blocks the conversion of cholesterol to pregnenolone (Figure 39–2) and also inhibits synthesis of all hormonally active steroids. It can be used in conjunction with other drugs for treatment of steroidproducing adrenocortical cancer. Metyrapone inhibits the normal synthesis of cortisol but not that of cortisol precursors; the drug can be used in diagnostic tests of adrenal function. 5. Which of the following best describes a glucocorticoid response element? (A) A protein regulator that controls the interaction between an activated steroid receptor and DNA (B) A short DNA sequence that binds tightly to RNA polymerase (C) A small protein that binds to an unoccupied steroid receptor protein and prevents it from becoming denatured (D) A specific nucleotide sequence that is recognized by a steroid hormone receptor-hormone complex (E) The portion of the steroid receptor that binds to DNA QUESTIONS 6. Glucocorticoids have proved useful in the treatment of which of the following medical conditions? (A) Chemotherapy-induced vomiting (B) Essential hypertension (C) Hyperprolactinemia (D) Parkinson’s disease (E) Type II diabetes 1. A 50-year-old woman, a known asthmatic for the past 30 years, presented to the emergency department with a 2-d history of worsening breathlessness and cough. Chest auscultation revealed bilateral polyphonic inspiratory and expiratory wheeze. Supplemental oxygen, nebulized albuterol (salbutamol) (5 mg) and ipratropium (250 µg), as well as intravenous methyl prednisolone (40 mg) were administered. Which of the following is a pharmacologic effect of exogenous glucocorticoids? (A) Increased muscle mass (B) Hypoglycemia (C) Inhibition of leukotriene synthesis (D) Improved wound healing (E) Increased excretion of salt and water 2. A 34-year-old woman with ulcerative colitis has required long-term treatment with pharmacologic doses of a glucocorticoid agonist. Which of the following is a toxic effect associated with long-term glucocorticoid treatment? (A) A lupus-like syndrome (B) Adrenal gland neoplasm (C) Hepatotoxicity (D) Osteoporosis (E) Precocious puberty in children 3. A 46-year-old male patient has Cushing’s syndrome due to an adrenal tumor. Which of the following drugs would be expected to reduce the signs and symptoms of this man’s disease? (A) Betamethasone (B) Cortisol (C) Fludrocortisone (D) Ketoconazole (E) Triamcinolone 4. A newborn girl exhibited ambiguous genitalia, hyponatremia, hyperkalemia, and hypotension as a result of genetic deficiency of 21α-hydroxylase activity. Treatment consisted of fluid and salt replacement and hydrocortisone administration. In this type of adrenal hyperplasia in which there is excess production of cortisol precursors, which of the following describes the primary therapeutic effect of glucocorticoid administration? (A) Increased adrenal estrogen synthesis (B) Inhibition of adrenal aldosterone synthesis (C) Prevention of hypoglycemia (D) Recovery of normal immune function (E) Suppression of ACTH secretion 7. A 56-year-old woman with systemic lupus erythematosus had been maintained on a moderate daily dose of prednisone for 9 months. Her disease has finally gone into remission and she now wishes to gradually taper and then discontinue the prednisone. Gradual tapering of a glucocorticoid is required for recovery of which of the following? (A) Depressed release of insulin from pancreatic B cells (B) Hematopoiesis in the bone marrow (C) Normal osteoblast function (D) The control by vasopressin of water excretion (E) The hypothalamic-pituitary-adrenal system 8. A patient presents with pain and stiffness in his wrists and knees. The stiffness is worse first thing in the morning. A blood test confirms rheumatoid arthritis. You advise a short course of steroids. Which one of the following is the most potent anti-inflammatory steroid? (A) Cortisol (B) Dexamethasone (C) Fludrocortisone (D) Prednisone (E) Triamcinolone 9. A 54-year-old man with advanced tuberculosis has developed signs of severe acute adrenal insufficiency. The patient should be treated immediately. Which of the following combinations is most rational? (A) Aldosterone and fludrocortisone (B) Cortisol and fludrocortisone (C) Dexamethasone and metyrapone (D) Fludrocortisone and metyrapone (E) Triamcinolone and dexamethasone 10. Which of the following is a drug that, in high doses, blocks the glucocorticoid receptor? (A) Aminoglutethimide (B) Beclomethasone (C) Ketoconazole (D) Mifepristone (E) Spironolactone CHAPTER 39 Corticosteroids & Antagonists ANSWERS 1. Glucocorticoids inhibit the production of both leukotrienes and prostaglandins via inhibition of phospholipase A2. This is a key component of their anti-inflammatory action. The answer is C. 2. One of the adverse metabolic effects of long-term glucocorticoid therapy is a net loss of bone, which can result in osteoporosis. The answer is D. 3. Ketoconazole inhibits many types of cytochrome P450 enzymes. It can be used to reduce the unregulated overproduction of corticosteroids by adrenal tumors. The answer is D. 4. A 21α-hydroxylase deficiency prevents normal synthesis of cortisol and aldosterone, and causes accumulation of cortisol precursors (Figure 39–2). The hypothalamic-pituitary system responds to the abnormally low levels of cortisol by increasing ACTH release. High levels of ACTH induce adrenal hyperplasia and excess production of adrenal androgens, which can cause virilization of females and prepubertal males. Glucocorticoid is administered to replace the missing mineralocorticoid and glucocorticoid activity and to suppress ACTH release, which removes the stimulus for excess adrenal androgen production. The answer is E. 5. Activated steroid hormone receptors mediate their effects on gene expression by binding to hormone response elements, which are short sequences of DNA located near steroidregulated genes. The answer is D. 6. Glucocorticoids are used in combination with other antiemetics to prevent chemotherapy-induced nausea and vomiting, which are commonly associated with anticancer drugs. The answer is A. 7. Exogenous glucocorticoids act at the hypothalamus and pituitary to suppress the production of CRF and ACTH. As a result, adrenal production of endogenous corticosteroids is suppressed. On discontinuance, the recovery of normal hypothalamic-pituitary-adrenal function occurs slowly. Glucocorticoid doses must be tapered slowly, over several months, to prevent adrenal insufficiency. The answer is E. 8. Of the drugs listed, cortisol has the lowest and dexamethasone the highest anti-inflammatory activity. The answer is B. 9. In acute adrenal insufficiency, there is loss of salt and water that is primarily due to reduced production of aldosterone. The loss of salt and water can lead to dehydration. A rational combination of drugs should include agents with complementary effects (ie, a glucocorticoid and a mineralocorticoid). The combination with these characteristics is cortisol and fludrocortisone. (Note that although fludrocortisone may have sufficient glucocorticoid activity for a patient with mild disease, a patient in severe acute adrenal insufficiency needs a full glucocorticoid such as cortisol.) The answer is B. 10. Mifepristone is a competitive antagonist of glucocorticoid and progesterone receptors. Ketoconazole and aminoglutethimide also antagonize corticosteroids; however, they act by inhibiting steroid hormone synthesis. The answer is D. SKILL KEEPER ANSWERS: ALDOSTERONE ANTAGONISTS AND CONGESTIVE HEART FAILURE (CHAPTERS 13 AND 15) 1. The reduction in cardiac output associated with heart failure decreases the effective arterial blood volume and renal blood flow. Decreased pressure in renal arterioles and increased sympathetic neural activity both stimulate renin release, which increases production of angiotensin II. Angiotensin II is a powerful stimulus of aldosterone secretion. 2. Acting through nuclear receptors in the epithelial cells that line renal collecting tubules, aldosterone promotes renal uptake of salt and water. This retention of salt and water exacerbates the peripheral and pulmonary edema associated with congestive heart failure and further overloads the weakened heart. In addition to these renal effects, aldosterone is also implicated in myocardial and vascular fibrosis and baroreceptor dysfunction. 3. The aldosterone antagonists are also known as “potassiumsparing diuretics” because, unlike other diuretics, they do not promote renal excretion of potassium. Because the excretion of potassium in the renal tubule is linked to the reuptake of sodium, the reduction in sodium uptake caused by spironolactone and eplerenone results in potassium retention and an increase in serum potassium. CHECKLIST When you complete this chapter, you should be able to: ❑ Describe the major naturally occurring glucocorticosteroid and its actions. ❑ List several synthetic glucocorticoids, and describe differences between these agents and the naturally occurring hormone. ❑ Describe the actions of the naturally occurring mineralocorticoid and 1 synthetic agent in this subgroup. ❑ List the indications for the use of corticosteroids in adrenal and nonadrenal disorders. ❑ Name 3 drugs that interfere with the action or synthesis of corticosteroids, and, for each, describe its mechanism of action. 327 328 PART VII Endocrine Drugs DRUG SUMMARY TABLE: Corticosteroids & Antagonists Subclass Mechanism of Action Clinical Applications Pharmacokinetics Activation of glucocorticoid receptor alters gene transcription Many inflammatory conditions, organ transplantation, hematologic cancers Duration of activity is longer than pharmacokinetic half-life of drug owing to gene transcription effects Toxicities, Drug Interactions Glucocorticoid agonists Prednisone Adrenal suppression, growth inhibition, muscle wasting, osteoporosis, salt retention, glucose intolerance, behavioral changes Many other glucocorticoids available for oral and parenteral use (see Table 39–1). Cortisol is the primary endogenous glucocorticoid hormone Mineralocorticoid agonist Fludrocortisone Strong agonist at mineralo-corticoid receptors and moderate activation of glucocorticoid receptors Adrenal insufficiency (Addison’s disease) Long duration of action (see Table 39–1) Salt and fluid retention, congestive heart failure, signs and symptoms of glucocorticoid excess (see above) Medical abortion (see Chapter 40) and very rarely Cushing’s syndrome Oral administration Vaginal bleeding in females, abdominal pain, gastrointestinal upset, diarrhea, headache Aldosteronism from any cause, hypokalemia due to other diuretics, post-myocardial infarction Slow onset and offset of effect Duration: 24–48 h Hyperkalemia, gynecomastia (spironolactone, not eplerenone), additive interaction with other K-retaining drugs Oral, topical administration Hepatic dysfunction, many drug-drug CYP450 interactions Glucocorticoid receptor antagonist Mifepristone Pharmacologic antagonist of glucocorticoid and progesterone receptors Mineralocorticoid receptor antagonists Spironolactone Pharmacologic antagonist of mineralocorticoid receptor, weak antagonism of androgen receptors Eplerenone: similar to spironolactone, more selective for mineralocorticoid receptor Synthesis inhibitors Ketoconazole Blocks fungal and mammalian CYP450 enzymes Inhibits mammalian steroid hormone synthesis and fungal ergosterol synthesis (see Chapter 48) Other adrenal steroid synthesis inhibitors: include aminoglutethimide and metyrapone C A P T E R 40 Gonadal Hormones & Inhibitors The gonadal hormones include the steroids of the ovary (estrogens and progestins) and testis (chiefly testosterone). Because of their importance as contraceptives, many synthetic estrogens and progestins have been produced. These include synthesis inhibitors, receptor antagonists, and some drugs with mixed effects (ie, agonist effects in some tissues and antagonist effects in other tissues). Mixed agonists with estrogenic effects H are called selective estrogen receptor modulators (SERMs). Synthetic androgens, including those with anabolic activity, are also available for clinical use. A diverse group of drugs with antiandrogenic effects is used in the treatment of prostate cancer and benign prostatic hyperplasia in men and androgen excess in women. Gonadal hormone agonists & antagonists Estrogens (ethinyl estradiol) Antiestrogens Receptor antagonists Aromatase inhibitors (anastrozole) Full antagonists (fulvestrant) Progestins (L-norgestrel) Antiprogestins Androgens (testosterone) Antiandrogens Mifepristone Other Receptor 5-α-Reductase Synthesis Other (GnRH agonists, antagonists inhibitors inhibitors (GnRH agonist, danazol) (flutamide) (ketoconazole) combined oral (finasteride) contraceptives) SERMs (tamoxifen) OVARIAN HORMONES The ovary is the primary source of gonadal hormones in women during the childbearing years (ie, between puberty and menopause). When properly regulated by follicle-stimulating hormone (FSH) and luteinizing hormone (LH) from the pituitary, each menstrual cycle consists of the following events: A follicle in the ovary matures, secretes increasing amounts of estrogen, releases an ovum, and is transformed into a progesterone-secreting corpus luteum. If the ovum is not fertilized and implanted, the corpus luteum degenerates; the uterine endometrium, which has proliferated under the stimulation of estrogen and progesterone, is shed as part of the menstrual flow, and the cycle repeats. The mechanism of action of both estrogen and progesterone involves entry into cells, binding to cytosolic receptors, and translocation of the receptor–hormone complex into the nucleus, where it modulates gene expression (see Figure 39–1). 329 330 PART VII Endocrine Drugs High-Yield Terms to Learn 5`-Reductase The enzyme that converts testosterone to dihydrotestosterone (DHT); it is inhibited by finasteride, a drug used to treat benign prostatic hyperplasia and prevent male-pattern hair loss in men Anabolic steroid Androgen receptor agonists used for anabolic effects (eg, weight gain, increased muscle mass) Breakthrough bleeding Vaginal bleeding that occurs outside of the period of regular menstrual bleeding Combined oral contraceptive (COC or OC) Hormonal contraceptive administered orally that contains an estrogen and a progestin Hirsutism A male pattern of body hair growth (face, chest, abdomen) in females that results from hyperandrogenism HRT Hormone replacement therapy; refers to estrogen replacement for women who have lost ovarian function and usually involves combination therapy with estrogen and a progestin SERM Selective estrogen receptor modulator, eg, tamoxifen A. Estrogens The major ovarian estrogen in women is estradiol. Estradiol has low oral bioavailability but is available in a micronized form for oral use. It can also be administered via transdermal patch, vaginal cream, or intramuscular injection. Long-acting esters of estradiol that are converted in the body to estradiol (eg, estradiol cypionate) can be administered by intramuscular (IM) injection. Mixtures of conjugated estrogens from biologic sources (eg, Premarin) are used orally for hormone replacement therapy (HRT). Synthetic estrogens with high bioavailability (eg, ethinyl estradiol, mestranol) are used in hormonal contraceptives. 1. Effects—Estrogen is essential for normal female reproductive development. It is responsible for the growth of the genital structures (vagina, uterus, and uterine tubes) during childhood and for the appearance of secondary sexual characteristics and the growth spurt associated with puberty. Estrogen has many metabolic effects: It modifies serum protein levels and reduces bone resorption. It enhances the coagulability of blood and increases plasma triglyceride levels while reducing low-density lipoprotein (LDL) cholesterol and increasing high-density lipoprotein (HDL) cholesterol. Continuous administration of estrogen, especially in combination with a progestin, inhibits the secretion of gonadotropins from the anterior pituitary (Figure 40–1). 2. Clinical use—Estrogens are used in the treatment of hypogonadism in young females (Table 40–1). Another use is as HRT in women with estrogen deficiency resulting from premature ovarian failure, menopause, or surgical removal of the ovaries. HRT ameliorates hot flushes and atrophic changes in the urogenital tract. It is effective also in preventing bone loss and osteoporosis. The estrogens are components of hormonal contraceptives (see later discussion). 3. Toxicity—In hypogonadal girls, the dosage of estrogen must be adjusted carefully to prevent premature closure of the epiphyses of the long bones and short stature. When used as HRT, estrogen increases the risk of endometrial cancer; this effect is prevented by combining the estrogen with a progestin. Estrogen use by postmenopausal women is associated with a small increase in the risk of breast cancer and cardiovascular events (myocardial infarction, stroke). Dose-dependent toxicity includes nausea, breast tenderness, increased risk of migraine headache, thromboembolic events (eg, deep vein thrombosis), gallbladder disease, hypertriglyceridemia, and hypertension. Diethylstilbestrol (DES), a nonsteroidal estrogenic compound, is associated with infertility, ectopic pregnancy, and vaginal adenocarcinoma in the daughters of women who were treated with the drug during pregnancy in a misguided attempt to prevent recurrent spontaneous abortion. These effects appear to be restricted to DES because there is no evidence that the estrogens and progestins in hormonal contraceptives have similar effects or other teratogenic effects. B. Progestins Progesterone is the major progestin in humans. A micronized form is used orally for HRT, and progesterone-containing vaginal creams are also available. Synthetic progestins (eg, medroxyprogesterone) have improved oral bioavailability. The 19-nortestosterone compounds differ primarily in their degree of androgenic effects. Older drugs (eg, L-norgestrel and norethindrone) are more androgenic than the newer progestins (eg, norgestimate, desogestrel). 1. Effects—Progesterone induces secretory changes in the endometrium and is required for the maintenance of pregnancy. The other progestins named above, also stabilize the endometrium but do not support pregnancy. Progestins do not significantly affect plasma proteins, but they do affect carbohydrate metabolism and stimulate the deposition of fat. High doses suppress gonadotropin secretion and often cause anovulation in women. 2. Clinical use—Progestins are used as contraceptives, either alone or in combination with an estrogen. They are used in combination CHAPTER 40 Gonadal Hormones & Inhibitors Hypothalamus GnRH GnRH antagonists – +/– Clomiphene Oral contraceptives, danazol + – Anterior pituitary GnRH agonists: + or – depending on timing FSH, LH Ovary Progesterone (Luteal phase) – Ketoconazole, danazol – Anastrozole, others Testosterone Androstenedione Estradiol Estrone – + /– Estriol Fulvestrant SERMS: + or – depending on tissue type Estrogen response element Expression in estrogen-responsive cells FIGURE 40–1 Control of ovarian secretion, the action of its hormones, and some sites of action of antiestrogens. In the follicular phase, the ovary produces mainly estrogens; in the luteal phase it produces estrogens and progesterone. SERMs, selective estrogen receptor modulators. (Reproduced, with permission, from Katzung BG, editor: Basic & Clinical Pharmacology, 12th ed. McGraw-Hill, 2012: Fig. 40–5.) with an estrogen in HRT to prevent estrogen-induced endometrial cancer. Progesterone is used in assisted reproductive technology methods to promote and maintain pregnancy. 3. Toxicity—The toxicity of progestins is low. However, they may increase blood pressure and decrease HDL. Long-term use of high doses in premenopausal women is associated with a reversible decrease in bone density (a secondary effect of ovarian suppression and decreased ovarian production of estrogen) and delayed resumption of ovulation after termination of therapy. 331 C. Hormonal Contraceptives Hormonal contraceptives contain either a combination of an estrogen and a progestin or a progestin alone. Hormonal contraceptives are available in a variety of preparations, including oral pills, longacting injections, subcutaneous implants, transdermal patches, vaginal rings, and intrauterine devices (IUDs) (Table 40–1). Three types of oral contraceptives for women are available in the United States: combination estrogen-progestin tablets that are taken in constant dosage throughout the menstrual cycle (monophasic preparations); combination preparations (biphasic, triphasic, and quadriphasic) in which the progestin or estrogen dosage, or both, changes during the month (to more closely mimic hormonal changes in a menstrual cycle); and progestin-only preparations. The postcoital contraceptives (also known as “emergency contraception”) prevent pregnancy if administered within 72 h after unprotected intercourse. Oral preparations containing a progestin (l-norgestrel) alone, estrogen alone, or the combination of an estrogen and a progestin are effective. The progestin-only preparation causes fewer side effects than the estrogen-containing preparations. 1. Mechanism of action—The combination hormonal contraceptives have several actions, including inhibition of ovulation (the primary action) and effects on the cervical mucus glands, uterine tubes, and endometrium that decrease the likelihood of fertilization and implantation. Progestin-only agents do not always inhibit ovulation and instead act through the other mechanisms listed. The mechanisms of action of postcoital contraceptives are not well understood. When administered before the LH surge, they inhibit ovulation. They also affect cervical mucus, tubal function, and the endometrial lining. 2. Other clinical uses and beneficial effects—Combination hormonal contraceptives are used in young women with primary hypogonadism to prevent estrogen deficiency. Combinations of hormonal contraceptives and progestins are used to treat acne, hirsutism, dysmenorrhea, and endometriosis. Users of combination hormonal contraceptives have reduced risks of ovarian cysts, ovarian and endometrial cancer, benign breast disease, and pelvic inflammatory disease as well as a lower incidence of ectopic pregnancy, iron deficiency anemia, and rheumatoid arthritis. 3. Toxicity—The incidence of dose-dependent toxicity has fallen since the introduction of the low-dose combined oral contraceptives. a. Thromboembolism—The major toxic effects of the combined hormonal contraceptives relate to the action of the estrogenic component on blood coagulation. There is a welldocumented increase in the risk of thromboembolic events (myocardial infarction, stroke, deep vein thrombosis, pulmonary embolism) in older women, smokers, women with a personal or family history of such problems, and women with genetic defects that affect the production or function of clotting factors. However, the risk of thromboembolism incurred by the use of these drugs is usually less than that imposed by pregnancy. 332 PART VII Endocrine Drugs TABLE 40–1 Representative applications for the gonadal hormones and hormone antagonists. Clinical Application Drugs Hypogonadism in girls, women Conjugated estrogens, ethinyl estradiol, estradiol esters Hormone replacement therapy Estrogen component: conjugated estrogens, estradiol, estrone, estriol Progestin component: progesterone, medroxyprogesterone acetate Oral hormonal contraceptive Combined: ethinyl estradiol or mestranol plus a progestin Progestin only: norethindrone or norgestrel Parenteral contraceptive Medroxyprogesterone as a depot IM injection Ethinyl estradiol and norelgestromin as a weekly patch Ethinyl estradiol and etonogestrel as a monthly vaginal ring L-Norgestrel as an intrauterine device (IUD) Etonogestrel as a subcutaneous implant Postcoital contraceptive L-Norgestrel, Intractable dysmenorrhea or uterine bleeding Conjugated estrogens, ethinyl estradiol, oral contraceptive, GnRH agonist, depot injection of medroxyprogesterone acetate Infertility Clomiphene; hMG and hCG; GnRH analogs; progesterone; bromocriptine Abortifacient Mifepristone (RU 486) and misoprostol Endometriosis Oral contraceptive, depot injection of medroxyprogesterone acetate, GnRH agonist, danazol Breast cancer Tamoxifen, aromatase inhibitors (eg, anastrozole) Osteoporosis in postmenopausal women Conjugated estrogens, estradiol, raloxifene (see also Chapter 42) Hypogonadism in boys, men; replacement therapy Testosterone enanthate or cypionate, methyltestosterone, fluoxymesterone, testosterone (patch) Anabolic protein synthesis Oxandrolone, stanozolol Prostate hyperplasia (benign) Finasteride Prostate carcinoma GnRH agonist, GnRH receptor antagonist, androgen receptor antagonist (eg, flutamide) Hirsutism Combined oral contraceptive, spironolactone, flutamide, GnRH agonist combined oral contraceptive b. Breast cancer—Evidence suggests that the lifetime risk of breast cancer in women who are current or past users of hormonal contraceptives is not changed, but there may be an earlier onset of breast cancer. c. Other toxicities—The low-dose combined oral and progestinonly contraceptives cause significant breakthrough bleeding, especially during the first few months of therapy. Other toxicities of the hormonal contraceptives include nausea, breast tenderness, headache, skin pigmentation, and depression. Preparations containing older, more androgenic progestins can cause weight gain, acne, and hirsutism. The high dose of estrogen in estrogen-containing postcoital contraceptives is associated with significant nausea. ANTIESTROGENS & ANTIPROGESTINS A. Selective Estrogen Receptor Modulators Selective estrogen receptor modulators (SERMs) are mixed estrogen agonists that have estrogen agonist effects in some tissues and act as partial agonists or antagonists of estrogen in other tissues. SKILL KEEPER: CYTOCHROME P450 AND HORMONAL CONTRACEPTIVES (SEE CHAPTERS 4 AND 61) Hormonal contraceptives usually contain the lowest doses of the estrogen and progestin components that prevent pregnancy. The margin between effective and ineffective serum concentrations of the steroids is narrow, which presents a risk of breakthrough bleeding and also unintended pregnancy resulting from drug–drug interactions. Most steroidal contraceptives are metabolized by cytochrome P450 isozymes. 1. How many drugs can you identify that decrease the efficacy of hormonal contraceptives by increasing their metabolism? 2. When one of these drugs is prescribed for a woman who already is using a combined hormonal contraceptive, what should be done to prevent pregnancy? The Skill Keeper Answers appear at the end of the chapter. CHAPTER 40 Gonadal Hormones & Inhibitors 1. Tamoxifen—Tamoxifen is an SERM that is effective in the treatment of hormone-responsive breast cancer, where it acts as an antagonist to prevent receptor activation by endogenous estrogens (Figure 40–2). Prophylactic use of tamoxifen reduces the incidence of breast cancer in women who are at very high risk. As an agonist of endometrial receptors, tamoxifen promotes endometrial hyperplasia Hypothalamus GnRH – GnRH antagonists (1) 333 and increases the risk of endometrial cancer. The drug also causes hot flushes (an antagonist effect) and increases the risk of venous thrombosis (an agonist effect). Tamoxifen has more agonist than antagonist action on bone and thus prevents osteoporosis in postmenopausal women. Toremifene is structurally related to tamoxifen and has similar properties, indications, and toxicity. 2. Raloxifene—Raloxifene, approved for prevention and treatment of osteoporosis in postmenopausal women, has a partial agonist effect on bone. Like tamoxifen, raloxifene has antagonist effects in breast tissue and reduces the incidence of breast cancer in women who are at very high risk. Unlike tamoxifen, the drug has no estrogenic effects on endometrial tissue. Adverse effects include hot flushes (an antagonist effect) and an increased risk of venous thrombosis (an agonist effect). Bazedoxifene, a newer SERM, is approved for treatment of menopausal symptoms and prophylaxis of postmenopausal osteoporosis in combination with conjugated estrogens. GnRH agonists (2) +/– 3. Clomiphene—Clomiphene is a nonsteroidal compound with tissue-selective actions. It is used to induce ovulation in anovulatory women who wish to become pregnant. By selectively blocking estrogen receptors in the pituitary, clomiphene reduces negative feedback and increases FSH and LH output. The increase in gonadotropins stimulates ovulation. Pituitary gonadotrophs LH B. Pure Estrogen Receptor Antagonists Fulvestrant is a pure estrogen receptor antagonist (in all tissues). It is used in the treatment of women with breast cancer that has developed resistance to tamoxifen. Testis – Ketoconazole, (3) spironolactone Testosterone – 5αReductase Finasteride (4) Dihydrotestosterone – – Flutamide, cyproterone, (5) spironolactone Androgen-receptor complex Androgen response element Expression of appropriate genes in androgen-responsive cells FIGURE 40–2 Control of androgen secretion and activity and some sites of action of antiandrogens: (1) competitive inhibition of GnRH receptors (see Chapter 37); (2) stimulation (+) or inhibition (-) by GnRH agonists; (3) inhibition of testosterone synthesis; (4) inhibition of dihydrotestosterone production by finasteride; (5) inhibition of androgen binding at its receptor by flutamide and other drugs. (Reproduced, with permission, from Katzung BG, editor: Basic & Clinical Pharmacology, 12th ed. McGraw-Hill, 2012: Fig. 40–6.) C. Synthesis Inhibitors 1. Aromatase inhibitors—Anastrozole and related compounds (eg, letrozole) are nonsteroidal competitive inhibitors of aromatase, the enzyme required for the last step in estrogen synthesis. Exemestane is an irreversible aromatase inhibitor. These drugs are used in the treatment of breast cancer. 2. Danazol—Danazol inhibits several cytochrome P450 enzymes involved in gonadal steroid synthesis and is a weak partial agonist of progestin, androgen, and glucocorticoid receptors. The drug is sometimes used in the treatment of endometriosis and fibrocystic disease of the breast. D. Gonadotropin-Releasing Hormone Analogs and Antagonists As discussed in Chapter 37, the continuous administration of gonadotropin-releasing hormone (GnRH) agonists (eg, leuprolide) suppresses gonadotropin secretion and thereby inhibits ovarian production of estrogens and progesterone. The GnRH agonists are used in combination with other agents in controlled ovarian hyperstimulation (Chapter 37) and are also used for treatment of precocious puberty in children and short-term (<6 mo) treatment of endometriosis and uterine fibroids in women. Treatment beyond 6 mo in premenopausal women can result in decreased bone density. The GnRH receptor antagonists 334 PART VII Endocrine Drugs ganirelix and cetrorelix are used for controlled ovarian hyperstimulation (see Chapter 37). E. Antiprogestins Mifepristone (RU 486) is an orally active steroid antagonist of progesterone and glucocorticoids (Chapter 39). Its major use is as an abortifacient in early pregnancy (up to 49 days after the last menstrual period). The combination of mifepristone and the prostaglandin E analog misoprostol (Chapters 18 and 59) achieves a complete abortion in over 95% of early pregnancies. The most common complication is failure to induce a complete abortion. Side effects, which are primarily due to the misoprostol, include nausea, vomiting, and diarrhea plus the cramping and bleeding associated with passing the pregnancy. Rarely, patients who used mifepristone and misoprostol for medical abortion have experienced serious infection, sepsis, and even death due to unusual infection (eg, Clostridium sordellii ). ANDROGENS Testosterone and related androgens are produced in the testis, the adrenal, and, to a small extent, the ovary. Testosterone is synthesized from progesterone and dehydroepiandrosterone (DHEA). In the plasma, testosterone is partly bound to sex hormone-binding globulin (SHBG), a transport protein. The hormone is converted in several organs (eg, prostate) to dihydrotestosterone (DHT), which is the active hormone in those tissues. Because of rapid hepatic metabolism, testosterone given orally has little effect. It may be given by injection in the form of long-acting esters or transdermal patch. Orally active variants are also available (Table 40–1). Many androgens have been synthesized in an effort to increase the anabolic effect (see Effects, discussed later) without increasing androgenic action. Oxandrolone and stanozolol are examples of drugs that, in laboratory testing, have an increased ratio of anabolic-androgenic action. However, all the so-called anabolic steroids have full androgenic agonist effects when used in humans. and increased red blood cell production. Excretion of urea nitrogen is reduced, and nitrogen balance becomes more positive. Testosterone also helps maintain normal bone density. C. Clinical Use The primary clinical use of the androgens is for replacement therapy in hypogonadism (Table 40–1). Androgens have also been used to stimulate red blood cell production in certain anemias and to promote weight gain in patients with wasting syndromes (eg, AIDS patients). The anabolic effects have been exploited illicitly by athletes to increase muscle bulk and strength and perhaps enhance athletic performance. D. Toxicity Use of androgens by females results in virilization (hirsutism, enlarged clitoris, deepened voice) and menstrual irregularity. In women who are pregnant with a female fetus, exogenous androgens can cause virilization of the fetus’s external genitalia. Paradoxically, excessive doses in men can result in feminization (gynecomastia, testicular shrinkage, infertility) as a result of feedback inhibition of the pituitary and conversion of the exogenous androgens to estrogens. In both sexes, high doses of anabolic steroids can cause cholestatic jaundice, elevation of liver enzyme levels, and possibly hepatocellular carcinoma. ANTIANDROGENS Reduction of androgen effects is an important mode of therapy for both benign and malignant prostate disease, precocious puberty, hair loss, and hirsutism. Drugs are available that act at different sites in the androgen pathway (Figure 40–2). A. Mechanism of Action Like other steroid hormones, androgens enter cells and bind to cytosolic receptors. The hormone-receptor complex enters the nucleus and modulates the expression of target genes. A. Receptor Inhibitors Flutamide and related drugs bicalutamide, nilutamide, and enzalutamide are nonsteroidal competitive antagonists of androgen receptors. These drugs are used to decrease the action of endogenous androgens in patients with prostate carcinoma. Spironolactone, a drug used principally as a potassium-sparing diuretic (Chapter 15), also inhibits androgen receptors and is used in the treatment of hirsutism in women. B. Effects Testosterone is necessary for normal development of the male fetus and infant and is responsible for the major changes in the male at puberty (growth of penis, larynx, and skeleton; development of facial, pubic, and axillary hair; darkening of skin; enlargement of muscle mass). After puberty, testosterone acts to maintain secondary sex characteristics, fertility, and libido. It also acts on hair cells to cause male-pattern baldness. The major effect of androgenic hormones, in addition to development and maintenance of normal male characteristics, is an anabolic action that involves increased muscle size and strength B. 5`-Reductase Inhibitors Testosterone is converted to DHT by the enzyme 5α-reductase. Some tissues, most notably prostate cells and hair follicles, depend on DHT rather than testosterone for androgenic stimulation. This enzyme is inhibited by finasteride, a drug used to treat benign prostatic hyperplasia and, at a lower dose, to prevent hair loss in men. Because the drug does not interfere with the action of testosterone, it is less likely than other antiandrogens to cause impotence, infertility, and loss of libido. Dutasteride is a newer 5α-reductase inhibitor with a much longer half-life than that of finasteride. CHAPTER 40 Gonadal Hormones & Inhibitors C. Gonadotropin-Releasing Hormone Analogs and Antagonists Suppression of gonadotropin secretion, especially LH, reduces the production of testosterone. This can be effectively accomplished with long-acting depot preparations of leuprolide or similar gonadotropin-releasing hormone (GnRH) agonists (Chapter 37). These analogs are used in prostatic carcinoma. During the first week of therapy, an androgen receptor antagonist (eg, flutamide) is added to prevent the tumor flare that can result from the surge in testosterone synthesis caused by the initial agonistic action of the GnRH agonist. Within several weeks, testosterone production falls to low levels. As discussed in Chapter 37, the GnRH receptor antagonists abarelix and degarelix are approved for advanced prostate cancer. D. Combined Hormonal Contraceptives Combined hormonal contraceptives are used in women with androgen-induced hirsutism. The estrogen in the contraceptive acts in the liver to increase the production of sex hormonebinding globulin, which in turn reduces the concentration of the free androgen in the blood that is causing the male-pattern hair growth characteristic of hirsutism. E. Inhibitors of Steroid Synthesis Ketoconazole, an antifungal drug (Chapter 48), inhibits gonadal and adrenal steroid synthesis. The drug has been used to suppress adrenal steroid synthesis in patients with steroid-responsive metastatic prostate cancer. QUESTIONS 1. A teenager seeks postcoital contraception. Which of the following preparations will be effective for this purpose? (A) Clomiphene (B) Ethinyl estradiol (C) Diethylstilbestrol (DES) (D) Mifepristone (E) Norgestrel 2. A 23-year-old woman desires a combined oral contraceptive for pregnancy protection. Which of the following patient factors would lead a health professional to recommend an alternative form of contraception? (A) Evidence of hirsutism (B) History of gastroesophageal reflux disease and is currently taking omeprazole (C) History of pelvic inflammatory disease (D) History of migraine headache that is well controlled by sumatriptan (E) She plans to use this contraceptive for about 1 yr and will then attempt to become pregnant 3. Men who use large doses of anabolic steroids are at increased risk of which of the following? (A) Anemia (B) Cholestatic jaundice and elevation of aspartate transaminase levels in the blood (C) Hirsutism (D) Hyperprolactinemia (E) Testicular enlargement 335 4. A 50-year-old woman with a positive mammogram undergoes lumpectomy and a small carcinoma is removed. Biochemical analysis of the cancer reveals the presence of estrogen and progesterone receptors. After this procedure, she will probably receive which of the following drugs? (A) Danazol (B) Flutamide (C) Leuprolide (D) Mifepristone (E) Tamoxifen 5. A 60-year-old man is found to have a prostate lump and an elevated prostate-specific antigen (PSA) blood test. Magnetic resonance imaging suggests several enlarged lymph nodes in the lower abdomen, and an x-ray reveals 2 radiolucent lesions in the bony pelvis. This patient is likely to be treated with which of the following drugs? (A) Anastrozole (B) Desogestrel (C) Leuprolide (D) Methyltestosterone (E) Oxandrolone 6. A young woman complains of abdominal pain at the time of menstruation. Careful evaluation indicates the presence of significant endometrial deposits on the pelvic peritoneum. Which of the following is the most appropriate medical therapy for this patient? (A) Flutamide, orally (B) Medroxyprogesterone acetate by intramuscular injection (C) Norgestrel as an IUD (D) Oxandrolone by intramuscular injection (E) Raloxifene orally 7. Diethylstilbestrol (DES) should never be used in pregnant women because it is associated with which of the following? (A) Deep vein thrombosis (B) Feminization of the external genitalia of male offspring (C) Infertility and development of vaginal cancer in female offspring (D) Miscarriages (E) Virilization of the external genitalia of female offspring 8. Which of the following is a unique property of SERMs? (A) Act as agonists in some tissues and antagonists in other tissues (B) Activate a unique plasma membrane-bound receptor (C) Have both estrogenic and progestational agonist activity (D) Inhibit the aromatase enzyme required for estrogen synthesis (E) Produce estrogenic effects without binding to estrogen receptors 9. Finasteride has efficacy in the prevention of male-pattern baldness by virtue of its ability to do which of the following? (A) Competitively antagonize androgen receptors (B) Decrease the release of gonadotropins (C) Increase the serum concentration of sex hormone-binding globulin (D) Inhibit the synthesis of testosterone (E) Reduce the production of dihydrotestosterone 336 PART VII Endocrine Drugs 10. A 52-year-old postmenopausal patient has evidence of low bone mineral density. She and her physician are considering therapy with raloxifene or a combination of conjugated estrogens and medroxyprogesterone acetate. Which of the following patient characteristics is most likely to lead them to select raloxifene? (A) Previous hysterectomy (B) Recurrent vaginitis (C) Rheumatoid arthritis (D) Strong family history of breast cancer (E) Troublesome hot flushes ANSWERS 1. Mifepristone, an antagonist at progesterone and glucocorticoid receptors, has a luteolytic effect and is effective as a postcoital contraceptive. When combined with a prostaglandin, it is also an effective abortifacient. The answer is D. 2. Estrogen-containing hormonal contraceptives increase the risk of episodes of migraine headache. The answer is D. 3. In men, large doses of anabolic steroids are associated with liver impairment, including cholestasis and elevation of serum concentrations of transaminases. The answer is B. 4. Tamoxifen has proved useful in adjunctive therapy of breast cancer; the drug decreases the rate of recurrence of cancer. The answer is E. 5. Leuprolide is a GnRH agonist used in the treatment of men with prostate cancer. Continuous use leads to downregulation of testosterone production. Initially, the agonist action increases testosterone, causing a tumor flare. To prevent this, flutamide, a competitive antagonist of the androgen receptor, is added until downregulation of testosterone is complete. The answer is C. 6. In endometriosis, suppression of ovarian function and production of gonadal steroids are useful. Intramuscular injection of relatively large doses of medroxyprogesterone provides 3 months of an ovarian suppressive effect because of inhibition of pituitary production of gonadotropins. The answer is B. 7. Diethylstilbestrol (DES) is a nonsteroidal estrogen agonist. Several decades ago, misguided use of the drug in pregnant women appears to have resulted in fetal damage that predisposed female offspring to infertility and a rare form of vaginal cancer. For this reason, the drug should be avoided in pregnant women. Other estrogenic drugs do not appear to have these effects. Although estrogens do increase the risk of deep vein thrombosis, this is not the reason why DES should be avoided. The answer is C. 8. SERMs such as tamoxifen and raloxifene exhibit tissuespecific estrogenic and antiestrogenic effects. The answer is A. 9. Finasteride inhibits 5α-reductase, the enzyme that converts testosterone to DHT, the principal androgen in androgensensitive hair follicles. The answer is E. 10. Conjugated estrogens and raloxifene both improve bone mineral density and protect against osteoporosis. The 2 advantages of raloxifene over full estrogen receptor agonists are that raloxifene has antagonist effects in breast tissue and lacks an agonistic effect in endometrium. If a patient’s uterus was removed by surgery, the difference in the endometrial effect is moot. In patients with a strong family history of breast cancer, raloxifene may be a better choice than a full estrogen agonist because it will not further increase the woman’s risk of breast cancer and may even lower her risk. The answer is D. SKILL KEEPER ANSWERS: CYTOCHROME P450 AND HORMONAL CONTRACEPTIVES (SEE CHAPTERS 4 AND 61) 1. Gonadal steroids and their derivatives are metabolized primarily by the cytochrome P450 3A4 (CYP3A4) family of enzymes. Inducers of CYP3A4 include barbiturates, carbamazepine, corticosteroids, griseofulvin, phenytoin, pioglitazone, rifampin, and rifabutin. The potential reduction in contraceptive efficacy of hormonal contraceptives by carbamazepine and phenytoin are of particular importance because these drugs are known teratogens. St. John’s wort, an unregulated herbal product, contains an ingredient that induces CYP3A4 enzymes and can reduce the efficacy of hormonal contraceptives. 2. To prevent an unwanted pregnancy, it would be advisable to use a combined hormonal contraceptive pill with a higher dose of estrogen (eg, a formulation containing 50 mcg of ethinyl estradiol). Alternatively, or additionally, women may use a barrier form of contraception or switch to an IUD. CHAPTER 40 Gonadal Hormones & Inhibitors 337 CHECKLIST When you complete this chapter, you should be able to: ❑ Describe the hormonal changes that occur during the menstrual cycle. ❑ Name 3 estrogens and 4 progestins. Describe their pharmacologic effects, clinical uses, and toxicity. ❑ List the benefits and hazards of hormonal contraceptives. ❑ List the benefits and hazards of postmenopausal estrogen therapy. ❑ Describe the use of gonadal hormones and their antagonists in the treatment of cancer in women and men. ❑ List or describe the toxic effects of anabolic steroids used to build muscle mass. ❑ Name 2 SERMs and describe their unique properties. DRUG SUMMARY TABLE: Gonadal Hormones & Inhibitors Subclass Mechanism of Action Clinical Applications Activation of estrogen receptors leads to changes in the rates of transcription of estrogen-regulated genes See Table 40–1 Pharmacokinetics Toxicities, Drug Interactions Oral, parenteral, or transdermal administration • metabolism relies on cytochrome P450 systems • enterohepatic recirculation occurs Moderate toxicity: Breakthrough bleeding, nausea, breast tenderness Estrogens Ethinyl estradiol Serious toxicity: Thromboembolism, gallbladder disease, hypertriglyceridemia, migraine headache, hypertension, depression In postmenopausal women: breast cancer, endometrial hyperplasia (unopposed estrogen) Combination with cytochrome P450 inducer can lead to breakthrough bleeding and reduced contraceptive efficacy Mestranol: a prodrug that is converted to ethinyl estradiol, contained in some contraceptives Estrogen esters (eg, estradiol cypionate): long-acting estrogens administered IM and used for hypogonadism in young females Progestins Norgestrel Activation of progesterone receptors leads to changes in the rates of transcription of progesterone-regulated genes See Table 40–1 Oral, parenteral, or transdermal administration • metabolism relies on cytochrome P450 systems • enterohepatic recirculation occurs Weight gain, reversible decrease in bone mineral density (high doses) Progesterone derivatives: medroxyprogesterone acetate, megestrol acetate Older 19-nortestosterone derivatives: norethindrone, ethynodiol Newer 19-nortestosterone derivatives: desogestrel, norelgestromin, norgestimate, etonogestrel Spironolactone derivative: drospirenone (Continued ) 338 PART VII Endocrine Drugs DRUG SUMMARY TABLE: Gonadal Hormones & Inhibitors (Continued ) Subclass Mechanism of Action Clinical Applications Estrogen antagonist actions in breast tissue and CNS • estrogen agonist effects in liver and bone Prevention and adjuvant treatment of hormone-responsive breast cancer Pharmacokinetics Toxicities, Drug Interactions Oral administration Hot flushes, thromboembolism, endometrial hyperplasia Antiestrogens SERMS Tamoxifen Toremifene: similar to tamoxifen Raloxifene: approved for osteoporosis and prevention of breast cancer in selected patients; antagonist effects in breast, CNS, and endometrium and agonist effects in the liver. Bazedoxifene: approved for treatment of menopausal symptoms and prophylaxis of postmenopausal osteoporosis in combination with conjugated estrogens. Clomiphene: used for ovulation induction; antagonist effect in pituitary increases gonadotropin secretion Receptor antagonist Fulvestrant Estrogen receptor antagonist in all tissues Adjuvant treatment of hormone-responsive breast cancer that is resistant to first-line antiestrogen therapy Intramuscular administration Hot flushes, headache, injection site reactions Reduces estrogen synthesis by inhibiting aromatase enzyme Adjuvant treatment of hormone-responsive breast cancer Oral administration Hot flushes, musculoskeletal disorders, reduced bone mineral density Aromatase inhibitors Anastrozole Joint symptoms (arthralgia, arthrosis, arthritis, cervical spondylosis, osteoarthritis, and disk herniation) Letrozole: similar to anastrozole Exemestane: irreversible aromatase inhibitor GnRH agonist Leuprolide See Chapter 37 GnRH receptor antagonist Ganirelix, cetrorelix See Chapter 37 Other Danazol Weak cytochrome P450 inhibitor and partial agonist of progestin and androgen receptors Endometriosis, fibrocystic breast disease Oral administration • drug interactions due to cytochrome P450 inhibition Acne, hirsutism, weight gain, menstrual disturbances, hepatic dysfunction Progestin and glucocorticoid receptor antagonist Used in combination with a prostaglandin (eg, misoprostol) for medical abortion Oral administration Gastrointestinal disturbances (mostly due to coadministration of misoprostol) • vaginal bleeding, atypical infection Antiprogestin Mifepristone (Continued ) CHAPTER 40 Gonadal Hormones & Inhibitors 339 DRUG SUMMARY TABLE: Gonadal Hormones & Inhibitors (Continued ) Subclass Mechanism of Action Clinical Applications Androgen receptor agonist Male hypogonadism • weight gain in patients with wasting syndromes Pharmacokinetics Toxicities, Drug Interactions Transdermal, buccal, subcutaneous implant In females, virilization • In men, high doses can cause gynecomastia, testicular shrinkage, infertility Androgens Testosterone Fluoxymesterone, methyltestosterone: oral androgens Testosterone esters (eg, testosterone cypionate): long-acting androgens for parenteral administration Anabolic steroids (eg, oxandrolone, nandrolone decanoate): increased ratio of anabolic-to-androgenic activity in laboratory animals, cholestatic jaundice, liver toxicity Antiandrogens 5α-reductase inhibitors Finasteride Inhibition of 5α-reductase enzyme that converts testosterone to dihydrotestosterone Benign prostatic hyperplasia (BPH), male-pattern hair loss Oral administration Rarely, impotence, gynecomastia Advanced prostate cancer Oral administration Gynecomastia, hot flushes, impotence, hepatoxicity Dutasteride: similar to finasteride Receptor antagonists Flutamide Competitive inhibition of androgen receptor Bicalutamide, nilutamide: similar to flutamide but lower risk of hepatotoxicity Spironolactone: mineralocorticoid receptor antagonist used mainly as a potassium-sparing diuretic (see Chapter 15); also has androgen-receptor antagonist activity, used for the treatment of hirsutism GnRH agonist Leuprolide See Chapter 37 GnRH receptor antagonist Abarelix, degarelix See Chapter 37 Synthesis inhibitor Ketoconazole (see Chapter 48) Inhibition of cytochrome P450 enzymes involved in androgen synthesis Advanced prostate cancer that is resistant to first-line antiandrogen drugs Oral administration Interferes with synthesis of other steroids • many drug interactions due to cytochrome P450 inhibition C A P T E R 41 Pancreatic Hormones, Antidiabetic Agents, & Glucagon In the endocrine pancreas, the islets of Langerhans contain at least 4 types of endocrine cells, including A (alpha, glucagon producing), B (beta, insulin, and amylin producing), D (delta, somatostatin producing), and F (pancreatic polypeptide producing). Of these, the B (insulin-producing) cells are the most numerous. H The most common pancreatic disease requiring pharmacologic therapy is diabetes mellitus, a deficiency of insulin production or effect. Diabetes is treated with several parenteral formulations of insulin and oral or parenteral noninsulin antidiabetic agents. Glucagon, a hormone that affects the liver, cardiovascular system, and gastrointestinal tract, can be used to treat severe hypoglycemia. Drugs for diabetes mellitus Insulins Rapid, shortacting (lispro, regular) Insulin secretagogues (glipizide) Noninsulin antidiabetic drugs Intermediateacting (NPH, lente) Biguanides (metformin) Slow, longacting (glargine) Alpha-glucosidase inhibitors (acarbose) Thiazolidinediones (pioglitazone) Amylin analogs (pramlintide) Incretin modulators GLP-1 analog (exenatide) DIABETES MELLITUS Diabetes mellitus is classified into four categories: type 1, type 2, other, and gestational diabetes mellitus. Here, we focus on type 1 and type 2. Type 1 diabetes usually has its onset during childhood 340 SGLT2 inhibitors (canagliflozin) DPP-4 inhibitor (sitagliptin) and results from autoimmune destruction of pancreatic B cells. Type 2 diabetes is a progressive disorder characterized by increasing insulin resistance and diminishing insulin secretory capacity. Type 2 diabetes is frequently associated with obesity and is much more common than type 1 diabetes. Although type 2 diabetes CHAPTER 41 Pancreatic Hormones, Antidiabetic Agents, & Glucagon 341 High-Yield Terms to Learn Alpha-glucosidase An enzyme in the gastrointestinal tract that converts complex starches and oligosaccharides to monosaccharides; inhibited by acarbose and miglitol Beta (B) cells in the islets of Langerhans Insulin-producing cells in the endocrine pancreas Hypoglycemia Dangerously lowered serum glucose concentration; a toxic effect of high insulin concentrations and the secretagogue class of oral antidiabetic drugs Lactic acidosis Acidemia due to excess serum lactic acid; can result from excess production or decreased metabolism of lactic acid Type 1 diabetes mellitus A form of chronic hyperglycemia caused by immunologic destruction of pancreatic beta cells Type 2 diabetes mellitus A form of chronic hyperglycemia initially caused by resistance to insulin; often progresses to insulin deficiency usually has its onset in adulthood, the incidence in children and adolescents is rising dramatically, in parallel with the increase in obesity in children and adolescents. The clinical history and course of these 2 forms differ considerably, but treatment in both cases requires careful attention to diet, fasting and postprandial blood glucose concentrations, and serum concentrations of hemoglobin A1c, a glycosylated hemoglobin that serves as a marker of glycemia. Type 1 diabetes requires treatment with insulin. The early stages of type 2 diabetes usually can be controlled with noninsulin antidiabetic drugs. However, patients in the later stages of type 2 diabetes often require the addition of insulin to their drug regimen. INSULIN A. Physiology Insulin is synthesized as the prohormone proinsulin, an 86-amino-acid single-chain polypeptide. Cleavage of proinsulin and cross-linking result in the 2-chain 51-peptide insulin molecule and a 31-amino-acid residual C-peptide. Neither proinsulin nor C-peptide appears to have any physiologic actions. B. Effects Insulin has important effects on almost every tissue of the body. When activated by the hormone, the insulin receptor, a transmembrane tyrosine kinase, phosphorylates itself and a variety of intracellular proteins when activated by the hormone. The major target organs for insulin action include: 1. Liver—Insulin increases the storage of glucose as glycogen in the liver. This involves the insertion of additional GLUT2 glucose transport molecules in cell plasma membranes; increased synthesis of the enzymes pyruvate kinase, phosphofructokinase, and glucokinase; and suppression of several other enzymes. Insulin also decreases protein catabolism. 2. Skeletal muscle—Insulin stimulates glycogen synthesis and protein synthesis. Glucose transport into muscle cells is facilitated by insertion of GLUT4 transporters into cell plasma membranes. 3. Adipose tissue—Insulin facilitates triglyceride storage by activating plasma lipoprotein lipase, increasing glucose transport into cells via GLUT4 transporters, and reducing intracellular lipolysis. C. Insulin Preparations Human insulin is manufactured by bacterial recombinant DNA technology. The available forms provide 4 rates of onset and durations of effect that range from rapid-acting to long-acting (Figure 41–1). The goals of insulin therapy are to control both basal and postprandial (after a meal) glucose levels while minimizing the risk of hypoglycemia. Insulin formulations with different rates of onset and effect are often combined to achieve these goals. 1. Rapid-acting—Three insulin analogs (insulin lispro, insulin aspart, and insulin glulisine) have rapid onsets and early peaks of activity (Figure 41–1) that permit control of postprandial glucose levels. The 3 rapid-acting insulins have small alterations in their primary amino acid sequences that speed their entry into the circulation without affecting their interaction with the insulin receptor. The rapid-acting insulins are injected immediately before a meal and are the preferred insulin for continuous subcutaneous infusion devices. They also can be used for emergency treatment of uncomplicated diabetic ketoacidosis. 2. Short-acting—Regular insulin is used intravenously in emergencies or administered subcutaneously in ordinary maintenance regimens, alone or mixed with intermediate- or long-acting preparations. Before the development of rapid-acting insulins, it was the primary form of insulin used for controlling postprandial glucose concentrations, but it requires administration 1 h or more before a meal. 3. Intermediate-acting—Neutral protamine Hagedorn insulin (NPH insulin) is a combination of regular insulin and protamine (a highly basic protein also used to reverse the action of unfractionated heparin, Chapter 34) that exhibits a delayed onset and peak of action (Figure 41–1). NPH insulin is often combined with regular and rapid-acting insulins. 342 PART VII Endocrine Drugs 8 Glucose infusion rate (mg/kg/min) Insulin lispro, aspart, glulisine 7 6 Regular 5 NPH 4 3 2 Insulin detemir Insulin glargine 1 1 2 3 4 5 6 7 8 9 10 11 12 13 14 15 16 17 18 19 20 21 22 23 24 Time (h) FIGURE 41–1 Extent and duration of action of various types of insulin as indicated by the glucose infusion rates (mg/kg/min) required to maintain a constant glucose concentration. The durations of action shown are typical of an average dose of 0.2–0.3 U/kg; the duration of regular and NPH insulin increases considerably when dosage is increased. (Reproduced, with permission, from Katzung BG, editor: Basic & Clinical Pharmacology, 12th ed. McGraw-Hill, 2012: Fig. 41–5.) 4. Long-acting—Insulin glargine and insulin detemir are modified forms of human insulin that provide a peakless basal insulin level lasting more than 20 h, which helps control basal glucose levels without producing hypoglycemia. 5. Insulin delivery systems—The standard mode of insulin therapy is subcutaneous injection with conventional disposable needles and syringes. More convenient means of administration are also available. Portable pen-sized injectors are used to facilitate subcutaneous injection. Some contain replaceable cartridges, whereas others are disposable. Continuous subcutaneous insulin infusion devices avoid the need for multiple daily injections and provide flexibility in the scheduling of patients’ daily activities. Programmable pumps deliver a constant 24-h basal rate, and manual adjustments in the rate of delivery can be made to accommodate changes in insulin requirements (eg, before meals or exercise). D. Hazards of Insulin Use The most common complication is hypoglycemia, resulting from excessive insulin effect. To prevent the brain damage that may result from hypoglycemia, prompt administration of glucose (sugar or candy by mouth, glucose by vein) or of glucagon (by intramuscular injection) is essential. Patients with advanced renal disease, the elderly, and children younger than 7 years are most susceptible to the detrimental effects of hypoglycemia. The most common form of insulin-induced immunologic complication is the formation of antibodies to insulin or noninsulin protein contaminants, which results in resistance to the action of the drug or allergic reactions. With the current use of highly purified human insulins, immunologic complications are uncommon. NONINSULIN ANTIDIABETIC DRUGS Four well-established groups of oral antidiabetic drugs are used most commonly to treat type 2 diabetes. These include insulin secretagogues (Figure 41–2), and the biguanide metformin, thiazolidinediones, and `-glucosidase inhibitors (Figure 41–3). Three novel agents—pramlintide, exenatide, and sitagliptin—target endogenous regulators of glucose homeostasis. The durations of action of important members of these groups are listed in Table 41–1. A. Insulin Secretagogues 1. Mechanism and effects—Insulin secretagogues stimulate the release of endogenous insulin by promoting closure of potassium channels in the pancreatic B-cell membrane (Figure 41–2). Channel closure depolarizes the cell and triggers insulin release. Insulin secretagogues are not effective in patients who lack functional pancreatic B cells. Most insulin secretagogues are in the chemical class known as sulfonylureas. The second-generation sulfonylureas (glyburide, glipizide, glimepiride) are considerably more potent and used more commonly than the older agents (tolbutamide, chlorpropamide, others). Repaglinide, a meglitinide, and nateglinide, a d-phenylalanine derivative, are also insulin secretagogues. Both have a rapid onset and short duration of action that make them useful for administration just before a meal to control postprandial glucose levels. 2. Toxicities—The insulin secretagogues, especially those with a high potency (eg, glyburide and glipizide), can precipitate hypoglycemia, although the risk is less than that associated with the insulins. The older sulfonylureas (tolbutamide and chlorpropamide) are extensively bound to serum proteins, and drugs that compete for CHAPTER 41 Pancreatic Hormones, Antidiabetic Agents, & Glucagon TABLE 41–1 Duration of action of representative oral antidiabetic drugs. Drug Duration of Action (hours) Secretagogues Chlorpropamide Tolbutamide Glimepiride Glipizide Glyburide Repaglinide Nateglinide Up to 60 6–12 12–24 10–24 10–24 4–5 4 Biguanides Metformin 10–12 Thiazolidinediones Pioglitazone Rosiglitazone 15–24 >24 Alpha-glucosidase inhibitors Acarbose Miglitol 3–4 3–4 Incretin modifiers Sitagliptin 8–14 SGLT2 inhibitors Canagliflozin 10–14 343 B. Biguanides 1. Mechanism and effects—Metformin, the primary member of the biguanide group, reduces postprandial and fasting glucose levels. Biguanides inhibit hepatic and renal gluconeogenesis (Figure 41–3). Other effects include stimulation of glucose uptake and glycolysis in peripheral tissues, slowing of glucose absorption from the gastrointestinal tract, and reduction of plasma glucagon levels. The molecular mechanism of biguanide reduction in hepatic glucose production appears to involve activation of an AMP-stimulated protein kinase. In patients with insulin resistance, metformin reduces endogenous insulin production presumably through enhanced insulin sensitivity. Because of this insulin-sparing effect and because it does not increase weight—unlike insulin, secretagogues, or the thiazolidinediones— metformin is increasingly the drug of first choice in overweight patients with type 2 diabetes. Recent clinical trials suggest that metformin reduces the risk of diabetes in high-risk patients. Metformin is also used to restore fertility in anovulatory women with polycystic ovary disease (PCOD) and evidence of insulin resistance. 2. Toxicities—Unlike the sulfonylureas, the biguanides do not cause hypoglycemia. Their most common toxicity is gastrointestinal distress (nausea, diarrhea), and they can cause lactic acidosis, especially in patients with renal or liver disease, alcoholism, or conditions that predispose to tissue anoxia and lactic acid production (eg, chronic cardiopulmonary dysfunction). protein binding may enhance their hypoglycemic effects. Occasionally these drugs cause rash or other allergic reactions. Weight gain is common and is especially undesirable in the large fraction of patients with type 2 diabetes who already are overweight. C. Thiazolidinediones 1. Mechanism and effects—The thiazolidinediones, rosiglitazone and pioglitazone, increase target tissue sensitivity to insulin K+ channel – Sulfonylurea drugs (block, depolarize) K+ (Closes, depolarizes) Glucose transporter ATP GLUT2 Glucose – Metabolism Ca 2+ Ca2+ channel (depolarization + opens) Ca2+ Insulin Exocytosis Insulin FIGURE 41–2 Control of insulin release from the pancreatic beta cell by glucose and by sulfonylurea drugs. When the extracellular glucose concentration increases, more glucose enters the cell via the GLUT2 glucose transporter and leads, through metabolism, to increased intracellular ATP production with subsequent closure of ATP-dependent K+ channels, membrane depolarization, opening of voltage-gated Ca2+ channels, increased intracellular Ca2+, and insulin secretion. Sulfonylurea and other insulin secretagogues enhance insulin release by blocking ATP-dependent K+ channels and thereby triggering the events subsequent to reduced K+ influx. (Reproduced, with permission, from Katzung BG, editor: Basic & Clinical Pharmacology, 12th ed. McGraw-Hill, 2012: Fig. 41–2.) 344 PART VII Endocrine Drugs Intestine Alpha-glucosidase inhibitors Dietary starch & sugar – GLP-1 (exenatide) Glucose Liver – Endocrine pancreas DPP-IV – + Glucose Insulin – DPP‐IV inhibitor (sitagliptin) + Metformin Secretagogues Glucose Adipocytes Skeletal Muscle Blood – + + Dapagliflozin SGLT-2 inhibitor Thiazolidinediones FIGURE 41–3 Urine Thiazolidinediones Major actions of the principal oral antidiabetic drugs used to treat type 2 diabetes. by activating the peroxisome proliferator-activated receptor-gamma nuclear receptor (PPAR-f receptor). This nuclear receptor regulates the transcription of genes encoding proteins involved in carbohydrate and lipid metabolism. A primary effect of the thiazolidinediones is increasing glucose uptake in muscle and adipose tissue (Figure 41–3). They also inhibit hepatic gluconeogenesis and have effects on lipid metabolism and the distribution of body fat. Thiazolidinediones reduce both fasting and postprandial hyperglycemia. They are used as monotherapy or in combination with insulin or other oral antidiabetic drugs. Like metformin, the thiazolidinediones have been shown to reduce the risk of diabetes in high-risk patients. 2. Toxicities—When these drugs are used alone, hypoglycemia is extremely rare. Thiazolidinediones can cause fluid retention, which presents as mild anemia and edema and may increase the risk of heart failure. Recent data have linked rosiglitazone to increased risk of myocardial infarction. The original thiazolidinedione (troglitazone) was removed from the market in several countries because of hepatotoxicity. Rosiglitazone and pioglitazone have not been linked to serious liver dysfunction but still require routine monitoring of liver function. Female patients taking thiazolidinediones appear to have an increased risk of bone fractures. Pioglitazone and troglitazone induce cytochrome P450 activity (especially the CYP3A4 isozyme) and can reduce the serum concentrations of drugs that are metabolized by these enzymes (eg, oral contraceptives, cyclosporine). D. Alpha-Glucosidase Inhibitors 1. Mechanism and effects—Acarbose and miglitol are carbohydrate analogs that act within the intestine to inhibit `-glucosidase, an enzyme necessary for the conversion of complex starches, oligosaccharides, and disaccharides to the monosaccharides that can be transported out of the intestinal lumen and into the bloodstream. As a result of slowed absorption, postprandial hyperglycemia is reduced. These drugs lack an effect on fasting blood sugar. Both drugs can be used as monotherapy or in combination with other antidiabetic drugs. They are taken just before a meal. Like metformin and the thiazolidinediones, the α-glucosidase inhibitors have been shown to prevent type 2 diabetes in prediabetic persons. 2. Toxicities—The primary adverse effects of the α-glucosidase inhibitors include flatulence, diarrhea, and abdominal pain resulting from increased fermentation of unabsorbed carbohydrate by bacteria in the colon. Patients taking an α-glucosidase inhibitor who experience hypoglycemia should be treated with oral glucose (dextrose) and not sucrose, because the absorption of sucrose will be delayed. E. Pramlintide Pramlintide is an injectable synthetic analog of amylin, a 37-amino acid hormone produced by pancreatic B cells. Amylin contributes to glycemic control by activating high-affinity receptors involved in both glycemic control and osteogenesis. Pramlintide suppresses glucagon release, slows gastric emptying, and works in the CNS to reduce appetite. After subcutaneous injection, it is rapidly absorbed and has a short duration of action. It is used in combination with insulin to control postprandial glucose levels. The major adverse effects associated with pramlintide are hypoglycemia and gastrointestinal disturbances. F. Exenatide Glucagon-like peptide-1 (GLP-1) is a member of the incretin family of peptide hormones, which are released from endocrine CHAPTER 41 Pancreatic Hormones, Antidiabetic Agents, & Glucagon cells in the epithelium of the bowel in response to food. The incretins augment glucose-stimulated insulin release from pancreatic B cells, retard gastric emptying, inhibit glucagon secretion, and produce a feeling of satiety. The GLP-1 receptor is a G proteincoupled receptor (GPCR) that increases cAMP and also increases the free intracellular concentration of calcium. Exenatide, a long-acting injectable peptide analog of GLP-1, is used in combination with metformin or a sulfonylurea for treatment of type 2 diabetes. The major adverse effects are gastrointestinal disturbances, particularly nausea during initial therapy, and hypoglycemia when exenatide is combined with a sulfonylurea. The drug has also caused serious and sometimes fatal acute pancreatitis. G. Sitagliptin Sitagliptin is an oral inhibitor of dipeptidyl peptidase-4 (DPP-4), the enzyme that degrades GLP-1 and other incretins. It is approved for use in type 2 diabetes as monotherapy or in combination with metformin or a thiazolidinedione. Like exenatide, sitagliptin promotes insulin release, inhibits glucagon secretion, and has an anorexic effect. The most common adverse effects associated with sitagliptin are headache, nasopharyngitis, and upper respiratory tract infection. H. Canagliflozin The sodium-glucose transporter 2 (SGLT2) accounts for 90% of renal glucose reabsorption, and its inhibition causes glycosuria and lowers glucose levels in patients with type 2 diabetes. The SGLT2 inhibitors canagliflozin and dapagliflozin are approved for clinical use. The main adverse effects are increased incidence of genital infections and urinary tract infections. The osmotic diuresis can also cause intravascular volume contraction and hypotension. TREATMENT OF DIABETES MELLITUS A. Type 1 Diabetes Therapy for type 1 diabetes involves dietary instruction, parenteral insulin (a mixture of shorter and longer acting forms to maintain control of basal and postprandial glucose levels) and possibly pramlintide for improved control of postprandial glucose levels, plus careful attention by the patient to factors that change insulin requirements: exercise, infections, other forms of stress, and deviations from the regular diet. Large clinical studies indicate that tight control of blood sugar, by frequent blood sugar testing and insulin injections, reduces the incidence of vascular complications, including renal and retinal damage. The risk of hypoglycemic reactions is increased in tight control regimens but not enough to obviate the benefits of better control. B. Type 2 Diabetes Because type 2 diabetes is usually a progressive disease, therapy for an individual patient generally escalates over time. It begins with weight reduction and dietary control. Initial drug therapy usually is oral monotherapy with metformin. Although initial responses 345 to monotherapy usually are good, secondary failure within 5 yr is common. Increasingly, noninsulin antidiabetic agents are being used in combination with each other or with insulin to achieve better glycemic control and minimize toxicity. Because type 2 diabetes often involves both insulin resistance and inadequate insulin production, it may be necessary to combine an agent that augments insulin’s action (metformin, a thiazolidinedione, or an α-glucosidase inhibitor) with one that augments the insulin supplies (insulin secretagogue or insulin). Long-acting drugs (sulfonylureas, metformin, thiazolidinediones, exenatide, sitagliptin, some insulin formulations) help control both fasting and postprandial blood glucose levels, whereas short-acting drugs (α-glucosidase inhibitors, repaglinide, pramlintide, rapid-acting insulins) primarily target postprandial levels. As is the case for type 1 diabetes, clinical trials have shown that tight control of blood glucose in patients with type 2 diabetes reduces the risk of vascular complications. SKILL KEEPER: DIABETES AND HYPERTENSION (SEE CHAPTER 11) Diabetes is linked to hypertension in several important ways. Obesity predisposes patients to hypertension as well as to type 2 diabetes, so many patients suffer from both diseases. Both diseases damage the kidney and predispose patients to coronary artery disease. A large clinical trial of patients with type 2 diabetes suggests that poorly controlled hypertension exacerbates the microvascular disease caused by long-standing diabetes. Because of these links, it is important to consider the treatment of hypertension in diabetic patients. 1. Identify the major drug groups used for chronic treatment of essential hypertension. 2. Which of these drug groups have special implications for the treatment of patients with diabetes? The Skill Keeper Answers appear at the end of the chapter. HYPERGLYCEMIC DRUGS: GLUCAGON A. Glucagon 1. Chemistry, mechanism, and effects—Glucagon is a protein hormone secreted by the A cells of the endocrine pancreas. Acting through G protein-coupled receptors in heart, smooth muscle, and liver, glucagon increases heart rate and force of contraction, increases hepatic glycogenolysis and gluconeogenesis, and relaxes smooth muscle. The smooth muscle effect is particularly marked in the gut. 2. Clinical uses—Glucagon is used to treat severe hypoglycemia in diabetics, but its hyperglycemic action requires intact hepatic glycogen stores. The drug is given intramuscularly or intravenously. In the management of severe β-blocker overdose, glucagon may be the most effective method for stimulating the depressed heart because it increases cardiac cAMP without requiring access to β receptors (Chapter 58). 346 PART VII Endocrine Drugs QUESTIONS Questions 1 and 2. A 13-year-old boy with type 1 diabetes is brought to the hospital complaining of dizziness. Laboratory findings include severe hyperglycemia, ketoacidosis, and a blood pH of 7.15. 1. Which of the following agents should be administered to achieve rapid control of the severe ketoacidosis in this diabetic boy? (A) Crystalline zinc insulin (B) Glyburide (C) Insulin glargine (D) NPH insulin (E) Tolbutamide 2. Which of the following is the most likely complication of insulin therapy in this patient? (A) Dilutional hyponatremia (B) Hypoglycemia (C) Increased bleeding tendency (D) Pancreatitis (E) Severe hypertension 3. A 24-year-old woman with type 1 diabetes wishes to try tight control of her diabetes to improve her long-term prognosis. Which of the following regimens is most appropriate? (A) Morning injections of mixed insulin lispro and insulin aspart (B) Evening injections of mixed regular insulin and insulin glargine (C) Morning and evening injections of regular insulin, supplemented by small amounts of NPH insulin at mealtimes (D) Morning injections of insulin glargine, supplemented by small amounts of insulin lispro at mealtimes (E) Morning injection of NPH insulin and evening injection of regular insulin 4. Which one of the following drugs promotes the release of endogenous insulin? (A) Acarbose (B) Canagliflozin (C) Glipizide (D) Metformin (E) Miglitol (F) Pioglitazone 5. Which of the following is an important effect of insulin? (A) Increased conversion of amino acids into glucose (B) Increased gluconeogenesis (C) Increased glucose transport into cells (D) Inhibition of lipoprotein lipase (E) Stimulation of glycogenolysis 6. A 54-year-old obese patient with type 2 diabetes has a history of alcoholism. In this patient, metformin should either be avoided or used with extreme caution because the combination of metformin and ethanol increases the risk of which of the following? (A) A disulfiram-like reaction (B) Excessive weight gain (C) Hypoglycemia (D) Lactic acidosis (E) Serious hepatotoxicity 7. Which of the following drugs is taken during the first part of a meal for the purpose of delaying the absorption of dietary carbohydrates? (A) Acarbose (B) Exenatide (C) Glipizide (D) Pioglitazone (E) Repaglinide 8. The PPAR-γ receptor that is activated by thiazolidinediones increases tissue sensitivity to insulin by which of the following mechanisms? (A) Activating adenylyl cyclase and increasing the intracellular concentration of cAMP (B) Inactivating a cellular inhibitor of the GLUT2 glucose transporter (C) Inhibiting acid glucosidase, a key enzyme in glycogen breakdown pathways (D) Regulating transcription of genes involved in glucose utilization (E) Stimulating the activity of a tyrosine kinase that phosphorylates the insulin receptor 9. Which of the following drugs is most likely to cause hypoglycemia when used as monotherapy in the treatment of type 2 diabetes? (A) Acarbose (B) Canagliflozin (C) Glyburide (D) Metformin (E) Miglitol (F) Rosiglitazone 10. Which of the following patients is most likely to be treated with intravenous glucagon? (A) An 18-year-old woman who took an overdose of cocaine and now has a blood pressure of 190/110 mm Hg (B) A 27-year-old woman with severe diarrhea caused by a flare in her inflammatory bowel disease (C) A 57-year-old woman with type 2 diabetes who has not taken her glyburide for the last 3 d (D) A 62-year-old man with severe bradycardia and hypotension resulting from ingestion of an overdose of atenolol (E) A 74-year-old man with lactic acidosis as a complication of severe infection and shock ANSWERS 1. Oral antidiabetic agents (listed in Table 41–1) are inappropriate in this patient because he has insulin-dependent diabetes. He needs a rapid-acting insulin preparation that can be given intravenously (see Figure 41–1). The answer is A. 2. Because of the risk of brain damage, the most important complication of insulin therapy is hypoglycemia. The other choices are not common effects of insulin. The answer is B. 3. Insulin regimens for tight control usually take the form of establishing a basal level of insulin with a small amount of a longacting preparation (eg, insulin glargine) and supplementing the insulin levels, when called for by food intake, with short-acting insulin lispro. Less tight control may be achieved with 2 injections of intermediate-acting insulin per day. Because intake of glucose is mainly during the day, long-acting insulins are usually given in the morning, not at night. The answer is D. CHAPTER 41 Pancreatic Hormones, Antidiabetic Agents, & Glucagon 4. Glipizide is a second-generation sulfonylurea that promotes insulin release by closing potassium channels in pancreatic B cells. The answer is C. 5. Insulin lowers serum glucose concentration in part by driving glucose into cells, particularly into muscle cells. The answer is C. 6. Biguanides, especially the older drug phenformin, have been associated with lactic acidosis. Thus, metformin should be avoided or used with extreme caution in patients with conditions that increase the risk of lactic acidosis, including acute ethanol ingestion. The answer is D. 7. To be absorbed, carbohydrates must be converted into monosaccharides by the action of α-glucosidase enzymes in the gastrointestinal tract. Acarbose inhibits α-glucosidase and, when present during digestion, delays the uptake of carbohydrates. The answer is A. 8. The PPAR-γ receptor belongs to a family of nuclear receptors. When activated, these receptors translocate to the nucleus, where they regulate the transcription of genes encoding proteins involved in the metabolism of carbohydrate and lipids. The answer is D. 9. The insulin secretagogues, including the sulfonylurea glyburide, can cause hypoglycemia as a result of their ability to increase serum insulin levels. The biguanides, thiazolidinediones, α-glucosidase inhibitors, and canagliflozin are euglycemics that are unlikely to cause hypoglycemia when used alone. The answer is C. 10. Glucagon acts through cardiac glucagon receptors to stimulate the rate and force of contraction of the heart. Because this bypasses cardiac β adrenoceptors, glucagon is useful in the treatment of β-blocker-induced cardiac depression. The answer is D. SKILL KEEPER ANSWERS: DIABETES AND HYPERTENSION (CHAPTER 11) 1. The major antihypertensive drug groups are (a) β-adrenoceptor blockers; (b) α1-selective adrenoceptor blockers (eg, prazosin); (c) centrally acting sympathoplegics (eg, clonidine or methyldopa); (d) calcium channel blockers (eg, diltiazem, nifedipine, verapamil); (e) angiotensin-converting enzyme (ACE) inhibitors (eg, captopril); (f) angiotensin receptor antagonists (eg, losartan); and (g) thiazide diuretics. 2. ACE inhibitors slow the progression of diabetic nephropathy and help stabilize renal function. Angiotensin receptor antagonists may have similar protective effects in patients with diabetes. Beta-adrenoceptor blockers can, in theory, mask the symptoms of hypoglycemia in diabetic patients; however, many patients with diabetes and cardiovascular disease are successfully treated with these drugs. A large clinical trial showed that control of hypertension decreases diabetes-associated microvascular disease. This trial included many patients being maintained on β-adrenoceptor blockers. Thiazide diuretics impair the release of insulin and tissue utilization of glucose, so they should be used with caution in patients with diabetes. CHECKLIST When you complete this chapter, you should be able to: ❑ Describe the effects of insulin on hepatocytes, muscle, and adipose tissue. ❑ List the types of insulin preparations and their durations of action. ❑ Describe the major hazards of insulin therapy. ❑ List the prototypes and describe the mechanisms of action, key pharmacokinetic features, and toxicities of the major classes of agents used to treat type 2 diabetes. ❑ Give 3 examples of rational drug combinations for treatment of type 2 diabetes mellitus. ❑ Describe the clinical uses of glucagon. 347 348 PART VII Endocrine Drugs DRUG SUMMARY TABLE: Antidiabetic Agents Subclass Clinical Applications Pharmacokinetics Type 1 and type 2 diabetes Parenteral administration, short-acting Hypoglycemia, weight gain Decreased endogenous glucose production Type 2 diabetes Oral administration Gastrointestinal (GI) disturbances, lactic acidosis (rare) Increases insulin secretion from pancreatic beta cells by closing ATP-sensitive K+ channels Type 2 diabetes Oral administration Hypoglycemia, weight gain Mechanism of Action Toxicities, Drug Interactions Insulins Regular insulin Activate insulin receptor Rapid-acting: lispro, aspart, glulisine Intermediate-acting: NPH Long-acting: detemir, glargine Biguanides Metformin Insulin secretagogues Glipizide Glyburide, glimepiride: like glipizide, sulfonylurea drugs with intermediate duration of action Chlorpropamide, tolbutamide: older sulfonylurea drugs, lower potency, greater toxicity; rarely used Repaglinide, nateglinide: fast-acting insulin secretagogues Alpha-glucosidase inhibitors Acarbose Inhibit intestinal α-glucosidases Type 2 diabetes Oral administration GI disturbances Regulates gene expression by binding to PPAR-γ Type 2 diabetes Oral administration Fluid retention, edema, anemia, weight gain, bone fractures in women, may worsen heart disease and increase risk of myocardial infarction Miglitol: similar to acarbose Thiazolidinediones Rosiglitazone Pioglitazone: similar to rosiglitazone, possibly fewer cardiovascular adverse effects Incretin-based drugs Exenatide Analog of glucagon-like peptide-1 (GLP-1) activates GLP-1 receptors Type 2 diabetes Parenteral administration GI disturbances, headache, pancreatitis Sitagliptin Inhibitor of the dipeptidyl peptidase-4 (DPP-4) that degrades GLP-1 and other incretins Type 2 diabetes Oral administration Rhinitis, upper respiratory infections, rare allergic reactions Analog of amylin activates amylin receptors Type 1 and type 2 diabetes Parenteral administration GI disturbances, hypoglycemia, headache Activates glucagon receptors Severe hypoglycemia, β-blocker overdose Parenteral administration GI disturbances, hypotension Inhibit renal glucose absorption via SGLT2 Type 2 diabetes Oral Osmotic diuresis, genital and urinary tract infections Amylin analog Pramlintide Glucagon Glucagon SGLT2 inhibitors Canagliflozin, dapagliflozin PPAR-γ, peroxisome proliferator-activated receptor-gamma; SGLT, sodium-glucose co-transporter. C A P T E R 42 Drugs That Affect Bone Mineral Homeostasis Calcium and phosphorus, the 2 major elements of bone, are crucial not only for the mechanical strength of the skeleton but also for the normal function of many other cells in the body. Accordingly, a complex regulatory mechanism has evolved to tightly regulate calcium and phosphate homeostasis. Parathyroid hormone (PTH), vitamin D, and H fibroblast growth factor 23 (FGF23) are primary regulators (Figure 42–1), whereas calcitonin, glucocorticoids, and estrogens play secondary roles. These hormones, or drugs that mimic or suppress their actions, are used in the treatment of bone mineral disorders (eg, osteoporosis, rickets, osteomalacia, Paget’s disease), as are several nonhormonal agents. Regulators of bone mineral homestasis Hormonal Nonhormonal PTH Bisphosphonates Vitamin D Calcitonin Fluoride Estrogen Glucocorticoids HORMONAL REGULATORS OF BONE MINERAL HOMEOSTASIS A. Parathyroid Hormone Parathyroid hormone (PTH), an 84-amino-acid peptide, acts on membrane G protein-coupled receptors to increase cyclic adenosine monophosphate (cAMP) in bone and renal tubular cells. In the kidney, PTH inhibits calcium excretion, promotes phosphate excretion, and stimulates the production of active vitamin D metabolites (Figure 42–1, Table 42–1). In bone, PTH promotes bone turnover by increasing the activity of both osteoblasts and osteoclasts Calcimimetics (Figure 42–2B). Osteoclast activation is not a direct effect and instead results from PTH stimulation of osteoblast formation of RANK ligand (RANKL), a member of the tumor necrosis factor (TNF) cytokine family that stimulates the activity of mature osteoclasts and the differentiation of osteoclast precursors. At the continuous high concentrations seen in hyperparathyroidism, the net effect of elevated PTH is increased bone resorption, hypercalcemia, and hyperphosphatemia. However, low intermittent doses of PTH produce a net increase in bone formation; this is the basis of the use of teriparatide, a recombinant truncated form of PTH, for parenteral treatment of osteoporosis. 349 350 PART VII Endocrine Drugs High-Yield Terms to Learn Hyperparathyroidism A condition of PTH excess characterized by hypercalcemia, bone pain, cognitive abnormalities, and renal stones. Primary disease results from parathyroid gland dysfunction. Secondary disease most commonly results from chronic kidney disease Osteoblast Bone cell that promotes bone formation Osteoclast Bone cell that promotes bone resorption Osteomalacia A condition of abnormal mineralization of adult bone secondary to nutritional deficiency of vitamin D or inherited defects in the formation or action of active vitamin D metabolites Osteoporosis Abnormal loss of bone with increased risk of fractures, spinal deformities, and loss of stature; remaining bone is histologically normal Paget’s disease A bone disorder, of unknown origin, characterized by excessive bone destruction and disorganized repair. Complications include skeletal deformity, musculoskeletal pain, kidney stones, and organ dysfunction secondary to pressure from bony overgrowth Rickets The same as osteomalacia, but occurs in the growing skeleton RANK ligand An osteoblast-derived growth factor that stimulates osteoclast activity and osteoclast precursor differentiation The synthesis and secretion of PTH is primarily regulated by the serum concentration of free ionized calcium; a drop in free ionized calcium stimulates PTH release. Active metabolites of vitamin D play a secondary role in regulating PTH secretion by inhibiting PTH synthesis (Figure 42–2A). B. Vitamin D Vitamin D, a fat-soluble vitamin (Figure 42–3), can be synthesized in the skin from 7-dehydrocholesterol under the influence of ultraviolet light or absorbed from the diet in the natural form (vitamin D3, cholecalciferol) or the plant form Ca, P TABLE 42–1 Actions of PTH and active vitamin D metabolites on intestine, kidney, and bone. D(+) Gut Serum Ca, P PTH Intestine Indirectly increases calcium and phosphate absorption by increasing vitamin D metabolites Increased calcium and phosphate absorption Kidney Decreased calcium excretion, increased phosphate excretion Increased resorption of calcium and phosphate but usually net increase in urinary calcium due to effects in GI tract and bone Bone Calcium and phosphate resorption increased by continuous high concentrations. Low intermittent doses increase bone formation Direct effect is increased calcium and phosphate resorption; indirect effect is promoting mineralization by increasing the availability of calcium and phosphate Net effect on serum levels Serum calcium increased, serum phosphate decreased Serum calcium and phosphate both increased D(+), PTH(+) Bone D(+), PTH(+) CT(–) Ca, P Kidney D(–) PTH(–) CT(+) Ca Active Vitamin D Metabolites Organ D(–) PTH(+) CT(+) FGF23(+) P FIGURE 42–1 Effects of active metabolites of vitamin D (D), parathyroid hormone (PTH), calcitonin (CT), and fibroblast growth factor 23 (FGF23) on calcium and phosphorus homeostasis. Active metabolites of vitamin D increase absorption of calcium from both gut and bone, whereas PTH increases reabsorption from bone. Vitamin D metabolites and PTH both reduce urinary excretion of calcium. In animals with vitamin D deficiency, active metabolites of vitamin D produce a net increase in bone mineralization by increasing the availability of serum calcium and phosphate. (Reproduced, with permission, from Katzung BG, editor: Basic & Clinical Pharmacology, 12th ed. McGraw-Hill, 2012: Fig. 42–1.) Reproduced and modified, with permission, from Katzung BG, editor: Basic & Clinical Pharmacology, 12th ed. McGraw-Hill, 2012. CHAPTER 42 Drugs That Affect Bone Mineral Homeostasis 351 Bone 1,25(OH)2D Gut + + Ca2+ in blood + Thyroid 1,25(OH)2D – PTH 1,25(OH)2D + – – FGF23 Kidney PTH – Calcitonin Parathyroids 25(OH)D Extracellular Ca2+ A Monocyte Stem cells + Preosteoclast PTH 1,25(OH)2D + + Preosteoblasts + Osteoclast Osteoblasts RANKL + MCSF + OPG – Osteoid – Bisphosphonates Calcitonin Estrogen Calcified bone B FIGURE 42–2 Hormonal interactions controlling bone mineral homeostasis. (A) The 1,25-dihydroxyvitamin D that is produced by the kidney under control of parathyroid hormone (PTH) and fibroblast growth factor 23 (FGF23) stimulates intestinal uptake of calcium and phosphate, and, in those with vitamin D deficiency, promotes bone formation. Calcitonin inhibits resorption from bone, whereas PTH stimulates bone resorption. Extracellular calcium and 1,25-dihydroxyvitamin D inhibit PTH production. (B) Both PTH and 1,25-dihydroxyvitamin D regulate bone formation and resorption. This is accomplished by their activation of precursor differentiation and by stimulation of osteoblast production of signaling factors, including RANK ligand (RANKL), macrophage colony-stimulating factor (MCSF), and osteoprotegerin (OPG). (Reproduced and modified, with permission, from Katzung BG, editor: Basic & Clinical Pharmacology, 12th ed. McGraw-Hill, 2012: Fig. 42–2.) (vitamin D2, ergocalciferol). Active metabolites are formed in the liver (25-hydroxyvitamin D or calcifediol) and kidney (1,25-dihydroxyvitamin D or calcitriol plus other metabolites). Renal synthesis of active vitamin D metabolites is stimulated by PTH. Synthesis of 1,25-dihydroxyvitamin D2 is inhibited by phosphate, fibroblast growth factor 23 (FGF23), and vitamin D metabolites (Figure 41–2). The action of vitamin D metabolites is mediated by activation of 1 or possibly a family of nuclear receptors that regulate gene expression. Active vitamin D metabolites cause a net increase in serum concentrations of calcium and phosphate by increasing intestinal absorption and bone resorption and decreasing renal excretion (Figure 42–1, Table 42–1). Because their effect in the gastrointestinal (GI) tract and bone is greater than their effect in the kidney, they also increase urinary calcium. Active vitamin D metabolites are required for normal mineralization of bone; deficiencies cause rickets in growing children and adolescents and osteomalacia in adults. Vitamin D metabolites inhibit PTH secretion directly and indirectly, by increasing serum calcium. Vitamin D, vitamin D metabolites, and synthetic derivatives are used to treat deficiency states, including nutritional deficiency, intestinal osteodystrophy, chronic kidney or liver disease, hypoparathyroidism, and nephrotic syndrome. They are also used, in combination with calcium supplementation, to prevent and 352 PART VII Endocrine Drugs 21 HO 22 20 18 19 12 1716 CH3 11 13 15 14 9 1 10 8 2 3 5 7 6 4 26 23 24 25 27 OH Heat Ultraviolet CH3 CH2 7-Dehydrocholesterol Pre D3 D3 (cholecalciferol) HO O HO H O H Liver P Ca +1,25(OH)2D Kidney CH2 HO 24,25(OH)2D3 (secalciferol) CH2 CH2 HO HO D3 25(OH)D3 P Ca +PTH –FGF23 O H 28 CH3 22 21 20 CH2 26 24 23 25 27 HO OH 1,25(OH)2D3 (calcitriol) FIGURE 42–3 Conversion of 7-dehydrocholesterol to vitamin D3 and metabolism of vitamin D3 to 1,25-dihydroxyvitamin D3 (1,25(OH)2D3) and to 24,25-dihydroxyvitamin D3 (24,25(OH)2D3). The inset shows the side chain for ergosterol. Ergosterol undergoes similar transformation to vitamin D2 (ergocalciferol), which, in turn is metabolized to 1,25-dihydroxyvitamin D2 and 24,25-dihydroxyvitamin D2. In humans, corresponding D2 and D3 have equivalent effects and potency. They are therefore referred to in the text without a subscript. (Reproduced, with permission, from Katzung BG, editor: Basic & Clinical Pharmacology, 12th ed. McGraw-Hill, 2012: Fig. 42–3.) treat osteoporosis in older women and men. Topical formulations are used in psoriasis, a hyperproliferative skin disorder. The 2 forms of vitamin D—cholecalciferol and ergocalciferol—are available as oral supplements and are commonly added to dairy products and other foods. In patients with conditions that impair vitamin D activation (chronic kidney disease, liver disease, hypoparathyroidism), an active form of vitamin D such as calcitriol is required. In the treatment of secondary hyperparathyroidism associated with chronic kidney disease, calcitriol reduces PTH levels, corrects hypocalcemia, and improves bone disease, but it can also result in hypercalcemia and hypercalciuria through direct effects on intestinal, bone, and renal handling of calcium and phosphate. Several forms of active vitamin D that selectively inhibit PTH formation while posing less risk of hypercalcemia have been developed. 1α-Hydroxyvitamin D2 (doxercalciferol) is a prodrug that is converted in the liver to 1,25-dihydroxyvitamin D, whereas 19-nor-1,25-dihydroxyvitamin D2 (paricalcitol) and calcipotriene (calcipotriol) are analogs of calcitriol. All cause less hypercalcemia and, in patients with normal renal function, less hypercalciuria than calcitriol. Oral and parenteral doxercalciferol and oral paricalcitol are approved for treatment of secondary hyperparathyroidism in patients with chronic kidney disease. Calcipotriene (calcipotriol) is approved for topical treatment of psoriasis. These and other analogs are being investigated for use in various malignancies and inflammatory disorders. The primary toxicity caused by chronic overdose with vitamin D or its active metabolites is hypercalcemia, hyperphosphatemia, and hypercalciuria. C. Fibroblast growth factor 23 (FGF23) FGF23 is secreted by osteocytes in bone and inhibits 1,25(OH)2D production and phosphate reabsorption in the kidney. It is not used as a drug. D. Calcitonin Calcitonin, a peptide hormone secreted by the thyroid gland, decreases serum calcium and phosphate by inhibiting bone resorption and inhibiting renal excretion of these minerals (Figure 42–1). Bone formation is not impaired initially, but ultimately both formation and resorption are reduced. The hormone has been used in conditions in which an acute reduction of serum calcium is needed (eg, Paget’s disease and hypercalcemia). Calcitonin is approved for treatment of osteoporosis and has been shown to increase bone mass and to reduce spine fractures. However, it is not as effective as teriparatide or bisphosphonates. Although human calcitonin is available, salmon calcitonin is most often selected for clinical use because of its longer half-life and greater potency. Calcitonin is administered by injection or as a nasal spray. CHAPTER 42 Drugs That Affect Bone Mineral Homeostasis E. Estrogens Estrogens and selective estrogen receptor modulators (SERMs; eg, raloxifene) can prevent or delay bone loss in postmenopausal women (see Chapter 40). Their action involves the inhibition of PTH-stimulated bone resorption (Figure 42–2B). F. Glucocorticoids The glucocorticoids (Chapter 39) inhibit bone mineral maintenance. As a result, chronic systemic use of these drugs is a common cause of osteoporosis in adults. However, these hormones are useful in the intermediate-term treatment of hypercalcemia. SKILL KEEPER: DIURETICS AND CALCIUM (SEE CHAPTER 15) The kidney is a key regulator of serum calcium concentrations. Several diuretics affect the kidney’s handling of filtered calcium. 1. Which 2 classes of diuretics have opposite effects on calcium elimination? 2. What mechanisms are responsible for their opposing effects? 3. What is the clinical importance of these effects? The Skill Keeper Answers appear at the end of the chapter. NONHORMONAL AGENTS A. Bisphosphonates The bisphosphonates (alendronate, etidronate, ibandronate, pamidronate, risedronate, tiludronate, and zoledronic acid) are short-chain organic polyphosphate compounds that reduce both the resorption and the formation of bone by an action on the basic hydroxyapatite crystal structure. The bisphosphonates have other complex cellular effects, including effects on vitamin D production and calcium absorption from the GI tract, and direct effects on osteoclasts, including inhibition of farnesyl pyrophosphate synthase, an enzyme that appears to play a critical role in osteoclast survival. Bisphosphonates are used to manage the hypercalcemia associated with some malignancies and to treat Paget’s disease. Chronic bisphosphonate therapy is used commonly to prevent and treat all forms of osteoporosis. It has been shown to increase bone density and reduce fractures. Pamidronate, zoledronic acid, or etidronate are available for parenteral treatment of hypercalcemia associated with Paget’s disease and malignancies. Etidronate and the other bisphosphonates listed above are available as oral medications. Oral bioavailability of bisphosphonates is low (<10%), and food impairs their absorption. Bisphosphonate treatment of osteoporosis is accomplished with daily oral dosing (alendronate, risedronate, ibandronate); weekly oral dosing (alendronate, risedronate); monthly oral dosing (ibandronate); quarterly injection dosing (ibandronate); or annual infusions (zoledronate). The primary toxicity of the low oral bisphosphonate doses used for osteoporosis is gastric and esophageal irritation. To reduce esophageal irritation, patients 353 are advised to take the drugs with large quantities of water and avoid situations that permit esophageal reflux. The higher doses of bisphosphonates used to treat hypercalcemia have been associated with renal impairment and osteonecrosis of the jaw. B. Rank Ligand (RANKL) Inhibitor Denosumab is a human monoclonal antibody that binds to and prevents the action of RANKL. Denosumab inhibits osteoclast formation and activity. It is at least as effective as the potent bisphosphonates in inhibiting bone resorption and can be used for treatment of postmenopausal osteoporosis. Denosumab is administered subcutaneously every 6 mo, which avoids gastrointestinal side effects. The drug appears to be well tolerated, but there could be an increased risk of infection due to RANKL’s role in the immune response. C. Calcimimetics Cinacalcet lowers PTH by activating the calcium-sensing receptor in the parathyroid gland. It is used for oral treatment of secondary hyperparathyroidism in chronic kidney disease and for the treatment of hypercalcemia in patients with parathyroid carcinoma. Its toxicities include hypocalcemia and adynamic bone disease, a condition of profoundly decreased bone cell activity. D. Fluoride Appropriate concentrations of fluoride ion in drinking water or as an additive in toothpaste have a well-documented ability to reduce dental caries. Chronic exposure to the ion, especially in high concentrations, may increase new bone synthesis. It is not clear, however, whether this new bone is normal in strength. Clinical trials of fluoride in patients with osteoporosis have not demonstrated a reduction in fractures. Acute toxicity of fluoride (usually caused by ingestion of rat poison) is manifested by gastrointestinal and neurologic symptoms. E. Other Drugs with Effects on Serum Calcium and Phosphate Strontium ranelate, an organic ion bound to 2 atoms of strontium, promotes osteoclast apoptosis and increases concentrations of bone formation markers; it is used in Europe for treatment of osteoporosis. Gallium nitrate is effective in managing the hypercalcemia associated with some malignancies and possibly Paget’s disease. It acts by inhibiting bone resorption. To prevent nephrotoxicity, patients need to be well hydrated and to have good renal output. The antibiotic plicamycin (mithramycin) has been used to reduce serum calcium and bone resorption in Paget’s disease and hypercalcemia. Because of the risk of serious toxicity (eg, thrombocytopenia, hemorrhage, hepatic and renal damage), plicamycin is mainly restricted to short-term treatment of serious hypercalcemia. Several diuretics, most notably thiazide diuretics and furosemide, can affect serum and urinary calcium levels (see this chapter’s Skill Keeper). The phosphate-binding gel sevelamer is used in combination with calcium supplements and dietary phosphate restriction to treat hyperphosphatemia, a common complication of renal failure, hypoparathyroidism, and vitamin D intoxication. 354 PART VII Endocrine Drugs QUESTIONS 1. Which of the following drugs is routinely added to calcium supplements and milk for the purpose of preventing rickets in children and osteomalacia in adults? (A) Cholecalciferol (B) Calcitriol (C) Gallium nitrate (D) Sevelamer (E) Plicamycin 2. Which of the following drugs is most useful for the treatment of hypercalcemia in Paget’s disease? (A) Fluoride (B) Hydrochlorothiazide (C) Pamidronate (D) Raloxifene (E) Teriparatide 3. The active metabolites of vitamin D act through a nuclear receptor to produce which of the following effects? (A) Decrease the absorption of calcium from bone (B) Increase PTH formation (C) Increase renal production of erythropoietin (D) Increase the absorption of calcium from the gastrointestinal tract (E) Lower the serum phosphate concentration 4. A 59-year-old female was referred to your clinic for evaluation of osteopenia. She was diagnosed with adult-onset cystic fibrosis (CF). She reported being treated with prednisone 2 times in the past for CF exacerbations. Since menopause at 52 years of age, she had been treated with raloxifene for osteoporosis prevention. She also was on daily calcium and vitamin D supplementation. Her bone mineral density test revealed a T score of –1.6 at the lumbar spine, –2.2 at the left femoral neck, and –1.6 at the total left hip. Which of the following drugs can be used to reduce the fracture risk by further stimulating bone formation in this patient? (A) Cholecalciferol (B) Ergocalciferol (C) Furosemide (D) Tamoxifen (E) Teriparatide Questions 5–7. A 58-year-old postmenopausal woman was sent for dual-energy x-ray absorptiometry to evaluate the bone mineral density of her lumbar spine, femoral neck, and total hip. The test results revealed significantly low bone mineral density in all sites. 5. Chronic use of which of the following medications is most likely to have contributed to this woman’s osteoporosis? (A) Lovastatin (B) Metformin (C) Prednisone (D) Propranolol (E) Thiazide diuretic 6. If this patient began oral therapy with alendronate, she would be advised to drink large quantities of water with the tablets and remain in an upright position for at least 30 min and until eating the first meal of the day. These instructions would be given to decrease the risk of which of the following? (A) Cholelithiasis (B) Diarrhea (C) Constipation (D) Erosive esophagitis (E) Pernicious anemia 7. The patient’s condition was not sufficiently controlled with alendronate, so she began therapy with a nasal spray containing a protein that inhibits bone resorption. The drug contained in the nasal spray was which of the following? (A) Calcitonin (B) Calcitriol (C) Cinacalcet (D) Cortisol (E) Teriparatide Questions 8–10. A 67-year-old man with chronic kidney disease was found to have an elevated serum PTH concentration and a low serum concentration of 25-hydroxyvitamin D. He was successfully treated with ergocalciferol. Unfortunately, his kidney disease progressed so that he required dialysis and his serum PTH concentration became markedly elevated. 8. Which of the following drugs is most likely to lower this patient’s serum PTH concentration? (A) Calcitriol (B) Cholecalciferol (C) Furosemide (D) Gallium nitrate (E) Risedronate 9. Although the drug therapy was effective at lowering serum PTH concentrations, the patient experienced several episodes of hypercalcemia. He was switched to a vitamin D analog that suppresses PTH with less risk of hypercalcemia. Which drug was the patient switched to? (A) Calcitriol (B) Cholecalciferol (C) Furosemide (D) Paricalcitol (E) Risedronate 10. In the treatment of patients like this with secondary hyperparathyroidism due to chronic kidney disease, cinacalcet is an alternative to vitamin D-based drugs. Cinacalcet lowers PTH by which of the following mechanisms? (A) Activating a steroid receptor that inhibits expression of the PTH gene (B) Activating the calcium-sensing receptor in parathyroid cells (C) Activating transporters in the GI tract that are involved in calcium absorption (D) Inducing the liver enzyme that converts vitamin D3 to 25-hydroxyvitamin D3 (E) Inhibiting the farnesyl pyrophosphate synthase found in osteoclasts CHAPTER 42 Drugs That Affect Bone Mineral Homeostasis ANSWERS 1. The 2 forms of vitamin D—cholecalciferol and ergocalciferol— are commonly added to calcium supplements and dairy products. Calcitriol, the active 1,25-dihydroxyvitamin D3 metabolite, would prevent vitamin D deficiency and is available as an oral formulation. However, because it is not subject to the complex mechanisms that regulate endogenous production of active vitamin D metabolites, it is not suitable for widespread use. The answer is A. 2. Paget’s disease is characterized by excessive bone resorption, poorly organized bone formation, and hypercalcemia. Bisphosphonates and calcitonin are first-line treatments. Pamidronate is a powerful bisphosphonate used parenterally to treat hypercalcemia. The answer is C. 3. The active metabolites of vitamin D increase serum calcium and phosphate by promoting calcium and phosphate uptake from the gastrointestinal tract, increasing bone resorption, and decreasing renal excretion of both electrolytes. They inhibit, rather than stimulate, PTH formation. The answer is D. 4. Cholecalciferol and ergocalciferol are precursors of vitamin D. Furosemide is a loop diuretic, which causes increased calcium excretion; tamoxifen is a selective estrogen receptor modulator (SERM) but is less selective for bone compared with raloxifene. Teriparatide increases bone formation and bone resorption; during the first 6 months, it causes a net gain in bone. Teriparatide should not be used longer than 2 yr due to risk of osteosarcoma. The answer is E. 5. Long-term therapy with glucocorticoids such as prednisone is associated with a reduction in bone mineral density and an increased risk of fractures. The other drugs are not known to have significant effects on bone or serum calcium. The answer is C. 6. Oral bisphosphonates such as alendronate can irritate the esophagus and stomach. The risk of this toxicity is reduced by drinking water and by remaining in an upright position for 30 min after taking the medication. The answer is D. 7. Calcitonin is a peptide hormone that prevents bone resorption. Salmon calcitonin is available as a nasal spray or a parenteral form for injection. The answer is A. 8. In patients with chronic kidney disease that requires dialysis, the impaired production of active vitamin D metabolites compounded with elevated serum phosphate due to renal impairment leads to secondary hyperparathyroidism. Administration of the active vitamin D metabolite calcitriol acts directly on the parathyroid to inhibit PTH production. Cholecalciferol, a form of vitamin D, is not effective in patients with advanced renal disease who cannot form adequate amounts of active vitamin D metabolites. The answer is A. 9. Paricalcitol is an analog of 1,25-dihyroxyvitamin D3 (calcitriol) that lowers serum PTH at doses that only rarely precipitate hypercalcemia. The molecular basis of this selective action is poorly understood but is of value in the management of hyperparathyroidism and psoriasis. The answer is D. 355 10. Cinacalcet is a member of a novel class of drugs that activate the calcium-sensing receptor in parathyroid cells. When this receptor is activated by cinacalcet or free ionized calcium, it activates a signaling pathway that suppresses PTH synthesis and release. The answer is B. SKILL KEEPER ANSWERS: DIURETICS AND CALCIUM (SEE CHAPTER 15) 1. Loop diuretics (eg, furosemide) and thiazide diuretics have opposite effects on urine calcium concentrations; loop diuretics increase urine concentrations of calcium, whereas the thiazides decrease urine calcium. 2. Loop diuretics inhibit the Na+/K+/2Cl– cotransporter in apical membranes of the thick ascending limb of the loop of Henle (see Figure 15–3). By disrupting the lumenpositive potential that normally serves as the driving force for resorption of Mg2+ and Ca2+, loop diuretics inhibit Mg2+ and Ca2+ resorption, leaving more Mg2+ and Ca2+ in the urine and less in the blood. In the distal convoluted tubule where thiazides act, Ca2+ is actively resorbed through the concerted action of an apical Ca2+ channel and a basolateral Na+/Ca2+ exchanger (see Figure 15–4). The system is under control of PTH. When thiazides inhibit the Na+/Cl– transporter in cells that line the distal convoluted tubule, they lower the intracellular concentration of sodium and thereby enhance the Na+/Ca2+ exchange that occurs on the basolateral surface. This, in turn, creates a greater driving force for passage of Ca2+ through the apical membrane calcium channels. The net effect is enhanced resorption of calcium. 3. In patients with hypercalcemia, treatment with a loop diuretic plus saline promotes calcium excretion and lowers serum calcium. In patients with intact regulatory function, increases in calcium resorption promoted by thiazides have minor impact on serum calcium because of buffering in bone and gut. However, thiazides can unmask hypercalcemia in patients with diseases that disrupt normal calcium regulation (eg, hyperparathyroidism, sarcoidosis, carcinoma). Thiazide diuretics are also used for treatment of persons who are prone to kidney stone formation as a result of idiopathic hypercalciuria. In such persons, it is crucial that primary hyperparathyroidism is ruled out before thiazide treatment is initiated. 356 PART VII Endocrine Drugs CHECKLIST When you complete this chapter, you should be able to: ❑ Identify the major and minor endogenous regulators of bone mineral homeostasis. ❑ Sketch the pathway and sites of formation of 1,25-dihydroxyvitamin D. ❑ Compare and contrast the clinical uses and effects of the major forms of vitamin D and its active metabolites. ❑ Describe the major effects of PTH and vitamin D derivatives on the intestine, the kidney, and bone. ❑ Describe the agents used in the treatment of hypercalcemia and the agents used in the treatment of osteoporosis. ❑ Recall the effects of adrenal and gonadal steroids on bone structure and the actions of diuretics on serum calcium levels. CHAPTER 42 Drugs That Affect Bone Mineral Homeostasis 357 DRUG SUMMARY TABLE: Drugs Affecting Bone Mineral Metabolism Subclass Mechanism of Action Clinical Applications Pharmacokinetics Vitamin D deficiency Oral administration Requires metabolism in liver or kidney to active forms Toxicities, Drug Interactions Vitamin D, metabolites, analogs Cholecalciferol, ergocalciferol Regulates gene transcription via the vitamin D receptor to produce the effects detailed in Table 42–1 Hypercalcemia, hyperphosphatemia, hypercalciuria Calcitriol: used for management of secondary hyperparathyroidism in patients with chronic kidney disease and for management of hypocalcemia in patients with hypoparathyroidism. Note that drug is active form, does not require metabolism Doxercalciferol (1-hydroxyvitamin D3 ): used for management of secondary hyperparathyroidism in patients with chronic kidney disease Paricalcitol: an analog of calcitriol used for management of secondary hyperparathyroidism in patients with chronic kidney disease Calcipotriene: an analog of calcitriol approved for psoriasis Bisphosphonates Alendronate Suppresses the activity of osteoclasts and inhibits bone resorption Osteoporosis, Paget’s disease Oral administration daily or weekly Adynamic bone, esophageal irritation, osteonecrosis of the jaw (rare) Risedronate, ibandronate, pamidronate, zoledronate: similar to alendronate Parathyroid hormone (PTH) analog Teriparatide Acts through PTH receptors to produce a net increase in bone formation Osteoporosis Subcutaneous injection Hypercalcemia, hypercalciuria • osteosarcoma in experimental animals Acts through calcitonin receptors to inhibit bone resorption Osteoporosis Subcutaneous injection or intranasal Rhinitis with the nasal spray Osteoporosis in postmenopausal women Oral administration Hot flushes, thromboembolism Binds to RANKL and prevents it from stimulating osteoclast differentiation and function Osteoporosis Subcutaneously every 6 mo May increase risk of infections Activates the calciumsensing receptor Hyperparathyroidism Oral administration Nausea, hypocalcemia, adynamic bone Calcitonin Calcitonin Selective estrogen-receptor modulator (see Chapter 40) Raloxifene Estrogen agonist effect in bone • estrogen antagonist effects in breast and endometrium RANK Ligand (RANKL) Inhibitor Denosumab Calcimimetic Cinacalcet This page intentionally left blank PART VIII CHEMOTHERAPEUTIC DRUGS INTRODUCTION TO ANTIMICROBIAL DRUGS The emergence of microbial resistance poses a constant challenge to the use of antimicrobial drugs. Mechanisms underlying microbial resistance include the production of antibiotic-inactivating enzymes, changes in the structure of target receptors, increased efflux via drug transporters, and decreases in the permeability of microbes’ cellular membranes to antibiotics. Strategies designed to combat microbial resistance include the use of adjunctive agents that can protect against antibiotic inactivation, the use of antibiotic combinations, the introduction of new (and often expensive) chemical derivatives of established antibiotics, and efforts to avoid the indiscriminate use or misuse of antibiotics. 359 C B Pe n N aow r spectrum r i c i l l Wider spectrum Penicillinase susceptible i ae i cr a t l a n s t h e o ss r i Wider spectrum 1st generation 2nd, 3rd, 4th generations A. Classification All penicillins are derivatives of 6-aminopenicillanic acid and contain a beta-lactam ring structure that is essential for antibacterial activity. Penicillin subclasses have additional chemical substituents that confer differences in antimicrobial activity, susceptibility to acid and enzymatic hydrolysis, and biodisposition. 360 ly l n Narrow spectrum PENICILLINS P T E R as important as the beta-lactam drugs. The selective toxicity of the drugs discussed in this chapter is mainly due to specific actions on the synthesis of a cellular structure that is unique to the microorganism. More than 50 antibiotics that act as cell wall synthesis inhibitors are currently available, with individual spectra of activity that afford a wide range of clinical applications. Cephalosporins Penicillinase resistant A 43 Beta-Lactam Antibiotics & Other Cell Wall Synthesis Inhibitors Penicillins and cephalosporins are the major antibiotics that inhibit bacterial cell wall synthesis. They are called beta-lactams because of the unusual 4-member ring that is common to all their members. The beta-lactams include some of the most effective, widely used, and well-tolerated agents available for the treatment of microbial infections. Vancomycin, fosfomycin, and bacitracin also inhibit cell wall synthesis but are not nearly H s c Miscellaneous Carbapenems Aztreonam Vancomycin B. Pharmacokinetics Penicillins vary in their resistance to gastric acid and therefore vary in their oral bioavailability. Parenteral formulations of ampicillin, piperacillin, and ticarcillin are available for injection. Penicillins are polar compounds and are not metabolized extensively. They are usually excreted unchanged in the urine via glomerular filtration and tubular secretion; the latter process is inhibited by probenecid. Nafcillin is excreted mainly in the bile and ampicillin e l sl CHAPTER 43 Beta-Lactam Antibiotics & Other Cell Wall Synthesis Inhibitors 361 High-Yield Terms to Learn Bactericidal An antimicrobial drug that can eradicate an infection in the absence of host defense mechanisms; kills bacteria Bacteriostatic An antimicrobial drug that inhibits antimicrobial growth but requires host defense mechanisms to eradicate the infection; does not kill bacteria Beta-lactam antibiotics Drugs with structures containing a beta-lactam ring: includes the penicillins, cephalosporins and carbapenems. This ring must be intact for antimicrobial action Beta-lactamases Bacterial enzymes (penicillinases, cephalosporinases) that hydrolyze the beta-lactam ring of certain penicillins and cephalosporins; confer resistance Beta-lactam inhibitors Potent inhibitors of some bacterial beta-lactamases used in combinations to protect hydrolyzable penicillins from inactivation Minimal inhibitory concentration (MIC) Lowest concentration of antimicrobial drug capable of inhibiting growth of an organism in a defined growth medium Penicillin-binding proteins (PBPs) Bacterial cytoplasmic membrane proteins that act as the initial receptors for penicillins and other beta-lactam antibiotics Peptidoglycan Chains of polysaccharides and polypeptides that are cross-linked to form the bacterial cell wall Selective toxicity More toxic to the invader than to the host; a property of useful antimicrobial drugs Transpeptidases Bacterial enzymes involved in the cross-linking of linear peptidoglycan chains, the final step in cell wall synthesis undergoes enterohepatic cycling. The plasma half-lives of most penicillins vary from 30 min to 1 h. Procaine and benzathine forms of penicillin G are administered intramuscularly and have long plasma half-lives because the active drug is released very slowly into the bloodstream. Most penicillins cross the blood-brain barrier only when the meninges are inflamed. C. Mechanisms of Action and Resistance Beta-lactam antibiotics are bactericidal drugs. They act to inhibit cell wall synthesis by the following steps (Figure 43–1): (1) binding of the drug to specific enzymes (penicillin-binding proteins [PBPs]) located in the bacterial cytoplasmic membrane; (2) inhibition of the transpeptidation reaction that cross-links the linear peptidoglycan chain constituents of the cell wall; and (3) activation of autolytic enzymes that cause lesions in the bacterial cell wall. Enzymatic hydrolysis of the beta-lactam ring results in loss of antibacterial activity. The formation of beta-lactamases (penicillinases) by most staphylococci and many gram-negative organisms is a major mechanism of bacterial resistance. Inhibitors of these bacterial enzymes (eg, clavulanic acid, sulbactam, tazobactam) are often used in combination with penicillins to prevent their inactivation. Structural change in target PBPs is another mechanism of resistance and is responsible for methicillin resistance in staphylococci and for resistance to penicillin G in pneumococci (eg, PRSP, penicillin resistant Streptococcus pneumoniae) and enterococci. In some gram-negative rods (eg, Pseudomonas aeruginosa), changes in the porin structures in the outer ce